Download as pdf or txt
Download as pdf or txt
You are on page 1of 320

பசித்திரு (Be hungry) தனித்திரு (Be individual) விழித்திரு (Be conscious)

HIGHER SECONDARY SECOND YEAR-PHYSICS

NAME :
STANDARD : 12 SECTION :
SCHOOL :
EXAM NO :

victory R. SARAVANAN. M.Sc, M.Phil, B.Ed.,


PG ASST (PHYSICS)
GBHSS, PARANGIPETTAI - 608 502
12 PHYSICS UNIT - 1 ELECTROSTATICS COMPLETE STUDY MATERIAL
4. An electric dipole is placed at an alignment angle of 30 o with an electric field
PART - I MULTIPLE CHOICE QUESTIONS & ANSWERS WITH SOLUTIONS
of 2 × 105 NC–1. It experiences a torque equal to 8 N m. The charge on the
1. Two identical point charges of magnitude –q are fixed as shown in the figure dipole if the dipole length is 1 cm is
below. A third charge +q is placed midway between the two charges at the (a) 4 mC (b) 8 mC (c) 5 mC (d) 7 mC
point P. Suppose this charge +q is displaced a small distance from the point P Solution :
in the directions indicated by the  Torque on the dipole is ; 𝜏 = 𝑝 𝐸 sin 𝜃 = 𝑞 2𝑎 𝐸 sin 𝜃
arrows, in which direction(s) will  Hence the charge ;
𝜏 8 8
+q be stable with respect to the 𝑞 = 2𝑎 𝐸 sin 𝜃 = 1 𝑋 10−2 𝑋 2 𝑋 105 𝑋 sin 30°
= 1 = 8 𝑋 10−3 𝐶 = 8 𝑚𝐶
2 𝑋 103 𝑋
displacement? 2
(a) A1 and A2 (b) B1 and B2 (c) both directions (d) No stable Answer (b) 𝟖 𝐦𝐂
Soultion : 5. Four Gaussian surfaces are given below with charges
 Stable means steady and unlikely to change inside each Gaussian surface. Rank the electric flux
 If +q is moved towards either A1 or through each Gaussian surface in increasing order.
A2, it will attracted by the nearer (a) D < C < B < A (b) A < B = C < D
charge -q. Thus +q moves away from (c) C < A = B < D (d) D > C > B > A
P and hence it is unstable Solution :
 But if +q is moved towards either B1 or B2, the vertical components of the forces 2𝑞
 Net charge inside A = +2𝑞 + 𝑞 − 𝑞 = 2𝑞 . Hence Φ𝐴 =
brings the charge +q to the original position P and hence it is stable. 𝜀0
𝑞
Answer (b) B1 and B2  Net charge inside B = 2𝑞 − 𝑞 = 𝑞 . Hence Φ𝐵 =
𝜀0
2. Which charge configuration produces a uniform electric field?  Net charge inside C = +𝑞 − 𝑞 = 0 . Hence Φ𝐶 = 0
(a) point charge (b) uniformly charged infinite line 𝑞
 Net charge inside D = − 𝑞 . Hence Φ𝐷 = −
𝜀0
(c) uniformly charged infinite plane (d) uniformly charged spherical shell
Solution : Answer (a) D<C<B<A
 Due to point charge ; 𝐸=
1 𝑞
(or) 𝐸 ∝ 2
1 6. The total electric flux for the following closed surface
2 4 𝜋 𝜀0 𝑟 𝑟 which is kept inside water
1 𝜆 1 80 𝑞 𝑞
 Due to charged infinite line ; 𝐸= (or) 𝐸∝ (a) (b)
2 𝜋 𝜀0 𝑟 𝑟
𝜎 𝜀0 40 𝜀0
 Due to uniformly charged infinite plane ; 𝐸 = (or) E = constant 𝑞 𝑞
2 𝜀0 (c) (d)
1 𝑞 1 80 𝜀0 160 𝜀0
 Due to uniformly charged spherical shell ; 𝐸 = (or) 𝐸∝ Solution :
4 𝜋 𝜀0 𝑟 2 𝑟2
𝑄 2 𝑞+𝑞−𝑞 2𝑞 𝑞
Answer (c) uniformly charged infinite plane  The total electric flux ; Φ = = 80 𝜀 = 80 𝜀 = 40𝜀
𝒒 𝜀0 𝜀𝑟 0 0 0
3. What is the ratio of the charges | 𝟏 | for the 𝑞
𝒒𝟐 Answer (b) 40 𝜀0
following electric field line pattern?
1 25 7. Two identical conducting balls having positive charges q1 and q2 are
(a) (b) separated by a centre to centre distance r. If they are made to touch each
5 11
11 other and then separated to the same distance, the force between them will be
(c) 5 (d)
25 (a) less than before (b) same as before (c) more than before (d) zero
Solution :
𝑞 Solution :
 Number of electric lines enters to 𝑞1 is Φ1 = 1 = 11 𝑞 𝑞
𝜀0  Initial force ; 𝐹𝑖𝑛𝑖𝑡𝑖𝑎𝑙 = 𝑘 1 2 2
𝑞2 𝑟
 Number of electric lines emerge out from 𝑞2 is Φ2 = = 25  If the two balls touch each other and then separated, then the charge on each
𝜀0
𝑞 +𝑞
 Thus ;
Φ1 𝒒
= | 𝟏| =
11 ball becomes 1 2
Φ2 𝒒𝟐 25 2
(𝑞 +𝑞 )2
𝟏𝟏  Then the new force ; 𝐹𝑓𝑖𝑛𝑎𝑙 = 𝑘 1 22
4𝑟
Answer (d)  Hence 𝐹𝑓𝑖𝑛𝑎𝑙 > 𝐹𝑖𝑛𝑖𝑡𝑖𝑎𝑙
𝟐𝟓
Answer (c) more than before
victory R. SARAVANAN. M.Sc., M.Phil., B.Ed PG ASST [PHYSICS], GBHSS, PARANGIPETTAI - 608 502
12 PHYSICS UNIT - 1 ELECTROSTATICS COMPLETE STUDY MATERIAL
8. Rank the electrostatic potential energies for the given system of charges in 11. Two points A and B are maintained at a potential of 7 V and - 4 V respectively.
increasing order. The work done in moving 50 electrons from A to B is
(a) 8.80 𝑋 10−17 𝐽 (b) − 8.80 𝑋 10−17 𝐽
(c) 4.40 𝑋 10−17 𝐽 (d) 5.80 𝑋 10−17 𝐽
Solution :
 Since, 𝑉𝐴 = 7 𝑉 ; 𝑉𝐵 = −4 𝑉,then potential difference 𝑉𝐵 − 𝑉𝐴 = −4 − 7 = −11 𝑉
(a) 1 = 4 < 2 < 3 (b) 2 = 4 < 3 < 1  Thus work done, 𝑊𝐴→𝐵 = 𝑞 (𝑉𝐵 − 𝑉𝐴 ) = 𝑛 𝑒 (𝑉𝐵 − 𝑉𝐴 )
(c) 2 = 3 < 1 < 4 (d) 3 < 1 < 2 < 4 𝑊𝐴→𝐵 = 50 𝑋 1.6 𝑋 10−19 𝑋 (−11) = 8.8 𝑋 10−17 𝐽
Solution : Answer (a) 𝟖. 𝟖𝟎 𝑿 𝟏𝟎−𝟏𝟕 𝑱
1 𝑄 (−𝑄) 1 𝑄2 12. If voltage applied on a capacitor is increased from V to 2V, choose the correct
 In fig (1) ⇒ 𝑈1 = =−
4 𝜋 𝜀0 𝑟 4 𝜋 𝜀0 𝑟 conclusion.
1 (−𝑄) (−𝑄) 1 𝑄2
 In fig (2) ⇒ 𝑈2 = = (a) Q remains the same, C is doubled
4 𝜋 𝜀0 𝑟 4 𝜋 𝜀0 𝑟
1 (−𝑄) (−2𝑄) 1 2 𝑄2 (b) Q is doubled, C doubled
 In fig (3) ⇒ 𝑈3 = = (c) C remains same, Q doubled
4 𝜋 𝜀0 𝑟 4 𝜋 𝜀0 𝑟
1 𝑄 (−2𝑄) 1 2 𝑄2 1 𝑄2 (d) Both Q and C remain same
 In fig (4) ⇒ 𝑈4 = = − =−
4 𝜋 𝜀0 𝑟 4 𝜋 𝜀0 2 𝑟 4 𝜋 𝜀0 𝑟 Solution :
Answer (a) 1=4<2<3  The charge stored in the capacitor is proportional to the potential difference

9. An electric field 𝑬 = 𝟏𝟎 𝒙 𝒊̂ exists in a certain region of space. Then the between the plates (i.e.) 𝑸 ∝ 𝑽 (or) 𝑸 = 𝑪 𝑽
potential difference V = Vo – VA, (where Vo is the potential at the origin and VA  Where the C is the proportionality constant called capacitance which depends
is the potential at x = 2 m ) is shape and size of the conductor
(a) 10 V (b) –20 V (c) +20 V (d) –10 V  When 𝑽 → 𝟐 𝑽 , then 𝑸 → 𝟐 𝑸
Solution : Answer (c) C remains same, Q doubled
 The potential at any point is given by ; 13. A parallel plate capacitor stores a charge Q at a voltage V. Suppose the area of
𝑥2 the parallel plate capacitor and the distance between the plates are each
𝑉𝑥 = − ∫ 𝐸⃗ . ⃗⃗⃗⃗
𝑑𝑟 = − ∫ 10 𝑥 𝑖̂ . 𝑑𝑥𝑖̂ = −10 ∫ 𝑥 𝑑𝑥 = −10 [ ] = −5 𝑥 2
2 doubled then which is the quantity that will change?
 At orgin (𝑥 = 0) ; 𝑉𝑜 = −5 (0)2 = 0 𝑉
(a) Capacitance (b) Charge
At 𝑥 = 2 ; 𝑉𝐴 = −5 (2)2 = − 5 𝑋 4 = −20 𝑉
(c) Voltage (d) Energy density
 Then the potential difference ; 𝑉 = 𝑉𝑜 − 𝑉𝐴 = 0 − (−20) = +20 𝑉
Solution :
Answer (c) + 20 V  When 𝑨𝟏 → 𝟐 𝑨 and 𝒅𝟏 → 𝟐 𝒅 then,
10. A thin conducting spherical shell of radius R has a charge Q which is uniformly 𝑸 𝜺𝒐 𝑨𝟏 𝜺𝒐 (𝟐 𝑨) 𝜺𝒐 𝑨
distributed on its surface. The correct plot for electrostatic potential due to a) New capacitance ; 𝑪𝟏 = = = = = 𝑪 (no change)
𝑽 𝒅𝟏 (𝟐 𝒅) 𝒅
this spherical shell is 𝟏 𝟏
b) New charge ; 𝑸 = 𝑪 𝑽 = 𝑪 𝑽 = 𝑸 (no change)
𝑸𝟏 𝒅𝟏 𝑸 (𝟐 𝒅) 𝑸𝒅
c) New voltage ; 𝑽𝟏 = =𝜺 = = 𝑽 (no change)
𝜺𝒐 𝑨𝟏 𝒐 (𝟐 𝑨) 𝜺𝒐 𝑨
d) New energy density ;
(a) (b) (c) (d) 𝟏 𝟐 𝟏 𝑸𝟏
𝟐
𝟏 𝑸 𝟐 𝟏 𝟏 𝑸 𝟐 𝟏 𝟏 𝟏
Solution : 𝒖𝟏𝑬 = 𝜺𝟎 𝑬𝟏 = 𝜺𝟎 [ ] = 𝜺𝟎 [ ] = ( 𝜺𝟎 [ ] )= [ 𝜺𝟎 𝑬𝟐 ] = 𝒖𝑬
𝟐 𝟐 𝜺𝒐 𝑨𝟏 𝟐 𝜺𝒐 (𝟐𝑨) 𝟒 𝟐 𝜺𝒐 𝑨 𝟒 𝟐 𝟒
1 𝑄
 For 𝑟 ≤ 𝑅 ; the electric potential 𝑉𝑟 = 𝑉𝑅 = = 𝑐𝑜𝑛𝑠𝑡𝑎𝑛𝑡 Answer (d) Energy density (quatered)
4 𝜋 𝜀0 𝑅
In this region, there is zero electric field and hence electric potential is constant
1 𝑄 1
 For 𝑟 > 𝑅 ; the electric potential 𝑉 = (or) 𝑉∝
4 𝜋 𝜀0 𝑟 𝑟
In this region, spherical shell acts similar to point charge.

Answer (b)

victory R. SARAVANAN. M.Sc., M.Phil., B.Ed PG ASST [PHYSICS], GBHSS, PARANGIPETTAI - 608 502
12 PHYSICS UNIT - 1 ELECTROSTATICS COMPLETE STUDY MATERIAL
14. Three capacitors are connected in triangle as shown in the figure. The
equivalent capacitance between the points A and C is
(a) 1 𝜇 𝐹 (b) 2 𝜇 𝐹
1
(c) 3 𝜇 𝐹 (d)
4
𝜇𝐹

Solution :

 2 𝜇 𝐹 and 2 𝜇 𝐹 are in series connection, their effective capacitance,


1 1 1
= + =1 (𝑜𝑟) 𝑪𝑺 = 𝟏 𝝁 𝑭
𝐶𝑆 2 1
 1 𝜇 𝐹 and 1 𝜇 𝐹 are in parallel connection, their effective capacitance,
𝑪𝑷 = 𝟏 + 𝟏 = 𝟐 𝝁 𝑭
Answer (b) 𝟐𝝁𝑭
15. Two metallic spheres of radii 1 cm and 3 cm are given charges of –1 × 10–2 C
and 5 × 10–2 C respectively. If these are connected by a conducting wire, the
final charge on the bigger sphere is
(a) 3 × 10–2 C (b) 4 × 10–2 C
(c) 1 × 10 C
–2 (d) 2 × 10–2 C
Solution :
 Let 𝑞1 = −1 𝑋 10−2 𝐶 and 𝑞2 = 5 𝑋 10−2 𝐶 , then the total charge will be,
𝑸 = 𝑞1 + 𝑞2 = (−1 + 5) 𝑋 10−2 𝐶 = 𝟒 𝑿 𝟏𝟎−𝟐 𝑪
 When these two spheres are connected by a conducting wire, distribution of
charge occurs between the two spheres but the total charge remains the same
(conservation of charges)
 In equilibrium stage, let 𝑄1 be the charge on small sphere, then charge on
bigger sphere will be 𝑄2 = 𝑸 − 𝑄1 = (𝟒 𝑿 𝟏𝟎−𝟐 ) − 𝑄1
 Since the two spheres are connected, the surfaces of both spheres together
form an equipotential surface. Hence
𝑉𝑠𝑚𝑎𝑙𝑙 = 𝑉𝑏𝑖𝑔𝑔𝑒𝑟
1 𝑄1 1 𝑄2
=
4 𝜋 𝜀0 𝑟𝑠𝑚𝑎𝑙𝑙 4 𝜋 𝜀0 𝑟𝑏𝑖𝑔𝑔𝑒𝑟
𝑄1 (4 𝑋 10−2 ) − 𝑄1
=
𝑟𝑠𝑚𝑎𝑙𝑙 𝑟𝑏𝑖𝑔𝑔𝑒𝑟
𝑄1 (4 𝑋 10−2 ) − 𝑄1
=
1 𝑋 10−2 3 𝑋 10−2
3 𝑄1 = (4 𝑋 10−2 ) − 𝑄1
4𝑄1 = (4 𝑋 10−2 )
𝑸𝟏 = 𝟏 𝑿 𝟏𝟎−𝟐 𝑪
 Therefore the charge on bigger sphere,
𝑄2 = (𝟒 𝑿 𝟏𝟎−𝟐 ) − 𝑄1 = (𝟒 𝑿 𝟏𝟎−𝟐 ) − (𝟏 𝑿 𝟏𝟎−𝟐 ) = 𝟑 𝑿 𝟏𝟎−𝟐 𝑪
Answer (a) 𝟑 𝑿 𝟏𝟎−𝟐 𝑪
victory R. SARAVANAN. M.Sc., M.Phil., B.Ed PG ASST [PHYSICS], GBHSS, PARANGIPETTAI - 608 502
12 PHYSICS UNIT - 1 ELECTROSTATICS COMPLETE STUDY MATERIAL

PART - II 2 MARK QUESTIONS AND ANSWERS 10. Distinguish between Coulomb force and Gravitational force.
Coulomb force Gravitational force
1. What is Electrostatics? It acts between two charges It acts between two masses
 The branch of electricity which deals with stationary charges is called
It can be attractive or repulsive It is always attractive
electrostatics.
2. What is called triboelectric charging? It is always greater in magnitude It is always lesser in magnitude
 Charging the objects through rubbing is called triboelectric charging. It depends on the nature of the medium It is independent of the medium
3. Like charges repels. Unlike charges attracts. Prove.
If charges are in motion, another force
 A negatively charged rubber rod is repeled by another negatively charged Gravitional force is the same whether
called Lorentz force come in to play in
rubber rod. But a negatively charged rubber rod is attracted by a positively two masses are at rest or in motion
addition to Coulomb force
charged glass rod.
 This proves like charges repels and unlike charges attracts. 11. Define superposition principle.
4. State conservation of electric charges.  According to Superposition principle, the total force acting on a given charge is
 The total electric charge in the universe is constant and charge can neither be equal to the vector sum of forces exerted on it by all the other charges.
created nor be destroyed 12. Define electric field.
 In any physical process, the net change in charge will be zero. This is called  The electric field at a point ‘P’ at a distance ‘r’ from the point charge ‘q’ is the
conservation of charges. force experienced by a unit charge. Its S.I unit is N C-1
5. State quantisation of electric charge. 13. Define electric field lines.
 The charge ‘q’ of any object is equal to an integral multiple of this fundamental  A set of continuous lines which are the visual representation of the electric field
unit of charge ‘e’ (i.e) 𝒒 = 𝒏 𝒆 in some region of space is calle electric field lines.
 where, n  integer and 𝒆 = 𝟏. 𝟔 𝑿 𝟏𝟎−𝟏𝟗 𝑪 14. Two electric field lines never intersect each other. Why?
6. State Coulomb’s law in electrostatics.  If two lines cross at a point, then there will be two different electric field vectors
 Coulomb law states that the electrostatic force between two point charges at the same point.
separated by a distance is  If some charge is placed at the intersection point, then it has to move in two
1) directly proportional to the product of the magnitude of two point charges different directions at the same time, which is physically impossible.
2) inversely proportional to the square of the distance between them  Hence electric field lines do not intersect.
7. Define one coulomb (1 C) 15. What is called electric dipole. Give an example.
 The S.I unit of charge is coulomb (C)  Two equal and opposite charges separated by a small distance constitute an
 One Coulomb is that charge which when placed in free space or air at a distance electric dipole.
1 m from an equal and similar charge repels with a force of 9 X 109 N (e.g) CO, HCl, NH4, H2O
8. Define relative permittivity. 16. Define electric dipole moment. Give its unit.
 The ratio of permittivity of the medium ( 𝜺) to the permittivity of free space ( 𝜺𝟎 )  The magnitude of the electric dipole moment (𝒑) is equal to the product of the
𝜺 magnitude of one of the charges (q) and the distance (2a) between them. (i.e)
is called relative permittivity or dielectric constant. [𝜺𝒓 = ]. |𝒑
⃗ | = 𝒒. 𝟐𝒂
𝜺𝟎
 It has no unit and for air 𝜀𝑟 = 1 and for other dielectric medium 𝜀𝑟 > 1  Its unit is C m.
9. Give the vector form of Coulomb’s law.  The electric dipole moment vector lies along the line joining two charges and is
 The force on the point charge 𝒒𝟐 exerted by another point charge 𝒒𝟏 is directed from – q to +q.
𝟏 𝒒𝟏 𝒒𝟐 17. Define potential difference. Give its unit.
⃗⃗⃗𝑭𝟐𝟏 = 𝒓̂𝟏𝟐  The electric potential difference is defined as the workdone by an external force
𝟒 𝝅 𝜺𝟎 𝒓 𝟐
 Simillarly the force on charge 𝒒𝟏 by charge 𝒒𝟐 is to bring unit positive charge from one point to another point against the electric
𝟏 𝒒𝟏 𝒒𝟐 field. Its unit is volt (V)
⃗⃗⃗𝑭𝟏𝟐 = 𝒓̂𝟐𝟏 18. Define electrostatic potential. Give its unit.
𝟒 𝝅 𝜺𝟎 𝒓 𝟐
 The electric potential at a point is equal to the work done by an external force to
 Here, 𝒓̂𝟏𝟐 → unit vector directed from 𝒒𝟏 to 𝒒𝟐
bring a unit positive charge with constant velocity from infinity to the point in
𝒓̂𝟐𝟏 → unit vector directed from 𝒒𝟐 to 𝒒𝟏
the region of the external electric field. Its unit is volt (V)

victory R. SARAVANAN. M.Sc., M.Phil., B.Ed PG ASST [PHYSICS], GBHSS, PARANGIPETTAI - 608 502
12 PHYSICS UNIT - 1 ELECTROSTATICS COMPLETE STUDY MATERIAL
19. Obtain the relation between electric field and electric potential. 28. What are called polar molecules? Give examples.
 The work done in moving a unit charge through a distance ‘dx’ in an electric  A polar molecule is one in which the positive and negative charges are
field ‘E’ is 𝑑𝑊 = − 𝐸 𝑑𝑥 separated even in the absence of an external electric field.
 Here negative sign indicates work done is against the electric field.  They have a permanent dipole moment. (e.g) H2O, N2O, HCl, NH4
 This work done is equal to the potential difference and hence, 29. Define dielectric polarization.
𝑑𝑉 = − 𝐸 𝑑𝑥 (𝑜𝑟) 𝑬= −
𝒅𝑽  In the presence of external electric field, dipole moment is induced in the
𝒅𝒙
dielectric along the direction of the field.
 Thus the electric field is the negative gradient of electric potential.
 Polarisation (𝑝) is defined as the total dipole moment per unit volume of the
20. Define electrostatic potential energy.
dielectric.
 The electric potential energy of two point charges is equal to the amount of
30. Define electric susceptibility.
workdone to assemble the charges or workdone in bringing a charge from
 For dielectrics, the polarization is directly proportional to the strength of the
infinite distance. (i.e) U = W = q V
21. Define electric flux. external electric field. (i.e) ⃗𝑷
⃗ = 𝝌𝒆 𝑬
⃗ 𝒆𝒙𝒕
 The number of electric field lines crossing a given area kept normal to the  where 𝝌𝒆 is a constant called the electric susceptibility which is defined as
electric field lines is called electric flux (𝐸 ). polarization per unit electric field. Its unit is 𝑪𝟐 𝑵−𝟏 𝒎
31. Define dielectric breakdown.
 Its S.I unit is 𝑵 𝒎𝟐 𝑪−𝟏 . It is a scalar quantity.
 When the external electric field applied to dielectric is very large, it tears the
22. State Gauss law.
atoms apart so that the bound charges become free charges. Then the dielectric
Gauss law states that if a charge ‘Q’ is enclosed by an arbitrary closed surface,
1 starts to conduct electricity. This is called dielectric breakdown.
then the total electric flux through the closed surface is equal to times the net 32. Define dielectric strength.
𝜀𝑂
𝑸𝒊𝒏𝒔𝒊𝒅𝒆
charge enclosed by the surface. ⃗ . 𝒅𝑨
𝑬 = ∮ 𝑬 ⃗⃗ =  The maximum electric field the dielectric can withstand before it breakdowns is
𝜺𝟎
called dielectric strength.
23. Define electrostatic shielding .
 The dielectric strength of air is 𝟑 𝑿 𝟏𝟎𝟔 𝑽 𝒎−𝟏 . If the applied electric field is
 By Gauss law, we conclude that the electric field inside the charged spherical
increases beyond this, a spark is produced in the air (i.e) it becomes a conductor
shell is zero.
33. What is called a capacitor?
 If a conductor has cavity, then whatever the charges at the surfaces or whatever
 Capacitor is a device used to store electric charge and electric energy.
the electrical diesturbances outside, the electric field inside the cavity is zero.
 It consists of two conducting plates or sheets separated by some distance.
 A sensitive electrical instrument which is to be protected from external
34. Define capacitance of a capacitor.
electrical disturnance is kept inside this cavity. This is called electrostatic
 The capacitance of a capacitor is defined as the ratio of the magnitude of charge
shielding. (e.g) Faraday cage
(Q) on either of the conductor plates to the potential difference (V) existing
24. During lightning, it is safer to sit inside bus than in an open ground or under tree.
between the conductors. (i.e) C = Q/V
Why?
 Its unit is farad (F) or C V-1
 The metal body of the bus provides electrostatic shielding, where the electric
35. Define energy density of a capacitor.
field is zero.
 The energy stored per unit volume of space is defined as energy density and it is
 During lightning the electric discharge passes through the body of the bus.
derived as,
 Thus it is safer to be inside the bus or car than in an open ground or under tree. 𝑼 𝟏
25. Define electrostatic induction. 𝒖𝑬 = = 𝜺𝒐 𝑬𝟐
𝒗𝒐𝒍𝒖𝒎𝒆 𝟐
 The phenomenom of charging without actual contact of charged body is called 36. Define action of point or corona discharge.
electrostatic induction.  Smaller the radius of curvature, larger the charge density. Hence charges are
26. Define dielectrics or insulators. accumulated at the sharp points.
 A dielectric is a non- conducting material and has no free electrons. The  Due to this, the electric field near this sharp edge is very high and it ionized the
electrons in a dielectric are bound within the atoms. surrounding air.
(e.g) Ebonite. glass and mica
 The positive ions are repelled and negative ions are attracted towards the sharp
27. What are called non-polar molecules? Give examples.
edge.
 A non-polar molecule is one in which centres of positive and negative charges
 This reduces the total charge of the conductor near the sharp edge. This is called
coincide. It has no permanent dipole moment. (e.g) H2, O2, CO2
action of points or corona discharge
victory R. SARAVANAN. M.Sc., M.Phil., B.Ed PG ASST [PHYSICS], GBHSS, PARANGIPETTAI - 608 502
12 PHYSICS UNIT - 1 ELECTROSTATICS COMPLETE STUDY MATERIAL
1 𝑞2
PART - III 3 MARK QUESTIONS AND ANSWERS ⃗⃗⃗𝐸2 =
2 𝑟̂2𝑃
4 𝜋 𝜀0 𝑟2𝑃
1. Discuss the basic properties of electric charge. 1 𝑞𝑛
(i) Electric charge : finally, ⃗⃗⃗𝐸𝑛 = 2 𝑟̂𝑛𝑃
4 𝜋 𝜀0 𝑟𝑛𝑃
 Like mass, the electric charge is also an intrinsic and fundamental  The total electric field at ‘P’ due to all these ‘n’ charges will be,
property of particles. The unit of electric charge is coulomb (C) ⃗⃗⃗𝐸𝑡𝑜𝑡 = ⃗⃗⃗𝐸1 + ⃗⃗⃗𝐸2 + … … + ⃗⃗⃗𝐸𝑛
(ii) Conservation of electric charge : 𝟏 𝒒𝟏 𝒒𝟐 𝒒𝒏
 The total electric charge in the universe is constant and charge can neither ⃗⃗⃗𝑬𝒕𝒐𝒕 = [ 𝒓̂ + 𝒓̂ + ⋯ + 𝟐 𝒓̂𝒏𝑷 ]
be created nor be destroyed. 𝟒 𝝅 𝜺𝟎 𝒓𝟐𝟏𝑷 𝟏𝑷 𝒓𝟐𝟐𝑷 𝟐𝑷 𝒓𝒏𝑷
 In any physical process, the net change in charge will be zero. This is called 4. List the properties of electric field lines.
conservation of charges Properties of electric field lines :
(iii) Quanisation of charge : 1) They starts from positive charge and end at negative charge or at infinity.
 The charge ‘q’ of any object is equal to an integral multiple of this 2) The electric field vector at a point in space is tangential to the electric field line
fundamental unit of charge ‘e’ (i.e) 𝒒 = 𝒏 𝒆 at that point.
3) The electric field lines are denser in a region where the electric field has larger
 where n  integer and 𝒆 = 𝟏. 𝟔 𝑿 𝟏𝟎−𝟏𝟗 𝑪
magnitude and less dense in region where the electric field is of smaller
2. Define superposition principle. Explain how superposition principle
magnitude. (i.e) the number of lines passing through a given surface area
explains the interaction between multiple charges.
perpendicular to the line is proportional to the magnitude of the electric field.
Superposition principle :
4) No two electric field lines intersect each other
 According to Superposition principle, the total force acting on a given charge is
5) The number of electric field lines that emanate from the positive charge or end
equal to the vector sum of forces exerted on it by all the other charges.
at a negative charge is directly proportional to the magnitude of the charges.
Explanation :
5. Derive an expression for torque experienced by an electric dipole placed in
 Consider a system of ‘n’ charges 𝑞1 , 𝑞2 , … , 𝑞𝑛
the uniform electric field.
 By Coulomb’s law, force on 𝑞1 by 𝑞2 , … , 𝑞𝑛 are Torque experienced by the dipole in electric field :
𝑞 𝑞
⃗⃗⃗𝐹12 = 𝑘 1 2 𝑟̂21  Let a dipole of moment ⃗⃗⃗𝒑 is placed
2
𝑟21
𝑞 𝑞 in an uniform electric field ⃗⃗⃗𝑬
⃗⃗⃗𝐹13 = 𝑘 1 2 𝑟̂31  The force on ‘+q’ = +𝒒⃗⃗⃗𝑬
2
𝑟31
𝑞1 𝑞2 The force on ‘-q’ = − 𝒒⃗⃗⃗𝑬
finally. ⃗⃗⃗𝐹1𝑛 = 𝑘 𝑟̂𝑛1
2
𝑟𝑛1  Then the total force acts on the
 By superposition principle, total force action on 𝑞1 due to all charges, dipole is zero.
⃗⃗⃗𝐹1𝑡𝑜𝑡 = ⃗⃗⃗𝐹12 + ⃗⃗⃗𝐹13 + … … + ⃗⃗⃗𝐹1𝑛  But these two forces constitute a
𝒒𝟏 𝒒𝟐 𝒒𝟏 𝒒𝟑 𝒒𝟏 𝒒𝒏 couple and the dipole experience a
⃗⃗⃗𝑭𝒕𝒐𝒕
𝟏 = 𝒌[ 𝟐 𝒓̂𝟐𝟏 + 𝟐 𝒓̂𝟑𝟏 + ⋯ + 𝟐 𝒓̂𝒏𝟏 ] torque which tend to rotate the
𝒓𝟐𝟏 𝒓𝟑𝟏 𝒓𝒏𝟏
dipole along the field.
3. Explain Electric field at a point due to system of charges (or) Superposition of
 The total torque on the dipole about the point ‘O’
electric fields.
Superposition of electric field : ⃗⃗𝜏 = ⃗⃗⃗⃗⃗
𝑂𝐴 𝑋 (− 𝒒⃗⃗⃗𝑬) + ⃗⃗⃗⃗⃗ 𝑂𝐵 𝑋 (+𝒒⃗⃗⃗𝑬)
 The electric field at an arbitrary point due to system of point charges is simply |⃗⃗𝜏| = |𝑂𝐴⃗⃗⃗⃗⃗ | |− 𝒒⃗⃗⃗𝑬| sin 𝜃 + |𝑂𝐵
⃗⃗⃗⃗⃗ | |𝒒⃗⃗⃗𝑬| sin 𝜃
equal to the vector sum of the electric fields created by the individual point 𝜏 = (𝑂𝐴 + 𝑂𝐵)𝑞 𝐸 sin 𝜃
charges. This is called superposition of electric fields. 𝜏 = 2 𝑎 𝑞 𝐸 sin 𝜃 ∵ [𝑂𝐴 = 𝑂𝐵 = 𝑎]
Explanation : 𝝉 = 𝒑 𝑬 𝐬𝐢𝐧 𝜽
 Consider a system of ‘n’ charges 𝑞1 , 𝑞2 , … , 𝑞𝑛  where, 2 𝑎 𝑞 = 𝑝 → dipole moment
 The electric field at ‘P’ due to ‘n’ charges  In vector notation, ⃗⃗𝝉 = ⃗⃗⃗𝒑 𝑿 ⃗⃗⃗𝑬
1 𝑞1  The torque is maximum, when 𝜃 = 90
⃗⃗⃗𝐸1 =
2 𝑟̂1𝑃
4 𝜋 𝜀0 𝑟1𝑃

victory R. SARAVANAN. M.Sc., M.Phil., B.Ed PG ASST [PHYSICS], GBHSS, PARANGIPETTAI - 608 502
12 PHYSICS UNIT - 1 ELECTROSTATICS COMPLETE STUDY MATERIAL
6. Obtain an expression electric potential at a point due to a point charge.  The potential at ‘C’ due to charges 𝒒𝟏 & 𝒒𝟐
Potential due to a point charge : 1 𝑞1 1 𝑞2
𝑉1𝐶 = & 𝑉2𝐶 =
 Consider a point charge +𝒒 at 4 𝜋 𝜀0 𝑟13 4 𝜋 𝜀0 𝑟23
origin.  To bring third charge 𝒒𝟑 to ‘C’ , work has to be done against the electric field
 ‘P’ be a point at a distance ‘r’ due to 𝒒𝟏 & 𝒒𝟐 . Thus work done on charge 𝒒𝟑 is ,
from origin. 1 𝑞1 𝑞2
1 𝑞 𝑊 = 𝑞3 (𝑉1𝐶 + 𝑉2𝐶 ) = 𝑞3 [ + ]
 By definition, the electric field at ‘P’ is ; ⃗⃗⃗𝐸 = 𝑟̂ 4 𝜋 𝜀0 𝑟13 𝑟23
4 𝜋 𝜀0 𝑟 2 𝟏 𝒒𝟏 𝒒𝟑 𝒒𝟐 𝒒𝟑
 Hence electric potential at ‘P’ is (𝑜𝑟) 𝑼 = [ + ] − − − − − − − −(𝟐)
𝑟 𝑟 𝟒 𝝅 𝜺𝟎 𝒓𝟏𝟑 𝒓𝟐𝟑
1 𝑞  Hence the the total electrostatic potential energy of system of three point
𝑉 = − ∫ ⃗⃗⃗𝐸 . ⃗⃗⃗⃗
𝑑𝑟 = − ∫ 𝑟̂ . ⃗⃗⃗⃗
𝑑𝑟
4 𝜋 𝜀0 𝑟 2 charges is
∞ ∞ 𝟏 𝒒𝟏 𝒒𝟐 𝒒𝟏 𝒒𝟑 𝒒𝟐 𝒒𝟑
𝑟
1 𝑞 𝑼 = [ + + ] − − − − − −(𝟑)
𝑉= − ∫ 𝑟̂ . 𝑑𝑟 𝑟̂ [∵ ⃗⃗⃗⃗
𝑑𝑟 = 𝑑𝑟 𝑟̂ ] 𝟒 𝝅 𝜺𝟎 𝒓𝟏𝟐 𝒓𝟏𝟑 𝒓𝟐𝟑
4 𝜋 𝜀0 𝑟 2 8. Obtain an expression for electrostatic potential energy of a dipole in a

𝑟 uniform electric field.
𝑞 1 Potential energy of dipole in uniform electric field:
𝑉= − ∫ 2 𝑑𝑟 [∵ 𝑟̂ . 𝑟̂ = 1]
4 𝜋 𝜀0 𝑟  Let a dipole of moment ⃗⃗⃗𝒑 is placed in a

𝑞 1𝑟 𝑞 1 1 1 uniform electric field ⃗⃗⃗𝑬
𝑉 = − [− ] = [ − ] [∵ = 0]  Here the dipole experience a torque,
4 𝜋 𝜀0 𝑟 ∞ 4 𝜋 𝜀0 𝑟 ∞ ∞
𝟏 𝒒 which rotate the dipole along the field.
𝑽=  To rotate the dipole from 𝜃 to 𝜃
𝟒 𝝅 𝜺𝟎 𝒓
 If the source charge is negative (−𝑞) , then the potential also negative (i.e,) against this torque, work has to be
𝟏 𝒒 done by an external torque (𝜏𝑒𝑥𝑡 ) and it
𝑽=− is given by,
𝟒 𝝅 𝜺𝟎 𝒓  
7. Obtain an expression for potential energy due to a collection of three point
charges which are separated by finite distances. 𝑊 = ∫ 𝜏𝑒𝑥𝑡 𝑑 = ∫ 𝑝 𝐸 sin 𝜃 𝑑
Potential energy of system of three charges :  
 Electrostatic potential energy of a system 𝑊 = 𝑝 𝐸 [− cos 𝜃] = −𝑝 𝐸 [𝑐𝑜𝑠 𝜃 − 𝑐𝑜𝑠 𝜃 ]
of charges is defined as the work done to 𝑊 = 𝑝 𝐸 [𝑐𝑜𝑠 𝜃 − 𝑐𝑜𝑠 𝜃]
assemble the charges  This work done is stored as electrostatic potential energy (U) of the dipole.
 consider a point charge 𝒒𝟏 at ‘A’  Let the initial angle be 𝜃 = 90 , then
 Electric potential at ‘B’ due to 𝒒𝟏 is, 𝑈 = 𝑝 𝐸 [𝑐𝑜𝑠 90 − 𝑐𝑜𝑠 𝜃]
1 𝑞1 𝑼 = − 𝒑 𝑬 𝐜𝐨𝐬  = − ⃗⃗⃗𝒑 . ⃗⃗⃗𝑬
𝑉1𝐵 =
4 𝜋 𝜀0 𝑟12  If 𝜃 = 180 , then potential energy is maximum
 To bring second charge 𝒒𝟐 to ‘B’, work has to be done against the electric field  If 𝜃 = 0 , then potential energy is mimimum
created by 𝒒𝟏
 The work done on the charge 𝒒𝟐 is,
1 𝑞1 𝑞2
𝑊 = 𝑞2 𝑉1𝐵 =
4 𝜋 𝜀0 𝑟12
 This work done is stored as electrostatic potential energy of system of two
charges 𝒒𝟏 and 𝒒𝟐
𝟏 𝒒𝟏 𝒒𝟐
𝑼= − − − −(𝟏)
𝟒 𝝅 𝜺𝟎 𝒓𝟏𝟐

victory R. SARAVANAN. M.Sc., M.Phil., B.Ed PG ASST [PHYSICS], GBHSS, PARANGIPETTAI - 608 502
12 PHYSICS UNIT - 1 ELECTROSTATICS COMPLETE STUDY MATERIAL
9. Explain the process of electrostatic induction. 11. Derive an expression for energy stored in capacitor
Electrostatic induction: Energy stored in capacitor:
 Capacitor is a device used to store charges and energy.
 When a battery is connected to the capacitor, electrons of total charge ‘-Q’ are
transferred from one plate to other plate. For this work is done by the battery.
 This work done is strored as electrostatic energy in capacitor.
 To transfer ′𝑑𝑄′ for a potential difference ‘V’, the work done is
𝑄 𝑄
𝑑𝑊 = 𝑉 𝑑𝑄 = 𝑑𝑄 [∵ 𝑉 = ]
𝐶 𝐶
 The type of charging without actual contact of charged body is called  The total work done to charge a capacitor,
electrostatic induction. 𝑄
𝑄
𝑄 1 𝑄2 𝑄2
 Let a negatively charged rubber rod is brought near to spherical conductor, the 𝑊= ∫ 𝑑𝑄 = [ ] =
0 𝐶 𝐶 2 0 2𝐶
electrons in the conductor are repelled to farther side and hence positive
charges are induced near the region of the rod. So the distribution of charges  This work done is stored as electrostatic energy of the capacitor, (i.e)
are not uniform, but the total charge is zero 𝑸𝟐 𝟏
𝑼𝑬 = = 𝑪 𝑽𝟐 [∵ 𝑄 = 𝐶 𝑉]
 If the conducting sphere is connected to ground, the electrons are flows to the 𝟐𝑪 𝟐
𝜀𝑂 𝐴
ground, but the positive charges will not flow to the ground, because they are  We know that, 𝑉 = 𝐸 𝑑 & 𝐶=
𝑑
attracted by the negative charges of the rod. ∴ 𝑈𝐸 =
1 𝜀𝑂 𝐴
(𝐸 𝑑)2 =
1
𝜀𝑂 (𝐴 𝑑) 𝐸 2
[here, (𝐴 𝑑) → 𝑣𝑜𝑙𝑢𝑚𝑒]
2 𝑑 2
 When the grounding wire is removed from the sphere, the positive charges
 The energy stored per unit volume of space is defined as energy density ((𝒖𝑬 ).
remain near the rod.
𝑼𝑬 𝟏
 If the charged rod is taken away, the positive charges are distributed uniformly 𝒖𝑬 = = 𝜺𝑶 𝑬𝟐
on the surface of the sphere. 𝒗𝒐𝒍𝒖𝒎𝒆 𝟐
12. Explain the principle, construction and action of lightning conductor.
 Thus the neutral conducting sphere becomes positively charged without any
Lightning conductor :
contact.
 This is a device used to protect tall building
10. Derive an expression for capacitance of parallel plate capacitor.
from lightning strikes;
Capacitance of parallel plate capacitor :
 It woks on the principle of acion of points or
 Consider a capacitor consists of two parallel plates each of area ‘A’ separated
corona discharge.
by a distance ‘d’
 It consists of a long thick copper rod passing
 Let ‘𝝈′ be the surface charge density of the plates.
from top of the building to the ground.
 The electric field between the plates,
𝝈 𝑸  The upper end of the rod has a sharp spike or a
𝑬= = − − − − − − − (1) sharp needle. The lower end of the rod is
𝜺𝑶 𝑨 𝜺𝑶
connected to the copper plate which is buried
 Since the field is uniform, the potential difference deep in to the ground.
between the plates,
𝑸  When a negatively charged cloud is passing
𝑽 =𝑬𝒅= [ ]𝒅 − − − − − (2) above the building, it induces a positive charge
𝑨 𝜺𝑶 on the spike.
 Then the capacitance of the capacitor,  Since the charge density is large at the spike,
𝑄 𝑄
𝐶= = action of point takes place.
𝑉 𝑄  This positive charge ionizes the surrounding air which in turn neutralizes
[ ]𝑑
𝐴 𝜀𝑂
𝜺𝑶 𝑨 the negative charge in the cloud.
𝑪= − − − − − − − (𝟑)  The negative charge pushed to the spikes passes through the copper rod
𝒅
and is safely diverted to the Earth.
 Thus capacitance is,
(i) directly proportional to the Area (A) and  Thus the lighting arrester does not stop the lightning, but it diverts the
(ii) inversely proportional to the separation (d) lightning to the ground safely

victory R. SARAVANAN. M.Sc., M.Phil., B.Ed PG ASST [PHYSICS], GBHSS, PARANGIPETTAI - 608 502
12 PHYSICS UNIT - 1 ELECTROSTATICS COMPLETE STUDY MATERIAL
13. Give the applications and disadvantage of capacitors
Applications of capacitor:
 Flash capacitors are used in digital camera to take photographs
 During cardiac arrest, a device called heart defibrillator is used to give a
sudden surge of a large amount of electrical energy to the patient’s chest to
retrieve the normal heart function. This defibrillator uses a capacitor of 175 µF
charged to a high voltage of around 2000 V
 Capacitors are used in the ignition system of automobile engines to eliminate
sparking.
 Capacitors are used to reduce power fluctuations in power supplies and to
increase the efficiency of power transmission.
Disadvantages :
 Even after the battery or power supply is removed, the capacitor stores
charges and energy for some time. It caused unwanted shock.
14. Define equipotential surface. Give its properties.
Equipotential surface:
 An equipotential surface is a surface on which all the points are at the same
potential.
1) For a point charge the equipotential surfaces are concentric spherical
surfaces.
2) For a uniform electric field, the equipotential surfaces form a set of planes
normal to the electric field.
Properties :
 The wor kdone to move a charge ‘q’ between any two points A and B is 𝑊 =
𝑞 (𝑉𝐴 − 𝑉𝐵 ). If A and B lie on the same equipotential surface then 𝑉𝐴 = 𝑉𝐵
Hence work done is zero (𝑊 = 0)
 The electric field is always normal to an equipotential surface.
15. Write a note on microwave oven.
Microwave oven :
 It works on the principle of torque acting on an electric dipole.
 The food we consume has water molecules which are permanent electric
dipoles. Oven produce microwaves that are oscillating electromagnetic fields
and produce torque on the water molecules.
 Due to this torque on each water molecule, the molecules rotate very fast and
produce thermal energy.
 Thus, heat generated is used to heat the food.

victory R. SARAVANAN. M.Sc., M.Phil., B.Ed PG ASST [PHYSICS], GBHSS, PARANGIPETTAI - 608 502
12 PHYSICS UNIT - 1 ELECTROSTATICS COMPLETE STUDY MATERIAL
2. Define electric field. Explain its various aspects.
PART - IV 5 MARK QUESTIONS AND ANSWERS Electric field :
1. Explain in detail Coulomb’s law and its various aspects.  The electric field at the point ‘P’ at a distance ‘r’ from the point charge ‘q’ is the
Coulomb’s law : force experienced by a unit charge and is given by
⃗⃗⃗𝑭 𝟏 𝒒
 Consider two point charges 𝒒𝟏 and ⃗⃗⃗𝑬 = = 𝒓̂
𝒒𝟐 separated by a distance ′𝒓′ 𝒒𝒐 𝟒 𝝅 𝜺𝒐 𝒓 𝟐
 According to Coulomb law, the force on the Important aspects :
point charge 𝒒𝟐 exerted by 𝒒𝟏 is  If ‘q’ is positive, the electric field points away and if ‘q’ is negative the electric
𝒒 𝒒 field points towards the source charge.
⃗⃗⃗𝑭𝟐𝟏 = 𝒌 𝟏 𝟐 𝒓̂𝟏𝟐
𝒓𝟐
 where, k → constant
𝒓̂𝟏𝟐 → unit vector directed from 𝒒𝟏 to 𝒒𝟐
Important aspects :  The force experienced by the test charge 𝒒𝒐 placed in electric field ⃗⃗⃗𝐸 is ,
 Coulomb law states that the electrostatic force is ⃗⃗⃗𝑭 = 𝒒𝒐 ⃗⃗⃗𝑬
1) directly proportional to the product of the magnitude of two point charges  The electric field is independent of test charge 𝒒𝒐 and it depends only on
2) inversely proportional to the square of the distance between them souce charge 𝒒
 The force always lie along the line joining the two charges.  Electric field is a vector quantity. So it has unique direction and magnitude at
𝟏
 In S.I units, 𝒌 = = 𝟗 𝑿 𝟏𝟎𝟗 𝑵 𝒎𝟐 𝑪−𝟐 every point.
𝟒 𝝅𝜺𝟎
 Since electric field is inversely proportional to the distance, as distance
 Here is the permittivity of free space or vacuum and its value is
𝟏 increases the field decreases.
𝜺𝟎 = = 𝟖. 𝟖𝟓 𝑿 𝟏𝟎−𝟏𝟐 𝑪𝟐 𝑵−𝟏 𝒎−𝟐  The test charge is made sufficiently small such that it will not modify the
𝟒 𝝅𝒌
 The magnitude of electrostatic force between two charges each of 1 C separated electric field of the source charge.
by a distance of 1 m is 𝟗 𝑿 𝟏𝟎𝟗 𝑵  For continuous and finite size charge distributions, integration techniques
must bt used
 The Coulomb law in vacuum and in medium are,
𝟏 𝒒𝟏 𝒒𝟐 𝟏 𝒒𝟏 𝒒𝟐  There are two kinds of electric field. They are
⃗𝑭𝟐𝟏 = 𝒓̂𝟏𝟐 & ⃗𝑭𝟐𝟏 = 𝒓̂𝟏𝟐 (1) Uniform or constant field
𝟒 𝝅𝜺𝟎 𝒓𝟐 𝟒 𝝅𝜺 𝒓𝟐
(2) Non uniform field
where, 𝜀 = 𝜀𝑜 𝜀𝑟 −→ permittivity of the medium
𝜀 3. Calculate the electric field due to a dipole on its axial line.
 Thus the relative permittivity of the given medium is defined as , 𝜀𝑟 = . For Electric field due to dipole on its axial line :
𝜀𝑜
air or vacuum, 𝜀𝑟 = 1 and for all other media 𝜀𝑟 > 1
 Coulomb’s law has same structure as Newton’s law of gravitation. (i.e)
𝑞1 𝑞2 𝑚1 𝑚2
𝐹𝐶𝑜𝑢𝑙𝑜𝑚𝑏 = 𝑘 2 & 𝐹𝑁𝑒𝑤𝑡𝑜𝑛 = 𝐺
𝑟 𝑟2
 Here 𝑘 = 9 𝑋 109 𝑁 𝑚2 𝐶 −2 and 𝐺 = 6.626 𝑋 10−11 𝑁 𝑚2 𝑘𝑔−2
 Since ‘k’ is much more greater than ‘G’, the electrostatic force is always greater  Consider a dipole AB along X - axis. Its diplole moment be 𝒑 = 𝟐𝒒𝒂 and its
than gravitational force for smaller size objects direction be along − 𝒒 to + 𝒒 .
 Electrostatic force between two point charges depends on the nature of the  Let ‘C’ be the point at a distance ‘r’ from the mid point ‘O’ on its axial line.
medium in which two charges are kept at rest.  Electric field at C due to +𝒒
 Depending upon the nature of the charges, it may either be attractive or 𝟏 𝒒
repulsive ⃗+=
𝑬 ̂
𝒑
𝟒 𝝅 𝜺𝒐 (𝒓 − 𝒂)𝟐
 If the charges are in motion, another force called Lorentz force come in to play  Electric field at C due to −𝒒
in addition with Coulomb force. 𝟏 𝒒
⃗−= −
𝑬 ̂
𝒑
 Electrostatic force obeys Newton’s third law. (i.e) ⃗𝑭𝟐𝟏 = − ⃗𝑭𝟏𝟐 𝟒 𝝅 𝜺𝒐 (𝒓 + 𝒂)𝟐
 Since +𝒒 is located closer to pont ‘C’ than −𝒒 , 𝑬 ⃗+> 𝑬 ⃗−

victory R. SARAVANAN. M.Sc., M.Phil., B.Ed PG ASST [PHYSICS], GBHSS, PARANGIPETTAI - 608 502
12 PHYSICS UNIT - 1 ELECTROSTATICS COMPLETE STUDY MATERIAL
 By superposition principle, the total electric field at ‘C’ due to dipole is,  Here the perpendicular components | 𝑬 ⃗ + | 𝒔𝒊𝒏 𝜽 and | 𝑬 ⃗ − | 𝒔𝒊𝒏 𝜽 are equal
𝐸⃗𝑡𝑜𝑡 = 𝐸⃗+ + 𝐸⃗− and opposite will cancel each other
1 𝑞 1 𝑞  But the horizontal components | ⃗𝑬+ | 𝒄𝒐𝒔 𝜽 and | ⃗𝑬− | 𝒄𝒐𝒔 𝜽 are equal and in
𝐸⃗𝑡𝑜𝑡 = 𝑝̂ − 𝑝̂
4 𝜋 𝜀𝑜 (𝑟 − 𝑎)2 4 𝜋 𝜀𝑜 (𝑟 + 𝑎)2 same direction (−𝒑 ̂) will added up to give total electric field. Hence
1 1 1 ⃗𝐸𝑡𝑜𝑡 = | 𝐸⃗+ | 𝑐𝑜𝑠 𝜃 (−𝑝̂ ) + | 𝐸⃗− | 𝑐𝑜𝑠 𝜃 (−𝑝̂ )
𝐸⃗𝑡𝑜𝑡 = 𝑞 [ − ] 𝑝̂
4 𝜋 𝜀𝑜 (𝑟 − 𝑎)2 (𝑟 + 𝑎)2
2
(𝑟 + 𝑎) − (𝑟 − 𝑎) 2 (𝑜𝑟) 𝐸⃗𝑡𝑜𝑡 = − 2 | 𝐸⃗+ | 𝑐𝑜𝑠 𝜃 𝑝̂
1
𝐸⃗𝑡𝑜𝑡 = 𝑞 [ ] 𝑝̂ 1 𝑞
4 𝜋 𝜀𝑜 (𝑟 − 𝑎)2 (𝑟 + 𝑎)2 𝐸⃗𝑡𝑜𝑡 = − 2 [ ] cos 𝜃 𝑝̂
4 𝜋 𝜀𝑜 (𝑟 2 + 𝑎2 )
1 𝑟 2 + 𝑎2 + 2 𝑟 𝑎 − 𝑟 2 − 𝑎2 + 2 𝑟 𝑎 1 2𝑞 𝑎
𝐸⃗𝑡𝑜𝑡 = 𝑞 [ ] 𝑝̂ 𝐸⃗𝑡𝑜𝑡 = − [ ]
4 𝜋 𝜀𝑜 〈(𝑟 − 𝑎)(𝑟 + 𝑎)〉2 1 𝑝̂
4 𝜋 𝜀𝑜 (𝑟 + 𝑎 ) (𝑟 2
2 2
1 4𝑟𝑎 + 𝑎 2 )2
𝐸⃗𝑡𝑜𝑡 = 𝑞 [ 2 ] 𝑝̂ 1 2𝑞𝑎
4 𝜋 𝜀𝑜 〈𝑟 − 𝑎2 〉2 𝐸⃗𝑡𝑜𝑡 = − 3 𝑝̂
 Here the direction of total electric field is the dipole moment ⃗⃗⃗𝒑. 4 𝜋 𝜀𝑜 (𝑟 2
+ 𝑎 2 )2
 If 𝑟 ≫ 𝑎 , then neglecting 𝑎2 . We get 1 𝑝 𝑝̂ 1 ⃗⃗⃗𝑝
1 4𝑟𝑎 1 4𝑎 𝐸⃗𝑡𝑜𝑡 = − 3 = − 3
𝐸⃗𝑡𝑜𝑡 = 𝑞 [ 4 ] 𝑝̂ = 𝑞 [ 3 ] 𝑝̂ 4 𝜋 𝜀𝑜 (𝑟 2 4 𝜋 𝜀𝑜 (𝑟 2
+ 𝑎2 )2 + 𝑎 2 )2
4 𝜋 𝜀𝑜 𝑟 4 𝜋 𝜀𝑜 𝑟
𝟏 𝟐 ⃗⃗⃗𝒑  If 𝑟 ≫ 𝑎 then neglecting 𝑎2
⃗ 𝒕𝒐𝒕 =
𝑬 [∵ 𝑞 2𝑎 𝑝̂ = ⃗⃗⃗𝑝 ] 𝟏 ⃗⃗⃗𝒑
𝟒 𝝅 𝜺𝒐 𝒓 𝟑 ⃗𝑬𝒕𝒐𝒕 = − [∵ 𝑞 2𝑎 𝑝̂ = 𝑝 𝑝̂ = ⃗⃗⃗𝑝 ]
𝟒 𝝅 𝜺𝒐 𝒓 𝟑
4. Calculate the electric field due to a dipole on its equatorial line.
5. Derive an expression for electro static potential due to electric dipole.
Electric field due to dipole on its equatorial line :
Electrostatic potential due to dipole :

 Consider a dipole AB along X - axis. Its diplole moment be 𝒑 = 𝟐𝒒𝒂 and its  Consider a dipole AB along X - axis. Its diplole moment be 𝒑 = 𝟐𝒒𝒂 and its
direction be along − 𝒒 to + 𝒒 . direction be along − 𝒒 to + 𝒒
 Let ‘C’ be the point at a distance ‘r’ from the mid point ‘O’ on its equatorial  Let ‘P’ be the point at a distance ‘r’ from the mid point ‘O’
plane.  Let ∠𝑃𝑂𝐴 = 𝜃, 𝐵𝑃 = 𝑟1 and 𝐴𝑃 = 𝑟2
 Electric field at C due to +𝒒 (along BC)  Electric potential at P due to +𝒒
𝟏 𝒒 1 q
| ⃗𝑬+ | = V1 =
𝟒 𝝅 𝜺𝒐 (𝒓𝟐 + 𝒂𝟐 ) 4 πε0 r1
 Electric field at C due to −𝒒 (along CA)  Electric potential at P due to −𝒒
𝟏 𝒒 1 q
⃗ −| =
| 𝑬 V2 = −
𝟒 𝝅 𝜺𝒐 (𝒓 + 𝒂𝟐 )
𝟐 4 πε0 r2
 Here | 𝑬⃗ +| = | 𝑬⃗ −|  Then total potential at ‘P’ due to dipole is
1 1 1
 ⃗ ⃗
Resolve 𝑬+ and 𝑬− in to two components. V = V1 + 𝑉2 = q [ − ] − − − − − (1)
4 πε0 r1 r2
victory R. SARAVANAN. M.Sc., M.Phil., B.Ed PG ASST [PHYSICS], GBHSS, PARANGIPETTAI - 608 502
12 PHYSICS UNIT - 1 ELECTROSTATICS COMPLETE STUDY MATERIAL
Apply cosine law in  BOP Apply cosine law in  AOP 6. Obtain an expression for electric field due to an infinitely long charged wire.
r22 = r 2 + a2 − 2 r a cos (180 − θ) Electric field due to infinitely long charged wire :
r12 = r 2 + a2 − 2 r a cos θ r22 = r 2 + a2 + 2 r a cos θ  Consider an infinitely long straight wire of
a2 2 a a2 2 a uniform linear charge density ‘’
r12 = r 2 [1 + 2 − cos θ] r22 = r 2 [1 + 2 + cos θ]  Let ‘P’ be a point at a distance ‘r’ from the wire.
r r r r
𝑎2 𝑎2 Let ‘E’ be the electric field at ‘P’
 If 𝑎 ≪ 𝑟 then neglecting 2  If 𝑎 ≪ 𝑟 then neglecting 2  Consider a cylindrical Gaussian surface of length
𝑟 𝑟
2 2
2 a 2 2
2a ‘L’ and radius ‘r’
r1 = r [1 − cos θ] r2 = r [1 + cos θ]
r r  The electric flux through the top surface,
1 1
2d 2 2a 2 ⃗⃗⃗⃗⃗ = ∫ 𝐸 𝑑𝐴 cos 90 = 0
Φ𝑡𝑜𝑝 = ∫ ⃗⃗⃗𝐸 . 𝑑𝐴
r1 = r [1 − cos θ] r2 = r [1 + cos θ]
r r

1 −
1  The electric flux through the bottom surface,
1 1 2a 2 1 1 2a 2
= [1 − cos θ]
r2
= [1 +
r r
cos θ] Φ𝑏𝑜𝑡𝑡𝑜𝑚 = ∫ ⃗⃗⃗𝐸 . ⃗⃗⃗⃗⃗
𝑑𝐴 = ∫ 𝐸 𝑑𝐴 cos 90 = 0
r1 r r
 We have, (1 − 𝑥)−𝑛 = 1 + 𝑛𝑥  We have, (1 + 𝑥)−𝑛 = 1 − 𝑛𝑥  The electric flux through the curved surface,
1 1 a 1 1 a
= [1 + cos θ] = [1 − cos θ] Φ𝑐𝑢𝑟𝑣𝑒 = ∫ ⃗⃗⃗𝐸 . ⃗⃗⃗⃗⃗
𝑑𝐴 = ∫ 𝐸 𝑑𝐴 cos 0 = 𝐸 ∫ 𝑑𝐴 = 𝐸 2 𝜋 𝑟 𝐿
r1 r r r2 r r
 Then equation (1) becomes,  Then the total electric flux through the Gaussian surface,
1 1 𝑎 1 𝑎 Φ𝐸 = Φ𝑡𝑜𝑝 + Φ𝑏𝑜𝑡𝑡𝑜𝑚 + Φ𝑐𝑢𝑟𝑣𝑒
𝑉 = 𝑞 { [1 + 𝑐𝑜𝑠 𝜃] − [1 − 𝑐𝑜𝑠 𝜃]} 𝚽𝑬 = 𝑬 (𝟐 𝝅 𝒓 𝑳)
4𝜋𝜀0 𝑟 𝑟 𝑟 𝑟
1 𝑞 𝑎 𝑎  Since  be the charge per unit length of the wire, the net charge enclosed in
𝑉 = [1 + 𝑐𝑜𝑠 𝜃 − 1 + 𝑐𝑜𝑠 𝜃] the Gaussian cylinder of length L is; 𝑸𝒊𝒏 = 𝝀 𝑳
4𝜋𝜀0 𝑟 𝑟 𝑟
1 𝑞 2𝑎 1 2𝑞𝑎  By Gauss law,
𝑉 = 𝑐𝑜𝑠 𝜃 = 𝑐𝑜𝑠 𝜃 𝑄𝑖𝑛
4𝜋𝜀0 𝑟 𝑟 4𝜋𝜀0 𝑟 2 Φ𝐸 =
𝟏 𝒑 𝜀𝑜
𝑽 = 𝒄𝒐𝒔 𝜽 [∵ 𝑝 = 2𝑞𝑎] 𝜆𝐿
𝟒𝝅𝝐𝟎 𝒓𝟐 𝐸 (2 𝜋 𝑟 𝐿) =
𝟏 ⃗⃗⃗⃗⃗𝒑 . 𝒓̂ 𝜀𝑜
(𝒐𝒓) 𝑽 = [∵ 𝑝 𝑐𝑜𝑠 𝜃 = ⃗⃗⃗⃗⃗
𝑝 . 𝑟̂ ] 𝝀
𝟒𝝅𝝐𝟎 𝒓𝟐 𝑬 =
 Here 𝑟̂ is the unit vector along OP 𝟐 𝝅 𝜺𝒐 𝒓
case -1 : If point ‘P’lies on axial line near positive charge, then  = 0°  In Vector notation,
𝝀
𝟏 𝒑 ⃗⃗⃗
𝑬 = 𝒓̂
∴ 𝑽 = 𝟐 𝝅 𝜺𝒐 𝒓
𝟒𝝅𝜺𝟎 𝒓𝟐
 Here ̂𝒓 → unit vector perpendicular to the curved surface outwards.
Case -2 : If point ‘P’lies on axial line near positive charge, then  = 180°
𝟏 𝒑  If 𝜆 > 0 , then ⃗⃗⃗𝐸 points perpendicular outward (𝑟̂ ) from the wire and
∴ 𝑽 =− if 𝜆 < 0 , then ⃗⃗⃗𝐸 points perpendicular inward (− 𝑟̂ )
𝟒𝝅𝜺𝟎 𝒓𝟐
Case -3 : If point ‘P’lies on equitorial plane, then  = 90° 7. Obtain an expression for electric field due to an charged infinite plane sheet.
∴ 𝑽 = 𝟎 Electric field due to charged infinite plane sheet :
 Consider an infinite plane
sheet of uniform surface
charge density ‘𝜎’
 Let ‘P’ be a point at a
distance ‘r’ from the sheet.
Let ‘E’ be the electric field at
‘P’

victory R. SARAVANAN. M.Sc., M.Phil., B.Ed PG ASST [PHYSICS], GBHSS, PARANGIPETTAI - 608 502
12 PHYSICS UNIT - 1 ELECTROSTATICS COMPLETE STUDY MATERIAL
 Here the direction of electric field is perpendicularly outward from the sheet.  The electric flux through the Gaussian surface,
 Consider a cylindrical Gaussian surface of length ‘2r’ and area of cross section ‘A’ ⃗⃗⃗⃗⃗ = ∮ 𝐸 𝑑𝐴 𝑐𝑜𝑠 0
Φ𝐸 = ∮ 𝐸⃗ . 𝑑𝐴
 The electric flux through plane surface ‘P’
⃗⃗⃗⃗⃗ = ∫ 𝐸 𝑑𝐴 cos 0 = ∫ 𝐸 𝑑𝐴
Φ𝑃 = ∫ ⃗⃗⃗𝐸 . 𝑑𝐴 𝚽𝑬 = 𝑬 ∮ 𝒅𝑨 = 𝑬 (𝟒 𝝅 𝒓𝟐 )
 The electric flux through plane surface ‘P’  By Gauss law,
𝑄𝑖𝑛
⃗⃗⃗⃗⃗ = ∫ 𝐸 𝑑𝐴 cos 0 = ∫ 𝐸 𝑑𝐴
𝚽𝑷 = ∫ ⃗⃗⃗𝐸 . 𝑑𝐴 Φ𝐸 =
𝜀𝑜
 The electric flux through the curved surface, 2
𝑄
𝐸 (4 𝜋 𝑟 ) =
Φ𝑐𝑢𝑟𝑣𝑒 = ∫ ⃗⃗⃗𝐸 . ⃗⃗⃗⃗⃗
𝑑𝐴 = ∫ 𝐸 𝑑𝐴 cos 90 = 0 𝜀𝑜
𝟏 𝑸
𝑬 =
 The total electric flux through through the Gaussian surface, 𝟒 𝝅 𝜺𝒐 𝒓 𝟐
Φ𝐸 = Φ𝑃 + 𝚽𝑷 + Φ𝑐𝑢𝑟𝑣𝑒  In vector notation,
𝚽𝑬 = ∫ 𝐸 𝑑𝐴 + ∫ 𝐸 𝑑𝐴 + 0 = 2 𝐸 ∫ 𝑑𝐴 𝟏 𝑸
⃗⃗⃗ =
𝑬 𝒓̂
𝟒 𝝅 𝜺𝒐 𝒓 𝟐
𝚽𝑬 = 𝟐 𝑬 𝑨  Here ̂𝒓 → unit vector acting radiallyh outward from the spherical surface.
 Since 𝜎 be the charge per unit area of the plane sheet, the net charge enclosed 2) At a point on the surface of the shell (𝒓 = 𝑹):
in the Gaussian cylinder of area A is; 𝑸𝒊𝒏 = 𝝈 𝑨  If the point lies on the surface of the charged shell, then = 𝑹 . Then the
 By Gauss law, electric field,
𝑄𝑖𝑛 𝟏 𝑸
Φ𝐸 = ⃗⃗⃗
𝑬 = 𝒓̂
𝜀𝑜 𝟒 𝝅 𝜺 𝒐 𝑹𝟐
𝜎𝐴
2EA= 3) At a point inside the shell (𝒓 < 𝑹) ∶
𝜀𝑜  Let ‘P’ be the point inside the charged shell
𝝈
𝐄 = at a distance ‘r’ from its centre.
𝟐 𝜺𝒐  Consider the spherical Gaussian surface of
 In vector notation, radius ‘r’
𝝈
⃗⃗⃗
𝑬 = 𝒏
̂  Since there is no charge inside the
𝟐 𝜺𝒐
Gaussian surface, 𝑄𝑖𝑛 = 0
 Here ̂𝒏 → unit vector perpendicular to the plane sheet outwards.
 Then from Gauss law,
 If 𝜎 > 0 , then ⃗⃗⃗𝐸 points perpendicular outward (𝑛̂) from the plane sheet and 𝑄𝑖𝑛
if 𝜎 < 0 , then ⃗⃗⃗𝐸 points perpendicular inward (− 𝑛̂) Φ𝐸 = ∮ 𝐸⃗ . ⃗⃗⃗⃗⃗
𝑑𝐴 =
𝜀𝑜
8. Obtain an expression for electric field due to an uniformly charged spherical 𝐸 (4 𝜋 𝑟 2 ) = 0
shell. 𝑬 = 𝟎
Electric field due to charged spherical shell :  Thus the electric field due to the uniform charged spherical shell is zero at
 Consider an uniformly charged spherical shell of radius ‘R’ and charge ‘Q’ all points inside the shell.
1) At a point outside the shell (𝒓 > 𝑹) : 9. Obtain Gauss law from Coulomb’s law.
 Let P be the point outside the shell at a Gauss law from Coulomb’s law :
distance ‘r’ from its centre.  Consider a charged particle of charge ‘+q’
 Here electric field points radially outwards if  Draw a Gaussian spherical surface of radius
Q >0 and radially inward if Q < 0. ‘r’ around this charge.
 Consider a spherical Gaussian surface of  Due to symmentry, the electric field ⃗⃗⃗𝐸 at all
radius ‘r’ which encloses the total charge ‘Q’ the points on the spherical surface have
 Since 𝐸⃗ and ⃗⃗⃗⃗⃗𝑑𝐴 are along radially outwards, same magnitude and radially outward in
we have 𝜃 = 0 direction.

victory R. SARAVANAN. M.Sc., M.Phil., B.Ed PG ASST [PHYSICS], GBHSS, PARANGIPETTAI - 608 502
12 PHYSICS UNIT - 1 ELECTROSTATICS COMPLETE STUDY MATERIAL
 If a test charge ‘𝑞𝑜 ’ is placed on the Gaussian surface, by Coulomb law the force Property - 2 : There is no net charge inside the conductors. The charges must
acting it is, reside only on the surface of the conductors.
1 𝑄 𝑞𝑜  Form Gauss’s law, this implies that there is no net charge inside the conductor.
|⃗⃗⃗𝐹 | =
4 𝜋 𝜀𝑜 𝑟 2 Even if some charge is introduced inside the conductor, it immediately reaches
 By definition, the electric field, the surface of the conductor.
|⃗⃗⃗𝐹 | 1 𝑄 Property - 3 : The electric field outside the conductor is perpendicular to the
|⃗⃗⃗𝐸 | = = − − − −(1) surface of the conductor and has a magnitude of
𝝈
, where 𝝈 is the surface
𝑞𝑜 4 𝜋 𝜀𝑜 𝑟 2 𝜺𝒐
 Since the area element 𝑑𝐴 ⃗⃗⃗⃗⃗ is along the electric field 𝐸⃗ , we have 𝜃 = 0. Hence charge density at that point
the electric flux through the Gaussian surface is,  If the electric field has components parallel to the surface of the conductor,
Φ𝐸 = ∮ 𝐸⃗ . ⃗⃗⃗⃗⃗
𝑑𝐴 = ∮ 𝐸 𝑑𝐴 cos 0° = 𝐸 ∮ 𝑑𝐴 then free electrons on the surface of the conductor would experience
acceleration. This means that the conductor is not in equilibrium.
Here ∮ 𝑑𝐴 = 4 𝜋 𝑟 2 → area of Gaussian sphere  Therefore at electrostactic equilibrium, the electric field must be
 Put equation (1). we get perpendicular to the surface of the conductor.
1 𝑄  For cylindrical Gaussian surface, the total electric flux is 𝚽𝑬 = 𝑬 𝑨 and the
Φ𝐸 = 𝑋 4 𝜋 𝑟2
4 𝜋 𝜀𝑜 𝑟 2 total charge inside the surface is 𝑸 = 𝝈 𝑨
𝑸  By Gauss law,
∴ 𝚽𝑬 =
𝜺𝒐 𝑄 σA
Φ𝐸 = (𝑜𝑟) E A =
 This is known as Gauss law. 𝜀𝑜 𝜀𝑜
Result : 𝛔
∴ 𝐄 =
 The total electric flux through the closed surface depends only on the charges 𝜺𝒐
enclosed by the surface and independent of charges outside the surface.  In vector notation,
 The total electric flux is independent of the location of charges inside the 𝛔
⃗⃗⃗𝐄 = ̂
𝐧
closed surface and shape on the closed surface. 𝜺𝒐
 Gauss law is another form of Coulomb law and also applicable to charges in Property - 4 : The electrostatic potential has the same value on the surface
motion. and inside of the conductor.
10. Discuss the various properties of conductors in electrostatic equilibrium.  The conductor has no parallel electric component on the surface which means
Conductors in electrostatic equilibrium : that charges can be moved on the surface without doing any work.
 An electrical conductor has a large number of mobile charges which are free to  This is possible only if the electrostatic potential is constant at all points on the
move in the material. surface and there is no potential difference between any two points on the
 The resultant motion is zero and it implies that the conductor is in electrostatic surface.
equilibrium.  Since the electric field is zero inside the conductor, the potential is the same as
 Thus at electrostatic equilibrium, there is no net current in the conductor. the surface of the conductor.
 A conductor at electrostatic equilibrium has the following properties.  Thus at electro static equilibrium, the conductor is always at equipotential.
Property - 1 : The electric field is zero everywhere inside the conductor. This 11. Explain dielectrics in detail and how an electric field is induced inside a
is true regardless of whether the conductor is solid or hollow. dielectric.
 The electric field is not zero inside the metal, then there will be a force on the Electric field induced inside a dielectric :
mobile charge carriers due to this electric field.  When an external
 As a result, there will be a net motion of the mobile charges, which contradicts electric field is applied
the conductors being in electrostatic equilibrium. on a conductor, the
 Thus the electric field is zero every where inside the conductor. charges are aligned in
such a way that an
internal electric field is
created which cancels
the external electric field.

victory R. SARAVANAN. M.Sc., M.Phil., B.Ed PG ASST [PHYSICS], GBHSS, PARANGIPETTAI - 608 502
12 PHYSICS UNIT - 1 ELECTROSTATICS COMPLETE STUDY MATERIAL
 But in dielectric, which has no free electrons, the external electric field only  Then the capacitance of a capacitor with dielectric,
realigns the charges so that an internal electric field is produced. 𝑄𝑜 𝑄𝑜 𝑄𝑜
𝐶= = = 𝜀𝑟 = 𝜀𝑟 𝐶𝑜
 The magnitude of the internal electric field is smaller than that of external 𝑉 𝑉 𝑉𝑜
[ 𝑜]
electric field. 𝜀𝑟
 Therefore the net electric field inside the dielectric field is not zero, but is  Since 𝜀𝑟 > 1, we have 𝐶 > 𝐶𝑜 .
parallel to an external electric field with magnitude less than that of the  Thus insertion of dielectric slab increases the capacitance.
𝜺 𝑨
external electric field.  We have, 𝑪𝒐 = 𝟎
𝒅
 For example, let a rectangular dielectric slab is placed between two oppositely 𝜺𝒓 𝜺𝟎 𝑨 𝜺 𝑨
charged plates. ∴ 𝑪 = =
𝒅 𝒅
 The uniform electric field between the plates acts as the external electric field Where, 𝜺𝒓 𝜺𝟎 = 𝜺 → permitivity of the dielectric medium
𝐸⃗ 𝑒𝑥𝑡 which polarizes the dielectric slab.  The energy stored in the capacitor without dielectric,
 Thus positive charges are induced on one side and negative charges are 1 𝑄𝑜2
induced on the other side of the slab. 𝑈𝑜 =
2 𝐶𝑜
 So the dielectric in the external field is equivalent to two oppositely charged  After the dielectric is inserted,
sheets with the surface charge densities . These charges are called bound 𝟏 𝑸𝒐𝟐 𝟏 𝑸𝒐𝟐 𝑼𝒐
charges. They are not free to move like free electrons in conductor. 𝑼= = =
𝟐 𝑪 𝟐 𝜺𝒓 𝑪𝒐 𝜺𝒓
12. Explain in detail the effect of dielectric placed in a parallel plate capacitor
 Since 𝜀𝑟 > 1, we have 𝑈 < 𝑈𝑜
when the capacitor is disconnected from the battery.
Effect of dielectrics when the capacitor is disconnected from the battery :  There is a decrease in energy because, when the dielectric is inserted, the
capacitor spend some energy to pulling the dielectric slab inside.
13. Explain in detail the effect of dielectric placed in a parallel plate capacitor
when the battery remains connected to the capacitor.
Effect of dielectrics when the battery remains connected to the capacitor:

 Consider a parallel plate capacitor.


 Area of each plates =A
Distance between the plates =𝑑
Voltage of battery = 𝑉𝑜
Total charge on the capacitor = 𝑄𝑜  Consider a parallel plate capacitor.
 So the capacitance of capacitor without dielectric,  Area of each plates =A
𝑄𝑜 Distance between the plates =𝑑
𝐶𝑜 = Voltage of battery = 𝑉𝑜
𝑉𝑜
Total charge on the capacitor = 𝑄𝑜
 The battery is then disconnected from the capacitor and the dielectric is
 So the capacitance of capacitor without dielectric,
inserted between the plates. This decreases the electric field. 𝑄𝑜
 Electric field without dielectric = 𝐸𝑜 𝐶𝑜 =
𝑉𝑜
Electric field with dielectric = E
Relative permittivity or dielectric constant = 𝜀𝑟  Dielectric is inserted between the plates and the battery is remains in
𝐸𝑜 connected with the capacitor.
∴ 𝐸=  So the charges stored in the capacitor is increased.
𝜀𝑟
 Total charge without dielectric = 𝑄𝑜
 Since 𝜀𝑟 > 1, we have 𝐸 < 𝐸𝑜
Total charge with dielectric = 𝑄
 Hence electrostatic potential between the plates is reduced and at the same
Relative permittivity (dielectric constat) = 𝜀𝑟
time the charge 𝑄𝑜 remains constant.
𝐸𝑜 𝑉𝑜 ∴ 𝑸 = 𝜺𝒓 𝑸𝒐
𝑉=𝐸𝑑= 𝑑=  Since 𝜀𝑟 > 1, we have 𝑄 < 𝑄𝑜
𝜀𝑟 𝜀𝑟
victory R. SARAVANAN. M.Sc., M.Phil., B.Ed PG ASST [PHYSICS], GBHSS, PARANGIPETTAI - 608 502
12 PHYSICS UNIT - 1 ELECTROSTATICS COMPLETE STUDY MATERIAL
 Here the potential difference between the plates remains constant. But the Capacitors in parallel :
charges increases and the new capacitance will be
𝑄 𝜀𝑟 𝑄𝑜
𝐶= = = 𝜀𝑟 𝐶𝑜
𝑉𝑜 𝑉𝑜
 Since 𝜀𝑟 > 1, we have 𝐶 > 𝐶𝑜
 Hence capacitance increases with the insertion of dielectric slab.
𝜺 𝑨
 We know that, 𝑪𝒐 = 𝟎
𝒅
𝜺𝒓 𝜺𝟎 𝑨 𝜺 𝑨
∴ 𝑪 = =
𝒅 𝒅
Where, 𝜺𝒓 𝜺𝟎 = 𝜺 → permitivity of the dielectric medium  Consider three capacitors of capacitance 𝐶1 , 𝐶2 and 𝐶3 connected in
 The energy stored in the capacitor without dielectric, parallel with a battery of voltage V .In parallel connection,
1 1) Each capacitor has same potential difference (V)
𝑈𝑜 = 𝐶 𝑉2 2) But charges on each capacitor will be different
2 𝑜 𝑜
 After the dielectric is inserted,  Let 𝑄1 , 𝑄2 , 𝑄3 be the charge on 𝐶1 , 𝐶2 , 𝐶3 respectively.
𝟏 𝟏  Let 𝐶𝑃 be the equivalent capacitance of capacitor in parallel connection, then
𝑼= 𝑪 𝑽𝒐𝟐 = 𝜺𝒓 𝑪𝒐 𝑽𝒐𝟐 = 𝜺𝒓 𝑼𝒐 𝑄 = 𝑄1 + 𝑄2 + 𝑄3
𝟐 𝟐
 Since 𝜀𝑟 > 1, we have 𝑈 > 𝑈𝑜 𝐶𝑃 𝑉 = 𝐶1 𝑉 + 𝐶2 𝑉 + 𝐶3 𝑉 [∵ 𝑄 = 𝐶 𝑉]
 So there is increase in energy when the dielectric is inserted 𝐶𝑃 𝑉 = 𝑉 [𝐶1 + 𝐶2 + 𝐶3 ]
14. Derive the expression for resultant capacitance, when capacitors are 𝑪𝑷 = 𝑪𝟏 + 𝑪𝟐 + 𝑪𝟑
connected in series and in parallel.  Thus the equivalent capacitance of capacitors connected in parallel is equal to
Capacitors in series : the sum of the individual capacitances.
 The equivalent capacitance 𝑪𝑷 in a parallel connection is always greater than
the largest individual capacitance.
15. Explain in detail how charges are distributed in a conductor and the principle
behind the lightning conductor.
Distribution of charges in a conductor :
 Consider two conducting spheres ‘A’
 Consider three capacitors of capacitance 𝐶1 , 𝐶2 and 𝐶3 connected in series with and ‘B’ of radii 𝒓𝟏 and 𝒓𝟐 . Let 𝒓𝟏 > 𝒓𝟐
a battery of voltage V . In series connection,  Let the two spheres are connected by
1) Each capacitor has same amount of charge (Q) a thin conducting wire.
2) But potential difference across each capacitor will be different.  If a charge ‘Q’ is given to either A or B,
 Let 𝑉1 , 𝑉2 , 𝑉3 be the potential difference across 𝐶1 , 𝐶2 , 𝐶3 respectively. this charge is redistributed in both the
 Let 𝐶𝑆 be the equivalent capacitance of capacitor in series connection, then spheres until their potential becomes
𝑉 = 𝑉1 + 𝑉2 + 𝑉3 same.
𝑄 𝑄 𝑄 𝑄  Now they are uniformly charged and attain electrostatic equilibrium.
= + + [∵ 𝑄 = 𝐶 𝑉]  At this stage, let the surface charge densities of A and B are 𝜎1 and 𝜎2
𝐶𝑆 𝐶1 𝐶2 𝐶3
𝑄 1 1 1 respectively, then
= 𝑄 [ + + ] Charge residing on suface of A = 𝑞1 = 𝜎1 4 𝜋 𝑟12
𝐶𝑆 𝐶1 𝐶2 𝐶3
𝟏 𝟏 𝟏 𝟏 Charge residing on suface of B = 𝑞2 = 𝜎2 4 𝜋 𝑟22
= + +  Then the total charge ; Q = 𝑞1 + 𝑞2
𝑪𝑺 𝑪𝟏 𝑪𝟐 𝑪𝟑
 There is no net charge inside the conductors.
 Thus the inverse of the equivalent capacitance of capacitors connected in  Electrostic potential on the surface of A and B is
series is equal to the sum of the inverses of each capacitance. 1 𝑞1 1 𝑞2
 This equivalent capacitance 𝑪𝑺 is always less than the smallest individual 𝑉𝐴 = & 𝑉𝐵 =
4 𝜋 𝜀0 𝑟1 4 𝜋 𝜀0 𝑟2
capacitance in the series
victory R. SARAVANAN. M.Sc., M.Phil., B.Ed PG ASST [PHYSICS], GBHSS, PARANGIPETTAI - 608 502
12 PHYSICS UNIT - 1 ELECTROSTATICS COMPLETE STUDY MATERIAL
 Under elecrostic equilibrium. 𝑉𝐴 = 𝑉𝐵 Working :
1 𝑞1 1 𝑞2  Due to the high electgric field near comb ‘D’, air between the belt and comb ‘D’
∴ =
4 𝜋 𝜀0 𝑟1 4 𝜋 𝜀0 𝑟2 gets ionized.
𝑞1 𝑞2  The positive charges are pushed towards the belt and negative charges are
=
𝑟1 𝑟2 attracted towards the comb ‘D’
𝜎1 4 𝜋 𝑟12 𝜎2 4 𝜋 𝑟22  The positive charges stick to the belt and move up.
=  When the positive charges reach the comb ‘E’ a large amount of negative and
𝑟1 𝑟2
positive charges are induced on either side of comb ‘E’ due to electrostatic
𝜎1 𝑟1 = 𝜎2 𝑟2 (𝑜𝑟) 𝝈 𝒓 = 𝒄𝒐𝒏𝒔𝒕𝒂𝒏𝒕
induction.
 Thus the surface charge density is inversely proportional to the radius of the  As a result. the positive charges are pushed away from the comb ‘E’ and they
sphere. reach the outer surface of the sphere.
 Hence for smaller radius , the charge density will be larger and vice versa  These positive charges are distributed uniformly on the outer surface of the
Principle of lightning conductor (Action of point) : hollow sphere.
 Action of point is the principle behind the lightning conductor.  At the same time, the negative charges neutralize the positive charges in the
 We know that smaller the radius of curvature, the larger is the charge density. belt due to corona discharge before it passes over the pulley.
 If the conductor has sharp end which has larger curvature (smaller radius), it  When the belt descends, it has almost no net charge.
has a large charge accumulation. As a result, the electric field near this edge is  This process continues until the outer surface produces the potential
very high and it ionizes the surrounding air. difference of the order of 107 𝑉 which is the limiting value.
 The positive ions are repelled at the sharp edge and negative ions are attracted  Beyond this, the charges starts leaking to the surroundings due to ionization of
towards the sharper edge. air.
 This reduces the total charge of the conductor near the sharp edge. This is  It is prevented by enclosing the machine in a gas filled steel chamber at very
called action of points or corona discharge. high pressure.
16. Explain in detail the construction and working of Van de Graff generator. Applications :
Van de Gralff generator :  The high voltage produced in this Van de Graff generator is used to accelerate
 It is designed by Robert Van de Graff. positive ions (protons and deuterons) for nuclear disintegrations and other
 It produce large electro static applications.
potential difference of about 107 𝑉
Principle :
 Electro static induction
 Action of points
Construction :
 It consists of large hollow spherical
conductor ‘A’ fixed on the insulating
stand.
 Pulley ‘B’ is mounted at the centre of
the sphere and another pulley ‘C’ is
fixed at the bottom.
 A belt made up of insulating material
like silk or rubber runs over the
pulleys.
 The pulley ‘C’ is driven continuously
by the electric motor.
 Two comb shaped metallic conductor D and E are fixed near the pulleys.
 The comb ‘D’ is maintained at a positive potential of 104 𝑉 by a power supply.
 The upper comb ‘E’ is connected to the inner side of the hollow metal sphere.
victory R. SARAVANAN. M.Sc., M.Phil., B.Ed PG ASST [PHYSICS], GBHSS, PARANGIPETTAI - 608 502
12 PHYSICS UNIT - 1 ELECTROSTATICS COMPLETE STUDY MATERIAL
3. Two small-sized identical equally charged spheres, each
EXAMPLE PROBLEMS WITH SOLUTIONS having mass 1 g are hanging in equilibrium as shown in the
1. Calculate the number of electrons in one coulomb of negative charge. figure. The length of each string is 10 cm and the
Solution :- q = 1 C ; e = 1.6 X 10-19 C ; n=? angle θ is 30° with the vertical. Calculate the magnitude of
 According to the quantisation of charge ; 𝑞=𝑛𝑒 the charge in each sphere.(Take g = 10 ms−2)
𝑞 1 1019 Solution:-: 𝑚 = 1 g = 10−3 𝑘g ; 𝜃 = 30 ;
(𝑜𝑟) 𝑛= = =
𝑒 1.6 𝑋 10−19 1.6 𝑔 = 10 𝑚𝑠 −2 ; 𝐿 = 10 𝑐𝑚 = 0.1 ; 𝑞 = ?
𝒏 = 𝟔. 𝟐𝟓 𝑿 𝟏𝟎𝟏𝟖 electrons  Since they are positively charged spheres, there will be a repulsive force
2. Consider two point charges q1 and q2 at rest as shown in between them and they will be at equilibrium with each other at an angle of 30°
the figure. They are separated by a distance of 1m. with the vertical.
Calculate the force experienced by the two charges for the  At equilibrium, each charge experiences zero
following cases: net force in each direction. We can draw a
(a) q1 = +2 μC and q2 = +3 μC free body diagram for one of the charged
(b) q1 = +2 μC and q2 = –3 μC spheres and apply Newton’s second law for
(c) q1= +2 μC and q2 = –3 μC kept in water (𝜺𝒓 = 𝟖𝟎) both vertical and horizontal directions.
Solution :-  In the x-direction, the acceleration of the
(a) 𝑞1 = +2 𝜇𝐶 ; 𝑞2 = +3 𝜇𝐶 ; r = 1 m then , ⃗⃗⃗𝐹12 = ? & ⃗⃗⃗𝐹21 = ? charged sphere is zero.
1 𝑞1 𝑞2 2 𝑋10−6 𝑋 3 𝑋 10−6 𝑇 sin 𝜃 ̂𝑖 + 𝐹𝑒 (− ̂)
𝑖 =0
⃗⃗⃗𝐹21 = 𝑟̂12 = 9 𝑋 10 9
𝑋 𝑖̂ (𝑜𝑟) 𝑇 sin 𝜃 ̂𝑖 = 𝐹𝑒 ̂𝑖
4 𝜋 𝜀𝑜 𝑟 2 12 (𝑜𝑟) 𝑇 sin 𝜃 = 𝐹𝑒 − − − (1)
⃗⃗⃗𝑭𝟐𝟏 = 𝟓𝟒 𝑿 𝟏𝟎−𝟑 𝑵 𝒊̂ ( along + X axis)
 In the y-direction also, the net acceleration experienced by the charge is
1 𝑞1 𝑞2 2 𝑋10−6 𝑋 3 𝑋 10−6 zero. 𝑇 cos 𝜃 ̂𝑗 + 𝑚 𝑔 (−̂) 𝑗 =0
& ⃗⃗⃗𝐹12 = 𝑟̂ = 9 𝑋 10 9
𝑋 (− 𝑖̂)
4 𝜋 𝜀𝑜 𝑟 2 21 12 (𝑜𝑟) 𝑇 cos 𝜃 ̂𝑗 = 𝑚 𝑔 ̂𝑗
⃗⃗⃗𝑭𝟏𝟐 = − 𝟓𝟒 𝑿 𝟏𝟎−𝟑 𝑵 𝒊̂ (along - X axis) (𝑜𝑟) 𝑇 cos 𝜃 = 𝑚 𝑔 − − − (2)
 Divide equation (1) by 92)
(b) 𝑞1 = +2 𝜇𝐶 ; 𝑞2 = −3 𝜇𝐶 ; r = 1 m then, ⃗⃗⃗𝐹12 = ? & ⃗⃗⃗𝐹21 = ?
𝑇 sin 𝜃 𝐹𝑒
1 𝑞1 𝑞2 2 𝑋10−6 𝑋 (−3) 𝑋 10−6 =
⃗⃗⃗𝐹21 = 𝑟̂12 = 9 𝑋 10 9
𝑋 𝑖̂ 𝑇 cos 𝜃 𝑚𝑔
4 𝜋 𝜀𝑜 𝑟 2 12 𝐹𝑒 1 1 𝑞2
⃗⃗⃗𝑭𝟐𝟏 = − 𝟓𝟒 𝑿 𝟏𝟎𝟗 𝑵 𝒊̂ (along - X axis) tan  = = [ ] [∵ 𝑎 = 𝐿 sin 𝜃]
𝑚𝑔 𝑚 𝑔 4 𝜋 𝜀0 (2𝑎)2
1 𝑞1 𝑞2 2 𝑋10−6 𝑋 (−3) 𝑋 10−6 1 1 𝑞2
& ⃗⃗⃗𝐹12 = 𝑟̂ = 9 𝑋 10 9
𝑋 (− 𝑖̂) tan  = [ ]
4 𝜋 𝜀𝑜 𝑟 2 21 12 𝑚𝑔 4 𝜋 𝜀0 (2 𝐿 sin 𝜃)2
2 2
⃗⃗⃗𝑭𝟏𝟐 = 𝟓𝟒 𝑿 𝟏𝟎𝟗 𝑵 𝒊̂ (along + X axis) ∴ 𝑞 = 𝑚 𝑔 [4 𝜋 𝜀0 ](2 𝐿 sin 𝜃) tan 𝜃
1
(c) 𝑞1 = +2 𝜇𝐶 and 𝑞2 = −3 𝜇𝐶 placed in water (𝜀𝑟 = 80), ⃗⃗⃗𝐹12 = ? & ⃗⃗⃗𝐹21 = ? (𝑜𝑟) 𝑞 = 2 𝐿 sin 𝜃 √𝑚 𝑔 [4 𝜋 𝜀0 ] tan 𝜃 [∵ = 9 𝑋 109 ]
4 𝜋 𝜀0
1 𝑞1 𝑞2 ⃗⃗⃗𝐹21 − 54 𝑋 10−3 10−3 𝑋 10 𝑋 tan 30
⃗⃗⃗𝐹21
𝑊
= 𝑟̂12 = = ̂𝒊 𝑞 = 2 𝑋 0.1 𝑋 sin 30 𝑋 √
4 𝜋 𝜀𝑜 𝜀𝑟 𝑟 2 𝜀𝑟 80 9 𝑋 109
⃗⃗⃗𝑭𝑾
𝟐𝟏 = − 𝟔. 𝟕𝟓 𝑿 𝟏𝟎
−𝟒
𝑵 ̂𝒊 (along - X axis) 1 10−3 𝑋 10 𝑋 0.5774 5.774 𝑋 10−12
𝑞 = 0.2 𝑋 𝑋 √ = 0.1 𝑋 √
1 𝑞1 𝑞2 ⃗⃗⃗𝐹12 54 𝑋 10−3 2 9 𝑋 109 9
⃗⃗⃗𝐹12
𝑊
= 𝑟̂ = = ̂𝒊 0. 1
4 𝜋 𝜀𝑜 𝜀𝑟 𝑟 2 21 𝜀𝑟 80 𝑞 = 𝑋 √5.774 𝑋 10−6
⃗⃗⃗𝑭𝑾 −𝟒 3
𝟐𝟏 = 𝟔. 𝟕𝟓 𝑿 𝟏𝟎 𝑵 ̂𝒊 (along + X axis)
𝑞 = 0.0333 𝑋 √5.774 𝑋 10−6
NOTE : All three cases obeys Newton’s third law of motion (𝑖. 𝑒) ⃗⃗⃗𝑭𝟏𝟐 = − ⃗⃗⃗𝑭𝟐𝟏 𝑞 = 8.001 𝑋 10−2 𝑋 10−6 = 8.001 𝑋 10−8 𝐶
𝑞 = 80. 01 𝑋 10−8 𝐶 = 80.01 𝑛 𝐶

victory R. SARAVANAN. M.Sc., M.Phil., B.Ed PG ASST [PHYSICS], GBHSS, PARANGIPETTAI - 608 502
12 PHYSICS UNIT - 1 ELECTROSTATICS COMPLETE STUDY MATERIAL
4. Calculate the electrostatic force and gravitational force between the proton  Since, 𝑟 = 1 𝑚 From figure, 𝑟12 = 𝑟14 = √2 𝑟2 = 𝑟√2 = √2 𝑚 ; 𝑟13 = 2 𝑟 = 2 𝑚
and the electron in a hydrogen atom. They are separated by a distance of  Magnitude of the forces,
5.3 × 10–11 m. The magnitude of charges on the electron and proton are
1 𝑞1 𝑞2 1 𝑋 10−6 𝑋 1 𝑋 10−6 9 𝑋 10−3
1.6 × 10–19 C. Mass of the electron is me = 9.1 × 10–31 kg and mass of proton is 𝐹12 = = 9 𝑋 10 9
𝑋 = = 4.5 𝑋 10−3 𝑁
mp = 1.6 × 10–27 kg. 4 𝜋 𝜀𝑜 𝑟122 (√2)
2
2
Solution :- 𝑟 = 5.3 𝑋 10−11 𝑚 ; 𝑒 = 1.6 𝑋 10−19 𝐶 ; 1 𝑞1 𝑞3 1 𝑋 10−6 𝑋 1 𝑋 10−6 9 𝑋 10−3
9
𝑚𝑒 = 9.1 𝑋 10 −31
𝑘𝑔 ; 𝑚𝑝 = 1,6 𝑋 10−27 𝑘𝑔 ; 𝐺 = 6.67 𝑋 10−11 𝑁 𝑚2 𝐶 −2 𝐹13 = 2 = 9 𝑋 10 𝑋 = = 2.25 𝑋 10−3 𝑁
4 𝜋 𝜀𝑜 𝑟13 (2) 2 4
 The magnitude of the electrostatic force between electron and proton 1 𝑞1 𝑞4 1 𝑋 10−6 𝑋 1 𝑋 10−6 9 𝑋 10−3
9
1 𝑒2 (1.6 𝑋 10−19 )2 𝐹14 = = 9 𝑋 10 𝑋 = = 4.5 𝑋 10−3 𝑁
𝐹𝐸 = = 9 𝑋 109 𝑋 4 𝜋 𝜀𝑜 𝑟142 (√2)
2
2
4 𝜋 𝜀𝑜 𝑟 2 (5.3 𝑋 10−11 )2
9 𝑋 10 𝑋 1.6 𝑋 1.6 𝑋 10−38
9  From the figure, 𝜃 = 45°, then the total force on 𝑞1
𝐹𝐸 = 𝐹1𝑡𝑜𝑡 = ⃗⃗⃗𝐹12 + ⃗⃗⃗𝐹13 + ⃗⃗⃗𝐹14
5.3 𝑋 5.3 𝑋 10−22
9 𝑋 2.56 𝑋 10−7 23.04 𝑋 10−7 𝐹1𝑡𝑜𝑡 = [𝐹12 cos 𝜃 ̂𝑖 + 𝐹12 sin 𝜃 (− 𝑗̂)] + 𝐹13 ̂𝑖 + [𝐹14 cos 𝜃 ̂𝑖 + 𝐹14 sin 𝜃 𝑗̂]
𝐹𝐸 = = = 8.202 𝑋 10−1 𝑋 10−7
28.09 29.09 𝐹1𝑡𝑜𝑡 = 𝐹12 cos 𝜃 ̂𝑖 − 𝐹12 sin 𝜃 𝑗̂ + 𝐹13 ̂𝑖 + 𝐹14 cos 𝜃 ̂𝑖 + 𝐹14 sin 𝜃 𝑗̂
𝑭𝑬 = 𝟖. 𝟐𝟎𝟐 𝑿 𝟏𝟎−𝟖 𝑵  Since 𝐹12 = 𝐹14 , we have
 The magnitude of the gravitational force between electron and proton 𝑡𝑜𝑡
⃗𝐹1 = 𝐹12 cos 𝜃 𝑖̂ + 𝐹13 𝑖̂ + 𝐹14 cos 𝜃 𝑖̂
𝐺 𝑚𝑃 𝑚𝑒 6.67 𝑋 10−11 𝑋 1,6 𝑋 10−27 𝑋 9.1 𝑋 10−31
𝐹𝐺 = = 𝐹1𝑡𝑜𝑡 = 2 𝐹12 cos 𝜃 𝑖̂ + 𝐹13 𝑖̂
𝑟2 (5.3 𝑋 10−11 )2 𝑡𝑜𝑡
6.67 𝑋 1.6 𝑋 9.1 𝑋 10−69 6.67 𝑋 1.6 𝑋 9.1 𝑋 10−47 ⃗ 1 = 2 (4.5 𝑋 10−3 ) cos 45° 𝑖̂ + 2.25 𝑋 10−3 𝑖̂
𝐹
𝐹𝐺 = = 1
28.09 𝑋 10−22 28.09 𝐹1𝑡𝑜𝑡 = 2 (4.5 𝑋 10−3 ) 𝑖̂ + 2.25 𝑋 10−3 𝑖̂
𝑭𝑮 = 𝟑. 𝟒𝟓𝟔 𝑿 𝟏𝟎−𝟒𝟕 𝑵 √2
 The ratio of the two forces, ⃗⃗⃗𝐹1𝑡𝑜𝑡 = [4.5 √2 + 2.25] 𝑋 10−3 𝑖̂ = [6.363 + 2.25] 𝑋 10−3 𝑖̂
𝐹𝐸 8.202 𝑋 10−8 ⃗𝑭𝒕𝒐𝒕
𝟏 = 𝟖. 𝟔𝟏𝟑 𝑿 𝟏𝟎
−𝟑
𝒊̂
= ≈ 2.4 𝑋 1039
𝐹𝐺 3. 456 𝑋 10−47 6. Calculate the electric field at points P, Q for the following two cases, as shown
(𝑜𝑟) 𝑭𝑬 ≈ 𝟏𝟎𝟑𝟗 𝑭𝑮 in the figure.
∴ 𝑭𝑬 ≫ 𝑭𝑮 (a) A positive point charge +1 μC is placed at the origin
5. Consider four equal charges q1, q2, q3 and q4 = q = +1 μC (b) A negative point charge –2 μC is placed at the origin
located at four different points on a circle of radius 1m, Solution :-
as shown in the figure. Calculate the total force acting on
(a) = +1 𝜇𝐶 ; 𝑟𝑃 = 2 𝑚 ; 𝑟𝑄 = 4 𝑚 ⃗𝐸𝑃 = ? ⃗𝐸𝑄 = ?
the charge q1 due to all the other charges.
Solution :- Electric field at P,
𝑡𝑜𝑡 1 𝑞 1 𝑋 10−6
𝑞1 = 𝑞2 = 𝑞3 = 𝑞4 = 𝑞 = +1 𝜇𝐶 ; 𝑟 = 1 𝑚 ; 𝐹 ⃗1 =? ⃗𝐸𝑃 = 𝑖 ̂ = 9 𝑋 10 9
𝑋 𝑖̂
 Figure shows forces and its components act on charge 𝑞1
4 𝜋 𝜀𝑜 𝑟𝑃2 (2)2
9 𝑋 103
𝐸⃗𝑃 = 𝑖̂
4
⃗𝑬𝑷 = 𝟐. 𝟐𝟓 𝑿 𝟏𝟎𝟑 𝒊̂ 𝑵 𝑪−𝟏
Electric field at Q ,
1 𝑞 1 𝑋 10−6
𝐸⃗𝑄 = 𝑗̂ = 9 𝑋 10 9
𝑋 𝑗̂
4 𝜋 𝜀𝑜 𝑟𝑄2 (4)2
9 𝑋 103
𝐸⃗𝑄 = 𝑗̂
16
⃗𝑬𝑸 = 𝟎. 𝟓𝟔 𝑿 𝟏𝟎𝟑 𝒋̂ 𝑵 𝑪−𝟏

victory R. SARAVANAN. M.Sc., M.Phil., B.Ed PG ASST [PHYSICS], GBHSS, PARANGIPETTAI - 608 502
12 PHYSICS UNIT - 1 ELECTROSTATICS COMPLETE STUDY MATERIAL
(b) 𝑞 = − 2 𝜇𝐶 ; 𝑟𝑃 = 2 𝑚 ; 𝑟𝑄 = 6𝑚 𝐸 ⃗ 𝑃 =? 𝐸⃗ 𝑄 =?  Its magnitude, |⃗⃗⃗𝐸𝐴 | = 2.25 𝑋 10 9
√12 + 12 = 2.25 𝑋 109 √2
Electric field at P  Then the direction of 𝐸 ⃗𝐴,
1 𝑞 2 𝑋 10−6 ⃗𝐸𝐴 2.25 𝑋 109 (𝑖̂ + 𝑗̂) (𝑖̂ + 𝑗̂)
⃗𝐸𝑃 = (−𝑖 ̂ ) = 9 𝑋 10 9
𝑋 (−𝑖̂) 𝐸̂𝐴 = = =
4 𝜋 𝜀𝑜 𝑟𝑃2 (2)2 ⃗⃗⃗
| 𝐸𝐴 | 9
2.25 𝑋 10 √2 √2
18 𝑋 103
𝐸⃗𝑃 = − 𝑖̂  Acceleration experienced by the electron placed at A,
4
⃗ 𝑷 = − 𝟒. 𝟓 𝑿 𝟏𝟎 𝒊̂ 𝑵 𝑪−𝟏𝟑 𝐹 𝑞 𝐸⃗𝐴
𝑬 𝑎𝐴 = =
Electric field at Q 𝑚 𝑚
1 𝑞 2 𝑋 10−6 − 1.6 𝑋 10−19 𝑋 2.25 𝑋 109 ( 𝑖̂ + 𝑗̂ )
𝐸⃗𝑄 = 𝑖̂ = 9 𝑋 10 9
𝑋 𝑖̂ 𝑎𝐴 =
4 𝜋 𝜀𝑜 𝑟𝑄2 (6)2 9.1 𝑋 10−31
− 1.6 𝑋 2.25 𝑋 1021 ( 𝑖̂ + 𝑗̂ )
18 𝑋 103 𝑎𝐴 =
𝐸⃗𝑄 = 𝑖̂ 9.1
36 𝑎𝐴 = − 3.956 𝑋 10−1 𝑋 1021 ( 𝑖̂ + 𝑗̂ )
⃗ 𝑸 = 𝟎. 𝟓 𝑿 𝟏𝟎 𝒊̂ 𝑵 𝑪−𝟏
𝑬 𝟑
⃗ 𝑨 = − 𝟑. 𝟗𝟓𝟔 𝑿𝟏𝟎𝟐𝟎 ( 𝒊̂ + 𝒋̂ )𝑵 𝒌𝒈−𝟏
𝒂
7. Consider the charge configuration as shown in the  ⃗𝐴
The electron is accelerated in a direction exactly opposite to 𝐸
figure. Calculate the electric field at point A. If an 8. A block of mass m carrying a positive charge q is placed on
electron is placed at points A, what is the acceleration an insulated frictionless inclined plane as shown in the
experienced by this electron? figure. A uniform electric field E is applied parallel to the
(mass of the electron = 9.1 × 10–31 kg and charge of inclined surface such that the block is at rest. Calculate the
electron = −1.6 × 10–19 C) magnitude of the electric field E.
Solution :- 𝑞1 = 𝑞2 = + 1 𝜇𝐶 ; 𝑟1 = 𝑟2 = 2 𝑚𝑚 ; ⃗𝐸𝐴 = ? Solution :-
 The free body diagram for the mass m is drawn below.
 Electric field due to 𝑞1 ,
1 𝑞1
𝐸⃗1 = 𝑗̂
4 𝜋 𝜀𝑜 𝑟12
1 𝑋 10−6
= 9 𝑋 109 𝑋 𝑗̂
(2 𝑋 10−3 )2
−6
1 𝑋 10
= 9 𝑋 109 𝑋 𝑗̂
4 𝑋 10−6
⃗𝑬𝟏 = 𝟐. 𝟐𝟓 𝑿 𝟏𝟎𝟗 𝒋̂
 Electric field due to 𝑞2 ,
1 𝑞2
𝐸⃗2 = 𝑖̂
4 𝜋 𝜀𝑜 𝑟22
1 𝑋 10−6  There are three forces that acts on the mass m:
= 9 𝑋 109 𝑋 𝑖̂ (i) The downward gravitational force exerted by the Earth (mg)
(2 𝑋 10−3 )2
1 𝑋 10−6 (ii) The normal force exerted by the inclined surface (N)
= 9 𝑋 109 𝑋 𝑖̂ (iii) The Coulomb force given by uniform electric field (qE)
4 𝑋 10−6
⃗ 𝟐 = 𝟐. 𝟐𝟓 𝑿 𝟏𝟎 𝒊̂
𝑬 𝟗  Along x-direction, applying Newton’s second law, we have
 Hence the resultant electric field at A, 𝑚𝑔 sin 𝜃 𝑖̂ + 𝑞 𝐸 (− 𝑖̂) = 0
𝑚𝑔 sin 𝜃 𝑖̂ − 𝑞 𝐸 𝑖̂ = 0
⃗⃗⃗𝐸𝐴 = 𝐸⃗1 + 𝐸⃗2
𝑚𝑔 sin 𝜃 − 𝑞 𝐸 = 0
= 2.25 𝑋 109 𝑗̂ + 2.25 𝑋 109 𝑖̂ 𝑚𝑔 sin 𝜃 = 𝑞 𝐸
⃗ 𝑨 = 𝟐. 𝟐𝟓 𝑿 𝟏𝟎𝟗 ( 𝒊̂ + 𝒋̂ )
𝑬 𝒎𝒈 𝐬𝐢𝐧 𝜽 𝒎𝒈 𝒉
𝑬= =
𝒒 𝒒𝑳

victory R. SARAVANAN. M.Sc., M.Phil., B.Ed PG ASST [PHYSICS], GBHSS, PARANGIPETTAI - 608 502
12 PHYSICS UNIT - 1 ELECTROSTATICS COMPLETE STUDY MATERIAL
9. The following pictures depict electric field lines for various charge (iii) In the figure (c), the electric field lines start at q1 and q3 and end at q2. This
configurations. implies that q1 and q3 are positive charges. The ratio of the number of field
lines
𝑞1 𝑁1 8 1
| |= = =
𝑞2 𝑁2 16 2
|𝑞2 | 20
|𝑞1 | = = = 10
2 2
𝑞3 𝑁3 8 1
& | |= = =
𝑞2 𝑁2 16 2
|𝑞2 | 20
|𝑞3 | = = = 10
𝒒 2 2
(i) In figure (a) identify the signs of two charges and find the ratio | 𝟏 | ∴ 𝒒𝟏 = 𝒒𝟑 = + 𝟏𝟎 𝒏𝑪
𝒒𝟐
(ii) In figure (b), calculate the ratio of two positive charges and identify the 10. Calculate the electric dipole moment for the following charge configurations.
strength the electric field at three points A, B, and C
(iii) Figure (c) represents the electric field lines for three charges. If
q2 = –20 nC, then calculate the values of q1 and q3
Solution :-
(i) The electric field lines start at q2 and end at q1. In figure (a), q2 is positive and q1
is negative.
The number of the lines ending at q1 ; 𝑁1 = 6
The number of the lines starting from q2 ; 𝑁2 = 18
𝑞1
𝑁1 = (𝑜𝑟) 𝑞1 = 𝑁1 𝜀𝑜 Solution :-
𝜀𝑜 Figure (a) :
𝑞2
𝑁2 = (𝑜𝑟) 𝑞2 = 𝑁2 𝜀𝑜 Position vector for the +q on the positive x-axis = 𝑎 𝑖̂
𝜀𝑜 Position vector for the +q charge the negative x axis = 𝑎 (− 𝑖̂)
𝑞1 𝑁1 𝜀𝑜 So the dipole moment is ; ⃗𝑝 = (+𝑞) 𝑎 𝑖̂ + (+𝑞) 𝑎 (− 𝑖̂) = 0
∴ =
𝑞2 𝑁2 𝜀𝑜 Figure (b) :
𝑞1 𝑁1 6 1 Position vector charge +𝑞 placed at the origin = 0
| |= = =
𝑞2 𝑁2 18 3 Position vector for the +q on the positive x-axis = 𝑎 𝑖̂
|𝒒𝟐 | = 𝟑 |𝒒𝟏 | So the dipole moment is ; ⃗𝑝 = 0 + (+𝑞) 𝑎 𝑖̂ = 𝑞 𝑎 𝑖̂
(ii) In figure (b), Figure (c) :
The number of field lines emanating from 𝒒𝟏 ; 𝑁1 = 18 Position vector for the −2𝑞 on the positive y-axis = 𝑎 𝑗̂
The number of field lines emanating from 𝒒𝟐 ; 𝑁2 = 18 Position vector for the +q charge on the negative y axis = 2 𝑎 (− 𝑗̂)
𝑞1 𝑁1 18 So the dipole moment is ; ⃗𝑝 = (−2𝑞) 𝑎 𝑗̂ + (+𝑞) 2 𝑎 (− 𝑗̂) = −4𝑞𝑎 𝑗̂
∴ = = = 1 Figure (d) :
𝑞2 𝑁2 18
𝒒𝟏 = 𝒒𝟐 Position vector for the −2𝑞 on the negative x axis = 𝑎 (− 𝑖̂)
At point A, the electric field lines are denser compared to the lines at point B. Position vector for the +q charge on the positive y axis = 𝑎 𝑗̂
So the electric field at t A is greater in magnitude compared to the field at t B. Position vector for the +q charge on the negative y axis = 𝑎 (− 𝑗̂)
Further, no electric field line passes through C, which implies that the resultant So the dipole moment is ; ⃗𝑝 = (−2𝑞) 𝑎 (− 𝑖̂) + (+𝑞) 𝑎 𝑗̂ + (+𝑞) 𝑎 (− 𝑗̂) = 2 𝑞 𝑎 𝑖̂
electric field at C due to these two charges is zero. (e.g) The water molecule (H2O) has this charge configuration.
The water molecule has three atoms (two H atom and one O atom). The
centres of positive (H) and negative (O) charges of a water molecule lie at
different points, hence it possess permanent dipole moment.
The electric dipole moment ; ⃗𝒑 = 𝟔. 𝟏 𝑿 𝟏𝟎−𝟑𝟎 𝑪 𝒎
victory R. SARAVANAN. M.Sc., M.Phil., B.Ed PG ASST [PHYSICS], GBHSS, PARANGIPETTAI - 608 502
12 PHYSICS UNIT - 1 ELECTROSTATICS COMPLETE STUDY MATERIAL
11. A sample of HCl gas is placed in a uniform electric field of magnitude 13. Consider a point charge +q placed at the origin and another point charge -2q
3 × 104 N C–1. The dipole moment of each HCl molecule is 3.4 × 10–30 Cm. placed at a distance of 9 m from the charge +q. Determine the point between
Calculate the maximum torque experienced by each HCl molecule. the two charges at which electric potential is zero.
Solution :- 𝐸 = 3𝑋 104 𝑁 𝐶 −1 ; 𝑝 = 3.4 𝑋 1030 𝐶 𝑚 ; 𝜏𝑚𝑎𝑥 = ? Solution :-
 The maximum torque experienced by the dipole is when it is aligned  According to the superposition principle, the total electric potential at a point is
perpendicular ( = 90) to the applied field. So equal to the sum of the potentials due to each charge at that point.
𝜏𝑚𝑎𝑥 = 𝑝 𝐸 sin 90 = 3.4 𝑋 1030 𝑋 3𝑋 104 𝑋 1  Consider the point at which
𝝉𝒎𝒂𝒙 = 𝟏𝟎. 𝟐 𝑿 𝟏𝟎−𝟐𝟔 𝑵 𝒎 the total potential zero is
12. (a) Calculate the electric potential at points P and Q as shown in the figure located at a distance 𝑥 from
below. the charge +q as shown in
(b) Suppose the charge +9 μC is replaced by –9 μC find the electrostatic the figure.
potentials at points P and Q 𝑉+𝑞 + 𝑉−2𝑞 = 0
(c) Calculate the work done to bring a test charge +2 μC from infinity to the 1 𝑞 1 (−2𝑞)
+ =0
point Q. Assume the charge +9 μC is held fixed at origin and +2 μC is brought 4 𝜋 𝜀𝑜 𝑥 4 𝜋 𝜀𝑜 9 − 𝑥
from infinity to P. 1 𝑞 1 2𝑞
=
4 𝜋 𝜀𝑜 𝑥 4 𝜋 𝜀𝑜 9 − 𝑥
1 2
=
𝑥 9−𝑥
9−𝑥 =2𝑥
9= 2𝑥+𝑥 =3𝑥
Solution :- ∴ 𝑥 =3𝑚
(a) Electric potential at point P is given by 14. The following figure represents the electric potential as a function of
1 𝑞 9 𝑋 10−6 81 𝑋 103 x – coordinate. Plot the corresponding electric field as a function of x.
𝑉𝑃 = = 9 𝑋 109 𝑋 = = 8.1 𝑋 103 = 8100 𝑉 Solution :-
4 𝜋 𝜀𝑜 𝑟𝑃 10 10
Electric potential at point Q is given by  The electric field is the negative gradient of the electric potential. In vector
1 𝑞 9 𝑋 10−6 81 𝑋 103 form
𝑉𝑄 = = 9 𝑋 109 𝑋 = = 5.06 𝑋 103 = 5060 𝑉 𝜕𝑉 𝜕𝑉 𝜕𝑉
4 𝜋 𝜀𝑜 𝑟𝑄 16 16 ⃗𝐸 = − [ 𝑖̂ + 𝑗̂ + 𝑘̂ ]
Here , 𝑉𝑃 < 𝑉𝑄 .The potential difference between the points P and Q is given by 𝜕𝑥 𝜕𝑦 𝜕𝑧
 In the given problem, since the potential depends
∆𝑉 = 𝑉𝑃 − 𝑉𝑄 = 8100 − 5060 = 3040 𝑉 = 3.04 𝑋 103 𝑉 𝜕𝑉 𝜕𝑉
(b) Suppose we replace the charge +9 μC by –9 μC, then the corresponding only on x, we have = = 0. Thus,
𝜕𝑦 𝜕𝑦
potentials at the points P and Q are, 𝜕𝑉
𝑉𝑃 = − 8.1 𝑋 103 = − 8100 𝑉 ⃗ = −
𝐸 𝑖̂
𝜕𝑥
𝑉𝑄 = − 5.06 𝑋 103 = − 5060 𝑉 (i) From 0 to 1 cm, the slope is constant and
Here , 𝑉𝑃 < 𝑉𝑄 . The potential difference between the points P and Q is given by 𝜕𝑉
= 25 So, ⃗𝑬 = −𝟐𝟓 𝒊̂ 𝑽 𝒄𝒎−𝟏
𝜕𝑥
∆𝑉 = 𝑉𝑃 − 𝑉𝑄 = − 8100 − (−5060) = − 3040 𝑉 = −3.04 𝑋 103 𝑉 (ii) From 1 to 4 cm, the potential is constant,
(c) The electric potential V at a point Q due to some charge is defined as the workdone 𝜕𝑉
by an external force to bring a unit positive charge from infinity to Q. So to bring the q V = 25 V and = 0 . So ⃗𝑬 = 𝟎
𝜕𝑥
amount of charge from infinity to the point Q, work done is given as follows. (iii) From 4 to 5 cm, the slope constant and
𝜕𝑉
𝑊𝑄 = 𝑞 𝑉𝑄 = 2 𝑋 10−6 𝑋 5.06 𝑋 103 = 10.12 𝑋 10−3 𝐽 = − 25 . So ⃗𝑬 = + 𝟐𝟓 𝒊̂ 𝑽 𝒄𝒎−𝟏
𝜕𝑥
 The plot of electric field for the various points along
the x axis is given below.

victory R. SARAVANAN. M.Sc., M.Phil., B.Ed PG ASST [PHYSICS], GBHSS, PARANGIPETTAI - 608 502
12 PHYSICS UNIT - 1 ELECTROSTATICS COMPLETE STUDY MATERIAL
15. Four charges are arranged at the corners of the square 16. A water molecule has an electric dipole moment of 6.3 × 10–30 Cm. A sample
PQRS of side a as shown in the figure.(a) Find the work contains 1022 water molecules, with all the dipole moments aligned parallel
required to assemble these charges in the given to the external electric field of magnitude 3 × 105 N C–1. How much work is
configuration. (b) Suppose a charge q′ is brought to the required to rotate all the water molecules from θ = 0o to 90o?
centre of the square, by keeping the four charges fixed at Solution :- 𝜃𝑖 = 0° ; 𝜃𝑓 = 90° ; 𝐸 = 3 𝑋 105 𝑁 𝐶−1 ; 𝑝 = 6.3 𝑋 10−30 𝐶 𝑚 ; 𝑛 = 1022
the corners, how much extra work is required for this?  When the water molecules are aligned in the direction of the electric field,
Solution :- it has minimum potential energy.
(a) The work done to arrange the charges in the corners of the square is  The work done to rotate the dipole from θ = 0o to 90o is equal to the
independent of the way they are arranged. We can follow any order. potential energy difference between these two configurations.
(i) First, the charge +q is brought to the corner P. This requires no work since no 𝑊 = 𝑈 (𝜃𝑓 ) − 𝑈 ( 𝜃𝑖 )
charge is already present, WP = 0
𝑊 = − 𝑝 𝐸 cos 𝜃𝑓 + 𝑝 𝐸 cos 𝜃𝑖 = 𝑝 𝐸 (cos 𝜃𝑖 − cos 𝜃𝑓 )
(ii) Let 𝑉𝑄 be the potential at a point Q due to +q located at a point P, then Work
𝑊 = 𝑝 𝐸 (cos 0° − cos 90°) = 𝑝 𝐸 (1 − 0) = 𝑝 𝐸
required to bring the charge –q to the corner Q 𝑊 = 6.3 𝑋 10−30 𝑋 3 𝑋 105 = 18.9 𝑋 10−25 𝐽
1 𝑞
𝑊𝑄 = (−𝑞) 𝑉𝑄 = (−𝑞)  Hence for 10 water molecules, the total work done is
22
4 𝜋 𝜀𝑜 𝑃𝑄 𝑊𝑡𝑜𝑡 = 1022 𝑋 18.9 𝑋 10−25
1 𝑞2 𝑾𝒕𝒐𝒕 = 𝟏𝟖. 𝟗 𝑿 𝟏𝟎−𝟑 𝑱
𝑊𝑄 = −
4 𝜋 𝜀𝑜 𝑎 17. Calculate the electric flux through the rectangle of sides 5 cm and 10 cm kept
(iii) Let 𝑉𝑅 be the potential at the point R due to charges at the point P and Q, in the region of a uniform electric field 100 NC–1. The angle θ is 60o. If θ
then Work required to bring the charge +q to the corner R becomes zero, what is the electric flux?
1 𝑞 1 (−𝑞) Solution :- E = 100 N C-1 ; 𝐴 = 𝑙 𝑏 = 5 𝑋 10−2 𝑋 10 𝑋 10−2 = 50 𝑋 10−4 𝑚2 ; 𝜃 = 60°
𝑊𝑅 = 𝑞 𝑉𝑅 = 𝑞 [ + ]  The electric flux through the rectangular area
4 𝜋 𝜀𝑜 𝑃𝑅 4 𝜋 𝜀𝑜 𝑄𝑅
1 𝑞 1 𝑞 Φ𝐸 = 𝐸⃗ . 𝐴 = 𝐸 𝐴 cos 𝜃 = 100 𝑋 50 𝑋 10−4 𝑋 cos 60°
𝑊𝑅 = 𝑞 [ − ] 1
4 𝜋 𝜀𝑜 𝑎√2 4 𝜋 𝜀𝑜 𝑎 Φ𝐸 = 5000 𝑋 10−4 𝑋 = 2500 𝑋 10−4
𝟏 𝒒𝟐 𝟏 2
𝑾𝑹 = [ − 𝟏] 𝚽𝑬 = 𝟎. 𝟐𝟓 𝑵 𝒎𝟐 𝑪−𝟏
𝟒 𝝅 𝜺𝒐 𝒂 √𝟐
 For 𝜽 = 𝟎° Φ𝐸 = ⃗𝐸 . ⃗𝐴 = 𝐸 𝐴 cos 𝜃 = 100 𝑋 50 𝑋 10−4 𝑋 cos 0°
(iv) Let 𝑉𝑠 be the potential at the centre point O due to all the four charges in the
Φ𝐸 = 5000 𝑋 10−4 𝑋 1
four corners, then Work required to bring the fourth charge –q at the position S
𝚽𝑬 = 𝟎. 𝟓 𝑵 𝒎𝟐 𝑪−𝟏
1 𝑞 1 (−𝑞) 1 𝑞
𝑊𝑆 = (−𝑞)𝑉𝑆 = (−𝑞) [ + + ] 18. (i) In figure (a), calculate the electric flux through the closed areas A1 and A2.
4 𝜋 𝜀𝑜 𝑃𝑆 4 𝜋 𝜀𝑜 𝑄𝑆 4 𝜋 𝜀𝑜 𝑅𝑆 (ii) In figure (b), calculate the electric flux through the cube
1 𝑞 𝑞 𝑞 1 𝑞2 1
𝑊𝑆 = (−𝑞) [ − + ]= − [1 − + 1]
4 𝜋 𝜀𝑜 𝑎 𝑎√2 𝑎 4 𝜋 𝜀𝑜 𝑎 √2
𝟐
𝟏 𝒒 𝟏
𝑾𝑺 = − [𝟐 − ]
𝟒 𝝅 𝜺𝒐 𝒂 √𝟐
(b) The potential created by the two +q charges are canceled by the potential
created by the –q charges which are located in the opposite corners. Therefore
the net electric potential at the centre O due to all the charges in the corners is
zero.
Hence no work is required to bring any charge to the point O. Physically this
Solution :-
implies that if any charge q′ when brought close to O, then it moves to the point O
(i) In figure (a), the area A1 encloses the charge +𝑄. So electric flux through this
without any external force. 𝑄
closed surface A1 is Φ𝐸 = . But the closed surface A2 contains no charges
𝜀𝑜
inside, so electric flux through A2 is zero.

victory R. SARAVANAN. M.Sc., M.Phil., B.Ed PG ASST [PHYSICS], GBHSS, PARANGIPETTAI - 608 502
12 PHYSICS UNIT - 1 ELECTROSTATICS COMPLETE STUDY MATERIAL
(ii) In figure (b), the net charge inside the cube is ; 𝑄 = +5𝑞 − 2𝑞 = +3𝑞 and the 20. A parallel plate capacitor has square plates of side 5 cm and separated by a
𝑸 𝟑𝒒
total electric flux in the cube is therefore, 𝚽𝑬 = = distance of 1 mm. (a) Calculate the capacitance of this capacitor. (b) If a 10 V
𝜺𝒐 𝜺𝒐
battery is connected to the capacitor, what is the charge stored in any one of
Note that the charge -10 q lies outside the cube and it will not contribute the total
flux through the surface of the cube. the plates? (The value of o = 8.85 × 10–12 N–1m–2 C2)
19. A small ball of conducting material having a charge Solution :- a = 5 cm = 5 X 10-2 m ; A = a2 = 25 X 10-4 m2 ; d = 1 mm =10-3 m ; V = 10 V
𝜺𝒐 𝑨
+q and mass m is thrown upward at an angle θ to (a) Capacitance of the capacitor ; 𝑪 =
𝒅
horizontal surface with an initial speed vo as 8.85 𝑋 10−12 𝑋 25 𝑋 10−4
shown in the figure. There exists an uniform 𝐶= = 221.2 𝑋 10−13 𝐹 = 𝟐𝟐. 𝟏𝟐 𝑿 𝟏𝟎−𝟏𝟐 𝑭 = 𝟐𝟐. 𝟏𝟐 𝒑𝑭
10−3
electric field E downward along with the (b) Charge stored in any one of the plate ; 𝑸 = 𝑪 𝑽
gravitational field g. Calculate the range, maximum 𝑄 = 22.12 𝑋 10−12 𝑋 10 = 221.2 𝑋 10−12 𝐶 = 𝟐𝟐𝟏. 𝟐 𝒑𝑪
height and time of flight in the motion of this 21. A parallel plate capacitor filled with mica having εr = 5 is connected to a 10 V
charged ball. Neglect the effect of air and treat the
battery. The area of each parallel plate is 6 cm2 and separation distance is 6 mm.
ball as a point mass.
(a) Find the capacitance and stored charge. (b) After the capacitor is fully
Solution :-
charged, the battery is disconnected and the dielectric is removed carefully.
 If the conductor has no net charge, then its motion is the same as usual
Calculate the new values of capacitance, stored energy and charge.
projectile motion of a mass m
Solution :- 𝜀𝑟 = 5 ; 𝑉 = 10 𝑉 ; 𝐴 = 6 𝑐𝑚2 = 6 𝑋 10−4 𝑚2 ; 𝑑 = 6 𝑚𝑚 = 6 𝑋 10−3 𝑚
 Here, in this problem, in addition to downward gravitational force, the charge 𝜀𝑜 𝜺𝒓 𝐴
also will experience a downward uniform electrostatic force. (a) The capacitance of the capacitor in the presence of dielectric is ; 𝐶 =
𝑑
 The acceleration of the charged ball due to gravity = −𝑔 𝑗̂ 8.85 𝑋 10−12 𝑋 5 𝑋 6 𝑋 10−4
𝑞𝐸 𝐶 = = 44.25 𝑋 10−13 𝐹 = 4.425𝑋 10−12 𝐹 = 𝟒. 𝟒𝟐𝟓 𝒑𝑭
6 𝑋 10−3
 The acceleration of the charged ball due to uniform electric field = − 𝑗̂
𝑚 The stored charge ; 𝑄 = 𝐶 𝑉
𝒒𝑬
 ⃗ = − [𝒈 +
Total acceleration of charged ball in downward direction; 𝒂 ] 𝒋̂ 𝑄 = 4.425 𝑋 10−12 𝑋 10 = 44.25 𝑋 10−12 𝐶 = 𝟒𝟒. 𝟐𝟓 𝒑𝑪
𝒎 1
 Its magnitude ; 𝒂 = 𝒈 +
𝒒𝑬 The stored charge ; 𝑈 = 𝐶 𝑉 2
2
𝒎
1
 Galileo’s conclusion that all objects fall 𝑼 = 𝑋 4.425 𝑋 10−12 𝑋 10 𝑋 10 = 𝟐. 𝟐𝟏𝟑 𝑿 𝟏𝟎−𝟏𝟎 𝑱
at the same rate towards the Earth is 2
(b) After the removal of the dielectric, since the battery is already disconnected
true only in a uniform gravitational
the total charge will not change. But the potential difference between the
field. When a uniform electric field is
plates increases. As a result, the capacitance is decreased. New capacitance is
included, the acceleration of a charged
object depends on both mass and 𝐶 4.425 𝑋 10−12
𝑪𝒐 = = = 0.885 𝑋 10−12 𝐹 = 0. 𝟖𝟖𝟓 𝒑𝑭
charge. 𝜺𝒓 5
 But still the acceleration is constant The stored charge remains same and 44.25 pC. Hence newly stored energy is
throughout the motion. In fact we can 1 1 𝑄2 1 𝑄2 𝜀𝑟
𝒒𝑬 𝑈𝑜 = 𝐶𝑜 𝑉2 = = = 𝜀𝑟 𝑈
simply replace g by (𝒈 + ) 2 2 𝐶𝑜 2 𝐶
𝒎
Expression for without charge with charge +q 𝑼𝒐 = 5 𝑋 2.213 𝑋 10−10 = 𝟏𝟏. 𝟎𝟔𝟓 𝑿 𝟏𝟎−𝟏𝟎 𝑱
(i) Time of flight 2 𝑣𝑂 sin 𝜃 𝟐 𝒗𝑶 𝐬𝐢𝐧 𝜽 The increased energy is,
𝑇𝑓 = [𝑻𝒇 ]𝑬 = 𝒒𝑬
𝑔 [𝒈+ ]
𝒎
∆𝑼 = 𝑈𝑜 − 𝑈 = (11.065 − 2.213) 𝑋 10−10 = 𝟖. 𝟖𝟓𝟐 𝑿 𝟏𝟎−𝟏𝟎 𝑱
(ii) Maximum height 𝑣𝑜2 𝑠𝑖𝑛2 𝜃 𝒗𝒐𝟐 𝒔𝒊𝒏𝟐 𝜽  When the dielectric is removed, it experiences an inward pulling force due to
ℎ𝑚𝑎𝑥 = [𝒉𝒎𝒂𝒙 ]𝑬 = 𝒒𝑬 the plates. To remove the dielectric, an external agency has to do work on the
2𝑔 𝟐 [𝒈+ ]
𝒎 dielectric which is stored as additional energy. This is the source for the extra
(iii) Range 𝑣𝑜2 sin 2𝜃 𝒗𝒐𝟐 𝐬𝐢𝐧 𝟐𝜽 energy 8.84 × 10–10 J.
𝑅= [𝑹]𝑬 = 𝒒𝑬
𝑔 [𝒈+ ]
𝒎
 For +𝒒 , we have [𝑻𝒇 ] < 𝑻𝒇 ; [𝒉𝒎𝒂𝒙 ]𝑬 < 𝒉𝒎𝒂𝒙 ; [𝑹]𝑬 < 𝑹
𝑬
 For −𝒒 , we have [𝑻𝒇 ] > 𝑻𝒇 ; [𝒉𝒎𝒂𝒙 ]𝑬 > 𝒉𝒎𝒂𝒙 ; [𝑹]𝑬 > 𝑹
𝑬

victory R. SARAVANAN. M.Sc., M.Phil., B.Ed PG ASST [PHYSICS], GBHSS, PARANGIPETTAI - 608 502
12 PHYSICS UNIT - 1 ELECTROSTATICS COMPLETE STUDY MATERIAL
22. Find the equivalent capacitance between P and  Total charge ; 𝑄 = 𝑞1 + 𝑞2
Q for the configuration shown below in the 𝑟1
𝑄 = 𝑞2 + 𝑞2
figure. 𝑟2
Solution :- 𝑟1 + 𝑟2
𝑄 = 𝑞2 [ ]
 In fig (i), 1 F, 3 F and 6 F, 2 F are in 𝑟2
parallel connection, then 𝑟2 2 200 𝑋 10−9
∴ 𝑞2 = 𝑄 [ ] = 100 𝑋 10−9 𝑋 [ ]=
𝐶𝑃 = 𝐶1 + 𝐶2 = 1 + 3 = 4 F 𝑟1 + 𝑟2 8+2 10
𝐶𝑃 = 𝐶1 + 𝐶2 = 6 + 2 = 8 F 𝒒𝟐 = 𝟐𝟎 𝑿 𝟏𝟎−𝟗 𝑪 = 𝟐𝟎 𝒏𝑪
 In fig (ii), 4 F, 4 F and 8 F, 8 F are in series connection, then  Thus, 𝑞1 = 𝑄 − 𝑞2 = 100𝑋10−9 − 20𝑋10−9 = (100 − 20)𝑋 10−9
1 1 1 1 1 2 1 ∴ 𝒒𝟏 = 𝟖𝟎 𝑿 𝟏𝟎−𝟗 𝑪 = 𝟖𝟎 𝒏𝑪
= + = + = = ⟹ 𝐶𝑆 = 2 F  The electric charge density on sphere A is
𝐶𝑆 𝐶1 𝐶2 4 4 4 2
1 1 1 1 1 2 1 𝑞1 𝑞1 80 𝑋 10−9
= + = + = = ⟹ 𝐶𝑆 = 2 F 𝜎1 = = =
𝐶𝑆 𝐶1 𝐶2 4 4 4 2 𝐴1 4 𝜋 𝑟12 4 𝑋 3.14 𝑋 (8 𝑋10−2 )2
80 𝑋 10−9 10 𝑋10−5
 In fig (iii), 2 F, 4 F are in parallel connection, then 𝜎1 = = = 9.96 𝑋10−2 𝑋10−5
𝐶𝑃 = 𝐶1 + 𝐶2 = 2 + 4 = 6 F 4 𝑋 3.14 𝑋 64 𝑋 10−4 100.48
𝝈𝟏 = 𝟗. 𝟗𝟔 𝑿𝟏𝟎−𝟕 𝑪 𝒎−𝟐 = 𝟎. 𝟗𝟗𝟔 𝑿𝟏𝟎−𝟔 𝑪 𝒎−𝟐
So the equivalent capacitance between P and Q is = 𝟔 𝐅
 The electric charge density on sphere B is,
 Thus the combination of capacitances in figure can be replaced by a single
𝑞2 𝑞2 20 𝑋 10−9
capacitance 6 μF as shown in fig (iv) 𝜎2 = = =
𝐴2 4 𝜋 𝑟22 4 𝑋 3.14 𝑋 (2 𝑋10−2 )2
20 𝑋10−9 20 𝑋10−5
𝜎2 = = = 3. 981 𝑋10−1 𝑋10−5
4 𝑋 3.14 𝑋 4 𝑋10−4 50.24
𝝈𝟐 = 𝟑. 𝟗𝟖𝟏 𝑿𝟏𝟎−𝟔 𝑪 𝒎−𝟐
 Note that the surface charge density is greater on the smaller sphere compared
1
to the larger sphere (σ2 ≈ 4 σ1) which confirms that ; 𝜎 ∝
𝑟
 Since the potential on both spheres is the same, we have
1 𝑞1 80 𝑋 10−9
𝑽𝑨 = 𝑽𝑩 = = 9 𝑋 109 𝑋 = 90 𝑋 102 = 9000 𝑉 = 𝟗 𝒌𝑽
23. Two conducting spheres of radius r1 = 8 cm and r2 = 2 cm are separated by a 4 𝜋 𝜀𝑜 𝑟1 8 𝑋 10−2
distance much larger than 8 cm and are 24. Dielectric strength of air is 3 × 106 V m–1. Suppose the radius of a hollow
connected by a thin conducting wire as sphere in the Van de Graff generator is R = 0.5 m, calculate the maximum
shown in the figure. A total charge of potential difference created by this Van de Graaff generator.
Q = +100 nC is placed on one of the Solution :- 𝐸𝑚𝑎𝑥 = 3 𝑋 106 𝑉 𝑚−1 ; 𝑅 = 0.5 𝑚 ; 𝑉𝑚𝑎𝑥 = ?
spheres. After a fraction of a second, the  The electric field on the surface of the sphere is given by (by Gauss law)
charge Q is redistributed and both the 1 𝑄
𝑬𝒎𝒂𝒙 =
spheres attain electrostatic equilibrium. 4 𝜋 𝜀𝑜 𝑅2
(a) Calculate the charge and surface charge density on each sphere.  So the maximum potential difference created is given by,
(b) Calculate the potential at the surface of each sphere. 1 𝑄 1 𝑄 𝑅
Solution :- 𝑽𝒎𝒂𝒙 = = = 𝑬𝒎𝒂𝒙 𝑅
4 𝜋 𝜀𝑜 𝑅 4 𝜋 𝜀𝑜 𝑅 𝑅
(a) At electrostatic equilibrium, 𝑉𝐴 = 𝑉𝐵 𝑽𝒎𝒂𝒙 = 3 𝑋 10 𝑋 0.5 = 1.5 𝑋 106 𝑉
6
1 𝑞1 1 𝑞2 𝑽𝒎𝒂𝒙 = 𝟏. 𝟓 𝒎𝒊𝒍𝒍𝒊𝒐𝒏 𝒗𝒐𝒍𝒕
=
4 𝜋 𝜀𝑜 𝑟1 4 𝜋 𝜀𝑜 𝑟2
𝑞1 𝑞2
=
𝑟1 𝑟2
𝑟1
𝑞1 = 𝑞2 − − − − − − (1)
𝑟2
victory R. SARAVANAN. M.Sc., M.Phil., B.Ed PG ASST [PHYSICS], GBHSS, PARANGIPETTAI - 608 502
12 PHYSICS UNIT - 1 ELECTROSTATICS COMPLETE STUDY MATERIAL
Resultant force on ‘q’ along Y- axis

EXERCISE PROBLEMS WITH SOLUTIONS 𝐹𝑦 = 𝐹1 (− 𝑗̂) + 𝐹2 s𝑖𝑛 45° (− 𝑗̂ ) + 𝐹5 𝑗̂ + 𝐹4 sin 45° 𝑗̂
1. When two objects are rubbed with each other, approximately a charge of 50 nC 1 1
𝐹𝑦 = 𝐹1 𝑗̂ − 𝐹2 𝑗̂ + +𝐹5 𝑗̂ + 𝐹4 𝑗̂
can be produced in each object. Calculate the number of electrons that must be √2 √2
transferred to produce this charge.  Since, 𝐹1 = 𝐹2 = 𝐹4 = 𝐹5 = 𝐹, we get ⃗𝐹𝑦 = ⃗⃗0
Solution :- q = 50 nC = 50 X 10-9 C ; e = 1.6 X 10-19 C ; n=?  Simillarly resultant force on ‘q’ along X- axis,
 According to quantization of electric charges, ⃗𝐹𝑋 = 𝐹3 𝑖̂ + 𝐹2 𝑐𝑜𝑠 45° 𝑖̂ + 𝐹4 cos 45° 𝑖̂
𝑞 =𝑛𝑒 1 1
𝑞 50 𝑋 10−9 50 𝑋 1010 ⃗𝐹𝑋 = 𝐹3 𝑖̂ + 𝐹2 𝑖̂ + 𝐹4 𝑖̂
∴ 𝑛= = −19
= √ 2 √ 2
𝑒 1.6 𝑋 10 1.6 1 1
𝑛 = 3.125 𝑋 101 𝑋 1010 ⃗𝐹𝑋 = 𝐹3 𝑖̂ + 2 𝐹2 𝑖̂ = 𝐹 [1 + 2 ] 𝑖̂
𝟏𝟎
𝒏 = 𝟑𝟏. 𝟐𝟓 𝑿 𝟏𝟎 𝒆𝒍𝒆𝒄𝒕𝒓𝒐𝒏𝒔 √2 √2
⃗𝐹𝑋 = 𝐹 [1 + √2 ]𝑖̂
2. The total number of electrons in the human body is typically in the order of  Thus total force on charge ‘q’,
1028. Suppose, due to some reason, you and your friend lost 1% of this 𝐹𝑡𝑜𝑡 = ⃗⃗⃗𝐹𝑋 = 𝐹 [1 + √2 ] 𝑖̂
number of electrons. Calculate the electrostatic force between you and your 𝟏 𝑸𝒒
friend separated at a distance of 1m. Compare this with your weight. Assume ⃗𝑭𝒕𝒐𝒕 = [𝟏 + √𝟐 ]𝒊̂
𝟒 𝝅 𝜺𝒐 𝑹 𝟐
mass of each person is 60 kg and use point charge approximation.
1  Hence, total force on the charge ‘q’ acts along X-axis whose magnitude is,
Solution:- 𝑁 = 1028 ; 𝑛 = 1028 𝑋 1% = 1028 𝑋 = 1026 ; 𝑟 = 1 𝑚 ; 𝑚 = 60 𝑘𝑔 𝟏 𝑸𝒒
100
 Charge on each person ; 𝑞 = 𝑛 𝑒 = 1028 𝑋 1.6 𝑋 1019 = 1.6 𝑋 107 𝐶 𝑭𝒕𝒐𝒕 = [𝟏 + √𝟐 ] 𝑵
𝟒 𝝅 𝜺𝒐 𝑹 𝟐
 Then electrostatic force between them, 4. Suppose a charge +q on Earth’s surface and another +q charge is placed on
1 𝑞1 𝑞2 1 𝑞2 the surface of the Moon. (a) Calculate the value of q required to balance the
𝐹𝐸 = =
4 𝜋 𝜀𝑜 𝑟 2 4 𝜋 𝜀𝑜 𝑟 2 gravitational attraction between Earth and Moon (b) Suppose the distance
(1.6 7 )2 between the Moon and Earth is halved, would the charge q change?
𝑋 10
𝐹𝐸 = 9 𝑋 109 𝑋 = 9 𝑋 109 𝑋 2.56 𝑋 1014 (Take mE = 5.9 × 1024 kg, mM = 7.9 × 1022 kg)
1
𝐹𝐸 = 𝟐𝟑. 𝟎𝟒 𝑿 𝟏𝟎 𝑵 𝟐𝟑 Solution :- 𝐺 = 6.67 𝑋 10−11 𝑁 𝑚2 𝑘𝑔−2 ; 𝑞 =?
 Weight of the person ; ; 𝑊 = 𝑚 𝑔 = 60 𝑋 9.8 = 588 𝑁  Let ‘R’ be the distance between earth and moon.
 Hence the ratio of the force and the weight, ,  Since the gravitational attraction is balanced by electrostatic force, 𝐹𝐸 = 𝐹𝐺
2
𝐹𝐸 23.04 𝑋 10 23 1 𝑞 𝑚 𝑚
𝐸 𝑀 𝐺 𝑚𝐸 𝑚𝑀
= = 3.918 𝑋10−2 𝑋1023 =𝐺 (𝑜𝑟) 𝑞2 =
4 𝜋 𝜀𝑜 𝑅 2 𝑅 2 1
𝑊 588 [ ]
𝑭𝑬 4 𝜋 𝜀𝑜
−𝟐𝟏
= 𝟑. 𝟗𝟏𝟖 𝑿𝟏𝟎
𝑾 𝐺 𝑚𝐸 𝑚𝑀 6.67 𝑋 10−11 𝑋 5.9 𝑋 1024 𝑋 7.9 𝑋 1022
3. Five identical charges Q are placed equidistant on a ∴ 𝑞= √ = √
1 9 𝑋 109
semicircle as shown in the figure. Another point charge q [ ]
4 𝜋 𝜀𝑜
is kept at the centre of the circle of radius R. Calculate the
electrostatic force experienced by the charge q. 6.67 𝑋 5.9 𝑋 7.9 𝑋 1026
Solution :- 𝑞= √
9
 Let the forces acts on ‘q’ be ⃗𝐹1 , ⃗𝐹2 , ⃗𝐹3 , ⃗𝐹4 , ⃗𝐹5 𝟏𝟑
 Here the magnitudes of these forces must be same. (i.e) 𝒒 = 𝟓. 𝟖𝟕𝟖 𝑿 𝟏𝟎 𝑪
 When the distance between earth and moon is halved,
𝐹1 = 𝐹2 = 𝐹3 = 𝐹4 = 𝐹5 = 𝐹
the charge ‘q’ remains the same, because the value of
 Here ⃗⃗𝐹2 and ⃗⃗𝐹4 are resolved into two perpendicular components. (i.e.) charge is independent of the distance.
(1) 𝐹2 cos 45° 𝑖̂ & 𝐹2 s𝑖𝑛 45° (− 𝑗̂ )
(2) 𝐹4 cos 45° 𝑖̂ & 𝐹4 sin 45° 𝑗̂

victory R. SARAVANAN. M.Sc., M.Phil., B.Ed PG ASST [PHYSICS], GBHSS, PARANGIPETTAI - 608 502
12 PHYSICS UNIT - 1 ELECTROSTATICS COMPLETE STUDY MATERIAL
5. Draw the free body diagram for the following charges as shown in the figure 7. A closed triangular box is kept in an
(a), (b) and (c). electric field of magnitude
E = 2 × 103 N C–1 as shown in the
figure. Calculate the electric flux
through the (a) vertical rectangular
surface (b) slanted surface and (c)
entire surface.
Solution :-
(a) Vertical rectangular surface : (𝑙 = 15 𝑐𝑚 ; 𝑏 = 5 𝑐𝑚 ; 𝜃 = 0°)
Solution :-
Φ𝐸 = ∮ 𝐸 𝑑𝑠 cos 𝜃 = 𝐸 ∮ 𝑑𝑠 cos 0°

Φ𝐸 = 𝐸 𝐴 = 𝐸 𝑙 𝑏
Φ𝐸 = 2 𝑋 103 𝑋 15 𝑋 10−2 𝑋 5 𝑋 10−2 = 150 𝑋 10−1
𝚽𝑬 = 𝟏𝟓 𝑵 𝒎𝟐 𝑪−𝟏
(b) Slanted surface : (𝑙 = 15 𝑐𝑚 ; 𝑥 ; 𝜃 = 60°)
5 5 5
sin 30 ° = ⟹ 𝑥 = = = 10 𝑐𝑚
𝑥 𝑠𝑖𝑛 30° 1
( )
2
1
6. Consider an electron travelling with a Φ𝐸 = ∮ 𝐸 𝑑𝑠 cos 𝜃 = 𝐸 ∮ 𝑑𝑠 cos 60° = 𝐸 𝐴 [ ]
2
speed 𝒗𝟎 and entering into a uniform 1
electric field ⃗𝑬 which is perpendicular Φ𝐸 = 𝐸𝑙𝑥
2
to 𝒗⃗⃗⃗ 𝟎 as shown in the Figure. Ignoring 2 𝑋 103 𝑋 15 𝑋 10−2 𝑋 10 𝑋 10−2
gravity, obtain the electron’s Φ𝐸 =
2
acceleration, velocity and position as
𝚽𝑬 = 𝟏𝟓 𝑵 𝒎𝟐 𝑪−𝟏
functions of time.
Solution :- (c) Entire surface :
(i) Acceleration :  In the figure electric field lines enters through the vertical rectangular
surface and leaves through the slanted surface. Also the closed surface
 From Newton’s second law ; ⃗𝐹 = 𝑚 𝑎 ⃗ contains no charge inside.
⃗ = 𝑒𝐸
 Force experienced by the electron in the electric field ; 𝐹 ⃗  In general the electric flux is negative if the electric field lines enter the closed
 Thus 𝑚𝑎 = 𝑒𝐸 ⃗ surface and positive if the electric field lines leave the closed surface. Thus the
𝑒 𝐸⃗ 𝑒𝐸 total electric flux through entire surface is zero (i.e.) 𝚽𝑬 = − 𝟏𝟓 + 𝟏𝟓 = 𝟎
𝑎 = = (−𝑗̂) 8. The electrostatic potential is given as a function of x in figure (a) and (b).
𝑚 𝑚
𝒆𝑬 Calculate the corresponding electric fields in regions A, B, C and D. Plot the
⃗𝒂 = − 𝒋̂ electric field as a function of x for the figure (b).
𝒎
(ii) Velocity :
 From equation of motion ; ⃗𝑣 = ⃗⃗𝑢 + ⃗𝑎 𝑡
𝒆𝑬
⃗ = 𝒗𝟎 𝒊̂ −
𝒗 𝒕 𝒋̂
𝒎
(iii) Position :
1
 From equation of motion ; 𝑆 = 𝑢 ⃗ 𝑡 + 𝑎 𝑡2
2
𝟏 𝒆𝑬 𝟐
⃗𝑺 = 𝒗𝟎 𝒕 𝒊̂ − 𝒕 𝒋̂
𝟐 𝒎

victory R. SARAVANAN. M.Sc., M.Phil., B.Ed PG ASST [PHYSICS], GBHSS, PARANGIPETTAI - 608 502
12 PHYSICS UNIT - 1 ELECTROSTATICS COMPLETE STUDY MATERIAL
Solution :- Solution :-
Figure (a) : According to the relation between E and V we have ; 𝐸= −
𝑑𝑉 𝑑 = 0.6 𝑚𝑚 = 0.6 𝑋 10−3 𝑚 ; 𝐸 = 3 𝑋 106 𝑉 𝑚−1 ; 𝑑  = 1 𝑚𝑚 = 1 𝑋 10−3 𝑚
𝑑𝑥 𝑉
 In region - A ; 𝑑𝑉 = 5 − 8 = − 3 ; 𝑑𝑥 = 0.2 − 0 = 0.2 (a) Let ‘V’ be the required potential difference, then ; 𝐸 =
𝑑
𝑑𝑉 (−3) 30 (𝑜𝑟) 𝑉 = 𝐸 𝑑 = 3 𝑋 106 𝑋 0.6 𝑋 10−3 = 1.8 𝑋 103
∴ 𝑬𝑨 = − = − = + = +𝟏𝟓 𝑽 𝒎−𝟏 𝑽 = 𝟏𝟖𝟎𝟎 𝑽
𝑑𝑥 0.2 2
 In region - B ; 𝑑𝑉 = 5 − 5 = 0 ; 𝑑𝑥 = 0.4 − 0.2 = 0.2 (b) Since potential difference is directly proportional to the separation (𝑉 = 𝐸 𝑑), if
𝑑𝑉 0 separation increases, potential difference also increases.
∴ 𝑬𝑩 = − = − = 𝟎
𝑑𝑥 0.2 (c) Let 𝑉 be the new potential difference, then ; 𝐸 =
𝑉
 In region - C ; 𝑑𝑉 = 7 − 5 = 2 ; 𝑑𝑥 = 0.6 − 0.4 = 0.2 𝑑
𝑑𝑉 2 20 (𝑜𝑟) 𝑉 = 𝐸 𝑑 = 3 𝑋 106 𝑋 1 𝑋 10−3 = 3 𝑋 103
∴ 𝑬𝑪 = − = − = − = − 𝟏𝟎 𝑽 𝒎−𝟏
𝑑𝑥 0.2 2 𝑽 = 𝟑𝟎𝟎𝟎 𝑽
 In region - D ; 𝑑𝑉 = 1 − 7 = − 6 ; 𝑑𝑥 = 0.8 − 0.6 = 0.2 10. A point charge of +10 μC is placed at a
𝑑𝑉 (−6) 60 distance of 20 cm from another identical
∴ 𝑬𝑫 = − = − = + = +𝟑𝟎 𝑽 𝒎−𝟏
𝑑𝑥 0.2 2 point charge of +10 μC. A point charge of -2
Figure (b) : μC is moved from point a to b as shown in
𝑑𝑉 30 the figure. Calculate the change in potential
𝑑𝑥 = 1 − 0 = 1 ; 𝑑𝑉 = 30 − 0 = 30 ; 𝑬=− = − = −𝟑𝟎 𝑽 𝒎−𝟏
𝑑𝑥 1 energy of the system? Interpret your result.
𝑑𝑉 (−30) Solution :-
𝑑𝑥 = 2 − 1 = 1 ; 𝑑𝑉 = 0 − 30 = −30 ; 𝑬=− = − = +𝟑𝟎 𝑽 𝒎−𝟏
𝑑𝑥 1
𝑑𝑉
𝑑𝑥 = 3 − 2 = 1 ; 𝑑𝑉 = 0 − 0 = 0 ; 𝑬=− = 𝟎
𝑑𝑥
𝑑𝑉 (−30)
𝑑𝑥 = 4 − 3 = 1 ; 𝑑𝑉 = −30 − 0 = −30 ; 𝑬 = − = − = +𝟑𝟎 𝑽 𝒎−𝟏
𝑑𝑥 1
𝑑𝑉 30
𝑑𝑥 = 5 − 4 = 1 ; 𝑑𝑉 = 0 − (−3)0 = +30 ; 𝑬 = − = − = −𝟑𝟎 𝑽 𝒎−𝟏
𝑑𝑥 1

𝑞1 = 𝑞2 = +10 𝜇𝐶 ; 𝑞 = −2 𝜇𝐶 ; 𝑟1 = 5 𝑐𝑚 ; 𝑟2 = 15 𝑐𝑚
𝑟1! = √ 52 + 52 = √50 = 5√2 𝑐𝑚 ; 𝑟2!= √152 + 52
= √250 = 5√10 𝑐𝑚
 Initial potential energy, when −2 𝜇𝐶 is at ‘a’
1 𝑞1 𝑞 1 𝑞2 𝑞 1 𝑞1 𝑞 𝑞2 𝑞
𝑈𝑖 = + = [ + ]
4 𝜋 𝜀𝑜 𝑟1 4 𝜋 𝜀𝑜 𝑟2 4 𝜋 𝜀𝑜 𝑟1 𝑟2
1 1 1
𝑈𝑖 = 𝑞 𝑞 [ + ] [∵ 𝑞1 = 𝑞2 ]
4 𝜋 𝜀𝑜 1 𝑟1 𝑟2
1 1
𝑈𝑖 = 9 𝑋 109 𝑋 10 𝑋 10−6 𝑋 (−2)𝑋 10−6 𝑋 [ + ]
5 𝑋 10−2 15 𝑋 10−2
9. A spark plug in a bike or a car is used to ignite the air-fuel mixture in the 1 1 1
engine. It consists of two electrodes separated by a gap 𝑈𝑖 = − 9 𝑋 2 𝑋 10−2 𝑋 [ + ] 𝑋
5 15 10−2
of around 0.6 mm gap as shown in the figure. To create 3+1 4 24
𝑈𝑖 = − 9 𝑋 2 𝑋 [ ] = −18 𝑋 = −
the spark, an electric field of magnitude 3 × 106 V m–1 is 15 15 5
required. (a) What potential difference must be 𝑼𝒊 = − 𝟒. 𝟖 𝑱
applied to produce the spark? (b) If the gap is increased,
does the potential difference increase, decrease or
remains the same? (c) find the potential
difference if the gap is 1 mm.
victory R. SARAVANAN. M.Sc., M.Phil., B.Ed PG ASST [PHYSICS], GBHSS, PARANGIPETTAI - 608 502
12 PHYSICS UNIT - 1 ELECTROSTATICS COMPLETE STUDY MATERIAL
 Final potential energy, when −2 𝜇𝐶 is at ‘b’ Figure (c):
1 𝑞1 𝑞 1 𝑞2 𝑞 1 𝑞1 𝑞 𝑞2 𝑞  Since 𝐶𝑂 , 𝐶𝑂 and 𝐶𝑂 are in parallel,
𝑈𝑓 = + = [ + ! ]
4 𝜋 𝜀𝑜 𝑟1! 4 𝜋 𝜀𝑜 𝑟2! 4 𝜋 𝜀𝑜 𝑟1!1 𝑟2 𝐶𝑃 = 𝐶𝑂 + 𝐶𝑂 + 𝐶𝑂
1 1 1 𝑪𝑷 = 𝟑 𝑪𝑶
𝑈𝑓 = 𝑞 𝑞 [ ! + !] [∵ 𝑞1 = 𝑞2 ] Figure (d):
4 𝜋 𝜀𝑜 1 𝑟1 𝑟2
1 1 (1) Resultant capacitors across P and Q :
𝑈𝑓 = 9 𝑋 109 𝑋 10 𝑋 10−6 𝑋 (−2)𝑋 10−6 𝑋 [ + ]
5√2 𝑋 10 −2 5√10 𝑋 10−2
1 1 1
𝑈𝑓 = − 9 𝑋 2 𝑋 10−2 𝑋 [ + ]𝑋
√2 √10 5 𝑋 10−2
1 1.023
𝑈𝑓 = − 9 𝑋 2 𝑋 [0.707 + 0.316] 𝑋 = −18 𝑋 = −18 𝑋 0.2046
5 5
𝑼𝒇 = − 𝟑. 𝟔𝟖𝟑 𝑱
 The change in potential energy ;
Here, 𝐶1 and 𝐶3 are in series,
∆ 𝑈 = 𝑈𝑓 − 𝑈𝑓 = −3.683 − (−4.8) = −3. 683 + 4.8 = +𝟏. 𝟏𝟏𝟕 𝑱 ≈ +𝟏. 𝟏𝟐 𝑱
1 1 1 𝐶1 + 𝐶3 𝐶1 𝐶3
 Positive sign implies that to move the charge –2μC external work is required. = + = ⇒ 𝐶𝑆1 =
𝐶𝑆1 𝐶1 𝐶3 𝐶1 𝐶3 𝐶1 + 𝐶3
11. Calculate the resultant capacitances for each of the following combinations of
capacitors. 𝐶2 and 𝐶4 are in series,
1 1 1 𝐶2 + 𝐶4 𝐶2 𝐶4
= + = ⇒ 𝐶 𝑆2 =
𝐶 𝑆2 𝐶2 𝐶4 𝐶2 𝐶4 𝐶2 + 𝐶4
𝐶𝑆1 and 𝐶𝑆2 are in parallel
𝐶𝑃𝑄 = 𝐶𝑆1 + 𝐶𝑆2
𝐶1 𝐶3 𝐶2 𝐶4
𝐶𝑃𝑄 = +
𝐶1 + 𝐶3 𝐶2 + 𝐶4
Solution :- ( 𝐶2 + 𝐶4 ) 𝐶1 𝐶3 + ( 𝐶1 + 𝐶3 ) 𝐶2 𝐶4
𝐶𝑃𝑄 =
 Effective capacitance of capacitors in series, ( 𝐶1 + 𝐶3 ) ( 𝐶2 + 𝐶4 )
1 1 1 1 𝐶1 𝐶2 𝐶3 + 𝐶1 𝐶3 𝐶4 + 𝐶1 𝐶2 𝐶4 + 𝐶2 𝐶3 𝐶4
= + + +⋯ 𝐶𝑃𝑄 =
𝐶𝑆 𝐶1 𝐶2 𝐶3 ( 𝐶1 + 𝐶3 ) ( 𝐶2 + 𝐶4 )
 Effective capacitance of capacitors in series, (2) Resultant capacitors across R and S :
𝐶𝑃 = 𝐶1 + 𝐶2 + 𝐶3 + ⋯
Figure (a) :
 Since 𝐶𝑂 and 𝐶𝑂 are in parallel,
𝐶𝑃 = 𝐶𝑂 + 𝐶𝑂 = 2 𝐶𝑂
 Also, 𝐶𝑂 and 2 𝐶𝑂 are in series,
1 1 1 3 𝟐 𝑪𝒐
= + = (𝑜𝑟) 𝑪𝑺 =
𝐶𝑆 𝐶𝑂 2 𝐶𝑜 2 𝐶𝑜 𝟑
Figure (b): Here, 𝐶1 and 𝐶2 are in series,
 Since 𝐶𝑂 and 𝐶𝑂 are in parallel, 1 1 1 𝐶1 + 𝐶2 𝐶1 𝐶2
= + = ⇒ 𝐶𝑆1 =
𝐶𝑃 = 𝐶𝑂 + 𝐶𝑂 = 2 𝐶𝑂 𝐶𝑆1 𝐶1 𝐶2 𝐶1 𝐶2 𝐶1 + 𝐶2
 Also, 2 𝐶𝑂 and 2 𝐶𝑂 are in series, 𝐶3 and 𝐶4 are in series,
1 1 1 2 1 1 1 1 𝐶3 + 𝐶4 𝐶3 𝐶4
= + = = (𝑜𝑟) 𝑪𝑺 = 𝑪𝒐 = + = ⇒ 𝐶 𝑆2 =
𝐶𝑆 2 𝐶𝑂 2 𝐶𝑜 2 𝐶𝑜 𝐶𝑜 𝐶 𝑆2 𝐶3 𝐶4 𝐶3 𝐶4 𝐶3 + 𝐶4

victory R. SARAVANAN. M.Sc., M.Phil., B.Ed PG ASST [PHYSICS], GBHSS, PARANGIPETTAI - 608 502
12 PHYSICS UNIT - 1 ELECTROSTATICS COMPLETE STUDY MATERIAL
Thus 𝐶𝑆1 and 𝐶𝑆2 are in parallel 1 𝑒𝐸 2 1 𝑒𝐸 2
ℎ =0+ 𝑡𝑒 = 𝑡
𝐶𝑅𝑆 = 𝐶𝑆1 + 𝐶𝑆2 2 𝑚 2 𝑚 𝑒
𝐶1 𝐶2 𝐶3 𝐶4 2𝑚ℎ
𝐶𝑅𝑆 = + (𝑜𝑟) 𝑡𝑒 2 =
𝐶1 + 𝐶2 𝐶3 + 𝐶4 𝑒𝐸
( 𝐶3 + 𝐶4 ) 𝐶1 𝐶2 + ( 𝐶1 + 𝐶2 ) 𝐶3 𝐶4 2𝑚ℎ 2 𝑋 9.1 𝑋 10−31 𝑋 1 X 10−3
𝐶𝑅𝑆 = ∴ 𝑡𝑒 = √ = √
( 𝐶1 + 𝐶2 ) ( 𝐶3 + 𝐶4 ) 𝑒𝐸 1.6 𝑋 10−19 𝑋 5 𝑋 103
𝐶1 𝐶2 𝐶3 + 𝐶1 𝐶2 𝐶4 + 𝐶1 𝐶3 𝐶4 + 𝐶2 𝐶3 𝐶4
𝐶𝑅𝑆 = 18.2 𝑋 10−18
( 𝐶1 + 𝐶2 ) ( 𝐶3 + 𝐶4 )
𝒕𝒆 = √ = √2. 275 𝑋 10−18 = 1.508 𝑋 10−9 𝑠
Figure (e) : 8
𝒕𝒆 ≈ 𝟏. 𝟓 𝒏 𝒔
𝐹 𝑒𝐸
Time of flight of proton : 𝑡 = 𝑡𝑃 ; 𝑆 = h ; u = 0 ; a = =
𝑚 𝑚
1 2
𝑆=𝑢𝑡+ 𝑎𝑡
2
1 𝑒𝐸 2 1 𝑒𝐸 2
ℎ=0+ 𝑡𝑃 = 𝑡
2 𝑚 2 𝑚 𝑒
2𝑚ℎ
∴ 𝑡𝑃 2 =
Here, 𝐶0 and 𝐶0 are in series, 𝑒𝐸
1 1 1 2 𝐶𝑂
= + = ⇒ 𝐶𝑆1 = 2𝑚ℎ 2 𝑋 1.6 𝑋 10−27 𝑋 1 X 10−3
𝐶𝑆1 𝐶𝑂 𝐶𝑂 𝐶𝑂 2 𝑡𝑃 = √ = √
𝑒𝐸 1.6 𝑋 10−19 𝑋 5 𝑋 103
Also, 𝐶0 and 𝐶0 are in series,
1 1 1 2 𝐶𝑂
= + = ⇒ 𝐶𝑆2 = 2 𝑋 10−14
𝐶𝑆2 𝐶𝑂 𝐶𝑂 𝐶𝑂 2 𝑡𝑃 = √ = √0.4 𝑋 10−14
5
Thus, 𝐶𝑆1 , 𝐶𝑆2 and 𝐶0 are in parallel,
𝐶𝑂 𝐶𝑂 𝑡𝑃 = √40 𝑋 10−16
𝐶𝑃 = 𝐶𝑆1 + 𝐶𝑆2 + 𝐶𝑂 = + + 𝐶𝑂 𝑡𝑃 = 6. 325 𝑋 10−8 = 63.25 𝑋 10−9 𝑠
2 2
𝑪𝑷 = 𝟐 𝑪𝑶 𝒕𝑷 ≈ 𝟔𝟑 𝒏 𝒔
12. An electron and a proton are allowed to fall through the separation between (b) Time of flight of neutron ; 𝑡 = 𝑡𝑛 ; 𝑆 = h ; u = 0 ; a = 𝑔
the plates of a parallel plate capacitor of voltage 5 V and separation distance 1
𝑆 = 𝑢 𝑡 + 𝑎 𝑡2
h = 1 mm as shown in the figure. (a) Calculate the time of flight for both 2
electron and proton (b) Suppose if a neutron is allowed to fall, what is the time 1 1
ℎ=0+ 𝑔 𝑡𝑛 2 = 𝑔 𝑡𝑛 2
of flight? (c) Among the three, which one will reach the bottom first? 2 2
2ℎ
∴ 𝑡𝑛 2 =
𝑔
2ℎ 2 𝑋 1 X 10−3
𝑡𝑛 = √ = √
𝑔 10

2 𝑋 10−3
(Take mp = 1.6 × 10–27
kg, me = 9.1 × 10–31
kg and g = 10 m s–2) 𝑡𝑛 = √ = √2 𝑋 10−4 = 1. 414 𝑋 10−2
10
𝑉 5
Solution :- h = 1 mm = 1 X 10−3 𝑚 ; 𝑉 = 5 𝑉 ; 𝐸 = = = 5 𝑋 103 𝑁𝐶 −1 𝒕𝒏 = 𝟏𝟒. 𝟏𝟒 𝑿 𝟏𝟎−𝟑 𝒔 = 𝟏𝟒. 𝟏𝟒 𝒎 𝒔
ℎ 10−3
𝐹 𝑒𝐸
(a) Time of flight of electron ; 𝑡 = 𝑡𝑒 ; 𝑆 = h ; u=0 ; a= = (c) Since, 𝒕𝒆 < 𝒕𝑷 < 𝒕𝒏 , out of three, electron reach first..
𝑚 𝑚
1
𝑆 = 𝑢 𝑡 + 𝑎 𝑡2
2

victory R. SARAVANAN. M.Sc., M.Phil., B.Ed PG ASST [PHYSICS], GBHSS, PARANGIPETTAI - 608 502
12 PHYSICS UNIT - 1 ELECTROSTATICS COMPLETE STUDY MATERIAL
13. During a thunder storm, the movement of (b) Potential difference across each capacitor :
water molecules within the clouds creates 𝑉 9 𝑉 9
friction, partially causing the bottom part of 𝑽𝒂 = = = 𝟑 𝑽 ; 𝑽𝒃 = = = 𝟑 𝑽
3 3 3 3
the clouds to become negatively charged. 𝑉 9 𝑉 9
This implies that the bottom of the cloud 𝑽𝒄 = = = 𝟑𝑽 ; 𝑽𝒅 = = = 𝟑 𝑽
3 3 3 3
and the ground act as a parallel plate
(c) Energy stored in each capacitor ;
capacitor. If the electric field between the
1 1
cloud and ground exceeds the dielectric 𝑼𝒂 = 𝐶𝑎 𝑉𝑎2 = 𝑋 8 𝑋 10−6 𝑋 32 = 36 𝑋 10−6 𝐽 = 𝟑𝟔 𝝁 𝑱
breakdown of the air (3 × 106 Vm–1 ), 2 2
1 1
lightning will occur. 𝑼𝒃 = 𝐶𝑏 𝑉𝑏2 = 𝑋 6 𝑋 10−6 𝑋 32 = 27 𝑋 10−6 𝐽 = 𝟐𝟕 𝝁 𝑱
(a) If the bottom part of the cloud is 1000 m above the ground, determine 2 2
the electric potential difference that exists between the cloud and ground. 1 1
𝑼𝒄 = 𝐶𝑐 𝑉𝑐 2 = 𝑋 2 𝑋 10−6 𝑋 32 = 9 𝑋 10−6 𝐽 = 𝟗 𝝁 𝑱
(b) In a typical lightning phenomenon, around 25 C of electrons are 2 2
transferred from cloud to ground. How much electrostatic potential energy is 1 1
𝑼𝒅 = 𝐶𝑑 𝑉𝑑2 = 𝑋 8 𝑋 10−6 𝑋 32 = 36 𝑋 10−6 𝐽 = 𝟑𝟔 𝝁 𝑱
transferred to the ground? 2 2
Solution :- 𝑬 = 3 𝑋 106 𝑉 𝑚−1 ; 𝑑 = 1000 𝑚 ; 𝑞 = 25 𝐶 15. Capacitors P and Q have identical
(a) Formula for electric field ; 𝐸 =
𝑉 cross sectional areas A and
𝑑 separation d. The space between the
Hence the potential difference ; 𝑉 = 𝐸 𝑑 = 3 𝑋 106 𝑋 1000 = 𝟑 𝑿 𝟏𝟎𝟗 𝑽 capacitors is filled with a dielectric
(b) Since the electrostatic potential energy is the amount of work done, then of dielectric constant 𝜺𝒓 as shown in
𝑊 = 𝑞 𝑉 = 25 𝑋 3 𝑋 109 = 𝟕𝟓 𝑿 𝟏𝟎𝟗 𝑱 the figure. Calculate the capacitance
14. For the given capacitor configuration (a) Find the of capacitors P and Q.
charges on each capacitor (b) potential Solution :-
difference across them (c) energy stored in each
capacitor.
Solution :- 𝐶𝑎 = 8 𝜇𝐹 ; 𝐶𝑏 = 6 𝜇𝐹 ; 𝐶𝑐 = 2 𝜇𝐹 ;
𝐶𝑑 = 8 𝜇𝐹 ; 𝑉 = 9 𝑉
 Here, 𝐶𝑏 and 𝐶𝑐 are in parellel,
𝐶𝑏𝑐 = 𝐶𝑏 + 𝐶𝑐 = 6 + 2 = 8 𝜇 𝐹 (1) Capacitor 𝑷 : Here two capacitors are connected in parallel. They are,
 Effective capacitance of this series connection, 𝐴 𝐴
𝜀𝑜 [ ] 𝜀𝑜 𝐴 𝜀𝑟 𝜀𝑜 [ ]
1 1 1 1 3 8
𝐶1 = 2 = & 𝐶2 = 2 = 𝜀𝑟 𝜀𝑜 𝐴
= + + = (𝑜𝑟) 𝐶𝑠 = 𝜇𝐹
𝐶𝑠 8 8 8 8 3 𝑑 2𝑑 𝑑 2𝑑
 Net charge on this series ciruit ;  Thus effective capacitance in parallel combination,
8 𝜀𝑜 𝐴 𝜀𝑟 𝜀𝑜 𝐴 𝜺𝒐 𝑨
𝑄 = 𝐶𝑠 𝑉 = 𝑋 10−6 𝑋 9 = 𝟐𝟒 𝑿 𝟏𝟎−𝟔 𝑪 𝐶𝑃 = 𝐶1 + 𝐶2 = + = (𝟏 + 𝜺𝒓 )
3 2𝑑 2𝑑 𝟐𝒅
(a) Charges on each capacitor : (2) Capacitor 𝑸 : Here two capacitors are connected in series. They are
 Since 𝐶𝑎 = 𝐶𝑏𝑐 = 𝐶𝑑 , potential difference across them also equal (i.e.) 𝜀𝑟 𝜀𝑜 𝐴 2 𝜀𝑟 𝜀𝑜 𝐴 𝜀𝑜 𝐴 2 𝜀𝑜 𝐴
𝐶1 = = & 𝐶2 = =
𝑉 𝑑 𝑑 𝑑 𝑑
𝑉𝑎 = 𝑉𝑏𝑐 = 𝑉𝑑 = =3𝑉 [ ] [ ]
3 2 2
 Thus charges on each capacitor ;  Thus effective capacitance in series combination,
𝒒𝒂 = 𝐶𝑎 𝑉𝑎 = 8 𝑋 10−6 𝑋 3 = 𝟐𝟒 𝑿 𝟏𝟎−𝟔 𝑪 1 1 1 𝑑 𝑑 𝑑 1 𝑑 1 + 𝜀𝑟
= + = + = [ + 1] = [ ]
𝒒𝒃 = 𝐶𝑏 𝑉𝑏𝑐 = 6 𝑋 10−6 𝑋 3 = 𝟏𝟖 𝑿 𝟏𝟎−𝟔 𝑪 𝐶𝑄 𝐶1 𝐶2 2 𝜀𝑟 𝜀𝑜 𝐴 2 𝜀𝑜 𝐴 2 𝜀𝑜 𝐴 𝜀𝑟 2 𝜀𝑜 𝐴 𝜀𝑟
𝒒𝒄 = 𝐶𝑐 𝑉𝑏𝑐 = 2 𝑋 10−6 𝑋 3 = 𝟔 𝑿 𝟏𝟎−𝟔 𝑪 𝟐 𝜺𝒐 𝑨 𝜺𝒓
∴ 𝑪𝑸 = [ ]
𝒒𝒅 = 𝐶𝑑 𝑉𝑑 = 8 𝑋 10−6 𝑋 3 = 𝟐𝟒 𝑿 𝟏𝟎−𝟔 𝑪 𝒅 𝟏 + 𝜺𝒓
victory R. SARAVANAN. M.Sc., M.Phil., B.Ed PG ASST [PHYSICS], GBHSS, PARANGIPETTAI - 608 502
12 PHYSICS UNIT - 1 ELECTROSTATICS COMPLETE STUDY MATERIAL
6. Ordinary rubber is an insulator. But the special rubber tyres of aircrafts are
UNIT - 1 ELECTROSTATICS CONCEPTUAL QUESTIONS & ANSWERS made slightly conducting. Why is this necessary?
1. When glass rod is rubbed with silk, both acquire charges. What is the source  During landing, the tyres of aircraft may get highly charged due to friction
of their electrification between tyres and the air strip.
 Before rubbing both objects are electrically neutral.  If the tyres are non - conducting, the accumulation of static electricity may
 For the electrification of a body, only electrons are responsible produce spark and result in fire.
 During rubbing glass rod with silk, some electrons are transferred from glass to  Since the tyres are made slightly conduting, they will lose the charge to the
silk and hence glass acquires positive charge (deficit of electrons) and silk earth and the landing is quite safe.
acquires negative charge (excess of electrons) 7. Vehicle carrying inflammable material usually have a metallic chains or ropes
 So both acquire charges due to frictional electrification. touching the ground during motion. Why?
2. Two identical metallic spheres of exactly equal masses are taken. One is given  When vehicle moves through air, they get charged due to friction. Charge will
a positive charge ‘q’ coulomb and other and equal negative charge. Are their be developed on the tyres also.
masses after charging are equal?  If the charge is sufficient, they will produce spark. The vapour which escapes
 No from the inflammable materials carried by the vehicle may catch fire.
 The positive charge of a body is due to deficit of electrons while the negative  To prevent this, metallic ropes touching the ground was suspended. Through
charge is due to surplus of electorns. this conducting metal, the charge flows to the earth.
 Hence the mass of negatively charged sphere will be slightly more than that of 8. An inflated balloon is charged by rubbing with fur. Will it stick readily to a
the positively charged sphere. conducting wall or to an insulating wall? Give reason.
3. Can two conductor having like charges on them attract each other? If yes, how?  When the negatively charged inflated balloon brought near to a wall, it will
 Yes . They can do so if the charge on one is much larger than that on the other. induces opposite type of charges on the wall and due to force of attraction it
 In that case, the charge of opposite nature is induced on second body is much will stick on the wall.
larger. This induced charge produces a resultant charge of opposite nature  It will stick readily to the conducting wall, since it induces equal amount of
after neutralizing the original charge on second body. opposite charge on the conducting wall and much snaller charge on insulating
 After neutralization the two bodies carry ulike charges and hence attracts each wall.
other.  So a large force of attraction acts between the balloon and the conducting wall.
4. An ebonite rod held in hand can be charged by rubbing with flannel, but a 9. Is it possible to produce high voltage on our body without getting a shock?
copper rod cannot be charged like this. Why?  Yes, possible. If we stand on an insulating surface or wearing shoes with
 Ebonite rod is insulating material. So whatever charge appears on it due to insulating soles and touch the live wire of high power supply, a high potential
rubbing stays on it. (high voltage) is developed on our body without causing any shock.
 On the other hand, copper is a good conductor. Any charge developed on it  Because potential difference between the body and the live wire is zero and
flows to the earth through our body which is also a conductor. So copper rod hence no current will flows through our body.
cannot be charged by holding it by our hand. 10. What does 𝒒𝟏 + 𝒒𝟐 = 𝟎 signify in electrostatics? or Two charges 𝒒𝟏 and 𝒒𝟐
5. A comb run through one’s dry hair attracts small bits of paper. Why? What separated by a small distance satisfy the equation 𝒒𝟏 + 𝒒𝟐 = 𝟎 . What does it
happens if the hair is wet or if it is a rainy day? tell about the charges?
 When the comb runs through dry hair, it gets charged (electrification) by friction  This equation signifies that the additive nature of charges.
 When the charged comb is brought near to the bits of paper, opposite nature of  Here the given two charges are equal in magnitude but opposite in nature.
charges are induced on the of paper, resulting in a net force of attraction. 11. Can a body have a charge of 𝟎. 𝟖 𝑿 𝟏𝟎−𝟏𝟗 𝑪 . Justify your answer by comment.
 Wet hair is a conductor or if it is a rainy day the humid air is more conductive  According to quantisation of charge, in nature the charge on any body is always
then dry air. So the charge goes out of comb and hence it will not attracts paper an integral multiple of a basic charge or electronic charge ‘e’ . (i.e.) 𝑞 = 𝑛 𝑒
bits.
victory R. SARAVANAN. M.Sc., M.Phil., B.Ed PG ASST [PHYSICS], GBHSS, PARANGIPETTAI - 608 502
12 PHYSICS UNIT - 1 ELECTROSTATICS COMPLETE STUDY MATERIAL
𝑞 0.8 𝑋 10−19 1 17. Two electrically charged particles having charges of different magnitude,
 Here, 𝑛 = = = = 0.5 = 𝑛𝑜𝑡 𝑎𝑛 𝑖𝑛𝑡𝑒𝑔𝑒𝑟
𝑒 1.6 𝑋 10−19 2 when placed at a distance ‘d’ from each other, experience a force of attraction
 So a body cannot have a charge 𝟎. 𝟖 𝑿 𝟏𝟎−𝟏𝟗 𝑪 ‘F’. These two particles are put in contact and again placed at the same
12. If the distance between two equal point charges is doubled and their individual distance from each other. What is the nature of new force between them? Is
charges are also doubled, what would happen to the force between them? the magnitude of the force on interaction between them now more or less
 According to Coulomb’s law, the magnitude of electrostic force is than F ?
1 𝑞𝑋𝑞  When the two charged particles are put in contact, they share the difference of
𝐹= − − − − − − − − − − − − − (1)
4 𝜋 𝜀𝑜 𝑟 2 charge identically and they become like chaged particles.
 Now q → 2q and r → 2r , then the force becomes,  Since their magnitude of charges decreases after contact, the two particles
1 2𝑞𝑋2𝑞 1 4 𝑋 𝑞 𝑋𝑞 1 𝑞𝑋𝑞
𝐹𝟏 = = = =𝐹 repels each other with a force less than ‘F’
4 𝜋 𝜀𝑜 (2𝑟)2 4 𝜋 𝜀𝑜 4 𝑋 𝑟 2 4 𝜋 𝜀𝑜 𝑟 2 18. What is the need of introducing the concept of electric field?
 Hence the force will remains the same.  Let a point chage is kept at a point in space. If another point charge is placed at
13. How does the force between two point charges change, if the dielectric some distance, it experience either attractive or repulsive force.
constant (relative permitivity) of the medium in which they are kept  Here how does the second charge know about the existance of the first charge
increases? Give an example. which is located at some distance away from it. To answer it Michael Farady
 According to Coulomb’s law, the magnitude of electrostic force in vacuum is introduced the ‘concept of field’.
1 𝑞1 𝑞2
𝐹=  According to Faraday, every charge in the universe creates an electric field in
4 𝜋 𝜀𝑜 𝑟 2
the surrounding space and if another charge is brought into its field, it will
 The magnitude of electrostic force in a medium of permittivity ‘𝜀 ’ is
interact with the electric field at that point will experience a force.
1 𝑞1 𝑞2 1 𝑞1 𝑞2 𝐹
𝐹𝑚 = 2
= 2
=  Simillarly the interaction of two massed can be explained by the conept of
4𝜋𝜀 𝑟 4 𝜋 𝜀𝑜 𝜀𝑟 𝑟 𝜀𝑟
gravitational field.
 So if the dielectric constant ‘𝜀𝑟 ′ of the medium increases, the force between the
 Thus field concept is required to explain and determine the force on a charge
two charges decreases.
or mass
Example :
19. An electron and a proton are kept in the same electric field. Will they
 The dielectric constant (relative permittivity) of water is 𝜀𝑟 = 80 . So the
experience same force and have same acceleration?
strength of the force between the two charges in water is reduced by 80 times
 Both electron and proton have same amount of charge and hence both will
compared to the force between the same two charges in vacuum.
experience force of same magnitude (i.e.) 𝑭 = 𝒆 𝑬 , when they placed in same
14. Water acts as a good solvent. Why?
electric field (E)
 The dielectric constant (relative permittivity) of water is 𝜀𝑟 = 80 . When
 But the mass of the proton is 1836 times that of an electron (i.e.) 𝑚𝑝 = 1836 𝑚𝑒
common salt (NaCl) is taken in water, the electrostatic force of attraction
Hence acceleration experienced by the proton is.
between Na+ and Cl- ions is reduced by 80 times due to the high relative
𝐹 𝐹 1
permittivity of water. This is the reason of water acts as a good solvent. 𝑎𝑝 = = = 𝑎
𝑚𝑝 1836 𝑚𝑒 1836 𝑒
15. The dilelectric constant of water is 80. What is its permittivity? 1
 Permitivity of the medium,  So proton acceleration will be times that of the electron.
1836
𝜀 = 𝜀𝑜 𝜀𝑟 = 8.854 𝑋 10−12 𝑋 80 = 7.083 𝑋 10−19 𝐶 2 𝑁 −1 𝑚−2 20. The electric line of force tend to contract lengthwise and expand laterally.
16. Give an example to illustrate that electrostatic forces are much stronger than What do they indicate?
gravitational forces.  The length wise contraction indicates attraction between unlike charges, while
 A charged glass rod or a charged rubber can lift a piece of paper against the lateral expansion indicates repulsion between like charges.
gravitational pull of the earth on this piece. This shows that the electrostatic
force on the piece of paper is much greater than the gravitational force on it.

victory R. SARAVANAN. M.Sc., M.Phil., B.Ed PG ASST [PHYSICS], GBHSS, PARANGIPETTAI - 608 502
12 PHYSICS UNIT - 1 ELECTROSTATICS COMPLETE STUDY MATERIAL
21. A point charge placed at any point on the axis of an electric dipole at some  If the pairs (2 - 3) is positively charged, then the pair (4 - 5) is negatively
large distance experiences a force ‘F’. What will be the force acting on the charged or vice versa.
point charge when its distance from the dipole is doubled?  Here the ball 1 is attracted both by ball 2 and 4 separately (i.e.) by both kind of
 The electric field at a point on the axis of the dipole at a distance ‘r’ from its mid charges. Hence the ball 1 is neutral
1 2𝑝 26. Distinguish between electric charge and mass.
point is, 𝐸=
4 𝜋 𝜀𝑜 𝑟 3
 Electric charge can be positive, negative or zero
 If a charge ‘q’ is placed at that point, it experience a force, But mass is always positive
1 2𝑝
𝐹=𝐸𝑞= 𝑞  Electric charge is always quantized.
4 𝜋 𝜀𝑜 𝑟 3
But quantisation of mass is yet not established.
 When 𝑟 → 2 𝑟 , the new force becomes,
 Charge on a body does not change with its speed.
1 2𝑝 1 2𝑝 𝐹
𝐹1 = 𝑞 = 𝑞= Mass of a body increases with its speed.
4 𝜋 𝜀𝑜 (2 𝑟)3 4 𝜋 𝜀𝑜 8 𝑟 3 8
𝐹
 Electric charge is always conserved.
 (i.e.) when the distance of the point charge is doubled, the force reduces to Mass is not conserved by itself as it can be changed in to energe and vice versa
8
22. A thin spherical shell carries a charge ‘Q’ on its surface. A point charge ‘Q/2’ is 27. A spherical rubber balloon carries a charge that is uniformly distributed over
placed at its centre ‘O’ and another charge ‘2Q’ is placed outside. If all the its surface. As the balloon is blown up, how does ‘E’ vary for point (i) inside the
charges are positive, what will be the force on the charge at the centre? balloon, (ii) on the surface of the balloon and (iii) outside the balloon
 Force on the charge at the centre is Zero.  According to Gauss theorem,
 Because, electric field inside a charged spherical shell is zero. So no force act on (i) For points inside the charged balloon (shell) is zero. (i.e.) 𝐸 = 0
the charge place inside the shell. (ii) As the balloon up, its surface area (A) increases and hence surface charge
𝑞 𝜎
23. What is the number of electric lines of force that radiate outwards from 1 C of density (𝜎 = ) decreases. So the field (𝐸 = ) on its surface decreases.
𝐴 𝜀𝑜
charge in vacuum? 1 𝑞
(iii) For points outside the balloon, the field is 𝐸 = . As the balloon
 According to Gauss theorem, when a charge ‘q’ is surrounded by a closed surface, 4 𝜋 𝜀𝑜 𝑟 2
then the total electric flux through the closed surface (i.e.) total number of blown up, the charge enclosed by the Gaussian surface remains same. So E
𝑄𝑖𝑛𝑠𝑖𝑑𝑒 does not change.
electric lines of force radiated outwards from the charge ‘q’ is, Φ𝐸 =
𝜀𝑜
28. Wht is meant by the statement that the electric field of a point charge has
 If 𝑄𝑖𝑛𝑠𝑖𝑑𝑒 = 1 𝐶 , then number of lines of force that radiate outwards from ‘q’ is
spherical symmentry, whereas that of an electric dipole is cylindrically
1
Φ𝐸 = = 4 𝜋 𝑋 9 𝑋 109 = 𝟏. 𝟏𝟑 𝑿 𝟏𝟎𝟏𝟏 symmentric?
𝜀𝑜 1 𝑞
24. An arbitrary surface encloses a dipole. What is the electric flux through this  ⃗ =
The electric field due to a point charge ‘q’ at a distance ‘r’ is 𝐸 𝑟̂ .
4 𝜋 𝜀𝑜 𝑟 2
surface? ⃗ will be same at all points on the surface of a sphere of
Clearly the magnitude of 𝐸
 Two equal and unlike charges separated by a small distance constitute a dipole.
radius ‘r’ drawn around the point charge and not depends on the direction of 𝑟̂
 So the total charge of a dipole is zero. By Gauss theorem, the electric flux
Hence the field due to a point charge is spherically symmentric.
through the closed surface is zero . (i.e.) Φ𝐸 = 0
 Electric field at a distance ‘r’ on the equatorial line of an electric dipole of dipole
25. Five balls numbered 1 to 5 are suspended using separate threads. Pairs 1 𝑝
(1 - 2), (2 - 4) and (4 - 1) shows electrostatic attraction, while pairs (2 - 3) and
⃗ =−
moment ‘p’ is given by 𝐸 𝑝̂ . The electric field ‘E’ is same
4 𝜋 𝜀𝑜 (𝑟 2 + 𝑎2 )3/2
(4 - 5) shows repulsion. What is the value of charge on ball 1? in all points which lie on a cylinder of radius ‘r’, with its axis on the dipole axis
 Repulsion is the surest test of electrification of the object. So balls 2, 3, 4 and 5 and the field pattern looks same in all planes passing through the dipole axis.
are electrically charged. Hence the field on an electric dipole is cylindrically symmentric.
 Here pairs (2 - 3) and (4 - 5) are similarly charged.
 As pair (2 - 4) shows attraction, the balls 2 and 4 are oppositely charged.

victory R. SARAVANAN. M.Sc., M.Phil., B.Ed PG ASST [PHYSICS], GBHSS, PARANGIPETTAI - 608 502
12 PHYSICS UNIT - 1 ELECTROSTATICS COMPLETE STUDY MATERIAL
29. No two lines of force intersect each other. Why? 35. ‘n’ small drops of same size are charged to ‘V’ volt each. If they coalesce to
 The tangent to the line of force gives the direction of electric field. form a single large drop, what will be the potential of large drop?
 If the two lines of force intersect, at the point of intersection the resultant  Let ‘q’ be the charge on each small drop of radius ‘r’. The electric potential of
electric intensity will have two different directions, which is impossible. Hence 1 𝑞
each drop is 𝑉 =
4 𝜋 𝜀𝑜 𝑟
two lines of force never intersect each other.
 Although ‘n’ drops coalesce together to form a single drop of radius ‘R’ , the
30. A bird perches on a bare high - power line and nothing happens to the bird. A
volume of sigle large drop is equal to sum of the volume of ‘n’ drops. Hence
man standing on the ground touches the same line and gets a fatal shock.
4 4
Why? 𝜋 𝑅3 = 𝑛 𝜋 𝑟3 (𝑜𝑟) 𝑅3 = 𝑛 𝑟 3 (𝑜𝑟) 𝑅 = 𝑛1/3 𝑟
3 3
 When a bird perches on the high power line, its body is at the same voltage as  The total charge on large drop (by additive law), 𝑄 = 𝑛 𝑞
the live wire. Since there is no potential difference between bird and wire, no  Then the potential of the large drop is,
current passes through the bird. So nothing will happen and the bird feel free. 1 𝑄 1 𝑛𝑞 1 𝑞
 When a man standing on earth touchs the same line, large potential diffence 𝑉1 = = = 𝑛2/3 [ ] = 𝑛2/3 𝑉
4 𝜋 𝜀𝑜 𝑅 4 𝜋 𝜀𝑜 𝑛1/3 𝑟 4 𝜋 𝜀𝑜 𝑟
developed between his hand and feet. So large current flows through his body
36. What is the difference between potential at a point and potential energy of a
and hence he gets a fatal shock.
charge at that point?
 But if the man wears shoes with insulating soles or standing in isulating surface,
 Electric potential at any point in an electric fleld is defined as the amount of
he will not get a shock.
work done in moving a unit positive charge from infinity to that point against
31. Is the electrostatic potential necessarily zero at a point where the electric field
the field. Its unit is volt (V) or joule/coulomb (J/C)
strength is zero. Give an example to illustrate your answer.
 But Electric potential energy at that point is defined as the amount of work done
 No
in moving the charge from infinity to that point against the field. Its unt is joule (J)
 For example, the electric field intensity at a point mid way between two equal
37. What is the amount of work done in displacing a charge of 1 C through a
like charges is zero. But the electrostatic potential at same point is twice that
distance of 1 m along the equipotential surface of potential ‘V’
due to a single charge.
 Since there is no potential difference between any two points of an equipotential
32. Is electrostatic potential is scalar or a vector quantity? Give its dimensional
surface, no amount of work is done in displacing any amount of charge through
formula?
any distance along the equipotential surface. (i.e.) 𝑊 = 𝑞 (𝑉 − 𝑉) = 0
 Since the electrostatic potential is work done per unit charge, it is a scalar
38. What is electrostatic shielding?
quantity.
 When a solid or hollow conductor is placed in an electric field, electric field
 The dimensional formula for electrostatic potential is,
intensiy inside is zero. Therefore any charge placed inside it does not
𝑊 𝐹𝑆 [𝑀 𝐿 𝑇 −2 ] [ 𝐿 ]
𝑉= = = = [𝑴 𝑳𝟐 𝑻−𝟑 𝑨−𝟏 ] experience a force and hence is said to be shielded from the electric field. This
𝑞 𝐼𝑡 [𝐴][𝑇]
phenomenon is called electrostatic shielding.
33. How can you transfer the whole of the charge on a conductor to another one?
39. What is the dielectric constant of a good conductor?
 If we place the given charged conductor inside a hollow conductor and connect
 Infinite.
the two together, whole of the charge will be transferred to the hollow
 Dielectric constant of a medium is defined as the ratio between electrice field in
conductor, since the charge tends to reside on the outer surface.
vacuum to the reduced electric field inside the medium.
34. Can two equipotential surfaces intersect each other?
 Since the field inside a conductor is zero, the dielectric constant of the
 Direction of lines of forces is always perpendicular to the equipotential surface.
conductor is infinite.
If the two equipotential surfaces intersect each other, we can draw two
40. The two charged conductors are touched mutully and then separated. What
perpendiculars at the common point of intersection indicating the possibility of
will be the charge on them?
two directions of electric field intensity, which is obviously not possible. Hence
 When the charged conductors touch each other, there occur distribution of
no two equipotential surfaces can intersect each other.
charges between them until they acquire same potential.

victory R. SARAVANAN. M.Sc., M.Phil., B.Ed PG ASST [PHYSICS], GBHSS, PARANGIPETTAI - 608 502
12 PHYSICS UNIT - 1 ELECTROSTATICS COMPLETE STUDY MATERIAL
 We know that , 𝑞 = 𝐶 𝑉 46. A battery is connected to a capacitor and then a dielectric slab is kept between
 Since V is constant, ‘q’ is directly proportional to ‘C’. So the charge on them will the plates. What is the change in the charge, the capacitance, the potential
be divided in the ratio of their capacitances. difference, the electric field and the stored energy?
 Dielectric is inserted when battery is connected, then the potential difference
41. The plates of charged capacitors are connected to a voltmeter. If the plates of
(𝑉𝑜 ) is constant and hence electric field intensity (𝐸𝑜 ) remains constant. But
the capacitors are separated further, what will be the effect on the reading of (i) Charge increases : 𝑄 = 𝜀𝑟 𝑄𝑜
the voltmeter? 𝑄 𝜀 𝑄
𝑞 𝜀𝑜 𝐴
(ii) Capacitance increases : 𝐶= = 𝑟 𝑜 = 𝜀𝑟 𝐶𝑜
𝑉𝑜 𝑉𝑜
 The capacitance of parallel plate capacitor is, 𝐶 = = 1 1
𝑉 𝑑
(iii) Stored energy also increases : 𝑈 = 2 𝐶 𝑉𝑜2 = 2 𝜀𝑟 𝐶𝑜 𝑉𝑜2 = 𝜀𝑟 𝑈𝑜
 As the separation between the plates (d) increases, capacitance (C) decreases
 Dielectric is inserted when the battery is disconnected, then the charge remains
 Since the charge (q) remains the same, value of voltage (V) increases. Hence the
constant (𝑄𝑜 ) . But
reading of the voltmeter will increase. 𝐸
(i) Electric field intensity decreases : 𝐸 = 𝑜
42. How does a spark discharge occur between two charged objects? 𝜀𝑟
𝐸𝑜 𝑉𝑜
 The air between the two charged objects is subjected to an electric field. (ii) Potential difference decreases : 𝑉=𝐸𝑑= 𝑑=
𝜀𝑟 𝜀𝑟
 If this electric field (potential gradient) in the intervening air column becomes 𝑄𝑜 𝜀 𝑄
(iii) Capacitance increases : 𝐶=
𝑉
= 𝑟𝑉 𝑜 = 𝜀𝑟 𝐶𝑜
high enough, the air is ionized and conducting path is formed for free electrons 𝑜
1 𝑄𝑜2 1 𝑄𝑜2 𝑈
which move across to discharge the surfaces. (iv) Stored energy decreases : 𝑈=
2 𝐶
= 2 𝜀𝑟 𝐶𝑜
= 𝜀𝑜
𝑟
 Stored electric potential energy is dissipated as heat, light and sound 47. Which is safe to remain inside the car or under tree during lighting?
43. Given solid metal sphere and a hollow metal sphere. Which will hold more  During lightning accompanied by an thunderstorm, it is always safer to sit
charge? Here both spheres are of same radius. inside a car or house than in open groun or under a tree.
 Both of the spheres will hold the same charge.  The metal body of the car or walls of the house provides electrostatic shielding
 It is because charge remains on the outer surface of a charged conductor since the electric field inside is zero
 Duing lightning, the charges flow through the body of the conductor or through
(whether solid or hollow) and the spheres have equal surface areas.
the walls of the house to the ground with no effect on the person inside the car
44. Two spheres of different capacitances are charged to different potentials. or house.
They are then joined by a wire. Will total energy increase, decrease or remains 48. Sometimes we notice that the ceiling fan does not start rotating as soon as it is
the same? switched on. But when we rotate the blades, it starts to rotate as usual. Why it
 The two spheres are at different potentials. Therefore , when they are is so?
connected by a wire, there will be redistribution of charge (i.e.) flow of charge  We know that to rotate any object, there must be a torque applied on the object.
For the ceiling fan, the initial torque is given by the capacitor widely known as a
through wire till the two sphrers attain the same potential
condenser. If the condenser is faulty, it will not give sufficient initial torque to
 Due to the flow of charge through the connecting wire, some energy will be lost rotate the blades when the fan is switched on.
as heat. 49. What principle is used in computer key board?
 Hence the total energy after connecting the spheres will decrease.  Computer keyboard keys are constructed using capacitors with a dielectric
45. Gauss theorem cannot be applied to find the electric field of a charged disc  When the key is pressed, the separation between the plates decreases leading to
and an electric dipole. Why? an increase in the capacitance. This in turn triggers the electronic circuits in the
 Gauss’law can applied to find the electric field of symmetric charge distributions computer to identify which key is pressed.
only. 50. A charged conical conductor loses its charge earlier than a similarly charged
 An electric dipole and a charged disc do not have sufficient symmetry. In these sphere. Why?
cases there is no simple surface over which normal component of ‘E’ is constant.  Surface charge density (charge per unit area) is inversely proportional to the
So Gauss law cannot be applied. radius of curvature of the charged conductor.
 So the surface charge density is high at pointed end and small at surface of large
radii. Due to action of points (corono discharge), the charge escapes into air.

victory R. SARAVANAN. M.Sc., M.Phil., B.Ed PG ASST [PHYSICS], GBHSS, PARANGIPETTAI - 608 502
12 PHYSICS UNIT - 1 ELECTROSTATICS COMPLETE STUDY MATERIAL
NAME : EXAM NO 7. Two identical conducting balls having positive charges q1 and q2 are
separated by a centre to centre distance r. If they are made to touch each
Time - 2 : 30 hours UNIT - 1 ELECTOSTATICS Total - 60 marks other and then separated to the same distance,the force between them will be
(a) less than before (b) same as before
PART - I 10 X 1 = 10
(c) more than before (d) zero
Note : (i) Answer all the questions 8. Rank the electrostatic potential energies for the given system of charges in
(ii) Choose the best answer and write the option code and increasing order.
corresponding answer
1. Two identical point charges of magnitude –q are fixed as shown in the figure
below. A third charge +q is placed midway between the two charges at the
point P. Suppose this charge +q is displaced (a) 1 = 4 < 2 < 3 (b) 2 = 4 < 3 < 1 (c) 2 = 3 < 1 < 4 (d) 3 < 1 < 2 < 4
a small distance from the point P in the 9. An electric field 𝑬⃗ = 𝟏𝟎 𝒙 𝒊̂ exists in a certain region of space. Then the
directions indicated by the arrows, in which potential difference V = Vo – VA, (where Vo is the potential at the origin and
direction(s) will +q be stable with respect VA is the potential at x = 2 m ) is
to the displacement? (a) 10 V (b) –20 V (c) +20 V (d) –10 V
(a) A1 and A2 (b) B1 and B2 (c) both directions (d) 10. A thin conducting spherical shell of radius R has a charge Q which is
No stable uniformly distributed on its surface. The correct plot for electrostatic
2. Which charge configuration produces a uniform electric field? potential due to this spherical shell is
(a) point charge (b) uniformly charged infinite line
(c) uniformly charged infinite plane (d) uniformly charged spherical shell
𝒒
3. What is the ratio of the charges | 𝟏 | for the
𝒒𝟐
(a) (b) (c) (d)
following electric field line pattern? 11. Two points A and B are maintained at a potential of 7 V and - 4 V
1 25
(a) (b) respectively. The work done in moving 50 electrons from A to B is
5 11
11 (a) 8.80 𝑋 10−17 𝐽 (b) − 8.80 𝑋 10−17 𝐽
(c) 5 (d) −17
25 (c) 4.40 𝑋 10 𝐽 (d) 5.80 𝑋 10−17 𝐽
4. An electric dipole is placed at an alignment angle of 30 o with an electric 12. If voltage applied on a capacitor is increased from V to 2V, choose the
field of 2 × 105 NC–1. It experiences a torque equal to 8 N m. The charge on correct conclusion.
the dipole if the dipole length is 1 cm is (a) Q remains the same, C is doubled (b) Q is doubled, C doubled
(a) 4 mC (b) 8 mC (c) C remains same, Q doubled (d) Both Q and C remain same
(c) 5 mC (d) 7 mC 13. A parallel plate capacitor stores a charge Q at a voltage V. Suppose the area
5. Four Gaussian surfaces are given below with charges inside of the parallel plate capacitor and the distance between the plates are each
each Gaussian surface. Rank the electric flux through each doubled then which is the quantity that will change?
Gaussian surface in increasing order. (a) Capacitance (b) Charge
(a) D < C < B < A (b) A < B = C < D (c) Voltage (d) Energy density
(c) C < A = B < D (d) D > C > B > A 14. Three capacitors are connected in triangle as shown in the
figure. The equivalent capacitance between the points A
6. The total electric flux for the following closed surface which is kept inside and C is
water (a) 1 𝜇 𝐹 (b) 2 𝜇 𝐹
80 𝑞 𝑞 1
(a) (b) (c) 3 𝜇 𝐹 (d)
4
𝜇𝐹
𝜀0 40 𝜀0
𝑞 𝑞 15. Two metallic spheres of radii 1 cm and 3 cm are given charges of –1 × 10–2 C
(c) (d) and 5 × 10–2 C respectively. If these are connected by a conducting wire, the
80 𝜀0 160 𝜀0
final charge on the bigger sphere is
(a) 3 × 10–2 C (b) 4 × 10–2 C (c) 1 × 10–2 C (d) 2 × 10–2 C

victory R. SARAVANAN. M.Sc., M.Phil., B.Ed PG ASST [PHYSICS], GBHSS, PARANGIPETTAI - 608 502
12 PHYSICS UNIT - 1 ELECTROSTATICS COMPLETE STUDY MATERIAL
PART - II 6 X 2 = 12
Note : (i) Answer any 6 of the following questions .
(ii) Question No. 23 is compulsory
16. State Coulomb’s law in electrostatics
17. Distinguish between Coulomb force and Gravitational force.
18. Define electric dipole moment. Give its unit.
19. Distinguish between potential difference and electric potential
20. State Gauss law.
21. What are called polar molecules? Give examples.
22. Define action of point or corona discharge.
23. A sample of HCl gas is placed in a uniform electric field of magnitude 3 × 104 N C–1. கல்விக்கு அழகு கசடற ம ொழிதல்
The dipole moment of each HCl molecule is 3.4 × 10–30 Cm. Calculate the maximum - குற்றமின்றி பபசுதபே கற்ற கல்விக்கு அழகு
torque experienced by each HCl molecule.
கற்கக நன்பற கற்கக நன்பற, ஐயம் புகினும் கற்கக நன்பற -
PART - III 6 X 3 = 18
பிச்கச எடுத்தொவது கல்வி கற்பது நல்ேது
Note : (i) Answer any 5 of the following questions .
(ii) Question No. 24 is compulsory
கல்ேொ ஒருவன் குேநேம் பபசுதல் மநல்லினுள் பிறந்த பதர்
24. Discuss the basic properties of electric charge.
25. List the properties of electric lines of force.
ஆகுப -
26. Derive an expression for torque experienced by an electric dipole placed in the கல்வியறிவில்ேொதவன் தன் குேப்மபருக பபசுவது மநல்லுக்கு
uniform electric field. நடுபவ பதொன்றும் குப்கபச் மசடிகயப் பபொன்றது
27. Derive an expression for capacitance of parallel plate capacitor.
28. Give the applications and disadvantage of capacitors எக்குடி பிறப்பினும் யொவபே ஆயினும் அக்குடியிற் கற்பறொகே
29. Write a note on microwave oven. ப ல்வருக என்பர் -
30. Dielectric strength of air is 3 × 106 V m–1. Suppose the radius of a hollow sphere கற்றவர்கள் எந்த குடியில் பிறந்தவர்களொக இருந்தொலும், அவர்ககள
in the Van de Graff generator is R = 0.5 m, calculate the maximum potential ற்ற கற்றவர்கள் ப பே வேபவற்று ஏற்றுக்மகொள்வொகள்
difference created by this Van de Graaff generator.
வெற்றி வெற்கை (நறுந்வ ொகை) - அதிவீர ரொம பொண்டியன்
PART - IV 3 X 5 = 15
Note : (i) Answer all the questions
31. Calculate the electric field due to a dipole on its axial line.
(OR)
Obtain an expression for electric field due to an infinitely long charged wire.
32. Derive the expression for resultant capacitance, when capacitors are connected in
series and in parallel.
(OR)
Derive an expression for electro static potential due to electric dipole.
33. Explain in detail the effect of dielectric placed in a parallel plate capacitor when
the capacitor is disconnected from the battery.
(OR)
Explain in detail the construction and working of Van de Graff generator.

victory R. SARAVANAN. M.Sc., M.Phil., B.Ed PG ASST [PHYSICS], GBHSS, PARANGIPETTAI - 608 502
பசித்திரு (Be hungry) தனித்திரு (Be individual) விழித்திரு (Be conscious)

HIGHER SECONDARY SECOND YEAR-PHYSICS

NAME :
STANDARD : 12 SECTION :
SCHOOL :
EXAM NO :

victory R. SARAVANAN. M.Sc, M.Phil, B.Ed.,


PG ASST (PHYSICS)
GBHSS, PARANGIPETTAI - 608 502
12 PHYSICS UNIT - 2 CURRENT ELECTRICITY CONCEPTUAL QUESTIONS & ANSWERS

PART - I MULTIPLE CHOICE QUESTIONS & ANSWERS WITH SOLUTION 5. What is the value of resistance of the following resistor?
(a)100 k Ω (b)10 k Ω
1. The following graph shows current versus voltage values (c) 1k Ω (d)1000 k Ω
of some unknown conductor. What is the resistance of Solution :
this conductor?  From colour code, 0 1 2 3 4 5 6 7 8 9
(a) 2 ohm (b) 4 ohm  𝐵𝑟𝑜𝑤𝑛 → 1 B B R O Y G B V G W
(c) 8 ohm (d)1 ohm 𝐵𝑙𝑎𝑐𝑘 → 0
Solution : 𝑌𝑒𝑙𝑙𝑜𝑤 → 4 (𝑧𝑒𝑟𝑜𝑠)
 From Ohm’s law, 𝑉 = 𝐼 𝑅  Thus the value of resistor = 100000 Ω = 100 𝑘 Ω
𝑉 4
 Hence the resistance of the conductor, 𝑅 = = =2Ω Answer (a) 100 k Ω
𝐼 2
Answer (a) 2 ohm 6. Two wires of A and B with circular cross section are made up of the same
material with equal lengths. Suppose RA = 3 RB, then what is the ratio of radius
2. A wire of resistance 2 ohms per meter is bent to
of wire A to that of B?
form a circle of radius 1m. The equivalent resistance 1 1
between its two diametrically opposite points, A and (a) 3 (b) √3 (c) (d)
√3 3
B as shown in the figure is Solution :
𝜋 𝜌𝑙 𝜌𝑙 1
(a) 𝜋 Ω (b) Ω  The resistance of the material ; 𝑅 = = (or) 𝑅 ∝
2 𝐴 𝜋 𝑟2 𝑟2
𝜋
(c) 2 𝜋 Ω (d) Ω 𝑅𝐴 𝑟𝐵2 𝑟𝐴2 𝑅𝐵 𝑟𝐴 𝑅 𝑅 1
4  Thus, = (or) = (or) =√ 𝐵 = √3 𝑅𝐵 = √3
Solution : 𝑅𝐵 𝑟𝐴2 𝑟𝐵2 𝑅𝐴 𝑟𝐵 𝑅𝐴 𝐵

 Since 𝑟 = 1 𝑚, Length of arc AB = 𝜋 𝑟 = 𝜋 𝑚𝑒𝑡𝑟𝑒 𝟏


Answer (c)
 Hence resistance of the arc AB of length 𝜋 is 2 𝜋 Ω √𝟑
 Here resistances 2 𝜋 and 2 𝜋 are in parallel, their effective resistance will be, 7. A wire connected to a power supply of 230 V has power dissipation P1. Suppose
1 1 1 2 1 the wire is cut into two equal pieces and connected parallel to the same power
= +2𝜋 = = (𝑜𝑟) 𝑹𝑨𝑩 = 𝝅 𝛀 𝑷𝟐
𝑅𝐴𝐵 2𝜋 2𝜋 𝜋 supply. In this case power dissipation is P2 . The ratio is
𝑷𝟏
Answer (a) 𝝅 𝛀 (a)1 (b) 2 (c) 3 (d) 4
3. A toaster operating at 240 V has a resistance of 120 Ω. Its power is Solution :
(a) 400 W (b) 2 W
(c) 480 W (d) 240 W
Solution :
𝑽𝟐 𝟐𝟒𝟎 𝑿 𝟐𝟒𝟎
 Power is given by, 𝑷 = 𝑽 𝑰 = = = 𝟐𝟒𝟎𝑿 𝟐 = 𝟒𝟖𝟎 𝑾
𝑹 𝟏𝟐𝟎
Answer (c) 𝟒𝟖𝟎 𝑾  Let ‘R’ be the resistance of the wire.
4. A carbon resistor of (47 ± 4.7) k Ω to be marked with rings of different colours 𝑹
 If the wire is cut into two equal pieces, then the resistance of each piece will be
for its identification. The colour code sequence will be 𝟐
(a) Yellow – Green – Violet – Gold (b) Yellow – Violet – Orange – Silver  If these two pieces are connected in parallel, their effective resistance will be
1 1 1 2 2 4 𝑅
(c) Violet – Yellow – Orange – Silver (d) Green – Orange – Violet - Gold 𝑅𝑃
= 𝑅/2 + 𝑅/2 = 𝑅 + 𝑅 = 𝑅 (or) 𝑅𝑃 =
4
Solution : 𝑉2 𝑉2 𝑉2 4 𝑉2 𝑷𝟐
 Resistance = 47 𝑘Ω = 47000 Ω ; Tolerance = 4.7 𝑘Ω = 4700 Ω = 10%  Hence, 𝑃1 = and 𝑃2 = = = ∴ =𝟒
𝑅 𝑅𝑃 𝑅/4 𝑅 𝑷𝟏
 10% - Silver 0 1 2 3 4 5 6 7 8 9 Answer (d) 4
 5% - Gold B B R O Y G B V G W
 From the code,
 4 → 𝑦𝑒𝑙𝑙𝑜𝑤, 7 → 𝑣𝑖𝑜𝑙𝑒𝑡, 3 (𝑧𝑒𝑟𝑜𝑠) → 𝑜𝑟𝑎𝑛𝑔𝑒, 10% ⟶ 𝑠𝑖𝑙𝑣𝑒𝑟
Answer (b) Yellow – Violet – Orange – Silver

victory R. SARAVANAN. M.Sc., M.Phil., B.Ed PG ASST [PHYSICS], GBHSS, PARANGIPETTAI - 608 502
12 PHYSICS UNIT - 2 CURRENT ELECTRICITY CONCEPTUAL QUESTIONS & ANSWERS
8. In India electricity is supplied for domestic use at 220 V. It is supplied at 110 V 12. The temperature coefficient of resistance of a wire is 0.00125 per °C. At 20°C,
in USA. If the resistance of a 60W bulb for use in India is R, the resistance of a its resistance is 1 Ω. The resistance of the wire will be 2 Ω at
60W bulb for use in USA will be (a) 800 °C (b) 700 °C (c) 850 °C (d) 820 °C
𝑅 𝑅 Solution :
(a) R ` (b) 2 R (c) (d)
4 2  The change in resistance due to change in temperature will be, Δ𝑅 = 𝑅𝑜 𝛼 Δ𝑡
Solution : (𝑜𝑟) 𝑅 − 𝑅𝑜 = 𝑅𝑜 𝛼 (𝑡 − 𝑡𝑜 ) (or) R = 𝑅𝑜 𝛼 (𝑡 − 𝑡𝑜 ) + 𝑅𝑜
𝑉2 𝑉2
 We know that, Power ; 𝑃 = ∴ Resistance ; 𝑅 = R = 𝑅𝑜 [𝛼 (𝑡 − 𝑡𝑜 ) + 1] (or)
R
= 𝛼 (𝑡 − 𝑡𝑜 ) + 1
𝑅 𝑃 𝑅𝑜
𝑉2 220 𝑋 220 𝑉2 110 𝑋 110 R 2
 Thus, 𝑅 = = and 𝑅𝑈𝑆𝐴 = = (𝑜𝑟) 𝛼 (𝑡 − 𝑡𝑜 ) = 𝑅 − 1 = 1 − 1 = 2 − 1 = 1
𝑃 60 𝑃 60 𝑜
𝑅𝑈𝑆𝐴 110 𝑋 110 60 1 𝑅
 Hence,
𝑅
=
60
𝑋 220 𝑋 220 = 4
∴ 𝑅𝑈𝑆𝐴 = 4 1 1 100000
𝑡 − 𝑡𝑜 = 𝛼 = 0.00125 = 125 = 800
𝑹
Answer (c) 𝑡 = 800 + 𝑡𝑜 = 800 + 20 = 820℃
𝟒
Answer (d) 𝟖𝟐𝟎℃
9. In a large building, there are 15 bulbs of 40 W, 5 bulbs of 100 W, 5 fans of 80 W
and 1 heater of 1k W are connected. The voltage of electric mains is 220 V. The 13. The internal resistance of a 2.1 V cell which gives a current of 0.2 A through a
maximum capacity of the main fuse of the building will be resistance of 10 Ω is
(a) 14 A (b) 8 A (c) 10 A (d) 12 A (a) 0.2 Ω (b) 0.5 Ω (c) 0.8 Ω (d) 1.0 Ω
Solution : Solution :
𝐸−𝑉 𝐸−𝐼 𝑅 2.1−(0.2 𝑋 10) 2.1−2 0.1
 Power of 15 bulbs each of 40 W = 15𝑋 40 = 600 𝑊  Internal resistance ; 𝑟 = = = = = = 0.5 Ω
𝐼 𝐼 0.2 0.2 0.2
Power of 5 bulbs each of 100 W = 5 𝑋 100 = 500 𝑊 0.5 Ω
Answer (b)
Power of 5 fans each of 80 W = 5 𝑋 80 = 400 𝑊
Power of 1 heater of 1 k W = 1 𝑋 1000 = 1000 𝑊 14. A piece of copper and another of germanium are cooled from room
temperature to 80 K. The resistance of
 Thus Total power ; 𝑃𝑡𝑜𝑡 = 2500 𝑊
𝑃𝑡𝑜𝑡 2500 (a) each of them increases
 Hence, 𝑃𝑡𝑜𝑡 = 𝑉 𝐼 (or) 𝐼 = = = 11.36 ≅ 12 𝐴 (b) each of them decreases
𝑉 220
Answer (d) 𝟏𝟐 𝑨 (c) copper increases and germanium decreases
10. There is a current of 1.0 A in the circuit shown (d) copper decreases and germanium increases
below. What is the resistance of P ? Solution :
(a) 1.5 Ω (b) 2.5 Ω  Copper is a conductor for which, 𝑅𝑒𝑠𝑖𝑠𝑡𝑎𝑛𝑐𝑒 ∝ 𝑇𝑒𝑚𝑝𝑒𝑟𝑎𝑡𝑢𝑟𝑒
1
(c) 3.5 Ω (d) 4.5 Ω  Germanium is a semiconductor for which, 𝑅𝑒𝑠𝑖𝑠𝑡𝑎𝑛𝑐𝑒 ∝
𝑇𝑒𝑚𝑝𝑒𝑟𝑎𝑡𝑢𝑟𝑒
Solution : copper decreases and germanium
𝑉 9
 By Ohm’s law ; 𝑉 = 𝐼 𝑅𝑒𝑓𝑓 (or) 𝑅𝑒𝑓𝑓 = = = 9 Ω Answer (d)
𝐼 1 increases
 From the circuit, 𝑅𝑒𝑓𝑓 = 3 + 2.5 + 𝑃 = 5.5 + 𝑃 (𝑜𝑟) 9 = 5.5 + 𝑃
15. In Joule’s heating law, when R and t are constant, if the H is taken along the y
(or) 𝑃 = 9 − 5.5 = 3.5 Ω axis and I2 along the x axis, the graph is
Answer (c) 𝟑. 𝟓 𝛀 (a) straight line (b) parabola (c) circle (d) ellipse
11. What is the current drawn out from the battery? Solution :
(a) 1A ( b) 2A  By Joule’s heating law ; 𝐻 = 𝐼 2 𝑅 𝑡
(c) 3A (d) 4A  Here R and t are constants. So 𝑯 = 𝑪 𝑰𝟐
Solution :  It is in the form of 𝒚 = 𝒎 𝒙 which is theequation of
 The effective resistance ; straight line
1 1 1 1 3
𝑅𝑃
= 15 + 15 + 15 = 15
(or) 𝑅𝑃 = 5 Ω
𝑉 5
 Then by Ohm’s law, 𝐼 = =5 =1𝐴
𝑅𝑃
Answer (a) 𝟏𝑨 Answer (a) straight line

victory R. SARAVANAN. M.Sc., M.Phil., B.Ed PG ASST [PHYSICS], GBHSS, PARANGIPETTAI - 608 502
12 PHYSICS UNIT - 2 CURRENT ELECTRICITY CONCEPTUAL QUESTIONS & ANSWERS
10. Current is a scalar quantity. Why?
PART - II TWO MARK SHORT ANSWER QUESTIONS WITH ANSWERS
 Current is defined as the scalar product of current density ( ⃗⃗⃗⃗𝐽) and area vector
WITH SOLUTION (⃗⃗⃗⃗
𝐴) in which charges crosses. (i.e.) 𝐈 = ⃗𝑱⃗ . ⃗⃗⃗⃗
𝑨 = 𝑱 𝑨 𝒄𝒐𝒔 𝜽
1. Define current electricity.
 The branch of physics deals with moving charges is called current electricity.  The current can be positive or negative depending on the choice of unit vector
2. Define electric current. normal to the surface area A.
 The electric current in a conductor is defined as the rate of flow of charges 11. Give the macroscopic form of Ohm’s law.
through a given cross - sectional area.  Let ‘V’ be the potential difference, ‘I’ be the current and ‘R’ be the resistance,
𝑄 𝑑𝑄 then the macroscopic form of Ohm’s law is V = I R
𝐼= (𝑜𝑟) 𝑖= 12. What are called ohmic and non ohmic materials?
𝑡 𝑑𝑡
 The S. I unit of current is ampere (A). It is a scalar quantity.  Materials for which the current against voltage graph is a straight line through
3. Define one ampere (1 A) the orgin are said to obey Ohm’s law and they are called ohmic materials.
 One ampere of current is equivalent to 1 coulomb of charge passing through a  But materials for which the
perpendicular cross section in 1 second. [𝟏 𝑨 = 𝟏 𝑪 𝒔−𝟏 ] current against voltage graph is
4. What is called conventional current? non - linear and they do not have
 By convention, this flow in the circuit should be from the positive terminal of the a constant resistance are called
battery to the negative terminal. This is called the conventional current or non - ohmic. They do not obey
simply current. Ohm’s law.
 It is in the direction in which a positive test charge would move. 13. Define resistance of the conductor.
5. What are called free electrons and positive ions?  The ratio of potential difference (V) across the given conductor to the current (I)
 Any material is made up of neutral atoms with equal number of electrons and passing through the conductor is called resistance (R).
protons. If the outermost electrons leave the atoms, they become free electrons 𝑽
𝑹=
and are responsible for electric current. 𝑰
 The atoms after losing their outer most electrons will have more positive  Its unit is ohm ( )
charges and hence are called positive ions. They will not move freely and hence 14. What are the factors that the resistance depend on?
the positive ions will not give rise to current.  The resistance of the conductor is,
6. Define drift velocity. 1) directly proportional to its length (l)
 The average velocity acquired by the free electrons inside the conductor when it 2) inversely proportional to its area of cross section (A)
𝒍 𝝆𝒍
is subjected to an electric field is called drift velocity (𝑣⃗𝑑 ). Its unit is 𝒎 𝒔−𝟏 𝑹= =
7. Define mobility. 𝝈𝑨 𝑨
where, 𝝈 → conductivity of the conductor
 The magnitude of drift velocity acquired by the free electrons per unit electric
𝝆 → resistivity of the conductor
field is called mobility (𝜇). Its unit is 𝒎𝟐 𝑽−𝟏 𝒔−𝟏
15. Define resistivity of the material.
8. Define current density.
 The electrical resistivity of a material is defined as the resistance offered to
 Current density (J) is defined as the current per unit area of cross section of the
current flow by a conductor of unit length having unit area of cross section.
conductor. 𝟏 𝑹𝑨
𝐼 𝝆= =
𝐽= 𝝈 𝒍
𝐴  Its unit is ohm - metre ( m )
 It is a vector quantity. Its unit is 𝑨 𝒎−𝟐
 It depends only the type of material and not the dimension of the material.
9. Give the microscopic form of Ohm’s law.
16. Define conductivity of the material.
 The current density is given by,
 The reciprocal of resistivity is called conductivity and it is given by,
𝑒𝜏 𝑛 𝑒2 𝜏 𝟏
⃗𝐽⃗ = 𝑛 𝑒 ⃗⃗⃗𝑣⃗𝑑 = 𝑛 𝑒 [ ⃗⃗⃗⃗
𝐸] = ⃗⃗⃗⃗
𝐸
𝑚 𝑚 𝝈=
⃗𝑱⃗ = 𝝈 ⃗⃗⃗⃗ 𝝆
(𝑜𝑟) 𝑬
 Its unit is mho- metre-1 (-1 m-1)
 Thus Currnt density is directly proportional to the applied electric field. This is
 It depends only the type of material and not the dimension of the material.
known as microscopic form of Ohm’s law.

victory R. SARAVANAN. M.Sc., M.Phil., B.Ed PG ASST [PHYSICS], GBHSS, PARANGIPETTAI - 608 502
12 PHYSICS UNIT - 2 CURRENT ELECTRICITY CONCEPTUAL QUESTIONS & ANSWERS
17. Reparing the electrical connection with the wet skin is always dangerous. 23. What is called electric cell (battery) ?
Why?  An electric cell is a device which converts chemical energy in to electrical energy
 The humam body contains a large amount of water which has low resistance of to produce electricity.
around 200  and the dry skin has high resistance of 500 k .  It contains two electrods (anode and cathode) immersed in an electrolyte.
 But when the skin is wet, the resistance is reduced to 1000  . 24. Define electromotive force.
𝑉
 By Ohm’s law [𝑅 = ] if resistance decreses, current increases. Hence reparing  The amount of work a battery or cell does to move a certain amout of charge
𝐼
around the circuit is called as electromotive force (𝜉). Its unit is volt (V)
electric connection with wet skin is dangerous.
 The emf of a battery or a cell is the voltage provided by the battery when no
18. Define temperature coefficient of resistivity.
current flows in the external circuit.
 It is defined as the ratio of increase in resistivity per degree rise in temperature
25. Define the internal resistance of the cell.
to its resistivity at 𝑇𝑜 . Its unit is 𝒑𝒆𝒓 ℃
 A real battery is made of electrodes and electrolyte.
19. Define Superconductivity.
 There is resistance to the flow of charges within the battery and this resistance is
 The resistance of certain material become zero below certain temperature called
called internal resistance (r)
critical or transition temperature (TC)
 A freshly prepared cell has low internal resistance and it increased with ageing.
 For mercury, TC = 4.2 K
26. State Kirchoff’s first law (current rule or junction rule)
 The materials which exhibit this property are known as super conductors.
 It states that the algebraic sum of currents at any junction in a circuit is zero
 The property of conducting current with zero resistance is called super
(∑ 𝐼 = 0).
conductivity.
 It is discovered by Kammerlingh Onnes.  It is a statement of conservation of electric charge.
20. Distinguish electric energy and electric power. 27. State Kirchoff’s second law (voltage rule or loop rule)
Electric energy Electric power  It states that in a closed circuit the algebraic sum of the products of the current
and reistance of each part of the circuit is equal to the total emf included in the
1) Work has to be done to move the 1) The rate at which the electrical
circuit ( ∑ 𝑰 𝑹 = ∑ 𝜺).
charge from one end to other end potential energy is delivered is
of the conductor and this called electric power.  It is a statement of conservation of energy for an isolated system.
workdone is called electric energy. 𝒅𝑼 28. Give the sign convention followed by the Kirchoff’s current rule.
𝑷= =𝑽𝑰  Current entering the junction is taken as positive and current leaving the
𝒅𝑾 = 𝒅𝑼 = 𝑽 𝒅𝑸 𝒅𝒕
2) Its S.I unit is joule ( J ) 2) Its S.I unit is watt (W) junction is taken as negative.
29. Give the sign convention followed by the Kirchoff’s votage rule.
3) Its practical unit is kilowatt hour 3) Its practical unit is horse power
(kWh) i.e. 1 𝑘𝑊ℎ = 3.6 𝑋 106 𝐽 (H P) i.e, 1 𝐻 𝑃 = 746 𝑊  The product of current and resistance is taken as positive when the direction of
21. Prove that the expression for power in an electrical circuit is 𝑷 = 𝑽 𝑰 the current is followed and is taken as negative when the direction of current is
opposite to the loop
 Electric energy is given by, 𝑑𝑈 = 𝑉 𝑑𝑄
 The emf is considered positive when proceeding from the negative to the
 By definition, the rate at which electric potential energy is delivered is called
positive terminal of the cell and negative when proceeding from the positive to
power. (i.e)
𝑑𝑈 𝑑 (𝑉 𝑑𝑄) 𝑑𝑄 the negative terminal of the cell.
𝑃= = =𝑉 = 𝑽𝑰 30. What is called Galvanometer?
𝑑𝑡 𝑑𝑡 𝑑𝑡
𝒅𝑸  A galvanometer is an instrument used for detecting and measuring even very
 Where, = 𝑰 → electric current small electric currents.
𝒅𝒕
22. Write down the various equations for power.  It is extensively useful to compare the potential difference between various parts
 The electric power is given by, of the circuit.
𝑷=𝑽𝑰 31. State the principle of potentiometer.
 By Ohm’s law, 𝑉 = 𝐼 𝑅 and hence  Let ‘I’ be the current, ′𝑟 ′ be the resistance per unit length and ′𝑙 ′ be the balancing
𝑷 = 𝑰𝟐 𝑹 length, then emf is
 Also, 𝐼 = 𝑉/ 𝑅 and hence, 𝜺= 𝑰𝒓𝒍 (𝒐𝒓) 𝜺∝𝒍
𝑽𝟐  The emf is directly proportional to the balancing length. This is the principle of
𝑷=
𝑹 potentiometer.

victory R. SARAVANAN. M.Sc., M.Phil., B.Ed PG ASST [PHYSICS], GBHSS, PARANGIPETTAI - 608 502
12 PHYSICS UNIT - 2 CURRENT ELECTRICITY CONCEPTUAL QUESTIONS & ANSWERS
32. What is called Joule’s heating effect of current? 39. What are the applications of Seebeck effect?
 When current flows through a resistor, some of the electrical energy delivered to  Seebeck effect is used in thermoelectric generators (Seebeck generators) which
the resistor is converted into heat energy and it is dissipated. This heating effect are used in power plants to convert waste heat into electricity.
of current is known as Joule’s heating effect.  This effect is utilized in automobiles as automotive thermoelectric generators for
33. State Joule’s law of heating. increasing fuel efficiency
 It states that the heat develop in an electrical circuit due to the flow, current  Seebeck effect is used in thermocouples and thermopiles to measure the
varies directly as temperature difference between the two objects.
(i) the square of the current 40. Define Peltier effect.
(ii) the resistance of the circuit and  When an electric current is passed through a circuit of a thermocouple, heat is
(iii) the time of flow evolved at one junction and absorbed at the other junction. This is known as
(𝑖. 𝑒) 𝑯 = 𝑰𝟐 𝑹 𝒕 Peltier effect.
34. What are the properties of the substance used as heating element?  Peltier effect is reversiable.
 An alloy of nickel and chromium called Nicrome is used as heating element. It 41. Define Thomson’s effect.
has  If two points in a conductor are at different temperatures, the density of
(i) a high specific resistance electrons at these points will differ and as a result the potential difference is
(ii) high melting point created between these points.
(iii) heated to very high temperature without oxidation  Thomson effect is reversiable.
35. Write a note on electric fuses.
 Fuses are connected in series in a circuit to protect the electric device from the
heat developed by the passage of excessive current.
 It melt and breaks the circuit if the current exceeds certain value.
 It is a short length of a wire made of a low melting point material.
36. Write a note on circuit breakers (trippers)
 Now a days in housed, circuit breakers are used instead of fuses.
 Whenever there is an ecessive current produced due to faulty wire connection,
the circuit breaker switch opens.
 After repairing the faulty connection, we can close the circuit breaker switch.
37. Write a note on electric bulb or lamp.
 It consists of a tungsten filament kept inside a glass bulb and heated to
incandescence by current.
 Melting point of tungsten is 3380 ℃
 In incandescent electric lamps, only 5% of electric energy is converted into light
and the rest is wasted as heat.
38. Define Seebeck effect.
 In a closed circuit consisting of two dissimilar metals, when the junctions are
maintained at different temperature an emf is developed. This phenomenom is
called Seebeck effect or thermoelectric effect.
 The current that flows due to the emf developed is called thermoelectric current.
 The two dissimilar metals connected to form two junctions is known as
thermocouple.

victory R. SARAVANAN. M.Sc., M.Phil., B.Ed PG ASST [PHYSICS], GBHSS, PARANGIPETTAI - 608 502
12 PHYSICS UNIT - 2 CURRENT ELECTRICITY CONCEPTUAL QUESTIONS & ANSWERS
 The number of electrons available in the volume of length ‘ 𝑑𝑥 ’ is
PART- III THREE MARK SHORT ANSWER QUESTIONS WITH ANSWERS = 𝐴 𝑑𝑥 𝑋 𝑛 = 𝐴 𝑣𝑑 𝑑𝑡 𝑋 𝑛
SOLUTION
1. Obtain an expression for drift velocity. How it is related with the mobility?  Then the total charge in this volume element is,
Drift velocity (⃗⃗⃗⃗
𝒗𝒅 ) : 𝑑𝑄 = 𝐴 𝑣𝑑 𝑑𝑡 𝑛 𝑒
 If there is no electric field, all the free electrons in a conductor are moves in  By definition, the current is given by
random directions. As a result no net flow of electrons in any direction and 𝑑𝑄 𝐴 𝑣𝑑 𝑑𝑡 𝑛 𝑒
𝐼= =
hence there will not be any current. 𝑑𝑡 𝑑𝑡
 If the conductor is subjected toan electric field ( ⃗⃗⃗⃗ 𝐸 ) free electrons experinces a 𝑰 = 𝒏 𝒆 𝑨 𝒗𝒅
force given by, 3. Write a note on carbon resistors.
⃗⃗⃗⃗
𝐹 = −𝑒 ⃗⃗⃗⃗
𝐸 − − − − − (1) Carbon resistors :
 So all the free electrons are accelerated in a direction opposite to the field. By  Carbon resistors consists of a ceramic core on which a thin layer of crystalline
Newton’s second law carbon is deposited.
⃗⃗⃗⃗  They ar inexpensive, stable and compact in size.
𝐹 −𝑒 ⃗⃗⃗⃗
𝐸
⃗⃗⃗⃗
𝑎= = − − − − − −(2)  Colour rings drawn over it are used to indicate the value of the resistance
𝑚 𝑚 according to the rules in the table.
 But the positive ions scatter the electrons and change its direction of motion. So
Colour Number Multiplier Coluur Tolerance
they move in zig-zag path.
Black 0 1 Gold 5%
 In addition to the zig-zag motion due to collisions, the electrons move slowly 1
Brown 1 10 Silver 10 %
along the conductor in a direction opposite to that of ⃗⃗⃗⃗ 𝐸
Red 2 10 2
No ring (colourless) 20 %
 This average velocity acquired by the free electrons inside the conductors, when it
is subjected to the electric field is called drift velocity (⃗⃗⃗𝑣⃗𝑑 ) Orange 3 103
 The average time between successive collision is called the mean free time or Yellow 4 104
relaxation time (𝜏). Green 5 105
 Hence the drift velocity is given by, Blue 6 106
−𝒆 ⃗⃗⃗⃗
𝑬 Violet 7 107
⃗⃗⃗⃗
𝒗𝒅 = ⃗⃗⃗⃗
𝒂𝝉= 𝝉 = − 𝝁 ⃗⃗⃗⃗𝑬 Grey 8 108
𝒎
𝑒𝜏 White 9 109
where , = 𝜇 → mobility of electrons
𝑚  There is three coloured bands on its left and one metallic coloured band on its
 The magnitude of the drift velocity acquired by the free electron per unit electric right side.
field is called mobility.  The first and second rings are the significant figures of the resistance and the
|⃗⃗⃗⃗
𝒗𝒅 | third ring indicate the decimal multiplier after them. The fourth metallic ring
𝝁=
⃗⃗⃗⃗
𝑬 shows the tolerance of the resistor.
 It unit is 𝒎𝟐 𝑽−𝟏 𝒔−𝟏 Example :
2. Derive the relation between the drift velocity and the current.
Drift velocity and current - Relation :  For the given carbon resistor,
First ring (Green) = 5
 Area of cross section of the conductor =𝐴 Second ring (Blue) = 6
Number of electrons per unit volume =𝑛 Third ring (Orange) = 103
Applied electric field = ⃗⃗⃗⃗
𝐸 Fourth metallic ring (Gold) = 5%
Drift velocity of electrons = 𝒗𝒅  Value of the resistor = 𝟓𝟔 𝑿 𝟏𝟎𝟑 𝛀 = 𝟓𝟔 𝐤 𝛀
Charge of an electrons =𝑒 Tolerance =5%
 Let ‘𝑑𝑥’ be the distance travelled by the electron in time ‘𝑑𝑡’, then
𝑑𝑥
𝑣𝑑 = (𝑜𝑟) 𝑑𝑥 = 𝑣𝑑 𝑑𝑡
𝑑𝑡

victory R. SARAVANAN. M.Sc., M.Phil., B.Ed PG ASST [PHYSICS], GBHSS, PARANGIPETTAI - 608 502
12 PHYSICS UNIT - 2 CURRENT ELECTRICITY CONCEPTUAL QUESTIONS & ANSWERS
4. Define temperature coefficient of resistivity. Obtain an expression for it. 6. Write a note on electric cells in parallel.
Temperature coefficient of resistivity : Cells in parallel :
 Resistivity of the substance depends on the temperature. Let  Let ‘n’ cells each of emf 𝜀 and internal
 Resistivity at 𝑇𝑜 ℃ = 𝜌𝑜 resistance ‘r’ are connected in parallel with
Resistivity at 𝑇℃ = 𝜌𝑇 an external resistance ‘R’.
∴ 𝜌𝑇 = 𝜌𝑜 [1 + 𝛼 (𝑇 − 𝑇𝑜 )] − − − −(1)  Total emf of the battery =𝑛𝜀
𝑟
Where, 𝛼 → Temperature coefficient of resistivity Total resistance of the circuit = +𝑅
𝑛
 From equation (1)  By Ohm’s law,
𝜌𝑇 = 𝜌𝑜 + 𝜌𝑜 𝛼 (𝑇 − 𝑇𝑜 ) 𝑇𝑜𝑡𝑎𝑙 𝑒𝑚𝑓 𝜀 𝑛 𝜀
𝜌𝑇 − 𝜌𝑜 = 𝜌𝑜 𝛼 (𝑇 − 𝑇𝑜 ) 𝐼= = 𝑟 = − − − − − (1)
𝝆𝑻 − 𝝆𝒐 ∆𝝆 𝑇𝑜𝑡𝑎𝑙 𝑟𝑒𝑠𝑖𝑠𝑡𝑎𝑛𝑐𝑒 +𝑅 𝑛𝑟+𝑅
∴ 𝜶= = 𝑛
𝝆𝒐 (𝑻 − 𝑻𝒐 ) 𝝆𝒐 ∆𝑻  If 𝑟 ≪ 𝑅, equation (1) becomes,
Where, ∆𝝆 = 𝝆𝑻 − 𝝆𝒐 → change in resistivity 𝑛 𝜀 𝜀
𝐼= ≈ 𝑛 𝐼1 [∵ = 𝐼1 ]
∆𝑻 = 𝑻 − 𝑻𝒐 → Change in temperature 𝑅 𝑅
 It is defined as the ratio of increase in resistivity per degree rise in (i.e.) if ‘r’ is negligible compared to ‘R’ the current supplied by the battery is ‘n’
temperature to its resistivity at 𝑇𝑜 . Its unit is 𝒑𝒆𝒓 ℃ times the that supplied by the single cell
 For conductors 𝜶 is positive (i.e) if the temperature of the conductor increases,  𝑟 ≪ 𝑅, equation (1) becomes,
𝑛 𝜀 𝜀
its resistivity also increases. 𝐼= = ≈ 𝐼1
𝑛𝑟 𝑟
 Thus resistance at 𝑇 ℃ (i.e.) if ‘r’ is very very greater than ‘R’, current due to the whole battery is same
𝑹𝑻 = 𝑹𝒐 [𝟏 + 𝜶 (𝑻 − 𝑻𝒐 )] as due to single cell.
 For semiconductor, 𝜶 is negative. (i.e.) if temperature increases, resistance 7. Explain the principle of
decreases. potentiometer.
 A semiconductor with a negative temperature coefficient of resistance is called a Principle of potentiometer:
thermistor.  A battery (Bt), key (K) and
5. Write a note on electric cells in series. potentiometer wire (CD) are
Cells in series : connected in series forms the
 Let ‘n’ cells each of emf 𝜀 and internal primary circuit.
resistance ‘r’ are connected in series with an  The positive terminal of
external resistance ‘R’. primary cell of emf ′𝜉 ′ is
 Total emf of the battery =𝑛𝜀 connected to the point C and
Total resistance of the circuit = 𝑛𝑟+𝑅 negative terminal is
 By Ohm’s law, connected to the point D
𝑇𝑜𝑡𝑎𝑙 𝑒𝑚𝑓 𝑛𝜀 through galvanometer (G)
𝐼= = − − − (1)
𝑇𝑜𝑡𝑎𝑙 𝑟𝑒𝑠𝑖𝑠𝑡𝑎𝑛𝑐𝑒 𝑛𝑟+𝑅 and high resistance (HR). This
 If 𝑟 ≪ 𝑅, equation (1) becomes, forms the secondary circuit.
𝑛𝜀 𝜀
𝐼= ≈ 𝑛 𝐼1 [∵ = 𝐼1 ]  Let contact be made at ‘J’ on the wire by jockey.
𝑅 𝑅  If the potential difference across CJ is equal to the emf (𝜉) of the cell, then the
(i.e.) if ‘r’ is negligible compared to ‘R’ the current supplied by the battery is ‘n’
galvanometer shows zero deflection. Here ‘CJ’ is the balancing length 𝒍
times the that supplied by the single cell
 If ‘r’ is the resistance per unit length of the wire, then by Ohm’s law,
 𝑟 ≪ 𝑅, equation (1) becomes,
𝑛𝜀 𝜀 Potential difference across CJ = 𝐼 𝑟 𝑙
𝐼= = ≈ 𝐼1 Hence, 𝜀 = 𝑰 𝒓 𝒍
𝑛𝑟 𝑟
(i.e.) if ‘r’ is very very greater than ‘R’, current due to the whole battery is same  Since I and r are constants, , 𝜀 ∝ 𝒍
as due to single cell.

victory R. SARAVANAN. M.Sc., M.Phil., B.Ed PG ASST [PHYSICS], GBHSS, PARANGIPETTAI - 608 502
12 PHYSICS UNIT - 2 CURRENT ELECTRICITY CONCEPTUAL QUESTIONS & ANSWERS
8. Explain Seebeck effect. Give its applications. 10. Distinguish between Peltier effect and Joule’s effect.
Seebeck effect : Peltier effect Joule’s effect
 Seebeck discoved that in a 1) Both heat liberated and absorbed 1) Heat liberated only occur
closed circuit consisting of two occur
dissimilar metals, when the 2) Occurs at junctions 2) Occurs all along the conductor
juctions are maintained at
different temperatures an emf 3) Reversiable effect 3) Irreversiable effect
(potential difference) is 11. Explain Thomson effect.
developed. This is called Thomson effect :
Seebeck effect.  Thomson showed that, if two points in a
 The current that flows due to the emf developed is called thermoelectric conductor are at different temperatures, the
current. density of electrons at these points will
 The two dissimilar metals connected to form two junctions is known as differ and as a result the potential difference
thermocouple. is created between these points. This is
known as Thomson effect.
 If hot and cold juntions are interchanged, the direction of current also reversed.
Hence Seebeck effect is reversiable.  Thomson effect is reversiable.
 The magnitude of emf developed in thermocouple depends on,  If current passed through copper bar AB
(i) Nature of the metals forming thermocouple which is heated at its mid point C, the point C
(ii) Temperature difference between the junctions will be at higer potential. This indicates that
Applications : the heat is absorbed along AC and evolved
along CB. Thus heat is transferred in the
 Seebeck effect is used in thermoelectric generators (Seebeck generators).
direction of the current. It is called positive
 This effect is utilized in automobiles as automotive thermoelectric generators.
Thomson effect.
 Seebeck effect is used in thermocouples and thermopiles.
(e.g) Ag, Zn. Cd
9. Explain Peltier effect.
Peltier effect :  When the copper bar is replaced by an iron
bar, heat is evolved along CA and absorbed
 When an electric current is passed through a circuit of a thermocouple, heat is
along BC. Thus heat is transferred in the
evolved at one junction and absorbed at the other junction. This is known as
direction opposite to the current. It is called
Peltier effect.
negative Thomson effect.
 (e.g.) Pt, Ni, Co, Hg

 In Cu - Fe thermocouple, the junctions A and B are maintained at the same


temperature.
 Let a current flow through the thermocouple.
 At junction ‘A’, where the current flows from Cu to Fe, heat is absorbed and it
becomes cold.
 At juction ‘B’, where the current flows from Fe to Cu, heat is liberated and it
becomes hot.
 When the direction current is reversed, junction ‘A’ becomes hot and junction ‘B’
becomes cold. Hence peltier effect is reversiable.
victory R. SARAVANAN. M.Sc., M.Phil., B.Ed PG ASST [PHYSICS], GBHSS, PARANGIPETTAI - 608 502
12 PHYSICS UNIT - 2 CURRENT ELECTRICITY CONCEPTUAL QUESTIONS & ANSWERS

PART - IV FIVE MARK LONG ANSWER QUESTIONS WITH ANSWERS 2. Obtain the macroscopic form of Ohm’s law from its microscopic form and
discuss its limitation.
1. Describe the microscopic model of current and obtain general form of Ohm’s Macroscopic form of Ohm’s law :
law.
Microscopic model of current and Ohm’ law :

 Consider a segment of wire of length ‘𝑙’ and area of cross section ‘𝐴’.
 Area of cross section of the conductor =𝐴  When a potential difference ‘V’ is applied across the wire, a net electric field is
Number of electrons per unit volume =𝑛 created in the wire which constitutes the current.
 If we assume the electric field is uniform in the entire length, the potential
Applied electric field along leftwads = ⃗⃗⃗⃗
𝐸
Drift velocity of the electrons = 𝒗𝒅 difference is given by,
𝑉
Charge of the electron = 𝒆 𝑉=𝐸𝑙 (𝑜𝑟) 𝐸=
 If ‘𝑑𝑥 ′ be the distance travelled by the electron in time ‘𝑑𝑡’, then 𝑙
𝑑𝑥  From the microscopic form of Ohm’s law,
𝑣𝑑 = (𝑜𝑟) 𝑑𝑥 = 𝑣𝑑 𝑑𝑡 𝑽
𝑑𝑡 𝑱= 𝝈𝑬= 𝝈
 The number of electrons available in the volume of length ‘ 𝑑𝑥 ’ is = 𝒍
 By definition, the current density is
𝐴 𝑑𝑥 𝑋 𝑛 = 𝐴 𝑣𝑑 𝑑𝑡 𝑋 𝑛 𝐈
 Then the total charge in this volume element is, 𝑱=
𝑨
𝑑𝑄 = 𝐴 𝑣𝑑 𝑑𝑡 𝑛 𝑒  Hence,
 By definition, the current is given by I 𝑉
𝑑𝑄 𝐴 𝑣𝑑 𝑑𝑡 𝑛 𝑒 = 𝜎
𝐼= = 𝐴 𝑙
𝑑𝑡 𝑑𝑡 𝑙
 𝑰 = 𝒏 𝒆 𝑨 𝒗𝒅 ∴ 𝑉=I [ ]
𝜎𝐴
Current density (𝑱⃗) : 𝑽 = 𝐈𝑹
 Current density (J) is defined as the current per unit area of cross section of the 𝒍
Where,
𝝈𝑨
= 𝑅 → Resistance of the conductor
conductor.
𝐼 𝑛 𝑒 𝐴 𝑣𝑑  This is called macroscopic form of Ohm’s law.
𝐽= =
𝐴 𝐴
𝑱 = 𝒏 𝒆 𝒗𝒅
 Its unit is 𝑨 𝒎−𝟐
 In vector notation,
⃗𝑱⃗ = 𝒏 𝒆 ⃗⃗⃗⃗⃗
𝒗𝒅
𝒆𝝉 𝒏 𝒆𝟐 𝝉
⃗𝑱⃗ = 𝒏 𝒆 [− ⃗⃗⃗⃗
𝑬] = − ⃗⃗⃗⃗
𝑬
𝒎 𝒎
𝒏 𝒆𝟐 𝝉
 where, = 𝝈 → conductivity Limittations:
𝒎
 From Ohm’s law, the graph between current versus voltage is straight line with a
∴ ⃗𝑱⃗ = − 𝝈 ⃗⃗⃗⃗ 𝑬
slope equal to the inverse of resistance (R) of the conductor.
 But conventionally, we take the dirction of current density as the direction of
 Materials for which the current against voltage graph is a straight line through
electric field. So the above equation becomes,
the origin are said to obey Ohm’s law and their behavior is said to be Ohmic.
⃗𝑱⃗ = 𝝈 ⃗⃗⃗⃗
𝑬  Materials that do not obey Ohm’s law are said to be non - ohmic. These materials
 This is called microscopic form of Ohm’s law. have more complex (non- linear) relationships between voltage and current.

victory R. SARAVANAN. M.Sc., M.Phil., B.Ed PG ASST [PHYSICS], GBHSS, PARANGIPETTAI - 608 502
12 PHYSICS UNIT - 2 CURRENT ELECTRICITY CONCEPTUAL QUESTIONS & ANSWERS
3. Explain the equivalent resistance of a series and parallel resistor network.  In parallel connection,
Resistor in series : (i) Potential difference across each resistance will be the same (V)
(ii) But current flows through different resistors will be different.
 Let 𝐼1 , 𝐼2 , 𝐼3 be the currents flow through 𝑅1 , 𝑅2 , 𝑅3 respectively.
 Let 𝑹𝑷 be the equivalent resistance in parallel connection, then from Ohm’s law
𝑉 𝑉 𝑉 𝑽
𝐼1 = ; 𝐼2 = ; 𝐼3 = ; 𝑰=
𝑅1 𝑅2 𝑅3 𝑹𝑷
 Hence the total current will be,
𝐼 = 𝐼1 + 𝐼2 + 𝐼3
 When two or more resistors are connected end to end, they are said to be in 𝑽 𝑉 𝑉 𝑉
series. = + +
𝑹𝑷 𝑅1 𝑅2 𝑅3
 Let 𝑅1 , 𝑅2 , 𝑅3 be the resistances of three resistors connected in series. 𝑉 1 1 1
 Let ‘V’ be the potential difference applied across this combination. =𝑉 [ + + ]
𝑅𝑃 𝑅1 𝑅2 𝑅3
 In Series connection, 𝟏 𝟏 𝟏 𝟏
(i) Current through each resistor will be same (I) ∴ = + +
(ii) But potential difference across different resistor will be different. 𝑹𝑷 𝑹𝟏 𝑹𝟐 𝑹𝟑
 Let 𝑉1 , 𝑉2 , 𝑉3 be the potential difference across 𝑅1 , 𝑅2 , 𝑅3 respectively.  When resistances are connected in parallel, the reciprocal of equivalent
 Let 𝑹𝑺 be the equivalent resistance in series connection, then from Ohm’s law resistance is equal to the sum of the reciprocal of the values of resistance of the
𝑉1 = 𝐼 𝑅1 individual resistor.
𝑉2 = 𝐼 𝑅2  The equivalent resistance in parallel connection will be lesser than each
𝑉3 = 𝐼 𝑅3 individual resistance.
𝑽 = 𝑰 𝑹𝑺 4. Explain the determination of the internal resistance of a cell using voltmeter.
 Total potential difference, Internal resistance of a cell :
𝑉 = 𝑉1 + 𝑉2 + 𝑉3
𝐼 𝑅𝑆 = 𝐼 𝑅1 + 𝐼 𝑅2 + 𝐼 𝑅3
𝐼 𝑅𝑆 = 𝐼 [𝑅1 + 𝑅2 + 𝑅3 ]
∴ 𝑹𝑺 = 𝑹𝟏 + 𝑹𝟐 + 𝑹𝟑
 When resistances are connected in series, the equivalent resistance is the sum of
the individual resistances.
 The equivalent resistance in series connection will be greater than each
individual resistance.
Resistors in parallel :
 A real battery is made of electrodes and electrolyte.
 There is resistance to the flow of charges within the battery and this resistance
is called internal resistance (r)
 The emf of the cell is measured by connecting high resistance voltmeter across
it without connecting the external resistance R
 This circuit may be considered as open, the voltmeter reading gives the emf (𝜀)
of the cell.
 Then external resistance is included in the circuit and current ‘I’ is established
 When two or more resistors are connected across the same potential difference,
in the circuit.
they are said to be in parallel.
 This circuit is then considered as close, the voltmeter reading gives the
 Let 𝑅1 , 𝑅2 , 𝑅3 be the resistances of three resistors connected in parallel.
potential difference (V) across ‘R’
 Let ‘V’ be the potential difference applied across this combination. 𝑉
 By Ohm’s law, 𝑉 = 𝐼 𝑅 (or) 𝐼 =
𝑅

victory R. SARAVANAN. M.Sc., M.Phil., B.Ed PG ASST [PHYSICS], GBHSS, PARANGIPETTAI - 608 502
12 PHYSICS UNIT - 2 CURRENT ELECTRICITY CONCEPTUAL QUESTIONS & ANSWERS
 Due to internal resistance of the cell, the voltmeter reads the value “V” which is  The product ‘IR’ is taken as positive when we proceed along the direction of
less than the emf 𝜉 current and taken as negative when we proceed opposite to the direction of
 It is because, certain amount of voltage (Ir) has dropped across the internal current.
resistance ‘r’. Hence  Simillarly, the emf is considered as positive, when we proceed from negative to
𝑉 = 𝜀−𝐼𝑟 − − − − (2) positive terminal of the cell and as negative, when we proceed from positive to
(𝑜𝑟) 𝐼𝑟= 𝜀−𝑉 negative terminal of the cell.
𝜺−𝑽 𝜺−𝑽 6. Obtain the condition for bridge balance in Wheatstone’s bridge.
∴ 𝒓= = [ ]𝑹
𝑰 𝑽 Wheatstone’s bridge :
 Since 𝜉 , V and R are known, internal resistance ‘r’ and total current ‘I’ can be
determined.
 The power delivered to the circuit is,
𝑃 = 𝐼 𝜀 = 𝐼 ( 𝑉 + 𝐼 𝑟) = 𝐼 (𝐼 𝑅 + 𝐼 𝑟)
𝑷 = 𝑰𝟐 𝑹 + 𝑰𝟐 𝒓
where , 𝐼 2 𝑅 → power deliverd to R
𝐼 2 𝑟 → power deliverd to 𝑟
5. Explain Kirchoff’s law.
Kirchoff first law (current law) :
 It states that the algebraic sum of currents at any junction in a circuit is zero
(∑ 𝐼 = 0).
Explanation :
 It is a statement of conservation of electric
charge.
 Thus all charges that enter a given junction in a
circuit must leave that junction.
 Current entering the junction is taken as positive
and current leaving the junction is taken as  An important application of Kirchoff’s laws is the Wheatstone’s bridge.
negative.  It is used to compare resistances and also helps in determining the unknown
 Applying this law at junction ‘A’ resistance in the electrical network
𝐼1 + 𝐼2 − 𝐼3 − 𝐼4 − 𝐼5 = 0  The bridge consists of four resistances P, Q, R, S connected as shown.
(𝑜𝑟) 𝐼1 + 𝐼2 = 𝐼3 + 𝐼4 + 𝐼5  A galvanometer ‘G’ is connected between B and D
Kirchoff second law (voltage law) :  A battery ‘𝜀 ′ is connected between A and C
 It states that in a closed circuit the algebraic sum of the products of the current  Let 𝐼1 , 𝐼2 , 𝐼3 , 𝐼4 currents through various branches and 𝐼𝐺 be the current through
and reistance of each part of the circuit is equal to the total emf included in the the galvanometer.
circuit ( ∑ 𝑰 𝑹 = ∑ 𝜺)  Applying Kirchoff’s current law at B and D,
Explanation : 𝐼1 − 𝐼𝐺 − 𝐼3 = 0 − − − − (1)
𝐼2 + 𝐼𝐺 − 𝐼4 = 0 − − − − (2)
 Applying Kirchoff’s voltage law ABDA and ABCDA,
𝐼1 𝑃 + 𝐼𝐺 𝐺 − 𝐼2 𝑅 = 0 − − − − (3)
𝐼1 𝑃 + 𝐼3 𝑄 − 𝐼2 𝑅 − 𝐼4 𝑆 = 0 − − − − (4)
 At balanced condition, the potential at B and D are same, and hence the
galvanometer shows zero deflection. So 𝑰𝑮 = 𝟎
 Put this in equation (1), (2) and (3)
𝐼1 − 𝐼3 = 0 (𝑜𝑟) 𝐼1 = 𝐼3 − − − − (5)
𝐼2 − 𝐼4 = 0 (𝑜𝑟) 𝐼2 = 𝐼4 − − − − (6)
 It is a statement of conservation of energy for an isolated system. 𝐼1 𝑃 − 𝐼2 𝑅 = 0 (𝑜𝑟) 𝐼1 𝑃 = 𝐼2 𝑅 − − − − (7)
victory R. SARAVANAN. M.Sc., M.Phil., B.Ed PG ASST [PHYSICS], GBHSS, PARANGIPETTAI - 608 502
12 PHYSICS UNIT - 2 CURRENT ELECTRICITY CONCEPTUAL QUESTIONS & ANSWERS
 Put equation (5) and (6) in (4)  Due to imperfect contace of wire at its ends, some resistance might be
𝐼1 𝑃 + 𝐼1 𝑄 − 𝐼2 𝑅 − 𝐼2 𝑆 = 0 introduced at the contact. These are called end resistances.
𝐼1 (𝑃 + 𝑄) − 𝐼2 (𝑅 + 𝑆) = 0  By interchange P and Q,tThis error can be eliminated, and the average value of P
∴ 𝐼1 (𝑃 + 𝑄) = 𝐼2 (𝑅 + 𝑆) − − − − (8) is found.
 Divide equation (8) by (7)  Let 𝑙 be the length and r be the radius of wire, its specific resistance
𝐼1 (𝑃 + 𝑄) 𝐼2 (𝑅 + 𝑆) (resistivity) is given be.
=
𝐼1 𝑃 𝐼2 𝑅 𝑷𝑨 𝑷 𝝅 𝒓𝟐
𝑃+𝑄 𝑅+𝑆 𝑄 𝑆 𝝆= = − − − −(𝟑)
= (𝑜𝑟) 1+ =1+ 𝒍 𝒍
𝑃 𝑅 𝑃 𝑅 8. How the emf of two cells are compared using potentiometer?
𝑄 𝑆 𝑷 𝑹 Comparision of emf of two cells :
= (𝑜𝑟) = − − (𝟗)
𝑃 𝑅 𝑸 𝑺
7. Explain the determination of unknown resistance using meterbridge.
Meterbridge:

 Potentiometer wire CD is connected to battery (Bt) and a key (K) in series. This
 Metrebridge is another form of Wheatstone’s bridge is the primary circuit.
 It consists of uniform manganin wire AB of 1m length.  The end C is connected to central terminal M of DPDT switch and another central
 This wire is stretched along a metre scale between two copper strips C and D terminal N is connected to jockey through a galvanometer G and high reistance
 E is another copper strip mounted with two gaps G1 and G2 HR. This is the secondary circuit.
 An unknown resistance P is connected in G 1 and standard resistance connected  The cell whose emf 𝜀1 and 𝜀2 to be compared are connected to 𝑀1 𝑁1 and 𝑀2 𝑁2
in G2 of DPDT switch.
 A jockey J is connected from E through a galvanometer G and high resistance HR.  Initially the cell of emf 𝜀1 is included in the secondary circuit and the balancing
length 𝑙1 is found by adjusting jockey for zero deflection.
 A Lechlanche cell 𝜀 and key K is connected across the bridge wire.
 Simillarly the cell of emf 𝜀2 is included in the secondary circuit and the balancing
 The position of jockey is adjusted so that the galvanometer shows zero
length 𝑙2 is found.
deflection. Let the point be ‘J’
 Let ‘r’ be the resistance per unit length and ‘I’ be the primary current, then by the
 The lengths AJ and JB now replace the resistance R and S of the Wheatstone’s
principle
bridge. Then
𝜀1 = 𝐼 𝑟 𝑙1 − − − − (1)
𝑃 𝑅 𝑅 𝐴𝐽
= =  𝜀2 = 𝐼 𝑟 𝑙2 − − − − (2)
𝑄 𝑆 𝑅 𝐽𝐵  Divide equantion (1) by (2),
Where 𝑅 → resistance per unit length 𝜀1 𝐼 𝑟 𝑙1
𝑷 𝑨𝑱 𝒍𝟏 =
= = − − − −(𝟏) 𝜀2 𝐼 𝑟 𝑙2
𝑸 𝑱𝑩 𝒍𝟐 𝜺𝟏 𝒍𝟏
𝒍𝟏 = − − − −(𝟑)
(𝒐𝒓) 𝑷= 𝑸 − − − −(𝟐) 𝜺𝟐 𝒍𝟐
𝒍𝟐

victory R. SARAVANAN. M.Sc., M.Phil., B.Ed PG ASST [PHYSICS], GBHSS, PARANGIPETTAI - 608 502
12 PHYSICS UNIT - 2 CURRENT ELECTRICITY CONCEPTUAL QUESTIONS & ANSWERS
9. Explain the method of measurement of internal resistance of a cell using
potentio meter.
Internal resistance by potentiometer :
 Potentiometer wire CD is
connected to battery (Bt) and a
key (K1) in series. This is the
primary circuit.
 The cell 𝜀 whose internal
resistance ‘r’ to be measured is
connected to the secondary
circuit.
 A resistance box R and a key K2 is
connected across the cell 𝜀
 With key K2 open, the balancing
point J is found out and balancing length CJ = 𝑙1 is measured.
 By the principle,
𝜀 ∝ 𝑙1 − − − −(1)
 A suitable resistance is included in R and key K2 is closed.
 The current flows through R and cell is,
𝜀
𝐼=
𝑅+𝑟
 Hence potential difference across R
𝜀
𝑉 =𝐼𝑅= 𝑅
𝑅+𝑟
 For this potential difference, again the balancing point J is found out and the
balancing length CJ = 𝑙2 is measured.
 By the principle,
𝜀
𝑅 ∝ 𝑙2 − − − −(2)
𝑅+𝑟
 Divide equation (1) by (2)
𝜀 𝑙1
𝜀 =
( 𝑅) 𝑙2
𝑅+𝑟
𝑅+𝑟 𝑙1
=
𝑅 𝑙2
𝑟 𝑙1
1+ =
𝑅 𝑙2
𝑟 𝑙1 𝑙1 − 𝑙2
= −1=
𝑅 𝑙2 𝑙2
𝒍𝟏 − 𝒍𝟐
𝒓=𝑹 [ ] − − − (𝟑)
𝒍𝟐
 By substituting 𝑅, 𝑙1 , 𝑙2 in equation (3) the internal resistance of the cell can be
measured.
 Here the internal reistance is not constant, and it increased with increase of
external resistance R.

victory R. SARAVANAN. M.Sc., M.Phil., B.Ed PG ASST [PHYSICS], GBHSS, PARANGIPETTAI - 608 502
12 PHYSICS UNIT - 2 CURRENT ELECTRICITY CONCEPTUAL QUESTIONS & ANSWERS
6. The resistance of a wire is 20 Ω. What will be new resistance, if it is stretched
WORKED EXAMPLE PROBLEMS WITH SOLUTIONS uniformly 8 times its original length?
Solution :- 𝑅1 = 20 Ω ; 𝑙1 = 𝑙 ; 𝑙2 = 8 𝑙 ; 𝑅2 = ?
1. Compute the current in the wire if a charge of 120 C is flowing through a copper  Though the wire is stretched, its volume remains unchanged.(i.e.)
wire in 1 minute. Initial volume = final volume
Solution :- 𝑡 = 1 𝑚𝑖𝑛 = 60 𝑠 ; 𝑞 = 120 𝐶 ; 𝐼 = ? 𝐴1 𝑙1 = 𝐴2 𝑙2
 By definition, electric current (i.e.) rate of flow of charge is given by, 𝐴1 𝑙 = 𝐴2 (8 𝑙)
𝑞 120 𝐴2 1
𝐼= = =𝟐𝑨 = − − − − − − (1)
𝑡 60 𝐴1 8
2. If an electric field of magnitude 570 N C , is applied in the copper wire, find the
–1
𝑙
acceleration experienced by the electron.  Initial resistance ; 𝑅1 = 𝜌 1
𝐴1
Solution :- 𝐸 = 570 𝑁 𝐶 −1 ; 𝑒 = 1.6 𝑋 10−19 𝐶 ; 𝑚 = 9.1 𝑋 10−31 𝑘𝑔 ; 𝑎 = ? 𝑙
 From Newton’s second law, force is given by ; 𝐹 = 𝑚 𝑎  New resistance ; 𝑅2 = 𝜌 2
𝐴2
 Hence the acceleration, 𝑅 𝑙 𝐴 𝐴 𝑙 1 𝑙 1
−19
 Hence the ratio ; 1 = 1 2 = 1 1 = 𝑋 = 64
𝐹 𝑒𝐸 1.6 𝑋 10 𝑋 570 912 𝑅 2 𝐴 1 2 𝑙 𝐴1 2𝑙 8 8 𝑙
𝑎= = = = 𝑋 1012 = 100.1 𝑋 1012 ∴ 𝑅2 = 64 𝑅1 = 64 𝑋 20 = 𝟏𝟐𝟖𝟎 𝛀
𝑚 𝑚 9.1 𝑋 10−31 9.1
𝒂 = 𝟏. 𝟎𝟎𝟏 𝑿 𝟏𝟎𝟏𝟒 𝒎 𝒔−𝟐  Hence, stretching the length of the wire has increased its resistance.
3. A copper wire of cross-sectional area 0.5 mm carries a current of 0.2 A. If the
2 7. Consider a rectangular block of metal of height A, width B and length C as
free electron density of copper is 8.4 × 10 m then compute the drift velocity
28 –3 shown in the figure. If a potential difference
of free electrons. of V is applied between the two faces A and B
2 −6 2
Solution :- 𝐴 = 0.5 𝑚𝑚 = 0.5 𝑋 10 𝑚 ; 𝐼 = 0.2 𝐴 ; 𝑛 = 8.4 𝑋 10 𝑚 28 −3 of the block [figure (a)], the current IAB is
 The relation between drift velocity of electrons and current in a wire of cross- observed. Find the current that flows if the
sectional area A is ; 𝐼 = 𝑛 𝐴 𝑒 𝑣𝑑 same potential difference V is applied
𝐼 between the two faces B and C of the block
 Hence, the drift velocity ; 𝑣𝑑 = [figure (b)]. Give your answers in terms of
𝑛𝐴𝑒
0.2 0.2 𝑋 10−3 IAB.
𝑣𝑑 = =
8.4 𝑋 1028 𝑋 0.5 𝑋10−6 𝑋 1.6 𝑋 10−19 6.72 Solution :-
−2 −3
𝒗𝒅 = 2.976 𝑋 10 𝑋 10 = 2.976 𝑋 10 𝑚 𝑠 −5 −1  ln first case ; length =C and area = AB. Hence resistance and current
𝑙𝑒𝑛𝑔𝑡ℎ 𝐶
𝒗𝒅 = 𝟎. 𝟎𝟐𝟗𝟕𝟔 𝑿 𝟏𝟎−𝟑 𝒎 𝒔−𝟏 ≈ 𝟎. 𝟎𝟑 𝑿𝟏𝟎−𝟑 𝒎 𝒔−𝟏 𝑅𝐴𝐵 = 𝜌 = 𝜌
4. Determine the number of electrons flowing per second through a conductor, 𝑎𝑟𝑒𝑎 𝐴𝐵
𝑉 𝑉 𝑉 (𝐴𝐵)
when a current of 32 A flows through it. 𝐼𝐴𝐵 = = = − − − − (1)
Solution :- 𝐼 = 32 𝐴 ; 𝑡 = 1 𝑠 ; 𝑒 = 1.6 𝑋 10 −19
; 𝑛=? 𝑅 𝐶 𝜌𝐶
𝐴𝐵 [𝜌 ]
𝑞 𝑛𝑒 𝐴𝐵
 By definition, current (rate of flow of charge) is given by ; 𝐼 = =  ln second case ; length =A and area = BC. Hence resistance and current
𝑡 𝑡
 Hence the number of electrons flowing per second ; 𝑙𝑒𝑛𝑔𝑡ℎ 𝐴
𝑅𝐵𝐶 = 𝜌 = 𝜌
𝐼𝑡 32 𝑋 1 𝑎𝑟𝑒𝑎 𝐵𝐶
𝒏= = = 20 𝑋 1019 = 𝟐 𝑿 𝟏𝟎𝟐𝟎 𝒆𝒍𝒆𝒄𝒕𝒓𝒐𝒏𝒔 𝑉 𝑉 𝑉 (𝐵𝐶)
𝑒 1.6 𝑋 10−19 𝐼𝐵𝐶 = = = − − − − (2)
5. A potential difference across 24 Ω resistor is 12 V. What is the current through 𝑅𝐵𝐶 𝐴 𝜌𝐴
[𝜌 ]
the resistor? 𝐵𝐶
Solution :- R = 24  ; V = 12 V ; I=?  Divide equation (2) by (1), we get
𝐼𝐵𝐶 [𝑉 (𝐵𝐶)/𝜌 𝐴] 𝑉 (𝐵𝐶) 𝜌𝐶 𝐶2
 From Ohm’s law; = = 𝑋 = 2
𝑉 12 1 𝐼𝐴𝐵 [𝑉 (𝐴𝐵)/𝜌 𝐶] 𝜌𝐴 𝑉 (𝐴𝐵) 𝐴
𝐼= = = 𝑪 𝟐
𝑅 24 2 𝑰𝑩𝑪 = 𝟐 𝑰𝑨𝑩
𝑰 = 𝟎. 𝟓 𝑨 𝑨
 Since 𝐶 > 𝐴 , the current 𝑰𝑩𝑪 > 𝑰𝑨𝑩

victory R. SARAVANAN. M.Sc., M.Phil., B.Ed PG ASST [PHYSICS], GBHSS, PARANGIPETTAI - 608 502
12 PHYSICS UNIT - 2 CURRENT ELECTRICITY CONCEPTUAL QUESTIONS & ANSWERS
8. Calculate the equivalent resistance for the circuit 11. Calculate the equivalent resistance between A and B in the given circuit.
which is connected to 24 V battery and also find
the potential difference across each resistors in
the circuit.
Solution :- 𝑅1 = 4 Ω ; 𝑅2 = 6 Ω ; V = 24 V
 Effective resistance in series circuit,
𝑹𝑺 = 𝑅1 + 𝑅2 = 4 + 6 = 𝟏𝟎 𝛀
𝑉 24 Solution :- 𝑅𝐴𝐵 = ?
 From Ohm’s law, current in the circuit ; 𝐼= 𝑹𝑺
= 10
= 𝟐. 𝟒 𝑨  Here, 2  and 2  , 4  and 4 , 6  and 6  are in parallel, then
 Hence voltage across the resistors, 1 1 1
= + = 1 (𝑜𝑟) 𝑅𝑃1 = 1 Ω
𝑉1 = 𝐼 𝑅1 = 2.4 𝑋 4 = 𝟗. 𝟔 𝑽 𝑅𝑃1 2 2
𝑉2 = 𝐼 𝑅2 = 2.4 𝑋 6 = 𝟏𝟒. 𝟒 𝑽 1 1 1 1
= + = (𝑜𝑟) 𝑅𝑃2 = 2 Ω
9. Calculate the equivalent resistance in the following 𝑅𝑃2 4 4 2
circuit and also find the values of current I, I1 and I2 in 1 1 1 2 6
the given circuit. = + = (𝑜𝑟) 𝑅𝑃2 = = 3 Ω
𝑅𝑃3 6 6 6 2
Solution :- 𝑅1 = 4 Ω ; 𝑅2 = 6 Ω ; V = 24 V  Thus the above circuit becomes,
 Effective resistance in parallel circuit,
1 1 1 1 1 6+4 10 1
= + = + = = =
𝑅𝑃 𝑅1 𝑅2 4 6 24 24 2.4
(or) 𝑹𝑷 = 𝟐. 𝟒 𝛀
 Then current flows through the resistors,  Here, 1 , 2  and 3  are in series, then the
𝑉 24 effective resistance becomes,
𝐼1 = = =𝟔𝑨 𝑅𝐴𝐵 = 𝑅𝑃1 + 𝑅𝑃2 + 𝑅𝑃3 = 1 + 2 + 3
𝑹𝟏 4
𝑉 24 𝑹𝑨𝑩 = 𝟔 𝛀
𝐼2 = = = 𝟒𝑨 12. Five resistors are connected in the configuration as shown in the figure.
𝑹𝟐 6
Calculate the equivalent resistance between the points a and b.
 The current I is the sum of the currents in the two branches. Then
Solution :-
𝑰 = 𝐼1 + 𝐼2 = 6 + 4 = 𝟏𝟎 𝑨
10. Two resistors when connected in series and parallel, their equivalent  Let us assume that a current is entering the
𝟓𝟔 junction at a.
resistances are 15 Ω and Ω respectively. Find the values of the resistances.  Since all the resistances in the outside loop are
𝟏𝟓
56 the same (1Ω), the current in the branches ac
Solution :- 𝑅𝑆 = 15 Ω ; 𝑅𝑃 = Ω ; 𝑅1 = ? ; 𝑅2 = ?
15 and ad must be equal. Hence the points C and D
 Effective resistance in series circuit ; 𝑹𝑺 = 𝑅1 + 𝑅2 are at the same potential and no current
15 = 𝑅1 + 𝑅2 − − − − − − − − − (1) through 5 Ω.
1 1 1 𝑅 +𝑅
 Effective resistance in parallel circuit ; = + = 1 2  It implies that the 5 Ω has no role in
𝑅𝑃 𝑅1 𝑅2 𝑅1 𝑅2
15 𝑅1 + 𝑅2 15 determining the equivalent resistance and it can
= = [∵ 𝑏𝑦 𝑒𝑞𝑛 (1)] be removed. So the circuit is simplified as shown in the figure.
56 𝑅1 𝑅2 𝑅1 𝑅2
∴ 56 = 𝑅1 𝑅2 − − − − − − − − (2)
 From equation (1) & (2) ; 𝑅1 + 𝑅2 = 15 and 𝑅1 𝑅2 = 56, we get
𝑅1 = 7 & 𝑅2 = 8 (𝑜𝑟) 𝑅1 = 8 & 𝑅2 = 7
 If 𝑅1 = 7 then, 𝑅2 = 15 − 𝑅1 = 15 − 7 = 8 . So 𝑹𝟏 = 𝟕 𝛀 ; 𝑹𝟐 = 𝟖 𝛀
 If 𝑅1 = 8 then, 𝑅2 = 15 − 𝑅1 = 15 − 8 = 7 . So 𝑹𝟏 = 𝟖 𝛀 ; 𝑹𝟐 = 𝟕 𝛀

victory R. SARAVANAN. M.Sc., M.Phil., B.Ed PG ASST [PHYSICS], GBHSS, PARANGIPETTAI - 608 502
12 PHYSICS UNIT - 2 CURRENT ELECTRICITY CONCEPTUAL QUESTIONS & ANSWERS
 Effective resistance in series connection is, 16. Two electric bulbs marked 20 W – 220 V and 100
𝑹𝑺𝟏 = 𝑅𝑎𝑐 + 𝑅𝑐𝑏 = 1 + 1 = 2 Ω W – 220 V are connected in series to 440 V supply.
𝑹𝑺𝟐 = 𝑅𝑎𝑑 + 𝑅𝑑𝑏 = 1 + 1 = 2 Ω Which bulb will get fused?
 Effective resistance in parallel connection is, Solution :-
1 1 1 1 1
= + = + =1  To check which bulb will get fused, the voltage
𝑅𝑎𝑏 𝑅𝑆1 𝑅𝑆2 2 2 drop across each bulb has to be calculated.
𝑹𝒂𝒃 = 𝟏 𝛀 𝑉2
13. If the resistance of coil is 3 Ω at 200 C and α = 0.004/0C then determine its  The power delivered by the battery ; 𝑃 = 𝑉 𝐼 =
𝑅
resistance at 100 0C.  Hence the resistance of the bulbs,
Solution :- 𝑇𝑜 = 20 𝐶 ; 𝑇 = 100  𝐶 ; 𝑅𝑜 = 3 Ω ; 𝑅𝑇 = ? 𝑉1 2 2202 48400
 Resistance at 𝑇 𝐶 ; 𝑅𝑇 = 𝑅𝑜 [1 + 𝛼 (𝑇 − 𝑇𝑜 )]
𝑅1 = 𝑃1
= 20
= 20 = 2420 Ω
𝑅𝑇 = 3 [ 1 + 0.004 (100 − 20)] = 3 [1 + 0.004 𝑋 80] 𝑉2 2202 48400
𝑅𝑇 = 3 [ 1 + 0.32] = 3 𝑋 1.32 𝑅2 = 2 = = = 484 Ω
𝑃2 100 100
𝑹𝑻 = 𝟑. 𝟗𝟔 𝛀  The two bulbs are connected in series, effective resistance
14. Resistance of a material at 200C and 400C are 45 Ω and 85 Ω respectively. Find 𝑅𝑡𝑜𝑡 = 2420 + 484 = 2904 Ω
its temperature coefficient of resistivity.  When the bulbs are connected in series, the current passing
Solution :- 𝑇𝑜 = 20 𝐶 ; 𝑇 = 40 𝐶 ; 𝑅𝑜 = 45 Ω ; 𝑅𝑇 = 85 Ω ; α = ? through each bulb is the same and it is given by,
 The temperature coefficient of resistivity is 𝑉 𝑉1 + 𝑉2 220 + 220 440
1 Δ𝑅 1 (𝑅𝑇 − 𝑅𝑜 ) 𝐼= = = = 𝐴
𝛼 = 𝑅 Δ 𝑇 = 𝑅 ( 𝑇− 𝑇 ) 𝑅𝑡𝑜𝑡 𝑅𝑡𝑜𝑡 2904 2904
𝑜 𝑜 𝑜
1 (85−45) 1 40 1 𝐼 = 1.515 𝑋 10−1 𝐴 = 0. 1515 𝐴
𝛼 = 45 𝑋 (40−20) = 45 𝑋 20 = 45 𝑋 2  The voltage drop across the 20W bulb is
𝜶 = 𝟎. 𝟎𝟒𝟒 / 𝑪 𝑉1 = 𝐼 𝑅1 = 0.1515 𝑋 2420 = 3.667 𝑋 102 = 𝟑𝟔𝟔. 𝟕 𝑽
15. A battery of voltage V is connected to 30 W bulb  The voltage drop across the 100W bulb is
and 60 W bulb as shown in the figure. (a) Identify 𝑉2 = 0.1515 𝑋 484 = 7. 333 𝑋 101 = 𝟕𝟑. 𝟑𝟑 𝑽
brightest bulb (b) which bulb has greater  The 20 W bulb will get fused because the voltage across it is more than the
resistance? (c) Suppose the two bulbs are voltage rating.
connected in series, which bulb will glow brighter? 17. A battery has an emf of 12 V and connected to a resistor of 3 Ω. The current in
Solution :- 𝑃1 = 30 𝑊 ; 𝑃2 = 60 𝑊 the circuit is 3.93 A. Calculate (a) terminal voltage and the internal resistance
(a) The power delivered by the battery ; 𝑃 = 𝑉 𝐼 . of the battery (b) power delivered by the battery and power delivered to the
Since the bulbs are connected in parallel, the resistor
voltage drop across each bulb is the same. If the voltage is kept fixed, then the Solution :- I = 3.93 A ; 𝜖 = 12 V ; R = 3 Ω
power is directly proportional to current (P ∝ I). Since 𝑃1 < 𝑃2 , we have 𝐼1 < (a) The terminal voltage of the battery is equal to voltage drop across the resistor
𝐼2 . So 60 W bulb draws twice as much as current as 30 W and hence 60 W bulb 𝑽 = 𝐼 𝑅 = 3.93 𝑋 3 = 𝟏𝟏. 𝟕𝟗 𝑽
will glow brighter than 30 W bulb. Internal resistance of the battery,
𝜖−𝑉 12 − 11.79
𝑉2 1 𝑟= [ ]𝑅 = [ ] 𝑋3
(b) The power delivered by the battery ; 𝑃 = 𝑉 𝐼 = . Hence 𝑃  𝑉 11.79
𝑅 𝑅
0.21𝑋 3 0.63
Since 𝑃1 < 𝑃2 , we have 𝑅1 > 𝑅2 . It implies that, the 30W has twice as much 𝒓 = = = 5.341 𝑋 10−2 Ω = 𝟎. 𝟎𝟓𝟑𝟒𝟏 𝛀
11.79 11.79
as resistance as 60 W bulb. (b) The power delivered by the battery,
(c) When the bulbs are connected in series, the current passing through each bulb is 𝑃 = ∈ 𝐼 = 12 𝑋 3.93 = 47.16 𝑊
the same. It is equivalent to two resistors connected in series. The bulb which has The power delivered to the resistor
higher resistance has higher voltage drop. So 30W bulb will glow brighter than 𝑃 = 𝑉 𝐼 = 11.79 𝑋 3.93 = 46.33 𝑊
60W bulb. So the higher power rating does not always imply more brightness and The remaining power P = 47.16 − 46.33 = 𝟎. 𝟖𝟑 𝑾 is delivered to the internal
it depends whether bulbs are connected in series or parallel. resistance and cannot be used to do useful work. (It is equal to 𝐼2 𝑟).
victory R. SARAVANAN. M.Sc., M.Phil., B.Ed PG ASST [PHYSICS], GBHSS, PARANGIPETTAI - 608 502
12 PHYSICS UNIT - 2 CURRENT ELECTRICITY CONCEPTUAL QUESTIONS & ANSWERS
18. From the given circuit, Find 21. The following figure shows a complex network
(a) Equivalent emf of the combination of conductors which can be divided into two
(b) Equivalent internal resistance closed loops like EACE and ABCA.
(c) Total current Solution :-
(d) Potential difference across external  Apply Kirchhoff’s voltage rule (KVR) in EACE
resistance ∑𝐼 𝑅 = ∑ ∈
(e) Potential difference across each cell 𝐼1 𝑅1 + 𝐼2 𝑅2 + 𝐼3 𝑅3 = ∈
Solution :- 𝑛 = 4; ∈= 9𝑉; 𝑟 = 0.1 Ω  Apply Kirchhoff’s voltage rule (KVR) in ABCA
∑𝐼 𝑅 = ∑ ∈
(a) Equivalent emf of the combination ; ∈𝒕𝒐𝒕 = 𝑛 ∈ = 4 𝑋 9 = 𝟑𝟔 𝑽
𝐼4 𝑅4 + 𝐼5 𝑅5 − 𝐼3 𝑅3 = 0
(b) Equivalent internal resistance ; 𝒓𝒕𝒐𝒕 = 𝑛 𝑟 = 4 𝑋 0.1 = 𝟎. 𝟒 𝛀
22. Calculate the current that flows in the 1 Ω resistor in the following circuit.
(c) Total current ;
𝑛 ∈ 4𝑋9
𝐼= =
𝑅+𝑛𝑟 10 + 4 𝑋 0.1
36 36
𝑰 = = = 𝟑. 𝟒𝟔𝟐 𝑨
10 + 0.4 10.4
(d) Potential difference across external resistance ; 𝑽 = 𝐼 𝑅 = 3.462 𝑋 10 = 𝟑𝟒. 𝟔𝟐 𝑽
𝑉 34.62
(e) Potential difference across each cell ; 𝐯 = = = 𝟖. 𝟔𝟓𝟓 𝑽
4 4
19. From the given circuit. Find Solution :-
(a) Equivalent emf
 We can denote the current that flows from 9V battery as I1 and it splits up into I2
(b) Equivalent internal resistance and (I1 – I2) at the junction E according Kirchhoff ’s current rule (KCR).
(c) Total current (I)  Now consider the loop EADFE and apply KVR, we get
(d) Potential difference across each cell ∑𝐼𝑅 = ∑ ∈
(e) Current from each cell 3 (𝐼1 − 𝐼2 ) − 𝐼2 = 6
Solution :- 3 𝐼1 − 3 𝐼2 − 𝐼2 = 6
(a) Equivalent emf ; ∈𝒕𝒐𝒕 = ∈ = 𝟓 𝑽 3 𝐼1 − 4 𝐼2 = 6 − − − − − − (1)
𝑟 0.5  Now consider the loop EFCBE and apply KVR, we get
(b) Equivalent internal resistance ; 𝒓𝒕𝒐𝒕 = = = 𝟎. 𝟏𝟐𝟓 𝛀
∈ 5
𝑛 4 ∑𝐼𝑅 = ∑ ∈
(c) Total current ; 𝐼 = 𝑟 = 0.5 𝐼2 + 3 𝐼1 + 2 𝐼1 = 9
𝑅+ 10+
𝑛 4 5 𝐼1 + 𝐼2 = 9 − − − − − − (2)
5 5 (2) 𝑋 4 ⇒
𝐼= = 20 𝐼1 + 4 𝐼2 = 36 − − − − − − (3)
10 + 0.125 10.125  (1) + (3) ⇒ 23 𝐼1 = 42
𝑰 = 4. 939 𝑋 10−1 𝐴 = 𝟎. 𝟒𝟗𝟑𝟗 𝑨 ≈ 𝟎. 𝟓 𝑨 42
(d) Potential difference across each cell ; 𝑽 = 𝐼 𝑅 = 0.5 𝑋 10 = 𝟓 𝑽 𝑰𝟏 = = 𝟏. 𝟖𝟐𝟔 𝑨
23
I 0.5  Put , 𝐼1 = 1.826 in equation (1)
(e) Current from each cell ; 𝑰𝟏 = = = 𝟎. 𝟏𝟐𝟓 𝑨
n 4 3 (1.826) − 4 𝐼2 = 6
20. For the given circuit find the value of I. 5.478 − 4 𝐼2 = 6
Solution :- −4 𝐼2 = 6 − 5.478 = 0.522
 From Kirchoff’s first law, at junction P ; ∑ 𝐼 = 0 (𝑜𝑟) 4 𝐼2 = −0.522
0.2 + (−0.4) + 0.6 + (−0.5) + 0.7 + (−𝐼) = 0 0.522
0.2 − 0.4 + 0.6 − 0.5 + 0.7 − 𝐼 = 0 𝑰𝟐 = − = − 𝟎. 𝟏𝟑𝟎𝟓 𝑨
4
∴ 𝐼 = 0.2 − 0.4 + 0.6 − 0.5 + 0.7  It implies that the current in the 1  resistor flows from F to E.
𝑰 = 𝟏. 𝟓 − 𝟎. 𝟗 = 𝟎. 𝟔 𝑨

victory R. SARAVANAN. M.Sc., M.Phil., B.Ed PG ASST [PHYSICS], GBHSS, PARANGIPETTAI - 608 502
12 PHYSICS UNIT - 2 CURRENT ELECTRICITY CONCEPTUAL QUESTIONS & ANSWERS
23. In a Wheatstone’s bridge P = 100 Ω, Q = 1000 Ω and R = 40 Ω. If the 28. An electric heater of resistance 10 Ω connected to 220 V power supply is
galvanometer shows zero deflection, determine the value of S. immersed in the water of 1 kg. How long the electrical heater has to be
Solution :- switched on to increase its temperature from 30°C to 60°C. (Specific heat
 From the balanced condition of Wheatstone’s bridge, capacity of water is s = 4200 J kg–1 K–1)
𝑃 𝑅 Solution :- R = 10 Ω ; V = 220 V ; m = 1 kg ; 𝑇1 = 30C ; 𝑇2 = 60C ; t = ?
= 𝑉2
𝑄 𝑆  From Joule’s law of heating ; 𝐻 = 𝐼 2 𝑅 𝑡 = 𝑡
𝑅
𝑅𝑄 40 𝑋 1000  Heat absorbed by the water ; 𝐻 = 𝑚 𝑠 Δ𝑇 = 𝑚 𝑠 (𝑇2 − 𝑇1 )
𝑺= = = 𝟒𝟎𝟎 𝛀
𝑃 100  Here, heat produced by the heater in time ‘t’ is equal to the heat absorbed by the
24. What is the value of x when the Wheatstone’s network is balanced? water to increase its temperature (i.e.)
Solution :- P = 500 Ω, Q = 800 Ω, R = x + 400, S = 1000 Ω 𝑉2
 From the balanced condition of Wheatstone’s bridge, 𝑡 = 𝑚 𝑠 (𝑇2 − 𝑇1 )
𝑅
𝑃 𝑅 𝑅
= 𝑡 = 2 𝑚 𝑠 (𝑇2 − 𝑇1 )
𝑄 𝑆 𝑉
500 𝑥 + 400 10
= 𝑡= 𝑋 1 𝑋 4200 𝑋 (60 − 30)
800 1000 2202
5 𝑥 + 400 10
= 𝑡= 𝑋 4200 𝑋 30
8 1000 220 𝑋 220
5000 = 8 (𝑥 + 400) 4200 𝑋 3 12600
5000 = 8 𝑥 + 3200 𝑡= = = 2.604 𝑋 101
22 𝑋 22 484
8 𝑥 = 5000 − 3200 = 1800 𝒕 = 𝟐𝟔. 𝟎𝟒 𝒔
1800
𝒙 = = 𝟐𝟐𝟓 𝛀
8
25. In a meter bridge experiment with a standard resistance of 15 Ω in the right
gap, the ratio of balancing length is 3:2. Find the value of the other resistance.
Solution :- Q = 15 Ω ; 𝑙1 ∶ 𝑙2 = 3 ∶ 2 ; 𝑃 = ?
𝑃 𝑙1
 From the theory of meter bridge ; =
𝑄 𝑙2
𝑙1 3 45
(𝑜𝑟) 𝑃=𝑄 = 15 𝑋 =
𝑙2 2 2
𝑷 = 𝟐𝟐. 𝟓 𝛀
26. In a meter bridge experiment, the value of resistance in the resistance box
connected in the right gap is 10 Ω. The balancing length is l1 = 55 cm. Find the
value of unknown resistance.
Solution :- Q = 10 Ω ; 𝑙1 = 55 𝑐𝑚 ; 𝑙2 = 100 − 𝑙1 = 45 𝑐𝑚 ; P = ?
𝑃 𝑙1
 From the theory of meter bridge ;
𝑄
= 𝑙2
𝑙1 55 550
(𝑜𝑟) 𝑃=𝑄 = 10 𝑋 =
𝑙2 45 45
𝑷 = 𝟏𝟐. 𝟐𝟐 𝛀
27. Find the heat energy produced in a resistance of 10 Ω when 5 A current flows
through it for 5 minutes.
Solution :- R = 10 Ω ; 𝐼 = 5 𝐴 ; 𝑡 = 5 𝑚𝑖𝑛 = 300 𝑠 ; 𝐻 = ?
 From Joule’s law of heating,
𝐻 = 𝐼 2 𝑅 𝑡 = 52 𝑋 10 𝑋 300 = 25 𝑋 3000
𝑯 = 𝟕𝟓𝟎𝟎𝟎 𝑱 = 𝟕𝟓 𝒌 𝑱
victory R. SARAVANAN. M.Sc., M.Phil., B.Ed PG ASST [PHYSICS], GBHSS, PARANGIPETTAI - 608 502
12 PHYSICS UNIT - 2 CURRENT ELECTRICITY CONCEPTUAL QUESTIONS & ANSWERS
3. A copper wire of 10–6 m2
area of cross section, carries a current of 2 A. If the
EXERSICE PROBLEMS WITH SOLUTIONS number of free electrons per cubic meter in the wire is 8 × 10 28, calculate the
1. The following graphs represent the current versus voltage and voltage versus current density and average drift velocity of electrons.
current for the six conductors A,B,C,D,E and F. Which conductor has least Solution :- 𝐴 = 10−6 𝑚2 ; 𝐼 = 2 𝐴 ; 𝑛 = 8 𝑋 1028 ; 𝐽 = ? ; 𝑣𝑑 = ?
resistance and which has maximum resistance?  Current density ;
𝐼 2
𝑱= = = 𝟐 𝑿 𝟏𝟎𝟔 𝑨 𝒎−𝟐
𝐴 10−6
 Aveage drift velocity ;
𝐼 𝐽
𝑣𝑑 = =
𝑛𝐴𝑒 𝑛𝑒
2 𝑋 106 1
𝑣𝑑 = 28 −19
= 𝑋 10−3
8 𝑋 10 𝑋 1.6 𝑋 10 6.4
𝒗𝒅 = 𝟏. 𝟓𝟔𝟐 𝑿 𝟏𝟎−𝟒 𝒎 𝒔−𝟏 = 𝟏𝟓. 𝟔𝟐 𝑿 𝟏𝟎−𝟓 𝒎 𝒔−𝟏
Solution :-
4. The resistance of a nichrome wire at 200C is 10 Ω. If its temperature coefficient
Figure (1) : of resistivity of nichrome is 0.004/0C, find the resistance of the wire at boiling
Δ𝑉 2
 Resistance of conductor - A ; 𝑅𝐴 = Δ𝐼 = 4 = 0.5 Ω point of water. Comment on the result.
Δ𝑉 4 Solution :- 𝑇𝑜 = 0 𝐶 ; 𝑇 = 100  𝐶 ; 𝑅𝑜 = 10 Ω ; 𝑅𝑇 = ?
Resistance of conductor - B ; 𝑅𝐵 = Δ𝐼 = 3 = 1.33 Ω
 Resistance of the conducting wire at T 0C is,
Δ𝑉 5 𝑅𝑇 = 𝑅𝑜 [1 + 𝛼 (𝑇 − 𝑇𝑜 )]
Resistance of conductor - C ; 𝑅𝐶 = = = 2.5 Ω
Δ𝐼 2 𝑅𝑇 = 10 [ 1 + 0.004 (100 − 0)]
Figure (2) : 𝑅𝑇 = 10 [1 + 0.004 𝑋 100]
Δ𝑉 4 𝑅𝑇 = 10 [ 1 + 0.4] = 10 𝑋 1.4
 Resistance of conductor - D ; 𝑅𝐷 = Δ𝐼 = 2 = 2 Ω
Δ𝑉 3
𝑹𝑻 = 𝟏𝟒 𝛀
Resistance of conductor - E ; 𝑅𝐸 = Δ𝐼 = 4 = 0.75 Ω  As the temperature increases the resistance of the wire also increases.
Δ𝑉 2 5. The rod given in the figure is made up of two different materials. Both have
Resistance of conductor - F ; 𝑅𝐹 = Δ𝐼 = 5 = 0.4 Ω square cross sections of 3 mm side. The resistivity of the first material is
 Thus conductor F has least resistance (i.e.) 𝑹𝑭 = 𝟎. 𝟒 𝛀 4 × 10–3 Ωm and that of second material has resistivity of 5 × 10–3 Ωm. What is
And conductor C has maximum resistance (i.e.) 𝑹𝑪 = 𝟐. 𝟓 𝛀 the resistance of rod between its ends?
2. Lightning is very good example of natural current. In
typical lightning, there is 109 J energy transfer across
the potential difference of 5 × 107 V during a time
interval of 0.2 s. Using this information, estimate the Solution :- 𝐴 = 3𝑚𝑚𝑋3𝑚𝑚 = 9 𝑚𝑚2 = 9 𝑋 10−6 𝑚2 ; 𝑙1 = 25 𝑐𝑚 = 25 𝑋 10−2 𝑚
following quantities 𝑙2 = 70 𝑐𝑚 = 70 𝑋 10−2 𝑚 ; 𝜌1 = 4 𝑋 10−3 ; 𝜌2 = 5 𝑋 10−3
(a) total amount of charge transferred between cloud  Resistance of first material ;
𝜌1 𝑙1 4 𝑋 10−3 𝑋 25 𝑋 10−2 1000
and ground (b) the current in the lightning bolt (c) the 𝑅1 = = = Ω
𝐴 9 𝑋 10 −6 9
power delivered in 0.2 s.
 Resistance of second material ;
Solution :- 𝑉 = 5 𝑋 107 𝑉 ; 𝑡 = 0.2 𝑠 ; 𝑈 = 109 𝐽 𝜌2 𝑙2 5 𝑋 10−3 𝑋 70 𝑋 10−2 3500
𝑈 109 1 𝑅2 = = = Ω
(a) Total charge ; 𝑸 =
𝑉
= 5 𝑋 107
= 5 𝑋 102 = 0.2 𝑋 102 = 𝟐𝟎 𝑪 𝐴 9 𝑋 10 −6 9
𝑄 20 200  Since the two materials are in series, their effective resistance,
(b) Current ; 𝑰= = = = 𝟏𝟎𝟎 𝑨 1000 3500 1000 + 3500 4500
𝑡 0.2 2 𝑅𝑡𝑜𝑡 = 𝑅1 + 𝑅2 = + = =
𝑈 10 9 9 9 9 9
(c) Power ; 𝑷 = 𝑡 = 0.2 = 𝟓 𝑿 𝟏𝟎𝟗 𝑾 = 𝟓 𝑮 𝑾 𝑹𝒕𝒐𝒕 = 𝟓𝟎𝟎 𝛀

victory R. SARAVANAN. M.Sc., M.Phil., B.Ed PG ASST [PHYSICS], GBHSS, PARANGIPETTAI - 608 502
12 PHYSICS UNIT - 2 CURRENT ELECTRICITY CONCEPTUAL QUESTIONS & ANSWERS
6. Three identical lamps each having a resistance R are connected to the battery of (d) Power delivered :
emf 𝝐 as shown in the figure Suddenly the switch S is closed. (a) Calculate the  When S is open,
current in the circuit when S is open 𝜖 𝜖 𝝐𝟐
and closed (b) What happens to the 𝑷𝑨 = 𝑉 𝐴 𝐼 = 𝑋 =
3 3𝑅 𝟗𝑹
intensities of the bulbs A,B and C. 𝜖 𝜖 𝝐𝟐
(c) Calculate the voltage across the 𝑷𝑩 = 𝑉𝐵 𝐼 = 𝑋 =
3 3𝑅 𝟗𝑹
three bulbs when S is open and 𝜖 𝜖 𝝐𝟐
closed (d) Calculate the power 𝑷 𝑪 = 𝑉𝐶 𝐼 = 𝑋 =
3 3𝑅 𝟗𝑹
delivered to the circuit when S is 𝝐𝟐 𝝐𝟐
Total power delivered; 𝑷𝒕𝒐𝒕 = 𝑷𝑨 + 𝑷𝑩 + 𝑷𝑪 = 𝟑 ( ) =
𝟗𝑹 𝟑𝑹
opened and closed (e) Does the
 When S is closed,
power delivered to the circuit
𝜖 𝜖 𝝐𝟐
decrease, increase or remain same? 𝑷𝑨 = 𝑉𝐴 𝐼 = 𝑋 =
Solution :- 2 2𝑅 𝟒𝑹
𝜖 𝜖 𝝐𝟐
(a) Current : 𝑷𝑩 = 𝑉𝐵 𝐼 = 𝑋 =
2 2𝑅 𝟒𝑹
 When S is open, all the bulbs A, B and C are in series, and hence their effective 𝑷𝑪 = 0
resistance becomes ; 𝑅𝑇 = 𝑅 + 𝑅 + 𝑅 = 3 𝑅 . Thus from Ohm’s law, 𝝐𝟐 𝝐𝟐
𝜖 𝝐 Total power delivered; 𝑷𝒕𝒐𝒕 = 𝑷𝑨 + 𝑷𝑩 + 𝑷𝑪 = 𝟐 ( ) =
𝟒𝑹 𝟐𝑹
𝑰= =
𝑅𝑇 𝟑𝑹 (e) When S is suddenly closed, the power delivered to the circuit will increases.
 When S is closed, only the bulbs A and B are in series and hence their 7. An electronics hobbyist is building a radio which requires 150 Ω in her circuit.
effective resistance becomes 𝑅𝑇 = 𝑅 + 𝑅 = 2 𝑅 and no current flows But she has only 220 Ω, 79 Ω and 92 Ω resistors available. How can she connect
through bulb C. Thus from Ohm’s law, the available resistors to get the desired value of resistance?
𝜖 𝝐 Solution :-
𝑰= =
𝑅𝑇 𝟑𝑹  The value of equivalent resistance in series connection will be greater than each
(b) Intensity : individual resistance. Similarly the value of equivalent resistance in parallel
 When S is open, all the bulbs A, B and C are in series. Here current through all connection will be lesser than each individual resistance.
the bulbs are same. So all the bulbs glow with equal intensity.  When all the three resistors are connected in series, their effective resistance
 When S is closed, only the bulbs A and B are in series . The intensities of the will be greater than 220 Ω and when all the three resistors are connected in
bulbs A and B equally increase. Bulb C will not glow since no current pass parallel, their effective resistance will be less than 79 Ω
through it.  Initially first two resistors 220 Ω and 79 Ω are connected in parallel and their
(c) Voltage (Potential difference) : effective value,
 When S is open, 1 1 1 220 + 79 299
𝜖 𝝐 = + = =
𝑽𝑨 = 𝐼 𝑅𝐴 = 𝑋𝑅 = 𝑅𝑃 220 79 220 𝑋 79 17380
3𝑅 𝟑 17380
𝜖 𝝐 ∴ 𝑅𝑃 = = 5. 811 𝑋 101 Ω ≈ 𝟓𝟖 𝛀
𝑽𝑩 = 𝐼 𝑅𝐵 = 𝑋𝑅 = 299
3𝑅 𝟑
𝜖𝜖 𝝐  Now 58 Ω and third resistor 92 Ω are connected in series its effective value,
𝑽𝑪 = 𝐼 𝑅𝐶 = 𝑋𝑅 = 𝑅𝑺 = 58 + 92 = 150 Ω
3𝑅 𝟑
 When S is closed,  Thus in order to get 𝟏𝟓𝟎 𝛀 resistance, 220 Ω and 79 Ω are connected in parallel
𝜖 𝝐 and this combination is connected in series with 9𝟐 𝛀
𝑽𝑨 = 𝐼 𝑅𝐴 = 𝑋𝑅 =
2𝑅 𝟐
𝜖 𝝐
𝑽𝑩 = 𝐼 𝑅𝐵 = 𝑋𝑅 =
2𝑅 𝟐
𝑽𝑪 = 0

victory R. SARAVANAN. M.Sc., M.Phil., B.Ed PG ASST [PHYSICS], GBHSS, PARANGIPETTAI - 608 502
12 PHYSICS UNIT - 2 CURRENT ELECTRICITY CONCEPTUAL QUESTIONS & ANSWERS
8. A cell supplies a current of 0.9 A through a 2 Ω resistor and a current of 0.3 A 10. A potentiometer wire has a length of 4 m and resistance of 20 Ω. It is connected
through a 7 Ω resistor. Calculate the internal resistance of the cell. in series with resistance of 2980 Ω and a cell of emf 4 V. Calculate the potential
Solution :- 𝐼1 = 0.9 𝐴 ; 𝐼2 = 0.3 𝐴 ; 𝑅1 = 2 Ω ; 𝑅2 = 7 Ω ; r = ? gradient along the wire.
 From Ohm’s law (V = I R), Solution :- 𝑙 = 4 𝑚 ; 𝑅 = 20  ; 𝑅𝑒𝑥 = 2980  ; ϵ = 4 V ; 𝑉 = ?
𝜖 = 𝐼1 (𝑅1 + 𝑟) = 0.9 (2 + 𝑟) − − − − (1)  Resistance per unit length of the potentiometer wire, 𝑟=
20
= 5 Ω 𝑚−1
4
𝜖 = 𝐼2 (𝑅2 + 𝑟) = 0.3 (7 + 𝑟) − − − − (2) 𝜖 4 4
 From equation (1) and (2)  From Ohm’s law ; 𝐼 = = (20+2980) = 3000
(𝑅+ 𝑅𝑒𝑥 )
0.9 (2 + 𝑟) = 0.3 (7 + 𝑟) 4
𝐼= = 1. 33 𝑋 10−3 𝐴
1.8 + 0.9 𝑟 = 2.1 + 0.3 𝑟 3 𝑋 103
0.9 𝑟 − 0.3 𝑟 = 2.1 − 1.8  Potential difference across the potentiometer wire
0.6 𝑟 = 0.3 𝑉 = 𝐼 𝑅 = 1.33 𝑋 10−3 𝑋 20 = 26.6 𝑋 10−3 𝑉
0.3 1  Then the potential gradient along the wire
𝑟= = 2 = 𝟎. 𝟓 𝛀
0.6 𝑽 = 𝐼 𝑟 = 1.33 𝑋 10−3 𝑋 5 = 𝟔. 𝟔𝟓 𝑿 𝟏𝟎−𝟑 𝑽𝒎−𝟏
9. Calculate the currents in the following circuit. 11. Determine the current flowing through the galvanometer (G) as shown in the
figure.
Solution :-
 Apply Kirchoff’s current law at the junction P,
2 = 𝐼1 + 𝐼2 (𝑜𝑟)
𝐼2 = 2 − 𝐼1 − − − − − − (1)
 Apply Kirchoff’s voltage law to the closed loop
PQSP ,
5 𝐼1 + 10 𝐼𝑔 − 15 𝐼2 = 0
Solution :- 5 𝐼1 + 10 𝐼𝑔 − 15 (2 − 𝐼1 ) = 0
 Apply Kirchoff’s current law at the junction B, 5 𝐼1 + 10 𝐼𝑔 − 30 + 15 𝐼1 = 0
𝐼1 = 𝐼2 + 𝐼3 − − − − − − (1) 20 𝐼1 + 10 𝐼𝑔 − 30 = 0
 Apply Kirchoff’s voltage law to the closed loop ABEFA and BCDEB, 20 𝐼1 + 10 𝐼𝑔 = 30 − − − − − − (2)
100 𝐼3 + 100 𝐼1 = 15 − − − − − − (2)  Similarly Apply Kirchoff’s voltage law to the closed loop QRSQ,
and 100 𝐼2 − 100 𝐼3 = − 9 − − − − − − (3) 10 (𝐼1 − 𝐼𝑔 ) − 10 𝐼𝑔 − 20 (𝐼2 + 𝐼𝑔 ) = 0
 Put equation (1) in (2) 10 𝐼1 − 10 𝐼𝑔 − 10 𝐼𝑔 − 20 𝐼2 − 20 𝐼𝑔 = 0
100 𝐼3 + 100( 𝐼2 + 𝐼3 ) = 15 10 𝐼1 − 40 𝐼𝑔 − 20 𝐼2 = 0
100 𝐼3 + 100 𝐼2 + 100 𝐼3 = 15
10 𝐼1 − 40 𝐼𝑔 − 20 (2 − 𝐼1 ) = 0
100 𝐼2 + 200 𝐼3 = 15 − − − − − − (4)
(4) − (3) ⟹ 300 𝐼3 = 24 10 𝐼1 − 40 𝐼𝑔 − 40 + 20 𝐼1 = 0
24 8 30 𝐼1 − 40 𝐼𝑔 − 40 = 0
𝐼3 = = = 0.08 𝐴 30 𝐼1 − 40 𝐼𝑔 = 40 − − − − − − (3)
300 100
 Put this in equation (3) (2)𝑋 3 ⟹ 60 𝐼1 + 30 𝐼𝑔 = 90 − − − − − − (4)
100 𝐼2 − 100 (0.08) = − 9 (3)𝑋 2 ⟹ 60 𝐼1 − 80 𝐼𝑔 = 80 − − − − − − (5)
100 𝐼2 − 8 = − 9 (4) − (5) ⟹ 110 𝐼𝑔 = 10
100 𝐼2 = − 9 + 8 = − 1 11 𝐼𝑔 = 1
1 𝟏
𝐼2 = − = − 0.01 𝐴 𝑰𝒈 = 𝑨
100 𝟏𝟏
 Then equation(1) becomes,
𝐼1 = − 0.01 + 0.08 = 0.07 𝐴
 Thus , 𝑰𝟏 = 𝟎. 𝟎𝟕 𝑨 ; 𝑰𝟐 = − 𝟎. 𝟎𝟏 𝑨 ; 𝑰𝟑 = 𝟎. 𝟎𝟖 𝑨

victory R. SARAVANAN. M.Sc., M.Phil., B.Ed PG ASST [PHYSICS], GBHSS, PARANGIPETTAI - 608 502
12 PHYSICS UNIT - 2 CURRENT ELECTRICITY CONCEPTUAL QUESTIONS & ANSWERS
12. Two cells each of 5V are connected in series with a 8 Ω resistor and three Solution :-
parallel resistors of 4 Ω, 6 Ω and 12 Ω. Draw a circuit diagram for the above
arrangement. Calculate (i) the current drawn from the cells (ii) current
through each resistor
Solution :-
 Circuit diagram for the above arrangement,

14. In a potentiometer arrangement, a cell of emf 1.25 V gives a balance point at 35


(i) The current drawn from the cells : cm length of the wire. If the cell is replaced by another cell and the balance
 Here, 4, 6  and 12  all are in parallel, their effective resistance, point shifts to 63 cm, what is the emf of the second cell?
1 1 1 1 3+2+1 6 1 Solution :- : 𝜖1 = 1.25 𝑉 ; 𝑙1 = 35 𝑐𝑚 ; 𝑙2 = 63 𝑐𝑚 ; 𝜖2 = ?
= + + = = = (𝑜𝑟) 𝑅𝑃 = 2 Ω
𝑅𝑃 4 6 12 12 12 2  The ratio of emf’s of two cells using potentiometer,
 Also, 8  and 2  are in series, their effective resistance, ‹ 𝜖1 𝑙1
𝑅𝑺 = 8 + 2 = 10  =
𝜖2 𝑙2
 Total potential difference(voltage) ; 𝑉 = 5 + 5 = 10 𝑉 𝑙2
 Then current in the circuit, 𝜖2 = 𝜖1
𝑙1
𝑉 10 63 𝑋 10−2 78.75
𝑰= = =𝟏𝑨 𝜖2 = 1.25 𝑋 =
𝑅𝑆 10 35 𝑋 10 −2 35
 The potential difference across parallel combination of effective 𝝐𝟐 = 𝟐. 𝟐𝟓 𝑽
resistance 𝑅𝑃 = 2 Ω is ; 𝑉𝑃 = 𝐼 𝑅𝑃 = 1 𝑋 2 = 2 𝑉
(ii) Current through each resistor :
 Current through 8 Ω resistor ; 𝑰 = 𝟏 𝑨
𝑉𝑃 𝟐
 Current through 4 Ω resistor ; 𝑰𝟏 = = = 𝟎. 𝟓 𝑨
4 𝟒
𝑉𝑃 𝟐
 Current through 6 Ω resistor ; 𝑰𝟐 = = = 𝟎. 𝟑𝟑 𝑨
6 𝟔
𝑉𝑃 𝟐
 Current through 12 Ω resistor ; 𝑰𝟑 = = = 𝟎. 𝟏𝟕 𝑨
12 𝟏𝟐
13. Four bulbs P, Q, R, S are connected in a circuit of unknown arrangement. When
each bulb is removed one at a time and replaced, the following behavior is
observed. Draw the circuit diagram for these bulbs.

victory R. SARAVANAN. M.Sc., M.Phil., B.Ed PG ASST [PHYSICS], GBHSS, PARANGIPETTAI - 608 502
12 PHYSICS UNIT - 2 CURRENT ELECTRICITY CONCEPTUAL QUESTIONS & ANSWERS
8. Is Ohm’s law universally applicable for all conducting elements? If not give
UNIT - 2 CONSEPTUAL QUESTIONS AND ANSWERS examples of elecment which do not obey Ohm’s law.
1. What is the charge on a wire, when a current flow through the wire?  According to Ohm’s law, 𝑉 = 𝐼 𝑅 (𝑜𝑟) 𝑉 ∝ 𝐼
 The net charge on the wire is zero.  For metallic conductor, the current versus voltage graph is a straight line
 Eventhough there is drifting of the free electrons. the total number of electrons through the orgin. Thus, Ohm’s law holds good in the case of metallic conductor
in the wire at any instant is equal toe the total number of protons. So the total and so they are called Ohmic conductor.
charge is zero.  But in case of liquids, vacuum tubes, semiconductor diodes, transistors,
2. A metal contains a large number of free electrons and they are in motion. Then thyristors etc., the current versus voltage graphis non - linear, having more
why there is no current in a piece of wire which it is not connected to a cell? comples relationship and they do not have a constant resistance. Thus these
 The free electrons in a wire moves in random direction. (i.e.) they move in all materials do not follow Ohm’s law and so they are called non - Ohmic.
possible directions. It is due to their collisions with the positive ions of the 9. Why is electric current a scalar quantity but current density a vecor quantity?
metal.So the net charge flowing in a fixed direction is zero. Hence there is no  For a quantity to be a vector it has to have magnitude, direction and has to obey
current in the wire. law of addition of vectors. Since current has magnitude and direction but it does
3. The electron drift velocity is estimated to be only few mm/s (i.e.) 10 -4 m/s for not obey the law of vector addition and it follows only algebraic addition current
currents in the range of a few ampere (A). Then how current established and is a scalar quantity. By definition, current ; 𝑰 = ⃗𝑱⃗ . ⃗⃗⃗⃗
𝑨
electric bulb glows almost the instant, when we switch on the battery?  But current density at a point inside the conductor is a vector, whose direction is
 For the flow of current to start through the wire, the electrons need not travel the direction of flow of charge at that point and whose magnitude is the current
from one end to the other end of the wire. When the circuit is closed, the free through the unit area perpendicular to the dirction of flow. Hence current
electrons begins to move away from the negative terminal of the battery and this density satisfies the conditions for a quantity to be a vector.
electron exerts force on the nearby electrons. 10. It is easier to start a car engine on a warm day than on a chilly day. Why?
 Thus an electric field is established in it and propagation of electric impulse  With increase in temperature on a warm day, the internal resistance of a car
takes place through the wire at the speed of light. in other words, the energy is battery decreases. Due to it, the battery can supply large current which helps in
transported from the battery to the bulb at the speed of light through starting the car engine easily
propagating electric field. 11. A high tension supply of say 6 KV must have a large internal resistance. Why?
 Due to this reason, the bulb glows as soon as the battery is switched on.  A high tension supply shoud be provided with large internal reistance so that
4. Why manganin is preferred for making standard resistance? current does not exceed the safely limit.
 The temperature coefficient of resistance of magnanin is nearly zero. So the  If the internal resistance is not large, then the current drawn can exceed the
resistance of magnanin does not change with temperature. safey limits which damage the circuits.
 This is the reason for magnanin is preferred for making standard resistance. 12. Give the dimensional formula for resistance and specific resistance.
5. If coil of wire is used to construct resistors, what is the length of the wire used  Dimensional formula for resistance, [S I unit is ohm () ]
for making infinite resistance? 𝑊 𝐹𝑠
𝑉 ( ) ( ) 𝐹𝑠 [𝑀 𝐿 𝑇 −2 ] [𝐿]
𝑞
 No wire is used to produce infinite resistance. 𝑅= = = 𝐼𝑡 = 2 ⟹ = [𝑴 𝑳𝟐 𝑻−𝟑 𝑨−𝟐 ]
 Infinite resistance is produced by an air gap. 𝐼 𝐼 𝐼 𝐼 𝑡 [𝐴2 ] [𝑇]
6. If the electron drift is so small and the electron charge is small, how can we still  Dimensional formula for specific resistance, [S I unit is ohm - metre ( m) ]
obtain large amount of current in the conductor? 𝑅𝐴 [𝑀 𝐿2 𝑇 −3 𝐴−2 ] [𝐿2 ]
𝜌= ⟹ = [𝑴 𝑳𝟑 𝑻−𝟑 𝑨−𝟐 ]
 We know that, electric current ; 𝐼 = 𝑛 𝐴 𝑒 𝑣𝑑 𝑙 [𝐿]
 Thus the current depends on the number of free electrons per unit volume ‘n’ 13. The light from a bath room bulb get dimmer for a moment when the geyser is
(i.e.) electron density , in addition to charge ‘e’ and drift velocity ‘𝑣𝑑 ’ swithched on. Why?
 Here the electron density is very large quantity. Hence the current is large.  Since geyser is initially at low temperature, its resistance is comparatively low.
7. When electrons drift in a metal from lower potential to higher potential, does it Thus power consumed by the geyser is high and it consumes more current when
mean that all the free electrons of the metal are moving in the same direction? switched on. So voltage drop across the geyser increses. Because of this,
 No. The free electrons always have their random thermal velocities. The drift available votage for the bulb comes down. So the light gets dimmer.
velocity is superposed over this. So all the electrons need not move from lower  After geyser become hot with consequent increase in resistance, it consumes
to a higher potential. lesser power. So the brightness of the bulb is restored.

victory R. SARAVANAN. M.Sc., M.Phil., B.Ed PG ASST [PHYSICS], GBHSS, PARANGIPETTAI - 608 502
12 PHYSICS UNIT - 2 CURRENT ELECTRICITY CONCEPTUAL QUESTIONS & ANSWERS
14. Why repairing the electrical connection with the wet skin is always dangerous? 20. Why the jockey should not be pressed too hard on the wire when sliding over it?
 The dry skin of the human body has high resistance of around 500 k  .  Sliding the jockey with a hard press, will scratch the wire and make its thickness
 But when the skin is wet, the resistance is reduced to around 1000  and hence non-uniform. Then the resistance per unit length of the wire will not remain
its conductivity increases. So it is not safe, while repairing electrical connection constant because resistance depend upon area of cross-section.
with wet hand. 21. Will the interchange of positions of cell and galvanometer in wheatstones
15. Two wires of equal lengths, one is copper and the other is mangnanin, have the bridge affect the balance condition?
same resistance. Which wire is thicker?  No. The condition of balanced Wheatstone bridge remains satisfied.
𝑹𝑨 22. What is meant by end - correction for a metre bridge?
 The resistivity is given by ; 𝝆 =
𝒍
 The two ends of the manganin wire not exactly coincide with the zero mark and
 Given that ‘R’ and ′𝑙′ are same for both copper and manganin. So 𝝆 ∝ 𝑨
100 cm mark of the metre scale. Also the two ends of the long wire need not be
(i.e.) area of cross section (thickness) is greater for metal with greater resistivity.
in good contact with the copper strips at the ends.
 The resistivity of copper is ; 𝝆 = 𝟏. 𝟕 𝑿 𝟏𝟎−𝟖  m
 A correction has to be applied for this. This is called end correction.
 The resistivity of manganin is ; 𝝆 = 𝟒. 𝟐 𝑿 𝟏𝟎−𝟕  m
23. Which is preferred for measuring emf of a cell, a volt meter or a potentiometer.
 Hence manganin wire is thicker than copper, because of its resistivity is greater Why?
than that of copper.
 For measuring emf of a cell, potentiometer is preferred. Because, at balancing
16. Wheatstone bridge method of determination of resistance is considered to be
point, the potentiometer does not draw any current from the source of emf.
more accurate than the voltmeter-ammeter method. Why?
 But a voltmeter always draws small current and hence it gives a reading less
 Wheatstone bridge method is a null method. The resistance determined does not
than the emf of the cell. Also there can be error in reading the deflections of the
depend on the deflection of the galvanometer and the supply voltage
voltmeter.
fluctuations. So it is more accurate than other methods.
24. Why is a ten-wire potentiometer more sensitive than a four- wire one?
17. Wheatstone bridge is not suitable for measuring very low resistance and very
 The potential gradient (i.e) fall of potential per unit length of the potentiometer
high resistance. Why?
wire decreases with an increase in the length of the potentiometer wire.
 While measuring a low resistance, all the other resistances must also be low, so
 Smaller the potential gradient, better its sensitivity.
that bridge is sensitive. But a low resistance galvanometer is insensitive. Further
the wire resistance and lead resistance come into the picture and affect the  Hence, a 10-wire potentiometer having a smaller potential gradient is more
sensitive than a 4-wire one.
result.
25. Why copper wire is not used in potentiometer? What is the preferred material
 While measuring a high resistance, the current through the galvanometer is very
used for making potentiometer wires?
low. Hence the galvanometer needs to be sensitive to sense the low current.
 The resistivity of copper is very small and its temperature coefficient of
Otherwise it shows zero deflection even if there is current flowing through it and
resistance large. Since change in resistance with increase in temperature is high,
the bridge is unbalanced.
copper wire is not used in potentiometer.
18. It is better to get the null point in the middle of the wire in a metre bridge.
Why?  Manganin or Constantan alloy are preferred material used for making
potentiometer wire because, it is characterised by a low temperature coefficient
 Because, when the null point is in the middle, the resistances of the ratio arms
of resistance and a high resistivity. This will enable us to have a large fall of
are nearly equal and the bridge is sensitive. Further it minimise the effect of
potential across any length of wire without any appreciable change in resistance
neglecting of end resistances in calculations.
due to change in temperature.
19. Why is a metre bridge so called? It is also called a slide bridge. Why? If the
26. While preforming an experiment with potentiometer, the deflection in the
length of the wire used be 120 cm, how will formula used to measure
galvanometer is found to be in the same direction, when jockey is pressed at
resistance of unknown resistance will change?
the starting and extreme end of the wire. Also it is seen that (i) the deflection is
 Since the bridge uses one metre long manganin wire, it is called a metre bridge.
less at the extreme end than at the starting end and (ii) the deflection is more
 It is also called slidebridge because, the jockey is slide over the bridge wire while
at the extreme end than at the starting end. Why?
performing the experiments.
(i) In this case, although the driver cell in the primary circuit and the source of emf
 If this bridge uses a manganin wire of 120 cm, then the formula for finding in the secondary circuit are connected with their positive terminals at the same
𝒍𝟏 𝒍
unknown resistance will be ; 𝑷 = 𝑸 =𝑸 [ 𝟏 ] end, we ae not getting the balancing point, because the emf of the driver cell is
𝒍𝟐 𝟏𝟐𝟎− 𝒍𝟏
less than the emf of the cell in the secondary circuit. So use a driver cell with
higher value of emf.
victory R. SARAVANAN. M.Sc., M.Phil., B.Ed PG ASST [PHYSICS], GBHSS, PARANGIPETTAI - 608 502
12 PHYSICS UNIT - 2 CURRENT ELECTRICITY CONCEPTUAL QUESTIONS & ANSWERS
(ii) In this case, the driver cell and the cell in the secondary circuit are connected 𝑽
 It can inferred from the given table that the ratio of votage with current (i.e.) is
with wrong polarities. Change the polarity of one of the cell to get balancing 𝑰
point. a constant, which is equal to 18. Hence manganin is an Ohmic conductor (i.e.) the
27. Two wires of A and B are of same metal and of same length have their areas of alloys obeys Ohm’s law.
cross section in the ratio 2 : 1 . If the same potential difference is applied across  According to Ohm’s law, the ratio of votage with current is the resistance (R) of
each wire in turn, what will be the ratio of current flowing in A and B? the conductor. Hence the resistance of given manganin is 𝑹 = 𝟏𝟖 
𝒍 𝟏 32. Two devices of rating 44W ; 220 V and 11W ; 220 V are connected in series. The
 The resistance is given by, 𝑹 = 𝝆 . Here 𝝆 and 𝒍 are constants. So 𝑹 ∝ . combination is connected across a 440 V mains. The fuse of which of the two
𝑨 𝑨
Since the ratio of area of cross section is 2 : 1, the ratio of resistance will be 1 : 2 devices is likely to burn when switch is on? Justify your answer.
𝑽 𝟏 𝑽𝟐 𝟐𝟐𝟎 𝑿 𝟐𝟐𝟎
 By Ohm’s law, 𝑰 = . Here 𝑽 is constant. So 𝑰 ∝  Resistance of 11 W device ; 𝑹 = = = 𝟒𝟒𝟎𝟎 
𝑹 𝑹 𝑷 𝟏𝟏
Therfore current will be in the ratio 2 : 1 𝑽𝟐 𝟐𝟐𝟎 𝑿 𝟐𝟐𝟎
28. A set of ‘n’ identical resistors, each of resistance ‘R’, when connected in series  Resistance of 44 W device ; 𝑹 =
𝑷
= 𝟒𝟒
= 𝟏𝟏𝟎𝟎 
have an effective resistance of ‘X’ and when they are connected in parallel have  Here the resistance of 11 W device will be four times that of the device 44 W
an effective resistance of ‘Y’. Find the relation between R, X and Y?  Since they are in series connection, the current flows through them will be same
 If ‘n’ resistors are connected in series, their effective resistance will be, 𝑽𝒎𝒂𝒊𝒏 𝟒𝟒𝟎 𝟒𝟒𝟎
and it is given by ; 𝑰 = = = = 𝟎. 𝟖 𝑨
𝑋 = 𝑅 + 𝑅 + 𝑅 + ⋯ (𝑛 𝑡𝑖𝑚𝑒𝑠) = 𝑛 𝑅 𝑹𝒆𝒇𝒇 𝟒𝟒𝟎𝟎+𝟏𝟏𝟎𝟎 𝟓𝟓𝟎𝟎
 If ‘n’ resistors are connected in parallel, their effective resistance will be,  The voltage across 11 W device = 𝑰 𝑹 = 𝟎. 𝟖 𝑿 𝟒𝟒𝟎𝟎 = 𝟑𝟓𝟐 𝑽
1
=
1 1
+𝑅
1
+𝑅 + ⋯ (𝑛 𝑡𝑖𝑚𝑒𝑠) =
𝑛
(𝑜𝑟) 𝑌=
𝑅 The voltage across 44 W device = 𝑰 𝑹 = 𝟎. 𝟖 𝑿 𝟏𝟏𝟎𝟎 = 𝟖𝟖 𝑽
𝑌 𝑅 𝑅 𝑛  Hence 352 V across the device of rating 11 W ; 220 V is sufficient to burn the
𝑅
 Then multiply above equations, we get 𝑋 𝑌 = 𝑛 𝑅 [ ] = 𝑅2 ∴ 𝑹 = √𝑿 𝒀 fuse of that device.
𝑛
29. The filament of a bulb glows, but the connecting wires supplying current flows 33. Several electric bulbs designed to be used on a 220 V electric supply line are
through the connecting wire are not heated. Why? rated 10 W. How many bulbs can be connected in parallel with each other
 From Joule heating effect, the heat produced in a conductor is ; 𝐻 = 𝐼 2 𝑅 𝑡 across the two wires of 220 V line, if the maximum allowable current is 5 A?
 Here, the same current flows through the wire and the bulb and hence. 𝑯 ∝ 𝑹  Let ‘n’ be the number of bulbs and ‘R’ be the resistance of each bulb. Then
𝑉2 220 𝑋 220
 Since the resistance of the filament is high and it gets heated up and glows. 𝑅= = = 4840 
𝑃 10
 But the resistance of the connecting wire is small. So it does not heated up.  For a flow of 5 A current, the effective resistance of the ‘n’ bulbs in parallel circuit
30. A circuit consists a piece of semiconductor in series with a variable resistor 𝑽 𝟐𝟐𝟎
(R), an ammeter (A) and a source of constant voltage (V). Would you increase By Ohm’s law, 𝑹𝒆𝒇𝒇 = = = 𝟒𝟒 
𝑰 𝟓
or decrease the value of R to keep the reading of ammeter constant, when  But the effective resistance in parallel combination is given by,
semiconductor is heated? Give reasons. 1 1 1 1 𝑛 𝑅
𝑅𝑒𝑓𝑓
= 𝑅
+ 𝑅 + 𝑅 + ⋯ (𝑛 𝑡𝑖𝑚𝑒𝑠) = 𝑅
(𝑜𝑟) 𝑅𝑒𝑓𝑓 = 𝑛
 In order to maintain flow of current as constant, the effective resistance of the
𝑹 𝟒𝟖𝟒𝟎
circuit will be remain unchanged. ∴ 𝒏= = 𝟒𝟒 = 𝟏𝟏𝟎
𝑹𝒆𝒇𝒇
 When the temperature of semiconductor is increased by heating, its resistance
decreases and hence the effective resistance of the circuit also decreases.  Thus 110 bulbs have to connected in parallel
 So the current in the circuit increases. In order to maintain the ammeter reading 34. Which uses more energy, a 250 W TV set in 1 hour or a 1200 W toaster in 10
as constant, the value of R can be increased. minutes?
31. What conclusion can you draw from the following observation on a resistor  Energy consumed by TV = Power X time = 250 𝑋 3600 = 9 𝑋 105 𝐽
made of alloy of manganin? Energy consumed by Toaster = Power X time = 1200 𝑋 600 = 7.2 𝑋 105 𝐽
Voltage V (volts) Current I (ampere)  Therfore energy consumed by TV set is more
0.2 3.6 35. Why is the series arrangement not used for domestic circuits?
 In a series connection, if one electrical appliance stops working due to some
0.4 7.2
defect, then all other appliances would stop working.
0.6 10.8  In a series connection, all electrical appliances have only one switch due to which
0.8 14.4 they cannot be turned on and off separately.
1.0 18.0
victory R. SARAVANAN. M.Sc., M.Phil., B.Ed PG ASST [PHYSICS], GBHSS, PARANGIPETTAI - 608 502
12 PHYSICS UNIT - 2 CURRENT ELECTRICITY CONCEPTUAL QUESTIONS & ANSWERS
 In a series conection, all the appliances do not get the same voltage as the voltage 𝟏 𝟏
 Power consumed by 100 W bulb ; 𝑃100 = 𝐼 2 𝑅100 = 𝐼 2 [ 𝑅50 ] = 𝑃50
𝟐 𝟐
gets divided in series combination.
(i.e.) the power consumed by 50 W bulb is twice that of 100 W bulb.
 In a series connection, the equivalent resistance increases too much due to
So 50 W bulb will be brighter.
which the amount of current flowing becomes very small.
 In Parallel connection, voltage ‘V’ is constant, then
36. Copper wire cannot be used as a filament in an electric bulb. Why?
𝑽𝟐
 The material used in filament bulb works on the principle of heating effect of  Power consumed by 50 W bulb ; 𝑃50 =
𝑅50
electric current. This effect requires the filament to be of a very high resistance 𝑽𝟐 𝑽𝟐
and high metling point, so that the filament heats up and gives out light for  Power consumed by 100 W bulb ; 𝑃100 =
𝑅100
= 𝟏 = 𝟐 𝑃50
[ 𝑅50 ]
𝟐
illumination.
(i.e.) the power consumed by 100 W bulb is twice that of 50 W bulb.
 But a copper wire has very low resistance, therefore it is used for wires for
So 100 W bulb will be brighter.
conduction and transmission of electricity, but cannot be used as a filament in a
41. A heater coil is cut into two equal parts and only one part is used in the heater.
bulb working on the principle of heating effect of electric current.
What is the ratio of the heat produced by this half coil to that by the original ?
37. Is therer any difference between fuse wire and a heating coil? 𝑽𝟐 𝑽𝟐 𝑽𝟐 𝒕 𝑨
 Fuse wire is used to protect an electrical appliance from getting overloaded. Its  The heat produced in the coil of length is ; 𝑯 = 𝒕= 𝝆𝒍 𝒕=
𝑹 ( ) 𝝆𝒍
melting point is low. 𝑨

 The heating coil is the reistance wire used in the electrical appliances for 𝑽𝟐 𝒕 𝑨 𝑽𝟐 𝒕 𝑨
 If length of the coil is halved, then , 𝑯𝑰 = 𝒍 =𝟐 [ ] =𝟐𝑯
generating heat. Its melting point will be high. 𝝆[ ] 𝝆𝒍
𝟐
38. What is the change in the power dissipated if the current flowing through the  Hence the heat produced will be doubled.
circuit is doubled? 42. What is the cause for the production of heat in a current carrying conductor?
 Power dissipated is given by ; 𝑷 = 𝑰𝟐 𝑹  In a conductor, the free electrons are always at random motion making collisions
 ‘I’ is doubled and keeping ‘R’ as constant the power dissipation becomes, with ions or atoms of the conductor.
𝑷𝟏 = (𝟐𝑰)𝟐 𝑹 = 𝟒 𝑰𝟐 𝑹 = 𝟒 𝑷  When a voltage V is applied between the ends of the conductor, resulting in the
 Therefore, when ‘I’ is doubled, P becomes 4 times. flow of current I, the free electrons are accelerated.
39. It takes 10 minutes to boil water in an electric kettle. By changing the length of  Hence the electrons gain energy at the rate = 𝑉 𝐼
the heating element in the kettle, using the same power supply, water is to be  The lattice ions or atoms receive this kinetic energy from the colliding electrons
boiled in 7 minutes. How it is to be done? in random bursts and appers as heat.
𝑽𝟐 𝑽𝟐 𝑽𝟐 𝒕 𝑨  Thus for a steady current I, the amount of heat produced in time t is ; 𝑯 = 𝑽 𝑰 𝒕
 By Joule’s effect, the heat produced is given by ; 𝑯 = 𝒕= 𝝆𝒍 𝒕=
𝑹 ( ) 𝝆𝒍 43. How conduction of electricity through metal varies with elecorolyte?
𝑨
𝒍𝟏 𝒕𝟏  Current flow is due to, drifting of free electrons in metals whereas bodily
 In this case H, V, A and  are constants. So 𝒍 ∝ 𝒕 (or) =
𝒍𝟐 𝒕𝟐 movement of positive and negative ions in electrolyte
𝒕𝟐 𝟕 𝑿 𝟔𝟎 𝟕
 Therefore, 𝒍𝟐 = 𝒍𝟏 = 𝒍𝟏 = 𝒍𝟏  Due to passage of electric current, there is, no chemical change in metals
𝒕𝟏 𝟏𝟎 𝑿 𝟔𝟎 𝟏𝟎
𝟕
whereas chemical decomposition takes place in electrolyte
 The length of the heating element in the second case must be [ ] times its initial  During flow of current, no transporation of matter in metals whereas
𝟏𝟎
length transporation of matter occurs in the form of positive and negative ions and are
40. Two electric bulbs of 50 W and 100 W are given. Which one will be brighter, deposited at the electrodes in elcrolyte
when they are connected (i) in series and (ii) in parallel  With rise in temperature, the resistance of the metal increases whereas the
𝑽𝟐 resistance of the electrolyte decreases
 The resistance of 50 W bulb ; 𝑅50 =  Conductivity is high for metals whereas very low for electrolyte
𝟓𝟎
𝑽𝟐 𝟏 𝑽𝟐 𝟏 44. Which thermocouple is more efficient? Why?
 The resistance of 100 W bulb ; 𝑅100 =
𝟏𝟎𝟎
= [ ]
𝟐 𝟓𝟎
= 𝟐
𝑅50
 An antimony - bismuth (Sb-Bi) thermocouple is more efficient than any other
(i.e.) the resistanceof 50 W bulb is twice that of 100 W bulb. thermocouple. Because antimony and bismuth are widely separated in the
 Brightness depends on the power consumed by the bulb. thermo electric series and hence the thermocouple formed by Sb-Bi produces
 In series connection, current ‘I’ is constant, then maximum thermo emf compared to Cu - Fe (or) any other thermocouple.
 Power consumed by 50 W bulb ; 𝑃50 = 𝐼 2 𝑅50

victory R. SARAVANAN. M.Sc., M.Phil., B.Ed PG ASST [PHYSICS], GBHSS, PARANGIPETTAI - 608 502
12 PHYSICS UNIT - 2 CURRENT ELECTRICITY CONCEPTUAL QUESTIONS & ANSWERS
45. Name the factors that neutral temperature and inversion temperature depends 50. A wire of resistance 8  is bent in the form of a circle.
on? What is the effective resistance between the ends of a
 Neutral temperature , depends on nature of the metals used as thermocouple diameter AB?
and independent of the junction temperature  Here 4  and 4  are in parallel, the effective reistance
 Inversion temperature, depends on temperature of the cold junction and also 1 1 1 2 1
between A and B is ;
𝑅𝐴𝐵
= 4
+ 4
= 4
= 2
depends on the nature of the metals used to form the thermocouple
46. Some heat is produced in a certain heating element when connected to mains.  Therefore, 𝑹𝑨𝑩 = 𝟐 
Now the element is doubled in itself by connecting its free ends together. How 51. Two students A and B were asked to pick a resistor of 15 kΩ from a collection
much heat will be produced in same time, if this doubled element is connected of carbon resistors. ‘A’ picked a resistor with bands of colours brown, green,
across the mains? orange while ‘B’ chose a resistor with bands of black, green, red. Who picked
𝑽𝟐 the correct resistor?
 Heat produced in singlel element of resistance ‘R’ is ; 𝑯= 𝒕  From colour code, 1  brown ; 5  green ; 103 (𝑘)  orange
𝑹
 When doubled in itself by connecting its free end together, it can be treated to be  So Student ‘A’ picked up the correct resistor of 15 kΩ.
𝑅 52. Why the terminal voltage of a battery decreases when current drawn from it is
parallel combination of two elements each of reistance ( ) . Now the effective
2 increases?
𝟏 𝟏 𝟏 𝟐 𝟐 𝟒 𝑹
resistance in this case is,
𝑹𝑰
= 𝑹/𝟐 + 𝑹/𝟐
= 𝑹
+ 𝑹 = 𝑹 (or) 𝑹𝑰 = 𝟒
 Terminal voltage is nothing but the potential difference between any two points
𝑽𝟐 𝑽𝟐 𝑽𝟐 in a closed circuit and it is given by; 𝑽𝒕 = ∈ − 𝑰 𝒓 (here r internal resistance)
 Hence the heat produced in this case ; 𝑯𝑰 = 𝒕= 𝒕=𝟒 𝒕=𝟒𝑯  It is clear from this equation, when ‘I’ is large, the factor ‘ 𝑰 𝒓 ’ becomes large and
𝑹𝑰 (𝑹/𝟒) 𝑹
 Therefore, heat produced will be increased by four times. 𝑽𝒕 becomes small. Hence terminal voltage of a battery decreases when current
47. Three equal resistors connected in series across a source of emf together drawn from it is increased.
dissipate 10 W of power. What should be the power dissipated if the same 53. Give the values of given carbon resistor.
resistors are connected in parallel across the same source of emf?
 The net resistance when they are connected in series ; 𝑹𝒔 = 𝑹 + 𝑹 + 𝑹 = 𝟑 𝑹
𝑽𝟐 𝑽𝟐
 Power dissipation ; 𝑷𝒔 =
𝑹𝑺
= 𝟑 𝑹 = 𝟏𝟎 𝑾
 The net resistance when they are connected in parallel ;
𝟏 𝟏 𝟏 𝟏 𝟑 𝑹  The colour code is easily memorized by,
= + + = (or) 𝑹𝑷 =
𝑹𝑷 𝑹 𝑹 𝑹 𝑹 𝟑
 Then power dissipation in this case ; Black Brown Red Orange Yellow Green Blue Violet Grey White
𝑽𝟐 𝑽𝟐 𝑽𝟐 𝑽𝟐
𝑷𝑷 =
𝑹𝑷
= 𝑹/ 𝟑 = 𝟑 ( 𝑹 ) = 𝟗 (𝟑 𝑹) = 𝟗 𝑷𝒔 = 𝟗 𝑿 𝟏𝟎 = 𝟗𝟎 𝑾 0 1 2 3 4 5 6 7 8 9
48. A wire is turned red hot by passing electric current through it. Half of this
heated wire is immersed in cold water. What will happen to the amount of heat (i) From colour code, (ii) From colour code,
produced per unit length in the second half? Brown - 1 (First digit) Red - 2 (First digit)
 Due to immersion of half the wire in cold water, its temperature and hence its Black - 0 (Second digit) Violet - 7 (Second digit)
resistance decreases. This results in an increase in current through the wire. Blue - 106 ( Number of zero) Brown - 101 ( Number of zero)
Therefore, second half of wire will start producing more heat in a given time. Resistance=10 X 106  (or) 10 M  Resistance=27 X 101  (or) 27 
49. What do you mean by the current capacity of a cell? Tolerance = 5 % (Gold Tolerance = 5% (Gold)
 It is the capability of the cell to supply a certain value of current for a certain
time. It is measured in ampere hour (A h).
 It is independent of rate of charging of cell.

victory R. SARAVANAN. M.Sc., M.Phil., B.Ed PG ASST [PHYSICS], GBHSS, PARANGIPETTAI - 608 502
12 PHYSICS UNIT - 2 CURRENT ELECTRICITY CONCEPTUAL QUESTIONS & ANSWERS
NAME : EXAM NO 8. In India electricity is supplied for domestic use at 220 V. It is supplied at 110 V
in USA. If the resistance of a 60W bulb for use in India is R, the resistance of a
Time - 2 : 30 hours UNIT - 2 CURRENT ELECTRICITY Total - 60 marks 60W bulb for use in USA will be
(a) R ` (b) 2 R
PART - I 15 X 1 = 15 𝑅 𝑅
Note : (i) Answer all the questions (c) (d)
4 2
(ii) Choose the best answer and write the option code and 9. In a large building, there are 15 bulbs of 40 W, 5 bulbs of 100 W, 5 fans of 80 W
corresponding answer and 1 heater of 1k W are connected. The voltage of electric mains is 220 V. The
1. The following graph shows current versus voltage values maximum capacity of the main fuse of the building will be
of some unknown conductor. What is the resistance of (a) 14 A (b) 8 A
this conductor? (c) 10 A (d) 12 A
(a) 2 ohm 10. There is a current of 1.0 A in the circuit shown below. What is the resistance of P ?
(b) 4 ohm (a) 1.5 Ω
(c) 8 ohm (b) 2.5 Ω
(d)1 ohm (c) 3.5 Ω
(d) 4.5 Ω
2. A wire of resistance 2 ohms per meter is bent to form a
circle of radius 1m. The equivalent resistance between
its two diametrically opposite points, A and B as shown 11. What is the current drawn out from
in the figure is the battery?
𝜋 (a) 1A
(a) 𝜋 Ω (b) Ω
2 ( b) 2A
𝜋
(c) 2 𝜋 Ω (d)
4
Ω (c) 3A
(d) 4A
3. A toaster operating at 240 V has a resistance of 120 Ω. Its power is
(a) 400 W (b) 2 W
(c) 480 W (d) 240 W
4. A carbon resistor of (47 ± 4.7) k Ω to be marked with rings of different colours 12. The temperature coefficient of resistance of a wire is 0.00125 per °C. At 20°C,
for its identification. The colour code sequence will be its resistance is 1 Ω. The resistance of the wire will be 2 Ω at
(a) Yellow – Green – Violet – Gold (b) Yellow – Violet – Orange – Silver (a) 800 °C (b) 700 °C
(c) Violet – Yellow – Orange – Silver (d) Green – Orange – Violet - Gold (c) 850 °C (d) 820 °C
5. What is the value of resistance of the following resistor? 13. The internal resistance of a 2.1 V cell which gives a current of 0.2 A through a
(a)100 k Ω (b)10 k Ω resistance of 10 Ω is
(c) 1k Ω (d)1000 k Ω (a) 0.2 Ω (b) 0.5 Ω
6. Two wires of A and B with circular cross section are made up of the same (c) 0.8 Ω (d) 1.0 Ω
material with equal lengths. Suppose RA = 3 RB, then what is the ratio of radius 14. A piece of copper and another of germanium are cooled from room
of wire A to that of B? temperature to 80 K. The resistance of
(a) 3 (b) √3 (a) each of them increases
1 1 (b) each of them decreases
(c) (d) (c) copper increases and germanium decreases
√3 3
7. A wire connected to a power supply of 230 V has power dissipation P1. Suppose (d) copper decreases and germanium increases
the wire is cut into two equal pieces and connected parallel to the same power 15. In Joule’s heating law, when R and t are constant, if the H is taken along the y
𝑷𝟐 axis and I2 along the x axis, the graph is
supply. In this case power dissipation is P2 . The ratio is
𝑷𝟏 (a) straight line (b) parabola
(a)1 (b) 2 (c) circle (d) ellipse
(c) 3 (d) 4
victory R. SARAVANAN. M.Sc., M.Phil., B.Ed PG ASST [PHYSICS], GBHSS, PARANGIPETTAI - 608 502
12 PHYSICS UNIT - 2 CURRENT ELECTRICITY CONCEPTUAL QUESTIONS & ANSWERS
PART - II 6 X 2 = 12
Note : (i) Answer any 5 of the following questions .
(ii) Question No. 17 is compulsory
16. Distinguish between drift velocity and mobility.
17. Define current density. Give its unit.
18. Define temperature coefficient of resistance. Give its unit.
19. Define Superconductivity.
20. State Kirchoff’s second law (voltage rule or loop rule)
21. Distinguish electric energy and electric power.
22. Define Seebeck effect.
23. A copper wire of 10–6 m2 area of cross section, carries a current of 2 A. If the number
of free electrons per cubic meter in the wire is 8 × 10 28, calculate the current density
and average drift velocity of electrons. கனியிடை ஏறிய சுடையும் - முற்றல் கடையிடை ஏறிய சாறும்
பனிமலர் ஏறிய ததனும் - காய்ச்சுப் பாகிடை ஏறிய சுடையும்
PART - III 6 X 3 = 18 நனிபசு பபாழியும் பாலும் - பதன்டை நல்கிய குளிரிை நீரும்,
Note : (i) Answer any 5 of the following questions . இனியை என்தபன் எனினும் - தமிடை என்னுயிர் என்தபன்
(ii) Question No. 24 is compulsory
24. Derive the relation between the drift velocity and the current. கண்டீர்
25. Write a note on carbon resistors.
26. Write a note on electric cells in parallel. நன்கு பழுத்த பழத்தின் சுளையில் இனிளை உண்டு- முற்றிய கரும்பு
27. Explain Thomson effect. சாற்றிலும் இனிளை உண்டு - ைலர் தரும் ததனிலும் இனிளைஉண்டு -
28. Obtain the macroscopic form of Ohm’s law from its microscopic form காய்ச்சிய வெல்லப்பாகிலும் இனிளை உண்டு - பசு தரும் பாலிலும்
29. Resistance of a material at 200C and 400C are 45 Ω and 85 Ω respectively. Find its இனிளை உண்டு - வதன்ளையின் இைநீரிலும் இனிளை உண்டு ஆைாலும்,
temperature coefficient of resistivity தமிழ் வைாழியாைது என் உயிராக உள்ைது. இந்த சுளெகளை நுகர உயிர்
30. The resistance of a wire is 20 Ω. What will be new resistance, if it is stretched தெண்டும் அல்லொ.
uniformly 8 times its original length?
தமிழின் இனிடம - பாதைந்தர் பாரதிதாசன்
PART - IV 3 X 5 = 15
Note : (i) Answer all the questions
31. Explain the equivalent resistance of a series and parallel resistor network.
(OR)
Explain the determination of the internal resistance of a cell using voltmeter.
32. Obtain the condition for bridge balance in Wheatstone’s bridge.
(OR)
Explain the determination of unknown resistance using meter bridge.
33. Explain the method of measurement of internal resistance of a cell using
potentiometer.
(OR)
Describe the microscopic model of current and obtain general form of Ohm’s law.

victory R. SARAVANAN. M.Sc., M.Phil., B.Ed PG ASST [PHYSICS], GBHSS, PARANGIPETTAI - 608 502
பசித்திரு (Be hungry) தனித்திரு (Be individual) விழித்திரு (Be conscious)

HIGHER SECONDARY SECOND YEAR - PHYSICS

NAME :
STANDARD : 12 SECTION :
SCHOOL :
EXAM NO :

victory R. SARAVANAN. M.Sc, M.Phil, B.Ed.,


PG ASST (PHYSICS)
GBHSS, PARANGIPETTAI - 608 502
12 PHYSICS UNIT - 3 MAGNETISM AND MAGNETIC EFFECTS OF CURRENT CONCEPTUAL QUESTIONS & ANSWERS

PART - I ONE MARK QUESTIONS & ANSWERS WITH SOLUTIONS 4. A circular coil of radius 5 cm and 50 turns carries a current of 3 ampere. The
magnetic dipole moment of the coil is nearly
1. The magnetic field at the centre O of the following
(a) 1.0 A m2
current loop is (b) 1.2 A m2
𝜇𝑜 𝐼 𝜇𝑜 𝐼
(a)
4𝑟
 (b) 4𝑟
 (c) 0.5 A m2
𝜇𝑜 𝐼 𝜇 𝐼 (d) 0.8 A m2
(c)  (d) 2𝑜𝑟 
2𝑟 -Solution :-
-Solution :-  Magnetic dipole moment ;
𝝁𝒐 𝑰
 Magnetic field at the centre of current carrying circular loop, 𝑩 = 𝑀 = 𝐼 𝐴 = 𝑛 𝑖 𝜋𝑟 2 = 50 𝑋 3 𝑋 3.14 𝑋 (5 𝑋 10−2 )2 = 1.17 𝐴𝑚2 ≅ 1.2 𝐴𝑚2
𝟐𝒓
 Hence at the centre of semi-circular loop, 𝐵1 =
𝑩
=
𝝁𝒐 𝑰 Answer (b) 𝟏. 𝟐 𝑨 𝒎𝟐
𝟐 𝟒𝒓
5. A thin insulated wire forms a plane spiral of N = 100 tight turns carrying a
 By Maxwell’s right hand rule, this magnetic field points into ( ) the page
𝝁𝒐 𝑰
current I = 8 m A (milli ampere). The radii of inside and outside turns are
Answer (a)  a = 50 mm and b = 100 mm respectively. The magnetic induction at the centre
𝟒𝒓
of the spiral is
2. An electron moves in a straight line inside a
(a) 5 μT
charged parallel plate capacitor of uniform
(b) 7 μT
charge density σ. The time taken by the electron
(c) 8 μT
to cross the parallel plate capacitor undeflected
(d) 10 μT
when the plates of the capacitor are kept under
-Solution :-
constant magnetic field of induction ⃗𝑩
⃗ is
 The magnetic induction at the centre of solenoid,
𝑒𝑙𝐵 𝑙𝐵 𝑙𝐵 𝑙𝐵
(a) 𝜀𝑜 (b) 𝜀𝑜 (c) 𝜀𝑜 (d) 𝜀𝑜 𝜇 𝑁𝐼 4 𝜋 𝑋 10−7 𝑋 100 𝑋 8 𝑋 10−3 1004.8 𝑋 10−6
𝜎 𝜎𝑙 𝑒𝜎 𝜎 𝐵 = 2𝑟𝑜 = 2 𝑋 75 𝑋 10−3
= 150
= 6.698 𝜇 𝑇 ≅ 7𝜇𝑇
-Solution :- 𝑎𝑣𝑒𝑟𝑎𝑔𝑒

 Electric field between the plates ; 𝐸 =


𝜎 Answer (b) 𝟕 𝝁𝑻
𝜀𝑜
6. Three wires of equal lengths are bent in the form of loops. One of the loops is
 When electron moves in straighgt line with out any deflection ; 𝑒 𝐸 = 𝐵 𝑒 𝑣
𝐸 𝜎 circle, another is a semi-circle and the third one is a square. They are placed in
 Thus the velocity of an electron ; 𝑣 = = a uniform magnetic field and same electric current is passed through them.
𝐵 𝜀𝑜 𝐵
𝑙 𝑙 𝜀𝑜 𝐵 Which of the following loop configuration will experience greater torque ?
 Hence the time taken to cross the length 𝑙 with velocity 𝑣 is ; 𝑡 = =
𝑣 𝜎 (a) Circle
𝒍𝑩 (b) Semi-circle
Answer (d) 𝜺𝒐 𝝈
(c) Square
3. A particle having mass m and charge q accelerated through a potential (d) All of them
difference V. Find the force experienced when it is kept under perpendicular -Solution :-
⃗⃗ .
magnetic field 𝑩  Let 𝑙 be the length of the wire. The maximum torque ; 𝜏𝑚𝑎𝑥 = 𝐵 𝐼 𝐴
3
2𝑞 𝐵𝑉 3 2
𝑞 𝐵 𝑉 3 2
2𝑞 𝐵 𝑉 3
2𝑞 𝐵𝑉  Here ‘B’ and ‘I’ are constants. So value of maximum torque ; 𝜏𝑚𝑎𝑥 ∝ 𝐴
(a)√ (b) √ (c) √ (d) √ 𝑙 𝜋 𝑙2 𝑙2
𝑚 2𝑚 𝑚 𝑚 3
 For circular loop, 𝑙 = 2 𝜋 𝑟 ⇒ 𝑟 = and 𝐴𝑐𝑖𝑟 = 𝜋 𝑟 2 = 2 =
2𝜋 4𝜋 4𝜋
-Solution :- 𝑙 𝜋 𝑟2 𝜋 𝑙2
 For semi-circle loop, 𝑙 = 𝜋 𝑟 + 2 𝑟 ⇒ 𝑟 = and 𝐴𝑠𝑒𝑚𝑖 = =
2𝑞𝑉 𝜋+2 2 2 (𝜋+2)2
 Velocity of the charged particle ; 𝑣 = √ 𝑙 𝑙2
𝑚  For square, 𝑙 = 4 𝑎 ⇒ 𝑎 = and 𝐴𝑠𝑞𝑢𝑎𝑟𝑒 = 𝑎2 =
4 16
2𝑞𝑉 2 𝑞3 𝐵2 𝑉  Since 𝐴𝑐𝑖𝑟 > 𝐴𝑠𝑒𝑚𝑖 > 𝐴𝑠𝑞𝑢𝑟𝑒 , hence circle experience greater torque.
 Lorentz force ; 𝐹 = 𝐵 𝑞 𝑣 = 𝐵 𝑞 √ = √
𝑚 𝑚
Answer (a) circle
𝟑 𝟐
Answer (c) √𝟐𝒒 𝑩 𝑽
𝒎

victory R. SARAVANAN. M.Sc., M.Phil., B.Ed PG ASST [PHYSICS], GBHSS, PARANGIPETTAI - 608 502
12 PHYSICS UNIT - 3 MAGNETISM AND MAGNETIC EFFECTS OF CURRENT CONCEPTUAL QUESTIONS & ANSWERS

7. Two identical coils, each with N turns and radius R are 10. A non-conducting charged ring carrying a charge of q, mass m and radius r is
placed coaxially at a distance R as shown in the figure. If rotated about its axis with constant angular speed ω. Find the ratio of its
I is the current passing through the loops in the same magnetic moment with angular momentum is
direction, then the magnetic field at a point P at a 𝑞 2𝑞
(a) (b)
distance of R/2 from the centre of each coil is 𝑚 𝑚
𝑞 𝑞
8 𝑁 𝜇𝑜 𝐼 8 𝑁 𝜇𝑜 𝐼 (c) (d)
(a) (b) 2𝑚 4𝑚
√5 𝑅 53/2 𝑅 -Solution :-
8 𝑁 𝜇𝑜 𝐼 4 𝑁 𝜇𝑜 𝐼
(c) (d) 𝑞 𝑞 𝜋 𝑟2 𝑞 𝑟2 𝜔
5𝑅 √5 𝑅  Magnetic moment ; 𝑝𝑚 = 𝐼 𝐴 =
𝑇
𝜋 𝑟2 = 2𝜋/𝜔
= 2
-Solution :-
 Magnetic field due to two coils,  Angular momentum ; 𝐿 = 𝑚 𝑟 2 𝜔
𝑞 𝑟2 𝜔
𝜇 𝑁 𝐼 𝑅2 𝜇𝑜 𝑁 𝐼 𝑅2 𝜇𝑜 𝑁 𝐼 𝑅2 8 𝜇𝑜 𝑁 𝐼 𝑅2 8 𝜇𝑜 𝑁 𝐼 𝑅2 8 𝜇𝑜 𝑁 𝐼 ( )
𝐵 = 2 [2 (𝑅𝑜2 +𝑍2 )3/2 ] = 3/2 = 3/2 = (5 𝑅2 )3/2
= (5)3/2 𝑅3
= (5)3/2 𝑅  Hence the ratio,
𝑝𝑚
=
2
=
𝑞
𝑅2 5 𝑅2 𝐿 𝑚 𝑟2 𝜔 2𝑚
(𝑅2 + ) ( )
4 4 𝒒
𝟖 𝝁𝒐 𝑵 𝑰 Answer (c) 𝟐𝒎
Answer (b) 𝟓𝟑/𝟐 𝑹 11. The BH curve for a ferromagnetic
8. A wire of length l carrying a current I along the Y direction is kept in a material is shown in the figure. The
⃗ = 𝜷 (𝒊̂ + 𝒋̂ + 𝒌
magnetic field given by ⃗𝑩 ̂ ) . The magnitude of Lorentz force material is placed inside a long solenoid
√𝟑
acting on the wire is, which contains 1000 turns/cm. The
current that should be passed in the
2 1 1
(a) √ 𝛽 𝐼 𝑙 (b) √ 𝛽 𝐼 𝑙 (c) √2 𝛽 𝐼 𝑙 (d) √ 𝛽 𝐼 𝑙 solenonid to demagnetize the
3 3 3
ferromagnet completely is
-Solution :- (a) 1.00 m A
 The force on current carrying wire placed in magnetic field, (b) 1.25 mA
𝛽 𝐼𝑙𝛽
⃗⃗⃗𝐹 = 𝐼 ⃗⃗⃗𝑙 𝑋 𝐵
⃗ = 𝐼 𝑙 ̂𝑗 𝑋 (𝑖̂ + 𝑗̂ + 𝑘̂ ) = (−𝑘̂ + 0 + 𝑖̂) (c) 1.50 mA
√3 √3
(d) 1.75 mA
𝐼𝑙𝛽 𝐼𝑙𝛽 2
 Hence its magnitude, 𝐹 = √3
√1 + 1 = √3 √2 = √3 𝛽 𝐼 𝑙 -Solution :-
 By definition, 𝐵 = 𝜇𝑜 𝐻
𝟐  By Ampere circuital law, 𝐵 = 𝜇𝑜 𝑛 𝐼
Answer (a) √ 𝜷𝑰𝒍
𝟑  Thus we have, 𝐻 = 𝑛 𝐼
9. A bar magnet of length l and magnetic moment pm is  From the figure, the coercivity ; 𝐻 = 150 𝐴 𝑚−1
bent in the form of an arc as shown in figure. The new 𝐻 150
∴ 𝐼= = = 15 𝑋 10−4 = 1.5 𝑋 10−3 𝐴 = 1.5 𝑚 𝐴
magnetic dipole moment will be 𝑛 1,00,000

(a) 𝑝𝑚 (b)
3
𝑝𝑚 Answer (c) 𝟏. 𝟓𝟎 𝒎 𝑨
𝜋
2 1 12. Two short bar magnets have magnetic moments 1.20 Am 2 and 1.00 Am2
(c)
𝜋
𝑝𝑚 (d)
2
𝑝𝑚 respectively. They are kept on a horizontal table parallel to each other with
-Solution :- their north poles pointing towards south. They have a common magnetic
 The initial magnetic dipole moment ; 𝑝𝑚 = 𝑞𝑚 𝑋 𝑙 equator and are separated by a distance of 20.0 cm. The value of the resultant
𝜋
 Since the angle subtended by the arc is 60° (𝑜𝑟) , the horizontal magnetic induction at the mid-point O of the line joining their
3
𝜋 3𝑙 centres is (Horizontal components of Earth’s magnetic induction is
length of the arc ; 𝑙 = 𝑟 ( ) and radius ; 𝑟 = 3.6 × 10–5 Wb m–2 )
3 𝜋
 The new magnetic moment ; (a) 3.60 × 10–5 Wb m–2 (b) 3.5 × 10–5 Wb m–2
1 1 3𝑙 3 (c) 2.56 × 10 Wb m
–4 –2 (d) 2.2 × 10–4 Wb m–2
𝑀 = 𝑞𝑚 𝑋 2 𝑟 sin 30° = 𝑞𝑚 𝑋 2 𝑟 (2) = 𝑞𝑚 𝑋 𝑟 = 𝑞𝑚 𝑋 𝜋
= 𝜋
𝑝𝑚
𝟑
Answer (b) 𝒑𝒎
𝝅
victory R. SARAVANAN. M.Sc., M.Phil., B.Ed PG ASST [PHYSICS], GBHSS, PARANGIPETTAI - 608 502
12 PHYSICS UNIT - 3 MAGNETISM AND MAGNETIC EFFECTS OF CURRENT CONCEPTUAL QUESTIONS & ANSWERS

-Solution :- 15. The potential energy of magnetic dipole whose dipole moment is
 Magnetic field due the short bar magnets along its ⃗⃗⃗𝒑𝒎 = (−𝟎. 𝟓 𝒊̂ + 𝟎. 𝟒 𝒋̂) 𝑨𝒎𝟐 kept in uniform magnetic field 𝑩 ⃗⃗ = 𝟎. 𝟐 𝒊̂ 𝑻
equatorial line, (a) –0.1 J (b) –0.8 J (c) 0.1 J (d) 0.8 J
𝜇𝑜 𝑀1 𝜇𝑜 𝑀2
𝐵1 = and 𝐵2 = -Solution :-
4 𝜋 𝑟3 4 𝜋 𝑟3
 The potential energy of magnetic dipole is ;
 Let horizontal component of earth’s magnetic field is 𝐵𝐻
⃗ = − (−0.5 𝑖̂ + 0.4 𝑗̂) . (0.2 𝑖̂) = −(−0.5)(0.2) = + 0.1 𝐽
𝑈 = − ⃗⃗⃗𝑝𝑚 . 𝐵
 Hence resultant horizontal magnetic induction will be,
𝐵 = 𝐵1 + 𝐵2 + 𝐵𝐻 =
𝜇𝑜
(𝑀1 + 𝑀2 ) + 𝐵𝐻 Answer (d) 𝟎. 𝟏 𝑱
4 𝜋 𝑟3
4 𝜋 𝑋 10−7
𝐵= (1.2 + 1) + 3.6 X 10−5
4 𝜋 𝑋 (10 𝑋 10−2 )3
𝐵 = 2.2 X 10−4 + 0.36 X 10−4 = 2.56 𝑋 10−4 wb m–2 (or) tesla
Answer (c) 𝟐. 𝟓𝟔 𝑿 𝟏𝟎−𝟒 wb m–2
13. The vertical component of Earth’s magnetic field at a place is equal to the
horizontal component. What is the value of angle of dip at this place?
(a) 30o (b) 45o (c) 60o (d) 90o
-Solution :-
𝐵𝐻
 Angle of dip ; 𝛿 = tan−1 ( )
𝐵𝑉
−1 (1)
 If 𝐵𝐻 = 𝐵𝑉 , then 𝛿 = tan = 45°
Answer (b) 𝟒𝟓°
14. A flat dielectric disc of radius R carries an excess charge on its surface. The
surface charge density is σ. The disc rotates about an axis perpendicular to its
plane passing through the centre with angular velocity ω. Find the magnitude
of the torque on the disc if it is placed in a uniform magnetic field whose
strength is B which is directed perpendicular to the axis of rotation
1 1
(a)
4
𝜎𝜔𝜋𝐵𝑅 (b) 4 𝜎 𝜔 𝜋 𝐵 𝑅 2
1 1
(c)
4
𝜎 𝜔 𝜋 𝐵 𝑅3 (d)
4
𝜎 𝜔 𝜋 𝐵 𝑅4
-Solution :-
 The charge on the ring of radius 𝑥 and thickness
𝑑𝑥 is, 𝑑𝑞 = 𝜎 𝑑𝐴 = 𝜎 2 𝜋 𝑥 𝑑𝑥
 The current due this charge,
𝑑𝑞 𝜎 2 𝜋 𝑥 𝑑𝑥
𝑑𝑖 = 𝑇
= 2 𝜋/𝜔
= 𝜎 𝜔 𝑥 𝑑𝑥
 Torque on the disc of radius 𝑥 ;
𝑑𝜏 = 𝐵 𝑑𝑖 𝑎 = 𝐵 𝜎 𝜔 𝑥 𝑑𝑥 𝜋 𝑥 2 = 𝐵 𝜎 𝜔 𝜋 𝑥 3 𝑑𝑥
 Then the net torque on the entire disc of radius R is,
𝑅
𝑅 𝑥4 1 4
𝜏 = ∫0 𝐵 𝜎 𝜔 𝜋 𝑥 3 𝑑𝑥 = 𝐵 𝜎 𝜔 𝜋 [ ] = 𝐵𝜎𝜔𝜋𝑅
4 0 4
𝟏
Answer (d) 𝑩 𝝈 𝝎 𝝅 𝑹𝟒
𝟒

victory R. SARAVANAN. M.Sc., M.Phil., B.Ed PG ASST [PHYSICS], GBHSS, PARANGIPETTAI - 608 502
12 PHYSICS UNIT - 3 MAGNETISM AND MAGNETIC EFFECTS OF CURRENT CONCEPTUAL QUESTIONS & ANSWERS

PART - II TWO MARK VERY SHORT ANSWER QUESTIONS & ANSWERS  Also, 𝑩𝑬 = √𝑩𝑯𝟐 + 𝑩𝑽𝟐
1. Define magnetism. Give its applications. 10. Define pol strength of the magnet.
 The property of attracting iron is called magnetism.  The attracting property of the magnet is concentrated at its poles only and this
 In olden days, magnets were used as magnetic compass for navigation, property is called pole strength (𝑞𝑚 ).
magnetic therapy for treatment and magic shows.  The S.I unit of pole strength is 𝑨 𝒎
 In modern days most of the things we use in daily life contains magnets. For 11. Define magnetic dipole moment.
example loud speaker, motors, dynamo, cell phones, pendrive, CD, hard disc in  Manetic dipole moment (𝒑𝒎 ) is defined as the product of the pole strength (𝑞𝑚 )
laptop etc and magnetic length (2 𝑙). i.e 𝒑𝒎 = 𝒒𝒎 𝟐 𝒍
2. Define Giomagnetism or Terrestrial magnetism.  In vector notation ; ⃗⃗⃗𝒑𝒎 = 𝒒𝒎 ⃗⃗⃗𝒅 [∵ |⃗⃗⃗𝑑 | = 2 𝑙]
 The branch of physics which deals with the Earth’s magnetic field is called 𝟐
 Its S.I unit is 𝐴 𝒎 . Its direction is from South pole to North poke.
Geomagnetism . 12. Define magnetic field.
3. What are the elements of the Earth’s magnetic field?  The magnetic field ⃗⃗⃗𝐵 at a point is defined as a force experienced by the bar
 To specify the Earth’s magnetic field, three quantities must be requied. They are magnet of unit pole strength.
(i) Magnetic declination (D) ⃗𝑭
(ii) Magnetic dip or inclination (I) ⃗⃗⃗𝑩 =
(iii) The horizontal component of the Earth’s magnetic field (BH) 𝒒𝒎
4. Define geographic meridian and magnetic meridian.  Its S.I unit is 𝑵 𝑨−𝟏 𝒎−𝟏
 A vertical plane passing through the geographic axis is called geographic 13. What are the types of magnet?
meridian and a great circle perpendicular to Earth’s geographic axis is called  Magnets are classified in to natural magnets and artificial magnets.
geographic equator.  Iron, cobalt, nickel etc are natural magnets. Strength of natural magnets are
 A vertical plane passing throuth magnetic axis is called magnetic meridian and very weak and the shape of the magnet are irregular.
a great circle perdicular to Earth’s magnetic axis is called magnetic equator.  Artificial magnets are made our desired shape and strength. Bar magnets,
5. Define magnetic declination. cylindrical magnets, horse shoe magnets are some examples for artificial
 The angle between magnetic meridian at a point and geographical meridian is magnets.
called the magnetic declination (D). 14. Define magnetic flux. Give its unit.
 At higher latitudes, the declination is greater whereas near the equator, the  the number of magnetic field lines crossing per unit area is called magnetic flux
declination is smaller. (Φ𝐵 )
6. For Chennai, the magnetic declination angle is − 𝟏°𝟖. Why it is negative? 𝚽𝑩 = ⃗𝑩 ⃗ . ⃗𝑨
⃗ = 𝑩 𝑨 𝐜𝐨𝐬 𝜽
 The negative sign indicates, that the magnetic meridian lies west to the  The S.I unit of magnetc flux is weber (Wb) and C.G.S unit is maxwell (1 Wb =
geographic meridian. 108 maxwell)
7. Define magnetic inclination or dip.  Its dimentional formula is [ML2T-2A-1]
 The angle subtended by the Earth’s total magnetic field 𝐵 ⃗ with the horizontal 15. Define magnetic flux density.
direction in the magnetic meridian is called dip or magnetic inclination (I)  The magnetic flux density can be defined as the number of magnetic field lines
 For Chennai, angle of dip is 14°16 crossing unit area kept normal to the direction of line of force.
8. Define horizontal component of Earth’s magnetic field.  Its S.I unit is tesla or 𝑾𝒃 𝒎−𝟐
 The componenet of Earth’s magnetic field along the horizontal direction in the 16. Distinguish between uniform and non-uniform magnetic field.
magnetic meridian is called horizontal component of Earth’s magnetic field (𝐵𝐻 ) Uniform magnetic field Non-uniform magnetic field
9. Calculate the tangent of magnetic inclination or angle of dip. 1) Magnetic field is said to be uniorm 1) Magnetic field is said to be non-
 Let 𝐵𝐸 be the net Earth’s magnetic field at a point ‘P’ and ‘I’ be the angle of If it has the same magnitude and uniform If the magnitude or
dip, then, Horizontal component ; 𝐵𝐻 = 𝐵𝐸 cos 𝐼 direction at all the points in a given direction or both varies at all its
Vertical componenet ; 𝐵𝑉 = 𝐵𝐸 s𝑖𝑛 𝐼 region. points.
𝐵𝐸 s𝑖𝑛 𝐼 𝐵𝐻 𝑩𝑯 2) (e.g) Locally Earth’s magnetic field 2) (e.g) Magnetic field of a bar
∴ = (𝑜𝑟) 𝐭𝐚𝐧 𝑰 = is uniform magnet
𝐵𝐸 cos 𝐼 𝐵𝑉 𝑩𝑽

victory R. SARAVANAN. M.Sc., M.Phil., B.Ed PG ASST [PHYSICS], GBHSS, PARANGIPETTAI - 608 502
12 PHYSICS UNIT - 3 MAGNETISM AND MAGNETIC EFFECTS OF CURRENT CONCEPTUAL QUESTIONS & ANSWERS

17. Discuss the types of force between two magnetic pole strength. 25. Define magnetic induction or total magnetic field.
 When north pole (N) of magnet A and north pole (N) of magnet B or south pole  The magnetic induction (⃗⃗⃗𝐵 ) inside the specimen is equal to the sum of the
(S) of magnet A and south pole (S) of magnet B are brought close together, they magnetic field (𝐵 ⃗ 𝑜 ) produced in vacuum due to magnetizing field and the
repels each other. ⃗ 𝑚 ) due to the induce magnetization of the substance,
magnetic field (𝐵
 On the other hand, when north pole of magnet A and south pole of magnet B or
south pole of magnet A and north pole of magnet B are brought close together [⃗⃗⃗𝐵 = 𝐵⃗𝑜 +𝐵 ⃗ 𝑚]
they attracts each other. 26. Define magnetic susceptibility.
 Thus like poles repels and unlike poles attracts.  Magnetic susceptibility (𝜒𝑚 ) is defined as the ratio of the intensity of
18. State Coulomb’s inverse square law of magnetism. magnetization (𝑀 ⃗⃗ ) induced in the material due to the magnetizing field (𝐻
⃗)
 The force of attraction or repulsion between two magnetic poles is directly  It is a dimensionless quantity.
proportional to the product of their pole strengths and inversely proportional 27. What are the classification of magnetic materials?
to the square of the distance between them.  Magnetic materials are generally classified in to three types. They are
19. What happens when a bar magnet is freely suspended in uniform and non- (i) Diamagnetic material
uniform magnetic field? (e.g.) bismuth, copper, water
 Even though Earth has non- uniform magnetic field, it is locally (at particular (ii) Paramagnetic material
place) taken as uniform. So bar magnet suspended freely in unifom magnetic (e.g.) Aluminum, platinum, chromium
field experience only torque (rotational motion) (iii) Ferro magnetic material
 When a bar magnet is freely suspended in non-uniform magnetic field, it (e.g.) Iron, nickel, cobalt
undergo translator motion due to net force and rotational motion due to torque. 28. Define Meissner effect.
20. State tangent law.  Super conductors are perfect diamagnetic materials.
 When a magnetic needle or magnet is freely suspended in two mutually  The exclusion of magnetic flux from a super conductor during its transition to
perpendicular uniform magnetic fields, it will come to rest in the direction of the superconducting state is known as Meisnner effect
the resultant of the two fields. 29. Define Curie’s law.
21. Define magnetizing field.  The susceptibility of the material is inversely proportional to its kelvin
 The magnetic field which is used to magnetize a sample or specimen is called temperature. (i.e.)the magnetic susceptibility decreases with increase in
the magnetizing field (𝐻 ⃗ ). Its unit is 𝑨 𝒎−𝟏 temperature.
𝟏 𝑪
22. Define magnetic permeability. 𝝌𝒎 ∝ (𝒐𝒓) 𝝌𝒎 =
 Magnetic permeability is defined as the measure of ability of the material to 𝑻 𝑻
allow the passage of magnetic lines through it or measure of the capacity of the  Where C  curie constant. This is called Curie law
substance to take magnetization or the degree of penetration of magnetic field 30. Define curie temperature.
through the substance.  As temperature increases, the ferromagnetism decreases due to the inceased
23. Define relative permeability. thermal agitation of the atomic dipoles.
 The relative permeability (𝝁𝒓 ) is defined as the ratio between absolute  At a particular temperature, ferromagnetic material becomes paramagnetic.
permeability (𝝁) of the medium to the permeability of free space (𝝁𝒐 ). This temperature is known as Curie temperature (𝑇𝐶 ).
𝝁 31. State Curie - Weiss law.
𝝁𝒓 =  The susceptibility of the material above the Curie temperature is given by
𝝁𝒐
𝐶
 It has no unit and it is dimensionless quantity. 𝜒𝑚 =
24. Define intensity of magnetization. 𝑇 − 𝑇𝑜
 The net magnetic moment per unit volume of the material or is known as where, C  Curie law ; T  Kelvin temperature
intensity of magnetization or magnetization vector or magnetization.  This relation is called Curie - Weiss law.
 For magnet the intensity of magnetization can be defined as the pole strength 32. What is Hysteresis?
per unit area  Hysterisis means ‘lagging behind’
𝑝𝑚 𝑞𝑚  The phenomenon of lagging of magnetic induction (B) behind the magnetizing
𝑀= = field (H) is called hysteresis.
𝑉 𝐴
 Its unit 𝑨 𝒎−𝟏 . It is a vector quantity

victory R. SARAVANAN. M.Sc., M.Phil., B.Ed PG ASST [PHYSICS], GBHSS, PARANGIPETTAI - 608 502
12 PHYSICS UNIT - 3 MAGNETISM AND MAGNETIC EFFECTS OF CURRENT CONCEPTUAL QUESTIONS & ANSWERS

33. Define hysteresis loss. 43. Define one tesla.


 During the magnetization of the specimen through a cycle, there is loss of  The strength of the magnetic field is one tesla if unit charge moving in it with
energy in the form of heat. This is known as hysteresis loss. unit velocity experiences unit force.
 The energy lost per unit volume of the material when it is carried through one 44. What are the limitations of cyclotron?
cycle of magnetization is equal to the area of the hysteresis loop.  The speed of the ion is limited.
34. What are the types of ferromagnetic materials?  Electron cannot be accelerated.
 Based on the shape and size of the bysterisis loop, ferromagnetic materials are  Uncharged particles cannot be accelerated.
classified as two types. They are 45. Write a note on fast-neutron cancer therapy.
(i) Hard magnetic material - (e.g) steel  When a deuteron is bombarded with a beryllium target, a beam of high energy
(ii) Soft magnetic material - (e.g) soft irom neutrons are produced.
35. State right hand thumb rule.  These high energy neutrons are sent into the patient’s cancerous region to
 If we hold the current carrying conductor in our right hand such that the thumb break the bonds in the DNA of the cancer cells.
points in the direction of current flow, then the fingers encircling the wire  This is used in treatment of fast-neutron cancer therapy.
points in the direction of the magnetic field lines produced. 46. State Flemming’s left hand rule (FLHR).
36. State Maxwell’s right hand cork screw rule.  Stretch fore finger, the middle finger and the thrumb of the left hand in mutully
 This rule is used to determine the direction of the magnetic field. perpendicular directions. If,
 If we advance a right handed screw along the direction of current, then the (i) fore finger points the direction of magnetic field,
direction of rotation of the screw gives the direction of the magnetic field. (ii) the middle finger points the direction of the electric current, then
37. Define magnetic dipole moment of current loop. (iii) thumb will point the direction of the force experienced by the conductor.
 The magnetic dipole moment of any current loop is equal to the product of the 47. Define one ampere.
current and area of the loop. [⃗⃗⃗𝑝𝑚 = 𝐼 ⃗⃗⃗𝐴]  One ampere is defined as that current when it is passed through each of the two
38. State right hand thumb rule. infinitely long parallel straight conductors kept at a distance of one metre apart
 This rule is used to determine the direction of magnetic moment. in vacuum caused each conductor to experience a force of 2 𝑋 10−7 newton per
 If we curl the fingers of right hand in the direction of current in the loop, then metre length of conductor.
the stretched thumb gives the direction of the magnetic moment associated 48. Define figure of merit of a galvanometer.
with the loop.  It is defined as the current which produces a deflection of one scale division in
39. Define gyro-magnetic ratio. the galvanometer.
 The ratio of magnetic moment (𝜇𝐿 ) of the electron to its angular momentum (L) 49. Define current sensitivity of a galvanometer.
is called gyro-magnetic ratio.  It is defined as the deflection produced per unit current flowing through it.
𝜇𝐿 𝑒 𝜽 𝑵𝑩𝑨 𝟏
= = 8.78 𝑋 1010 𝐶 𝑘𝑔−1 𝑰𝑺 = = =
𝐿 2𝑚 𝑰 𝑲 𝑮
40. Define Bohr magneton. 50. How the current sensitivity of galvanometer can be increased?
 It is the unit of atomic magnetic moment.  By increasing the number of turns N
 The minimum value of atomic magnetic moment is called Bohr magneton.  By increasing the magnetic induction B
𝒆𝒉  By increasing the area of the coil A
𝟏 𝒃𝒐𝒉𝒓𝒎𝒂𝒈𝒏𝒆𝒕𝒐𝒏 = 𝝁𝑩 = = 𝟗. 𝟐𝟕 𝑿 𝟏𝟎−𝟐𝟒 𝑨 𝒎𝟐  By decreasing the couple per unit twist of the suspension wire
𝟒𝝅𝒎
41. State Ampere’s circuital law. 51. Why Phosphor - bronze is used as suspension wire?
 It state that the line integral of magnetic field over a closed loop is 𝝁𝒐 times net  Because, for phosphor - bronze wire, the couple per unit twist is very small.
current enclosed by the loop. 52. Define voltage sensitivity of the galvanometer.
⃗⃗⃗⃗ = 𝝁𝒐 𝑰𝒐
⃗⃗ . 𝒅𝒍  It is defined as the deflection produced per unit voltage applied across it.
∮𝑩 𝜽 𝑵𝑩𝑨 𝟏
𝑰𝑺 = = =
42. Define Lorentz force. 𝑰 𝑲 𝑮
⃗⃗⃗ ) and magnetic field (𝐵
 If the charge is moving in the electric field (𝐸 ⃗ ), the total 53. How galvanometer can be converted in to ammeter?
force experienced by the charge is given by ⃗⃗⃗𝑭 = 𝒒 [⃗⃗⃗𝑬 + (⃗⃗⃗𝒗 𝑿 ⃗⃗⃗𝑩)]  A galvanometer is converted in to an ammeter by connecting a low resistance
 It is known as Lorentz forec. (shunt) in parallel with the galvanometer.

victory R. SARAVANAN. M.Sc., M.Phil., B.Ed PG ASST [PHYSICS], GBHSS, PARANGIPETTAI - 608 502
12 PHYSICS UNIT - 3 MAGNETISM AND MAGNETIC EFFECTS OF CURRENT CONCEPTUAL QUESTIONS & ANSWERS

54. How galvanometer can be converted in to voltmeter?


 A galvanometer is converted into a voltmeter by connecting high reistance in PART - III THREE MARK SHORT ANSWER QUESTIONS & ANSWERS
series with galvanometer. 1. What are the properties of bar magnet?
55. Why ammeter should always connected in series to the circuit? Properties of magnet:
 The ammeter must offer low resistance such that it will not change the current (i) A freely suspended bar magnet wil always point along the north - south
passing through it. So ammeter is connected in series to measure the circuit direction.
current. (ii) The attractive property of the magnet is maximum near its end or pole. This is
 An ideal ammeter has zero resistance. called pole strength.
56. Why voltmeter should always connected in parallel to the circuit? (iii) Two poles of a magnet have pole strength equal to one another.
 The voltmeter must offer high resistance so that it will not draw appreciable (iv) When a magnet is broken into pieces, each piece behave like a magnet with
current. So voltmeter is connected in paralle to measure the potential poles at its ends.
difference. (v) The length of the bar magnet is called geometrical length and length between
 An ideal voltmeter has infinite resistance. two magnetic poles in a bar magnet is called magnetic length. The magnetic
length is always slightly smaller than geometrical length. (i.e.)
magnetic length : geometrical length = 5 ∶ 6
2. Write a note on pole strength.
Pole strength :
 The attracting property of the magnet is concentrated at its poles only and this
property is called pole strength (𝑞𝑚 ).
 It is a scalar quantity with dimension [𝐿 𝐴]. Its S.I unit is 𝑨 𝒎 (𝑜𝑟) 𝑵 𝑻−𝟏
 North pole of the magnet experiences a force in the direction of the magnetic
field and south pole experiences force opposite to the magnetic field.
 Pole strength depends on the nature of materials of the magnet, area of cross-
section and the state of magnetization.
 If a magnet is cut in to two equal halves along the length, then pole strength is
reduced to half.
 If the magnet is cut into two equal halves perperdicular to the length, then pole
strength remains same.
 If we cut the magnet in to two pieces, we will not separate north and south
poles. Instead we get two magnets. (i.e) isolated mono pole does not exist in
nature
3. Give the properties of magnetic field lines.
Properties of magnetic field lines:
 They are continuous closed lines. Their direction is from North pole to South
pole outside the magnet and South pole to North pole inside the magnet.
 The tangent drawn at any point on the magnetic field lines gives the direction of
magnetic field at that point.
 They never intersect each other.
 The degree of closeness of the field lines determines the relative strength of the
magnetic field. The magnetic field is strong where magnetic field lines crowd
and weak where magnetic field lines thin out.

victory R. SARAVANAN. M.Sc., M.Phil., B.Ed PG ASST [PHYSICS], GBHSS, PARANGIPETTAI - 608 502
12 PHYSICS UNIT - 3 MAGNETISM AND MAGNETIC EFFECTS OF CURRENT CONCEPTUAL QUESTIONS & ANSWERS

4. Explain Coulomb’s inverse square law in magnetism.  Hence total force ; ⃗⃗⃗𝐹 = ⃗⃗⃗𝐹𝑁 + ⃗⃗⃗𝐹𝑆 = 0 ⃗
Coulomb’ inverse square law in magnetism :  So that there is no translator motion.
 But these two forces constitute a couple, which tends to rotate the magnet
along the direction of the field ⃗⃗⃗𝐵 .
 Hence moment of force or torque about ‘O’ is
⃗⃗𝜏 = ⃗⃗⃗⃗⃗⃗
𝑂𝑁 𝑋 ⃗⃗⃗𝐹𝑁 + ⃗⃗⃗⃗⃗
𝑂𝑆 𝑋 ⃗⃗⃗𝐹𝑆
⃗⃗𝜏 = ⃗⃗⃗⃗⃗⃗
𝑂𝑁 𝑋 𝑞𝑚 𝐵 ⃗ + 𝑂𝑆 ⃗⃗⃗⃗⃗ 𝑋 (−𝑞𝑚 𝐵 ⃗)
 Consider two bar magnets A and B as shown.
 Here, |𝑂𝑁⃗⃗⃗⃗⃗⃗ | = |𝑂𝑆
⃗⃗⃗⃗⃗ | = 𝑙 and |𝑞𝑚 𝐵 ⃗ | = |−𝑞𝑚 𝐵⃗|
 Let , Pole strength of A = 𝑄𝑚𝐴
Pole strength of B = 𝑄𝑚𝐵  Hence the magnitude of the torque,
𝜏 = 𝑙 𝑞𝑚 𝐵 sin 𝜃 + 𝑙 𝑞𝑚 𝐵 sin 𝜃
Distance between A and B = 𝑟
𝜏 = 2 𝑙 𝑞𝑚 𝐵 sin 𝜃 [𝑞𝑚 2𝑙 = 𝑝𝑚 ]
 Then by Coulomb’s law, the force of attraction or repulsion between two
𝝉 = 𝒑𝒎 𝑩 𝐬𝐢𝐧 𝜽
mannetic poles is directly proportional to the product of their pole strengths
and inversely proportional to the square of the distance between them. Hence  In vector notation, ⃗⃗𝝉 = 𝒑 ⃗ 𝒎 𝑿 ⃗𝑩 ⃗
𝑄 𝑄 𝑄 𝑄 6. Obtain an expression for potential energy of a bar magnet placed in an
⃗⃗⃗𝐹 ∝ 𝑚𝐴 𝑚𝐴 𝑟̂ (𝑜𝑟) ⃗⃗⃗𝐹 = 𝑘 𝑚𝐴 𝑚𝐴 𝑟̂ uniform magnetic field.
𝑟2 𝑟2
 In magnitude, Potential energy of a bar magnet :
𝑄𝑚 𝑄𝑚  Let a bar magnet of dipole moment ⃗⃗⃗𝑝𝑚 is
𝐹 =𝑘 𝐴2 𝐴 placed in a uniform magnetic field ⃗⃗⃗𝐵 at an
𝑟
 where, 𝑘 → proportionality constant. angle 𝜃
 In S. I unit, the value of 𝑘 is  The magnitude of the torque acting on the
𝜇𝑜 dipole is ; 𝝉 = 𝒑𝒎 𝑩 𝐬𝐢𝐧 𝜽
𝑘= ≅ 10−7 𝐻 𝑚−1
4𝜋  So work done bt external torque (𝜏𝑒𝑥𝑡 ) for
 Then the force, a small angular displacement against the
𝝁𝒐 𝑸 𝒎𝑨 𝑸𝒎𝑨 torque (𝝉) is
𝑭 =
𝟒𝝅 𝒓𝟐 𝑑𝑊 = 𝜏𝑒𝑥𝑡 𝑑𝜃 = 𝜏 𝑑𝜃 = 𝑝𝑚 𝐵 sin 𝜃 𝑑𝜃
 where, 𝜇𝑜 → permiability of free space or vacuum  Hence the total work done to rotate the bar magnet from 𝜃 to 𝜃 is ,
[𝜇𝑜 = 4 𝜋 𝑋 10−7 𝐻 𝑚−1 ] 𝜃 𝜃
5. Calculate the torque acting on a bar magnet in uniform magnetic field. 𝑊 = ∫ 𝑑𝑊 = ∫ 𝑝𝑚 𝐵 sin 𝜃 𝑑𝜃
𝜽 𝜽
Torque acting on a bar magnet :
 Consider a mannet of length ‘2𝑙’ of pole strength ‘𝑞𝑚 ’ kept in uniform magnetic 𝑊 = 𝑝𝑚 𝐵 [− cos 𝜃]𝜃𝜽 = − 𝑝𝑚 𝐵 [cos 𝜃 − cos 𝜃 ]
 This workdone is stored as potential energy of the bar magnet.
field ⃗⃗⃗𝐵 .
Hence 𝑼 = − 𝒑𝒎 𝑩 [𝐜𝐨𝐬 𝜽 − 𝐜𝐨𝐬 𝜽 ]
 If initial angle be  = 90 then,∶ 𝑼 = − 𝒑𝒎 𝑩 𝒄𝒐𝒔 𝜽
 The potential energy stored in a bar magnet placed in a uniform magnetic field
is, 𝑼 = − ⃗⃗⃗𝒑𝒎 . ⃗⃗⃗𝑩
(i) If  = 0 then, ∶ 𝑼 = − 𝒑𝒎 𝑩 = minimum
(ii) If  = 180 then, ∶ 𝑼 = 𝒑𝒎 𝑩 = maximum
 Thus the potential energy of a bar magnet is minimum when it is align along the
external field and maximum when it align anti parallel with the external field.
 Force experienced by the North pole along the direction of the field ;
⃗⃗⃗𝐹𝑁 = 𝑞𝑚 𝐵 ⃗
 Force experienced by the South pole opposite to the direction of the field ;
⃗⃗⃗𝐹𝑆 = − 𝑞𝑚 𝐵

victory R. SARAVANAN. M.Sc., M.Phil., B.Ed PG ASST [PHYSICS], GBHSS, PARANGIPETTAI - 608 502
12 PHYSICS UNIT - 3 MAGNETISM AND MAGNETIC EFFECTS OF CURRENT CONCEPTUAL QUESTIONS & ANSWERS

7. What are the precausions taken wile using tangent galvanometer (TG)  The induced moment disappears as soon as the external field is removed.
Precausions:  When placed in a non-uniform magnetic field, it has tendency to move the
 All the neaby magnets and magnetic materials are kept away from the material from stronger to weaker part of the field.
instrument.  This action is called diamagnetic action and such materials are known as
 Using sprit level, the levelling screws at the base are adjusted so that the small diamagnetic materials.
magnetic needle is exactly horizontal and also the circular coil is exactly (e.g.) Bismuth, Copper, Water
vertical. 11. Explain paramagnetism.
 The plane of the coil is kept along the magnetic meridian. Paramagnetic material :
 The pointer in the compass box should read 0° − 0°  In some magnetic material, each atom or molecule has net dipole magnetic
8. Using the relation ⃗𝑩 ⃗ = 𝝁𝒐 (𝑯⃗⃗⃗ + ⃗𝑴 ⃗⃗ ) , show that 𝝌 𝒎 = 𝝁𝒓 − 𝟏 moment which is vector sum of orbital and spin magnetic moments of
Proof : The total magnetic induction, electrons.
⃗ = 𝜇𝑜 (𝐻⃗ + 𝑀 ⃗⃗ )  But due to random orientation of these moments, the net magnetic moment of
𝐵 − − − − − (1)
the material is zero.
 By definition,  In the presence of external magnetic field, the torque acting on the atomic
⃗⃗
𝑀
𝜒𝑚 = (𝑜𝑟) 𝑀 ⃗⃗ = 𝜒𝑚 𝐻 ⃗ dipoles will align them in the field direction.
𝐻⃗  Thus a net magnetic dipole moment induced in the direction of the applied field.
& ⃗ = 𝜇𝐻
𝐵 ⃗  The induced dipole moment is present as long as the external field exists.
 Put this in equation (1),  When placed in a non-magntic field, these materials will have a tendency to
𝜇𝐻 ⃗ = 𝜇𝑜 (𝐻 ⃗ + 𝜒𝑚 𝐻
⃗) move from weaker to stronger part of the field.
⃗ ⃗  Materials which exhibit weak magnetim in the direction of the applied field are
𝜇 𝐻 = 𝜇𝑜 𝐻 (1 + 𝜒𝑚 )
𝜇 known as paramagnetic materials.
= 1 + 𝜒𝑚 (e.g.) Aluminium, Platinum, Chromium
𝜇𝑜
(𝑜𝑟) 𝜇𝑟 = 1 + 𝜒𝑚 12. Explain ferro magnetism.
∴ 𝝌 𝒎 = 𝝁𝒓 − 𝟏 Ferromangntic material:
9. What are called dia, para and ferro magnetic material?  Ferro magnetic material also
 Materials which exhibit weak magnetim in the direction opposite to the applied possesses net magnetic dipole
field are known as diamagnetic materials. They are repelled by the magnet. moment as paramagnetic
(e.g.) Bismuth, Copper, Water material.
 Materials which exhibit weak magnetim in the direction of the applied field are  A ferro magnetic material is
known as paramagnetic materials. They are feebly attracted by the magnets made up of smaller rigions
(e.g.) Alluminium, Platinum, Chromium called ferromagnetic domain.
 Materials which exhibit strong magnetim in the direction of the applied field are  Within each domain, the magnetic moments are aligned in same direction due
known as feromagnetic materials. They are strongly attracted by the magnets to strong interaction arising from electron spin. So each domain has net
(e.g.) Iron, Cobalt, Nickel magnetization in a direction.
10. Explain dia magnetism.  But the direction of magnetization is different for different domains. Hence the
Diamagnetic material : net magnetization of the specimen is zero.
 The orbital motion of electron produce a magnetic field perpendicular to the  In the presence of external magnetic field, the domain having magnetic
plane of the orbit. moments parallel to the field grow in size and the other domains are aligned
 Thus each electron orbit has finite orbital magnetic dipole moment. But the with the field.
resultant magnetic moment for each atom is zero.  It results, a strong net magnetization of the material in the direction of the
 In the presence of an external magnetic moment, some electrons are speeded applied field is produced.
up and some are slowed down.  Materials which exhibit strong magnetism in the direction of the applied field is
 According to Lenz’s law, the electrons whose moments were anti-parallel are called ferro magnetic materials.
speeded up which produces induced magnetic moment in a direction opposite (e.g.) Iron, Nickel, Cobalt
to the field.
victory R. SARAVANAN. M.Sc., M.Phil., B.Ed PG ASST [PHYSICS], GBHSS, PARANGIPETTAI - 608 502
12 PHYSICS UNIT - 3 MAGNETISM AND MAGNETIC EFFECTS OF CURRENT CONCEPTUAL QUESTIONS & ANSWERS

13. List the properties of Diamagnetic materials. 17. What are the differences between soft and hard ferromagnetic materials?
Properties of Diamagnetic materials : Soft ferromagnetic materials :
 Magnetic susceptibility is negative.  When external field is removes, its magnetization will disappears.
 Relative permeability is slightly less than one  Area of the loop is small
 The magnetic field lines are excluded by diamagnetic materials when placed in  Low retentivity
a magnetic fields.  Low coercivity
 Susceptibility is nearly temperature independent.  High susceptibility and magnetic permeability
14. List the properties of Paramagnetic materials.  Less hysteresis loss
Properties of Paramagnetic materials :  Used as solenoid core, transformer core and electromagnets
 Magnetic susceptibility is small positive value. (e.g.) Soft iron, Mumetal, Stalloy
 Relative permeability is greater than one Hard ferromagnetic materials :
 The magnetic field lines are attracted in to paramagnetic materials when placed  When external field is removes, its magnetization will persists.
in a magnetic field.  Area of the loop is large
 Susceptibility is inversely proportional to temperature.  High retentivity
15. List the properties of Ferromagnetic materials.  High coercivity
Properties of Ferromangnetic materials :  Low susceptibility and magnetic permeability
 Magnetic susceptibility is positive and large  More hysteresis loss
 Relative permeability is very very greater than one  Used as permanent magnets
 The magnetic fleld lines are stronglyattracted in to the ferromagnetic materials (e.g.) Steel, Alnico, Lodestone
when placed in a magnetic field. 18. Explain the magnetic field around a straight current carrying conductor.
 Susceptibility is inversely proportional to temperature. Current carrying straight conductor :
16. Explain the applications of hysteresis loop.
Applications of hysteresis loop :
 The main significance of hysteresis loop is that it provides the following
information.
(i) Retentivity
(ii) Coercivity
(iii) Permiability
(iv) Susceptibility
(v) Energy loss during on cycle of magnetization  When a magnetic compass is kept near a current carrying straight conductor,
 These information will help us in selecting proper and suitable material for a the magnetic needle deflects which indicates there exists a magnetic field.
given purpose.  If we trace the direction shown by the magnetic needle, we can draw the
 For example, the materials (Steel and Alnico) with high retentivity, high magnetic field lines which are concentric circles having their centre at the axis
coercivity and high permeability are suitable for making permanent magnets. of the conductor.
 The materials (Soft iron and Mumetal) with high initial permeability, low  It may be either clockwise or anticlock wise depending on the direction of
retentivity, low coercivity and thin hysteresis loop with smaller area are current in the conductor.
preferred to make electro mangnet.  If strength of the current is increased, then the density of the magnetic field will
 The materials (Soft irom) with high initial permeability, large mangnetic also increases.
induction and thin hysteresis loop with smaller area are needed to desigh  The strength of the magnetic field decreases at the distance from the conductor
transformer cores. increases.

victory R. SARAVANAN. M.Sc., M.Phil., B.Ed PG ASST [PHYSICS], GBHSS, PARANGIPETTAI - 608 502
12 PHYSICS UNIT - 3 MAGNETISM AND MAGNETIC EFFECTS OF CURRENT CONCEPTUAL QUESTIONS & ANSWERS

19. Explain the magnetic field around the current carrying circular loop.  From superposition principole the total magnetic field due to entire conductor
Circular coil carrying current : is,
 If we keep a magnetic compass 𝝁𝒐 𝑰 ⃗⃗⃗⃗⃗ 𝒅𝒍 𝑿 𝒓̂
near a current carrying circular ⃗⃗ = ∫ ⃗⃗⃗⃗⃗⃗
𝑩 𝒅𝑩 = ∫
𝟒𝝅 𝒓𝟐
conductor, then the magnetic 21. Give the difference between Coulomb’s law and Biot-Savart’s law.
needle deflects which indicates Coulomb’s law Biot-Savart’s law
the existence of magnetic field. 1) Electric field is calculated 1) Magnetic field is calculated
 Tracing the direction of the
2) Produced by a scalar source (i.e) 2) Produced be vector source (i.e.)
deflection, it shows the magnetic
lines are circular near A and B
charge ‘q’ current element ‘𝐼 ⃗⃗⃗𝑑𝑙 ’
and nearly parallel to each other 3) It is directed along the position 3) It is directed perpendicular to the
near the centre of the loop. vector joining the source and the position vector and the current
 Thus the field present near the point at which the field is element
centre of the coil is almost calculated.
uniform. 4) Does not depends on angle 4) Depends on the angle between
 The strength of the magnetic field is increased if either the current in the coil or 𝐼 ⃗⃗⃗⃗
𝑑𝑙 and 𝑟̂
the number of turns or both are increased. 22. Explain the current loop acts as a magnetic dipole and calculate its dipole
 The polarity (north pole or south pole) depends on the direction of current in moment.
the loop. Current loop as a magnetic dipole :
20. State and explain Biot-Savart law.  The magnetic field from the centre of a currnt loop of radius ‘R’ along the axis
Biot - Savart law : 𝜇𝑜 𝐼 𝑅 2
⃗⃗⃗𝐵 = ̂
 According to Biot - Savart law, the 3 𝑘
2 (𝑅 + 𝑧 )2
2 2
magnitude of magnetic field ⃗⃗⃗⃗⃗ 𝑑𝐵 at a  At larger distance, 𝑧 ≫ 𝑅 and hence 𝑅2 + 𝑧 2 ≈ 𝑧 2
point ‘P’ at a distance ‘r’ from the 𝜇 𝐼 𝑅2 𝜇 𝐼 𝜋 𝑅2
small elemental length ‘dl’ of the ⃗⃗⃗𝐵 = 𝑜 ̂ = 𝑜
𝑘 𝑘̂
current ‘I’ carrying conductor varies, 2 𝑧3 2 𝜋 𝑧3
Here, 𝜋 𝑅2 → area of the loop
(i) 𝑑𝐵 ∝ 𝐼 𝜇 𝐼𝐴 𝜇𝑜 2 𝐼 𝐴
(ii) 𝑑𝐵 ∝ 𝑑𝑙 ⃗⃗⃗𝐵 = 𝑜 𝑘̂ = 𝑘̂ − − − −(1)
2𝜋𝑧 3 4 𝜋 𝑧3
(iii) 𝑑𝐵 ∝ sin 𝜃  We know that, magnetic field at a distance ‘𝑧’ along the axial line is
1
(iv) 𝑑𝐵 ∝ 2 𝜇 2 ⃗⃗⃗𝑝𝑚
𝑟 ⃗⃗⃗𝐵 = 𝑜 − − − − − −(2)
 Hence, 4 𝜋 𝑧3
𝐼 𝑑𝑙 sin 𝜃  Compare equation (1) and (2)
𝑑𝐵 ∝ ⃗⃗⃗𝒑𝒎 = 𝑰 ⃗⃗⃗𝑨
𝑟2
𝐼 𝑑𝑙 sin 𝜃 (𝑜𝑟) 𝒑𝒎 = 𝑰 𝑨
(𝑜𝑟) 𝑑𝐵 = 𝑘 − − − − (1)
𝑟2  This implies that a current carrying circular loop behaves as a magnetic dipole
 where, 𝑘 → constant of dipole moment 𝒑𝒎
𝜇
 In S. I. units, 𝑘 = 𝑜  So the magnetic dipole moment of any current loop is equal to the product of
4𝜋
 Hence, the current and area of the loop.
𝝁𝒐 𝑰 𝒅𝒍 𝐬𝐢𝐧 𝜽
𝒅𝑩 = − − − − (2)
𝟒𝝅 𝒓𝟐
 In vector notation,
𝝁𝒐 𝑰⃗⃗⃗⃗⃗
𝒅𝒍 𝑿 𝒓̂
⃗⃗⃗⃗⃗⃗
𝒅𝑩 = − − − − (3)
𝟒𝝅 𝒓𝟐
 Here ⃗⃗⃗⃗⃗⃗
𝒅𝑩 is perpendicular to both 𝑰⃗⃗⃗⃗⃗ 𝒅𝒍 and 𝒓̂
victory R. SARAVANAN. M.Sc., M.Phil., B.Ed PG ASST [PHYSICS], GBHSS, PARANGIPETTAI - 608 502
12 PHYSICS UNIT - 3 MAGNETISM AND MAGNETIC EFFECTS OF CURRENT CONCEPTUAL QUESTIONS & ANSWERS

23. Explain current carrying solenoid behaves like a bar magnet. Properties of Lorentz magnetic force :
Current carrying conductor: (i) ⃗⃗⃗𝑭𝒎 is directly proportional to the magnetic field (𝑩 ⃗⃗ )
(ii) ⃗⃗⃗𝑭𝒎 is directlty proportional to the velocity (𝒗
⃗)
(iii) ⃗⃗⃗𝑭𝒎 is directly proportional to sine of the angle between the velocity and
magnetic field.
(iv) ⃗⃗⃗𝑭𝒎 is directly proportional to the magnitude of the charge
(v) The direction of ⃗⃗⃗𝑭𝒎 is always perpendicular to ⃗⃗⃗𝑣 and 𝐵 ⃗
 A solenoid is a long coil of wire closely wound in the form of helix. (vi) The direction of 𝑭𝒎 on negative chanrge is opposite to the direction of ⃗⃗⃗𝑭𝒎 on
⃗⃗⃗
 When current flows through the solenoid, magnetic field is produced. positive charge
 It is due to the superposition of magnetic fields of each turn of the solenoid. (vii) If the of the charge is along the manetic field, then ⃗⃗⃗𝑭𝒎 is zero.
 Inside the solenoid, the magnetic field is nearly uniform and parallel to its axis. 26. Write a note on velocity selector.
 But outside the solenoid, the field is negligibly small. Velocity selector:
 Depending on the direction of current, one end of the solenoid behaves like  Let an electric charge ‘q’ of
North pole and the other end behaves like South pole. mass ‘m’ enters in to a region
 The direction of magnetic field is given by right hand palm rule. (i.e.) if the of uniform magnetic field
current carrying solenoid is held in right hand such that the fingers curl in the 𝐵⃗ with velocity ⃗⃗⃗𝑣
direction of current, then extended thumb gives the direction of magnetic field.  Dut to Lorentz force, the
 Hence magnetic field of a solenoid looks like the magnetic field of a bar magnet. charged particle moves in
Uses : helical path.
 Solenoid can be used as electromagnets which produces strong magnetic field  By applying proper electric field 𝐸⃗ , the Lorentz force can be balanced by
that can be turned ON or OFF. Coulomb force
 The strength of the magnetic field can be increased by keeping iron bar inside  Here Coulomb force acts along the direction of electric field, whereas the
the solenoid. Lorentz force is perpendicular to the direction of magnetic field.
 They are useful in designing variety of electrical appliences.  Therefore in order to balance these forces, both electric and magnetic fields must
24. Write a note in MRI. be perpendicular to each other. Such an arrangement of perpendicular electric
MRI : and magnetic fields are known as cross fields.
 MRI is Magnetic Resonance Imaging which helps the physicians to diagonise or  The force on electric charge due to these fields is ; ⃗⃗⃗𝐹 = 𝑞 [𝐸⃗ + (𝑣 𝑋 𝐵 ⃗ )]
monitor treatment for a variety of abnormal conditions happening within the  For a positive charge, the electric force on the charge acts in downward
head, chest, abdomen and pelvis. direction whereas the Lorentz force acts upwards.
 It is a non invasive medical test.
When these two forces balance one another, the net force ⃗⃗⃗𝐹 = 0. Hence
 The patient is placed in a circular opening and large current is sent through the
𝑞 𝐸 = 𝐵 𝑞 𝑣𝑜
superconduction wire to produce a strong magnetic field. 𝑬
 This magnetic field produces radio frequency pulses which are fed to a ∴ 𝒗𝒐 =
computer which produce pictures of organs which helps the physicians to 𝑩
 This means for a given magnitude of electric field 𝐸⃗ and magnetic field 𝐵 ⃗ , the
examine various parts of the body
forces act only for the particle moving with particular speed 𝒗𝒐 .
25. Define Lorentz force. Give the properties of Lorentz magnetic force.
 This speed is independent of mass and charge,
Lorentz force :
(i) If > 𝑣𝑜 , then charged particle deflects in the direction of Lorentz force.
 When an electric charge ′𝑞′ moves in the magnetic field 𝐵 ⃗ , it experience a force
(ii) If 𝑣 < 𝑣𝑜 , then charged particle deflects in the direction of Coulomb force.
called Lorentz magnetic force.
(iii) If , then no deflection and the charged particle moves in straight line.
𝑭𝒎 = 𝑩 𝒒 𝒗 𝐬𝐢𝐧 𝜽
 Thus by proper choice of electric and magnetic fields, the particle with
 In vector notation, particular speed can be selected. Such an arrangement of fields is called a
⃗⃗⃗𝑭𝒎 = 𝒒 (𝒗
⃗ 𝑿𝑩⃗⃗ )
velocity selector.
 This principle is used in Bainbridge mass spectrograph to separate the isotopes.

victory R. SARAVANAN. M.Sc., M.Phil., B.Ed PG ASST [PHYSICS], GBHSS, PARANGIPETTAI - 608 502
12 PHYSICS UNIT - 3 MAGNETISM AND MAGNETIC EFFECTS OF CURRENT CONCEPTUAL QUESTIONS & ANSWERS

27. How Galvanometer can be converted in to Ammeter.  Galvanometer resistance = 𝑅𝐺


Galvanometer to an Ammeter : High resistanc = 𝑅ℎ
 Ammeter is an instrument used to Current flows through galvanometer = 𝐼𝐺
measure current. Voltage to be measured = 𝑉
 A galvanometer is converted into an Total resistance of this circuit = 𝑅𝐺 + 𝑅ℎ
ammeter by connecting a low resistance  Here the current in the electrical circuit is same as the current passing through
called shunt in parallel with the the galvanometer. (i.e.)
galvanometer. 𝐼𝐺 = 𝐼
 The scale is calibrated in amperes. 𝑽
𝑰𝑮 =
 Galvanometer resistance = 𝑅𝐺 𝑹𝑮 + 𝑹𝒉
Shunt resistance = 𝑆 𝑉
Current flows through galvanometer = 𝐼𝐺 (𝑜𝑟) 𝑅𝐺 + 𝑅ℎ =
𝐼𝐺
Current flows through shunt resistance = 𝐼𝑆 𝑽
Current to be measured = 𝐼 ∴ 𝑹𝒉 = − 𝑹𝑮
𝑰𝑮
The potential difference across galvanometer is same as the potential difference  Let 𝑅𝑣 be the resistance of voltmeter, then
shunt resistance. (i.e.) 𝑉𝐺𝑎𝑙𝑣𝑎𝑛𝑜𝑚𝑒𝑡𝑒𝑟 = 𝑉𝑠ℎ𝑢𝑛𝑡 𝑹𝒗 = 𝑹𝑮 + 𝑹𝒉
𝐼𝐺 𝑅𝐺 = 𝐼𝑆 𝑆  Here, 𝑅𝐺 < 𝑅ℎ < 𝑅𝑣
𝐼𝐺 𝑅𝐺 = (𝐼 − 𝐼𝐺 ) 𝑆 − − − −(1)  Thus an voltmeter is a highresistance instrument, and it always connected in
𝑰𝑮 parallel to the circuit element.
𝑺= 𝑹
𝑰 − 𝑰𝑮 𝑮  An ideal ammeter has zero resistance.
 From equation (1), 29. Differentiate Scalar, Vector and Tensor.
𝐼𝐺 𝑅𝐺 = 𝑆 𝐼 − 𝐼𝐺 𝑆 Scalar :
𝐼𝐺 (𝑆 + 𝑅𝐺 ) = 𝑆 𝐼  It has only one component.
𝑺  It has no direction (i.e) no unit vector
𝑰𝑮 = 𝑰
𝑺 + 𝑹𝑮  Since it has no direction, its rank is zero.
 Let 𝑅𝑎 be the resistance of ammeter, then Vector :
1 1 1  It haIs resolved in to components.
= +
𝑅𝑎 𝑅𝐺 𝑆  It has only one direction. (i.e.) has one unit vector
𝑹𝑮 𝑺  Since each component have one direction, its rank is one
⟹ 𝑹𝒂 =
𝑹𝑮 + 𝑺 Tensor :
 Here, 𝑅𝐺 > 𝑆 > 𝑅𝑎  It has resolved into components.
 Thus an ammeter is a low resistance instrument, and it always connected in  It has more than one direction (i.e) has more than one unit vector
series to the circuit.  If each component associated with two direction, then its rank is two and if
 An ideal ammeter has zero resistance. each component associated with three direction, then its rank is three.
28. How Galvanometer can be converted in to voltmeter?  In general, if each component associated with ‘n’ direction, then it is called
Galvanometer to a voltmeter : tensor of rank ‘n’
 A voltmeter is an instrument used to
measure potential difference across
any two points.
 A galvanometer is converted in to
voltmeter by connecting high
resistance in series with the
galvanometer.
 The scale is calibrated in volts.

victory R. SARAVANAN. M.Sc., M.Phil., B.Ed PG ASST [PHYSICS], GBHSS, PARANGIPETTAI - 608 502
12 PHYSICS UNIT - 3 MAGNETISM AND MAGNETIC EFFECTS OF CURRENT CONCEPTUAL QUESTIONS & ANSWERS

 Let BH be the net Earth’s magnetic field at a point on the surface of the Earth,
PART - IV FIVE MARK LONG ANSWER QUESTIONS & ANSWERS
then
1. Discuss Earth’s magnetic field in detail. Horizontal component ; 𝐵𝐻 = 𝐵𝐸 cos 𝐼 − − − (1)
Earth’s magnetic field : Vertical component : 𝐵𝑉 = 𝐵𝐸 s𝑖𝑛 𝐼 − − − (2)
(2) 𝐵𝑉
⟹ tan 𝐼 =
(1) 𝐵𝐻
(i) At magnetic equator :
 At magnetic equator, 𝐼 = 0°, then
𝐵𝐻 = 𝐵𝐸
𝐵𝑉 = 0
(ii) At magnetic poles :
 At magnetic poles, 𝐼 = 90° , then
𝐵𝐻 = 0
𝐵𝑉 = 𝐵𝐸
2. Calculate the magnetic induction at a point on the axial line of a bar magnet.
 A freely suspended magnet comes to rest approximately along the geographical ⃗ 𝑎𝑥𝑖𝑠 ) :
Magnetic field at axial line ( 𝐵
north - south direction.
 To explain this, William Gilbert proposed that, Earth itself like a gigantic
powerful magnet, but this theory was not accepted.
 Gover suggested that the Earth’s magnetic field is due to hot rays coming out
from the Sun.
 So many theories have been proposed, but none of the theory completely
explains the cause for the Earth’s magnetism.
 The north pole of magnetic compass needle is attracted towards the magnetic
south pole of the Earth which is near the geographic north pole.
 Simillarly the south pole of magnetic compass needle is attracted towards the  Consider a bar magnet ‘NS’ of moment 𝑝𝑚 = 𝑞𝑚 2𝑙
magnetic north pole of the Earth which is near the the geographic south pole.  Let C be the point on its axis at a distance ‘r’ from centre ‘O’
 The branch of physics which deals with the Earth’s magnetic field is called  Let unit north pole (𝑞𝑚𝐶 = 1 𝐴 𝑚) is placed at ‘C’
Geomagnetism (or) Terrestrial magnetism.
 The repulsive force experienced by unit north pole (i.e.) magnetic field at ‘C’ due
 The Earth spins about an axis called geographic axis and vertical line passing
to north pole
through the geographic axis is called geographic meridian, and a great circle ⃗⃗⃗𝐹 𝜇 𝑞𝑚
perpendicular to Earth’s geographic axis is called geographic equator. ⃗⃗⃗𝐵𝑁 = 𝑁 = 𝑜 𝑖̂ − − − − (1)
 The straight line which connects magnetic poles of Earthis known as magnetic 𝑞𝑚𝐶 4 𝜋 (𝑟 − 𝑙)2
axis and the vertical lise passing throuth magnetic axis is called magnetic  The attractive force experienced by unit north pole (i.e.) magnetic field at ‘C’
meridian and a great circle perpendicular to Earth’s magnetic axis is called due to south pole
magnetic equator. ⃗⃗⃗𝐹 𝜇 𝑞𝑚
⃗⃗⃗𝐵𝑆 = 𝑆 = − 𝑜 𝑖̂ − − − − (2)
 The angle between magnetic meridian at a point and geographical meridian is 𝑞𝑚𝐶 4 𝜋 (𝑟 + 𝑙)2
called the magnetic declination (D).  Then total magnetic field at ‘C’ is
 The angle subtended by the Earth’s total magnetic field wih the horizontal ⃗ 𝑎𝑥𝑖𝑠 = ⃗⃗⃗𝐵𝑁 + ⃗⃗⃗𝐵𝑆
𝐵
direction in the magnetic meridian is called dip or magnetic inclination (I) at
𝜇𝑜 𝑞𝑚 𝜇𝑜 𝑞𝑚
that point. = 𝑖̂ + [− 𝑖̂ ]
 The component of Earth’s magnetic field along the horizontal direction in the 4 𝜋 (𝑟 − 𝑙) 2 4 𝜋 (𝑟 + 𝑙)2
magnetic meridian is called horizontal component of Earth’s magnetic field 𝜇𝑜 1 1
= 𝑞𝑚 [ − ] 𝑖̂
(BH) 4𝜋 (𝑟 − 𝑙) 2 (𝑟 + 𝑙)2

victory R. SARAVANAN. M.Sc., M.Phil., B.Ed PG ASST [PHYSICS], GBHSS, PARANGIPETTAI - 608 502
12 PHYSICS UNIT - 3 MAGNETISM AND MAGNETIC EFFECTS OF CURRENT CONCEPTUAL QUESTIONS & ANSWERS

𝜇𝑜 (𝑟 + 𝑙)2 − (𝑟 − 𝑙)2  Resolve these two magnetic fields into their components. Hence
= 𝑞𝑚 [ ] 𝑖̂ ⃗⃗⃗𝐵𝑁 = − 𝐵𝑁 cos 𝜃 𝑖̂ + 𝐵𝑁 sin 𝜃 𝑗̂
4𝜋 (𝑟 − 𝑙)2 (𝑟 + 𝑙)2
𝜇𝑜 𝑟2 + 𝑙2 + 2 𝑟 𝑙 − 𝑟2 − 𝑙2 + 2 𝑟 𝑙 𝐵 ⃗ 𝑆 = − 𝐵𝑆 cos 𝜃 𝑖̂ − 𝐵𝑆 sin 𝜃 𝑗̂
= 𝑞𝑚 [ ] 𝑖̂  Then the total magnetic field at ‘C’ is
4𝜋 {(𝑟 − 𝑙) (𝑟 + 𝑙)}2
𝜇𝑜 4𝑟𝑙 ⃗ 𝑒𝑞𝑢𝑎𝑡𝑜𝑟 = ⃗⃗⃗𝐵𝑁 + 𝐵
𝐵 ⃗𝑺
= 𝑞 𝑖̂ = − 𝐵𝑁 cos 𝜃 𝑖̂ + 𝐵𝑁 sin 𝜃 𝑗̂
4 𝜋 𝑚 (𝑟 2 − 𝑙 2 )2
𝜇𝑜 2 𝑟 (𝑞𝑚 2 𝑙) − 𝐵𝑺 cos 𝜃 𝑖̂ − 𝐵𝑆 sin 𝜃 𝑗̂
= 𝑖̂ ⃗ 𝑒𝑞𝑢𝑎𝑡𝑜𝑟 = − 𝐵𝑁 cos 𝜃 𝑖̂ − 𝐵𝑺 cos 𝜃 𝑖̂
4 𝜋 (𝑟 2 − 𝑙 2 )2 𝐵
𝜇 2 𝑟 𝑝𝑚 ⃗ 𝑒𝑞𝑢𝑎𝑡𝑜𝑟 = − 2 𝐵𝑁 cos 𝜃 𝑖̂
𝐵 [∵ 𝐵𝑁 = 𝐵𝑺 ]
⃗ 𝑎𝑥𝑖𝑠 = 𝑜
𝐵 𝑖̂ − − − − (3)
4 𝜋 (𝑟 2 − 𝑙 2 )2 𝜇𝑜 𝑞𝑚
= − 2 cos 𝜃 𝑖̂
 where 𝑞𝑚 2 𝑙 = 𝑝𝑚 → magnetic dipole moment 4 𝜋 𝑟 !𝟐
 If 𝑟 ≫ 𝑙, then (𝑟 2 − 𝑙 2 )2 ≈ 𝑟 4 . So 𝜇 2 𝑞𝑚
⃗ 𝑒𝑞𝑢𝑎𝑡𝑜𝑟 = − 𝑜
𝐵 cos 𝜃 𝑖̂ − − − (3)
𝜇 2 𝑟 𝑝𝑚 4 𝜋 (𝑟 + 𝑙 2 )
2
𝐵⃗ 𝑎𝑥𝑖𝑠 = 𝑜 𝑖̂
4𝜋 𝑟4  But in ∆ 𝑁𝑂𝐶,
𝜇 2 𝑝𝑚 𝑂𝑁 𝑙 𝑙
𝐵⃗ 𝑎𝑥𝑖𝑠 = 𝑜 𝑖̂ [𝑝𝑚 𝑖̂ = ⃗⃗⃗𝑝𝑚 ]
4 𝜋 𝑟3 cos 𝜃 = = != 1
𝐶𝑁 𝑟 (𝑟 2 + 𝑙 2 )2
𝝁 𝟐 ⃗⃗⃗⃗⃗
𝒑𝒎
⃗⃗ 𝒂𝒙𝒊𝒔 = 𝒐
𝑩 − − − − (𝟒)  Then equation (3) becomes,
𝟒𝝅 𝒓𝟑
3. Obtain the magnetic induction at a point on the equatorial line of a bar 𝜇 𝑞𝑚 𝑙
𝐵⃗ 𝑒𝑞𝑢𝑎𝑡𝑜𝑟 = − 2 𝑜 1 𝑖̂
magnet. 4𝜋 (𝑟 2 2 )
+ 𝑙 (𝑟 2
+ 𝑙 2 )2
Magnetic field at equatorial line ( 𝑩 ⃗⃗ 𝒆𝒒𝒖𝒂 ): 𝜇 𝑞𝑚 2 𝑙
𝐵⃗ 𝑒𝑞𝑢𝑎𝑡𝑜𝑟 = − 𝑜 3 𝑖̂
4 𝜋 (𝑟 2
+ 𝑙 2 )2
𝜇 𝑝𝑚
𝐵⃗ 𝑒𝑞𝑢𝑎𝑡𝑜𝑟 = − 𝑜 3 𝑖̂
4 𝜋 (𝑟 2
+ 𝑙 2 )2
 where 𝑞𝑚 2 𝑙 = 𝑝𝑚 → magnetic dipole moment
3
 If 𝑟 ≫ 𝑙, then (𝑟 2 + 𝑙 2 )2 ≈ 𝑟 3 . So
𝜇 𝑝
𝐵⃗ 𝑒𝑞𝑢𝑎𝑡𝑜𝑟 = − 𝑜 𝑚 𝑖̂ [𝑝𝑚 𝑖̂ = 𝑝𝑚 ]
4 𝜋 𝑟3
𝝁 ⃗⃗⃗⃗⃗
𝒑
⃗⃗ 𝒆𝒒𝒖𝒂𝒕𝒐𝒓 = − 𝒐 𝒎
𝑩 − − − −(𝟒)
𝟒 𝝅 𝒓𝟑
4. What is tangent law? Discuss in detail. Explain the principle, construction and
 Consider a bar magnet ‘NS’ of moment 𝑝𝑚 = 𝑞𝑚 2𝑙 working of tangent galvanometer.
 Let C be the point on its equatorial line at a distance ‘r’ from centre ‘O’ Tangent Galvanometer :
 Let unit north pole (𝑞𝑚𝐶 = 1 𝐴 𝑚) is placed at ‘C’  It is a device used to measure very small currents.
 The repulsive force experienced by unit north pole (i.e.) magnetic field at ‘C’  It is a moving magnet type galvanometer.
due to north pole  Its working is based on tangent law.
𝐹𝑁 𝜇𝑜 𝑞𝑚 Tangent law :
𝐵𝑁 = = (𝑎𝑙𝑜𝑛𝑔 𝑁𝐶) − − − − (1)
𝑞𝑚𝐶 4 𝜋 𝑟 !𝟐  When a magnetic needle or magnet is freely suspended in two mutually
 The attractive force experienced by unit north pole (i.e.) magnetic field at ‘C’ perpendicular uniform magnetic fields, it will come to rest in the direction of
due to south pole the resultant of the two fields.
𝐹𝑆 𝜇𝑜 𝑞𝑚  Let B be the magnetic field produced by passing current through the coil of
𝐵𝑆 = = (𝑎𝑙𝑜𝑛𝑔 𝐶𝑆) − − − − (2) tangent galvanometer and B H be the horizontal component of Earth’s magnetic
𝑞𝑚𝐶 4 𝜋 𝑟 !𝟐
 Here, 𝑩𝑵 = 𝑩𝑺 field.
victory R. SARAVANAN. M.Sc., M.Phil., B.Ed PG ASST [PHYSICS], GBHSS, PARANGIPETTAI - 608 502
12 PHYSICS UNIT - 3 MAGNETISM AND MAGNETIC EFFECTS OF CURRENT CONCEPTUAL QUESTIONS & ANSWERS

 Under the action of two magnetic fields, the needle comes to rest at an angle  5. Define Hysterisis. Explain it with help of diagram.
with 𝐵𝐻 , such that Hysterisis :
𝑩 = 𝑩𝑯 𝒕𝒂𝒏 𝜽  Hysterisis means ‘lagging behind’
Construction :  The phenomenon of lagging of magnetic induction (𝐵⃗ ), behind the magnetizing
 It consists of copper coil wound on a non-magnetic circular frame. ⃗ ) is called hysteresis.
field (𝐻
 It is fixed vertically on a horizontal turn table providing with three levelling Hysterisis loop :
screws.
 At centre, a compass box is placed which consists of a small magnetic needle
which is pivoted at its centre.
 A thin aluminium pointer is attached to the magnetic needle normally and
moves over circular scale.
 The circular scale is divided in to four quadrants and graduated in degrees.
 In order to avoid parallax error in measurement, a mirror is placed below the
aluminium pointer.
 Here the centre of magnetic needle will exactly coincide with the centre of the
circular coil.
 The coil has three sections of 2, 5 and 50 turns which are different thickness
and are used to measuring currents of different strengths.
Theory :
 Let a ferro magnetic material (iron) is magnetized slowly by a magnetizing field
 When no current is passed through the coil, the small magnetic needle lies
𝐻⃗
along horizontal component of Earth’s magnetic field
 When current pass through the coil, it produces  The magnetic induction 𝐵 ⃗ is increases from point A and attains saturated level
magnetic field in direction perpendicular to the plane of at C. This is shown by the path AC
the coil.  The maximum point up to which the material cn be magnetized by applying the
 Now there are two fields, which are acting mutually magnetizing field is called Saturation magnetization.
perpendicular to each other. They are  If magnetizing field is now reduced, the magnetic induction also decreases but
(i) The magnetic field ‘B’ due to current in the coil in different path CA.
(ii) Horizontal component of Earth’s magnetic field ‘BH’  When magnetizing field is zero, the magnetic induction is not zero and it has
 Thus the magnetic needle deflects through an angle ‘’. By tangent law, positive value. (i.e.) some magnetism is left in the material even when H=0.
B = Bh tan θ − − − − − − − − (1)  The ability of the material to retain the magnetism in them even magnetizing
 When current ‘I’ passing through a circular coil of radius ‘R’ having ‘N’ turns, field vanishes is called remanence or retentivity.
the magnitude of magnetic field at the centre is,  To remove the remance, the magnetizing field is gradually increased in the
μo N I reverse direction, so that the magnetic induction decreases along DE and
B = − − − − − − − − (2) becomes zero at ‘E’
2R
 Put equation (2) in (1)  The magnitude of the reverse magnetizing field for which the residual magnetism
μo N I of the material vanishes is called its coercivity.
= BH tan θ ⃗ in the reverse direction, the mangetic indiuction
2R  Further increase of 𝐻
μo N 𝐼 increases along EF until it reaches saturation at F in the reverese direction.
𝐁𝐇 = ( ) − − − − − −(3)
2 R tan 𝜃  If magnetizing field is decreased and then increased with direction reversed,
 Also the current is , the magnetic induction traces the path FGKC.
𝟐 𝐑 𝐁𝑯  This closed curve ACDEFGKC is called hysteresis loop and it represents a cycle
𝐈 = 𝐭𝐚𝐧 𝛉 = 𝐊 𝐭𝐚𝐧 𝛉 − −(4)
𝛍𝐨 𝐍 of magnetization.
 where,  In the entire cycle, the magnetic induction ‘B’ lags behind the magnetizing field
𝟐 𝐑 𝐁𝑯 ‘H’.
K= 𝛍𝐨 𝐍
−→ Reduction factor of TG
 This phenomenon is called hysteresis
victory R. SARAVANAN. M.Sc., M.Phil., B.Ed PG ASST [PHYSICS], GBHSS, PARANGIPETTAI - 608 502
12 PHYSICS UNIT - 3 MAGNETISM AND MAGNETIC EFFECTS OF CURRENT CONCEPTUAL QUESTIONS & ANSWERS

Hysterisis Loss :  For infinitely long conductor, φ1 = φ2 = 90°


 Due to hysterisis there is a loss of energy in the form of heat and It is found that μ I
⃗⃗⃗𝐵 = 0 [ 2 ] 𝑛̂
the energy lost per unit volume of the material when it is carried through one 4π a
cycle of magnetization is equal to the area of the hysteresis loop. 𝛍𝟎 𝐈
⃗⃗⃗𝑩 = 𝒏
̂
 Thus the loss of energy for a complete cycle is, 𝟐𝛑 𝐚
7. Obtain a relation for the magnetic induction at a point along the axis of a
⃗⃗ . ⃗⃗⃗⃗⃗⃗
∆𝑬 = ∮ ⃗𝑯 𝒅𝑩 circular coil carrying current.
6. Deduce the relation for magnetic induction at a point due to an infinitely long Magnetic field due to current carrying circular coil :
straight conductor carrying current.  Consider a circular coil of radius ‘R’
Magnetic field due to long straight current carrying conductor : carrying a current ‘I’ in anticlock wise
 Consider a long straight wire YYI direction.
carrying a current I  Let ‘P’ be the point on the axis at a distance
 Let P be a point at a distance ‘a’ from ‘O’ ‘z’ from centre ‘O’
 Consider an element of length ‘𝑑𝑙’ of the  Consider two diametrically opposite line
wire at a distance ‘𝑙’ from point ‘O’ elements of the coil of each of length ⃗⃗⃗ 𝑑𝑙 at
 Let ⃗⃗𝑟 be the vector joining the element C and D.
‘𝑑𝑙’ with the point ‘P’ and ‘𝜃’ be the angle  Let ⃗⃗𝑟 be the vector joining the current
between ⃗⃗𝑟 and ⃗⃗⃗ 𝑑𝑙 element (𝐼 𝑑𝑙 ⃗⃗⃗ ) at C to the point ‘P’
 Then the magnetic field at ‘P’ due to the  From Pythogorous theorem,
element is, 𝑃𝐶 = 𝑃𝐷 = 𝑟 = √𝑅2 + 𝑧 2
𝝁𝒐 𝑰 𝒅𝒍 𝐬𝐢𝐧 𝜽 and ∠ 𝐶𝑂𝑃 = ∠𝐷𝑂𝑃 = ϕ
⃗⃗⃗⃗⃗⃗
𝒅𝑩 = 𝒏̂ − −(1)
𝟒𝝅 𝒓𝟐  According to Biot - Savart law, the
 where, 𝒏 ̂ → points into the page magnetic field at ‘P’ due to the current
AC AC
 In ABC, sin θ = = elements 𝐶 𝑎𝑛𝑑 𝐷 are,
AB dl
AC = dl sin θ − − − − − (2) 𝜇𝑜 𝐼 ⃗⃗⃗
𝑑𝑙 𝑋 ⃗⃗𝑟
⃗⃗⃗⃗⃗
𝑑𝐵 =
 In AC, AC = r dφ − − − − − (3) 4𝜋 𝑟2
 From equation (2) and (3)  Their magnitudes are same and it is given by,
dl sin θ = r dφ − − − − − − − (4) 𝜇𝑜 𝐼 𝑑𝑙
𝑑𝐵 = [ ∵ 𝜃 = 90°]
 Put this in eqation (1) 4 𝜋 𝑟2
μ I r dφ μ0 I dφ  Here, ⃗⃗⃗⃗⃗ 𝑑𝐵 can be resolved in to two componenets.
⃗⃗⃗⃗⃗ = 0
𝑑𝐵 𝑛̂ = 𝑛̂ − − − (5)
4π r 2 4π r (i) 𝑑𝐵⃗⃗⃗⃗⃗ cos ϕ − horizontal component (Y - axis)
 In OAP (ii) ⃗⃗⃗⃗⃗
𝑑𝐵 sin ϕ − vertical component (Z - axis)
a a
cos φ = (or) r = − − − (6)  Here horizontal components of each element cancel each other.
r cos φ  But vertical components alone contribute to total magnetic field at the point ‘P’
 Put this in equation (5)
μ0 I dφ μ0 I B ⃗⃗⃗⃗⃗ = ∫ 𝑑𝐵 sin ϕ 𝑘̂
⃗ = ∫ 𝑑𝐵
⃗⃗⃗⃗⃗⃗
𝒅𝑩 = 𝑛̂ = cos φ 𝑛̂
a
4 π ( ⁄cos φ) 4π a 𝜇𝑜 𝐼 𝑑𝑙
⃗ =
B ∫ 2 sin ϕ 𝑘̂ − − − − − (1)
 The total magnetic field at ‘P’ due to conductor YYI 4𝜋 𝑟
φ2 φ2
μ0 I  Also from ∆𝐶𝑂𝑃,
⃗⃗⃗𝐵 = ∫ ⃗⃗⃗⃗⃗⃗ 𝑑𝐵 = ∫ cos φ 𝑛̂ 𝑅 𝑅
−φ1 −φ1 4 π a sin ϕ = = 1
μ I 𝑟 (𝑅2 + 𝑧 2 )2
⃗⃗⃗𝐵 = 0 ⌊sin φ⌋φ −φ1 𝑛
2
̂
4π a  But from equation (1)
μ0 I
⃗⃗⃗𝐵 = ⌊sin φ1 + sin φ2 ⌋ 𝑛̂ − − − − (7)
4π a
victory R. SARAVANAN. M.Sc., M.Phil., B.Ed PG ASST [PHYSICS], GBHSS, PARANGIPETTAI - 608 502
12 PHYSICS UNIT - 3 MAGNETISM AND MAGNETIC EFFECTS OF CURRENT CONCEPTUAL QUESTIONS & ANSWERS
𝜇𝑜 𝐼 𝑑𝑙 𝑅  By definition, angular momentum of the electron about ‘O’ is ⃗⃗𝐿 = ⃗⃗⃗𝑅 𝑋 ⃗⃗⃗𝑝
⃗ =
B ∫ 2 ̂
(𝑅 + 𝑧 ) 2 2 1 𝑘
4𝜋 (𝑅 + 𝑧 2 )2  In magnitude, angular momentum is given by,
𝜇𝑜 𝐼 𝑅 𝐿 =𝑅𝑝=𝑚𝑣𝑅 − − − − (3)
⃗B = ̂
3 ∫ 𝑑𝑙 𝑘  Dividing equation (2) by (3),
4 𝜋 (𝑅 + 𝑧 )
2 2 2 𝜇𝐿 𝑒𝑣𝑅 𝑒
 where, ∫ 𝑑𝑙 = 2 𝜋 𝑅 → total length of the coil. = − =−
𝐿 2𝑚𝑣𝑅 2𝑚
𝜇𝑜 𝐼 𝑅  In vector notation,
⃗ =
B 3
[2 𝜋 𝑅] 𝑘̂ 𝒆
4 𝜋 (𝑅2 + 𝑧 2 )2 ⃗⃗⃗𝝁𝑳 = − ⃗𝑳 − − − − (4)
𝟐𝒎
𝝁 𝒐 𝑰 𝑹𝟐  Here negative sign indicates that the magnetic dipole moment and angular
⃗ =
𝐁 ̂
𝒌
𝟑 momentum are in opposite direction. In magnitude,
𝟐 (𝑹 + 𝒛 )𝟐
𝟐 𝟐
𝜇𝐿 𝑒
 If the circular coil contains ‘N’ turns, then = = 8.78 𝑋 1010 𝐶 𝑘𝑔−1 = 𝑐𝑜𝑛𝑠𝑡𝑎𝑛𝑡
𝐿 2𝑚
𝝁 𝒐 𝑵 𝑰 𝑹𝟐
⃗ =
𝐁 ̂  This constant is called gyro-magnetic ratio.
𝟑 𝒌
𝟐 (𝑹 + 𝒛 )𝟐
𝟐 𝟐  According to Bohr quantization rule, angular momentum of an electron is,
 The magnetic field at the centre of the coil is, ℎ
𝐿=𝑛ℏ=𝑛
𝛍 𝐍𝐈 2𝜋
⃗ = 𝟎
𝐁 ̂
𝒌 (𝒛 = 𝟎)  where, ℎ → Plank’s constant (ℎ = 6.63 𝑋 10−34 𝐽 𝑠)
𝟐𝐑
8. Compute the magnetic dipole moment of revolving electron. And hence define 𝑛 → Positive integer (𝑛 = 1, 2, 3, … . ..)
bohr magneton. 𝑒 𝑒 ℎ
Magnetic dipole moment of revolving electron : ∴ 𝜇𝐿 = 𝐿= 𝑛
2𝑚 2 𝑚 2𝜋
 Let an electron moves in 𝒆𝒉
𝝁𝑳 = 𝒏 − − − − (𝟓)
circular motion around the 𝟒𝝅𝒎
nucleus. The circulating  The minimum magnetic moment can be obtained by substituting 𝑛 = 1
electron in a loop is like 𝒆𝒉
(𝝁𝑳 )𝒎𝒊𝒏 = 𝝁𝑩 = = 𝟗. 𝟐𝟕 𝑿 𝟏𝟎−𝟐𝟒 𝑨 𝒎𝟐
current in a circular loop. 𝟒𝝅𝒎
 The magnetic dipole  The minimum value of magnetic moment of revolving electron is called Bohr
moment due to current magneton (𝝁𝑩 )
carrying circular loop is, 9. Using Ampere’s law, obtain an expression for magnetic field due to the
⃗⃗⃗𝝁𝑳 = 𝑰 ⃗𝑨
⃗ current carrying wire of infinite length.
 In magnitude, 𝝁𝑳 = 𝑰 𝑨 − − − − − − (𝟏) Magnetic field due to current carrying straight wire using Ampere’s law :
 If T is thetime period of an electron, the current due to revolving electron is,  Consider a straight conductor of infinite length carrying current ‘I’
𝑒  Imagine an Amperian circular loop at a
𝐼= − distance ‘r’ from the centre of the
𝑇
where ‘- e’  charge of an electron. conductor.
 If ‘R’ be the radius and ‘𝑣’ be the velocity of electron in the circular orbit, then  From Ampere’s circuital law,
2𝜋 2𝜋𝑅 ⃗ . ⃗⃗⃗
∮𝐵 𝑑𝑙 = 𝜇𝑜 𝐼
𝑇= =
𝜔 𝑣
 Then equation (1) becomes,  Here ⃗⃗⃗
𝑑𝑙 is the line element along the
𝑒 𝑒 tangent to the Amperian loop. So the
𝝁𝑳 = − 𝐴 = − 𝜋 𝑅2
𝑇 2𝜋𝑅 angle between 𝐵 ⃗⃗⃗ is zero (𝜃 = 0°).
⃗ and 𝑑𝑙
[ ]
𝑣
where, 𝐴 = 𝜋𝑅2 → area of the circular orbit Thus,
𝒆𝒗𝑹 ∮ 𝐵 𝑑𝑙 = 𝜇𝑜 𝐼
∴ 𝝁𝑳 = − − − − − (2)
𝟐

victory R. SARAVANAN. M.Sc., M.Phil., B.Ed PG ASST [PHYSICS], GBHSS, PARANGIPETTAI - 608 502
12 PHYSICS UNIT - 3 MAGNETISM AND MAGNETIC EFFECTS OF CURRENT CONCEPTUAL QUESTIONS & ANSWERS

 Due to symmetry, the magnitude of the magnetic field is uniform over the  Here ab = h . If we take large loop such that it is equal to length of the solenoid,
Amperian loop and hence, we have
𝐵 ∮ 𝑑𝑙 = 𝜇𝑜 𝐼 ⃗ . ⃗⃗⃗
∮B dl = B 𝑳 − − − − − − − (2)
 For circular loop, ∮ 𝑑𝑙 = 2 𝜋 𝑟  Let ‘I’ be the current passing through the solenoid of ‘N’ turns, then
𝐵 (2 𝜋 𝑟) = 𝜇𝑜 𝐼 I0 = N I − − − − − − − − (3)
𝝁𝒐 𝑰  Put equation (2) and (3) in (1)
𝑩= B 𝐿 = μ0 N I
𝟐𝝅𝒓
 In vector notation, N
𝝁 𝑰 B = μ0 I − − − − (4)
⃗⃗ = 𝒐 𝒏
𝑩 ̂ 𝐿
𝐍
𝟐𝝅𝒓  Let ‘n’ be the number of turns per unit length, then = 𝒏 . Hence,
𝑳
10. Obtain an expression for magnetic field due to long current carrying 𝛍𝟎 𝐍 𝐈
solenoid. 𝐁 = = 𝛍𝟎 𝐧 𝐈 − − − − (5)
Mangnetic field due to current carrying solenoid : 𝑳
 Since ‘n’ and μ0 are constants, for fixed current ‘I’ the magnetic field ‘B’ inside
the solenoid is also constant.
11. Obtain the magnetic fields at various points on the toroid.
Toroid :
 A solenoid is bent in such a way its ends are joined together
to form a closed ring shape is called toroid.

Open space interior to the toroid (P) :


 To calculate the magnetic field 𝐵𝑃 at ‘P’, consider an Amperian loop (1) of
radius 𝒓𝟏
 Consider a solenoid of length ‘L’ having ‘N’ turns.  Then Amperian circuital law for loop 1 is
 To calculate the magnetic field at any point inside the solenoid, consider an ⃗⃗⃗ = 𝜇𝑜 𝐼𝑜
⃗ 𝑃 . 𝑑𝑙
∮𝐵
Amperian loop ‘abcd’
 From Ampere circuital law,  Since the loop 1 encloses no current, 𝐼𝑜 = 0,
⃗⃗⃗ = 𝜇𝑜 𝐼𝑜 then
⃗ . 𝑑𝑙
∮𝐵 − − − − − (1)
⃗ 𝑃 . ⃗⃗⃗
∮𝐵 𝑑𝑙 = 0
 The LHS of equation (1) can be written as
𝑏 𝑐 𝑑 𝑎
∴ ⃗𝑩
⃗𝑷=𝟎
⃗ . ⃗⃗⃗
∮𝐵 ⃗ . ⃗⃗⃗
𝑑𝑙 = ∫ 𝐵 ⃗ . ⃗⃗⃗
𝑑𝑙 + ∫ 𝐵 ⃗ . ⃗⃗⃗
𝑑𝑙 + ∫ 𝐵 ⃗ . ⃗⃗⃗
𝑑𝑙 + ∫ 𝐵 𝑑𝑙 Open space exterior to the toroid (Q):
𝑎 𝑏 𝑐 𝑑  To calculate magnetic field 𝑩𝑸 at ‘Q’
 Here, construct Amperian loop (3) of radius 𝒓𝟑
𝑏 𝑏
b  Then Amperian circuital law for loop 3 is
∫ ⃗B. ⃗⃗⃗
d𝒍 = ∫ 𝐵 𝑑𝑙 cos 0° = B ∫ 𝑑𝑙 = B 𝒉 ⃗ 𝑄 . ⃗⃗⃗
∮𝐵 𝑑𝑙 = 𝜇𝑜 𝐼𝑜
a
𝑎 𝑎
c
∫b B⃗ . ⃗⃗⃗
d𝒍 =
𝑐
∫𝑏 𝐵 𝑑𝑙 cos 90° = 0  Since in each turn of the toroid loop, current coming out of the plane of paper is
d cancelled by the current going into plane of the paper. Thus 𝐼𝑜 = 0
∫ ⃗B. ⃗⃗⃗
d𝒍 = 0 [∵ B = 0] ⃗⃗⃗ = 0
⃗ 𝑄 . 𝑑𝑙
∮𝐵
c
𝑐
a
∴ ⃗⃗ 𝑸 = 𝟎
𝑩
⃗⃗⃗ = ∫ 𝐵 𝑑𝑙 cos 90° = 0
⃗ . d𝒍
∫B
d
𝑏

victory R. SARAVANAN. M.Sc., M.Phil., B.Ed PG ASST [PHYSICS], GBHSS, PARANGIPETTAI - 608 502
12 PHYSICS UNIT - 3 MAGNETISM AND MAGNETIC EFFECTS OF CURRENT CONCEPTUAL QUESTIONS & ANSWERS

Inside the toroid (S) :  From equantion (2), (3) and (4), we infer that time period (T), frequency (f)
 To calculate magnetic field 𝑩𝑺 at ‘S’ construct Amperian loop (2) of radius 𝒓𝟐 and angular frequency (𝝎) depends only on specific charge, but not velocity or
 The length of the loop 2 ; 𝐿2 = 2 𝜋 𝑟2 the radius of the circular path.
and the loop encloses the current ; 𝐼𝑜 = 𝑁 𝐼 Special cases :
 Then Amperian circuital law for loop 2 is  If a charged particle moves in uniform magnetic field, such that its velocity is
⃗ 𝑆 . ⃗⃗⃗ not perpendicular to the magnetic field, then its velocity is resolved into two
∮𝐵 𝑑𝑙 = 𝜇𝑜 𝐼𝑜
components.
 One component is parallel to the fjeld and the other component is
𝐵𝑆 ∮ 𝑑𝑙 = 𝜇𝑜 𝑁 𝐼
perpendicular to the field.
𝐵𝑆 (2𝜋𝑟2 ) = 𝜇𝑜 𝑁 𝐼  Here parallel component remains
𝝁𝒐 𝑵 𝑰 unchanged and the perpendicular
𝑩𝑺 =
𝟐𝝅𝒓𝟐 component keeps on changing due to
𝐍
 Let ‘n’ be the number of turns per unit length, then = 𝒏. Hence Lorentz force.
𝟐𝝅𝒓𝟐
 Hence the path of the paricle is not circle, it is helix around the field.
𝑩𝑺 = 𝛍𝟎 𝐧 𝐈
13. Describe the principle, construction and working of Cylotron.
12. Obtain the expression for force on a moving charge in a magnetic field. Cylotron :
Force on moving charge in a magnetic field :
 It is a device used to accelerate the charged particles to gain large kinetic
 Consider a charged particle of charge ‘q’ having mass ‘m’ enters perpendicular energy. It is also called as high energy accelerator.
to uniform magnetic field ‘B’ with velocity 𝑣 ⃗⃗⃗
 It is invented by Lawrence and Livingston.
 So this charged particle experience Lorentz force which acts perpendicular to Principle :
both 𝐵 ⃗ and 𝑣⃗⃗⃗ and it is ; ⃗⃗⃗
𝐹 = 𝑞 (𝑣 ⃗ )
⃗⃗⃗ 𝑋 𝐵  When a charged particle moves normal to the magnetic field, it experience
 Since Lorentz force alone acts on the particle, the magnetic Lorentz force.
magnitude of this force is ; 𝐹 = 𝐵 𝑞 𝑣 ] Construction :
 Hence charged particle moves in a circular orbit and
the necessary centripetal force is provided by
Lorentz force. (i.e.)
𝑚 𝑣2
𝐵𝑞𝑣=
𝑟
 The radius of the circular path is,
𝑚𝑣 𝑝
𝑟= = − − − − (1)
𝐵𝑞 𝐵𝑞
where, 𝑚 𝑣 = 𝑝 → linear momentum
 Let ‘T’ be the time period, then
2𝜋𝑟 2𝜋𝑚𝑣
𝑇= =
𝑣 𝑣𝐵𝑞
𝟐𝝅𝒎
𝑻= − − − − (2)
𝑩𝒒
It is called cyclotron time period.
 Let ‘f’ be the frequency, then
𝟏 𝑩𝒒  It consists two semi circular metal containers called Dees.
𝒇= = − − − − (𝟑)
𝑻 𝟐𝝅𝒎  The Dees are enclosed in an evacuated chamber and it is kept in a region of
 In terms of angular frequency, uniform magnetic field acts normal to the plane of the Dees.
𝑩𝒒  The two Dees are kept separated with a gap and the source ‘S’ of charged
𝝎=𝟐𝝅𝒇= − − − − (4)
𝒎 particles to be accelerated is placed at the centre in the gap between the Dees.
It is called cyclotron frequency or gyro-frequency.  Dees are connected to high frequency alternating potential difference.

victory R. SARAVANAN. M.Sc., M.Phil., B.Ed PG ASST [PHYSICS], GBHSS, PARANGIPETTAI - 608 502
12 PHYSICS UNIT - 3 MAGNETISM AND MAGNETIC EFFECTS OF CURRENT CONCEPTUAL QUESTIONS & ANSWERS

Working :  When a current carrying conductor is placed in a magnetic field, the force
 Let the positive ions are ejected from source ‘S’ experienced by the wire is equal to the sum of Lorentz forces on the individual
 It is accelerated towards a Dee-1 which has negative potential at that instant. chage carriers in the wire.
 Since the magnetic field is normal to the plane of the Dees, the ion undergoes  Let a current ‘I’ flows through a conductor of lengh ‘L’ and area of cross-section
circular path. ‘A’
 After one semi-circular path in Dee-1, the ion reaches the gap between Dees.  Consider a small segment of wire of length ‘𝑑𝑙’
 At this time the polarities of the Dees are reversed, so that the ion is now  The free electorns drift opposite to the direction of current with drift velocity
accelerated towards Dee-2 with a greater velocity. 𝑣𝑑
 For this circular motion, the centripetal force of the charged particle is  The relation between current and drift velocity is,
provided by Lorentz force, then 𝐼 = 𝑛 𝐴 𝑒 𝑣𝑑 − − − − − (1)
𝑚 𝑣2  If the wire is kept in a magnetic field, then average force experienced by the
=𝐵𝑞𝑣 electron in the wire is
𝑟
𝑚𝑣 𝐹 = − 𝑒 (𝑣𝑑 𝑋 𝐵 ⃗)
𝑟=
𝐵𝑞  Let ‘n’ be the number of free electrons per unit volume, then the total number of
∴ 𝒓 ∝𝒗 electrons in the small element of volume (𝑉 = 𝐴 𝑑𝑙) is 𝑁 = 𝑛 𝐴 𝑑𝑙
 Thus the increase in velocity increases the radius of the circular path. Hence the  Hence Lorentz force on the small element,
particle undergoes spiral path of increasing radius. ⃗⃗⃗⃗⃗ ⃗ ) − − − (1)
𝑑𝐹 = −𝑒 𝑛 𝐴 𝑑𝑙 (𝑣𝑑 𝑋 𝐵
 Once it reaches near the edge, it is taken out with help of deflector plate and
 Here length 𝑑𝑙 is along the length of the wire and hence the current element is
allowed to hit the target T
⃗⃗⃗ = − 𝑛 𝐴 𝑒 𝑑𝑙 𝑣
I 𝑑𝑙 ⃗⃗⃗ 𝑑
 The important condition in cyclotron is the resonance condition. (i.e.) the
frequency ‘𝑓’ of the charged particle must be equal to the frequency of the  Put this in equation (1),
electrical oscillator ‘𝑓𝑜𝑠𝑐 ’ . Hence ⃗⃗⃗⃗⃗
𝑑𝐹 = 𝐼 ⃗⃗⃗ 𝑑𝑙 𝑋 𝐵 ⃗ − − − (2)
𝑩𝒒  Therefore, the force in a straight current carrying conductor of length ‘𝒍’ placed
𝒇𝒐𝒔𝒄 = in a uniform magnetic field
𝟐𝝅𝒎
 The time period of oscillation is , ⃗𝑭 = 𝐈 𝒍 𝑿 𝑩 ⃗⃗ − − − (3)
𝟐𝝅𝒎  In magnitude,
𝑻=
𝑩𝒒 𝑭 = 𝑩 𝐈 𝒍 𝐬𝐢𝐧 𝜽 − − − (4)
 The kinetic energy of the charged particle is, Special cases :
𝟏 𝑩𝟐 𝒒𝟐 𝒓𝟐 (i) If the current carrying conductor placed along the direction of magnetic field,
𝑲𝑬 = 𝒎 𝒗𝟐 = then  = 0°
𝟐 𝟐𝒎
Limitations of cyclotron : ∴ 𝑭=𝟎
(i) the speed of the ion is limited (ii) If the current carrying conductor is placed perpendicular to the magnetic field,
(ii) electron cannot be accelerated then  = 90°
(iii) uncharged paricles cannot be accelerated. ∴ 𝑭 = 𝑩 𝐈 𝒍 = 𝒎𝒂𝒙𝒊𝒎𝒖𝒎
14. Obtain an expression for the force on a current carrying conductor placed in a 15. Obtain a force between two long parallel current carrying conductors. Hence
magnetic field. define ampere.
Force on current carrying conductor in magnetic field : Force between two parallel conductors carrying current :

victory R. SARAVANAN. M.Sc., M.Phil., B.Ed PG ASST [PHYSICS], GBHSS, PARANGIPETTAI - 608 502
12 PHYSICS UNIT - 3 MAGNETISM AND MAGNETIC EFFECTS OF CURRENT CONCEPTUAL QUESTIONS & ANSWERS

 Consider two straight parallel current carrying conductors ‘A’ and ‘B’ separated 16. Deduce an expression for torque on a current loop placed in uniform
by a distance ‘r’ kept in air. ⃗⃗ .
magneitic field 𝑩
 Let I1 and I2 be the currents passing through the A and B in same direction Torque on a current loop :
(z-direction)  Consider a rectangular current loop PQRS kept in uniform magnetic field ⃗⃗⃗𝐵
 The net magnetic field due to I1 at a distance ‘r’ with its plane parallel to the field
𝜇 𝐼 𝜇 𝐼
𝐵⃗ 1 = 𝑜 1 (− 𝑖̂) = − 𝑜 1 𝑖̂  Let 𝑃𝑄 = 𝑅𝑆 = 𝑎 → Length of the loop
2𝜋𝑟 2𝜋𝑟 𝑄𝑅 = 𝑆𝑃 = 𝑏 → Breadth of the loop
 Here 𝐵 ⃗ 1 acts perpendicular to plane of paper and inwards.  Let 𝒏
̂ be the unit vector normal to the plane of the current loop.
 Then Lorentz force acts on the length element 𝑑𝑙 in conductor ‘B’ carrying
current I2 due to this magnetic field 𝐵 ⃗1
𝜇 𝐼
⃗⃗⃗⃗⃗ = 𝐼2 𝑑𝑙
𝑑𝐹 ⃗⃗⃗ 𝑋 𝐵⃗ 1 = − 𝐼2 𝑑𝑙 𝑘̂ 𝑋 𝑜 1 𝑖̂
2𝜋𝑟
𝜇𝑜 𝐼1 𝐼2 𝑑𝑙
⃗⃗⃗⃗⃗
𝑑𝐹 = − ̂
(𝑘 𝑋 𝑖̂)
2𝜋𝑟
𝜇𝑜 𝐼1 𝐼2 𝑑𝑙
⃗⃗⃗⃗⃗
𝑑𝐹 = − 𝑗̂
2𝜋𝑟
 By Flemming’s left hand rule, this force acts left wards. The force per unit
length of the conductor B
⃗⃗⃗𝑭 𝝁𝒐 𝑰𝟏 𝑰𝟐  Let the loop is divided in to four sections PQ, QR, RS and SP. The Lorentz force
= − 𝒋̂ − − − − − (𝟏) on each loop can be calculated as follows.
𝒍 𝟐𝝅𝒓  Magnitude of Force on section 𝑃𝑄
 Simillarly, net magnetic field due to 𝐼2 at a distance ‘r’ is
𝜇 𝐼 F𝑃𝑄 = B I (PQ) sin 90° = 𝐁 𝐈 𝐚
⃗ 2 = 𝑜 2 𝑖̂
𝐵 From right hand cork screw rule, its direction is vertically upwards.
2𝜋𝑟
 Here 𝐵 ⃗ 2 acts perpendicular to plane of paper and outwards.  Magnitude of Force on section 𝑄𝑅
 Then Lorentz force acts on the length element 𝑑𝑙 in conductor ‘A’ carrying F𝑄𝑅 = B I (QR) sin( 90 − ) = 𝐁 𝐈 𝐛 𝐜𝐨𝐬 𝛉
⃗2 Its direction is along the loop downwards
current I1 due to this magnetic field 𝐵
𝜇 𝐼  Magnitude of Force on section 𝑅𝑆
⃗⃗⃗⃗⃗
𝑑𝐹 = 𝐼1 ⃗⃗⃗ 𝑑𝑙 𝑋 𝐵 ⃗ 2 = 𝐼1 𝑑𝑙 𝑘̂ 𝑋 𝑜 2 𝑖̂ F𝑅𝑆 = B I (RS) sin 90° = 𝐁 𝐈 𝐚
2𝜋𝑟
𝜇 𝐼 𝐼 𝑑𝑙 From right hand cork screw rule, its direction is vertically downwards.
⃗⃗⃗⃗⃗ = 𝑜 1 2
𝑑𝐹 (𝑘̂ 𝑋 𝑖̂)  Magnitude of Force on section 𝑆𝑃
2𝜋𝑟
𝜇 𝐼 𝐼 𝑑𝑙 F𝑆𝑃 = B I (SP) sin( 90 − ) = 𝐁 𝐈 𝐛 𝐜𝐨𝐬 𝛉
⃗⃗⃗⃗⃗ = 𝑜 1 2
𝑑𝐹 𝑗̂ Its direction is along the loop upwards
2𝜋𝑟
 By Flemming’s left hand rule, this force acts right wards. The force per unit  Since the forces FQR and FSP are equal, opposite and collinear, they cancel each
length of the conductor A other.
⃗⃗⃗𝑭 𝝁𝒐 𝑰𝟏 𝑰𝟐  But the forces FPQ and FRS, which are equal in magnitude and opposite in
= 𝒋̂ − − − − − (𝟐) direction, are not acting along same straight line. Therefore, FPQ and FRS
𝒍 𝟐𝝅𝒓 constitute a couple which exerts a torque on the loop.
 Thus the force experienced by two parallel current carrying conductors is
attractive if they carry current in same direction.
 On the other hand, the force experienced by two parallel current carrying
conductors is repulsive if they carry current in opposite direction.
Definition of ampere :
 One ampere is defined as that current when it is passed through each of two
infinitely long parallel conductors kept a a distance of one metre apart in
vacuum causes each conductor experience a force of 2 𝑋 10−7 newton per
meter length of conductor.
victory R. SARAVANAN. M.Sc., M.Phil., B.Ed PG ASST [PHYSICS], GBHSS, PARANGIPETTAI - 608 502
12 PHYSICS UNIT - 3 MAGNETISM AND MAGNETIC EFFECTS OF CURRENT CONCEPTUAL QUESTIONS & ANSWERS

 The magnitude of torque acting on the arm PQ about AB is Working :


𝑏 𝑏
τ𝑃𝑄 = FPQ [ sin 𝜃] = 𝐁 𝐈 𝐚 [ sin 𝜃]
2 2
It points in the direction of AB
 The magnitude of torque acting on the arm RS about AB is
𝑏 𝑏
τ𝑅𝑆 = FRS [ sin 𝜃] = 𝐁 𝐈 𝐚 [ sin 𝜃]
2 2
It points in the direction of AB
 The total torque acting on the entire loop about an axis AB is given by
𝑏 𝑏
𝛕 = τ𝑃𝑄 + τ𝑅𝑆 = 𝐁 𝐈 𝐚 [ sin 𝜃] + 𝐁 𝐈 𝐚 [ sin 𝜃]  Consider a single turn of rectangular coil PQRS of length 𝑙 and breadth 𝑏, such
2 2 that
𝛕 = 𝐁 𝐈 𝐚 𝐛 𝐬𝐢𝐧 𝜽 = 𝐁 𝐈 𝐀 𝐬𝐢𝐧 𝜽
𝑃𝑄 = 𝑅𝑆 = 𝑙 ; 𝑄𝑅 = 𝑆𝑃 = 𝑏
where, 𝑎 𝑏 = 𝐴 → area of the rectangular loop
 Let ‘I’ be the electric current flowing through the rectangular coil
 𝛕 is along the direction of AB
 The horse-shoe type magnet has hemi-spherical magnetic poles which produces
 In vector form, ⃗⃗⃗𝝉 = 𝑰⃗⃗⃗𝑨 𝑿 𝑩
⃗⃗
a radial magnetic field.
 In terms of magnetic dipole moment, ⃗⃗⃗𝝉 = 𝒑 ⃗ 𝒎𝑿𝑩⃗⃗  Due to this radial field, the sides QR and SP are always parallel to the magnetic
where, 𝒑 ⃗ 𝒎 = 𝑰𝑨 ⃗⃗⃗ field ‘B’ and experience no force.
 The tendency of the torque is to rotate the loop so as to align its normal vector  But the sides PQ and RS are always perpendicular to the magnetic field ‘B’ and
with the direction of the magnetic field. experience force and due to ths torque is produced.
 If there are N turns in the rectangular loop, then the torque is given by  For single turn, the deflecting couple is,
𝛕 = 𝐍 𝐁 𝐈 𝐀 𝐬𝐢𝐧  𝜏𝑑𝑒𝑓 = 𝐹 𝑏 = 𝐵 𝐼 𝑙 𝑏 = 𝐵 𝐼 𝐴
Special cases:  For coil with N turns, we get
a) When θ = 90°or the plane of the loop is parallel to the magnetic field, the torque 𝝉𝒅𝒆𝒇 = 𝑵 𝑩 𝑰 𝑨 − − − − (1)
on the current loop is maximum. 𝛕 𝒎𝒂𝒙 = 𝐍 𝐁 𝐈 𝐀  Due to this deflecting torque, the coil get twisted and restoring torque is
b) When θ = 0°/180° or the plane of the loop is perpendicular to the magnetic developed.
field, the torque on the current loop is zero.  The magnitude of restoring torque is proportional to amount of twist and it is
17. Describe the principle, construction and working of moving coil given by
galvanometer. 𝝉𝒓𝒆𝒔 = 𝑲 𝜽 − − − − (2)
Moving coil galvanometer : where 𝐾 → restoring couple per unit twist (or)
 It is a device which is used to indicate the flow of current. torsional constant
Principle :  At equilibrium, 𝝉𝒅𝒆𝒇 = 𝝉𝒓𝒆𝒔
 When a current carrying loop is placed in a uniform magnetic field it 𝑁𝐵𝐼𝐴= 𝐾𝜃
experiences a torque. 𝑲
𝑰= 𝜽 = 𝑮 𝜽 − − − (𝟑)
Construction : 𝑵𝑩𝑨
𝑲
 It consists of a rectangular coil PQRS of insulated thin copper wire. where, 𝐺 = → galvanometer constant (or)
𝑵𝑩𝑨
 A cylindrical soft-iron core is placed symmentrically inside the coil. current reduction factor
 This rectangular coil is suspended freely between two pole pieces of a horse-
shoe magnet by means of phosphor - bronze wire.
 Lower end of the coil is connected to a hair spring which is also made up of
phosphor bronze.
 A small plane mirror is attached on the suspension wire to measure the
deflection of the coil with help of lamp and scale arrangement.
 In order to pass electric current through the galvanometer, the suspension strip
W and the spring S are connectee to terminals.

victory R. SARAVANAN. M.Sc., M.Phil., B.Ed PG ASST [PHYSICS], GBHSS, PARANGIPETTAI - 608 502
12 PHYSICS UNIT - 3 MAGNETISM AND MAGNETIC EFFECTS OF CURRENT CONCEPTUAL QUESTIONS & ANSWERS

3. Compute the magnetic length of a uniform bar


EXAMPLE PROBLEMS WITH SOLUTION
magnet if the geometrical length of the magnet is
1. The horizontal component and vertical component of Earth’s magnetic field at 12 cm. Mark the positions of magnetic pole points.
a place are 0.15 G and 0.26 G respectively. Calculate the angle of dip and Solution :
resultant magnetic field. (G-gauss, cgs unit for magnetic field 1G = 10–4 T)  We know that, Magnetic length : Geometrical length = 5 : 6
𝑀𝑎𝑔𝑛𝑒𝑡𝑖𝑐 𝑙𝑒𝑛𝑔𝑡ℎ 5
Solution : 𝐵𝐻 = 0.15 𝐺 ; 𝐵𝑉 = 0.26 𝐺 ; 𝐼 = ? ; 𝐵 = ? (i.e.) =
𝐺𝑒𝑜𝑚𝑒𝑡𝑟𝑖𝑐𝑎𝑙 𝑙𝑒𝑛𝑔𝑡ℎ 6
 Angle of dip ‘I’ is given by, 5 5
𝐵𝑉 0.26 26 𝑀𝑎𝑔𝑛𝑒𝑡𝑖𝑐 𝑙𝑒𝑛𝑔𝑡ℎ = 𝑋 𝐺𝑒𝑜𝑚𝑒𝑡𝑟𝑖𝑐𝑎𝑙 𝑙𝑒𝑛𝑔𝑡ℎ = 𝑋 12
tan 𝐼 = = = = 1. 733 6 6
𝐵𝐻 0.15 15 𝑀𝑎𝑔𝑛𝑒𝑡𝑖𝑐 𝑙𝑒𝑛𝑔𝑡ℎ = 10 𝑐𝑚
𝑰 = tan−1 (1.733) = 𝟔𝟎°  In this figure, the dot implies the pole points.
 Resultant magnetic field,
𝐵 = √𝐵𝐻2 + 𝐵𝑉2 = √0.152 + 0.262
𝐵 = √0.0225 + 0.0676 = √0.0901
𝑩 = 𝟎. 𝟑 𝑮
2. Let the magnetic moment of a bar magnet be 𝒑 ⃗ 𝒎 whose magnetic length is d =
2l and pole strength is 𝒒𝒎 . Compute the magnetic moment of the bar magnet 4. Calculate the magnetic flux coming out from
when it is cut into two pieces (a) along its length (b) perpendicular to its closed surface containing magnetic dipole
length. (say, a bar magnet) as shown in figure.
Solution : Solution :
(a) Bar magnet cut into two pieces along its length :  The total flux emanating from the closed
surface S enclosing the dipole is zero. So,
⃗⃗⃗⃗⃗ = 𝟎
⃗ . 𝒅𝑨
𝚽𝑩 = ∮ 𝑩
Here the integral is taken over closed surface. Since no isolated magnetic pole
 When the bar magnet is cut along the axis into two pieces, the magnetic (called magnetic monopole) exists, this integral is always zero. This is similar to
length does change , but magnetic pole strength becomes halved . So the new Gauss’s law in electrostatics.
 𝑞𝑚 5. The repulsive force between two magnetic poles in air is 9 × 10 –3 N. If the two
magnetic pole strength is ; 𝑞𝑚 =
2 poles are equal in strength and are separated by a distance of 10 cm, calculate
1 1
 Hence the magnetic moment ; 𝑝𝑚 = 𝑞𝑚 𝑑 = 𝑞𝑚 2 𝑙 = 𝑝𝑚 the pole strength of each pole.
2 2
 𝟏 Solution : 𝐹 = 9 𝑋 10−3 𝑁 ; 𝑟 = 10 𝑐𝑚 = 10 𝑋 10−2 𝑚 ; 𝑞𝑚𝐴 = 𝑞𝑚𝐵 = 𝑞𝑚 = ?
 In vector notation ; 𝒑 ⃗𝒎= 𝟐𝒑 ⃗𝒎  The magnitude of the force between two poles is given by
(b) Bar magnet cut into two pieces perpendicular to the axis : 𝜇𝑜 𝑞𝑚𝐴 𝑞𝑚𝐵
𝐹=
4𝜋 𝑟2
−7
4 𝜋 𝑋 10 𝑞𝑚2
9 𝑋 10−3 =
4𝜋 (10 𝑋 10−2 )2
2
𝑞𝑚
 When the bar magnet is cut perpendicular to the axis into two pieces, the 9 𝑋 10−3 = 10−7 𝑋
10−2
magnetic pole strength does change , but magnetic length becomes halved. 9 𝑋 10−3
 𝑑 𝑞𝑚2 = = 900
So the new magneticlength is ;: 𝑑 = = 𝑙 10−5
2 −𝟏
1 1 1 ∴ 𝒒𝒎 = 𝟑𝟎 𝑵 𝑻

 ∴ Magnetic moment ; 𝑝𝑚 = 𝑞𝑚 𝑑 = 𝑞𝑚 𝑑 = 𝑞𝑚 (2 𝑙) = 𝑝𝑚
2 2 2
 𝟏
 ⃗𝒎=
In vector notation ; 𝒑 ⃗𝒎
𝒑
𝟐
victory R. SARAVANAN. M.Sc., M.Phil., B.Ed PG ASST [PHYSICS], GBHSS, PARANGIPETTAI - 608 502
12 PHYSICS UNIT - 3 MAGNETISM AND MAGNETIC EFFECTS OF CURRENT CONCEPTUAL QUESTIONS & ANSWERS

6. A short bar magnet has a magnetic moment of 0.5 J T –1. Calculate magnitude 8. Compute the intensity of magnetisation of the bar magnet whose mass,
and direction of the magnetic field produced by the bar magnet which is kept magnetic moment and density are 200 g, 2 A m2 and 8 g cm–3, respectively.
at a distance of 0.1 m from the centre of the bar magnet along (a) axial line of Solution : 𝑚 = 200g = 200 𝑋10−3 𝑘g ; 𝑝𝑚 = 2 𝐴𝑚2 ; 𝜌 = 8 𝑔𝑐𝑚−3 = 8 𝑋103 𝑘𝑔 𝑚−3
the bar magnet and (b) normal bisector of the bar magnet.  Density of the bar magnet ;
Solution : 𝑝𝑚 = 0.5 𝐽 𝑇−1 ; 𝑟 = 0.1 𝑚 ; 𝐵𝑎𝑥𝑖𝑠 = ? ; 𝐵𝑒𝑞𝑢𝑎𝑡𝑜𝑟 = ? 𝑚𝑎𝑠𝑠 𝑚
𝜌= =
(a) Magnetic field at a point on the axial line of the bar magnet, 𝑉𝑜𝑙𝑢𝑚𝑒 𝑉
𝜇𝑜 2 𝑝𝑚  Hence the volume ;
𝐵 ⃗ 𝑎𝑥𝑖𝑠 = 𝑖̂
4 𝜋 𝑟3 𝑚 200 𝑋10−3
−7
4 𝜋 𝑋 10 2 𝑋 0.5 𝑉 = = 3
= 25 𝑋 10−6 𝑚3
⃗𝐵𝑎𝑥𝑖𝑠 = 𝑖̂ 𝜌 8 𝑋10
4𝜋 (0.1)3  So the intensity of magnetization ;
1 1 𝑝𝑚 2 0. 08
𝐵⃗ 𝑎𝑥𝑖𝑠 = 10−7 𝑋 𝑖̂ = 10−7 𝑋 𝑖̂ 𝑀= = = = 0.08 𝑋 106 𝐴 𝑚−1
0.001 10−3 𝑉 25 𝑋 10−6 10−6
⃗⃗ 𝑎𝑥𝑖𝑠 = 𝟏 𝑿 𝟏𝟎−𝟒 𝒊̂
𝑩 𝑴 = 𝟖 𝑿 𝟏𝟎𝟒 𝑨 𝒎−𝟏
 Hence, the magnitude of the magnetic field along axial is Baxis = 1 × 10–4 T 9. Using the relation 𝑩 ⃗⃗ = 𝝁𝒐 (𝑯 ⃗⃗⃗ + 𝑴 ⃗⃗⃗ ), show that 𝝌 = 𝝁 − 𝟏 .
𝒎 𝒓
and direction is towards South to North. Solution :
(b) Magnetic field at a point on the normal bisector of the bar magnet, ⃗⃗
𝑀
𝜇𝑜 𝑝𝑚  By definition, the magnetic susceptibility is ; 𝜒𝑚 = ⃗
𝐵⃗ 𝑒𝑞𝑢𝑎𝑡𝑜𝑟 = − 𝑖̂ 𝐻
4 𝜋 𝑟3 (𝑜𝑟) ⃗⃗ = 𝜒𝑚 𝐻
𝑀 ⃗ − − − − − (1)
4 𝜋 𝑋 10−7 0.5
⃗𝐵𝑒𝑞𝑢𝑎𝑡𝑜𝑟 = − 𝑖̂ ⃗
 By definition, the magnetic field is ; 𝐵 = 𝜇 𝐻 − − − − − (2) ⃗
4𝜋 (0.1)3 ⃗⃗ = 𝝁𝒐 (𝑯 ⃗⃗⃗ + 𝑴 ⃗⃗⃗ )
0.5 0.5  The given relation, 𝑩
𝐵 ⃗ 𝑒𝑞𝑢𝑎𝑡𝑜𝑟 = − 10−7 𝑋 𝑖̂ = 10−7 𝑋 𝑖̂  put equation (1) and (2), we get
0.001 10−3
⃗⃗⃗𝐵𝑒𝑞𝑢𝑎𝑡𝑜𝑟 = − 𝟎. 𝟓 𝑿𝟏𝟎−𝟒 𝒊̂ 𝜇𝐻 ⃗ = 𝜇𝑜 (𝐻 ⃗ + 𝜒𝑚 𝐻 ⃗)
 Hence, the magnitude of the magnetic field along equatorial is ⃗
𝜇 𝐻 = 𝜇𝑜 𝐻 + 𝜒𝑚 ) ⃗ (1
Bequator = 0.5 × 10 T and direction is towards North to South.
–4 𝜇 = 𝜇𝑜 (1 + 𝜒𝑚 )
𝜇
Note : The magnitude of Baxis is twice that of magnitude of Bequator and the direction of = (1 + 𝜒𝑚 )
Baxis and Bequator are opposite. 𝜇 𝑜
7. Consider a magnetic dipole which on switching ON external magnetic field (𝑜𝑟) 𝜇𝑟 = 1 + 𝜒𝑚
orient only in two possible ways i.e., one along the direction of the magnetic (𝑜𝑟) 𝝌 𝒎 = 𝝁𝒓 − 𝟏
field (parallel to the field) and another anti-parallel to magnetic field. Compute 10. Two materials X and Y are magnetised whose values of intensity of
the energy for the possible orientation. magnetisation are 500 A m–1 and 2000 A m–1 respectively. If the magnetising
Solution : field is 1000 A m–1, then which one among these materials can be easily
 Let 𝒑𝒎 be the dipole and before switching ON the external magnetic field, there magnetized?
Solution :
is no orientation. Therefore, the energy U = 0.
 Susceptibility of material X,
 As soon as external magnetic field is switched ON, the magnetic dipole orient
|𝑀 ⃗⃗ | 500 1
parallel (θ = 0o ) to the magnetic field with energy, 𝜒𝑚𝑋 = = = = 0.5
𝑜
𝑼𝒑𝒂𝒓𝒂𝒍𝒍𝒆𝒍 = − 𝒑𝒎 𝑩 𝐜𝒐𝒔 0 = − 𝒑𝒎 𝑩 = 𝒎𝒊𝒏𝒊𝒎𝒖𝒎 |𝐻 ⃗| 1000 2
 Otherwise, the magnetic dipole orients anti-parallel (θ = 180 ) to the magnetico  Susceptibility of material Y
field with energy, |𝑀 ⃗⃗ | 2000
𝑼𝒂𝒏𝒕𝒊 𝒑𝒂𝒓𝒂𝒍𝒍𝒆𝒍 = − 𝒑𝒎 𝑩 𝐜𝒐𝒔 180𝑜 = + 𝒑𝒎 𝑩 = 𝒎𝒂𝒙𝒊𝒎𝒖𝒎 𝜒𝑚𝑌 = = = 2

|𝐻 | 1000
 Here 𝜒𝑚 < 𝜒𝑚 .. It implies that material Y can be easily magnetized.
𝑋 𝑌

victory R. SARAVANAN. M.Sc., M.Phil., B.Ed PG ASST [PHYSICS], GBHSS, PARANGIPETTAI - 608 502
12 PHYSICS UNIT - 3 MAGNETISM AND MAGNETIC EFFECTS OF CURRENT CONCEPTUAL QUESTIONS & ANSWERS

11. The following figure shows the variation of 14. A coil of a tangent galvanometer of diameter 0.24 m has 100 turns. If the
intensity of magnetisation with the applied horizontal component of Earth’s magnetic field is 25 × 10–6 T then, calculate
magnetic field intensity for three magnetic the current which gives a deflection of 60o.
materials X, Y and Z. Identify the materials Solution : 𝑁 = 100 ; 𝑑 = 0.24 𝑚 ; 𝑟 = 0.12 𝑚 ; 𝐵𝐻 = 25 𝑋 10−6 𝑇 ; 𝜃 = 60°
X, Y and Z.  From the theory of TG, the current through the circular coil is,
Solution : 2 𝑟 𝐵𝐻
𝐼= tan 𝜃
 The slope of M-H graph is a measure of the 𝜇𝑜 𝑁
magnetic susceptibility, which is given by 2 𝑋 0.12 𝑋 25 𝑋 10−6
𝐼= tan 60
|⃗𝑀
⃗| 4 𝜋 𝑋 10−7 𝑋 100
𝑆𝑙𝑜𝑝𝑒 = = 𝜒𝑚 6 𝑋 10 −6
|⃗𝐻| 𝐼= 𝑋 √3
(a) Material X : Slope is positive and larger value. So, it is a ferromagnetic 4 𝑋 3.14 𝑋 10−5
−1
6 𝑋 1.732 𝑋 10
material. 𝐼=
(b) Material Y : Slope is positive and lesser value than X. So, it could be a 12.56
paramagnetic material. 𝐼 = 8. 274 𝑋 10−1 𝑋 10−1 = 8.274 𝑋 10−2 𝐴
(c) Material Z : Slope is negative and hence, it is a diamagnetic material. 𝑰 = 𝟎. 𝟎𝟖𝟐𝟕𝟒 𝑨
12. The magnetic field shown in the figure is due to the current carrying wire. In 15. Compute the magnitude of the magnetic field of a long, straight wire carrying
which direction does the current flow in the wire?. a current of 1 A at distance of 1m from it. Compare it with Earth’s magnetic
Solution : field.
Solution : 𝑰 = 1 𝐴 ; 𝑎 = 1 𝑚 ; 𝐵 = ? ; 𝐵 ∶ 𝐵𝑒𝑎𝑟𝑡ℎ = ?
 Magnetic field due to long straight current carrying wire,
𝜇𝑜 𝐼 4 𝜋 𝑋 10−7 𝑋 1
𝐵= = = 2 𝑋 10−7 𝑇
2𝜋𝑎 2𝜋𝑋1
 But the Earth’s magnetic field is ; 𝐵𝑒𝑎𝑟𝑡ℎ ≈ 10−5 𝑇
 Thus, 𝑩 ∶ 𝑩𝒆𝒂𝒓𝒕𝒉 = 𝟏 ∶ 𝟏𝟎𝟎
(i.e.) Bstraightwire is one hundred times smaller than BEarth
16. Calculate the magnetic field inside a solenoid, when (a) the length of the
solenoid becomes twice with fixed number of turns (b) both the length of the
solenoid and number of turns are doubled (c) the number of turns becomes
twice for the fixed length of the solenoid
 Using right hand rule, current flows upwards. Solution :
13. What is the magnetic field at the centre of the loop shown in figure?
 The magnetic field inside a solenoid of length L and turns N is
𝜇𝑜 𝑁 𝐼
𝐵𝐿,𝑁 = − − − − − − (1)
𝐿
𝜇 𝑁𝐼 𝐵
(a) If 𝐿 → 2𝐿 then ; 𝐵2𝐿,𝑁 = 𝑜 = 𝐿,𝑁
2𝐿 2
𝜇𝑜 2 𝑁 𝐼 𝜇 𝑁𝐼
(b) If 𝐿 → 2𝐿, 𝑁 → 2𝑁 then ; 𝐵2𝐿,2𝑁 = = 𝑜 = 𝐵𝐿,𝑁
2𝐿 𝐿
𝜇 2𝑁𝐼
(c) If 𝑁 → 2𝑁 then ; 𝐵𝐿,2𝑁 = 𝑜 𝐿 = 2 𝐵𝐿,𝑁
Solution :
 Here 𝑩𝑳,𝟐𝑵 > 𝑩𝟐𝑳,𝟐𝑵 > 𝑩𝟐𝑳,𝑵 . Thus, strength of the magnetic field is
 The magnetic field due to current in the upper semicircle and lower semicircle of
the circular coil are equal in magnitude but opposite in direction. increased when we pack more loops into the same length for a given current
 Hence, the net magnetic field at the centre of the loop (at point O) is zero
(i.e.) ⃗𝑩 = ⃗𝟎.

victory R. SARAVANAN. M.Sc., M.Phil., B.Ed PG ASST [PHYSICS], GBHSS, PARANGIPETTAI - 608 502
12 PHYSICS UNIT - 3 MAGNETISM AND MAGNETIC EFFECTS OF CURRENT CONCEPTUAL QUESTIONS & ANSWERS

⃗⃗ along positive y - direction in a


17. A particle of charge q moves with velocity 𝒗 19. An electron moving perpendicular to a uniform magnetic field 0.500 T
magnetic field ⃗⃗⃗
𝑩 . Compute the Lorentz force experienced by the particle undergoes circular motion of radius 2.50 mm. What is the speed of electron?
(a) when magnetic field is along positive y-direction (b) when magnetic field Solution : 𝐵 = 0.500 𝑇 ; 𝑟 = 2.5 𝑚𝑚 = 2.5 𝑋 10−3 𝑚 ; |𝑒| = 1.6 𝑋 10−19 𝐶 ; 𝑣 = ?
points in positive z - direction (c) when magnetic field is in zy - plane and  Lorentz force acts as centripetal force for the particle causing it to execute
𝑚𝑣
making an angle θ with velocity of the particle. Mark the direction of magnetic circular motion. The radius of the circular path is ; 𝑟 =
𝐵𝑒
force in each case.. 𝐵𝑒𝑟
 Therefore the speed of the electron ; 𝑣 =
𝑚
Solution : ⃗⃗𝑣 = 𝑣 𝑗̂ 0.5 𝑋 1.6 𝑋 10−19 𝑋 2.5 𝑋 10−3
𝐵 = 𝐵 𝑗̂
(a) When magnetic field is along positive y - direction, then ⃗⃗⃗ 𝑣=
9.1 𝑋 10−31
Lorentz force ; 𝐹 ⃗ ) = 𝑞 (𝑣 𝑗̂ 𝑋 𝐵 𝑗̂) = 𝑞 𝑣 𝐵 (𝑗̂ 𝑋 𝑗̂) = 0
⃗ 𝑚 = 𝑞 (⃗𝑣 𝑋 𝐵 0.5 𝑋 1.6 𝑋 2.5 𝑋 109
𝑣 =
So, no force acts on the particle when it moves along the direction of 9.1
magnetic field. 𝑣 = 2. 197 𝑋 10−1 𝑋 109
𝒗 = 𝟐. 𝟏𝟗𝟕 𝑿 𝟏𝟎𝟖 𝒎 𝒔−𝟏
(b) When magnetic field is along positive z - direction, then ⃗⃗⃗ 𝐵 = 𝐵 𝑘̂
20. A proton moves in a uniform magnetic field of strength 0.500 T magnetic field
Lorentz force ; 𝐹 ⃗ ) = 𝑞 (𝑣 𝑗̂ 𝑋 𝐵 𝑘̂ ) = 𝑞 𝑣 𝐵 (𝑗̂ 𝑋 𝑘̂ ) = 𝑞 𝑣 𝐵 𝑖̂
⃗ 𝑚 = 𝑞 (⃗𝑣 𝑋 𝐵 is directed along the x-axis. At initial time, t = 0 s, the proton has velocity
Therefore, the magnitude of the Lorentz force is qvB and direction is along ⃗ = [𝟏. 𝟗𝟓 𝑿 𝟏𝟎𝟓 𝒊̂ + 𝟐, 𝟎𝟎 𝑿 𝟏𝟎𝟓 𝒌̂ ] 𝒎 𝒔−𝟏 .Find
𝒗
positive x - direction.
(a) At initial time, what is the acceleration of the proton?
(c) When magnetic field is in zy -plane, then 𝐵 ⃗⃗⃗ = 𝐵 cos 𝜃 𝑗̂ + 𝐵 sin 𝜃 𝑘̂
(b) Is the path circular or helical? If helical, calculate the radius of helical
Lorentz force ; 𝐹𝑚 = 𝑞 (𝑣 𝑋 𝐵 ⃗ ) = 𝑞 (𝑣 𝑗̂ 𝑋 𝐵[cos 𝜃 𝑗̂ + 𝐵 sin 𝜃 𝑘̂])
trajectory and also calculate the pitch of the helix (Note: Pitch of the helix is
𝐹𝑚 = 𝑞 𝑣 𝐵 cos 𝜃 (𝑗̂ 𝑋 𝑗̂) + 𝑞 𝑣 𝐵 sin 𝜃 (𝑗̂ 𝑋 𝑘̂) = 𝑞 𝑣 𝐵 sin 𝜃 𝑖 the distance travelled along the helix axis per revolution).
⃗ = 0.500 𝑖̂ ; ⃗𝑣 = [1.95 𝑋 105 𝑖̂ + 2.00 𝑋 105 𝑘̂ ] ; 𝑡 = 0 𝑠 ; 𝒂 = ? ; 𝑻 = ?
Solution : 𝐵
(a) Mass of proton ; 𝑚𝑝 = 1.67 𝑋 10−27 𝑘𝑔
Charge of proton ; 𝑒 = 1.6 𝑋 10−19 𝐶
From Newton’s II law, acceleration is
𝐹𝐵 𝑞 (𝑣 𝑋 𝐵 ⃗)
𝑎= =
𝑚𝑝 𝑚𝑝
1.6 𝑋 10−19 [(1.95 𝑋 105 𝑖̂ + 2.00 𝑋 105 𝑘̂ ) 𝑋 0.500 𝑖̂ ]
𝑎=
18. Compute the work done and power delivered by the Lorentz force on the 1.67 𝑋 10−27
1.6 𝑋 10−19 [1 𝑋 105 𝑗̂] 1.6 𝑋 10−14
particle of charge q moving with velocity 𝒗 ⃗ . Calculate the angle between 𝑎= = 𝑗̂
1.67 𝑋 10−27 1.67 𝑋 10−27
Lorentz force and velocity of the charged particle and also interpret the result. 1.6 𝑋 10 13
Solution : 𝑎= 𝑗̂ = 9. 581 𝑋 10−1 𝑋 1013 𝑗̂
1.67
 Lorentz force on a charged particle moving on a magnetic field is, ⃗ = 𝟗. 𝟓𝟖𝟏 𝑿 𝟏𝟎𝟏𝟐 𝒋̂ (𝑜𝑟)
𝒂 𝒂 = 𝟗. 𝟓𝟖𝟏 𝑿 𝟏𝟎𝟏𝟐 𝒎 𝒔−𝟐
⃗⃗⃗𝐹𝑚 = 𝑞 (𝑣 𝑋 𝐵
⃗) (b) Here this acceleration directed perpendicular to the magnetic field, due to the
 Work done by the magnetic field ; 𝑊 = ∫ 𝐹 . ⃗⃗⃗⃗
𝑑𝑟 = 𝑞 ∫(⃗⃗𝑣 𝑋 𝐵⃗ ) . 𝑣 𝑑𝑡 = 0 Lorentz force, the velocity component 𝑣𝑧 = 2.00 𝑋 105 𝑘̂ along Z-axis alone
𝑑𝑊 continuously changed. Thus the path of proton is helical. The radius of helical
 And power is given by ; 𝑃 = =0 path is
𝑑𝑡
 Since, ⃗𝐹 . ⃗𝑣 = 0 we have ⃗𝑭 ⊥ 𝒗
⃗ . The angle between Lorentz force and velocity 𝑚𝑝 𝑣𝑧 1.67 𝑋 10−27 𝑋 2.00 𝑋 105
𝑟= =
of the charged particle is 90o. 𝐵𝑒 0.500 𝑋 1.6 𝑋 10−19
 Thus Lorentz force changes the direction of the velocity but not the magnitude 3.34 𝑋 10−3 33.4 𝑋 10−3
𝑟= =
of the velocity. Hence Lorentz force does no work and also does not alter kinetic 0.8 8
−𝟑
energy of the particle. 𝒓 = 𝟒. 𝟏𝟕𝟓 𝑿 𝟏𝟎 𝒎 = 𝟒. 𝟏𝟕𝟓 𝒎𝒎

victory R. SARAVANAN. M.Sc., M.Phil., B.Ed PG ASST [PHYSICS], GBHSS, PARANGIPETTAI - 608 502
12 PHYSICS UNIT - 3 MAGNETISM AND MAGNETIC EFFECTS OF CURRENT CONCEPTUAL QUESTIONS & ANSWERS

Time period ; 22. Let E be the electric field of magnitude 6.0 × 106 N C–1 and B be the magnetic
2𝜋 2 𝜋 𝑚𝑝 field magnitude 0.83 T. Suppose an electron is accelerated with a potential of
𝑇= =
𝜔 𝐵𝑒 200 V, will it show zero deflection?. If not, at what potential will it show zero
2 𝑋 3.14 𝑋 1.67 𝑋 10−27 10. 4876 𝑋 10−8 104. 876 𝑋 10−8 deflection?
𝑇= = =
0.500 𝑋 1.6 𝑋 10−19 0.8 8 Solution : 𝐸 = 6.0 𝑋 106 𝑁 𝐶 −1 ; 𝐵 = 0.83 𝑇 ; 𝑉 = 200 𝑉 ; 𝑣 = ? ; 𝑣200 = ? ; 𝑉0 = ?
𝑻 = 𝟏𝟑. 𝟏𝟎𝟗𝟓 𝑿 𝟏𝟎−𝟖 𝒔 = 𝟏𝟑. 𝟏𝟏 𝑿 𝟏𝟎−𝟖 𝒔  At zero deflection, the velocity of electron,
Hence, pitch of the helix is 𝐸 6.0 𝑋 106
𝑋 = 𝑣𝑥 𝑇 = 1.95 𝑋 105 𝑋 13.11 𝑋 10−8 = 25.5645 𝑋 10−3 𝑣= = = 𝟕. 𝟐𝟐𝟗 𝑿 𝟏𝟎𝟔 𝒎 𝒔−𝟏
−𝟑 𝐵 0.83
𝑿 = 𝟐𝟓. 𝟓𝟔 𝑿 𝟏𝟎 𝒎 = 𝟐𝟓. 𝟓𝟔 𝒎𝒎  Since the accelerating potential is 200 V, the electron
The proton experiences appreciable acceleration in the magnetic field, hence the acquires kinetic energy because of this accelerating potential.
pitch of the helix is almost six times greater than the radius of the helix. Hence
21. Two singly ionized isotopes of uranium 𝟐𝟑𝟓 𝟗𝟐𝑼 and 1 2
𝟐𝟑𝟖
𝑼 (isotopes have same atomic number but 𝑚 𝑣200 =𝑒𝑉
𝟗𝟐 2
different mass number) are sent with velocity 2
2𝑒𝑉
𝑣200 =
1.00 × 105 m s–1 into a magnetic field of strength 𝑚
0.500 T normally. Compute the distance between 2𝑒𝑉
the two isotopes after they complete a semi- ∴ 𝑣200 = √
𝑚
circle. Also compute the time taken by each
isotope to complete one semi-circular path. 2 𝑋 1.6 𝑋 10−19 𝑋 200 640 𝑋 1012
𝑣200 = √ = √
(Given: masses of the isotopes: m 235 = 3.90 × 10–25 kg and m238 = 3.95 × 10–25 9.1 𝑋 10−31 9.1
kg) 𝒗𝟐𝟎𝟎 = 𝟖. 𝟑𝟖𝟔 𝑿 𝟏𝟎𝟔 𝒎 𝒔−𝟏
Solution : 𝐵 = 0.500 𝑇 ; 𝑣 = 1.00 𝑋 105 𝑚 𝑠 −1 ; 𝑚235 = 3.90 𝑋 10−25 𝑘𝑔 ;  Since the speed 𝒗𝟐𝟎𝟎 > 𝒗 , the electron is deflected towards direction of Lorentz
−25 −19
𝑚238 = 3.95 𝑋 10 𝑘𝑔 ; 𝑞 = |𝑒| = 1.6 𝑋 10 𝐶 ; 𝑑 =? force. So, in order to have null deflection, the potential 𝑉0 , we have to supply is
 Since isotopes are singly ionized, they have equal charge which is equal to the 1
𝑚 𝑣 2 = 𝑒 𝑉0
charge of an electron 2
𝑚 𝑣2 9.1 𝑋 10−31 𝑋 (7.229 𝑋 106 )2
 The radius of the path of 𝟐𝟑𝟓 𝟗𝟐𝑼 is r235 ∴ 𝑉0 = =
𝑚235 𝑣 3.90 𝑋 10−25 𝑋 1.00 𝑋 105 3.90 𝑋 10−1 3.90 2𝑒 2 𝑋 1.6 𝑋 10−19
𝑟235 = = = = 9.1 𝑋 7.229 𝑋 7.229 𝑋 100
𝐵𝑒 0.500 𝑋 1.6 𝑋 10−19 0.8 8 𝑉0 =
𝑟235 = 0.4875 𝑚 3.2
𝑽𝟎 = 𝟏𝟒𝟖. 𝟔 𝑽
Hence the diameter ; 𝑑235 = 2 𝑟235 = 2 𝑋 0.4875 = 0.975 𝑚 = 97.5 𝑐𝑚 23. Suppose a cyclotron is operated to accelerate protons with a magnetic field of
 The radius of the path of 𝟐𝟑𝟖 𝟗𝟐𝑼 is r235 strength 1 T. Calculate the frequency in which the electric field between two
𝑚238 𝑣 3.95 𝑋 10−25 𝑋 1.00 𝑋 105 3.95 𝑋 10−1 3.95
𝑟238 = = = = Dees could be reversed.
𝐵𝑒 0.500 𝑋 1.6 𝑋 10−19 0.8 8 Solution : B = 1 T ; 𝑚𝑝 = 1.67 𝑋 10−27 𝑘𝑔 ; 𝑞 = |𝑒| = 1.6 𝑋 10−19 𝐶 ; 𝑓 = ?
𝑟238 = 0.49375 𝑚
Hence the diameter ; 𝑑238 = 2 𝑟238 = 2 𝑋 0.49375 = 0.9875 𝑚 = 98.75 𝑐𝑚  The frequency is,
𝐵𝑞
 Therefore the separation distance between the isotopes is ; 𝑓=
∆𝒅 = 𝑑238 − 𝑑235 = 0.9875 = 0.975 = 𝟎. 𝟎𝟏𝟐𝟓 𝒎 = 𝟏. 𝟐𝟓 𝒄𝒎 2 𝜋 𝑚𝑝
 The time taken by each isotope to complete one semi-circular path are 1 𝑋 1.6 𝑋 10−19
𝑓=
𝑑235 0.975 2 𝑋 3.14 𝑋 1.67 𝑋 10−27
𝑡235 = = 5
= 0.975 𝑋 10−5 = 9.75 𝑋 10−6 𝑠 = 9.75 𝜇𝑠 1.6 𝑋 108
𝑣 1.00 𝑋 10 𝑓 =
𝑑238 0.9875 10.4876
𝑡238 = = 5
= 0.9875 𝑋 10−5 = 9.875 𝑋 10−6 𝑠 = 9.875 𝜇𝑠 𝑓 = 1.525 𝑋 10−1 𝑋 108 = 1.525 𝑋 107 𝐻𝑧
𝑣 1.00 𝑋 10
𝒇 = 𝟏𝟓. 𝟐𝟓 𝑿 𝟏𝟎𝟔 𝑯𝒛 = 𝟏𝟓. 𝟐𝟓 𝑴𝑯𝒛
victory R. SARAVANAN. M.Sc., M.Phil., B.Ed PG ASST [PHYSICS], GBHSS, PARANGIPETTAI - 608 502
12 PHYSICS UNIT - 3 MAGNETISM AND MAGNETIC EFFECTS OF CURRENT CONCEPTUAL QUESTIONS & ANSWERS

24. A metallic rod of linear density is 0.25 kg m –1 is lying (c) Resistance of the galvanometer,
horizontally on a smooth inclined plane which makes an 1
𝑰𝑺 ( −6 ) 1
angle of 45o with the horizontal. The rod is not allowed to 𝑹𝒈 = = 10 = = 0.5 𝑋 103 = 𝟓𝟎𝟎 𝛀 = 𝟎. 𝟓 𝒌𝛀
𝑽𝑺 2 𝑋 10 3 2 𝑋 10 𝑋 10−6
3
slide down by flowing a current through it when a magnetic
26. The resistance of a moving coil galvanometer is made twice its original value
field of strength 0.25 T is acting on it in the vertical
in order to increase current sensitivity by 50%. Find the percentage change in
direction. Calculate the electric current flowing in the rod to
voltage sensitivity.
keep it stationary.
𝑚 Solution :
Solution : = 0.25 𝑘𝑔 𝑚−1 ; 𝐵 = 0.25 𝑇 ; 𝐼 = ?
𝑙  Let 𝐼𝑆 be the initial current sensitivity. If current sensitivity is increased by
 Magnetic Lorentz force experienced by the current carrying conductor placed in 50%, then new current sensitivity,
magnetic field ; 𝐹𝐵 = 𝐵 𝐼 𝑙 sin 90° = 𝐵 𝐼 𝑙 50 1 3
 The direction of this force 𝐵 𝐼 𝑙 is given by Fleming’s left hand rule. 𝑰𝑺 = 𝐼𝑆 + 50% 𝐼𝑆 = 𝐼𝑆 [1 + ] = 𝐼𝑆 [1 + ] = 𝑰𝑺 = 1.5 𝐼𝑆
100 2 2
 This force is resolved in to two perpendicular components  Let 𝑉𝑆 be the initial Voltage sensitivity. When the resistance is doubled, then
1) 𝐵 𝐼 𝑙 𝑐𝑜𝑠 45° − along the inclined plane
new voltage sensitivity is
upwards
3
2) 𝐵 𝐼 𝑙 𝑠𝑖𝑛 45° − perpendicular to the 𝐼𝑆 ( ) 𝐼𝑆 3

𝑽𝑺 = 2
incline plane downwards  = 2𝑅 = 𝑽𝑺 = 0.75 𝑉𝑆
4
𝑅𝑔 𝑔
 Similarly, weight 𝑚𝑔 also resolved in to two
perpendicular components.  Hence the voltage sensitivity decreases. The percentage decrease in voltage
1) 𝑚𝑔 𝑐𝑜𝑠 45° − perpendicular to the sensitivity is
incline plane downwards 𝑽𝑺 − 𝑽𝑺 𝑽𝑺 − 𝟎. 𝟕𝟓 𝑽𝑺
𝑿 𝟏𝟎𝟎 % = 𝑿 𝟏𝟎𝟎% = 𝟎. 𝟐𝟓 𝑿 𝟏𝟎𝟎% = 𝟐𝟓 %
2) 𝑚𝑔 𝑠𝑖𝑛 45° − along the inclined plane downwards 𝑽𝑺 𝑽𝑺
 For equilibrium of the rod ; 𝑚𝑔 sin 45 ° = 𝐵 𝐼 𝑙 cos 45 °
𝑚
𝑚𝑔 sin 45 ° ( )𝑔 0.25 𝑋 9.8
∴ 𝐼= = 𝑙 = = 𝟗. 𝟖 𝑨
𝐵 𝑙 cos 45 ° 𝐵 0.25
 So, we need to supply current of 9.8 A to keep the metallic rod stationary.
25. The coil of a moving coil galvanometer has 5 turns and each turn has an
effective area of 2 × 10–2 m2. It is suspended in a magnetic field whose strength
is 4 × 10–2 Wb m–2. If the torsional constant K of the suspension fibre is
4 × 10–9 N m deg–1.
(a) Find its current sensitivity in division per micro - ampere.
(b) Calculate the voltage sensitivity of the galvanometer for it to have full scale
deflection of 50 divisions for 25 mV.
(c) Compute the resistance of the galvanometer.
Solution :𝑛 = 5 ; 𝐴 = 2 𝑋 10−2 𝑚2 ; 𝐵 = 4 𝑋10−2 𝑊𝑏 𝑚−2 ; 𝐾 = 4 𝑋 10−9 𝑁 𝑚 𝑑𝑒𝑔−1
(a) Current sensitivity,
𝜃 𝑁 𝐵 𝐴 5 𝑋 4 𝑋10−2 𝑋 2 𝑋 10−2 1
𝑰𝑺 = = = = −6 𝑑𝑒𝑔/𝐴 = 𝟏 𝒅𝒆𝒈/𝝁𝑨
𝐼 𝐾 4 𝑋 10−9 10
(b) Voltage sensitivity,
𝜃 50
𝑽𝑺 = = = 2 𝑋 103 𝑑𝑒𝑔/𝑣𝑜𝑙𝑡
𝑉 25 𝑋 10−3

victory R. SARAVANAN. M.Sc., M.Phil., B.Ed PG ASST [PHYSICS], GBHSS, PARANGIPETTAI - 608 502
12 PHYSICS UNIT - 3 MAGNETISM AND MAGNETIC EFFECTS OF CURRENT CONCEPTUAL QUESTIONS & ANSWERS

3. A circular coil with cross-sectional area 0.1 cm2 is kept in a uniform magnetic
EXERSICE PROBLEMS WITH SOLUTION field of strength 0.2 T. If the current passing in the coil is 3 A and plane of the
1. A bar magnet having a magnetic moment 𝒑 ⃗ 𝒎 is cut into four pieces i.e., first cut loop is perpendicular to the direction of magnetic field. Calculate
into two pieces along the axis of the magnet and each piece is further cut along (a) total torque on the coil
the axis into two pieces. Compute the magnetic moment of each piece. (b) total force on the coil
Solution : (c) average force on each electron in the coil due to the magnetic field. (The
free electron density for the material of the wire is 1028 m–3).
Solution : 𝑨 = 0.1 𝑐𝑚2 = 0.1 𝑋 10−4 𝑚2 ; 𝐵 = 0.2 𝑇 ; 𝐼 = 3 𝐴 ; 𝑛 = 1028 𝑚−3 ; 𝜃 = 0°
(a) Total torque on the coil
𝜏 = 𝑝𝑚 𝐵 sin 𝜃 = 𝐼 𝐴 𝐵 𝑠𝑖𝑛 𝜃
 Initial magnetic moment of the magnet ; 𝒑 ⃗ 𝒎 = 𝒒𝒎 ⃗𝒅 = 𝒒𝒎 ⃗⃗⃗⃗
𝟐𝒍 𝜏 = 3 𝑋 0.1 𝑋 10−4 𝑋 0.2 𝑋 sin 0° [∵ sin 0° = 0]
 When it cut along the axis into four pieces, the pole strength of each piece 𝝉=𝟎
𝑞
becomes 𝑚 , but the magnetic length will not change. Hence magnetic moment of (b) Total force on the coil,
4
𝐹 = 𝐵 𝐼 𝑙 sin 𝜃
each piece,
𝑞 1 𝟏 𝑭=𝟎 [∵ sin 0° = 0]

⃗𝒎= 𝑚 𝑑
𝒑 ⃗ = (𝑞𝑚 ⃗⃗⃗⃗2 𝑙) = 𝒑 ⃗𝒎 (c) Charge of electron ; 𝑞 = |𝑒| = 1.6 𝑋 10−19 𝐶
4 2 𝟒 𝑙
2. A conductor of linear mass density 0.2 g m–1 suspended If 𝑙 is the length of the coil, the drift velocity ; 𝑣𝑑 =
𝑡
by two flexible wire as shown in figure. Suppose the Lorentz force on each electron,
tension in the supporting wires is zero when it is kept 𝐹𝐵 = 𝐵 𝑞 𝑣 = 𝐵 𝑒 𝑣𝑑 [∵ 𝐼 = 𝑛 𝐴 𝑒 𝑣𝑑 ]
inside the magnetic field of 1 T whose direction is into 𝐼 𝐵𝐼
𝐹𝐵 = 𝐵 𝑒 =
the page. Compute the current inside the conductor and 𝑛𝐴𝑒 𝑛𝐴
0.2 𝑋 3 0.6
also the direction of the current. Assume g = 10 m s–2 𝐹𝐵 = = 𝑋 10−24 = 𝟔 𝑿 𝟏𝟎−𝟐𝟒 𝑵
𝒎 10 28 𝑋 0.1 𝑋 10−4 0.1
−𝟏 −𝟑 −𝟏 −𝟐
Solution : = 𝟎. 𝟐 𝒈 𝒎 = 𝟎. 𝟐 𝑿 𝟏𝟎 𝒌𝒈 𝒎 ; 𝑩 = 𝟏 𝑻 ; 𝒈 = 𝟏𝟎 𝒎 𝒔 4. A bar magnet is placed in a uniform magnetic field whose strength is 0.8 T.If
𝒍
the bar magnet is oriented at an angle 30o with the external field experiences
a torque of 0.2 Nm. Calculate: (a) the magnetic moment of the magnet
(b) the work done by the magnetic field in moving it from most stable
configuration to the most unstable configuration and also compute the work
done by the applied magnetic field in this case.
Solution : 𝐵 = 0.8 𝑇 ; 𝜃 = 30° ; 𝜏 = 0.2 𝑁 𝑚
(a) We know that, the torque ; 𝜏 = 𝑝𝑚 𝐵 sin 𝜃
Hence magnetic moment is given by,
𝜏 0.2 0.2 0.2 2
𝑝𝑚 = = = = =
 Weight of the conductor due to gravity in downward direction, 𝐵 sin 𝜃 0.8 𝑋 sin 30° 1 0.4 4
0.8 𝑋
𝐹𝑔 = 𝑚 𝑔 2
 Magnetic Lorentz force acting perpendicular to conductor in upward direction, 𝒑𝒎 = 𝟎. 𝟓 𝑨 𝒎𝟐
𝐹𝐵 = 𝐵 𝐼 𝑙 (b) For most stable configuration ; 𝜃 = 0° and energy 𝑈𝑆
 When the tension of the supporting wire becomes zero, we have For most unstable configuration ; 𝜃 = 180 and energy 𝑈𝑈𝑆
𝐹𝐵 = 𝐹𝑔
𝐵𝐼𝑙=𝑚𝑔
𝑚
𝑚𝑔 ( )𝑔 0.2 𝑋 10−3 𝑋 10
𝑰= = 𝑙 = = 𝟐 𝑿 𝟏𝟎−𝟑 𝑨 = 𝟐 𝒎 𝑨
𝐵𝑙 𝐵 1
victory R. SARAVANAN. M.Sc., M.Phil., B.Ed PG ASST [PHYSICS], GBHSS, PARANGIPETTAI - 608 502
12 PHYSICS UNIT - 3 MAGNETISM AND MAGNETIC EFFECTS OF CURRENT CONCEPTUAL QUESTIONS & ANSWERS

 From the figure, 6. Calculate the magnetic field at the centre of a square loop which carries a
𝑈𝑆 = − 𝑝𝑚 𝐵 cos 𝜃 = − 𝑝𝑚 𝐵 cos 0° = − 𝑝𝑚 𝐵 current of 1.5 A, length of each side being 50 cm.
𝐿
𝑈𝑈𝑆 = − 𝑝𝑚 𝐵 cos 𝜃 = − 𝑝𝑚 𝐵 cos 180° = − 𝑝𝑚 𝐵 (−1) = 𝑝𝑚 𝐵 Solution : 𝐿 = 50 𝑐𝑚 = 50 𝑋 10−2 𝑚 ; 𝐼 = 1.5 𝐴 𝑎 = = 25𝑐𝑚 = 25 𝑋 10−2 𝑚
2
 Hence the work done by the magnetic field in moving it from most stable 𝜙1 = 𝜙 = 45° ; 𝜙2 = 180° − 𝜙 ; 𝜃 = 45°
configuration to the most unstable configuration is,  Let the square loop is made up of four straight conductors AB, BD, DC and CA
𝑊𝐹 = 𝑈𝑈𝑆 − 𝑈𝑆 = 𝑝𝑚 𝐵 − (− 𝑝𝑚 𝐵 ) = 2 𝑝𝑚 𝐵  From Biot - Savart law, the magnetic field at a
𝑊𝐹 = 2 𝑋 0.5 𝑋 0.8 distance ‘a’ due to straigtht current carrying
𝑾𝑭 = 𝟎. 𝟖 𝑱 conductor AB is,
 Work done by the applied magnetic field, 𝜇𝑜 𝐼
𝐵𝐴𝐵 = [sin 𝜙1 + sin 𝜙2 ]
𝑊𝐵 = 𝑈𝑆 − 𝑈𝑈𝑆 = − 𝑝𝑚 𝐵 − 𝑝𝑚 𝐵 = − 2 𝑝𝑚 𝐵 4𝜋𝑎
𝑊𝐵 = − 2 𝑋 0.5 𝑋 0.8 4 𝜋 𝑋 10−7 𝑋 1.5
𝐵𝐴𝐵 = [sin 45° + sin 45° ]
𝑊𝐵 = − 𝟎. 𝟖 𝑱 4 𝜋 𝑋 25 𝑋 10−2
5. A non - conducting sphere has a mass of 100 g and 10−5 𝑋 1.5
radius 20 cm. A flat compact coil of wire with turns 𝐵 𝐴𝐵 = [2 sin 45° ]
25
5 is wrapped tightly around it with each turns 1
concentric with the sphere. This sphere is placed on 𝐵𝐴𝐵 = 0. 06 𝑋 10−5 𝑋 2 𝑋 = 6 √2 𝑋 10−7 𝑇
√2
an inclined plane such that plane of coil is parallel  Similarly, magnetic field due to BD, DC and CA
to the inclined plane. A uniform magnetic field of 0.5
𝐵𝐵𝐷 = 6 √2 𝑋 10−7 𝑇
T exists in the region in vertically upward direction.
Compute the current I required to rest the sphere in 𝐵𝐷𝐶 = 6 √2 𝑋 10−7 𝑇
equilibrium. 𝐵𝐶𝐴 = 6 √2 𝑋 10−7 𝑇
−2
Solution : 𝑀 = 100 g = 0.1 kg ; R = 20 cm =20 X 10 m ; N = 5 ; B = 0.5 T ; I = ?  From Fleming’s left hand rule, the magnetic field at the centre, due to all four
 Here sphere be at two types of equilibrium. They are, conductors is directed perpendicularly inwards to the plane of the paper. Hence
(i) Straight line equilibrium the total magnetic field
(ii) Rotational equilibrium 𝐵 = 𝐵𝐴𝐵 + 𝐵𝐵𝐷 + 𝐵𝐷𝐶 + 𝐵𝐶𝐴
 Under straight line equilibrium ; 𝑓𝑆 = 𝑀 𝑔 𝑠𝑖𝑛 𝜃 − − − − − − (1) 𝐵 = 4 𝑋 6 √2 𝑋 10−7
 Torque on the sphere due to magnetic field about its centre ; 𝐵 = 24 𝑋 1.414 𝑋 10−7
𝜏 = 𝑝𝑚 𝐵 sin 𝜃 (anticlockwise direction) 𝐵 = 33. 936 𝑋 10−7 = 3.3936 𝑋 10−6 𝑇
Torque on the sphere due to friction about its centre ; 𝑩 = 𝟑. 𝟒 𝑿 𝟏𝟎−𝟔 𝑻
𝜏 = 𝑓𝑆 𝑅 (clockwise direction)
Under rotational equilibrium ; 𝑓𝑆 𝑅 = 𝑝𝑚 𝐵 sin 𝜃 − − − − − − (2)
 Put equation (1) in (2)
𝑀 𝑔 𝑠𝑖𝑛 𝜃 𝑅 = 𝑝𝑚 𝐵 sin 𝜃
𝑀 𝑔 𝑅 = 𝑝𝑚 𝐵 [∵ 𝑝𝑚 = 𝑁 𝐼 𝐴]
𝑀𝑔𝑅 = 𝑁𝐼𝐴𝐵
𝑀 𝑔 𝑅 = 𝑁 𝐼 (𝜋 𝑅2 ) 𝐵
𝑀𝑔
𝐼 =
𝑁𝜋𝑅𝐵
0.1 𝑋 10
𝐼 =
5 𝑋 𝜋 𝑋 20 𝑋 10−2 𝑋 0.5
102 100
𝐼 = =
50 𝑋 𝜋 50 𝑋 𝜋
𝟐
𝑰 = 𝑨
𝝅
victory R. SARAVANAN. M.Sc., M.Phil., B.Ed PG ASST [PHYSICS], GBHSS, PARANGIPETTAI - 608 502
12 PHYSICS UNIT - 3 MAGNETISM AND MAGNETIC EFFECTS OF CURRENT CONCEPTUAL QUESTIONS & ANSWERS

7. A strong horse shoe magnet is able to lift a soft iron bar when two limbs touch
UNIT - 3 CONCEPTUAL QUESTIONS & ANSWERS
its ends. If the two limbs of the magnet is short circuited by another iron strip,
1. A pair of magnets to be stored have two soft iron bars which are kept pressed the first strip falls down. What is the reason?
against their free faces on the two sides of the pair. What are they used for?  When the two limbs of the horse -shoe magnet are short circuited by means of
 The two iron bars are called magnetic keepers. another iron strip, the lines of induction form closed circuits. This results in a
 A magnet which has two free poles is likely to be demagnetized automatically decrease in pole strength at the magnet.
after a long time when left free.  This reduced pole strength is insufficient to hold the first strip against its weight.
 This is due to effect of external field (earth’s magnetic field etc) which exerts a So it falls down.
torque on each individual magnetice dipole, thus diverting it away from the 8. Give the dimension formula for magnetic induction, magnetic moment and pole
regular orientation. strength.
 For that two magnetic keepers are placed against the unlike poles of the two  Dimension of Magnetic Inducion ‘B’ is given by,
magnets thus by making that system as closed chain. 𝐹 [𝑀 𝐿 𝑇 −2 ]
𝐵 = 𝐼𝑙 ⟹ [𝐴][𝐿]
= [𝑴 𝑻−𝟐 𝑨−𝟏 ]
 Now there is no free poles in the system, and this will save the pairs of magnets
from demagnetization.  Dimension of Magnetic moment ‘𝑝𝑚 ’ is given by,
2. A magnet placed in a non uniform magnetic field experiences a force and a 𝜏 𝐹 .𝑥 [𝑀 𝐿 𝑇 −2 ] [𝐿]
torque. How can you explain this?
𝑝𝑚 = = ⟹ = [𝑳𝟐 𝑨]
𝐵 sin 𝜃 𝐵 sin 𝜃 [𝑀 𝑇 −2 𝐴−1 ]
 In non uniform magnetic field, the forces acting on the two poles of the magnet  Dimension of Pole strength ‘’ is given by,
will be unlike and unequal parallel forces. 𝑝𝑚 [𝑳𝟐 𝑨]
 Thus there is combination of a net force and a moment of couple (torque) acting 𝑞 𝑚 = ⟹ [𝐿]
= [𝑳 𝑨]
2𝑙
on the magnet. 9. What are the directions of magnetic moments of two substances A
3. An isolated magnetic pole is not possible. How can we have it if we need it for (diamagnetic) and B (paramagnetic) when they are placed in same
some practical purposes? magnetizing field?
 If we take a sufficiently long magnet, one of its pole can be considered to be  Substance A which is diamagnetic in nature will get magnetized opposite to
located at a large distance from the other. Hence both of its poles can be treated direction of magnetizing field, whereas substance B which is paramagnetic in
as isolated magnetic poles for all practical purposes. nature will get magnetized in the direction on magnetizing field.
4. What will we do if we want to save a sensitive instrument from a magnetic field?  Thus the direction of magnetic moment vectors in the two samples will be
 The instrument should be enclosed in a hollow conductor of soft iron, since there opposite to each other.
cannot be any magnetic lines of force inside the conductor. 10. What type of hysteresis loop do you expect in case of a permanent magnet?
5. A magnetic needle is placed in a cork floating on a still lake in the northern  In case of permanent magnet, both the value of retentivity (it is the ability of the
hemisphere. What will happen to the system? materials to retain the magnetism in them even after the magnetising field
 There is no net translatory motion of the system, since it experience a torque disappears) and coercivity (it is the magnitude of the reverse magnetising field
only. for which the residual magnetism of the material vanishes) are large.
 Due to the torque the system rotates in such a way that the axis of the needle  Therefore the hysteresis loop should be tall and wide.
becomes parallel to the magnetic meridian. 11. What type of hysteresis loop do you expect in case a material forming the core
6. There are two apparently identical soft iron bars ‘A’ and ‘B’ . One of them is of a transformer?
magnetized while the other is not. How will you distinguish between them?  For the core of a transformer, it is essential that the direction of magnetization of
Neither you have a magnetic compass or any other magnetic material nor a the core should get reversed quickly. For this coercivity has to be small.
thread for suspension purposes.  Secondly there should be lesser loss of energy per cycle, so that the core is not
 Place one bar on the table and touch its middle point with one end of the other. heated up. For this the area of the loop should be less.
 If there is attraction, the bar in your hand is magnetized.  At the same time the core should have greater retentivity or remenance.
 If there is no attraction, the bar in your hand is unmagnetised.  Thererfore, the loop should be tall and narrow.
 This is because the fact there is no magnetic effect at the middle of the magnet
and concentrated only in its poles called pole strength.

victory R. SARAVANAN. M.Sc., M.Phil., B.Ed PG ASST [PHYSICS], GBHSS, PARANGIPETTAI - 608 502
12 PHYSICS UNIT - 3 MAGNETISM AND MAGNETIC EFFECTS OF CURRENT CONCEPTUAL QUESTIONS & ANSWERS

12. A bar magnet of magnetic moment’ 𝒑𝒎 ′ and length ′𝟐𝒍′ is broken up in 16. If magnetic monopole exist, how would Gauss’law of magnetgism be modified?
following three different ways . What is the magnetic moment of the shaded 𝑑𝑠 will not be zero , but ∮ ⃗⃗⃗𝑩 . ⃗⃗⃗⃗⃗⃗
 If monopole exist, then the ∮ ⃗⃗⃗𝐵 . ⃗⃗⃗⃗⃗ 𝒅𝒔 = 𝝁𝒐 𝒒𝒎
portion in each case? where 𝑞 is the magnetic charge of the monopole enclosed by the closed path.
𝑚
17. Does a bar magnet exert a torque on itself due to its own field? Does one
element of the current carrying wire exert a force on another element of the
same wire?
 A bar magnet does not exert a torque on itself due to its own field.
 In the case of a wire, there is a force or torque on an element of wire due to the
 Initial magnetic moment of the given bar magnet ; 𝒑𝒎 = 𝒒𝒎 𝟐𝒍
field produced by another element of the same wire. But for a straight wire, the
(i) If it broken as shown in fig (i), its pole strength remains the same while the force will be zero.
length is reduced to half. Therefore the new magnetic moment of 18. Magnetic field arises due to charges in motion. Can a system have magnetic
𝒑
shaded portion is ; 𝒑𝒎 𝑰 = 𝒒𝒎 𝒍 = 𝒎 moment eventhough its net charge is zero?
𝟐
(ii) If it broken as shown in fig (ii), then its pole strength is reduced to half,  Yes. The net charge of a system may be zero, still it can have magnetic moment.
while the length remains the same. Therefore the new magnetic moment of  For example, the charge of a neutron is zero, but it has a non zero magnetic
𝒒 𝒑 moment.
shaded portion is ; 𝒑𝒎 𝑰 = 𝒎 𝟐𝒍 = 𝒎
𝟐 𝟐 19. If earth’s magnetic field is due to a large current loop within the earth, then
(iii) If it broken as shown in fig (iii), then its pole strength is reduced to half and
what is plane of the loop and the direction of the loop?
the length also reduced to half. Therefore the new magnetic moment of
𝒒 𝒒 𝒑  Since the earth’s magnetic field is along north - south plane, the current loop
shaded portion is ; 𝒑𝒎 𝑰 = 𝒎 𝒍 = 𝒎 𝟐𝒍 = 𝒎 must be in the east-west plane (i.e.) nearly in the equatorial plane.
𝟐 𝟒 𝟒
13. When a charged particle moves in a magnetic field the work done by the  According to right hand thumb rule, the current should be flowing in the
magnetic force on the charged particle is zero. Why? But when an iron nail held clock -wise direction as seen from the geographic northpole.
near a magnet is released, the nail moves towards the magnet and cling to it. 20. Given that the horizontal and vertical components of the earth’s magnetic field
There is an increase in the kinetic energy (i.e) work done on the nail due to the are equal at a place. What is the angle of dip at the place?
attraction. How will you explain this?  When horizontal component (𝐵𝐻 ) and vertical component (𝐵𝑉 ) of earth’s
 The work done by the magnetic Lorentz force is zero, because the force is 𝑩
manetic field are equal (i.e.) 𝐵𝐻 = 𝐵𝑉 ,then 𝐭𝐚𝐧 𝑰 = 𝑯 = 𝟏 (𝑜𝑟) 𝑰 = 𝟒𝟓°
𝑩𝑽
perpendicular to the velocity of the charged particle. Hence the magnitude of
the velocity remains constant and the direction of velocity alone changes and 21. How will compass needle stay, if it is kept on the magnetic north pole of the
that force makes the charged particle to move along a circular path. earth?
 But in the case of iron nail, it can be assumed to be made up of number of  It will stay in any direction, because at poles the angle of dip is ; 𝑰 = 𝟗𝟎° and
magnetic dipoles arising due to the orbital and spin motion of the electrons hence only the vertical component of earth’s magnetic field acts.
in the nail. These dipoles are placed in a non-uniform magnetic field. So they 22. A vector needs three quantities for its specification.Name the three independent
experience a net force which increases the kinetic energy of the iron nail. quantities conventionally used to specify the earth’s magnetic field.
14. A magnetic needle placed in uniform magnetic field experience a torque but no  A quantity to be a vector, then it must have both magnitude and direction and
net force. But an iron nail placed near a bar magnet experience a force of also it will obeys the laws of vector addition.
attraction in addition to torque. Why?  Similarly the three quantities required to specify earth’s magnetic field are
 Due to the bar maget, the iron nail experieces a non-uniform magnetic field. The i. Magnetic decienation (D)
induced magnetic moment in the nail therefore experiences both force and ii. Magnetic dip or inclination (I)
torque. The net force is attractive, because the induced southpole in the nail is iii. The horizontal component of earth’s magnetic field (𝐵𝐻 )
closer to the north pole of the magnet than the induced northpole. 23. Interstellar space has an extremely weak magnetic field of the order 𝟏𝟎−𝟏𝟐 𝑻 .
15. Can you think of a magnetic field with three poles? Can such weak field be of any significant consequence? Explain.
 Yes. Bring two bar magnets such that the two northpoles are together.  A charged particle entering a uniform field ‘B’ with velocity ‘𝑣’ trace a circular
𝒎𝒗
 Fix the two northpoles using glue, we have magnet with south-north-south pole. path of radius ‘r’ given by, = . If ‘B’ is small, ‘r’ is very large. So the charged
𝑩𝒒
particle traces circular path of large radius. For a small distances, this may not be
noticeable, but over the inter-stellar distances, it is of significant value.
victory R. SARAVANAN. M.Sc., M.Phil., B.Ed PG ASST [PHYSICS], GBHSS, PARANGIPETTAI - 608 502
12 PHYSICS UNIT - 3 MAGNETISM AND MAGNETIC EFFECTS OF CURRENT CONCEPTUAL QUESTIONS & ANSWERS

24. Jupiter’s magnetic field is stronger than earth’s magnetic field, But Venus 30. How will the reduction factor of a tangent galvanometer change if the number
magnetic field is weaker than Earth’s magnetic field. Why? of turns ‘N’ of the coil is doubled?
 The magnetic field of a planet is related to its rate of rotation. 𝟐 𝒓 𝑩𝑯
 The reduction factor is given by, 𝑲 =
𝝁𝒐 𝑵
 Jupiter rotates faster, whereas Venus rotates slower than our Earth.
 So Jupiter’s magnetic field is stronger and Venus magnetic field is weaker than  If number of turns are doubled (𝑁 → 2𝑁), then the reduction factor becomes
𝟐 𝒓 𝑩𝑯 𝑲
our Earth’s magnetic field. 𝑲𝑰 = 𝝁𝒐 (𝟐𝑵)
= 𝟐
(i.e.) halved
25. The angle of dip at two places on the surface of the Earth are respectively
31. Why does a paramagnetic sample display greater magnetization (for the same
𝟎° and ° , where are these places located?
magnetizing) when cooled?
 The angle of dip 𝐼 = 0° , implies that the Earth’s magnetic field is paralled to
 At low temperature the random thermal motion which distrupts the alignment
the surface of the Earth (i.e.) horizontal. This is correspondace to Magnetic of dipoled is reduced and hence the paramagnetic sample displays magnetization
equator. to a greater degree.
 The angle of dip 𝐼 = 90° , implies that the Earth’s magnetic field is 32. Alnico is preferred for making permanent magnet. Why?
perpendiculr to the surface of the Earth (i.e.) vertical. This is correspondace to  The materials with high retentivity, high coercivity and high permeability are
Magnetic poles. suitable for making permanent magnets.
26. The magnetic needle used in tangent galvanometer (TG) is very small. Why?
 Since Alnico has (i) high retentivity (ii) high coercivity and (iii) area of the
 The magnetic field produced by the current carrying circular coil at its centre is hysteresis loop is large, it is preferred for making permanent magnets.
uniform only over a small area, So the magnetic needle is snall so that it remains 33. When a ferromagnetic material is placed in a magnetic field, the flux density
in an uniform filed even in deflected position. inside the material is greater than that outside. Why?
27. Before performing experiment using a tangent galvanometer, the plane of its
 Because the susceptibility 𝜒𝑚 (which measures how easily and how strongly a
coil is adjusted in the earth’s magnetic meridian. Why?
material can be magnetized) and permeability 𝜇 (which measure of ability of
 The magnetic field (B) produced by a current carrying circular coil is the material to allow the passage of magnetic field lines through it) of ferro
perpendicular to the palne of the coil. magnetic materials are very high.
 Hence the plane of the circular coil is placed along the magnetic meridian (i.e.) 34. Why diamagnetism almost independent of temperature?
parallel to the horizontal component of Earth’s magnetic field (B H), so that the
 In a diamagnetic substance, the induced dipole moment always opposite to the
magnetic field produced by the coil (B) is perpendicular to the horizontan magnetizing field and it does not depend on the internal thermal motion of the
component of Earth’s magnetic field (BH), dipoles.
 Under this condition only, the pivoted magnetic needle in the compass box is 35. A length of wire carries a steady current. It is bent first to form a circular plane
subjected to two crossed fields and it deflected through an angle ‘’ according to coil of one turn. The same length is now bent more sharply to give a double
the tangent law. loop of smaller radius. How does the magnetic induction at the centre changes?
28. The value of the dip at a place in south India is 18. Will it be more or less in 𝝁 𝑵𝒊
England?  Magnetic induction in the first case ; 𝑩𝟏 = 𝒐
𝟐𝒓
𝑟
 We know that dip ‘I’ is the angle substended by the Earth’s total magnetic field  If it bent more sharply to give double loop, then, 𝑁 → 2𝑁 & 𝑟 → . So the
2
(𝐵⃗ ) with the horizontal direction in the magnetic meridian. The value of dip is 0° 𝝁𝒐 (𝟐𝑵) 𝒊 𝝁 𝑵𝒊
new magnetic induction at the centre is ; 𝑩𝟐 = 𝒓 =𝟒 [ 𝒐𝟐 𝒓 ] = 𝟒 𝑩𝟏
at the equator and 90° at the poles 𝟐( )
𝟐
 England is nearer to the poles and south India is nearer to the equator. So the 36. A constant voltage is applied to the opposite corners of one diagonals of a cube
value of dip in England will be more than that in south India. formed by means of homogeneous wire frame. What will be the strength of
29. In India, declination angle is very small and for Chennai, magnetic declination magnetic field at the centre of the cube?
angle is –1 16ʹ. Why it is very small and negative value?  When a constant potential difference is applied to the opposite ends of diagonal
 Declination or magnetic declination (D) is the angle between magnetic meridian of the cube, same current flows through each wire. Corresponding to one wire,
at a point and geographical meridian. It is large at higher latitudes and small there exists another wire carrying current in the same direction. Therefore the
near the equator. resultant magnetic field at the centre of cube is zero.
 Chennai is nearer to equator and so declination angle at Chennai is very small.
 At Chennai, the magnetic meridian lies on west side of geographic meridian so it
is negative.
victory R. SARAVANAN. M.Sc., M.Phil., B.Ed PG ASST [PHYSICS], GBHSS, PARANGIPETTAI - 608 502
12 PHYSICS UNIT - 3 MAGNETISM AND MAGNETIC EFFECTS OF CURRENT CONCEPTUAL QUESTIONS & ANSWERS

37. The lower end of a soft spiral spring just dips in a trough of mercury. The spiral 42. A charged particle goes undeviated through a region, simultaneously subjected
can be connected to a D.C source through the mercury. What happens when the to an electric field and a magnetic field. What is the relative orientation of the
key is closed? two fields?
 When the key is closed, the current flows through each loop of the spiral is same.  For charged particle to be undeviated, the two fields should be uniform and
 This results in attraction between the consecutive loops and hence the spial acting in directions mutually perpendicular to each other.
contracts. So its free end comes out of mercury, thereby breaking the circuit. 43. Does the application of a uniform magnetic field to a moving charged particle
 Now the attractive force vanishes, restoring the spiral back to its original shape result in a change in kinetic energy of the particle?
and the connection is made again and repeating the above process.  The force acting on the charged particle moving in uniform magnetic field is,
 Threrfoe the free end of the spiral keeps on moving up and down. ⃗⃗⃗𝑭 = 𝒒 (⃗⃗⃗𝒗 𝑿 ⃗𝑩
⃗ ). Here ⃗⃗⃗𝑭 is always perpendicular to both ⃗⃗⃗𝒗 and ⃗⃗⃗𝑩
38. Two long thin straight wires each carrying a current of 10 A are placed at right  Since the force is right angles to the motion of charged particle, the field does not
angles to each other. Calculate the magnetic field at a point having do any work on the particle. Therefore its kinetic energy throughout remains
co-ordinates (5m, 2m) constant.
 Magnetic field at ‘P’ due to current carrying 44. Two particles ‘X’ and ‘Y’ having equal charge ‘q’, after being accelerated
conductor ‘X’ through the same potential difference ‘V’ enter a region of uniform magnetic
−7
⃗⃗𝐵1 = 𝜇𝑜 𝑖 𝑘 ̂ = 4𝜋𝑋10 𝑋 10 𝑘 ̂ = 10 𝑋 10−7 𝑘̂ field ‘B’ and describe circular paths of radii 𝑹𝟏 and 𝑹𝟐 respectively. What will
2 𝜋 𝑟1 2 𝜋 (2) be the ratio of their masses?
 Magnetic field at ‘P’ due to current carrying  Here kinetic energy is provided by electrostatic potential energy of the charge.
conductor ‘Y’ 1 2𝑞𝑉 𝟐𝒒𝑽
−7 (i.e.) 𝑚 𝑣2 = 𝑞 𝑉 (𝑜𝑟) 𝑣 2 = (𝑜𝑟) 𝒗= √
⃗⃗𝐵2 = 𝜇𝑜 𝑖 (−𝑘 ̂ ) = 4𝜋𝑋10 𝑋 10 (−𝑘 ̂ ) = − 4 𝑋 10−7 𝑘
̂ 2 𝑚 𝒎
2 𝜋 𝑟2 2 𝜋 (5)
 Also the necessary centripetal force is provided by magnetic Lorentz force
 The resultant magnetic field at ‘P’ is
𝑚 𝑣2 𝑚𝑣 𝑚 𝟐𝒒𝑽 𝟏 𝟐𝒎𝑽 𝟐𝒎𝑽
⃗⃗⃗𝐵 = 𝐵
⃗1 +𝐵 ̂
⃗ 2 = 10 𝑋 10−7 𝑘̂ − 4 𝑋 10−7 𝑘̂ = 𝟔 𝑿 𝟏𝟎−𝟕 𝒌 (i.e.) = 𝐵 𝑞 𝑣 (𝑜𝑟) 𝑅 = = √ = √ (𝒐𝒓) 𝑹𝟐 = 𝟐
𝑅 𝐵𝑞 𝐵𝑞 𝒎 𝑩 𝒒 𝑩 𝒒
 Thus the magnitude of the resultant field at ‘P’ is 𝟔 𝑿 𝟏𝟎−𝟕 𝒕𝒆𝒔𝒍𝒂 and its 𝒎𝟏 𝑹𝟏𝟐
𝟐 𝟐 𝟐
direction is along positive Z axis  Hence 𝒎 ∝ 𝑹 (𝑜𝑟) = 𝟐 (𝑜𝑟) 𝒎𝟏 ∶ 𝒎𝟐 = 𝑹𝟏 ∶ 𝑹𝟐
𝒎𝟐 𝑹𝟐
39. A wire AB is bent in the form of an arc of radius ‘r’ having centre at ‘O’. 45. A charged particle ‘P’ enter a region from let to right in which a uniform
Calculate the magnetic induction at ‘O’, if the wire subtends an angle of  at its magnetic field has been applied perpendicular to the plane of the paper
centre. towards the reader deflected upwards in the plane of the paper. Determine the
 We know that, the magnetic induction at centre ‘O’ due to current carrying nature of the charge on the particle?
𝜇𝑜 𝑖
circular loop ; 𝐵 =
2𝑟
 The magnetic force on charged particle moves in a uniform magnetic field is
𝜃 ⃗⃗⃗𝑭 = 𝒒 (⃗⃗⃗𝒗 𝑿 ⃗⃗⃗𝑩) . According cross product rule, the force should be directed
 Since the given wire has a length which times the circumference of the
360
𝝁 𝒊 𝜽 downwards along the paper.
circular loop, the magnetic induction at centre of the arc AB is ; 𝑩𝑰 = 𝒐 [ ]  Since the charged particle ‘P’ deflected upwards, the force must be acts upwards
𝟐 𝒓 𝟑𝟔𝟎
40. A charged particle moves with uniform velocity in a certain region. Does it and it is possible only if the nature of the charge on the particle ‘P’ must be
means that the region is free from magnetic field? negative.
 Not necessary. When the charged particle moves parallel or antiparallel to the 46. A uniform electric field and a magnetic field act in the same direction. A proton
direction of lines of force of uniform magnetic field, there is no resultant force on is projected to the space with uniform velocity in opposite direction. What will
the charge. In this case also the charge moves with constant velocity. happen to proton?
41. A charged particle undergoes uniform circular motion in a plane where there is  Since motion of proton is along the lines of force of magnetic field, there is no
a non uniform magnetic field. What is the angle between direction of motion of force on it due to the magnetic field.
charge and the lines of force of the field?  But due to electric field, proton will suffer retardation and hence its velocity will
 Charged particle can execute uniform circular motion, only if it experience a decrease till it comes to rest.
force at right angles to the direction of motion called centripetal force.  Then the proton will come back along the same path and shall emerge from the
 For this happen, the charge must move right angles to the lines of force of the space with same speed with which it was projected.
magnetic field.
victory R. SARAVANAN. M.Sc., M.Phil., B.Ed PG ASST [PHYSICS], GBHSS, PARANGIPETTAI - 608 502
12 PHYSICS UNIT - 3 MAGNETISM AND MAGNETIC EFFECTS OF CURRENT CONCEPTUAL QUESTIONS & ANSWERS

47. A proton is moving under the influence of a perpendicular magnetic field ‘B’ 51. Two straight conductors carring currents 𝒊𝟏 and 𝒊𝟐 are oriented at right angles
and possesses energy ‘E’. What will be the energy of the proton if the magnetic to each other. What will be the force between them if the two conductors lie in
filed is increased to ‘4B’ while it is compelled to move in the circular path of same plane?
same radius?  In such a case, the magnetic field due current in one conductor is parallel to the
 The force acting on the proton moves in magnetic field ‘B’ is ; 𝑭 = 𝑩 𝒆 𝒗 length of second conductor. Since 𝜃 = 0°, the force [𝐹 = 𝐵 𝐼 𝑙 sin 𝜃 ] between
 Since this force acts at right angles to the motion of proton, it becomes them will be zero
𝑚 𝑣2 𝑩𝒆𝒓 52. A rectangular loop ABCD carrying current as shown in
centripetal force. Hence, = 𝐵 𝑒 𝑣 (𝑜𝑟) 𝒗 = 𝒎
𝑟 the figure is placed near a fixed straight conductor XY
𝐼
 Let 𝑣 be the new velocity of proton, when magnetic field is increased to ‘4B’, carrying current. What will happen to the current loop?
𝟒𝑩𝒆𝒓
then ; 𝒗 =𝑰
=𝟒𝒗  Here the segments AB and CD are right angles to XY.
𝒎
1 2 1 1 So the magnetic field due to the current in XY is parallel
 New kinetic energy ; 𝑬𝑰 = 𝑚 𝑣 𝐼 = 𝑚 (4 𝑣)2 = 16 [ 𝑚 𝑣2 ] = 𝟏𝟔 𝑬
2 2 2 to AB and CD. Therefore these two segments will not
48. An electron is shot in steady electric and magnetic field such that its experience any force
velocity ‘v” electric field ‘E’ and the magnetic field ‘B’ are mutully  But the magnetic field due to the current in XY is perpendicular to both
perpendicular. Now if it so happens that Lorentz force cancels the electrostatic AD and BC, they experience a force. Force on AD will be attractive while BC will
force on the electron, what is the velocity of the electron? be repulsive.
 Magnetic Lorentz force acting on electron; 𝑭𝑩 = 𝑩 𝒆 𝒗  Since AD is located near XY than BC, attractive force is stronger than repulsive
Electrostatic force acting on electron ; 𝑭𝑬 = 𝒆 𝑬 force. Therefore rectangular loop is move closer to the fixed straight conductor.
 Since the two fields are perpendicular to each other, the forces due to these two 53. A direct current flows through a cylindrical conductor. Will current density be
fields must be on the same plane. constant across the entire cross section of the conductor?
 Given that, these two forces are opposed to each other in same plane, their  Here currents flows through various elements of the conductor in same
𝑬 direction. Thus there exists a force of attraction between different current
magnitudes must be equal. (i.e.) 𝐹𝐵 = 𝐹𝐸 𝑜𝑟 𝐵 𝑒 𝑣 = 𝑒 𝐸 𝑜𝑟 𝒗 =
( ) ( )
𝑩
elements. Therefore the density of current increases a little near the axis of the
49. An electron and a proton possessing equal momenta are injected to a region at
cylindrical conductor.
right angles to a uniform magnetic field. Calculate the ratio of their radii of
54. A hollow copper pipe leads a lighting conductor to the earth. It was observed
curvature while moving inside a magnetic field.
that the pipe turned into a rod after the lightning struck. Explain.
 Since the electron and proton possess same momentum,
𝒗𝑷 𝒎𝒆  Due to sudden flow of charge from lightning conductor to the earth, large
𝑚𝑃 𝑣𝑃 = 𝑚𝑒 𝑣𝑒 (𝑜𝑟) = instantaneous currents started flowing through various elements of hollow pipe
𝒗𝒆 𝒎𝑷
 The force due to magnetic field provides centripetal force and hence in same direction. This resulted in mutual attraction between various elements
𝑚 𝑣2 𝐵𝑒𝑟 of pipes which in turn resulted in formation of a solid rod.
=𝐵𝑒𝑣 (𝑜𝑟) 𝑣= 𝑚 55. If the polarity of Hall emf in a material is opposite to that obtained from a
𝑟
𝐵 𝑒 𝑟𝑃 𝐵𝑒𝑟 𝒗 𝒎 𝑟
 Thus we have, 𝑣𝑃 = & 𝑣𝑒 = 𝑚 𝑒 ∴ 𝒗𝑷 = 𝒎𝒆 𝑟𝑃 copper strip, what conclusion can you draw?
𝑚𝑃
𝒎 𝒎 𝑟
𝑒
𝑟
𝒆 𝑷 𝑒  In copper conductor, current carriers are electrons.
 Comparing the two equations, 𝒎 𝒆 = 𝒎𝒆 𝑟𝑃 (𝒐𝒓) 𝑟𝑃 = 𝟏 ∴ 𝒓𝑷 ∶ 𝒓𝒆 = 𝟏 ∶ 𝟏  If the Hall emf is opposite to that obtained for a copper strip, this means that
𝑷 𝑷 𝑒 𝑒
50. Two parallel straight conductors carry currents of same magnitude and in current carriers are positively charged particle. This is the case for P - type
same direction. Will there be any point having zero magnetic field intensity? If semiconductor in which charge carriers are holes.
so, where? 56. Is it possible to accelerate electrons and neutrons using cyclotron? If no, why?
 If two straight parallel conductors are separated by a distance ‘a’, then the  Eventhough electron have negative charge, its mass is very very small compared
𝒂 to protons and nertrons. So at high frequencies, relativistic variation of mass of
point ‘P situated in between the two conductors at a distance from either of
𝟐
the electron is appreciable and hence electrons cannot be accelerated by
the conductor have zero magnetic intensity.
cyclotron.
 Since the currents in them are equal, the magnitude of magnetic fields at ‘P’ due to
 Only charged particls can be accelerated by cyclotron. Since netrons are neutral
the two conductors are equal, while their directions are opposite.
𝒂 partile, they cannot be accelerated by cyclotron.
 Therefore the resultant magnetic field at ‘P’ at a distance from either side of
𝟐
conductor is zero.
victory R. SARAVANAN. M.Sc., M.Phil., B.Ed PG ASST [PHYSICS], GBHSS, PARANGIPETTAI - 608 502
12 PHYSICS UNIT - 3 MAGNETISM AND MAGNETIC EFFECTS OF CURRENT CONCEPTUAL QUESTIONS & ANSWERS

57. What is current element? Give its significance. 63. Two parallel wires carrying current in same direction attracts each other. But
 The product of current ‘I’ and the geomentrical length ‘𝑑𝑙 ⃗⃗⃗ ’ is called the current two electron beams moving in the same direction repels each other. Why?
element (i.e.) 𝑰 𝒅𝒍 ⃗⃗⃗⃗  In case of two parallel conductors carrying current, they attracts each other due
 This can be considered as the source of magnetic field, just like an electric charge to magnetic Lorentz force.
(q) considered as source of electric field.  But in the case of electron beam, there is both electric and magnetic forces. Here
58. Find the magnetic field for the paths 1, 2 and 3 electric force dominates and hence they repels each other.
show in the figure by applying Ampere’s 64. What is meant by radial field? Why such a field is necessary in a moving coil
circuital law. galvanometer?
 Let ‘I’ be the current flows through thewire.  A radial field is one in which the field lines in the gap between the hemi-spherical
 According to Ampere’s circuital law, pole pieces appears to radiate from the axis of the cylinder.
 When the field is radial, the plane of the coil in the moving coil galvanometer will
For path 1 ; ∮ ⃗⃗⃗𝐵 . ⃗⃗⃗
𝑑𝑙 = 𝜇𝑂 𝐼
be always parallel to the magnetic lines of force, whatever be the position of
For path 2 ; ∮ 𝐵 . ⃗⃗⃗
⃗⃗⃗ 𝑑𝑙 = 𝜇𝑂 𝐼 the coil. So the deflecting torque is always maximum [𝜏𝑚𝑎𝑥 = 𝑁 𝐵 𝐼 𝐴]
⃗⃗⃗ = 0 (because net current is zero)
For path 3 ; ∮ ⃗⃗⃗𝐵 . 𝑑𝑙 65. In a moving coil galvanometer, the coil is suspended using phosphor-bronze
59. An electron is deflected both in the electric and magnetic fields. Mention the alloy. Why?
differences between the deflections.  The restoring couple per unit twist is small for phosphor - bronze alloy. Hence
 In electric field, the force acts along the direction of the field and hence electron the galvanometer is more sensitive.
get deflected in the same plane. But in magnetic field, the force acts in a plane  Tensile strength is high and hence thin wires can be used to suspend the coil
perpendicular to the velocity and hence electron deflected in a plane such that the wire does not break under the weight of the coil.
perpendicular the both velocity and the field.  The alloy does not get rusted and hence it is independent of the weather
 The path is parabolic in electric field and it is circular in magnetic field. conditions.
 There is increase in kinetic energy of the particle when it moves in electric field. 66. What is the purpose of the soft iron core used in the moving coil galvanometer?
But there is no change in kinetic energy in an magnetic field.  To make the magnetic field radial so that a linear scale may be used to measure
60. There is no net charge in a current carrying conductor. Then why it current.
experiences a force in a magnetic field?  It concentrates the lines of force and thereby increases the strength of the
 Eventhough the net charge in a conductor is zero, it contains number of free magnetic field. This inturn increases the sensitivity of the galvanometer.
electrons which moves with drift velocity results electric current. 67. The earth’s magnetic field do not affect the working of a moving coil
 Thus when current carrying conductor is subjected to magnetic field, every galvanometer. Why?
drifted electron experiences magnetic force which is transmitted to the  The earth’s magnetic field is weak. The magnetic field in which the coil is
conductor. suspended is very strong. So the earth’s field do not affect the working of the
61. A solenoid tends to contract when a current is passed through it. Why? moving coil galvanometer.
 The currents in the adjacent parallel turns of the solenoid flow in the same 68. Which has higher resistance, an ammeter or a voltmeter? Why?
direction.  Ammeter has low resistance, because it has a low resistance called shunt is in
 We know that two parallel current carrying conductors attracts each other if the parallel with the galvanometer. The resistance of ideal ammeter is zero.
currents are in the same direction.  Voltmeter has high resistance, because it has a high resistance is in series with
 So there is a force of attractiohn between the adjacent turns and hence the the galvanometer. The resistance of ideal voltmeter is infinity.
solenoid tends to contract. 69. Which of the two has lesser resistance milli ammeter or ammeter?
62. A current is passed though a copper pipe. A straight wire carrying current is  To convert a galvanometer in to an ammeter of range ‘I’, the low resistance to
initially placed inside the pipe and then outside the pipe. Where the straight 𝑰𝒈
be connected in parallel to it is , 𝑺 = 𝑮
wire experience a force? Why? 𝑰− 𝑰𝒈
 Inside the copper pipe there is no magnetic field. So the straight wire will not  Here, 𝑰𝒈 and 𝑮 are same for both cases. So ‘S’ will be low, when range ‘I’ is high.
experience any force.  The range of milli-ammeter is low and ammeter is high. So ammeter has low
 But outside the pipe, there exist a definite value of magnetic field. So the straight resistance than milli-ammeter.
wire placed outside experience a force.

victory R. SARAVANAN. M.Sc., M.Phil., B.Ed PG ASST [PHYSICS], GBHSS, PARANGIPETTAI - 608 502
12 PHYSICS UNIT - 3 MAGNETISM AND MAGNETIC EFFECTS OF CURRENT CONCEPTUAL QUESTIONS & ANSWERS

70. By connecting suitable high resistance in series, two identical galvanometers 76. Why a freely suspended bar magnet in your laboratory experiences only
are converted in to (i) a voltmeter and (ii) a mill-voltmeter. Which has higher torque (rotational motion) but not any translatory motion even though Earth
resistances? has non-uniform magnetic field? Suppose we keep a freely suspended bar
 To convert a galvanometer in to a voltmeter of range ‘V’, the high resistance to magnet in a non-uniform magnetic field. What will happen?
𝑽  It is because Earth’s magnetic field is locally (physics laboratory) uniform.
be connected in series is, 𝑹 = − 𝑮 . Here, 𝑰𝒈 and 𝑮 are same for both
𝑰𝒈
 It will undergo translatory motion (net force) and rotational motion (torque).
cases.So ‘R’ will be high, when range ‘V’ is high. 77. Neutrons cannot be accelerated by cyclotron because it is electrically neutral
 The range of milli-voltmeter is low and voltmeter is high. So voltmeter has particle. Then how we use it in fast-neutron cancer therapy?
higher resistance.  Deutrons (bundles of one proton and one neutron) can be accelerated because it
71. We have the zero position of the scale of a galvanometer in the centre of scale, has same charge as that of proton. When a deutron is bombarded with a
while those of ammeter and voltmeter is on one side. Why is it so? beryllium target, a beam of high energy neutrons are produced.
 Galvanometer is a current detecting instrument. We have the zero position of  These high-energy neutrons are sent into the patient’s cancerous region to break
the scale of a galvanometer in the centre, so that current flowing in either the bonds in the DNA of the cancer cells (killing the cells). This is used in
direction could be detected. treatment of fast-neutron cancer therapy.
 Voltmeter and ammeter are measuring instruments. To use them we already 78. An electron and a proton move with the same speed in a plane perpendicular
know the direction of current flowing through the circuit. We can always to a uniform magnetic field. Compare the magnetic forces on these particles,
connect them in such a manner that deflection is obtained only in one direction. acclerations, radii and periods of their orbits.
Therfore we have the zero position of the scale on one side.  The mass of the proton is 1836 times greater than that of the mass of elctron
 For the same reason, terminals of voltmeter and ammeter are marked ‘+’ and ‘-‘  The magnitude of magnetic forces [𝑭𝑩 = 𝑩 𝒆 𝒗 ] on electron and proton are the
while those galvanometers are not. same, since they have the same amount of charge. But the direction of these forces
72. Why an ammeter always connected in series with the circuit? are opposite of each other.
 An ammeter is a low resistance device. When it connected in series, it does not 𝑭 𝑩𝒆𝒗
 The acceleration [𝒂 = 𝑩 = ] are in opposite direction and the electron has a
produce any effective change in resistance or current in the circuit. Therefore it 𝒎 𝒎
is always connected in series with the circuit. larger acceleration than the proton due to its smaller mass.
73. Why a voltmeter always connected in parallel with the circuit?  The magnetic force acts at right angles to the motion of charged particle provides
𝒎𝒗
 A voltmeter is a high resistance device. When it connected in parallel, it does centripetal force and make the particle to move along a circle of radius ; 𝒓 =
𝑩𝒆
not produce any effective change in resistance or current in the circuit. 𝟐𝝅𝒓 𝟐𝝅𝒎
Therefore it is always connected in parallel with the circuit. The time period (time for one complete rotation) is given by ; 𝑻 = 𝒗
= 𝑩𝒆
74. By mistake, a voltmeter is connected in series and an ammeter is connected in  Proton has more mass than the electron ; therefore its radius and period will be
parallel with a resistance in an electrical circuit. What will happen to the larger.
instruments? 79. Discuss the similarities and differences between the electrical force onf a
 Voltmeter is a high resistance instrument. When it connected in series with a charge and the magnetic force on a charge.
resistance, the effective resistance of the circuit will become very high and no  Both electric force and magnetic force are field dependent.
current will flow through the circuit and so no voltage drop occur. On other  Electrical force dependent on charge, whereas magnetic force is dependent on
words, it will lead to an open circuit and the meter will be safe. current or rate of flow of charge
 Ammeter is a low resistance instrument. When it connected in parallel with a
resistance, we will be shorting that element. Because of this, the current
through the circuit will increase which will definitely damage the meter
75. How earth’s magnetic field helps to birds and animals?
 Many birds and animals have magnetic sense in their eyes using Earth’s
magnetic field for navigation.
 For example, Zebra finch bird, due to protein cryptochromes Cry4 present in
retina; the bird uses Earth's magnetic field for navigation

victory R. SARAVANAN. M.Sc., M.Phil., B.Ed PG ASST [PHYSICS], GBHSS, PARANGIPETTAI - 608 502
12 PHYSICS UNIT - 3 MAGNETISM AND MAGNETIC EFFECTS OF CURRENT CONCEPTUAL QUESTIONS & ANSWERS

EXAM NO 7. Two identical coils, each with N turns and radius R are placed coaxially at a
NAME : distance R as shown in the figure. If I is the current passing through the loops
UNIT - 3 MAGNETISM AND MAGNETIC EFFECTS OF CURRENT in the same direction, then the magnetic field at a point P at a distance of R/2
from the centre of each coil is
Time - 2 : 30 hours Total - 60 marks 8 𝑁 𝜇𝑜 𝐼
(a)
PART - I 15 X 1 = 15 √5 𝑅
8𝑁𝜇 𝐼
Note : (i) Answer all the questions (b) 3/2𝑜
5 𝑅
8 𝑁 𝜇𝑜 𝐼
(ii) Choose the best answer and write the option code and (c)
5𝑅
corresponding answer 4 𝑁 𝜇𝑜 𝐼
1. The magnetic field at the centre O of the following (d)
√5 𝑅
current loop is 8. A wire of length l carrying a current I along the Y direction is kept in a
𝜇𝑜 𝐼 𝜇𝑜 𝐼
(a)  (b)  ⃗⃗ = 𝜷 (𝒊̂ + 𝒋̂ + 𝒌
magnetic field given by 𝑩 ̂ ) . The magnitude of Lorentz force
4𝑟 4𝑟 √𝟑
𝜇𝑜 𝐼 𝜇𝑜 𝐼 acting on the wire is,
(c)
2𝑟
 (d) 2 𝑟 
2 1 1
2. An electron moves in a straight line inside a (a) √ 𝛽 𝐼 𝑙 (b) √ 𝛽 𝐼 𝑙 (c)√2 𝛽 𝐼 𝑙 (d)√ 𝛽 𝐼 𝑙
charged parallel plate capacitor of uniform 3 3 3
charge density σ. The time taken by the electron 9. A bar magnet of length l and magnetic moment pm is
to cross the parallel plate capacitor undeflected bent in the form of an arc as shown in figure. The new
when the plates of the capacitor are kept under magnetic dipole moment will be
3
constant magnetic field of induction 𝑩⃗⃗ is (a) 𝑝𝑚 (b) 𝑝𝑚
𝜋
𝑒𝑙𝐵 𝑙𝐵 𝑙𝐵 𝑙𝐵 2 1
(a) 𝜀𝑜 (b) 𝜀𝑜 (c) 𝜀𝑜 (d) 𝜀𝑜 (c) 𝑝𝑚 (d) 𝑝𝑚
𝜎 𝜎𝑙 𝑒𝜎 𝜎 𝜋 2
3. A particle having mass m and charge q accelerated through a potential 10. A non-conducting charged ring carrying a charge of q, mass m and radius r is
difference V. Find the force experienced when it is kept under perpendicular rotated about its axis with constant angular speed ω. Find the ratio of its
⃗⃗ .
magnetic field 𝑩 magnetic moment with angular momentum is
𝑞 2𝑞
2 𝑞3 𝐵 𝑉 𝑞3 𝐵2 𝑉 2 𝑞3 𝐵2 𝑉 2 𝑞3 𝐵 𝑉 (a) (b)
(a)√ (b) √ (c) √ (d) √ 𝑚 𝑚
𝑚 2𝑚 𝑚 𝑚3 𝑞 𝑞
(c) (d)
4. A circular coil of radius 5 cm and 50 turns carries a current of 3 ampere. The 2𝑚 4𝑚
magnetic dipole moment of the coil is nearly 11. The BH curve for a ferromagnetic material
(a) 1.0 A m2 (b) 1.2 A m2 is shown in the figure. The material is
(c) 0.5 A m2 (d) 0.8 A m2 placed inside a long solenoid which
5. A thin insulated wire forms a plane spiral of N = 100 tight turns carrying a contains 1000 turns/cm. The current that
current I = 8 m A (milli ampere). The radii of inside and outside turns are should be passed in the solenonid to
a = 50 mm and b = 100 mm respectively. The magnetic induction at the centre demagnetize the ferromagnet completely is
of the spiral is (a) 1.00 m A
(a) 5 μT (b) 7 μT (b) 1.25 mA
(c) 8 μT (d) 10 μT (c) 1.50 mA
6. Three wires of equal lengths are bent in the form of loops. One of the loops is (d) 1.75 mA
circle, another is a semi-circle and the third one is a square. They are placed in
a uniform magnetic field and same electric current is passed through them.
Which of the following loop configuration will experience greater torque ?
(a) Circle (b) Semi-circle
(c) Square (d) All of them

victory R. SARAVANAN. M.Sc., M.Phil., B.Ed PG ASST [PHYSICS], GBHSS, PARANGIPETTAI - 608 502
12 PHYSICS UNIT - 3 MAGNETISM AND MAGNETIC EFFECTS OF CURRENT CONCEPTUAL QUESTIONS & ANSWERS

12. Two short bar magnets have magnetic moments 1.20 Am2 and 1.00 Am2 PART - III 6 X 3 = 18
respectively. They are kept on a horizontal table parallel to each other with
their north poles pointing towards south. They have a common magnetic
Note : (i) Answer any 6 of the following questions .
equator and are separated by a distance of 20.0 cm. The value of the resultant (ii) Question No. 30 is compulsory
horizontal magnetic induction at the mid-point O of the line joining their 24. What are the properties of bar magnet?
centres is (Horizontal components of Earth’s magnetic induction is 25. Give the properties of magnetic field lines.
3.6 × 10–5 Wb m–2 ) 26. State and explain Biot-Savart law.
(a) 3.60 × 10–5 Wb m–2 (b) 3.5 × 10–5 Wb m–2 27. Give the properties of Lorentz magnetic force
(c) 2.56 × 10–4 Wb m–2 (d) 2.2 × 10–4 Wb m–2 28. How Galvanometer can be converted in to Ammeter
13. The vertical component of Earth’s magnetic field at a place is equal to the 29. Explain the current loop acts as a magnetic dipole and calculate its dipole moment.
horizontal component. What is the value of angle of dip at this place? 30. Calculate the magnetic field at the centre of a square loop which carries a current of
(a) 30o (b) 45o 1.5 A, length of each side being 50 cm.
(c) 60 o (d) 90o PART - IV 3 X 5 = 15
14. A flat dielectric disc of radius R carries an excess charge on its surface. The
surface charge density is σ. The disc rotates about an axis perpendicular to its Note : (i) Answer all the questions
plane passing through the centre with angular velocity ω. Find the magnitude 31. Deduce the relation for magnetic induction at a point due to an infinitely long
of the torque on the disc if it is placed in a uniform magnetic field whose straight conductor carrying current.
strength is B which is directed perpendicular to the axis of rotation (OR)
1 1 Obtain the expression for force on a moving charge in a magnetic field.
(a) 𝜎𝜔𝜋𝐵𝑅 (b) 4 𝜎 𝜔 𝜋 𝐵 𝑅 2
4 32. Using Ampere’s circuital law, Obtain an expression for magnetic field due to long
1 1
(c)
4
𝜎 𝜔 𝜋 𝐵 𝑅3 (d)
4
𝜎 𝜔 𝜋 𝐵 𝑅4 current carrying solenoid.
15. The potential energy of magnetic dipole whose dipole moment is (OR)
⃗⃗ = 𝟎. 𝟐 𝒊̂ 𝑻
⃗⃗⃗𝒑𝒎 = (−𝟎. 𝟓 𝒊̂ + 𝟎. 𝟒 𝒋̂) 𝑨𝒎𝟐 kept in uniform magnetic field 𝑩
(a) –0.1 J (b) –0.8 J Obtain an expression for the force on a current carrying conductor placed in a
(c) 0.1 J (d) 0.8 J magnetic field.
33. Obtain a force between two long parallel current carrying conductors. Hence define
PART - II 6 X 2 = 12 ampere.
Note : (i) Answer any 6 of the following questions . (OR)
(ii) Question No. 23 is compulsory Describe the principle, construction and working of moving coil galvanometer.
16. Calculate the tangent of magnetic inclination or angle of dip.
17. Define magnetic dipole moment. Give its unit.
18. Define magnetic flux. Give its unit.
19. State Ampere’s circuital law.
20. State Fleming’s left hand rule (FLHR). தமிழுக்கும் அமுததன்று பபர் ! -அந்தத்தமிழ் இன்பத்தமிழ் எங்கள் உயிருக்கு பேர் !
21. How the current sensitivity of galvanometer can be increased? தமிழுக்கு நிலதென்று பபர் ! - இன்பத் தமிழ் எங்கள் சமுகத்தின் விளைவுக்கு நீர் !
22. Define voltage sensitivity of the galvanometer. தமிழுக்கு மணதமன்று பபர் ! - இன்பத் தமிழ் எங்கள் ொழ்வுக்கு நிருமித்த ஊர் !
23. The repulsive force between two magnetic poles in air is 9 × 10 –3 N. If the two poles தமிழுக்கு மதுதென்று பபர் ! - இன்பத் தமிழ் எங்கள் உரிளமச்தசம் பயிருக்கு பெர் !
are equal in strength and are separated by a distance of 10 cm, calculate the pole
தமிழ் எங்கள் இைளமக்குப் பால் ! - இன்பத்தமிழ் ேல்ல புகழ்மிக்க புலெர்க்குபெல்
strength of each pole
தமிழ் எங்கள் உயர்வுக்கு ொன் ! - இன்பத் தமிழ் எங்கள் அசதிக்குச் சுடர்தந்த பதன் !
தமிழ் எங்கள் அறிவுக்குத் பதாள் ! - இன்பத் தமிழ் எங்கள் கவிளதக்கு ெயிரத்தின் ொள் !
தமிழ் எங்கள் பிறவிக்குத் தாய் ! - இன்பத் தமிழ் எங்கள் ெைமிக்க உைமுற்ற தீ !
இன்பத்தமிழ் - பாபெந்தர் பாரதிதாசன்

victory R. SARAVANAN. M.Sc., M.Phil., B.Ed PG ASST [PHYSICS], GBHSS, PARANGIPETTAI - 608 502
பசித்திரு (Be hungry) தனித்திரு (Be individual) விழித்திரு (Be conscious)

HIGHER SECONDARY SECOND YEAR-PHYSICS

NAME :
STANDARD : 12 SECTION :
SCHOOL :
EXAM NO :

victory R. SARAVANAN. M.Sc, M.Phil, B.Ed.,


PG ASST (PHYSICS)
GBHSS, PARANGIPETTAI - 608 502
12 PHYSICS UNIT - 4 ELECTOMAGNETIC INDUCTION AND ALTERNATING CURRENT COMPLETE GUIDE AND MODEL QUESTION

PART – I MULTIPLE CHOICE 1 MARK QUESTIONS & ANSWERS WITH SOLUTIONS


4. When the current changes from +2A to −2A in 0.05 s, an emf of 8 V is induced
1. An electron moves on a straight line path XY as shown in a coil. The co-efficient of self-induction of the coil is
in the figure. The coil abcd is adjacent to the path of (a) 0.2 H (b) 0.4 H
the electron. What will be the direction of current, if (c) 0.8 H (d) 0.1 H
any, induced in the coil? -Solution :-
(a) The current will reverse its direction as the 𝑑𝐼
 The self induced emf is given by, 𝜖 = −𝐿
𝑑𝑡
electron goes past the coil
 Hence coefficient of self induction ;
(b) No current will be induced (c) abcd (d) adcb 𝜖 𝜖 𝑑𝑡 𝜖𝑡 8 𝑋 0.05 0.4
-Solution :- 𝐿=− 𝑑𝐼 =− 𝑑𝐼
= − = − (−2−2)
= − (−4) = 0.1 𝐻
( ) (𝐼𝑓 −𝐼𝑖 )
 Conventional current flows opposite to flow of electron (i.e) from Y to X 𝑑𝑡

 When current approaches the coil, the magnetic flux linked with it increases and Answer (d) 0.1 H
hence current induced in it which flows in the direction ‘abcd’ according to 5. The current i flowing in a coil varies with time as
Lenz law. When current receeding away from the coil, the magnetic flux linked shown in the figure. The variation of induced emf
with it decreases and hence current induced in it which flows along ‘adcb’ with time would be
 Thus the current will reverse its direction
The current will reverse its direction
Answer (a)
as the electron goes past the coil
2. A thin semi-circular conducting ring (PQR) of radius r
is falling with its plane vertical in a horizontal magnetic
field B, as shown in the figure. The potential difference
developed across the ring when its speed v is
𝐵 𝑣 𝜋 𝑟2 (a) (b) (c) (d)
(a) Zero (b) and P is at higher potential
2 -Solution :-
(c) 𝜋 r B v and R is at higher potential (d) 2 r B v and R is at higher potential 𝑑𝐼
-Solution :-  The self induced emf is given by, 𝜖 = −𝐿
𝑑𝑡
 The motional emf induced is, 𝑒 = 𝐵 𝑙𝑃𝑅 𝑣 = 𝐵 [2 𝑟] 𝑣 = 2 𝑟 𝐵 𝑣  When ‘I’ increases linearly with time, the rate
 By Lenz’s law, the induced current opposes the flux change by producing 𝑑𝐼
of change of current becomes positive
𝑑𝑡
another magnetic field pointing outward (i.e.) opposite to the existing magnetic
field. constant. So induced emf also constant
 From the direction of magnetic field thus produced, the direction of the induced but with negative sign.
current is found to be along RQP by using right-hand thumb rule. Thus R is at  When ‘I’ decreases linearly with time, the
𝑑𝐼
higher potential. rate of change of current becomes 𝑑𝑡
Answer (d) 2 r B v and R is at higher potential negative constant. So induced emf also
3. The flux linked with a coil at any instant t is given by 𝚽𝑩 = 𝟏𝟎 𝒕𝟐 − 𝟓𝟎 𝒕 + 𝟐𝟓𝟎 . constant but with positive sign.
The induced emf at t = 3 s is  When ‘I’ is constant, the rate of change of
(a) −190 V (b) −10 V (c) 10 V (d) 190 V current
𝑑𝐼
becomes zero and hence the
-Solution :- 𝑑𝑡
 By Laws of electromagnetic induction ; induced emf also zero
𝑑Φ 𝑑
𝜖 = − 𝐵 = − (10 𝑡 2 − 50 𝑡 + 250) = −(20 𝑡 − 50) = −20 𝑡 + 50
𝑑𝑡 𝑑𝑡
 When 𝑡 = 3 𝑠, the induced emf ; 𝜖 = −20(3) + 50 = −10 𝑉 Answer (a)
Answer (b) −𝟏𝟎 𝑽

victory R. SARAVANAN. M.Sc., M.Phil., B.Ed PG ASST [PHYSICS], GBHSS, PARANGIPETTAI - 608 502
12 PHYSICS UNIT - 4 ELECTOMAGNETIC INDUCTION AND ALTERNATING CURRENT COMPLETE GUIDE AND MODEL QUESTION
6. A circular coil with a cross-sectional area of 4 cm2
has 10 turns. It is placed at 10. In a series RL circuit, the resistance and inductive reactance are the same.
the centre of a long solenoid that has 15 turns/cm and a cross-sectional area Then the phase difference between the voltage and current in the circuit is
𝜋 𝜋
of 10 cm2. The axis of the coil coincides with the axis of the solenoid. What is (a) (b)
their mutual inductance? 4 2
𝜋
(a) 7.54 μH (b) 8.54 μH (c) 9.54 μH (d) 10.54 μH (c) (d) zero
6
-Solution :- -Solution :-
 The mutual inductance is given by ; 𝑋
 In RL- circuit, the phase difference will be; 𝜙 = tan−1 [ 𝐿 ]
𝜇𝑜 𝑁1 𝑁2 𝐴2 4𝜋 𝑋 10−7 𝑋 10 𝑋 1500 𝑋 4 𝑋 10−4 𝑅
𝑀= 𝐼1
= 1
= 7.536𝑋10−6 𝐻 = 7.54 𝜇𝐻  Since 𝑋𝐿 = 𝑅, phase difference becomes, 𝜙 = tan −1 [1] 𝜋
= 45° = 4
Answer (a) 7.54 μH 𝝅
Answer (a)
7. In a transformer, the number of turns in the primary and the secondary are 𝟒
410 and 1230 respectively. If the current in primary is 6A, then that in the 11. In a series resonant RLC circuit, the voltage across 100 Ω resistor is 40 V. The
secondary coil is resonant frequency ω is 250 rad/s. If the value of C is 4 μF, then the voltage
(a) 2 A (b) 18 A across L is
(c) 12 A (d) 1 A (a) 600 V (b) 4000 V
-Solution :- (c) 400V (d) 1 V
𝐸𝑆 𝑁 𝐼𝑃 -Solution :-
 From transformer equation ;
𝐸𝑃
= 𝑁𝑆 = 𝐼𝑆
=𝑘 1 1 1
𝑁𝑆 𝐼𝑃 𝑁
𝑃
410 2460  Capacitive reactance ; 𝑋𝐶 =
𝜔𝐶
= 250 𝑋 4 𝑋 10−6 = 10−3
= 103 Ω
 Thus,
𝑁𝑃
= 𝐼𝑆
(or) 𝐼𝑆 = 𝐼𝑃 𝑁𝑃 = 6 𝑋 1230
= 1230 =2𝐴 𝑉 40
𝑆  And the current ; 𝐼 = = = 0.4 𝐴
𝑅 100
Answer (a) 2A 3
 At resonance, 𝑋𝐿 = 𝑋𝐶 = 10 Ω
8. A step-down transformer reduces the supply voltage from 220 V to 11 V and 3
increase the current from 6 A to 100 A. Then its efficiency is  The voltage across L is ; 𝑉𝐿 = 𝐼 𝑋𝐿 = 0.4 𝑋 10 = 400 𝑉
(a) 1.2 (b) 0.83 Answer (c) 400 V
(c) 0.12 (d) 0.9 12. An inductor 20 mH, a capacitor 50 μF and a resistor 40 Ω are connected in
-Solution :- series across a source of emf V = 10 sin 340 t. The power loss in AC circuit is
𝑜𝑢𝑡𝑝𝑢𝑡 𝑝𝑜𝑤𝑒𝑟 𝐸𝑆 𝐼𝑆 11 𝑋 100 5
 Efficiency ; 𝜂 = = = = = 0.83 (a) 0.76 W (b) 0.89 W
𝑖𝑛𝑝𝑢𝑡 𝑝𝑜𝑤𝑒𝑟 𝐸𝑃 𝐼𝑃 220 𝑋 6 6
(c) 0.46 W (d) 0.67 W
Answer (b) 𝟎. 𝟖𝟑 Solution :-
9. In an electrical circuit, R, L, C and AC voltage source are all connected in series.  Inductive reactance ; 𝑋𝐿 = 𝜔𝐿 = 340 𝑋 20 𝑋 10−3 = 6.8 Ω
1 1
When L is removed from the circuit, the phase difference between the voltage  Capacitive reactance ; 𝑋𝐶 = = −6 = 58.8 Ω
𝝅 𝜔𝐶 340 𝑋 50 𝑋 10
and current in the circuit is . Instead, if C is removed from the circuit, the  Resultant reactance ; 𝑋𝐶 − 𝑋𝐿 = 58.8 − 6.8 = 52 Ω
𝟑
𝝅
phase difference is again . The power factor of the circuit is  Impeadance ; 𝑍 = √𝑅2 + (𝑋𝐿 − 𝑋𝐶 )2 = √402 + 522 = √402 + 522 = 65.6 Ω
𝟑
𝑉𝑅𝑀𝑆 𝑉𝑚 /√2 10𝑋 0.707
1 1 √3  RMS current ; 𝐼𝑅𝑀𝑆 = = = = 0.108 𝐴
(a) (b) (c) 1 (d) 𝑍 𝑍 65.6
2 √2 2 2
 Then power loss ; 𝑃𝑎𝑣𝑔 = 𝐼𝑅𝑀𝑆 𝑅 = 0.108 𝑋 0.108 𝑋 40 = 0.46 𝑊
-Solution :-
 L is removed, we get RC- circuit and the phase difference ; 𝜙 = tan−1 [
𝑋𝐶
] =
𝜋 Answer (c) 0.46 W
𝑅 3
𝑋 𝜋
 C is removed, we get RL- circuit and the phase difference ; 𝜙 = tan−1 [ 𝑅𝐿 ] = 3
 Hence, 𝑋𝐿 = 𝑋𝐶 and the impedance becomes, 𝑍 = √𝑅2 + (𝑋𝐿 − 𝑋𝐶 )2 = 𝑅
𝑅 𝑅
 Thus power factor ; cos 𝜙 = 𝑍 = 𝑅
=1
Answer (c) 1
victory R. SARAVANAN. M.Sc., M.Phil., B.Ed PG ASST [PHYSICS], GBHSS, PARANGIPETTAI - 608 502
12 PHYSICS UNIT - 4 ELECTOMAGNETIC INDUCTION AND ALTERNATING CURRENT COMPLETE GUIDE AND MODEL QUESTION
13. The instantaneous values of alternating current and voltage in a circuit are
𝟏 𝟏 𝝅
𝒊= 𝐬𝐢𝐧(𝟏𝟎𝟎 𝝅 𝒕) 𝑨 and 𝑽 = 𝐬𝐢𝐧 (𝟏𝟎𝟎 𝝅 𝒕 + ) 𝑽.The average power
√𝟐 √𝟐 𝟑
in watts consumed in the circuit is
1 √3
(a) (b)
4 4
1 1
(c) (d)
2 8
-Solution :-
 The average power,
𝑉𝑚 𝐼𝑚 1
𝑃𝑎𝑣𝑔 = 𝐸𝑅𝑀𝑆 𝐼𝑅𝑀𝑆 𝑐𝑜𝑠𝜙 = 𝑐𝑜𝑠𝜙 = 2 𝑉𝑚 𝐼𝑚 𝑐𝑜𝑠𝜙
√2 √2
1 1 𝟏 𝟏 𝝅 1
𝑃𝑎𝑣𝑔 = 2 𝑉𝑚 𝐼𝑚 𝑐𝑜𝑠𝜙 = 2 ( ) ( ) 𝑐𝑜𝑠 [ 𝟑 ] = 8
√𝟐 √𝟐
𝟏
Answer (d) 𝟖
14. In an oscillating LC circuit, the maximum charge on the capacitor is Q. The
charge on the capacitor when the energy is stored equally between the electric
and magnetic fields is
𝑄 𝑄
(a) (b)
2 √3
𝑄
(c) (d) Q
√2
-Solution :-
𝑄2
 Maximum energy stored in capacitor ; 𝑈𝑚𝑎𝑥 =
2𝐶
 Let ‘q’ be the charge, when energy is stored equally between the electric and
magnetic fields, then
𝑈𝑚𝑎𝑥 𝑞2 𝑄2 𝑞2 𝑄2 𝑄
= (or) = (or) = 𝑞 2 (or) 𝑞=
2 2𝐶 4𝐶 2𝐶 2 √2
𝑸
Answer (c) √𝟐
𝟐𝟎
15. 𝑯 inductor is connected to a capacitor of capacitance C. The value of C in
𝝅𝟐
order to impart maximum power at 50 Hz is
(a) 50 μF (b) 0.5 μF
(c) 500 μF (d) 5 μF
-Solution :-
 For maximum power, current must be maximum. Hence
1 1 1
𝑋𝐿 = 𝑋𝐶 (or) 𝜔𝐿= 𝜔𝐶
(or) 𝐶 =
𝜔2 𝐿
(or) 𝐶 = 4 𝜋2 𝑓 2 𝐿
1 1 1
𝐶= 𝟐𝟎 = 4 𝑋 2500 𝑋 20 = 200000 = 0.5 𝑋 10−5 = 5 𝑋 10−6 𝐹 = 5 𝜇𝐹
4 𝜋2 502 ( 𝟐 )
𝝅

Answer (d) 𝟓 𝝁𝑭

victory R. SARAVANAN. M.Sc., M.Phil., B.Ed PG ASST [PHYSICS], GBHSS, PARANGIPETTAI - 608 502
12 PHYSICS UNIT - 4 ELECTOMAGNETIC INDUCTION AND ALTERNATING CURRENT COMPLETE GUIDE AND MODEL QUESTION
10. What is called self induction?
PART – II 2 MARK SHORT ANSWER QUESTIONS & ANSWERS  The phenomenon of inducing an emf in a coil, when the magnetic flux linked
1. Define magnetic flux. with the coil itself changes is called self induction.
 The magnetic flux through an area ‘A’ in a magnetic field is defined as the  The emf induced is called self-induced emf.
number of magnetic field lines passing through that area normally. 11. Define self inductance or coeffient of self induction.
 The S.I unit of magnetic flux is 𝑻 𝒎𝟐 (or) weber  Self inductance of a coil is defined as the flux linkage of the coil, when 1 A
2. Define electromagnetic induction. current flows through it.
 Whenever the magnetic flux linked with a closed coil changes, an emf is induced  Its S.I unit is 𝑯 (𝑜𝑟) 𝑾𝒃 𝑨−𝟏 (𝑜𝑟) 𝑽 𝒔 𝑨−𝟏 and its dimension is [𝑴 𝑳𝟐 𝑻−𝟐 𝑨−𝟐 ]
and hence an electric current flows in the circuit. 12. Define the unit of self inductance (one henry)
 This emf is called induced emf and the current is called induced current. This  The inductance of the coil is one henry, if a current changing at the rate of 1 A s -1
phenomenon is called electromagnetic induction. induces an opposing emf of 1 V in it.
3. What is the importance of electromagnetic induction? 13. What is called mutual induction?
 There is an ever growing demand for electric power for the operation of almost  When an electric current passing through a coil changes with time, an emf is
all the devices used in present day life. induced in the neighbouring coil. This phenomenon is known as mutual
 All these are met with the help of electric generators and transformer which induction and the emf is called mutually induced emf.
function on electromagnetic induction. 14. Define mutual inductance or coefficient of mutual induction.
4. State Faraday’s laws of electromagnetic induction.  Mutual inductance is also defined as the opposing emf induced in the one coil,
(i) Whenever magnetic flux linked with a closed circuit changes, an emf is induced when the rate of change of current through the other coil is 1 A s-1
in the circuit.  Its S.I unit is 𝑯 (𝑜𝑟) 𝑾𝒃 𝑨−𝟏 (𝑜𝑟) 𝑽 𝒔 𝑨−𝟏 and its dimension is [𝑴 𝑳𝟐 𝑻−𝟐 𝑨−𝟐 ]
(ii) The magnitude of induced emf in a closed circuit is equal to the time rate of 15. What the methods of producing induced emf?
change of magnetic flux linked with the circuit.  By changing the magnetic field ‘B’
5. State Lenz’s law.  By changing the area ‘A’ of the coil
 Lenz’s law states that the direction of the induced current is such that is always  By changing the relative orientation ‘’ of the coil with magnetic field.
opposes the cause responsible for its production. 16. How an emf is induced by changing the magnetic field?
6. State Flemming’s right hand rule (generator rule).  Change in magnetic flux of the field is brought about by,
 The thumb, index finger and middle finger of right hand are stretched out in (i) The relative motion between the circuit and the magnet
mutually perpendicular directions. If index finger points the direction of (ii) Variation in current flowing through the nearby coil
magnetic field and the thumb points the direction of motion of the conductor, 17. What is called AC generator or alternator?
then the middle finger will indicate the direction of the induced current.  AC generator is a device which converts mechanical energy used to rotate the
7. What are called eddy currents? How are they produced? coil or field magnet in to electrical energy.
 When magnetic flux linked with a conductor in the form of a sheet or a plate 18. State the principle of AC generator (alternator)
changes, an emf is induced. As a result, the induced current flow in concentric  It work on the principle of electromagnetic induction. (i.e.) The relative
circular paths which resembles eddies of water. Hence these are known as Eddy motion between a conductor and a magnetic field changes the magnetic flux
currents or Foucault currents. linked with the conductor which in turn induces an emf.
8. A spherical strone and a spherical metallic ball of same size and mass are  The magnitude of the induced emf is given by Faraday’s law and its direction by
dropped from the same height. Which one will reach earth’s surface first? Flemming’s right hand rule.
Justify your answer. 19. State single phase AC generator.
 The stone will reach the earth’s surface earlier than the metal ball.  In a single phase AC generator, the armature conductors are connected in series
 Because when the metal ball falls through the magnetic field of earth, the eddy so as to form a single circuit which generates a single - phase alternating emf
currents are produced in it which opposed its motion. and hence it is called single-phase alternator.
 But in the case of stone, no eddy currents are produced and it falls freely. 20. State three phase AC generators.
9. What is called inductor?  If there are three separate coils, which would give three separate emf’s then
 Inductor is a device used to store energy in a mangnetic field when an electric they are called three phase AC generators.
current flows through it.
(e.g.) solenoids and toroids
victory R. SARAVANAN. M.Sc., M.Phil., B.Ed PG ASST [PHYSICS], GBHSS, PARANGIPETTAI - 608 502
12 PHYSICS UNIT - 4 ELECTOMAGNETIC INDUCTION AND ALTERNATING CURRENT COMPLETE GUIDE AND MODEL QUESTION
21. What are the advantages of three phase AC generators? 30. Define effective value of alternating current.
 For a given dimension of the generator, three -phase machine produces higher  RMS value of AC is also called effective value of AC
power output than a single -phase machine.  The effective value of AC (𝐼𝑒𝑓𝑓 ) is defined as the value of steady current which
 For the same capacity, three phase alternator is smaller in size when compared when flowing through a given circuit for a given time produces the same
to single phase genarators. amount of heat as produced by the alternating current when flowing through
 Three phase transmission system is cheaper. A relatively thinner wire is the same circuit for the same time.
sufficient for transmission of three phase power. 31. The common house hold appliences, the voltage rating is specified as 230 V,
22. What is called poly phase generator? 50 Hz. What is the meaning of it?
 Some AC generators may have more than one coil in the armature core and each  The voltage rating specified in the common house hold appliences indicates the
coil produces an alternating emf. In these generators, more than one emf is RMS value or effective value of AC. (i.e.) 𝑽𝒆𝒇𝒇 = 𝟐𝟑𝟎 𝑽
produced. Thus they are called poly-phase generators.  Its peak value will be,
23. What is called transformer? 𝑽𝒎 = 𝑽𝒆𝒇𝒇 √𝟐 = 𝟐𝟑𝟎 𝑿 𝟏. 𝟒𝟏𝟒 = 𝟑𝟐𝟓 𝑽
 It is a stationary device used to transform electrical power from one circuit to
 Also 50 Hz indicates, the frequency of domestic AC supply.
another without changing its frequency.
32. Define phasor and phasor diagram.
 The applied alternating voltage is either increased or decreased with
 A sinusoidal alternating voltage or current can be represented by a vector
corresponding decrease or increase in current in the circuit.
which rotates about the orgin in anti-clockwise direction at a constant angular
24. Distinguish between step up and step down transformer.
velocity ‘𝜔’. Such a rotating vector is called a phasor.
Step up transformer Step down transformer
 The diagram which shows various phasors and phase relations is called phasor
If the transformer converts an If the transformer converts an
diagram.
alternating current with low voltage in alternating current with high voltage
33. Draw the phasor diagram for an alternating voltage 𝒗 = 𝑽𝒎 𝒔𝒊𝒏 𝝎𝒕
to an alternating current with high in to an alternating current with low
voltage is called step up transformer. voltage is called step down
transformer.
25. State the principle of transformer.
 The principle of transformer is the mutual induction between two coils. (i.e.)
when an electric current passing through a coil changes with time, and emf is
induced in the other coil.
26. Define the efficiency of the transformer.
 The efficiency (𝜂) of a transformer is defined as the ratio of the useful output 34. Define inductive reactance.
power to the input power.  The resistance offered by the inductor in an ac circuit is called inductive
𝑜𝑢𝑡𝑝𝑢𝑡 𝑝𝑜𝑤𝑒𝑟
𝜂= 𝑋 100 % reactance and it is given by ; 𝑿𝑳 = 𝝎 𝑳 = 𝟐 𝝅 𝒇 𝑳
𝑖𝑛𝑝𝑢𝑡 𝑝𝑜𝑤𝑒𝑟  Its unit is ohm (𝜴)
27. Define Sinusoidal alternating voltage. 35. An inductor blocks AC but it allows DC. Why?
 If the waveform of alternating voltage is a sine wave, then it is known as  The DC current flows through an inductor produces uniform mangetic field and
sinusoidal alternating voltage and it is given by ; 𝒗 = 𝑽𝒎 𝐬𝐢𝐧 𝝎𝒕 the magnetic flux linked remains constant. Hence there is no self induction and
28. Define mean value or average value of AC. self induced emf (opposing emf). So DC flows through an inductor.
 The mean or average value of alternating current is defined as the average of all  But AC flows through an inductor produces time varying magnetic field which
values of current over a positive half cycle or negative half cycle. inturn induces self induced emf and this opposes any change in the current.
𝟐 𝑰𝒎
𝑰𝒂𝒗𝒈 = = 𝟎. 𝟔𝟑𝟕𝟏 𝑰𝒎 Since AC varies both in magnitude and direction, it flow is opposed by the back
𝝅 emf induced in the inductor and hence inductor blocks AC
29. Define RMS value of AC. 36. Define capacitive reactance.
 The root mean square value of an alternating current is defined as the square  The resistance offered by the capacitor is an ac circuit is called capacitive
root of the mean of the square of all currents over one cycle. 𝟏 𝟏
𝑰𝒎 reactance and it is given by ; 𝑿𝑪 = =
𝝎𝑪 𝟐𝝅𝒇𝑪
𝑰𝑹𝑴𝑺 = = 𝟎. 𝟕𝟎𝟕 𝑰𝒎
√𝟐  Its unit is ohm (𝜴)
victory R. SARAVANAN. M.Sc., M.Phil., B.Ed PG ASST [PHYSICS], GBHSS, PARANGIPETTAI - 608 502
12 PHYSICS UNIT - 4 ELECTOMAGNETIC INDUCTION AND ALTERNATING CURRENT COMPLETE GUIDE AND MODEL QUESTION
37. A capacitor blocks DC but it allows AC. Why? 45. What are called LC oscillations?
 When DC flows through capacitor, electrons flows from negative terminal and  Whenever energy is given to a circuit containing a pure inductor of inductance L
accumulated at one plate making it negative and hence another plate becomes and a capacitor of capacitance C, the energy oscillates back and forth between
positive. This process is known as charging and once capacitor is fully charged, the magnetic field of the inductor and the electric field of the capacitor.
the current will stop and we say capacitor blocks DC.  Thus the electrical oscillations of definite frequency are generated. These
 But AC flows through capacitor, the electron flow in one direction while oscillations are called LC oscillations.
charging the capacitor and its direction is reversed while discharging. Though 46. Define Flux linkage.
electrons flow in the circuit, no electrons crosses the gap between the plates. In  The product of magnetic flux (Φ𝐵 ) linked with each turn of the coil and the total
this way, AC flows through a capacitor. number of turns (N) in the coil is called flux linkage (NΦ𝐵 )
38. Define resonance. 47. Define impedeance of RLC circuit.
 When the frequency of the applied sourch is equal to the natural frequency of  The effective opposion by resistor, inductor and capacitor to the circuit current
the RLC circuit, the current in the circuit reaches it maximum value. Then the in the series RLC circuit is called impedance (Z)
circuit is said to be in electrical resonance. 𝒁 = √ 𝑹𝟐 + (𝑿𝑳 − 𝑿𝑪 ) 𝟐
 The frequency at which resonance takes place is called resonant frequency.
 Hence the condition for resonance is : 𝑿𝑳 = 𝑿𝑪
39. What are the applications of series RLC resonant circuit?
 RLC circuits have many applications like filter circuits, oscillators, voltage
multipliers etc.,
 An important use of series RLC resonant circuits is in the tuning circuits of radio
and TV systems. To receive the signal of a particular station among various
broadcasting stations at different frequencies, tuning is done.
40. Resonance will occur only in LC circuits. Why?
 When the circuits contains both L and C, then voltage across L and C cancel one
another when 𝑉𝐿 and 𝑉𝐶 are 180 out of phase and the circuit becomes purely
resistive.
 This implies that resonance will not occur in a RL and RC circuits.
41. Define Q - factor or quality factor.
 Q - factor is defined as the ratio of voltage across L or C to the applied voltage
at resonance.
42. Define power in an AC circuits.
 Power of a circuits is defined as the rate of consumption of electric energy in
that circuit.
 It is the product of the voltage and current.
43. Define power factor.
 Power factor (cos 𝜙) of a circuit is defined as the cosine of the angle of lead or
lag
 Power factor is also defined as the ratio of true power to the apparent power.
44. Define wattles current.
 If the power consumed by an AC circuit is zero, then the current in that circuit is
said to be wattless current.
 This wattles current happens in a purely inductive or capacitive circuit.

victory R. SARAVANAN. M.Sc., M.Phil., B.Ed PG ASST [PHYSICS], GBHSS, PARANGIPETTAI - 608 502
12 PHYSICS UNIT - 4 ELECTOMAGNETIC INDUCTION AND ALTERNATING CURRENT COMPLETE GUIDE AND MODEL QUESTION
 Consider a closed circuit called primary consisting of coil ‘P’, a battery ‘B’ and a
PART – III 3 MARK SHORT ANSWER QUESTIONS & ANSWERS key ‘K’
1. Establish the fact that the relative motin between the coil and the magnet  Consider an another closed circuit called secondary consisting of coil ‘S and a
induces an emf in the coil of a closed circuit. galvanometer ‘G’
Faraday’s experiment - 1 :  Here the two coils ‘P’ and ‘S’ are kept at rest in close proximity with respect to
one another.
 When the primary circuit is closed, current starts flowing in this circuit. At this
time, the galvanometer gives a momentary deflection. After that, when current
reaches a steady value, no deflection is observed in the galvanometer.
 Similarly, if the primary circuit is broken, current starts decreasing and there is
again a momentary deflection but in the opposite direction. When current
becomes zero, the galvanometer shows no deflection.
 From the above observations, it is concluded that whenever the electric current
 Consider a closed circuit consisting of a coil ‘C’ and a galvanometer ‘G’. Initially
in the primary changes, the galvanometer in secondary shows a deflection.
the galvanometer shows no deflection.
3. How we understood the conclusions obtained from Faraday’s experiment.
 When a bar magnet move towards the stationary coil with its north pole (N) Faraday’s experiment - Explanation :
facing the coil, there is a momentary deflection in the galvanometer. This Experiment - 1 :
indicates that an electric current is set up in the coil
 In the first experiment, when a bar magnet is placed close to a coil, then there is
 If the magnet is kept stationary inside the coil, the galvanometer does not some magnetic flux linked with the coil.
indicate deflection.
 When the barmagneti and coil approach each other, the magnetic flux linked
 The bar magnet is now withdrawn from the coil, the galvanometer again gives a with the coil increases and this increase in magnetic flux induces an emf and
momentary deflection but is opposite direction. This indicates current flows in hence a transient current flows in one direction.
opposite direction.
 At the same time, when they recede away from one another, the magnetic flux
 Now if the magnet is moved faster, it gives a larger deflection due to a greater linked with the coil decreases. The decrease in magnetic flux again induces an
current in the circuit. emf in opposite direction and hence an electric current flows in opposite
 The bar magnet is reversed (i.e.) the south pole now faces the coil and the direction.
experiment is repeated, same results are obtained but the directions of  So there is deflection in the galvanometer, when there is a relative motion
deflection get reversed. between the coil and the magnet.
 Simillarly if the magnet is kept stationary and the coil moved towards or away Experiment - 2 :
from the coil, similar results are obtained.  In the second experiment, when the primary coil ‘P’ carries an electric current, a
 Thus the above experiments concluded that, whenever there is a relative motion magnetic field is established around it. The magnetic lines of this field pass
between the coil and the magnet, ther is a deflection in the galvanometer, through itself and the neighbouring secondary coil ‘S’
indicating the electric current set up in the coil.  When the primary circuit is open, no current flows in it and hence the magnetic
2. Prove that experimentaly if the current in a one closed circuit changes, an emf flux linked with secondary coil is zero
is induced in another circuit.
 When the primary circuit is closed, the increasing current increases the
Faraday’s experiment - 2 :
magnetic flux linked with primary as well as secondary coil. This increasing flux
induces a current in the secondary coil.
 When the current in the primary coil reaches a steady value, the magnetic flux
linked with the secondary coil does not change and the current in it will
disappear.
 Similarly, when the primary circuit is broken, the decreasing current induces an
electric current in the secondary coil, but in opposite direction.
 So there is a deflection in the galvanometer, whenever there is a change in the
primary current.

victory R. SARAVANAN. M.Sc., M.Phil., B.Ed PG ASST [PHYSICS], GBHSS, PARANGIPETTAI - 608 502
12 PHYSICS UNIT - 4 ELECTOMAGNETIC INDUCTION AND ALTERNATING CURRENT COMPLETE GUIDE AND MODEL QUESTION
4. State and explain Faraday’s laws of electromagnetic induction. 6. Show that Lenz’s law is in accordance with the law of conservation of energy.
Faraday’s first law : Conservation of energy - Lenz’s law :
 Whenever magnetic flux linked with a closed circuit changes, an emf is induced  According to Lenz’s law, when a magnet is moved either towards or away from
in the circuit. a coil, the induced current produced opposes its motion.
 The induced emf lasts so long as the change in magnetic flux continues.  As a result, there will always be a resisting force on the moving magnet. So work
Faraday’s second law : has to be done by some external agency to move the magnet against this
 The magnitude of induced emf in a closed circuit is equal to the time rate of resistive force.
change of magnetic flux linked with the circuit.  Here the mechanical energy of the moving magnet is converted into the
 If magnetic flux linked with the coil changes by 𝑑Φ𝐵 in time 𝑑𝑡 , then the electrical energy which inturn gets converted in to Joule heat in the coil. (i.e)
induced emf is given by, energy is conserved from one form to another
𝑑Φ𝐵  On the contrary to Lenz’s law, let us assume that the induced current helps the
𝜖= −
𝑑𝑡 cause responsible for its production.
 The negative sign in the above equation gives the direction of the induced  If we push the magnet little bit towards the coil, the induced current helps the
current movement of the magnet towards the coil.
 If a coil consisting of ‘N’ turns, then  Then the magnet starts moving towards the coil without any expense of energy,
𝒅𝚽𝑩 𝒅 ( 𝐍 𝚽𝑩 ) which is impossible in practice.
𝝐= −𝑵 = −
𝒅𝒕 𝒅𝒕  Therefore the assumption that the induced current helps the cause is wrong.
 Here N Φ𝐵 is called flux linkage. 7. Obtain an expression for motional emf from Lorentz force.
5. Give an illustration of determining direction of induced current by using Motional emf from Lorentz force:
Lenz’s law.  Consider a straight conductor
Explanation of Lenz’s law : rod AB of length ‘𝑙’ in a uniform
magnetic field 𝐵 ⃗ which is
directed perpendicularly in to
plane of the paper.
 Let the rod move with a constant
velocity 𝑣⃗⃗⃗ towards right side.
 When the rod moves, the free
electrons present in it also move with same velocity 𝑣 ⃗
⃗⃗⃗ in 𝐵
 As a result, the Lorentz forec acts on free electron in the direction from B to A
 Let a bar magnet move towards the solenoid with its north pole pointing the and it is given by,
solenoid. ⃗⃗⃗𝐹𝐵 = −𝑒 (⃗⃗⃗𝑣 𝑋 ⃗⃗⃗𝐵 ) − − − − (1)
 This motion increases the magnetic flux linked with the solenoid and hence an  Due to this force, all the free electrons are accumulate at the end A which
electric current is induced. Due to the flow of induced current, the coil become a produces the potential difference across the rod which inturn establishes an
magnetic dipole whose two magnetic poles are on either end of the coil. electric field ⃗⃗⃗𝐸 directed along BA
 Here the cause producing the induced current is the movement of the magnet.  Due to the electric field, the Coulomb force starts acting on the free electron
 According to Lenz’s law, the induced current should flow in such a way that it along AB and it is given by,
opposed the movement of the north pole towards coil. ⃗⃗⃗𝐹𝐸 = − 𝑒 ⃗⃗⃗𝐸 − − − − − (2)
 It is possible if the end nearer to the magnet becomes north pole. Then it repels ⃗⃗⃗ ⃗⃗⃗
 At equilibrium, | 𝐹𝐵 | = | 𝐹𝐸 |
the north pole of the bar magnet and opposed the movement of the magnet.
 Once pole end are known, the direction of the induced current could be found |−𝑒 (⃗⃗⃗𝑣 𝑋 ⃗⃗⃗𝐵 )| = |−𝑒 ⃗⃗⃗𝐸 |
by using right hand thumb rule. 𝐵 𝑒 𝑣 sin 90° = 𝑒 𝐸
 Whwn the bar magnet is with drawn, the nearer end becomes south pole which 𝐵𝑣 = 𝐸 − − − − (3)
attracts north pole of the bar magnet, opposing the receding of the magnet.  The potential difference between two ends of the rod is ,
 Thus the direction of the induced current can be found from Lenz’s law. 𝑉=𝐸𝑙=𝐵𝑣𝑙

victory R. SARAVANAN. M.Sc., M.Phil., B.Ed PG ASST [PHYSICS], GBHSS, PARANGIPETTAI - 608 502
12 PHYSICS UNIT - 4 ELECTOMAGNETIC INDUCTION AND ALTERNATING CURRENT COMPLETE GUIDE AND MODEL QUESTION
 Thus the Lorentz force on the free electrons is responsible to maintain this  Let Φ𝐵 be the magnetic flux linked with each turn of the coil of turn ‘N’, then
potential difference and hence produces an emf total flux linkage (𝑁Φ𝐵 )
𝝐=𝑩𝒍𝒗 − − − − (4) is directly proportional to the current ‘𝑖’
 Since this emf is produced due to the movement of the rod, it is often called as N Φ𝐵 ∝ 𝑖 (𝑜𝑟) N Φ𝐵 = 𝐿 𝑖
motional emf. 𝐍 𝚽𝑩
∴ 𝐋=
8. Define eddy currents. Demonstrate the production of eddy currents. 𝒊
Eddy currents:  Where, L  constant called coefficient of self induction (or) self inductance
 When magnetic flux linked with a conductor in the form of a sheet or a plate  When the current (𝑖) changes with time, an emf is induced in the coil and it is
changes, an emf is induced. given by,
 As a result, the induced current flow in concentric circular paths which 𝑑(N Φ𝐵 ) 𝑑 (𝐿 𝑖) 𝒅𝒊
∈= − = − = −𝑳
resembles eddies of water. Hence these are known as Eddy currents or Foucault 𝑑𝑡 𝑑𝑡 𝒅𝒕
currents. ∈
∴ 𝑳 = −
Demonstration : 𝒅𝒊
( )
 Let a pendulum that can be freely suspended between the poles of a powerful 𝒅𝒕
Coefficient of self induction - Definition :
electromagnet.
 Self inductance of a coil is defined as the flux linkage of the coil, when 1 A
 Keeping the magnetic field switched off, If the pendulum is made to oscillate, it
current flows through it.
executes a large number of oscillations before stops. Here air friction is a only
 Self inductance of a coil is also defined as the opposing emf induced in the coil,
damping force.
when the rate of change of current through the coil is 1 A s-1
 When the electro magnet is switched on, and the pendulum is made to oscillate, it
11. How will you define the unit of inductance?
comes to rest within a few oscillations. Because eddy currents are produced in it
Unit of inductance :
and it will oppose the oscillations (Lenz’s law)
 Inductance is a scalar and its unit is 𝑾𝒃 𝑨−𝟏 (or) 𝑽 𝒔 𝑨=𝟏 (or) henry (H)
 However some slots are cut in the disc, the eddy currents are reduced and now
 It dimension is [𝑴 𝑳𝟐 𝑻−𝟐 𝑨−𝟐 ]
the pendulum executes several oscillations before coming to rest.
Definition - 1 :
 This clearly demonstrates the production of eddy current in the disc of the 𝐍 𝚽𝑩
pendulum.  The self inductance is given by, 𝐋 =
𝒊
9. What are the drawbacks of Eddy currents. How it is minimized?  The inductance of the coil is one henry if a current of 1 A produces unit fux
Drawbacks of Eddy currents : linkage in the coil.
 When eddy currents flow in the conductor, a large amout of energy is dissipated Definition - 2 :

in the form of heat.  The self inductance is given by, 𝑳 = − 𝒅𝒊
( )
 The energy loss due to flow of eddy current is inevitable but it can be reduced. 𝒅𝒕

 To reduce eddy current losses, the core of the transformer is made up of thin  The inductance of the coil is one henry if a current changing at the rate of
laminas insulated from one another. In case of electric motor the winding is 𝟏 𝑨 𝒔−𝟏 induces an opposing emf of 1 V in it.
made up of a group of wire insulated from one another. 12. Discuss the physical significance of inductance.
 The insulation used does not allow huge eddy currents to flow and hence losses Physical inductance of inductance :
are minimized.
10. Explain self induction and define coefficient of self induction on the basis of
(1) magnetic flux and (2) induced emf
Self induction :
 When an electric current flowing
through a coil changes, an emf is
induced in the same coil. This  Generally inertia means opposition to change the state of the body.
phemomenon is known as self  In translational motion, mass is a measure of inertia, whereas in rotational
induction. The emf induced is called motion, moment of inertia is a measure of rotational inertia.
self-induced emf.  Simillarly inductance plays the same role in a circuit as the mass and moment of
inertia play in mechanical motion.
victory R. SARAVANAN. M.Sc., M.Phil., B.Ed PG ASST [PHYSICS], GBHSS, PARANGIPETTAI - 608 502
12 PHYSICS UNIT - 4 ELECTOMAGNETIC INDUCTION AND ALTERNATING CURRENT COMPLETE GUIDE AND MODEL QUESTION
 When a ciruit is switched on, the increasing current induces an emf which  Let ‘dW’ be the workdone in moving a charge ‘dq’ in a time ‘dt’ against the
opposes the growth of current in a circuit. opposition, then
 Similllarly, when a circuit is broken, the decreaing current induces an emf in the 𝑑𝑊 = − ∈ 𝑑𝑞 = − ∈ 𝑖 𝑑𝑡
reverese direction which opposed the decay of the current. 𝑑𝑖
𝑑𝑊 = − [−𝐿 ] 𝑖 𝑑𝑡 = 𝐿 𝑖 𝑑𝑖
 Thus inductance on the coil opposes any change in current and tries to maintain 𝑑𝑡
the original state.  Total wor done in establishing the current ‘𝑖’ is
𝑖
13. Assuming that the length of the solenoid is large when compared to its 𝑖2 1
diameter, find the equation for its inductance. 𝑊 = ∫ 𝑑𝑊 = ∫ 𝐿 𝑖 𝑑𝑖 = 𝐿 [ ] = 𝐿 𝑖 2
2 0 2
Self inductance of a long solenoid (L) :
 Consider a long solenoid of length ‘𝑙’, area  This work done is stored as magnetic potential energy. (i.e)
𝟏
of cross section ‘A’ having ‘N’ number of 𝑼𝑩 = 𝑳 𝒊𝟐
turns 𝟐
 The energy stored per unit volume of the space is called energy density (𝑢𝐵 )
 Let ‘𝑛’ be number of turns per unit length
and it is given by,
(i.e.) turn density 1 2
 When an electric current ‘𝑖’ is passed 𝑒𝑛𝑒𝑟𝑔𝑦 (𝑈𝐵 ) 2 𝐿 𝑖 1 (𝜇𝑜 𝑛2 𝐴 𝑙) 𝑖 2
through the coil, a magnetic field at any 𝑢𝐵 = = =
𝑣𝑜𝑙𝑢𝑚𝑒 (𝐴 𝑙) 𝐴𝑙 2 𝐴𝑙
point inside the solenoid is, 𝝁𝒐 𝒏 𝟐 𝒊𝟐
𝐵 = 𝜇𝑜 𝑛 𝑖 𝒖𝑩 =
𝟐
 Due to this field, the magnetic flux linked with the solenoid is, 𝑩𝟐
𝒖𝑩 = [∵ 𝐵 = 𝜇𝑜 𝑛 𝑖]
⃗⃗⃗ = ∮ 𝐵 𝑑𝐴 cos 90° = 𝐵 𝐴
Φ𝐵 = ∮ ⃗⃗⃗𝐵 . 𝑑𝐴 𝟐 𝝁𝒐
15. Explain mutual induction. Define coefficient of mutual induction on the basis
Φ𝐵 = [𝜇𝑜 𝑛 𝑖] 𝐴
of (1) magnetic flux and (2) induced emf
 Hence the total magnetic flux linked (i.e.) flux linkage
Mutual induction :
𝑁 Φ𝐵 = 𝑁 𝜇𝑜 𝑛 𝑖 𝐴 = (𝑛 𝑙) 𝜇𝑜 𝑛 𝑖 𝐴
𝑵 𝚽𝑩 = 𝝁𝒐 𝒏𝟐 𝒊 𝑨 𝒍
 Let ‘L’ be the self inductance of the solenoid, then
𝑁 Φ𝐵 𝜇𝑜 𝑛2 𝑖 𝐴 𝑙
𝐿= =
𝑖 𝑖
𝟐
𝑳 = 𝝁𝒐 𝒏 𝑨 𝒍
 If the solenoid is filled with a dielectric medium of relative permeability ‘𝜇𝑟 ’,
then ; 𝑳 = 𝝁𝒐 𝝁𝒓 𝒏 𝟐 𝑨 𝒍 = 𝝁 𝒏 𝟐 𝑨 𝒍
 Thus, the inductance depens on  When an electric current passing through a coil changes with time, an emf is
(i) geomentry of the solenoid induced in the neighbouring coil. This phenomenon is known as mutual
(ii) medium present inside the solenoid induction and the emf is called mutually induced emf.
14. An inductor of inductance ‘L’ carries an electric current ‘𝒊’. How much energy  Consider two coils 1 and 2 which are placed close to each other. If an electric
is stored while establishing the current in it? current ‘𝑖1 ’ is sent through coil -1, the magnetic field produced by it also linked
Energy stored in an solenoid : with the coil -2
 Whenever a current is established in the circuit, the inductance opposes the  Let ‘Φ21 ’ be the magnetic flux linked with each turn of the coil-2 of 𝑁2 turns due
growth of the current. to coil -1, then the total flux linked with coil -2 is proportional to the current ‘𝑖1 ’
 To establish the current, work has to done against this opposition. This work in the coil -`1 (i.e.)
done is stored as magnetic potential energy. 𝑁2 Φ21 ∝ 𝑖1 (𝑜𝑟) 𝑁2 Φ21 = 𝑀21 𝑖1
 Consider an inductor of negligible resistance, the induced emf ‘∈’ at any instant 𝑵𝟐 𝚽𝟐𝟏
∴ 𝑴𝟐𝟏 = − − − − (𝟏)
‘t’ is 𝒊𝟏
𝑑𝑖  Here 𝑀21 → constant called coefficient of mutual induction or mutual
∈ = −𝐿
𝑑𝑡 inductance coil -2 with respect to coil -1
victory R. SARAVANAN. M.Sc., M.Phil., B.Ed PG ASST [PHYSICS], GBHSS, PARANGIPETTAI - 608 502
12 PHYSICS UNIT - 4 ELECTOMAGNETIC INDUCTION AND ALTERNATING CURRENT COMPLETE GUIDE AND MODEL QUESTION
 When the current ‘𝑖1 ’ changes with time, an emf ‘∈2 ’ is induced in coil -2 and it is  Simillarly, Let ‘𝑖2 ’ be the current flowing through solenoid -2, then the magnetic
given by, field produced inside it is,
𝑑 (𝑁2 Φ21 ) 𝑑 (𝑀21 𝑖1 ) 𝑑𝑖1 𝐵2 = 𝜇𝑜 𝑛2 𝑖2
∈2 = − = − = − 𝑀21
𝑑𝑡 𝑑𝑡 𝑑𝑡  Hence the magnetic flux linked with each turn of solenoid -1 due to solenoid -2 is
∈𝟐
∴ 𝑴𝟐𝟏 = −
𝒅𝒊
− − − − (2) Φ12 = ∮ ⃗⃗⃗𝐵2 . ⃗⃗⃗⃗⃗
𝑑𝐴2 = ∮ 𝐵2 𝑑𝐴2 cos 0° = 𝐵2 𝐴2
( 𝟏)
𝒅𝒕 Φ12 = (𝜇𝑜 𝑛2 𝑖2 ) 𝐴2
 Simillarly,  Then total flux linkage of solenoid -1 of 𝑁1 turns is
𝑵𝟏 𝚽𝟏𝟐
𝑴𝟏𝟐 = − − − − (𝟑) 𝑁1 Φ12 = (𝑛1 𝑙 ) (𝜇𝑜 𝑛2 𝑖2 ) 𝐴2
𝒊𝟏𝟐 𝑁1 Φ12 = 𝜇𝑜 𝑛1 𝑛2 𝐴2 𝑙 𝑖2 − − − (3)
∈𝟏
& 𝑴𝟏𝟐 = − − − − − (4)  So the mutual inductance of solenoid -1 with respect to solenoid -2 is given by,
𝒅𝒊𝟐 𝑁1 Φ12 𝜇𝑜 𝑛1 𝑛2 𝐴2 𝑙 𝑖2
( ) 𝑀12 = =
𝒅𝒕
 Here 𝑀21 → constant called coefficient of mutual induction or mutual 𝑖2 𝑖2
inductance coil -2 with respect to coil -1 𝑴𝟏𝟐 = 𝝁𝒐 𝒏𝟏 𝒏𝟐 𝑨𝟐 𝒍 − − − − (4)
Coefficient of mutual induction - Definition :  From equation (2) and (4), 𝑴𝟏𝟐 = 𝑴𝟐𝟏
 The mutual inductance is defined as the flux linkage of the one coil, when 1 A  In general, the mutual inductance between two long co-axial solenoids is ,
current flow through other coil. 𝑴 = 𝝁 𝒐 𝒏 𝟏 𝒏 𝟐 𝑨𝟐 𝒍
 Mutual inductance is also the opposing emf induced in one coil, when the rate of  If the solenoid is filled with a dielectric medium of relative permeability ‘𝜇𝑟 ’,
change of current through other coil is 1 𝐴 𝑠 −1 then 𝑴 = 𝝁 𝒐 𝝁 𝒓 𝒏 𝟏 𝒏 𝟐 𝑨𝟐 𝒍 = 𝝁 𝒏 𝟏 𝒏 𝟐 𝑨𝟐 𝒍
16. Show that the mutual inductance between a pair of coils is same (𝑴𝟏𝟐 = 𝑴𝟐𝟏 )  Thus, the inductance depens on
Mutual inductance between a pair of coils : (i) geomentry of the solenoids
 Consider two long co-axial solenoids of (ii) medium present inside the solenoids
same length ‘𝑙’ (iii) proximity of the two soienoids
 Let 𝐴1 and 𝐴2 be the area of cross section 17. How will you induce an emf by changing the area enclosed by the coil.
of the solenoids. Here 𝐴1 > 𝐴2 EMF induced by changing area enclosed by the coil
 Let the turn density of these solenoids are  Consider a conducting rod of length ‘𝑙’
𝑛1 and 𝑛2 resectively. moving with a velocity ‘𝑣’ towards left
 Let ‘𝑖1 ’ be the current flowing through on a rectangular metallic frame work.
solenoid -1, then the magnetic field  The whole arangemetn is placed in a
produced inside it is, uniform magnetic field ‘ ⃗⃗⃗𝐵 ’ acting
𝐵1 = 𝜇𝑜 𝑛1 𝑖1 perpendicular to the plane of the coil
 Hence the magnetic flux linked with each turn of solenoid -2 due to solenoid -1 inwards.
is  As the rod moves from AB to DC in a
time ‘dt’, the area enclosed by the loop and hence the magnetic flux through the
Φ21 = ∮ ⃗⃗⃗𝐵1 . ⃗⃗⃗⃗⃗
𝑑𝐴2 = ∮ 𝐵1 𝑑𝐴2 cos 0° = 𝐵1 𝐴2 loop decreases.
Φ21 = (𝜇𝑜 𝑛1 𝑖1 ) 𝐴2  The change in magnetic flux in time ’dt’ is
 Then total flux linkage of solenoid -2 of 𝑁2 turns is 𝑑Φ𝐵 = 𝐵 𝑑𝐴 = 𝐵 (𝑙 𝑋 𝑣 𝑑𝑡)
𝑁2 Φ21 = (𝑛2 𝑙 ) (𝜇𝑜 𝑛1 𝑖1 ) 𝐴2 𝑑Φ𝐵
=𝐵𝑙𝑣
𝑁2 Φ21 = 𝜇𝑜 𝑛1 𝑛2 𝐴2 𝑙 𝑖1 − − − − (1) 𝑑𝑡
 So the mutual inductance of solenoid -2 with respect to solenoid -1 is given by,  This change in magnetic flux results and induced emf and it is given by,
𝑁2 Φ21 𝜇𝑜 𝑛1 𝑛2 𝐴2 𝑙 𝑖1 𝑑Φ𝐵
𝑀21 = = ∈= = 𝑩𝒍𝒗
𝑖1 𝑖1 𝑑𝑡
𝑴𝟐𝟏 = 𝝁𝒐 𝒏𝟏 𝒏𝟐 𝑨𝟐 𝒍 − − − − (2)  This emf is called motional emf. The direction of induced current is found to be
clock wise from Fleming’s right hand rule.

victory R. SARAVANAN. M.Sc., M.Phil., B.Ed PG ASST [PHYSICS], GBHSS, PARANGIPETTAI - 608 502
12 PHYSICS UNIT - 4 ELECTOMAGNETIC INDUCTION AND ALTERNATING CURRENT COMPLETE GUIDE AND MODEL QUESTION
18. What are the advantages of stationary armature - rotating field alternator? Illustration :
Advantages of stationary armature - rotating field alternator :  Let an electric power of 2 MW is transmitted through the transmission lines of
 The current is drawn directly from fixed terminals on the stator without the use resistance 40 Ω at 10 𝑘𝑉 and 100 𝑘𝑉
of brush contacts. 𝑃 2 𝑋 106
(i) 𝑃 = 2 𝑀𝑊, 𝑅 = 40 Ω, 𝑉 = 10 𝑘𝑉, then ; 𝐼 = = = 200 𝐴
 The insulation of stationary armature winding is easier. 𝑉 10 𝑋 103
Power loss = 𝐼 2 𝑅 = (200)2 𝑋 40 = 1.6 𝑋 106 𝑊
 The number of slip rings is reduced. Moreover the sliding contacts are used for
1.6 𝑋 106
low-voltage DC source. % of Power loss = = 0.8 = 𝟖𝟎 %
 Armature windings can be constructed more rigidly to prevent deformation due 2 𝑋 106
𝑃 2 𝑋 106
to any mechanical stress. (ii) 𝑃 = 2 𝑀𝑊, 𝑅 = 40 Ω, 𝑉 = 100 𝑘𝑉 , then; 𝐼 = = 3 = 20 𝐴
𝑉 100 𝑋 10
19. Explain various energy losses in a transformer. Power loss = 𝐼 2 𝑅 = (20)2 𝑋 40 = 0.016 𝑋 106 𝑊
Energy losses in a transformer : 0.016 𝑋 106
(i) Core loss or Iron loss : % 𝑜𝑓 𝑝𝑜𝑤𝑒𝑟 𝑙𝑜𝑠𝑠 = = 0.008 = 𝟎. 𝟖 %
2 𝑋 106
 Hysterisis loss and eddy current loss are known as core loss or Iron loss.
 Thus it is clear that, when an electric power is transmitted at high voltage, the
 When transformer core is magnetized or demangnetized repeatedly by the
power loss is reduced to a large extent.
alternating voltage applied across primary coil, hyterisis takes place and
 So at transmitting point the voltage is increased and the corresponding current
some energy lost in the form of heat. It is minimized by using silicone steel
is decreased by using step-up transformer. At receiving point, the voltage is
in making transformer core.
decreased and the current is increased by using step-down transformer
 Alternating magnetic flux in the core induces eddy currents in it. Therefore
21. Obtain the expression for average value of alternating current.
there is energy loss due to the flow of eddy current called eddy current loss.
Average or Mean value of AC :
It is minimized by using very thin laminations of transformer core.
 The average value of AC is defined as the average of all values of current over a
(ii) Copper loss :
positive half-cycle or negative half-cycle.
 The primary and secondary coils in transformer have electrical resistance.
Expression :
 When an electric current flows through them, some amount of energy is
 The average or mean value of AC over one
dissipated due to Joule’s heating and it is known as copper loss. It is
complete cycle is zero. Thus the average or
minimized by using wires of larger diameter (thicki wire)
mean value is measured over one half of a
(iii) Flux leakage :
cycle.
 The magnetic flux linked with primary coil is not completely linked with
 The alternating current at any instant is
secondary.
𝑖 = 𝐼𝑚 sin 𝜔𝑡 = 𝐼𝑚 sin 𝜃
Energy loss due to this flux leakage is minimize by winding coils one over
 The sum of all currents over a half-cycle is
the other.
given by area of positive half-cycle (or)
20. Discuss the advantages of AC in long distance power transmission.
negative half-cycle.
Long distance power transmission :
 Consider an elementary strip of thickness ‘𝑑𝜃’ in positive half-cycle,
 The electric power is generated in power stations using AC generators are
Area of the elementary strip = 𝑖 𝑑𝜃
transmitted over long distances through transmission lines to reach towns or
cities. This process is called power transmission.  Then area of positive half-cycle,
𝜋
𝜋
 But during power transmission, due to Joules’s heating (( 𝐼 2 𝑅 ) in the
= ∫ 𝑖 𝑑𝜃 = ∫ 𝐼𝑚 sin 𝜃 𝑑𝜃 = 𝐼𝑚 [− cos 𝜃]𝜋0
transmission lines, sizable fraction of electric power is lost. 0
0
 This power loss can be reduced either by reducing current (I) or by reducing = − 𝐼𝑚 [cos 𝜋 − cos 0] = − 𝐼𝑚 [−1 − 1] = 2 𝐼𝑚
resistance (R)
 Then Average value of AC,
 Here the resistance ‘R’ can be reduced with thick wires of copper or aluminium. 𝑎𝑟𝑒𝑎 𝑜𝑓 𝑝𝑜𝑠𝑖𝑡𝑣𝑒 𝑜𝑟 𝑛𝑒𝑔𝑎𝑡𝑖𝑣𝑒 ℎ𝑎𝑙𝑓 − 𝑐𝑦𝑐𝑙𝑒
But this increases the cost of production of transmission lines and hence this 𝐼𝑎𝑣 =
𝑏𝑎𝑠𝑒 𝑙𝑒𝑛𝑔𝑡ℎ 𝑜𝑓 ℎ𝑎𝑙𝑓 − 𝑐𝑦𝑐𝑙𝑒
method is not economically viable. 𝟐 𝑰𝒎
 Thus by using transformer, the current is reduced by stepped up the alternating 𝐈𝒂𝒗𝒈 = = 𝟎. 𝟔𝟑𝟕 𝑰𝒎
𝝅
voltage and thereby reducing power losses to a greater extent.
 For negative half-cycle ; 𝐈𝒂𝒗𝒈 = − 𝟎. 𝟔𝟑𝟕 𝑰𝒎

victory R. SARAVANAN. M.Sc., M.Phil., B.Ed PG ASST [PHYSICS], GBHSS, PARANGIPETTAI - 608 502
12 PHYSICS UNIT - 4 ELECTOMAGNETIC INDUCTION AND ALTERNATING CURRENT COMPLETE GUIDE AND MODEL QUESTION
22. Obtain an expression for RMS value of alternating current. 23. Draw the phasor diagram and wave diagram for that current ‘𝒊’ leads the
RMS value of AC (𝐼𝑅𝑀𝑆 ) : voltage ‘V’ by phase angle of ‘𝝓’
 The root mean squae value of an alternating current is defined as the square Phasor and wave diagram of ‘𝒊’ leads ‘V’ by ‘𝝓’
root of the mean of the squares of all currents over one cycle.  Let the alternating current and voltage at any instant is,
Expression : 𝑣 = 𝑉𝑚 sin 𝜔𝑡
 The alternating current at any 𝑖 = 𝐼𝑚 sin(𝜔𝑡 + 𝜙)
instant is
𝑖 = 𝐼𝑚 sin 𝜔𝑡 = 𝐼𝑚 sin 𝜃
 The sum of the squares of all
currents over one cycle is given by
the area of one cycle of squared
wave.
 Consider an elementary area of
thickness ‘𝑑𝜃’ in the first half-cycle of the squared current wave.
Area of the element = 𝑖 2 𝑑𝜃
 Area of one cycle of squared wave,
2𝜋
2𝜋
= ∫ 𝑖 2 𝑑𝜃 = ∫ 𝐼𝑚 2 sin2 𝜃 𝑑𝜃 24. Find out the phase relation ship between voltage and current in a pure
0
0 resistive circuit.
2𝜋
1 − cos 2𝜃 AC circuit containing pure resistor :
= 𝐼𝑚 2 ∫ [ ] 𝑑𝜃 [∵ cos 2𝜃 = 1 − 2 sin2 𝜃 ]
0 2
𝐼𝑚 2 2𝜋 2𝜋
= [∫ 𝑑𝜃 − ∫ cos 2𝜃 𝑑𝜃]
2 0 0
𝐼𝑚 2 sin 2𝜃 2𝜋
= [𝜃− ]
2 2 0
𝐼𝑚 2 sin 4𝜋 sin 0
= [2𝜋 − −0+ ]
2 2 2
[∵ sin 0 = sin 4𝜋 = 0]
 Let a pure resistor of resistance ‘R’ connected across an alternating voltage
𝐼𝑚 2 source ‘𝑣’
= [2 𝜋] = 𝐼𝑚 2 𝜋
2  The instantaneous value of the alternating voltage is given by,
 Hence, 𝑣 = 𝑉𝑚 sin 𝜔𝑡 − − − − (1)
𝑎𝑟𝑒𝑎 𝑜𝑓 𝑜𝑛𝑒 𝑐𝑦𝑐𝑙𝑒 𝑜𝑓 𝑠𝑞𝑢𝑎𝑟𝑒𝑑 𝑤𝑎𝑣𝑒  Let ‘𝑖’ be the alternating current flowing in the circuit due to this voltage, then
𝐼𝑅𝑀𝑆 = √ the potential drop across ‘R’ is
𝑏𝑎𝑠𝑒 𝑙𝑒𝑛𝑔𝑡ℎ 𝑜𝑓 𝑜𝑛𝑒 𝑐𝑦𝑐𝑙𝑒
𝑉𝑅 = 𝑖 𝑅 − − − − (2)
𝐼𝑚 2 𝜋 𝐼𝑚 2  From Kirchoff’s loop rule, 𝑣 − 𝑉𝑅 = 0
IRMS = √ = √ (𝑜𝑟) 𝑣 = 𝑉𝑅
2𝜋 2
𝑉𝑚 sin 𝜔𝑡 = 𝑖 𝑅
𝑰𝒎 𝑉𝑚
𝐈𝐑𝐌𝐒 = = 𝟎. 𝟕𝟎𝟕 𝑰𝒎 𝑖= sin 𝜔𝑡
√𝟐 𝑅
 Simillarly for alternating voltage, it can be shown that, 𝒊 = 𝑰𝒎 𝐬𝐢𝐧 𝝎𝒕 − − − − (3)
𝑽𝒎 𝑉𝑚
𝐕𝐑𝐌𝐒 = = 𝟎. 𝟕𝟎𝟕 𝑽𝒎 Here, = 𝐼𝑚 → Peak value of AC
𝑅
√𝟐  From equation (1) and (3), it is clear that, the applied voltage and the current
 RMS value of AC is also called effective value (𝐼𝑒𝑓𝑓 )
are in phase with each other. This is indicated in the phasor and wave diagram.
victory R. SARAVANAN. M.Sc., M.Phil., B.Ed PG ASST [PHYSICS], GBHSS, PARANGIPETTAI - 608 502
12 PHYSICS UNIT - 4 ELECTOMAGNETIC INDUCTION AND ALTERNATING CURRENT COMPLETE GUIDE AND MODEL QUESTION
 From equation (1) and (3), it is clear that current lags behind the applied
𝝅
voltage by . This is indicated in the phasor and wave diagram.
𝟐

25. Find out the phase relation ship between voltage and current in a pure
inductive circuit.
AC circuit containing pure inductor: Inductive reactance (𝑿𝑳 ) :
 Let a pure inductor of inductance  In pure inductive circuit, ‘𝜔 𝐿’ is the resistance offered by the inductor and it is
‘L’ connected across an alternating called inductive reactance (𝑋𝐿 ). Its unit is ohm (𝜴)
voltage source ‘𝑣’ 𝑿𝑳 = 𝝎 𝑳 = 𝟐 𝝅 𝒇 𝑳
 The instantaneous value of the 26. Find out the phase relation ship between voltage and current in a pure
alternating voltage is given by, capacitive circuit.
𝑣 = 𝑉𝑚 sin 𝜔𝑡 − − − − (1) AC circuit containing pure capacitor :
 Let ‘𝑖’ be the alternating current  Let a pure capacitor of capacitance
flowing in the circuit due to this ‘C’ connected across an alternating
voltage, which induces a self induced emf (back emf) across ‘L’ and it is given by voltage source ‘𝑣’
𝑑𝑖  The instantaneous value of the
∈= − 𝐿 − − − − (2)
𝑑𝑡 alternating voltage is given by,
 From Kirchoff’s loop rule, 𝑣 − (−∈) = 0 𝑣 = 𝑉𝑚 sin 𝜔𝑡 − − − − (1)
(𝑜𝑟) 𝑣 = −∈  Let ‘𝑞’ be the instantaneous charge
𝑑𝑖 on the capacitor. The emf across the capacitor at that instant is,
𝑉𝑚 sin 𝜔𝑡 = − (− 𝐿 ) 𝑞
𝑑𝑡
𝑑𝑖 ∈= − − − − (2)
𝑉𝑚 sin 𝜔𝑡 = 𝐿 𝐶
𝑑𝑡  From Kirchoff’s loop rule, 𝑣 − ∈= 0
𝑉𝑚 (𝑜𝑟) 𝑣 = ∈
∴ 𝑑𝑖 = sin 𝜔𝑡 𝑑𝑡 𝑞
𝐿 𝑉𝑚 sin 𝜔𝑡 =
 Integrate on both sides, 𝐶
𝑉𝑚 ∴ 𝑞 = 𝐶 𝑉𝑚 sin 𝜔𝑡
𝑖= ∫ sin 𝜔𝑡 𝑑𝑡  By the definition of current,
𝐿
𝑉𝑚 − cos 𝜔𝑡 𝑉𝑚 𝜋 𝑑𝑞 𝑑(sin 𝜔𝑡)
𝑖= ( )= [− sin ( − 𝜔𝑡)] 𝑖= = 𝐶 𝑉𝑚 = 𝐶 𝑉𝑚 (cos 𝜔𝑡) 𝜔
𝐿 𝜔 𝜔𝐿 2 𝑑𝑡 𝑑𝑡
𝑉𝑚 𝜋 𝜋 𝑉𝑚 𝜋
𝑖= sin (𝜔𝑡 − ) 𝑖 = 𝜔 𝐶 𝑉𝑚 sin ( + 𝜔𝑡) = sin ( + 𝜔𝑡)
𝜔𝐿 2 2 1
( ⁄𝜔 𝐶 ) 2
𝝅
𝒊 = 𝑰𝒎 𝐬𝐢𝐧 (𝝎𝒕 − ) − − − − (3) 𝒊 = 𝑰𝒎 𝐬𝐢𝐧 (𝝎𝒕 + − )
𝝅
− − − − (3)
𝟐 𝟐
𝑉𝑚 𝑉𝑚
Where, = 𝐼𝑚 → peak value of AC where, = 𝐼𝑚 → Peak value of AC
𝜔𝐿 (1⁄𝜔 𝐶 )
𝝅
 From equation (1) and (3), it is clear that current leads the applied voltage by .
𝟐

victory R. SARAVANAN. M.Sc., M.Phil., B.Ed PG ASST [PHYSICS], GBHSS, PARANGIPETTAI - 608 502
12 PHYSICS UNIT - 4 ELECTOMAGNETIC INDUCTION AND ALTERNATING CURRENT COMPLETE GUIDE AND MODEL QUESTION
 This is indicated in the phasor and wave diagram. 28. Define quality factor. Obtain an expression for it.
Definition :
 Q - factor is defined as the ratio of voltage across L (or) C to the applied
voltage at resonance.
Expression :
 The current in the series RLc circuit becomes maximum at resonance.
 Due to the increase in current, the voltage across L and C are also increased,
 This magnification of voltages at series resonance is termed as Q - factor.
 By definition,
𝑣𝑜𝑙𝑡𝑎𝑔𝑒 𝑎𝑐𝑟𝑜𝑠𝑠 𝐿 (𝑜𝑟) 𝐶
Capacitive reactance (𝑿𝑪 ) : 𝑄 − 𝑓𝑎𝑐𝑡𝑜𝑟 =
𝑎𝑝𝑝𝑙𝑖𝑒𝑑 𝑣𝑜𝑙𝑡𝑎𝑔𝑒
 In pure capacitive circuit, ‘1⁄𝜔 𝐶 ’ is the resistance offered by the capacitor and
𝐼𝑚 𝑋𝐿 𝑋𝐿 𝜔𝑅 𝐿 1 𝐿
it is called capacitive reactance (𝑋𝐶 ). Its unit is ohm (𝜴) 𝑄 − 𝑓𝑎𝑐𝑡𝑜𝑟 = = = =
𝟏 𝟏 𝐼𝑚 𝑅 𝑅 𝑅 √𝐿 𝐶 𝑅
𝑿𝑪 = =
𝝎𝑪 𝟐𝝅𝒇𝑪 𝟏 𝑳
𝑸 − 𝒇𝒂𝒄𝒕𝒐𝒓 = √
27. Explain resonance in series RLC circiuit. 𝑹 𝑪
Resonance on series in RLC circuit :
 The physical meaning is that Q - factor indicates the number of times the
 When the frequency of applied alternating source is increases, the inductive voltage across L (or) C is greaterthan the applied voltage at resonance.
reactance (𝑿𝑳 ) increases, where as capacitive reactance (𝑿𝑪 ) decreases.
29. Obtain an expression for average power of AC over a cycle. Discuss its special
 At particular frequency (𝜔𝑅 ), 𝑿𝑳 = 𝑿𝑪 cases.
 At this stage, the frequency of applied source (𝜔𝑅 ) is equal to the natural Average power of AC :
frequency of the RLC circuit, the current in the circuit reaches its maximum  Power of a circuit is defined as the rate of consumption. It is given by the
value. product of the voltage and current.
 Then the circuit is said to be in electrical resonance. The frequency at which  The alternating voltage and alternating current in the series RLC circuit at an
resonance takes place is called resonant frequency. instance are given by,
 Thus at resonance, 𝑋𝐿 = 𝑋𝐶 𝑣 = 𝑉𝑚 sin 𝜔𝑡
1 1
𝜔𝑅 𝐿 = (𝑜𝑟) 𝜔𝑅2 = 𝑖 = 𝐼𝑚 sin(𝜔𝑡 + 𝜙)
𝜔𝑅 𝐶 𝐿𝐶  Then the instantaneous power is given by,
 Hence the resonant angular frequency, 𝑃 = 𝑣 𝑖 = 𝑉𝑚 sin 𝜔𝑡 𝐼𝑚 sin(𝜔𝑡 + 𝜙)
1 𝑃 = 𝑉𝑚 𝐼𝑚 sin 𝜔𝑡 (sin 𝜔𝑡 cos 𝜙 − cos 𝜔𝑡 sin 𝜙)
𝜔𝑅 =
√𝐿 𝐶 𝑃 = 𝑉𝑚 𝐼𝑚 (𝑠𝑖𝑛2 𝜔𝑡 cos 𝜙 − sin 𝜔𝑡 cos 𝜔𝑡 sin 𝜙 )
 And resonant frequency,
1 1
𝑓𝑅 =  Here the average of 𝑠𝑖𝑛2 𝜔𝑡 over a cycle is and that of sin 𝜔𝑡 cos 𝜔𝑡 is zero.
2 𝜋 √𝐿 𝐶 2
Effects of series resonance :  Thus average power over a cycle is,
1 𝑉𝑚 𝐼𝑚
 When series resonance occurs, the impedance 𝑃𝑎𝑣𝑔 = 𝑉𝑚 𝐼𝑚 ( cos 𝜙) = cos 𝜙
of the circuit is minimum and is equal to the 2 √2 √2
resistance of the circuit. So the current in the 𝑷𝒂𝒗𝒈 = 𝑽𝑹𝑴𝑺 𝑰𝑹𝑴𝑺 𝐜𝐨𝐬 𝝓
circuit becomes maximum. Where, 𝑉𝑅𝑀𝑆 𝐼𝑅𝑀𝑆 → apparent power and cos 𝜙 → power factor
 (i.e.) At resonance, Z = R & 𝐼𝑚 = 𝑚
𝑉 Special cases :
𝑅 (i) For purely resistive circuit, 𝜙 = 0 and cos 𝜙 = 1
 The maximum current at resonance depends on ∴ 𝑷𝒂𝒗𝒈 = 𝑽𝑹𝑴𝑺 𝑰𝑹𝑴𝑺
the value of resistance (R) 𝜋
(ii) For purely inductive or capacitive circuit, 𝜙 = ± and cos 𝜙 = 0.
 For smaller resistance, larger the current with sharper curve is obtained. But 2
for larger resistance, smaller the current with flat curve is obtained. ∴ 𝑷𝒂𝒗𝒈 = 𝟎

victory R. SARAVANAN. M.Sc., M.Phil., B.Ed PG ASST [PHYSICS], GBHSS, PARANGIPETTAI - 608 502
12 PHYSICS UNIT - 4 ELECTOMAGNETIC INDUCTION AND ALTERNATING CURRENT COMPLETE GUIDE AND MODEL QUESTION
𝑋 −𝑋 33. Show that the total energy is conserved during LC oscillations.
(iii) For series RLC circuit, 𝜙 =tan−1 [ 𝐿 𝐶 ]
𝑅
Conservation of energy LC oscillations :
∴ 𝑷𝒂𝒗𝒈 = 𝑽𝑹𝑴𝑺 𝑰𝑹𝑴𝑺 𝐜𝐨𝐬 𝝓
 During LC oscillations, the energy of the system oscillates between the electric
(iv) For series RLC circuit at resonance, 𝜙 = 0 and cos 𝜙 = 1. field of the capacitor and the magnetic field of the inductor.
∴ 𝑷𝒂𝒗𝒈 = 𝑽𝑹𝑴𝑺 𝑰𝑹𝑴𝑺
 Although these two energies vary with time, the total energy remains constant.
30. Write a note on wattful current and wattles current. (i.e)
Wattful current and Wattless current : 𝑞2 1
 Consider an AC circuit in which 𝑈 = 𝑈𝐸 + 𝑈𝐵 = + 𝐿 𝑖 2 = 𝑐𝑜𝑛𝑠𝑡𝑎𝑛𝑡
2𝐶 2
the voltage ( 𝑽𝑹𝑴𝑺 ) leads the Case (i) :
current (𝑰𝑹𝑴𝑺 ) by phase angle ‘𝜙’  When the charge of in the ccapacitor ; 𝑞 = 𝑄𝑚
 Resolve the current in to two and the current through the inducor ; 𝑖 = 0
perpendicular components, 𝑄𝑚2 𝑄𝑚2
(i) 𝑰𝑹𝑴𝑺 𝒄𝒐𝒔 𝝓 - Component 𝑈= +0= − − − − (1)
2𝐶 2𝐶
along 𝑽𝑹𝑴𝑺
 The total energy is wholly electrical.
(ii) 𝑰𝑹𝑴𝑺 𝒔𝒊𝒏 - Component
Case (ii) :
perpendicular to 𝑽𝑹𝑴𝑺
 When charge 𝑞 = 0 ; Current « 𝑖 = 𝐼𝑚 , the total energy,
 Here the component of current (𝑰𝑹𝑴𝑺 𝒄𝒐𝒔 𝝓) which is inphase with the voltage 1 1
is called ative component. The power consumed by this component = 𝑈 = 0 + 𝐿 𝐼𝑚2 = 𝐿 𝐼𝑚2
𝑽𝑹𝑴𝑺 𝑰𝑹𝑴𝑺 𝒄𝒐𝒔 𝝓 . It is known as wattfull current 2 2
𝑑𝑞 𝑑
 The other component of current which has a phase angle of with the voltage is [∵ 𝑖 = − =− (𝑄 cos 𝜔𝑡) = 𝑄𝑚 𝜔 sin 𝜔𝑡 = 𝐼𝑚 sin 𝜔𝑡]
𝑑𝑡 𝑑𝑡 𝑚
called reactive component. The power consumed by this current is zero. It is 𝑄
 Hence, 𝐼𝑚 = 𝑄𝑚 𝜔 = 𝑚
known as wattles current. √𝐿𝐶
31. Define power factor in various ways. Give some examples for power factor. 1 𝑄𝑚2 𝑄𝑚2
∴ 𝑈= 𝐿 [ ] = − − − − (2)
Power factor - Definitions : 2 𝐿𝐶 2𝐶
(i) The cosine of the angle lead or lag is called power factor (power factor =  Here the total energy is wholly magnetic
= cos 𝜙) Case (iii) :
𝑅 𝑅𝑒𝑠𝑖𝑠𝑡𝑎𝑛𝑐𝑒
(ii) Power factor = = 𝐼𝑚𝑝𝑒𝑑𝑎𝑛𝑐𝑒  When charge = 𝑞 , Current = 𝑖, then the total energy,
𝑍
𝑉 𝐼 cos 𝜙 𝑇𝑟𝑢𝑒 𝑝𝑜𝑤𝑒𝑟 𝑞2 1
(iii) Power factor = = 𝑈= + 𝐿 𝑖2
𝑉𝐼 𝐴𝑝𝑝𝑎𝑟𝑒𝑛𝑡 𝑝𝑜𝑤𝑒𝑟 2𝐶 2
Examples :  Here, 𝑞 = 𝑄𝑚 cos 𝜔𝑡 & 𝑖 = 𝑄𝑚 𝜔 sin 𝜔𝑡. So
 For purely resistive circuit, 𝜙 = 0 and cos 𝜙 = 1 𝑄𝑚2 𝑐𝑜𝑠 2 𝜔𝑡 1
𝑈= + 𝐿 𝑄𝑚2 𝜔2 sin2 𝜔𝑡
 For purely inductive or capacitive circuit, 2𝐶 2
𝜋 1
𝜙=± and cos 𝜙 = 0  Since, 𝜔2 =
2 𝐿𝐶
 For RLC circuit, power factor lies between 0 and 1 𝑄𝑚2 𝑐𝑜𝑠 2 𝜔𝑡 𝐿 𝑄𝑚2 sin2 𝜔𝑡
32. What are the advantages and disadvantages of AC over DC? 𝑈= +
2𝐶 2 𝐿𝐶
2
Advantages of AC over DC : 𝑄𝑚 𝑄𝑚2
𝑈= (𝑐𝑜𝑠 2 𝜔𝑡 + sin2 𝜔𝑡) = − − − (3)
 The generation of AC is cheaper than that of DC 2𝐶 2𝐶
 When AC is supplied at higher voltages, the transmission losses are small  From equation (1), (2) and (3) it is clear that the total energy of the system
compared to DC transmission. remains constant
 AC can easily be converted into DC with the help of rectifier.
Disadvantages of AC over DC :
 Alternating voltages cannot be used for certain application. (e.g) charging of
batteries, electroplating, electric traction etc.,
 At high voltages, it is more dangerous to work with AC than DC.

victory R. SARAVANAN. M.Sc., M.Phil., B.Ed PG ASST [PHYSICS], GBHSS, PARANGIPETTAI - 608 502
12 PHYSICS UNIT - 4 ELECTOMAGNETIC INDUCTION AND ALTERNATING CURRENT COMPLETE GUIDE AND MODEL QUESTION
2. Show mathematically that the rotation of a coil in a magnetic field over one
PART – IV 5 MARK LONG ANSWER QUESTIONS & ANSWERS rotation induces an alternating emf of one cycle.
1. Explain the applications of eddy currents (or) Focault currents. Induction of emf by changing relative orientation of the coil with the magnetic
Induction stove : field :
 It is used to cook food quickly and safely with less consumption. Below the
cooking zone, there is a tightly woind coil of insulated wire.
 A suitable cooking pan is placing over the cooking zone.
 When the stove is switched on, an AC flowing in the coil produces high
frequency alternating magnetic field which induces very strong eddy currents
in the cooking pan.
 The eddy currents in the pan produce so much of heat due to Joule heating
which is used to cook the food.
Eddy current brake :
 This types of brakes are generally used in high speed trains and roller coasters.
 Strong electromagnets are fixed just above the rails.To stop the train,
electromagnets are swiched on. The magnetic field of these magnets induces
eddy currents in the rails which oppose the movement of the train. This is eddy
current linear brake.
 In some cases, the circular disc connected in train is made to rotate in between
the pole of a electromagnet. When there is a relative motion between the disc
and the magnet, eddy currents are induced in the disc which stop the train. Ths
is eddy current circular brake.  Consider a rectangular coil of ‘N’ turns kept in a uniform magnetic field ‘B’
Eddy current testing :  The coil rotates in anti-clockwise direction with an angular velocity ‘𝜔’ about an
 It is one of the non - destructive testing methods to find defects like surface axis.
craks, air bubbles present in a specimen.  Initially let the plane of the coil be perpendicular to the field (𝜃 = 0) and the
 A coil of insulated wire is given an alternating electric current, so that it flux linked with the coil has its maximum value. (i.e.) Φ𝑚 = 𝐵 𝐴
produces an alternating magnetic field.  In time ‘t’, let the coil be rotated through an angle 𝜃 (= 𝜔𝑡), then the total flux
 When this coil is brought near the test surface, eddy current is induced in it, and linked is
the presence of defects caused the change in phase and amplitude of the eddy 𝑁 Φ𝐵 = 𝑁 𝐵 𝐴 cos 𝜔𝑡 = 𝑁 Φ𝑚 cos 𝜔𝑡
current.  According to Faraday’s law, the emf induced at that instant is,
 Thus the defects present in the specimen are identified. 𝑑 𝑑
∈= − (𝑁Φ𝐵 ) = − (𝑁 Φ𝑚 cos 𝜔𝑡)
Electro magnetic damping : 𝑑𝑡 𝑑𝑡
 The armature of the galvanometer coil is wound on a soft irom cylinder. = − 𝑁 Φ𝑚 (− sin 𝜔𝑡) 𝜔
 Once the armature is deflected, the relative motion between the soft irom ∈ = 𝑵 𝚽𝒎 𝝎 𝐬𝐢𝐧 𝝎𝒕 − − − − − (1)
cylinder and the radial magnetic field induces eddy current in the cylinder.  When 𝜃 = 90°, then the induced emf becomes maximum and it is given by,
 The damping force due to the flow of eddy current brings the armature to rest ∈𝒎 = 𝑵 𝚽𝒎 𝝎 = 𝑵 𝑩 𝑨 𝝎 − − − − − (2)
immediately and the galvanometer shows a steady deflection.  Therefore the value of induced emf at that instant is then given by,
 This is called electromagnetic damping. ∈ = ∈𝒎 𝐬𝐢𝐧 𝝎𝒕 − − − − − (3)
 Thus the induced emf varies as sine function of the time angle and this is called
sinusoidal emf or alternating emf.
 If this alternating voltage is given to a closed circuit, a sinusoidally varying
current flows in it. This current is called alternating current an is given by,
𝒊 = 𝑰𝒎 𝐬𝐢𝐧 𝝎𝒕 − − − − − (4)
 where, 𝑰𝒎 → peak value of induced current

victory R. SARAVANAN. M.Sc., M.Phil., B.Ed PG ASST [PHYSICS], GBHSS, PARANGIPETTAI - 608 502
12 PHYSICS UNIT - 4 ELECTOMAGNETIC INDUCTION AND ALTERNATING CURRENT COMPLETE GUIDE AND MODEL QUESTION
3. Elaborate the standard construction details of AC generator.  Assume the initial position of the field magnet is horizontal. At that instant, the
AC generator - construction : direction of magnetic field is perpendicular to the plane of the loop PQRS. The
 AC generator (alternator) is an energy conversion device. It converts induced emf is zero. It is represented by origin ‘O’ in the graph
mechanical energy used to rotate the coil or field magnet in to electrical energy.
 It works on the principle of electromagnetic induction.
 It consists of two major parts stator and rotor.
 In commercial alternators, the armature winding is mounted on stator and the
field magnet on rotor
Stator : It has three components
(i) Stator core (Armature) :
 It is made up of iron or steel alloy.
 It is a hollo cylinder and is laminated to minimize eddy current loss.
 The slots are cut on inner surface of the core to accommodate armature
windings.
(ii) Armature windings :
 It the coil wound on slots provided in the armature core. One or more than
one coil may be employed, depending on the type of alternator.  Let the magnetic field rotate in clock-wise direction.
Rotar :  When the field magnet rotates through 90, the magnetic field becomes parallel
 It consists magnetic field windings to PQRS. The induced emf’s across PQ and RS would become maximum.
 The magnetic poles are magnetized bhy DC source According to Flemming’s right hand rule, the direction of induced emf for PQ is
 The ends of field windings are connected to a pair of slip rings, attached to a downwards and for RS is upwards. Therefore the current flows along PQRS. The
common shaft about which rotor rotates. Slip rings rotate along with rotor. point A in the graph represents this maximum emf.
 To maintain connection between the DC source and field windings, two brushed  When field magnet rotates 180, the field is again perpendicular to PQRS and
are used which continuously slide over the slip rings the induced emf becomes zero. This is represented by point B
4. Explain the working of a single - phase AC generator with necessary diagram.  When field magnet rotates 270, the field is again parallel to PQRS, the induced
Single phase AC generator : emf is maximum but the direction is reversed. Thus the current flows along
 In a single phase AC generator, the armature conductors are connected in series SRQP. This is represented by point C.
so as to form a single circuit which generates a single-phase alternating emf and  On completion of 360, the induced emf becomes zero and it is represented by
hence it is called single-phase alternator. the point D.
Principle :  From the graph, it is clear that, when field magnet completes one rotation, the
 Electro magnetic induction emf induced in PQRS is alternating in nature.
Construction : 5. How are the three different emfs generated in a three-phase AC generator?
 Consider a stator core consisting Show the graphical representation of these three emfs.
of 2 slots in which 2 armature Three phase AC generator :
conductor PQ and RS are  If the AC generator consists three separate
mounted to form single - turn coils, which would give three separate
rectangular loop PQRS emfs, then it is called three-phase
 Rotor has 2 salient poles with generators.
field windings which can be Construction :
magnetized by means of DC  It has 6 slots, cut in its inner rim. Each slot
source. is 60 away from one another. six
Working : armature conductors are mounted in these
 The loop PQRS is stationary and slots.
is perpendicular to the plane of  The conductors 1 - 4, 2 - 5 and 3 - 6 are joined in series to form coils 1, 2 and 3
the paper.  So these coils are rectangular in shape and are 120 apart from one another.
victory R. SARAVANAN. M.Sc., M.Phil., B.Ed PG ASST [PHYSICS], GBHSS, PARANGIPETTAI - 608 502
12 PHYSICS UNIT - 4 ELECTOMAGNETIC INDUCTION AND ALTERNATING CURRENT COMPLETE GUIDE AND MODEL QUESTION
Working : Working :
 The initial position of the  The alternating voltage given to the primary coil, set up an alternating magnetic
field magnet is horizontal flux in the laminated core.
and field direction is  As the result of flux change, emf is induced in both primary and secondary coils.
perpendicular to the  The emf induced in the primary coil ‘∈𝑃 ’ is almost equal and opposite to the
plane of the coil - 1. applied voltage ‘𝑉𝑃 ’ and is given by,
 When it rotated from that 𝑑Φ𝐵
position in clock-wise 𝑉𝑃 = ∈𝑃 = − 𝑁𝑃 − − − − (1)
𝑑𝑡
direction, alternating emf  The frequency of alternating magnetic flux is same as the frequency of applied
‘∈1 ’ in coil - 1 begins a voltage. Therefore induced in secondary will also have same frequency as that
cycle from origin ‘O’ of applied voltage,
 When it rotated through  The emf induced in the secondary coil ‘∈𝑆 ’ is,
120, alternating emf ‘∈2 ’ 𝑑Φ𝐵
𝑉𝑆 = ∈𝑆 = − 𝑁𝑆 − − − − (2)
in coil - 2 statrs at point 𝑑𝑡
‘A’  Dividing equation (1) by (2),
 When it rotated through 240, alternating emf ‘∈3 ’ in coil - 3 statrs at point ‘B’ 𝑽𝑺 𝑵𝑺
= − − − − (3)
 Thus these emfs produced in the three phase AC generator have 120 phase 𝑽𝑷 𝑵𝑷
difference between one another. Where, K  transformation ratio
6. Explain the principle , construction and working of transformer.  For an ideal transformer,
Transformer : input power = output power
 It is a stationary device used to transform electrical power from one circuit to 𝑉𝑃 𝑖𝑃 = 𝑉𝑆 𝑖𝑆
another without changing its frequency. 𝑽𝑺 𝒊𝑷
 It is done with either increasing or decreasing the applied alternationg voltage = − − − − (4)
𝑽𝑷 𝒊𝑺
with corresponding decrease or increase of current in the circuit.  From equation (3) and (4), we have
 If the transformer converts an alternating current with low voltage in to an 𝑽𝑺 𝑵𝑺 𝒊𝑷
alternating current with high voltage, it is called step-up transformer. = = =𝑲 − − − − (5)
𝑽𝑷 𝑵𝑷 𝒊𝑺
 If the transformer converts an alternating current with high voltage in to an (i) If K > 1 (or) 𝑵𝑺 > 𝑵𝑷 , then 𝑽𝑺 > 𝑽𝑷 and 𝒊𝑺 < 𝒊𝑷
alternating current with low voltage, it is called step-down transformer. This is step up transformer in which voltage increased and the corresponding
Principle : current is decreased.
 Mutual induction between two coils. (ii) If K < 1 (or) 𝑵𝑺 < 𝑵𝑷 , then 𝑽𝑺 < 𝑽𝑷 and 𝒊𝑺 > 𝒊𝑷
Construction : This is step down transformer in which voltage decreased and the
 It consists of two coils of high corresponding current is increased.
mutual inductance wound Efficiency of a transformer :
over the same transformer  The efficiency (𝜂) of a transformer is defined as the ratio of the useful output
core made up of silicone power to the input power.
steel. 𝑜𝑢𝑡𝑝𝑢𝑡 𝑝𝑜𝑤𝑒𝑟
 To avoid eddy current loss, 𝜂= 𝑋 100 %
𝑖𝑛𝑝𝑢𝑡 𝑝𝑜𝑤𝑒𝑟
the core is generally
laminated
 The alternating voltage is
applied across primary coil
(P), and the output is taken
across secondary coil (S)
 The assemnbled core and coils are kept in a container which is filled with
suitable medium for better insulation and cooling purpose.
victory R. SARAVANAN. M.Sc., M.Phil., B.Ed PG ASST [PHYSICS], GBHSS, PARANGIPETTAI - 608 502
12 PHYSICS UNIT - 4 ELECTOMAGNETIC INDUCTION AND ALTERNATING CURRENT COMPLETE GUIDE AND MODEL QUESTION
7. Derive an expression for phase angle between the applied voltage and current (iii) When 𝑋𝐿 = 𝑋𝐶 , the phase angle 𝝓 𝒊𝒔 𝒛𝒆𝒓𝒐. It means that 𝒗 inphase with 𝒊
in a series RLC circuit. (𝒊. 𝒆. ) 𝒗 = 𝑽𝒎 𝐬𝐢𝐧 𝝎𝒕 & 𝒊 = 𝑰𝒎 𝐬𝐢𝐧 𝝎𝒕
Series RLC circuit : This circuit is resistive
 Consider a circuit containing 8. What are called LC oscillations? Explain the generation of LC oscilations.
a resistor of resistance ‘R’, a LC oscillations :
inductor of inductance ‘L’ and  Whenever energy is given to a circuit containing a pure inductor of inductance L
a capacitor of capacitance ‘C’ and a capacitor of capacitance C, the energy oscillates back and forth between
connected across an the magnetic field of the inductor and the electric field of the capacitor.
alternating voltage source.  Thus the electrical oscillations of definite frequency are generated. These
 The applied alternating oscillations are called LC oscillations.
voltage is given by, Generation of LC oscillations :
𝑣 = 𝑉𝑚 sin 𝜔𝑡 − − − − − (1)  Whenever energy is given to a circuit containing
 Let ‘𝑖’ be the current in the circuit at that instant. a pure inductor of inductance L and a capacitor
 Hence the voltage developed across R, L and C of capacitance C, the energy oscillates back and
𝑉𝑅 = 𝑖 𝑅 ( 𝑉𝑅 𝑖𝑠 𝑖𝑛 𝑝ℎ𝑎𝑠𝑒 𝑤𝑖𝑡ℎ 𝑖) forth between the magnetic field of the inductor
𝜋
𝑉𝐿 = 𝑖 𝑋𝐿 (𝑉𝐿 𝑙𝑒𝑎𝑑𝑠 𝑖 𝑏𝑦 ) and the electric field of the capacitor.
2
𝜋  Thus the electrical oscillations of definite
𝑉𝐶 = 𝑖 𝑋𝐶 (𝑉𝐶 𝑙𝑎𝑔𝑠 𝑖 𝑏𝑦 )
2 frequency are generated. These oscillations are
 ⃗⃗⃗⃗⃗ , 𝑉𝑅 𝑎𝑙𝑜𝑛𝑔 𝑂𝐴
The phasor diagram is drawn by representing current along 𝑂𝐼 ⃗⃗⃗⃗⃗⃗ , called LC oscillations.
⃗⃗⃗⃗⃗⃗⃗
𝑉𝐿 𝑎𝑙𝑜𝑛𝑔 𝑂𝐵 and 𝑉𝐶 𝑎𝑙𝑜𝑛𝑔 𝑂𝐶 ⃗⃗⃗⃗⃗⃗ Stage -1 :
 If 𝑉𝐿 > 𝑉𝐶 , then the net voltage drop  Consider the capacitor is fully charged with maximum charge 𝑄𝑚 . So that the
across LC combination is (𝑉𝐿 − 𝑉𝐶 ) 𝑄𝑚2
energy stored in the capacitor is maximum (i.e.) 𝑈𝐸 =
which is represented by ⃗⃗⃗⃗⃗⃗⃗
𝐴𝐷 2𝐶
 By parallogram law, the diagonal  As there is no current in the inductor, 𝑈𝐵 = 0
⃗⃗⃗⃗⃗⃗⃗  Therefore the total energy is wholly electrical.
𝑂𝐸 gives the resultant voltage ‘𝑣’
Stage - 2 :
∴ 𝑣 = √𝑉𝑅2 + (𝑉𝐿 − 𝑉𝐶 ) 2  The capacitor now begins to discharge through the inductor that establishes
current ‘𝑖’ clockwise direction.
𝑣 = √𝑖 2 𝑅2 + (𝑖 𝑋𝐿 − 𝑖 𝑋𝐶 ) 2  This current produces a magnetic field around the inductor and energy stored
𝑣 = 𝑖√ 𝑅2 + (𝑋𝐿 − 𝑋𝐶 ) 2 𝐿 𝑖2
𝑣 𝒗 in the inductor which is given by 𝑈𝐵 =
2
(𝑜𝑟) 𝑖 = =
√ 𝑅2 + (𝑋𝐿 − 𝑋𝐶 ) 2 𝒁  As the charge in the capacitor decreases, the energy stored in it also decreases
𝑞2
 Where, 𝒁 = √ 𝑹 + (𝑿𝑳 − 𝑿𝑪 ) is called impedance of the circuit, which
𝟐 𝟐 and is given by 𝑈𝐸 =
2𝐶
refers to the effective opposition to the circuit current by the series RLC circuit.  Thus the total energy is the sum of electrical and magnetic energies.
 From the phasor diagram, the phase angle between ‘𝑣’ and ‘𝑖’ is found out by Stage - 3 :
𝑽𝑳 − 𝑽𝑪 𝑿𝑳 − 𝑿𝑪  When the charge in the capacitor becomes zero, its energy becomes zero (i.e.)
𝐭𝐚𝐧 𝝓 = =
𝑽𝑹 𝑹 𝑈𝐸 = 0
Special cases :  In this stage maximum current (𝐼𝑚 ) flows through inductor and its energy
(i) When 𝑋𝐿 > 𝑋𝐶 , the phase angle 𝝓 𝒊𝒔 𝒑𝒐𝒔𝒊𝒕𝒊𝒗𝒆. It means that 𝒗 leads 𝒊 by 𝜙. 𝐿 𝐼𝑚2

(𝒊. 𝒆. ) 𝒗 = 𝑽𝒎 𝐬𝐢𝐧 𝝎𝒕 & 𝒊 = 𝑰𝒎 𝐬𝐢𝐧(𝝎𝒕 − 𝝓) becomes maximum. (i.e.) 𝑈𝐵 =


2
This circuit is inductive.  Thus the total energy is wholly magnetic.
(ii) When 𝑋𝐿 < 𝑋𝐶 , the phase angle 𝝓 𝒊𝒔 𝒏𝒆𝒈𝒂𝒕𝒊𝒗𝒆. It means that 𝒗 lags behind 𝒊
by 𝜙.
(𝒊. 𝒆. ) 𝒗 = 𝑽𝒎 𝐬𝐢𝐧 𝝎𝒕 & 𝒊 = 𝑰𝒎 𝐬𝐢𝐧(𝝎𝒕 + 𝝓)
This circuit is capacitive
victory R. SARAVANAN. M.Sc., M.Phil., B.Ed PG ASST [PHYSICS], GBHSS, PARANGIPETTAI - 608 502
12 PHYSICS UNIT - 4 ELECTOMAGNETIC INDUCTION AND ALTERNATING CURRENT COMPLETE GUIDE AND MODEL QUESTION
Stage - 4 : Angular frequency of LC oscillations :
 Eventhough the charge in the capacitor is zero, the current will continue to flow  We know that the angular frequency of mechanical oscilations,
in the same direction.
𝒌
 Since the current flow is in decreasing magnitude, the capacitor begins to 𝝎= √
charge in the opposite direction. 𝒎
𝟏
 Thus a part of the energy is transferred from the inductor back to the capacitor.  From the above table, 𝒌 → & 𝒎 →𝑳
𝑪
The total energy is the sum of the electrical and magnetic energies.
 Thus the angular frequency of LC oscillations is,
Stage - 5 : 𝟏
 When the current in the circuit reduces to zero, the capacitor becomes fully 𝝎=
charged in the opposite direction. √𝑳 𝑪
 Thus the energy stored in the capacitor becomes maximum and the energy
stored in the inductor is zero.
 So the total energy is wholly electrical.
Stage - 6 :
 This state of the circuit is similar to the initial state but the difference is that the
capacitor is charged in opposite direction. So it will starts discharge through
inductor in anti-clockwise direction.
 The total energy is the sum of the the electrical and magnetic energies.
Stage - 7 :
 The processes are repeated in opposite direction and finally the circuit returns
to the initial state.
 Thus when the circuit goes through these stages, an alternating current flows in
the circuit.
 As this process is repeated again and again, the electrical oscillations of definte
frequency are generated. These are known as LC oscillations.
9. Compare the electromagnetic oscillations of LC circuit with the mechanical
oscillations of block-spring system to find the expression for angular
frequency of LC oscillatiors mathematically.
Analogies between LC oscillations and simple harmonic oscillations :
Electromagnetic oscilations Mechanical oscilations
This circuit consists inductor and
This circuit consists spring and block
capacitor
Charge ‘q’ Displacement ‘𝑥’
𝑑𝑞 𝑑𝑥
Current 𝑖 = Velocity 𝑣 =
𝑑𝑡 𝑑𝑡
Inductance ‘L’ Mass ‘m’
1
Reciprocal if capacitance Force constant ‘𝑘’
𝐶
1 1 1
Electrical energy = [ ] 𝑞 2
Potential energy = 𝑘𝑥 2
2 𝐶 2
1 1
Magnetic energy = 𝐿𝑖 2 Kinetic energy = 𝑚𝑣 2
2 2
Electro magnetic energy Mechanical energy
1 1 1 1 1
= [ ] 𝑞 2 + 𝐿𝑖 2 = 𝑘𝑥 2 + 𝑚𝑣 2
2 𝐶 2 2 2

victory R. SARAVANAN. M.Sc., M.Phil., B.Ed PG ASST [PHYSICS], GBHSS, PARANGIPETTAI - 608 502
12 PHYSICS UNIT - 4 ELECTOMAGNETIC INDUCTION AND ALTERNATING CURRENT COMPLETE GUIDE AND MODEL QUESTION
4. A closed coil of 40 turns and of area 200 cm 2, is rotated in a magnetic field of
EXAMPLE PROBLEMS WITH SOLUTIONS flux density 2 Wb m–2. It rotates from a position where its plane makes an
1. A circular antenna of area 3 m2 is installed at a place in Madurai. The plane of angle of 30o with the field to a position perpendicular to the field in a time 0.2
the area of antenna is inclined at 47o with the direction of Earth’s magnetic s. Find the magnitude of the emf induced in the coil due to its rotation.
field. If the magnitude of Earth’s field at that place is 4.1 × 10 –5 T find the Solution : 𝐴 = 200 𝑐𝑚2 = 200 𝑋 10−4 𝑚2 ; 𝐵 = 2 𝑇 ; 𝑁 = 40 ; 𝑡 = 0.2 𝑠
magnetic flux linked with the antenna.  Initially, 𝜃 = 90° − 30° = 60 ; Hence initial magnetic flux
Solution : 𝑨 = 3 𝑚2 ; 𝜃 = 90° − 47° = 43 ° ; 𝐵 = 4.1 X 10−5 T Φ𝐵𝑖 = 𝐵 𝐴 cos 𝜃
 Magnetic flux, Φ𝐵 = 𝐵 𝐴 cos 𝜃 Φ𝐵𝑖 = 2 𝑋 200 𝑋 10−4 X cos 60°
Φ𝐵 = 4.1 X 10−5 X 3 X cos 43° 1
Φ𝐵 = 4.1 X 10−5 X 3 X 0.7314 Φ𝐵𝑖 = 400 𝑋 10−4 X = 200 𝑋 10−4
2
Φ𝐵 = 8. 997 X 10−5 = Φ𝐵𝑖 = 2 𝑋 10−2 𝑊𝑏
𝚽𝑩 = 𝟖𝟗. 𝟗𝟕 𝐗 𝟏𝟎−𝟔 𝑾𝒃 = 𝟖𝟗. 𝟗𝟕 𝝁 𝑾𝒃  Finally, 𝜃 = 90° − 90° = 0° ; Hence final magnetic flux
2. A circular loop of area 5 ×10–2 m2 rotates in a uniform Φ𝐵𝑓 = 𝐵 𝐴 cos 𝜃
magnetic field of 0.2 T. If the loop rotates about its Φ𝐵𝑓 = 2 𝑋 200 𝑋 10−4 X cos 0°
diameter which is perpendicular to the magnetic field as Φ𝐵𝑓 = 400 𝑋 10−4 X 1 = 400 𝑋 10−4
shown in figure. Find the magnetic flux linked with the
Φ𝐵𝑓 = 4 𝑋 10−2 𝑊𝑏
loop when its plane is (a) normal to the field (b)
inclined 60o to the field and (c) parallel to the field.  Since the magnetic flux changes, an emf is induced which is given by
Solution : 𝐴 = 5 𝑋 10−2 𝑚2 ; 𝐵 = 0.2 𝑇 𝑑Φ𝐵 Φ𝐵𝑓 − Φ𝐵𝑖
∈= 𝑁 = 𝑁
(a) When circular loop normal to the magnetic field, then 𝜽 = 𝟎°. The magnetic flux 𝑑𝑡 𝑡
Φ𝐵 = 𝐵 𝐴 cos 𝜃 4 𝑋 10−2 − 2 𝑋 10−2 40 𝑋 2 𝑋 10−2
∈ = 40 𝑋 =
Φ𝐵 = 0.2 𝑋 5 𝑋 10−2 X cos 0° 0.2 0.2
Φ𝐵 = 1 𝑋 10−2 X 1 = 𝟏 𝑿 𝟏𝟎−𝟐 𝐖𝐛 ∈ = 400 𝑋 10−2 = 𝟒 𝑽
(b) When circular loop inclined 𝟔𝟎° to the magnetic field, then 𝜽 = 𝟗𝟎° − 𝟔𝟎° = 𝟑𝟎° 5. A straight conducting wire is dropped horizontally from a certain height with
The magnetic flux Φ𝐵 = 𝐵 𝐴 cos 𝜃 its length along east – west direction. Will an emf be induced in it? Justify your
Φ𝐵 = 0.2 𝑋 5 𝑋 10−2 X cos 30° answer.
√3 1.732 Solution :
Φ𝐵 = 1 𝑋 10−2 X = 𝑋 10−2  Yes! An emf will be induced in the wire because it moves perpendicular to the
2 2
𝚽𝑩 = 𝟎. 𝟖𝟔𝟔 𝑿𝟏𝟎−𝟐 = 𝟖. 𝟔𝟔 𝑿 𝟏𝟎−𝟑 𝐖𝐛 horizontal component of Earth’s magnetic field and hence it cuts the magnetic
(c) When circular loop parallel to the magnetic field, then 𝜽 = 𝟗𝟎°. The magnetic lines of Earth's magnetic filed.
flux Φ𝐵 = 𝐵 𝐴 cos 𝜃 6. If the current i flowing in the straight conducting wire as shown in the figure
Φ𝐵 = 0.2 𝑋 5 𝑋 10−2 X cos 90° decreases, find out the direction of induced current in the metallic square
Φ𝐵 = 1 𝑋 10−2 X 0 = 𝟎 loop placed near it.
3. A cylindrical bar magnet is kept along the axis of a circular solenoid. If the Solution :
magnet is rotated about its axis, find out whether an electric current is induced  From right hand rule, the magnetic field by the
in the coil. straight wire is directed into the plane of the square
Solution : loop perpendicularly and its magnetic flux is
 The magnetic field of a cylindrical magnet is decreasing.
symmetrical about its axis. As the magnet is  The decrease in flux is opposed by the current
rotated along the axis of the solenoid, there is induced in the loop by producing a magnetic field in
no induced current in the solenoid because the same direction as the magnetic field of the wire.
the flux linked with the solenoid does not  Again from right hand rule, for this inward magnetic
change due to the rotation of the magnet field, the direction of the induced current in the loop
is clockwise.

victory R. SARAVANAN. M.Sc., M.Phil., B.Ed PG ASST [PHYSICS], GBHSS, PARANGIPETTAI - 608 502
12 PHYSICS UNIT - 4 ELECTOMAGNETIC INDUCTION AND ALTERNATING CURRENT COMPLETE GUIDE AND MODEL QUESTION
7. The magnetic flux passes perpendicular to the plane of the circuit and is 9. A copper rod of length l rotates about one of
directed into the paper. If the magnetic flux varies with respect to time as per its ends with an angular velocity ω in a
the following relation: 𝚽𝑩 = (2t3 + 3t2 + 8t + 5) mWb , what is the magnitude of magnetic field B as shown in the figure. The
the induced emf in the loop when t = 3 s? Find out the direction of current plane of rotation is perpendicular to the field.
through the circuit. Find the emf induced between the two ends of
Solution : Φ𝐵 = (2 𝑡 3 + 3 𝑡 2 + 8 𝑡 + 5) 𝑋 10−3 𝑊𝑏 ; 𝑁 = 1 ; 𝑡 = 3 𝑠 ; ∈ = ? ; 𝑖 = ? the rod.
 From laws of electromagnetic induction, Solution :
𝑑Φ𝐵 𝑑  Consider a small element of length 𝑑𝑥 at a
∈= 𝑁 = (2 𝑡 3 + 3 𝑡 2 + 8 𝑡 + 5) 𝑋 10−3
𝑑𝑡 𝑑𝑡 distance 𝑥 from the centre O of the circle
∈ = (2 𝑋 3 𝑡 2 + 3 𝑋 2 𝑡 + 8 + 0)𝑋 10−3
described by the rod.
∈ = (6 𝑡 2 + 6 𝑡 + 8) 𝑋 10−3
 As this element moves perpendicular to the
 At t = 3 s, the magnitude of induced emf
field with a linear velocity 𝑣 = 𝑥 𝜔, the emf
∈ = [6 (3)2 + 6 (3) + 8]𝑋 10−3
developed in the element 𝑑𝑥 is
∈ = [54 + 18 + 8]𝑋 10−3
∈ = 𝟖𝟎 𝑿 𝟏𝟎−𝟑 𝑽 = 𝟖𝟎 𝒎 𝑽 𝑑 ∈ = 𝐵 𝑑𝑥 𝑣 = 𝐵 𝑑𝑥 (𝑥 𝜔) = 𝐵 𝜔 𝑥 𝑑𝑥
 As time passes, the magnetic flux linked with the  This rod is made up of many such elements, moving perpendicular to the field.
loop increases. The emf developed across two ends is
𝑙 𝑙
 According to Lenz’s law, the direction of the 𝑥2
induced current should be in a way so as to ∈ = 𝐵 𝜔 ∫ 𝑥 𝑑𝑥 = 𝐵 𝜔 [ ]
2 0
oppose the flux increase. 0
𝟏
 So, the induced current flows in such a way to ∈ = 𝑩 𝝎 𝒍𝟐
produce a magnetic field opposite to the given 𝟐
10. A solenoid of 500 turns is wound on an iron core of relative permeability 800.
field. This magnetic field is perpendicularly
outwards. The length and radius of the solenoid are 40 cm and 3 cm respectively.
 Therefore, the induced current flows in anti- Calculate the average emf induced in the solenoid if the current in it changes
clockwise direction. from 0 to 3 A in 0.4 second.
8. A conducting rod of length 0.5 m falls freely from the top of a building of height Solution : : 𝜇𝑟 = 800 ; 𝑁 = 500 ; 𝑙 = 40 𝑐𝑚 = 40 𝑋 10−2 𝑚 ;
7.2 m at a place in Chennai where the horizontal component of Earth’s 𝑟 = 3 𝑐𝑚 = 3 𝑋 10−2 𝑚 ; 𝑑𝑖 = 3 − 0 = 3 𝐴 ; 𝑑𝑡 = 0.4 𝑠 ; ∈ = ?
magnetic field is 4.04 × 10–5 T. If the length of the rod is perpendicular to  Self inductance,
Earth’s horizontal magnetic field, find the emf induced across the conductor 𝜇0 𝜇𝑟 𝑁 2 𝐴 𝜇0 𝜇𝑟 𝑁 2 𝜋 𝑟 2
𝐿= =
when the rod is about to touch the ground. (Assume that the rod falls down 𝑙 𝑙
with constant acceleration of 10 m s–2) 4 𝜋 𝑋 10−7 𝑋 800 𝑋 (500)2 𝑋 3.14 𝑋 (3 𝑋 10−2 )2
𝐿=
Solution : 𝐵𝐻 = 4.04 𝑋 10−5 𝑇 ; ℎ = 7.2 𝑚 ; 𝑙 = 0.5 𝑚 40 𝑋 10−2
 From the equation of motion, the final velocity of the rod is 4 𝑋 3.14 𝑋 10−7 𝑋 800 𝑋 250000 𝑋 3.14 𝑋 9 𝑋 10−4
𝐿=
𝑣 2 = 𝑢2 + 2 𝑔 ℎ [∵ 𝑢 = 0] 40 𝑋 10−2
𝑣 2 = 0 + (2 𝑋 10 𝑋 7.2) 𝐿 = 4 𝑋 3.14 𝑋 20 𝑋 250000 𝑋 3.14 𝑋 9 𝑋 10−9
𝑣 2 = 144 𝐿 = 4 𝑋 3.14 𝑋 2 𝑋 25 𝑋 3.14 𝑋 9 𝑋 10−4
𝒗 = 𝟏𝟐 𝒎 𝒔−𝟏 𝐿 = 3.14 𝑋 3.14 𝑋 1800 𝑋 10−4
𝐿 = 1.775 𝑋 104 𝑋 10−4
 The magnitude of the induced emf when the rod is about to touch the ground is
𝑳 = 𝟏. 𝟕𝟕𝟓 𝑯
∈ = 𝐵𝐻 𝑙 𝑣
∈ = 4.04 𝑋 10−5 𝑋 0.5 𝑋 12  Hence induced emf,
∈ = 24.24 𝑋 10−5 𝑉 𝑑𝑖 3 30
∈=−𝐿 = − 1.775 𝑋 = −1.775 𝑋 = − 1.775 𝑋 7. 5
∈ = 𝟐𝟒𝟐. 𝟒 𝑿 𝟏𝟎−𝟔 𝑽 = 𝟐𝟒𝟐. 𝟒 𝝁 𝑽 𝑑𝑡 0.4 4
∈ = − 𝟏𝟑. 𝟑𝟏𝟐𝟓 𝑽

victory R. SARAVANAN. M.Sc., M.Phil., B.Ed PG ASST [PHYSICS], GBHSS, PARANGIPETTAI - 608 502
12 PHYSICS UNIT - 4 ELECTOMAGNETIC INDUCTION AND ALTERNATING CURRENT COMPLETE GUIDE AND MODEL QUESTION
11. The self-inductance of an air-core solenoid is 4.8 mH. If its core is replaced by 13. Consider two coplanar, co-axial circular coils A
iron core, then its self-inductance becomes 1.8 H. Find out the relative and B as shown in figure. The radius of coil A is
permeability of iron. 20 cm while that of coil B is 2 cm. The
Solution : 𝐿𝑎𝑖𝑟 = 4.8 𝑚𝐻 = 4.8 𝑋 10−3 𝐻 ; 𝐿𝑖𝑟𝑜𝑛 = 1.8 𝐻 ; 𝜇𝑟 = ? number of turns in coils A and B are 200 and
 Self inductance of air core solenoid ; 1000 respectively. Calculate the mutual
𝜇0 𝑁 2 𝐴 inductance between the coils. If the current in
𝐿𝑎𝑖𝑒 =
𝑙 coil A changes from 2 A to 6 A in 0.04 s,
 Self inductance of iron core solenoid ; determine the induced emf in coil B and the
𝜇0 𝜇𝑟 𝑁 2 𝐴 rate of change of flux through the coil B at that
𝐿𝑖𝑟𝑜𝑛 =
𝑙 instant.
 Hence, Solution : 𝑁𝐴 = 200 ; 𝑁𝐵 = 1000 ; 𝑟𝐴 = 20𝑐𝑚 = 20 𝑋 10−2 𝑚 ; 𝑟𝐵 = 2 𝑐𝑚 = 2 𝑋10−2 𝑚
𝜇0 𝑁 2 𝐴 𝑑𝑖𝐴 = 6 − 2 = 4 𝐴 ; 𝑑𝑡 = 0.04 𝑠 ; 𝑀𝐵𝐴 = ? ; ∈𝐵 = ?
𝐿𝑎𝑖𝑟 ( ) 1
𝑙
= 2 =  Mutual inductance between the coils,
𝐿𝑖𝑟𝑜𝑛 𝜇0 𝜇𝑟 𝑁 𝐴 𝜇𝑟 𝜇 𝑁 𝑖
( ) 𝑁𝐵 ( 0 𝐴 𝐴 ) 𝜋 𝑟𝐵2
𝑙 𝑁𝐵 Φ𝐵 𝑁𝐵 B𝐴 A𝐵 2 𝑟𝐴 𝑁𝐵 𝜇0 𝑁𝐴 𝑖𝐴 𝜋 𝑟𝐵2
𝐿𝑖𝑟𝑜𝑛 1.8 3 𝑋 103 3000 𝑀𝐵𝐴 = = = =
∴ 𝜇𝑟 = = = = 𝑖𝐴 𝑖𝐴 𝑖𝐴 2 𝑟𝐴 𝑖𝐴
𝐿𝑎𝑖𝑟 4.8 𝑋 10−3 8 8 𝜇0 𝑁𝐴 𝑁𝐵 𝜋 𝑟𝐵2
𝝁𝒓 = 𝟑𝟕𝟓 (𝒏𝒐 𝒖𝒏𝒊𝒕) 𝑀𝐵𝐴 =
2 𝑟𝐴
12. The current flowing in the first coil changes from 2 A to 10 A in 0.4 s. Find the 4 𝜋 𝑋 10−7 𝑋 200 𝑋 1000 𝑋 𝜋 𝑋 (2 𝑋10−2 )2
mutual inductance between two coils if an emf of 60 mV is induced in the 𝑀𝐵𝐴 =
2 𝑋 20 𝑋 10−2
second coil. Also determine the magnitude of induced emf in the second coil if 𝑀𝐵𝐴 = 8 𝑋 3.14 𝑋 3.14 𝑋 10−5
the current in the first coil is changed from 4 A to 16 A in 0.03 s. Consider only 𝑀𝐵𝐴 = 7. 887 𝑋 101 𝑋 10−5 = 𝟕. 𝟖𝟖𝟕 𝑿 𝟏𝟎−𝟒 𝑯
the magnitude of induced emf.  Magnitude of the induced emf in the coil B,
Solution : 𝑑𝑖𝐴 4
∈𝐵 = 𝑀𝐵𝐴 = 7. 887 𝑋 10−4 𝑋 = 7.887 𝑋 10−4 𝑋 100
(i) 𝑑𝑖1 = 10 − 2 = 8 𝐴 ; 𝑑𝑡 = 0.4 𝑠 ; ∈2 = 60 𝑚𝑉 = 60 𝑋 10−3 𝑉 ; 𝑀 = ? 𝑑𝑡 0.04
 Magnitude of mutual induced emf is ; ∈𝑩 = 7.887 𝑋 10−2 𝑉 = 78.87 𝑋 10−3 𝑉 = 𝟕𝟖. 𝟖𝟕 𝒎𝑽
𝑑𝑖1  The rate of change of magnetic flux of coil B is
∈2 = 𝑀21 𝒅
𝑑𝑡 (𝐍 𝚽 ) = ∈𝑩 = 78.87 𝑋 10−3 𝑉 = 𝟕𝟖. 𝟖𝟕 𝒎 𝑾𝒃 𝒔−𝟏
 Hence mutual inductance between the coils, 𝒅𝒕 𝑩 𝑩
∈2 14. A circular metal of area 0.03 m2 rotates in a uniform magnetic field of 0.4 T.
𝑀21 = The axis of rotation passes through the centre and perpendicular to its plane
𝑑𝑖
( 1) and is also parallel to the field. If the disc completes 20 revolutions in one
𝑑𝑡
60 𝑋 10−3 60 𝑋 10−3 𝑋 0.4 second and the resistance of the disc is 4 Ω, calculate the induced emf between
𝑀21 = = = 60 𝑋 10−3 𝑋 0.05
8 8 the axis and the rim and induced current flowing in the disc.
( )
0.4 Solution : : 𝐵 = 0.4 𝑇 ; 𝐴 = 0.3 𝑚2 ; 𝑓 = 20 𝑟𝑝𝑠 ; 𝑅 = 4  ; ∈ = ? ; 𝑖 = ?
𝑴𝟐𝟏 = 𝟑 𝑿 𝟏𝟎−𝟑 𝑯 = 𝟑 𝒎 𝐻
 Area swept out by the disc in unit time = Area of the disc × frequency
(ii) 𝑑𝑖1 = 16 − 4 = 12 𝐴 ; 𝑑𝑡 = 0.03 𝑠 ; ∈2 = ? 𝑑𝐴
 Magnitude of Induced emf in the second coil due to the rate of change of = 0.3 𝑋 20 = 0.6 𝑚2
current in the first coil is 𝑑𝑡
 Hence induced emf,
𝑑𝑖1
∈2 = 𝑀21 𝑑Φ𝐵 𝑑 (B A) 𝑑𝐴
𝑑𝑡 ∈= = = 𝐵 = 0.4 𝑋 0.6 = 𝟎. 𝟐𝟒 𝑽
12 𝑑𝑡 𝑑𝑡 𝑑𝑡
∈2 = 3 𝑋 10−3 𝑋 = 100 𝑋 10−3 𝑋 12  Thus induced current,
0.03 ∈ 0.24
∈2 = 𝟏. 𝟐 𝑽 𝒊 = = = 𝟎. 𝟎𝟔 𝑨
𝑅 4
victory R. SARAVANAN. M.Sc., M.Phil., B.Ed PG ASST [PHYSICS], GBHSS, PARANGIPETTAI - 608 502
12 PHYSICS UNIT - 4 ELECTOMAGNETIC INDUCTION AND ALTERNATING CURRENT COMPLETE GUIDE AND MODEL QUESTION
15. rectangular coil of area 70 cm2 having 600 turns rotates about an axis 17. An inverter is common electrical device which we use in our homes. When
perpendicular to a magnetic field of 0.4 Wb m–2. If the coil completes 500 there is no power in our house, inverter gives AC power to run a few
revolutions in a minute, calculate the instantaneous emf when the plane of the electronic appliances like fan or light. An inverter has inbuilt step-up
coil is (a) perpendicular to the field (b) parallel to the field and (c) inclined at transformer which converts 12 V AC to 240 V AC. The primary coil has 100
60o with the field. turns and the inverter delivers 50 mA to the external circuit. Find the number
Solution : 𝑁 = 600 ; 𝐴 = 70 𝑐𝑚2 = 70 𝑋 10−4 𝑚2 ; 𝐵 = 0.4 𝑇 ; of turns in the secondary and the primary current.
500 50 Solution : 𝑉𝑃 = 12 𝑉 ; 𝑉𝑆 = 240 𝑉 ; 𝑁𝑃 = 100 𝑉 ; 𝐼𝑆 = 50 𝑚𝐴 = 50 𝑋 10−3 𝐴
𝑓 = 500 𝑟𝑝𝑚 = = = 8.333 𝑟𝑝𝑠
60 6  By transformer equation;.
(a) When perpendicular to the field, 𝜃 = 𝜔 𝑡 = 0° 𝑉𝑆 𝑁𝑆 𝐼𝑃
∈ = ∈𝑚 sin 𝜔 𝑡 = 𝑁 𝐵 𝐴 𝜔 sin 𝜔 𝑡 = 𝑁 𝐵 𝐴 2 𝜋 𝑓 sin 𝜔 𝑡 = = =𝐾
𝑉𝑆 𝑁𝑃 𝐼𝑆
50
∈ = 600 𝑋 0.4 𝑋 70 𝑋 10−4 𝑋 2 𝜋 𝑋 𝑋 sin 0° = 𝟎  Hence the transformation ratio ;
6 𝑉𝑆 240
(b) When parallel to the field, 𝜃 = 𝜔 𝑡 = 90° 𝐾= = = 20
𝑉𝑆 12
∈ = ∈𝑚 sin 𝜔 𝑡 = 𝑁 𝐵 𝐴 𝜔 sin 𝜔 𝑡 = 𝑁 𝐵 𝐴 2 𝜋 𝑓 sin 𝜔 𝑡  Number of turns in secondary coil ; 𝑵𝑺 = 𝑁𝑃 𝐾 = 100 𝑋 20 = 𝟐𝟎𝟎𝟎
50
∈ = 600 𝑋 0.4 𝑋 70 𝑋 10−4 𝑋 2 𝜋 𝑋 𝑋 sin 90°  Primary current ; 𝑰𝑷 = 𝐼𝑆 𝐾 = 50 𝑋 10−3 𝑋 20 = 1000 𝑋 10−3 = 𝟏 𝑨
6
22 18. Write down the equation for a sinusoidal voltage of 50 Hz and its peak value is
∈ = 100 𝑋 0.4 𝑋 70 𝑋 2 𝑋 𝑋 50 𝑋 1 𝑋 10−4 20 V. Draw the corresponding voltage versus time graph.
7
∈ = 88 𝑋 104 𝑋 10−4 = 𝟖𝟖 𝑽 Solution : 𝑓 = 50 𝐻𝑧 ; 𝑉𝑚 = 20 𝑉 ; 𝑉 = ? ; 𝑇 = ?
(c) When inclined at 60o with the field, 𝜃 = 𝜔 𝑡 = 90° − 60° = 30°  Voltage at any instant,
∈ = ∈𝑚 sin 𝜔 𝑡 = 𝑁 𝐵 𝐴 𝜔 sin 𝜔 𝑡 = 𝑁 𝐵 𝐴 2 𝜋 𝑓 sin 𝜔 𝑡 𝑉 = 𝑉𝑚 sin 𝜔 𝑡 = 𝑉𝑚 sin 2 𝜋 𝑓 𝑡
50 𝑉 = 20 sin(2 𝑋 3.14 𝑋 50 𝑋 𝑡)
∈ = 600 𝑋 0.4 𝑋 70 𝑋 10−4 𝑋 2 𝜋 𝑋 𝑋 sin 30° 𝑽 = 𝟐𝟎 𝐬𝐢𝐧 𝟑𝟏𝟒 𝒕
6
22 1  Time for one cycle ,
∈ = 100 𝑋 0.4 𝑋 70 𝑋 2 𝑋 𝑋 50 𝑋 1 𝑋 10−4 𝑋 1 1
7 2
∈ = 44 𝑋 104 𝑋 10−4 = 𝟒𝟒 𝑽 𝑇= = = 0.02 𝑠
𝑓 50
16. An ideal transformer has 460 and 40,000 turns in the primary and secondary 𝑻 = 𝟐𝟎 𝑿 𝟏𝟎−𝟑 𝒔 = 𝟐𝟎 𝒎𝒔
coils respectively. Find the voltage developed per turn of the secondary if the  Corresponding Wave form is shown
transformer is connected to a 230 V AC mains. The secondary is given to a load 19. The equation for an alternating current is given by i = 77 sin 314t. Find the
of resistance 104 Ω. Calculate the power delivered to the load. peak current, frequency, time period and instantaneous value of current at t =
𝑉 2 ms.
Solution : 𝑁𝑃 = 460 ; 𝑁𝑆 = 40000 ; 𝑉𝑃 = 230 𝑉 ; 𝑅𝑆 = 104 Ω ; 𝑆 = ? ; 𝑃 = ?
𝑁𝑆
Solution : : 𝑖 = 77 sin 314 𝑡 ; 𝑡 = 2 𝑚𝑠 = 2 𝑋 10−3 𝑠
 From the transformer equation, voltage per turn of the secondary is ;  General equation for alternating current ; 𝑖 = 𝐼𝑚 sin 𝜔 𝑡 = 𝐼𝑚 sin 2 𝜋 𝑓 𝑡
𝑉𝑆 𝑉𝑃 230 1
= = =  Comparing this equation with given equation, we get
𝑁𝑆 𝑁𝑃 460 2 (a) Peak current ; 𝑰𝒎 = 𝟕𝟕 𝑨
𝑽𝑺 𝝎 314 314 100
∴ = 𝟎. 𝟓 𝑽/𝒕𝒖𝒓𝒏 (b) Frequency ; 𝒇 = 2𝜋 = = 2 𝑋 3.14 = = 𝟓𝟎 𝑯𝒛
𝑵𝑺 2𝜋 2
1 1
 Total secondary voltage ; 𝑽𝑺 = 𝑁𝑆 𝑋 0.5 = 40000 𝑋 0.5 = 𝟐𝟎𝟎𝟎𝟎 𝑽 (c) Time period ; 𝑻= = = 𝟎. 𝟎𝟐 𝒔
𝑓 50
 Power delivered to the load ,
𝑉𝑆 (d) At 𝑡 = 2 𝑚𝑠, instantaneous current
𝑃𝑆 = 𝑉𝑆 𝐼𝑆 = 𝑉𝑆 𝑖 = 77 sin (314 𝑋 2 𝑋 10−3 𝑟𝑎𝑑)
𝑅𝑆 180°
20000 𝑋 20000 𝑖 = 77 sin (314 𝑋 2 𝑋 10−3 𝑋 )
𝑃𝑆 = = 40000 𝑊 3.14
104 𝑖 = 77 𝑋 sin 36°
𝑷𝑺 = 𝟒𝟎 𝒌𝑾
𝒊 = 77 𝑋 0. 5877 = 𝟒𝟓. 𝟐𝟔 𝐀

victory R. SARAVANAN. M.Sc., M.Phil., B.Ed PG ASST [PHYSICS], GBHSS, PARANGIPETTAI - 608 502
12 PHYSICS UNIT - 4 ELECTOMAGNETIC INDUCTION AND ALTERNATING CURRENT COMPLETE GUIDE AND MODEL QUESTION
20. A 400 mH coil of negligible resistance is connected to an AC circuit in which an  Phase angle between voltage and current ;
effective current of 6 mA is flowing. Find out the voltage across the coil if the 𝑋𝐿 − 𝑋𝐶
tan 𝜙 =
frequency is 1000 Hz. 𝑅
Solution : 𝐼𝑒𝑓𝑓 = 6 𝑚𝐴 = 6 𝑋 10−3 𝐴 ; 𝐿 = 400 𝑚𝐻 = 400 𝑋 10−3 𝐻 ; 𝑓 = −1
𝑋𝐿 − 𝑋𝐶 184 − 144
𝜙 = tan [ ] = tan−1 [ ]
1000 𝐻𝑧 𝑅 30
40 4
 Voltage across the coil of inductance 𝐿 𝜙 = tan−1 [ ] = tan−1 [ ]
𝑉𝐿 = 𝐼 𝑋𝐿 = 𝐼 𝜔 𝐿 = 𝐼 (2 𝜋 𝑓) 𝐿 30 3
𝝓 = tan−1 [1. 333] = 𝟓𝟑. 𝟏𝟐°
𝑉𝐿 = 6 𝑋 10−3 𝑋 2 𝑋 3.14 𝑋 1000 𝑋 400 𝑋 10−3
 Since the phase angle is positive, voltage leads current by 53.12° for this
𝑉𝐿 = 150.72 𝑋 10−1
inductive circuit.
𝑽𝑳 = 𝟏𝟓. 𝟎𝟕𝟐 𝑽 𝟖𝟎
𝟏𝟎𝟐 23. A 500 μH inductor, pF capacitor and a 628 Ω resistor are connected to form
21. A capacitor of capacitance μF is connected across a 220 V, 50 Hz A.C. mains. 𝝅𝟐
𝝅 a series RLC circuit. Calculate the resonant frequency and Q-factor of this
Calculate the capacitive reactance, RMS value of current and write down the
circuit at resonance.
equations of voltage and current. 80 80
102 102
Solution : 𝐿 = 500 𝜇𝐻 = 500 𝑋 10−6 𝐻 ; 𝐶 = 2 𝑝𝐹 = 2 𝑋 10−12 𝐹 ; R = 628 Ω
𝜋 𝜋
Solution : 𝑉𝑅𝑀𝑆 = 220 𝑉 ; 𝑓 = 50 𝐻𝑧 ; 𝐶 = 𝜇𝐹 = 𝑋 10−6 𝐹 ; 𝑋𝐶 = ? ; 𝐼𝑅𝑀𝑆 = ?
𝜋 𝜋  Resonance frequency,
1 1
 Capacitive reactance ; 𝑋𝐶 = = 1 1 1
𝜔𝐶 2𝜋𝑓𝐶 𝑓𝑟 = = = −9
1 1 2 𝜋 √𝐿 𝐶 80 10
2𝜋𝑋 𝑋 80
𝑋𝐶 = = 2 𝜋 √500 𝑋 10−6 𝑋 2 𝑋 10−12
𝜋 𝜋 √500
102 10−2
2 𝑋 𝜋 𝑋 50 𝑋 ( ) 𝑋 10−6 109 109 109 109 109 1
𝜋
𝑓𝑟 = = = = = = 𝑋 107
𝑿𝑪 = 𝟏𝟎𝟐 𝛀 = 𝟏𝟎𝟎 𝛀 2√500 𝑋 80 2√40000 2 𝑋 200 400 4 𝑋 10 2 4
𝑉𝑅𝑀𝑆 220 𝑓𝑟 = 0.25 𝑋 107 = 2500 𝑋 103 𝐻𝑧 = 𝟐𝟓𝟎𝟎 𝒌 𝑯𝒛
 RMS value of alternating current ; 𝑰𝑹𝑴𝑺 = = = 𝟐. 𝟐 𝛀
𝑋𝐶 100  Q -factor,
 Equation for alternating voltage ; 𝑉𝐿 𝐼 𝑋𝐿 𝑋𝐿 𝜔𝑟 𝐿 2 𝜋 𝑓𝑟 𝐿
𝑉 = 𝑉𝑚 sin 𝜔𝑡 𝑄= = = = =
𝑉𝑅 𝐼𝑅 𝑅 𝑅 𝑅
𝑉 = 𝑉𝑅𝑀𝑆 √2 sin 2 𝜋 𝑓 𝑡 2 𝑋 3.14 𝑋 2500 𝑋 103 𝑋 500 𝑋 10−6
𝑉 = 220 𝑋 1.414 sin 2 𝑋 3.14 𝑋 50 𝑋 𝑡 𝑄=
𝑽 = 𝟑𝟏𝟏 𝐬𝐢𝐧 𝟑𝟏𝟒 𝒕 628
2 𝑋 2500 𝑋 103 𝑋 500 𝑋 10−6 25000 𝑋 10−3
 Equation for alternating current ; 𝑄= =
𝜋 200 2
𝑖 = 𝐼𝑚 sin (𝜔𝑡 + ) 𝑸 = 𝟏𝟐𝟓𝟎𝟎 𝑿 𝟏𝟎−𝟑 = 𝟏𝟐. 𝟓
2
𝜋 24. Find the instantaneous value of alternating voltage 𝒗 =
𝑖 = 𝐼𝑅𝑀𝑆 √2 sin (2 𝜋 𝑓 𝑡 + )
2 𝟏𝟎 𝐬𝐢𝐧(𝟑𝝅 𝑿 𝟏𝟎𝟒 𝒕) volt at (a) 0 s (b) 50 μs (c) 75 μs.
𝜋
𝑖 = 2.2 𝑋 1.414 sin (2 𝑋 3.14 𝑋 50 𝑋 𝑡 + ) Solution :
2
𝝅  Voltage at any instant ; 𝑣 = 𝑉𝑚 sin 𝜔 𝑡
𝒊 = 𝟑. 𝟏𝟏 𝐬𝐢𝐧 (𝟑𝟏𝟒 𝒕 + )
𝟐  Given voltage equation ; 𝑣 = 10 sin(3𝜋 𝑋 104 𝑡)
22. Find the impedance of a series RLC circuit if the inductive reactance, (a) At 𝑡 = 0 𝑠 ; 𝑣 = 10 sin 0° = 𝟎
capacitive reactance and resistance are 184 Ω, 144 Ω and 30 Ω respectively. (b) At 𝑡 = 50 𝜇𝑠 ,
Also calculate the phase angle between voltage and current. 𝑣 = 10 sin(3𝜋 𝑋 104 𝑋 50 𝑋 10−6 )
Solution : 𝑋𝐿 = 184 Ω ; 𝑋𝐶 = 144 Ω ; R = 30 Ω ; Z = ? ; ϕ = ? 𝑣 = 10 sin(150 𝜋 𝑋 10−2 𝑟𝑎𝑑)
 Impedance ; 𝑍 = √𝑅2 + (𝑋𝐿 − 𝑋𝐶 )2 180°
𝑣 = 10 sin (150 𝜋 𝑋 10−2 𝑋 )
𝑍 = √302 + (184 − 144)2 = √302 + 402 𝜋
𝑣 = 10 sin(1.5 𝑋 180° ) = 10 sin(270° )
𝑍 = √900 + 1600 = √2500 𝑣 = 10 𝑋 (−1) = − 𝟏𝟎 𝑽
𝒁 = 𝟓𝟎 𝛀
victory R. SARAVANAN. M.Sc., M.Phil., B.Ed PG ASST [PHYSICS], GBHSS, PARANGIPETTAI - 608 502
12 PHYSICS UNIT - 4 ELECTOMAGNETIC INDUCTION AND ALTERNATING CURRENT COMPLETE GUIDE AND MODEL QUESTION
4 −6 )
(c) At 𝑡 = 75 𝜇𝑠 ; 𝑣 = 10 sin(3𝜋 𝑋 10 𝑋 75 𝑋 10 (c) New Q- factor,
−2 −2 180°
𝑣 = 10 sin(225 𝜋 𝑋 10 𝑟𝑎𝑑) = 10 sin (225 𝜋 𝑋 10 𝑋 ) 1 2𝐿 1 2 𝑋 80 𝑋 10−6
𝜋
𝑣 = 10 sin(2.25 𝑋 180° ) = 10 sin(405° ) 𝑄𝑛𝑒𝑤 = √ = 𝑋√
𝑅 𝐶𝑛𝑒𝑤 50 1000 𝑋 10−12
𝑣 = 10 sin(360 + 45° ) = 10 sin(45° )
𝑣 = 10 𝑋 (0.7071) = 𝟕. 𝟎𝟕𝟏 𝑽 1 160 𝑋 106 1 16 𝑋 107 1
25. The current in an inductive circuit is given by 0.3 sin (200t – 40°) A. Write the 𝑄𝑛𝑒𝑤 = 𝑋√ = 𝑋√ = 𝑋 √16 𝑋 104
50 1000 50 1 𝑋103 50
equation for the voltage across it if the inductance is 40 mH.
4 𝑋 102 400
Solution : 𝑖 = 0.3 𝑠𝑖𝑛 (200 𝑡 − 40°) 𝐴 ; 𝐿 = 40 𝑚𝐻 = 40 𝑋 10−3 𝐻 ; 𝑉 = ? 𝑄𝑛𝑒𝑤 = = =𝟖
50 50
 In an inductive circuit, the voltage leads the current by 90o. Therefore, 𝟏𝟎 −𝟒 𝟐
𝑣 = 𝑉𝑚 sin(𝜔 𝑡 + 90°) 27. capacitor of capacitance F, an inductor of inductance H and a resistor of
𝝅 𝝅
𝑣 = 𝐼𝑚 𝑋𝐿 sin(𝜔 𝑡 + 90°) resistance 100 Ω are connected to form a series RLC circuit. When an AC
𝑣 = 𝐼𝑚 𝜔 𝐿 sin(𝜔 𝑡 + 90°) supply of 220 V, 50 Hz is applied to the circuit, determine (a) the impedance of
𝑣 = 0.3 𝑋 200 𝑋 40 𝑋 10−3 sin(200 𝑡 − 40° + 90°) the circuit (b) the peak value of current flowing in the circuit (c) the power
𝒗 = 𝟐. 𝟒 𝐬𝐢𝐧(𝟐𝟎𝟎 𝒕 + 𝟓𝟎°) 𝒗𝒐𝒍𝒕 factor of the circuit and (d) the power factor of the circuit at resonance.
26. A series RLC circuit which resonates at 400 kHz has 80 μH inductor, 2000 pF 10−4 2
Solution : 𝐶 = 𝐹 ; 𝐿= 𝐻 ; 𝑅 = 100 Ω ; 𝑉𝑟𝑚𝑠 = 220 𝑉 ; 𝑓 = 50 𝐻𝑧
capacitor and 50 Ω resistor. Calculate (a) Q-factor of the circuit (b) the new 𝜋 𝜋

value of capacitance when the value of inductance is doubled and (c) the new (a) Inductive reactance,
2
Q-factor. 𝑋𝐿 = 𝜔 𝐿 = 2 𝜋 𝑓 𝐿 = 2 𝑋 𝜋 𝑋 50𝑋 = 𝟐𝟎𝟎 𝛀
Solution : 𝑓𝑟 = 400 𝑘𝐻𝑧 = 400 𝑋 103 𝐻𝑧 ; 𝐿 = 80 𝜇𝐻 = 80 𝑋 10−6 𝐻 ; 𝜋
Capacitive reactance,
𝐶 = 2000 𝑝𝐹 = 2000 𝑋 10−12 𝐹 ; 𝑅 = 50 Ω
1 1 1 104
(a) Q-factor of the circuit, 𝑋𝐶 = = = = = 𝟏𝟎𝟎 𝛀
𝜔𝐶 2𝜋𝑓𝐶 10−4 100
1 𝐿 1 80 𝑋 10−6 1 80 𝑋 106 1 8 𝑋 107 2 𝜋 𝑋 50 𝑋
𝜋
𝑄= √ = 𝑋√ −12
= 𝑋√ = 𝑋√ Impedance ; 𝑍 = √𝑅2 + (𝑋𝐿 − 𝑋𝐶 )2
𝑅 𝐶 50 2000 𝑋 10 50 2000 50 2 𝑋103
1 2 𝑋 102 200 𝑍 = √1002 + (200 − 100)2
𝑄= 𝑋 √4 𝑋 104 = = =𝟒
50 50 50 𝑍 = √1002 + 1002
(b) Resonance frequency ; 𝑍 = √2 𝑋 1002 = √2 𝑋 100 = 1.414 𝑋 100
1 1
𝑓𝑟 = (𝑜𝑟) 𝑓𝑟 2 = 𝒁 = 𝟏𝟒𝟏. 𝟒 𝛀
2
4𝜋 𝐿𝐶
2 𝜋 √𝐿 𝐶 (b) Peak value of current,
1
∴ 𝐶= 𝑉𝑚 𝑉𝑟𝑚𝑠 √2
4 𝜋 2 𝐿 𝑓𝑟 2 𝐼𝑚 = =
When inductance 𝐿 is doubled, new capacitance , 𝑍 𝑍
220 𝑋 1.414 220
1 𝐼𝑚 = =
𝐶𝑛𝑒𝑤 = 141.1 100
4 𝜋 (2 𝐿) 𝑓𝑟 2
2
𝑰𝒎 = 𝟐. 𝟐 𝑨
1 (c) Power factor of the circuit,
𝐶𝑛𝑒𝑤 =
4 𝑋 (3.14) 𝑋 2 𝑋 80 𝑋 10−6 𝑋 (400 𝑋 103 )2
2 𝑅 100 1 1
1 𝐜𝐨𝐬 𝝓 = = = = = 𝟎. 𝟕𝟎𝟕
𝐶𝑛𝑒𝑤 = 𝑍 141.4 1.414 √2
4 𝑋 3.14 𝑋 3.14 𝑋 2 𝑋 80 𝑋 160000 (d) Power factor of the circuit at resonance (𝑍 = 𝑅),
1
𝐶𝑛𝑒𝑤 = 𝑅 𝑅
3.14 𝑋 3.14 𝑋 640 𝑋 160000 𝐜𝐨𝐬 𝝓 = = = 𝟏
𝐶𝑛𝑒𝑤 = 9. 9 06 𝑋 10−10 𝑍 𝑅
𝐶𝑛𝑒𝑤 ≅ 10 𝑋 10−10 = 1000 𝑋 10−12 𝐹
𝑪𝒏𝒆𝒘 ≅ 𝟏𝟎𝟎𝟎 𝒑𝑭

victory R. SARAVANAN. M.Sc., M.Phil., B.Ed PG ASST [PHYSICS], GBHSS, PARANGIPETTAI - 608 502
12 PHYSICS UNIT - 4 ELECTOMAGNETIC INDUCTION AND ALTERNATING CURRENT COMPLETE GUIDE AND MODEL QUESTION
5. A rectangular coil of area 6 cm2 having 3500 turns is kept in a uniform
EXERSICE PROBLEMS WITH SOLUTIONS magnetic field of 0.4 T. Initially, the plane of the coil is perpendicular to the
1. A square coil of side 30 cm with 500 turns is kept in a uniform magnetic field field and is then rotated through an angle of 180 o. If the resistance of the coil
of 0.4 T. The plane of the coil is inclined at an angle of 30o to the field. is 35 Ω, find the amount of charge flowing through the coil.
Calculate the magnetic flux through the coil. Solution : 𝐴 = 6 𝑐𝑚2 = 6 𝑋 10−4 𝑚2 ; 𝑁 = 3500 ; 𝐵 = 0.4 𝑇 ; 𝜃𝑖 = 90° − 90° = 0°
Solution : 𝑁 = 500 ; 𝑎 = 30 𝑐𝑚 ; 𝐴 = 𝑎2 = 900 𝑐𝑚2 = 900 𝑋 10−4 𝑚2 ; 𝜃𝑓 = 180° − 90° = 90° ; 𝑅 = 35Ω ; 𝑞 = ?
𝜃 = 90° − 30° = 60 ° ; 𝐵 = 0.4 T ; N Φ𝐵 = ?  Initial magnetic flux ; 𝑁 Φ𝐵 = 𝑁 𝐵 𝐴 cos 𝜃𝑖 = 𝑁 𝐵 𝐴 cos 0° = 𝑁 𝐵 𝐴
 Total Magnetic flux through the coil,  Final magnetic flux ; 𝑁 Φ𝐵 = 𝑁 𝐵 𝐴 cos 𝜃𝑓 = 𝑁 𝐵 𝐴 cos 180° = − 𝑁 𝐵 𝐴
𝑁 Φ𝐵 = 𝑁 𝐵 𝐴 cos 𝜃  Change in magnetic flux ; 𝑑 (𝑁 Φ𝐵 ) = 𝑁 𝐴 𝐵 − (− 𝑁 𝐵 𝐴) = 2 𝑁 𝐵 𝐴
𝑁 Φ𝐵 = 500 X 0.4 X 900 𝑋 10−4 cos 60°  Hence rate of change in magnetic flux (i.e.) induced emf;
1 1 𝑑 (𝑁 Φ𝐵 )
𝑁 Φ𝐵 = 180000 X 10−4 X = 18 X = 𝟗 𝑾𝒃 ∈ = = 2𝑁𝐵𝐴
2 2 𝑑𝑡
2. A straight metal wire crosses a magnetic field of flux 4 mWb in a time 0.4 s. ∈ = 2 𝑋 3500 𝑋 0.4 𝑋 6 𝑋 10−4 = 16800 𝑋 10−4
Find the magnitude of the emf induced in the wire. ∈ = 168 𝑋 10−2 𝑉
Solution : dΦ𝐵 = 4 𝑚𝑊𝑏 = 4 𝑋 10−3 𝑊𝑏 ; 𝑑𝑡 = 0.4 𝑇 ; ∈ = ?  Thus induced current (rate of flow of electric charge).
 Magnitude of the emf induced in the wire, ∈ 168 𝑋 10−2
𝒊 = = = 𝟒. 𝟖 𝑿 𝟏𝟎−𝟐 𝑨
𝑑 Φ𝐵 4 𝑋 10−3 𝑅 35
∈= = = 𝟏𝟎 𝑿𝟏𝟎−𝟑 𝑽 = 𝟏𝟎 𝒎𝑽  So the amount of charge flowing through the coil,
𝑑𝑡 0.4
3. The magnetic flux passing through a coil perpendicular to its plane is a 𝑞 = 𝑖 𝑡 = 4. 8 𝑋 10−2 𝑋 1 = 𝟒. 𝟖 𝑿 𝟏𝟎−𝟐 𝑪
function of time and is given by 𝚽𝑩 = (𝟐 𝒕𝟑 + 𝟒 𝒕𝟐 + 𝟖 𝒕 + 𝟖 ) Wb. If the 6. An induced current of 2.5 mA flows through a single conductor of resistance
resistance of the coil is 5 Ω, determine the induced current through the coil at 100 Ω. Find out the rate at which the magnetic flux is cut by the conductor.
𝑑 Φ𝐵
a time t = 3 second. Solution : 𝑅 = 100  ; 𝑖 = 2.5 𝑚𝐴 = 2.5 𝑋 10−3 𝐴 ; =?
𝑑𝑡
Solution : Magnitude of the induced emf,  The rate of change in magnetic flux (i.e.) induced emf
𝑑 Φ𝐵 𝑑 𝒅 𝚽𝑩
∈= = (2 𝑡 3 + 4 𝑡 2 + 8 𝑡 + 8 ) = ∈ = 𝑖 𝑅 = 2.5 𝑋 10−3 𝑋 100 = 250 𝑋 10−3 = 𝟐𝟓𝟎 𝒎 𝑾𝒃 𝒔−𝟏
𝑑𝑡 𝑑𝑡 𝒅𝒕
∈ = 2 𝑋 3𝑡 2 + 4 𝑋 2 𝑡 + 8 + 0 = 6 𝑡 2 + 8 𝑡 + 8 7. A fan of metal blades of length 0.4 m rotates normal to a magnetic field of
If t = 3 s , ∈ = 6 (3)2 + 8 (3) + 8 = 54 + 24 + 8 = 𝟖𝟔 𝑽 𝟒 𝑿 𝟏𝟎−𝟑 𝑻. If the induced emf between the centre and edge of the blade is
 Then the induced current through the coil, 0.02 V, determine the rate of rotation of the blade.
∈ 86 Solution : 𝑙 = 0.4 m ; A = π 𝑙 2 = 0.16 𝜋 ; B = 4 𝑋 10−3 𝑇 ; ∈𝒎 = 0.02 𝑉 ; 𝜔 =?
𝑖= = = 𝟏𝟕. 𝟐 𝑨  Maximum value of induced emf,
𝑅 5
4. A closely wound circular coil of radius 0.02 m is placed perpendicular to the ∈𝑚 = 𝑁 𝐵 𝐴 𝜔
magnetic field. When the magnetic field is changed from 8000 T to 2000 T in ∈𝑚 0.02
∴ 𝜔= =
6 s, an emf of 44 V is induced in it. Calculate the number of turns in the coil. 𝑁𝐵𝐴 1 𝑋 4 𝑋 10−3 𝑋 0.16 𝜋
0.02
Solution : 𝑟 = 0.02 m ; dt = 6 s ; 𝑑𝐵 = 8000 − 2000 = 6000 𝑇 ; ∈ = 44 𝑉 𝜔=
𝜃 = 90° − 90° = 0° ; 𝑁 = ? 0.64 𝑋 10−3 𝑋 3.14
0.02
 Magnitude of the induced emf ; 𝜔=
0.64 𝑋 10−3 𝑋 3.14
𝑑 Φ𝐵 𝑑 𝑑𝐵 0.02 𝑋 103
∈= 𝑁 =𝑁 (𝐵 𝐴 cos 𝜃) = 𝑁 𝐴 cos 𝜃 ( ) 𝜔=
𝑑𝑡 𝑑𝑡 𝑑𝑡 2.0096
∈ ∈ 44 𝜔 = 9.954 𝑋 10−3 𝑋103
∴ 𝑁= = =
𝑑𝐵 𝑑𝐵 22 6000 𝝎 = 𝟗. 𝟗𝟓𝟒 revolutions/second
𝐴 cos 𝜃 ( ) 𝜋 𝑟 2 cos 𝜃 ( ) 𝑋 (0.02)2 cos 0° ( )
𝑑𝑡 𝑑𝑡 7 6
44 𝑋 7 𝑋 6 84 840
𝑁= = = = 35 𝑡𝑢𝑟𝑛𝑠
22 𝑋 0.0004 𝑋 1 𝑋 6000 2.4 24

victory R. SARAVANAN. M.Sc., M.Phil., B.Ed PG ASST [PHYSICS], GBHSS, PARANGIPETTAI - 608 502
12 PHYSICS UNIT - 4 ELECTOMAGNETIC INDUCTION AND ALTERNATING CURRENT COMPLETE GUIDE AND MODEL QUESTION
−7
8. A bicycle wheel with metal spokes of 1 m long rotates in Earth’s magnetic 4 𝜋 𝑋 10 20000 𝑋 3.14 𝑋 0.022
2

field. The plane of the wheel is perpendicular to the horizontal component of Φ𝐵 = 𝑋1


0.5
Earth’s field of 𝟒 𝑿 𝟏𝟎−𝟓 𝑻.If the emf induced across the spokes is 31.4 mV, −7 8
4 𝑋 3.14 𝑋 10 𝑋 4 𝑋 10 𝑋 3.14 𝑋 0.0004
calculate the rate of revolution of the wheel. Φ𝐵 = 𝑋1
0.5
Solution : 𝑙 = 1 𝑚 ; 𝐵𝐻 = 4 𝑋 10−5 𝑇 ; ∈ = 31.4 𝑚 𝑉 = 31.4 𝑋 10−3 𝑉 ; 𝜔 = ? 64 𝑋 3.14 𝑋 𝑋 3.14 𝑋 10−3
 Maximum value of induced emf, Φ𝐵 =
0.5
∈𝑚 = 𝑁 𝐵 𝐴 𝜔 = 𝑁 𝐵 (𝜋 𝑙 2 )𝜔 Φ𝐵 = 128 𝑋 3.14 𝑋 3.14 𝑋 10−3
∈𝑚 31.4 𝑋 10−3 Φ𝐵 = 1.262 𝑋 103 𝑋 10−3 = 𝟏. 𝟐𝟔𝟐 𝑾𝒃
∴ 𝜔= =  Hence magnetic flux linked with each turn
𝑁 𝐵 𝜋 𝑙2 1 𝑋 4 𝑋 10−5 𝑋 3.14 𝑋12
10 𝑋 102 1000 𝚽𝑩 1. 262
𝜔= = = = 𝟎. 𝟔𝟑𝟏 𝑿 𝟏𝟎−𝟒 𝑾𝒃
4 4 𝑁 20000
𝝎 = 𝟐𝟓𝟎 revolutions/second 12. A coil of 200 turns carries a current of 0.4 A. If the magnetic flux of 4 mWb is
9. Determine the self-inductance of 4000 turn air-core solenoid of length 2m and linked with each turn of the coil, find the inductance of the coil.
diameter 0.04 m. Solution : 𝑁 = 200 ; 𝐼 = 0.4 𝐴 ; Φ𝐵 = 4 𝑚𝑊𝑏 = 4 𝑋 10−3 𝑊𝑏 ; 𝐿 = ?
Solution : : 𝑙 = 2 𝑚 ; 𝑑 = 0.04 𝑚 ; 𝑟 = 0.02 𝑚 ; 𝑁 = 4000 ; 𝐿 = ?  Self inductance of the coil,
 Self inductance of air core solenoid, 𝑁 Φ𝐵 200 𝑋 4 𝑋 10−3
𝐿= = = 2000 𝑋 10−3 = 𝟐 𝑯
𝜇𝑜 𝑁 2 𝐴 𝜇𝑜 𝑁 2 𝜋 𝑟 2 𝐼 0.4
𝐿= == 13. Two air core solenoids have the same length of 80 cm and same cross–
𝑙 𝑙
4 𝜋 𝑋 10−7 𝑋 40002 𝑋 𝜋 𝑋 0.02 𝑋 0.02 sectional area 5 cm2. Find the mutual inductance between them if the number
𝐿 = of turns in the first coil is 1200 turns and that in the second coil is 400 turns.
2
𝐿 = 2 𝑋 3.14 𝑋 3.14 𝑋 10−7 𝑋 16 𝑋106 𝑋 0.02 𝑋 0.02 Solution : 𝑙 = 80 𝑐𝑚 = 80 𝑋 10−2 𝑚 ; 𝐴 = 5 𝑐𝑚2 = 5 𝑋 10−4 𝑚2 ;
𝐿 = 2 𝑋 3.14 𝑋 3.14 𝑋 16 𝑋 4 𝑋 10−5 𝑁1 = 1200 ; 𝑁2 = 400 ; 𝑀 = ?
𝐿 = 128 𝑋 3.14 𝑋 3.14 𝑋 10−5  Mutual inductance between the coils,
𝐿 = 1.262 𝑋 103 𝑋 10−5 = 1.262 𝑋 10−2 𝐻 = 12.62 𝑋 10−3 𝐻 𝜇𝑜 𝑁1 𝑁2 𝐴 4 𝜋 𝑋 10−7 𝑋1200 𝑋 400 𝑋 5 𝑋 10−4
𝑀= =
𝑳 = 𝟏𝟐. 𝟔𝟐 𝒎 𝑯 𝑙 80 𝑋 10−2
10. A coil of 200 turns carries a current of 4 A. If the magnetic flux through the coil 𝑀 = 4 𝜋 𝑋15 𝑋 400 𝑋 5 𝑋 10−9 = 3.14 𝑋 12 𝑋10−5
is 6 × 10–5 Wb, find the magnetic energy stored in the medium surrounding 𝑀 = 37.68 𝑋10−5 𝐻 = 0.3768 𝑋 10−3 𝐻 = 𝟎. 𝟑𝟕𝟔𝟖 𝒎 𝑯
the coil. 14. A long solenoid having 400 turns per cm carries a current 2A. A 100 turn coil
Solution : 𝑁 = 200 ; 𝐼 = 4 𝐴 ; Φ𝐵 = 6 𝑋 10−5 𝑊𝑏 ; 𝑈𝐵 = ? of cross-sectional area 4 cm2 is placed co-axially inside the solenoid so that
 Magnetic energy stored, the coil is in the field produced by the solenoid. Find the emf induced in the
1 1 𝑁 Φ𝐵 2 1 coil if the current through the solenoid reverses its direction in 0.04 sec.
𝑈𝐵 = 𝐿 𝐼 2 = [ ] 𝐼 = 𝑁 Φ𝐵 𝐼 Solution : : 𝑛1 = 400 ; 𝑁1 = 400 𝑋 100 = 4 𝑋 104 ; 𝑁2 = 100 ;
2 2 𝐼 2
1 𝐴 = 4 𝑐𝑚2 = 4 𝑋 10−4 𝑚2 ; 𝐼1 = 2 𝐴 ; 𝑡 = 0.04 𝑠
𝑈𝐵 = 𝑋 200 𝑋 6 𝑋 10−5 𝑋 4  Mutual inductance,
2
𝑈𝐵 = 100 𝑋 6 𝑋 10−5 𝑋 4 = 2400 𝑋10−5 𝐽 𝜇𝑜 𝑁1 𝑁2 𝐴 4 𝜋 𝑋 10−7 𝑋 4 𝑋 104 𝑋 100 𝑋 4 𝑋 10−4
𝑀= =
𝑼𝑩 = 𝟎. 𝟎𝟐𝟒 𝑱 𝑙 1
11. A 50 cm long solenoid has 400 turns per cm. The diameter of the solenoid is 𝑀 = 64 𝜋 𝑋 10−5 = 64 𝑋 3.14 𝑋 10−5
0.04 m. Find the magnetic flux linked with each turn when it carries a current 𝑀 = 200.96 𝑋 10−5 = 2.0096 𝑋 10−3
of 1 A. 𝑴 ≅ 𝟐 𝑿 𝟏𝟎−𝟑 𝑯
Solution : 𝑙 = 50 𝑐𝑚 = 0.5 𝑚 ; 𝑛 = 400 ; 𝑁 = 𝑛 𝑙 = 400 𝑋 50 = 20000 ;  If the current through the solenoid is reversed its direction, the emf inuced in the
𝑑 = 0.04 𝑚 ; 𝑟 = 0.02 𝑚 ; 𝐼 = 1 𝐴 ; Φ𝐵 = ? coil is,
 Let ‘L’ be the self inductance of the solenoid, magnetic flux is, 𝑑𝐼1 2 − (−2) 4
𝜇𝑜 𝑁 2 𝐴 𝜇𝑜 𝑁 2 𝜋 𝑟 2 ∈2 = 𝑀 = 2 𝑋 10−7 𝑋 = 2 𝑋 10−3 𝑋
𝑑𝑡 0.04 0.04
Φ𝐵 = 𝐿 𝐼 = 𝐼= 𝐼 ∈2 = 2 𝑋 10−3 𝑋 100 = 2 𝑋 10−1 𝑉 = 𝟎. 𝟐 𝑽
𝑙 𝑙

victory R. SARAVANAN. M.Sc., M.Phil., B.Ed PG ASST [PHYSICS], GBHSS, PARANGIPETTAI - 608 502
12 PHYSICS UNIT - 4 ELECTOMAGNETIC INDUCTION AND ALTERNATING CURRENT COMPLETE GUIDE AND MODEL QUESTION
15. A 200 turn circular coil of radius 2 cm is placed co-axially within a long 18. A 200V/120V step-down transformer of 90% efficiency is connected to an
solenoid of 3 cm radius. If the turn density of the solenoid is 90 turns per cm, induction stove of resistance 40 Ω. Find the current drawn by the primary of
then calculate mutual inductance of the coil and the solenoid. the transformer.
Solution : 𝑟 = 2 𝑐𝑚 = 2 𝑋 10−2 𝑚 ; 𝑁1 = 200 ; 𝑁2 = 90 𝑋100 = 9000 Solution : 𝜂 = 90 % ; 𝑉𝑃 = 200 𝑉 ; 𝑉𝑆 = 120 𝑉 ; 𝑅 = 40 
 Mutual inductance of the coil and the solenoid, 𝑉𝑆 𝑉𝑆2 1202 120 𝑋 120
 Output power ; 𝑃𝑆 = 𝑉𝑆 𝐼𝑆 = 𝑉𝑆 = = = = 360 𝑊
𝜇𝑜 𝑁1 𝑁2 𝐴 𝜇𝑜 𝑁1 𝑁2 𝜋 𝑟 2 𝑅
𝑃𝑆
𝑅
360
40 40
𝑀= =  Thus secondary current is; 𝐼𝑆 = = = 𝟑𝑨
𝑙 𝑙 𝑉𝑆 120
4 𝜋 𝑋 10−7 𝑋 200 𝑋 9000 𝑋 3.14 𝑋 (2 𝑋 10−2 )2  We know, the efficiency of the transformer ; 𝜂 = 𝑆 = 𝑆 𝑆
𝑃 𝑉 𝐼
𝑀= 𝑃𝑃 𝑉𝑃 𝐼 𝑃
1 90 360
𝑀 = 3.14 𝑋 3.14 𝑋 288 𝑋 10−6 = 2.839 𝑋 103 𝑋 10−6 =
𝑴 = 𝟐. 𝟖𝟑𝟗 𝑿 𝟏𝟎−𝟑 𝑯 = 𝟐. 𝟖𝟑𝟗 𝒎 𝑯 100 200 𝑋 𝐼𝑃
16. The solenoids S1 and S2 are wound on an iron-core of relative permeability  The current drawn by the primary of the transformer
900. Their areas of their cross-section and their lengths are the same and are 360 𝑋 100 36000
𝐼𝑃 = = = 𝟐𝑨
4 cm2 and 0.04 m respectively. If the number of turns in S 1 is 200 and that in S2 200 𝑋 90 18000
is 800, calculate the mutual inductance between the solenoids. If the current 19. The 300 turn primary of a transformer has resistance 0.82 Ω and the
in solenoid 1 is increased form 2A to 8A in 0.04 second, calculate the induced resistance of its secondary of 1200 turns is 6.2 Ω. Find the voltage across the
emf in solenoid 2. primary if the power output from the secondary at 1600V is 32 kW. Calculate
Solution : 𝜇𝑟 = 900 ; 𝐴 = 4 𝑐𝑚2 = 4 𝑋 10−4 𝑚2 ; 𝑙 = 0.04 𝑚 ; 𝑁1 = 200 ; the power losses in both coils when the transformer efficiency is 80%.
𝑁2 = 800 ; 𝑑𝐼1 = 8 − 2 = 6 𝐴 ; 𝑑𝑡 = 0.04 𝑠 ; 𝑀 =? ; ∈2 = ? Solution : 𝑁𝑃 = 300 ; 𝑁𝑆 = 1200 ; 𝑅𝑃 = 0.82  ; 𝑅𝑆 = 6.2  ; 𝑉𝑆 = 1600 𝑉
80
 Mutual inductance, 𝑃𝑆 = 32 𝑘𝑊 = 32 𝑋 103 𝑊 ; 𝜂 = 80 % =
𝜇𝑜 𝜇𝑟 𝑁1 𝑁2 𝐴 4 𝜋 𝑋 10−7 𝑋 900 𝑋 200 𝑋 800 𝑋 4 𝑋 10−4 100
𝑀= = 𝑃𝑆 32 𝑋 103 32000
𝑙 0.04  Output power ; 𝑃𝑆 = 𝑉𝑆 𝐼𝑆 (𝑜𝑟) 𝑰𝑺 = = = = 𝟐𝟎 𝑨
𝑉𝑆 1600 1600
𝑀 = 100 𝜋 𝑋 10−7 𝑋 900 𝑋 200 𝑋 800 𝑋 4 𝑋 10−4 𝑉𝑆 𝑁𝑆 𝑉𝑆 𝑁𝑃 1600 𝑋 300 4800
 Transformer equation ; = (𝑜𝑟) 𝑽𝑷 = = = = 𝟒𝟎𝟎 𝑽
𝑀 = 3.14 𝑋 576 𝑋 10−3 𝑉𝑃 𝑁𝑃 𝑁𝑆 1200 12
𝑃𝑆 𝑃𝑆
𝑴 = 1.808 𝑋 103 𝑋 10−3 ≅ 𝟏. 𝟖𝟏 𝑯  Efficiency ; 𝜂= =
𝑃𝑃 𝑉𝑃 𝐼𝑃
 The induced emf in solenoid 2 is,
80 32 𝑋 103
𝑑𝐼1 6 600 =
∈2 = − 𝑀 = −1.81 𝑋 = −1.81 𝑋 100 400 𝑋 𝐼𝑃
𝑑𝑡 0.04 4
∈2 = −1.81 𝑋 150 = −𝟐𝟕𝟏. 𝟓 𝑽 32 𝑋 103 𝑋 100 3200
𝑰𝑷 = = = 𝟏𝟎𝟎 𝑨
17. A step-down transformer connected to main supply of 220 V is used to operate 400 𝑋 80 32
2 2
11V, 88 W lamp. Calculate (a) Voltage transformation ratio and (b) Current in  Power loss in primary coil = 𝐼𝑃 𝑅𝑃 = 100 𝑋 0.82 = 8200 𝑊 = 𝟖. 𝟐 𝒌𝑾
the primary.  Power loss in secondary coil = 𝐼𝑆2 𝑅𝑆 = 202 𝑋 6.2 = 2480 𝑊 = 𝟐. 𝟒𝟖 𝒌𝑾
Solution : 𝑉𝑃 = 220 𝑉 ; 𝑉𝑆 = 11 𝑉 ; 𝑃𝑆 = 88 𝑊 ; 𝐾 = ? ; 𝐼𝑃 = ? 20. Calculate the instantaneous value at 60o, average value and RMS value of an
𝑉𝑆 𝑁𝑆 𝐼𝑃 alternating current whose peak value is 20 A.
(a) Voltage transformation ratio ; 𝐾 = = =
𝑉𝑃 𝑁𝑃 𝐼𝑆 Solution : 𝐼𝑚 = 20 𝐴 ; 𝜔𝑡 = 60° ; 𝑖 = ? ; 𝐼𝑎𝑣𝑔 = ? ; 𝐼𝑟𝑚𝑠 = ?
𝑉𝑆 11 𝟏  Alternating current at any instant,
∴ 𝑲= = =
𝑉𝑃 220 𝟐𝟎 √3
(b) We know the output power ; 𝑃𝑆 = 88 𝑊 𝑖 = 𝐼𝑚 sin 𝜔𝑡 = 20 sin 60° = 20 𝑋 = 10 𝑋 1.732 = 𝟏𝟕. 𝟑𝟐 𝑨
(𝑖. 𝑒) 2
𝑉𝑆 𝐼𝑆 = 88 𝑊
 Average value of alternating current,
Hence, the secondary current ; 2 𝐼𝑚
88 88 𝐼𝑎𝑣𝑔 = = 0.637 𝐼𝑚 = 0.637 𝑋 20 = 𝟏𝟐. 𝟕𝟒 𝑨
𝐼𝑆 = = =8𝐴 𝜋
𝑉𝑆 11  RMS value of alternating current,
 Thus, current in the primary 𝐼𝑚
1 4 𝐼𝑟𝑚𝑠 = = 0.707 𝐼𝑚 = 0.707 𝑋 20 = 𝟏𝟒. 𝟏𝟒 𝑨
𝐼𝑃 = 𝐾 𝐼𝑆 = 𝑋8= = 𝟎. 𝟒 𝑨 √2
20 10
victory R. SARAVANAN. M.Sc., M.Phil., B.Ed PG ASST [PHYSICS], GBHSS, PARANGIPETTAI - 608 502
12 PHYSICS UNIT - 4 ELECTOMAGNETIC INDUCTION AND ALTERNATING CURRENT COMPLETE GUIDE AND MODEL QUESTION
6. Two co-axial circular coils carrying equal currents, flowing in the same
UNIT – 4 CONCEPTUAL QUESTIONS AND ANSWERS direction approach near each other. What do you expect to happen to the
1. State the condition under which the magnetic flux linked with a circuit is currents in them?
positive, negative and zero.  As the two coils approaching each other, each tends to increase the strength of
 Magnetic flux (Φ𝐵 ) is given by, 𝚽𝑩 = ⃗𝑩 ⃗ . ⃗𝑨
⃗ = 𝑩 𝑨 𝐜𝐨𝐬 𝜽 ; where 𝜃 is the angle magnetic field at the centre of the other. According to Lenz’s law the induced emf
made by the lines of force with normal to the surface tends to oppose this tendency. Therefore it will send a current in opposite
Case (i) : If is 𝜃 acute, (i.e.) magnetic lines of force comes out of the surface, direction, thereby reducing the strength of net current through each coil.
then cos 𝜃 = +𝑣𝑒 and hence 𝚽𝑩 = +𝒗𝒆 7. When a fan is switched off, a spark can be seen in the switch while this is not so
Case (ii) : If is 𝜃 obtuse, (i.e.) magnetic lines of force enters the surface, when the fan was switched on. Explain.
then cos 𝜃 = −𝑣𝑒 and hence 𝚽𝑩 = −𝒗𝒆  At make or break, the current changes through the coil of fan, thus causing an
Case (iii) : If is 𝜃 = 90° (i.e.) magnetic lines of force is parallel to the surface, induced emf to be set up in the coil due to the property of self induction.
then , cos 𝜃 = 0 and hence 𝚽𝑩 = 𝟎  At make, there is no gap any where in the circuit so there is no spark.
2. A straight permanent magnet falls through a fixed metal ring. Will the magnet  At the break, there is a small gap in the cirucuit. Moreover the rate of change of
fall with the acceleration of a freely falling body? magnetic flux and hence the induced emf is very large at break than that at make.
 As the magnet approaches the ring, the strength of magnetic field and hence the This high self induced emf across the gap results in a spark.
magnetic flux linked with the ring changes. This results in induction of an 8. Two coils having co-efficient of self induction 𝑳𝟏 and 𝑳𝟐 are connected in series.
induced emf in the ring which sends a current through the ring in such a Whether the coefficient of self induction of the combination will decrease or
direction that it opposes the motion of magnet. increase?
 Therefore the magnet will fall through the ring with an acceleration less than  In series combination, they carry equal currents but have different potential
that of acceleration due to gravity. diffence across their ends.
3. A copper ring is suspended by means of a thread that its plane is vertical. A bar 𝒅𝒊
 If be the rate of change of current through of this combination, then the self
magnet approaches the centre of ring, horizontally from right side. What will 𝒅𝒕
𝒅𝒊 𝒅𝒊
happen to the ring? induced emf in the two coils will be, 𝑬𝟏 = − 𝑳𝟏 and 𝑬𝟐 = − 𝑳𝟐
𝒅𝒕 𝒅𝒕
 Due to approaching magnet, an emf is induced in the ring which tends to oppose  Hence the net emf induced in this combination ;
the motion of magnet towards the ring. It means the ring tends to exert a force 𝒅𝒊 𝒅𝒊 𝒅𝒊
𝑬 = 𝑬𝟏 + 𝑬𝟐 = − 𝑳𝟏 − 𝑳𝟐 = − (𝑳𝟏 + 𝑳𝟐 )
on magnet from left to right. As a reaction the magnet exerts a force on ring from 𝒅𝒕 𝒅𝒕 𝒅𝒕
right to left. Thus the ring will shift towards left and the string will make some  If ‘L’ is the coefficient of self induction of the combination, then self induced emf
𝒅𝒊
angle with the vertical. is given by ; 𝑬 = − 𝑳 . By comparing above two equations, we get,
𝒅𝒕
4. Obtain the dimensional formula for coefficient of self induction. 𝑳 = 𝑳𝟏 + 𝑳𝟐 (i.e.) Coefficient of self induction of this combination will increase
 Induced emf ‘𝜀’ is given by, 9. Two coils having co-efficient of self induction 𝑳𝟏 and 𝑳𝟐 are connected in
𝑑𝑖 𝜀 𝑑𝑡
𝜀 = −𝐿 (𝑜𝑟) 𝐿 = − parallel. Whether the coefficient of self induction of the combination will
𝑑𝑡 𝑑𝑖 decrease or increase?
[𝑀 𝐿2 𝑇 −3 𝐴−1 ] [𝑇]
⟹ 𝐿= = [𝑴 𝑳𝟐 𝑻−𝟐 𝑨−𝟐 ]  In parallel combination, they carry different currents but have same potential
[𝐴] diffence across their ends.
5. A piece of metal and a piece of stone are dropped from same height. Will they 𝒅𝒊𝟏 𝒅𝒊𝟐
reach the earth in same time?  If and be the rate of change of current through the two coils , then the
𝒅𝒕 𝒅𝒕
𝒅𝒊 𝒅𝒊
 Both the pieces fall through earth’s magnetic field. So eddy currents are setup self induced emf in the two coils will be, 𝑬 = − 𝑳𝟏 𝟏 and 𝑬 = − 𝑳𝟐 𝟐
inside the metal piece while it is not so in the case of stone. 𝒅𝒕 𝒅𝒕
𝒅𝒊 𝑬 𝒅𝒊𝟐 𝑬 𝒅𝒊 𝒅𝒊 𝟏 𝟏
 According to Lenz law, the eddy current always opposes the cause that produce  Therefore, 𝟏 = − and = − ⟹ 𝟏 + 𝟐 = −𝑬 [ + ]
𝒅𝒕 𝑳𝟏 𝒅𝒕 𝑳𝟐 𝒅𝒕 𝒅𝒕 𝑳𝟏 𝑳𝟐
it and this results in retardation of metal piece.  If ′𝑖 ′ be the instantaneous current and ‘L’ is the coefficient of self induction of the
 So that metal piece take longer time to reach the surface of earth 𝒅𝒊 𝒅𝒊 𝑬
combination, then 𝑬 = − 𝑳 (𝒐𝒓) =−
𝒅𝒕 𝒅𝒕 𝑳
𝟏 𝟏 𝟏 𝑳𝟏 𝑳𝟐
 By comparing above two equations, we get, = + (𝒐𝒓) 𝑳 =
𝑳 𝑳𝟏 𝑳𝟐 𝑳𝟏 +𝑳𝟐
(i.e.) Coefficient of self induction of this combination will decrease

victory R. SARAVANAN. M.Sc., M.Phil., B.Ed PG ASST [PHYSICS], GBHSS, PARANGIPETTAI - 608 502
12 PHYSICS UNIT - 4 ELECTOMAGNETIC INDUCTION AND ALTERNATING CURRENT COMPLETE GUIDE AND MODEL QUESTION
10. A wire and a circular loop are placed as shown. The wire and 14. A conducting loop is held stationary, normal to the field between the plates of a
the plane of loop are in the plane of the paper. A current large capacitor. Is a current induced in the loop when it is (i) wholly inside the
starts increasing in the wire, while flowing form P to Q. What capacitor (ii) partially outside the plates of the capacitor?
will be the direction of current, if any induced in the coil?  No current is induced in both cases.
 According Maxwell’s right hand cork screw rule, a current  For the generation on induced current there must be change in magnetic flux and
flowing from P to Q produces a magnetic flux through the not electric flux.
coil directed outwards () 15. A rectangular loop and circular loop are moving out of a uniform magnetic field
 Increase of current results in increase of flux and thereby inducing current in the region to a field free region with a constant velocity. In which loop do you
loop due to electromagnetic induction. expect the induced emf to be constant during the passage out of the field
 According to Lenz’s law, the induced current must flow in such a direction that region? The field is normal to the field.
the net flux through the loop decreases.. This is possible only the induced field is  In the rectangular loop the induced emf will be the constant.
directed in to the plane of the coil (). For this happen the direction of current  In the case of circular loop, the rate of change of area is not constant. Hence the
through should be clockwise. induced emf will not be constant.
11. How can the concept of electromagnetic induction be used to apply breaks to 16. A coil is wound on an iron core and looped back itself, so that the core has two
fast moving automobiles? sets of closely wound wires in series, carrying currents in the opposite senses.
 To retard the motion of a fast moving automobile, a strong magnetic field should What do you expect about its self-inductance? Will it be large or small?
be applied across the wheels. The consequent change in magnetic flux induces  The self inductive will be small, because the effects due to the induced back emf
emf which according to Lenz’s law, opposes the cause (motion), thus retarding will get cancelled, as the currents flow in the opposite senses.
the motion. 17. An iron bar falling vertically through the hollow region of a thick cylindrical
12. Three identical coils A, B and C are placed with shell made of copper, experiences a retarding force. What can you conclude
their planes parallel to one another. Coil A and about the nature of the iron bar?
C carry currents as shown. Coil B and C are fixed  The iron bar must be a magnet.
in position and coil A moves towards B with  When the bar magnet falls vertically through the hollow space of the shell, eddy
uniform motion. Is there any induced current in currents are produced in the shell due to changing magnetic flux.
B? If no give reason. If yes make the direction of  By Lenz’s, the eddy currents oppose the fall of the magnet and hence it
induced current in the diagram. experiences a retarding force.
 As the coils B and C are fixed, there is no relative displacement between them, 18. An electron moves in a circle, with uniform speed, in a stationary magnetic
and hence no current is induced in coil B due to C field, normal to the plane of the circle. If the field magnitude is made to
 Current flowing through A produces magnetic field perpendicular to its plane. increase with time, will the electron speed up or speed down? Will it continue
When coil A moves towards B, the magnetic flux linked with B changes and an to revolve in a same circle?
emf is induced in it.  When ‘B’ increased with time, the speed of the electron will increase, because, =
 According to Lenz’s law, the induced current in B flows such that it opposes the 𝑩𝒆𝒓
.
cause producing it (i.e.) change in magnetic flux. For this happen, the current 𝒎
induced in B will be anticlockwise.  When speed increases, the radius will also change. But by selecting non-uniform
13. When a car is started, the intensity of light of its head light undergoes a magnetic field ‘B’, the electron can be made to revolve in a circle of constant
momentary decrease. Why is it so? radius.
19. A small resistor (say a lamp) is usually put in parallel to the current carrying
 When starter is pushed, motor gets started. In the beginning the speed of
coil of an electromagnet. What purpose does it serve?
armature is small. Therefore there is small back emf. Thus there is a greater fall
of potential across it. So the intensity of light is decreases.  When the current in the electromagnet is switched off, the flux changes from a
large value to zero. This produces a high induced emf.
 When the armature has acquired sufficient speed, there is appreciable back emf
which reduces the fall of potential across it, thereby restoring the brightness of  When the induced current passes across the switch, sparks are produced. To
head lights. avoid this, a small resistance is connected in parallel and this provides a
conducting path for the induced voltage thereby spark is avoided.

victory R. SARAVANAN. M.Sc., M.Phil., B.Ed PG ASST [PHYSICS], GBHSS, PARANGIPETTAI - 608 502
12 PHYSICS UNIT - 4 ELECTOMAGNETIC INDUCTION AND ALTERNATING CURRENT COMPLETE GUIDE AND MODEL QUESTION
20. Two identical loops, one of copper and another of nichrome are rotated with  In case of LC-parallel circuit, voltage across both is same, but current in L lags
the same speed in a magnetic field. In which case will the induced (i) emf voltage by /2 and current in C leads voltage by /2 . So currents in L and C in
(ii) current be greater? Why? paralle has a phase difference of  (or) 180
 Since the rate of change of magnetic flux linked with the two loops are equal, the 28. Power factor often be improved by the use of a capacitor of appropriate
emf induced in both the loops will be the same. But the current will be more in capacitance in the circuit. Explain.
the loop which has small resistance.  We know that, 𝑷𝒐𝒘𝒆𝒓 𝒇𝒂𝒄𝒕𝒐𝒓 = 𝒄𝒐𝒔𝝓 =
𝑹
 Copper has small resistance than nichrome and hence induced current will be 𝒁
more in copper loop.  When capacitance is introduced ‘Z’ is reduced and can be made equal to ‘R’. Then
21. Explain whether an induced current will be developed in a conductor, if it is power factor becomes unity.
moved in a direction parallel to the magnetic field. 29. An applied voltage signal consists of a superposition of a d.c voltage and
 When the conductor is moved parallel to the field, there is no change in the an a.c voltage of high frequency. The circuit consists of an inductor and a
magnetic flux. capacitor in series. Show that the d.c signal will appear across C and the
 The Lorentz force on the free electrons will be zero and hence the potential a.c signal across L
difference across the two ends of the conductor will be zero.  For d.c, the inductive reactance (𝑋𝐿 = 𝜔𝐿 = 2𝜋𝜈𝐿) is zero and the capacitive
1 1
22. Can we use ordinary moving coil galvanometer for measuring a.c? reactance (𝑋𝐶 = = ) is infinity. Because for d.c, frequency ; 𝜈 = 0.
𝜔𝐶 2𝜋𝜈𝐶
 No. Because during the first half of a.c the needle of the galvanometer deflects in So d.c signal appears across ‘C’
one direction and in the second half in the opposite direction. Since the  But for a.c of high frequency, the inductive reactance (𝑋𝐿 = 𝜔𝐿 = 2𝜋𝜈𝐿) is high
frequency of a.c is high the average deflection will be zero. 1 1
and the capacitive reactance (𝑋𝐶 = = ) is small. So a.c signal appears
23. What is choke? 𝜔𝐶 2𝜋𝜈𝐶
 Choke is an inductor of high inductance and negligible resistance. across ‘L’
 It is connected in series in a circuit to reduce current without much loss of power 30. A choke coil in series with a lamp is connected to a d.c line. The lamp is seen to
24. Explain reactance and impedance. shine brightly. Insertion of an iron core in the choke causes no change in the
lamp’s brightness. Predict the corresponding observations if the connection is
 The resistance offered by a pure inductor or pure capacitor to the flow of a.c
through it is called reactance. to an a.c line.
 The combined resistance offered by RL or RC or RLC circuit is called impedance.  ‘L’ has no effect when d.c flows in the circuit.
25. What is the difference between resistance and reactance?  But for a.c when the choke is introduced, it offers an impedance and so the lamp
will shine dimly. With iron core, ‘L’ for choke further increased and hence the
 Both resistance and reactance are the properties of an electrical circuit that
opposes current. lamp will further dim.
31. A lamp is connected in series with a capacitor. Predict your observations for d.c
 But resistance measures the opposition to a flow of current, where as reactance
and a.c connections. What will happen in each case, if the capacity is reduced?
measures the opposition to a change in current. 𝟏 𝟏
 Resistance depends on the dimension of the conductor whereas reactance  For a capacitor; 𝑿𝑪 = = 𝟐𝝅𝝂𝑪 . For d.c, the frequency, 𝜈 = 0 and hence
𝝎𝑪
depends on the frequency of the alternating source. capacitor offers infinite resistance to d.c. So the lamp will not glow in d.c line
26. In any a.c circuit, is the applied instantaneous voltage equal to the algebraic  But for a.c, capacitor offers small resistance and so the lamp will glow. If ‘C’ is
sum of the instantaneous voltage across the series elements of the circuit? Is reduced, 𝑿𝑪 will increase and so the brightness will decrease
the same true for r.m.s voltage? 32. The frequency of a.c passing through a capacitor and a bulb connected in series
 Yes. The applied instantaneous voltage is always equal to the algebraic sum of is increased. What will happen?
the instantaneous voltage across the series elements of the circuit.  The bulb will glow brighter, because due to increase in frequency, the capacitive
 But it is not correct for r.m.s voltage, because there is phase difference between reactance is decreased.
r.m.s voltages across different elements of the circuit. 33. Can we transform d.c voltage using a transformer?
27. Explain why voltages across L and C in series are 180 out of phase, while the  No. Because d.c voltage remains constant always and no emf is induced as there
current flows through L and C in parallel are out of phase by 180 is no change in magnetic flux.
 In case of LC-series circuit, current is same in both, but voltage across L leads 34. What is meant by electrical inertia?
current by /2 and voltage across C lags current by /2 . So voltage in L and C in  The inability of a body to change its state of rest or uniform straight line motion
series has a phase difference of  (or) 180 by itself is called inertia. Like that self inductance of a coil always opposes the
changes in current and hence this property is called electrical inertia.
victory R. SARAVANAN. M.Sc., M.Phil., B.Ed PG ASST [PHYSICS], GBHSS, PARANGIPETTAI - 608 502
12 PHYSICS UNIT - 4 ELECTOMAGNETIC INDUCTION AND ALTERNATING CURRENT COMPLETE GUIDE AND MODEL QUESTION
35. What is the nature of impedance in RLC circuit while at resonance and what is 42. If the frequency of a.c. source is increased , how does its impedance change?
the phase angle between voltage and current when RLC circuit is in resonance?  If the frequency of a.c. source increases gradually, the impedance Z decreases
 At resonance 𝑋𝐿 = 𝑋𝐶 , Hence the nature of impedance is purely resistive (𝑍 = 𝑅) first, becomes minimum while at resonance and starts increasing afterwards.
 Since at resonance the impedance is purely resistive, so the phase angle between 43. Can electrolysis proceed with help of an a.c. source?
voltage and current is zero (i.e.) 𝜙 = 0°  No. The ion inside the electrolyte will keep on vibrating about its mean position
36. What is the impedance of a pure inductor and pure capacitor? Does it depend due to continuous reversal of polarity of a.c. source.
upon the nature of source of a.c. connected across it? 44. An inductor blocks AC but it allows DC. Why? and How?
 The impedance of a pure inductor is called inductive reactance (𝑋𝐿 ) and it is  An inductor L is a closely wound helical coil. The steady DC current flowing
given by ; 𝑋𝐿 = 𝜔 𝐿 = 2 𝜋 𝜈 𝐿 through L produces uniform magnetic field around it and the magnetic flux
 The impedance of a pure capacitor is called capacitive reactance (𝑋𝐶 ) and it is linked remains constant. Therefore there is no self-induction and self-induced
1 1 emf (back emf). Since inductor behaves like a resistor, DC flows through an
given by ; 𝑋𝐶 = =
𝜔𝐶 2𝜋𝜈𝐶
inductor.
 Yes. The impedance depends upon the nature of source because of different
 The AC flowing through L produces time-varying magnetic field which in turn
values of frequency.
induces self-induced emf (back emf). This back emf, according to Lenz’s law,
37. What information can you get from a vector-impedance diagram?
opposes any change in the current. Since AC varies both in magnitude and
 It tells us about the net impedance of the circuit due to all the circuit elements.
direction, its flow is opposed in L. For an ideal inductor of zero ohmic resistance,
 It tells about the phase difference between effective voltage and current the back emf is equal and opposite to the applied emf. Therefore L blocks AC.
 It gives us the value of the power factor of the a.c. circuit. 45. A capacitor blocks DC but it allows AC. Why? and How?
 It gives the information regarding the natrure of circuit (i.e.) whether the circuit  When DC battery is connected, electrons start to flow from the negative terminal
behaves like RL - circuit, pure resistive circuit or RC-circuit. and are accumulated at the right plate of the capacitor, making it negative. Due to
38. Two identical heaters are heated by two sources, one of d.c. and second of a.c., this negative potential, the electrons present in the nearby left plate are repelled
having same potential difference across their terminals. Will the rate of heat and are moved towards positive terminal of the battery. When electrons leave
production in both the heaters be the same? Explain. the left plate, it becomes positively charged. This process is known as charging.
 No. The rate of heat produced in certain time in the heater fed with a.c. source The charging of the plates continues till the level of the battery. Once C is fully
will be greater than the heater fed with d.c. source. charged and current will stop. At this time, we say that capacitor is blocking DC
 Because the net impedance of the circuit, in the case of a.c. source is  Now an AC source is connected across C At an instant, the right side of the source
𝑍 = √𝑅2 + 𝜔 2 𝐿2 . Since 𝑍 > 𝑅, greater heat will produced in the heater fed with is at negative potential, then the electrons flow from negative terminal to the
a.c. source. right plate and from left plate to the positive, but no electron crosses the gap
39. What steps would you take to save the energy consumed in an a.c. circuit between the plates. Thus, the charging of the plates takes place and the plates
without affecting its emf and current? become fully charged. After a short time, the polarities of AC source are reversed
 The energy consumed in a circuit is given by, 𝑊 = 𝐸𝑟𝑚𝑠 𝐼𝑟𝑚𝑠 cos 𝜙 and the right side of the source is now positive. The electrons which were
 Smaller the value of cos 𝜙, smaller is the energy consumed in the circuit. accumulated in the right plate start to flow to the positive terminal and the
 Here cos 𝜙 can be decreased by increasing the value of 𝜙 and 𝜙 can be increased electrons from negative terminal flow to the left plate to neutralize the positive
1
by decreasing capacitive reactance ( ) . Thus if a capacitor of high ‘C’ is charges stored in it. As a result, the net charges present in the plates begin to
𝜔𝐶
decrease and this is called discharging. Once the charges are exhausted, C will be
connected in the circuit, it will result in decrease in actual energy consumed
charged again but with reversed polarities Thus the electrons flow in one
40. What is watt-less current? State the conditions under which it is possible.
direction while charging the capacitor and its direction is reversed while
 The reactive component of 𝐼𝑟𝑚𝑠 (i.e.) 𝐼𝑟𝑚𝑠 sin 𝜙 which is perpendicular 𝑉𝑟𝑚𝑠 is
discharging (the conventional current is also opposite in both cases). Though
called Watt-less current, since the power consumed by this is zero.
electrons flow in the circuit, no electron crosses the gap between the plates. In
 This is possible only in the case of pure inductive or a pure capacitive circuit. this way, AC flows through a capacitor.
41. What is the power factor of the a.c. circuit while at resonance?
 At resonance, the circuit behaves as a pure resistive circuit. Thus 𝜙 = 0°
 Hence, power factor = cos 𝜙 = 1

victory R. SARAVANAN. M.Sc., M.Phil., B.Ed PG ASST [PHYSICS], GBHSS, PARANGIPETTAI - 608 502
12 PHYSICS UNIT - 4 ELECTOMAGNETIC INDUCTION AND ALTERNATING CURRENT COMPLETE GUIDE AND MODEL QUESTION
EXAM NO 7. In a transformer, the number of turns in the primary and the secondary are
NAME : 410 and 1230 respectively. If the current in primary is 6A, then that in the
UNIT - 4 ELECTROMAGNETIC INDUCTION AND ALTERNATING CURRETNT secondary coil is
(a) 2 A (b) 18 A
Time - 2 : 30 hours Total - 60 marks
(c) 12 A (d) 1 A
PART - I 15 X 1 = 15 8. A step-down transformer reduces the supply voltage from 220 V to 11 V and
Note : (i) Answer all the questions increase the current from 6 A to 100 A. Then its efficiency is
(ii) Choose the best answer and write the option code and (a) 1.2 (b) 0.83
corresponding answer (c) 0.12 (d) 0.9
1. An electron moves on a straight line path XY as shown 9. In an electrical circuit, R, L, C and AC voltage source are all connected in series.
in the figure. The coil abcd is adjacent to the path of When L is removed from the circuit, the phase difference between the voltage
𝝅
the electron. What will be the direction of current, if and current in the circuit is . Instead, if C is removed from the circuit, the
𝟑
any, induced in the coil? 𝝅
phase difference is again . The power factor of the circuit is
(a) The current will reverse its direction as the 𝟑
electron goes past the coil 1 1 √3
(a) (b) (c) 1 (d)
(b) No current will be induced (c) abcd (d) adcb 2 √2 2
2. A thin semi-circular conducting ring (PQR) of radius r 10. In a series RL circuit, the resistance and inductive reactance are the same.
is falling with its plane vertical in a horizontal magnetic Then the phase difference between the voltage and current in the circuit is
𝜋 𝜋 𝜋
field B, as shown in the figure. The potential difference (a) (b) (c) (d) zero
4 2 6
developed across the ring when its speed v is 11. In a series resonant RLC circuit, the voltage across 100 Ω resistor is 40 V. The
𝐵 𝑣 𝜋 𝑟2 resonant frequency ω is 250 rad/s. If the value of C is 4 μF, then the voltage
(a) Zero (b) and P is at higher potential
2 across L is
(c) 𝜋 r B v and R is at higher potential (d) 2 r B v and R is at higher potential (a) 600 V (b) 4000 V
3. The flux linked with a coil at any instant t is given by 𝚽𝑩 = 𝟏𝟎 𝒕𝟐 − 𝟓𝟎 𝒕 + 𝟐𝟓𝟎 . (c) 400V (d) 1 V
The induced emf at t = 3 s is 12. An inductor 20 mH, a capacitor 50 μF and a resistor 40 Ω are connected in
(a) −190 V (b) −10 V (c) 10 V (d) 190 V series across a source of emf V = 10 sin 340 t. The power loss in AC circuit is
4. When the current changes from +2A to −2A in 0.05 s, an emf of 8 V is induced (a) 0.76 W (b) 0.89 W
in a coil. The co-efficient of self-induction of the coil is (c) 0.46 W (d) 0.67 W
(a) 0.2 H (b) 0.4 H 13. The instantaneous values of alternating current and voltage in a circuit are
(c) 0.8 H (d) 0.1 H 𝟏 𝟏 𝝅
5. The current i flowing in a coil varies with time as 𝒊= 𝐬𝐢𝐧(𝟏𝟎𝟎 𝝅 𝒕) 𝑨 and 𝑽 = 𝐬𝐢𝐧 (𝟏𝟎𝟎 𝝅 𝒕 + 𝟑 ) 𝑽.The average power
√𝟐 √𝟐
shown in the figure. The variation of induced emf in watts consumed in the circuit is
with time would be 1 √3 1 1
a) (b) (c) (d)
4 4 2 8
14. In an oscillating LC circuit, the maximum charge on the capacitor is Q. The
charge on the capacitor when the energy is stored equally between the electric
and magnetic fields is
𝑄 𝑄
(a) (b)
2 √3
𝑄
(c) (d) Q
(a) (b) (c) (d) √2
𝟐𝟎
6. A circular coil with a cross-sectional area of 4 cm2 has 10 turns. It is placed at 15.
𝝅𝟐
𝑯 inductor is connected to a capacitor of capacitance C. The value of C in
the centre of a long solenoid that has 15 turns/cm and a cross-sectional area order to impart maximum power at 50 Hz is
of 10 cm2. The axis of the coil coincides with the axis of the solenoid. What is (a) 50 μF (b) 0.5 μF
their mutual inductance? (c) 500 μF (d) 5 Μf
(a) 7.54 μH (b) 8.54 μH (c) 9.54 μH (d) 10.54 μH
victory R. SARAVANAN. M.Sc., M.Phil., B.Ed PG ASST [PHYSICS], GBHSS, PARANGIPETTAI - 608 502
12 PHYSICS UNIT - 4 ELECTOMAGNETIC INDUCTION AND ALTERNATING CURRENT COMPLETE GUIDE AND MODEL QUESTION
PART - II 6 X 2 = 12 33. Find out the phase relation ship between voltage and current in a pure inductive
circuit. Define inductive reactance.
Note : (i) Answer any 6 of the following questions .
(OR)
(ii) Question No. 23 is compulsory
16. State Fleming’s right hand rule. Find out the phase relation ship between voltage and current in a pure capacitive
17. Define mutual inductance or coefficient of mutual induction. circuit. Define capacitive reactance
18. Define RMS value of AC.
19. Define power factor.
20. Define wattles current.
21. Define Q – factor.
22. Define resonance.
23. A circular metal of area 0.03 m2 rotates in a uniform magnetic field of 0.4 T. The axis
of rotation passes through the centre and perpendicular to its plane and is also
parallel to the field. If the disc completes 20 revolutions in one second and the
resistance of the disc is 4 Ω, calculate the induced emf between the axis and the rim
and induced current flowing in the disc.
PART - III 6 X 3 = 18
Note : (i) Answer any 6 of the following questions .
(ii) Question No. 30 is compulsory
24. Assuming that the length of the solenoid is large when compared to its diameter, find
the equation for its inductance.
25. How will you induce an emf by changing the area enclosed by the coil.
26. Explain various energy losses in a transformer.
27. Find out the phase relation ship between voltage and current in a pure resistive
circuit.
28. Obtain an expression for average power of AC over a cycle. Discuss its special cases.
29. An inductor of inductance ‘L’ carries an electric current ‘i’. How much energy is
stored while establishing the current in it?
30. A 200V/120V step-down transformer of 90% efficiency is connected to an induction
stove of resistance 40 Ω. Find the current drawn by the primary of the transformer.

PART - IV 3 X 5 = 15
Note : (i) Answer all the questions
31. Show mathematically that the rotation of a coil in a magnetic field over one rotation
induces an alternating emf of one cycle.
(OR)
Explain the principle , construction and working of transformer. இனிமைத் தமிழ் மைொழி எைது - எைக்கின்பந் தரும்படிவொய்த்த நல்அமுது !
32. Derive an expression for phase angle between the applied voltage and current in a கனிமை பிழிந்திட்ட சொறு - எங்கள் கதியில் உைர்ந்திட ைொம்மபற்ற பபறு !
series RLC circuit. தனிமைச் சுமவயுள்ள மசொல்மை - எங்கள் தமிழினும் பவமறங்கும் ைொங்கண்டதில்மை !
நனியுண்டு நனியுண்டு கொதல் - தமிழ் நொட்டினர் ைொவர்க்குபை தமிழ் மீதில் !
(OR)
எங்கள் தமிழ் - பொபவந்தர் பொரதிதொசன்
What are called LC oscillations? Explain the generation of LC oscillations.

victory R. SARAVANAN. M.Sc., M.Phil., B.Ed PG ASST [PHYSICS], GBHSS, PARANGIPETTAI - 608 502
பசித்திரு (Be hungry) தனித்திரு (Be individual) விழித்திரு (Be conscious)

HIGHER SECONDARY SECOND YEAR-PHYSICS

NAME :
STANDARD : 12 SECTION :
SCHOOL :
EXAM NO :

victory R. SARAVANAN. M.Sc, M.Phil, B.Ed.,


PG ASST (PHYSICS)
GBHSS, PARANGIPETTAI - 608 502
12 PHYSICS UNIT - 5 ELECTROMAGNETIC WAVES COMPLETE GUIDE AND MODEL QUESTION

PART – I 1 MARK MULTIPLE CHOICE QUESTIONS & ANSWERS 5. Consider an oscillator which has a charged particle oscillating about its mean
𝟏 position with a frequency of 300 MHz. The wavelength of electromagnetic
1. The dimension of is
𝜺𝒐 𝝁𝒐 waves produced by this oscillator is
(a) [LT−1] (b) [L2 T−2] (a) 1 m (b) 10 m
(c) [L−1 T] (d) [L−2 T2] (c) 100 m (d) 1000 m
-Solution :- -Solution :-
1 1
 Velocity of electromagnetic waves ; 𝑐 = (or) 𝑐 2 =  We know that ; 𝑐 = 𝜆 𝑓 (or) 𝜆 = =
𝑐 3 𝑋 108
= 1𝑚
√𝜀𝑜 𝜇𝑜 𝜀𝑜 𝜇𝑜
𝑓 300 𝑋 106
 Since the dimension of velocity ‘c’ be [ L T-1], then
1
Answer (a) 1m
the dimension of 𝑐 2 (= ) will be [ L2 T-2] 6. The electric and the magnetic fields, associated with an electromagnetic wave,
𝜀𝑜 𝜇𝑜
propagating along negative X axis can be represented by
Answer (b) [ L2 T-2] ⃗ = 𝐵𝑜 𝑘̂
(a) ⃗⃗⃗𝐸 = 𝐸0 𝑖̂ and 𝐵 (b) ⃗⃗⃗𝐸 = 𝐸𝑜 𝑘̂ and ⃗⃗⃗𝐵 = 𝐵𝑜 𝑗̂
2. If the amplitude of the magnetic field is 3 × 10−6 T, then amplitude of the ⃗ ⃗
(c) 𝐸 = 𝐸𝑜 𝑖̂ and 𝐵 = 𝐵𝑜 𝑗̂ (d) ⃗⃗⃗𝐸 = 𝐸𝑜 𝑗̂ and ⃗⃗⃗𝐵 = 𝐵𝑜 𝑖̂
electric field for a electromagnetic waves is -Solution :-
(a) 100 V m−1 (b) 300 V m−1 1
 The Poynting vector for EM-waves ; ⃗⃗𝑆 = (⃗⃗⃗𝐸 𝑋⃗⃗⃗𝐵 )
(c) 600 V m −1 (d) 900 V m−1 𝜇𝑜
-Solution :-  Thus electric vector (⃗⃗⃗𝐸 ), magnetic vector (⃗⃗⃗𝐵 ) and propagation
1
 Velocity of electromagnetic waves ; 𝑐 = = 3 𝑋 108 𝑚𝑠 −1 vector (⃗⃗𝑆) are all mutually perpendicular to each other. (i.e.) If
√𝜀𝑜 𝜇𝑜 electric and magnetic fields are as shown in the figure, then the
𝐸𝑜
 But; 𝑐 =
𝐵0
(or) 𝐸𝑜 = 𝑐 𝐵0 = 3 𝑋 10 𝑋 3 𝑋 10−6 = 9 𝑋 102 = 900 𝑉𝑚−1
8 direction of propagation will be along x- direction.
Answer (b) ̂ and ⃗⃗⃗𝑩 = 𝑩𝒐 𝒋̂
⃗⃗⃗𝑬 = 𝑬𝒐 𝒌
Answer (d) 𝟗𝟎𝟎 𝑽𝒎−𝟏
7. In an electromagnetic wave travelling in free space the rms value of the
3. Which of the following electromagnetic radiations is used for viewing objects
electric field is 3 V m−1. The peak value of the magnetic field is
through fog
(a) 1.414 × 10−8 T (b) 1.0 × 10−8 T
(a) microwave (b) gamma rays
(c) 2.828 × 10 T
−8 (d) 2.0 × 10−8 T
(c) X- rays (d) infrared
-Solution :
-Solution :-
𝐸𝑜 𝐸 √2
 According to Rayleigh scattering law, shorter wavelengths are scattered much  We know that; 𝑐 =
𝐵0
= 𝑅𝑀𝑆 𝐵0
more than the longer wavelengths. 𝐸 √2 3 𝑋 1.414
 Since the wavelength of IR radiaions are greater than visible light, then the (or) 𝐵0 = 𝑅𝑀𝑆𝑐
= 3 𝑋 108 = 1.414 𝑋 10−8 𝑇
scattering of IR- radiations is less than visible light and hence IR radiations are
used for viewing objects through fog.
Answer (a) 𝟏. 𝟒𝟏𝟒 𝑿 𝟏𝟎−𝟖 𝑻
8. An e.m. wave is propagating in a medium with a velocity ⃗⃗⃗𝒗 = 𝒗 𝒊̂ . The
Answer (d) infrared instantaneous oscillating electric field of this e.m. wave is along +y-axis, then
4. Which of the following is false for electromagnetic waves ? the direction of oscillating magnetic field of the e.m. wave will be along:
(a) transverse (b) non-mechanical waves (a) –y direction (b) –x direction
(c) longitudinal (d) produced by accelerating charges (c) +z direction (d) –z direction
-Solution :- -Solution :
 Electromagnetic waves are transverse not longitudinal, because the oscillating  Electric vector (⃗⃗⃗𝐸 ), magnetic vector (⃗⃗⃗𝐵) and propagation
electric field vector, oscillating magnetic field vector and propagation vector are vector (⃗⃗⃗𝑣) are all mutually perpendicular to each other.
all mutually perpendicular direction.  Since velocity is along +x -axis, oscillating electric field is
Answer (c) Longitudinal along +y -axis, then oscillating magnetic field is along +z -axis
Answer (c) +z direction

victory R. SARAVANAN. M.Sc., M.Phil., B.Ed PG ASST [PHYSICS], GBHSS, PARANGIPETTAI - 608 502
12 PHYSICS UNIT - 5 ELECTROMAGNETIC WAVES COMPLETE GUIDE AND MODEL QUESTION
9. If the magnetic monopole exists, then which of the Maxwell’s equation to be 13. If E =Eo sin[106 x - ωt] be the electric field of a plane electromagnetic wave, the
modified?. value of ω is
𝑄
𝐸 . ⃗⃗⃗⃗⃗
(a) ∮ ⃗⃗⃗ 𝑑𝐴 = 𝑒𝑛𝑐𝑙𝑜𝑠𝑒𝑑 ⃗ . ⃗⃗⃗⃗⃗
(b) ∮ 𝐵 𝑑𝐴 = 0 (a) 0.3 × 10−14 rad s−1 (b) 3 × 10−14 rad s−1
𝜀 𝑜
𝑑 𝑑 (c) 0.3 × 1014 rad s−1 (d) 3 × 1014 rad s−1
⃗ . ⃗⃗⃗
(c) ∮ 𝐵 𝑑𝑙 = 𝜇𝑜 𝑖𝐶 + 𝜇𝑜 𝜀𝑜 𝐸 . ⃗⃗⃗⃗⃗
∮ ⃗⃗⃗ 𝑑𝐴 (d) ∮ 𝐸⃗ . ⃗⃗⃗
𝑑𝑙 = − Φ -Solution :-
𝑑𝑡 𝑑𝑡 𝐵
-Solution :-  The equation of a plane electromagnetic wave ; 𝐸 = 𝐸𝑜 sin(𝑘 𝑥 − 𝜔 𝑡)
𝜔
 Here option (a) indicates Gauss law in electrostatics  Thus, 𝑘 = (or) 𝜔 = 𝑘 𝑐 = 106 𝑋 3 𝑋 108 = 3 𝑋 1014 𝑟𝑎𝑑 𝑠 −1
𝑐
 Option (c) indicates Ampere-Maxwell’s law
 Option (d) indicates Faraday’s law Answer (d) 𝟑 𝑿 𝟏𝟎𝟏𝟒 𝒓𝒂𝒅 𝒔−𝟏
 Option (b) indicates Gauss law in magnetism which implies that the lines of 14. Which of the following is NOT true for electromagnetic waves?.
force form a continuous closed path. It means that no isolated magnetic (a) it transports energy
⃗ . ⃗⃗⃗⃗⃗ (b) it transports momentum
monopole exists. Thus if magnetic monopole exists, equation ∮ 𝐵 𝑑𝐴 = 0
(c) it transports angular momentum
should be modified.
(d) in vacuum, it travels with different speeds which depend on their frequency
Answer (b) ⃗⃗ . ⃗⃗⃗⃗⃗
∮𝑩 𝒅𝑨 = 𝟎 -Solution :-
10. Fraunhofer lines are an example of _______________________ spectrum.  Here option (a) (b) and (c) are correct statements. But option (d) is incorrect.
(a) line emission (b) line absorption  Electromagnetic waves travel with speed which is equal to the speed of light in
(c) band emission (d) band absorption 1
vacuum or free space ; 𝑐 = = 3 𝑋 108 𝑚𝑠 −1
-Solution :- √𝜀𝑜 𝜇𝑜
 When light from hot source pass through a cold gas, the atoms in the gas absorbs in vacuum, it travels with
their own characteristic wavelengths from the continuous spectrum gives rise to Answer (d) different speeds which depend
several dark lines. Such spectrum is called line absorbtion spectrum.
on their frequency
 The contiuous spectrum obtained from the Sun consists several dark lines and
15. The electric and magnetic fields of an electromagnetic wave are
these dark lines (absorbed lines) in the solar spectrum are called Fraunhofer
(a) in phase and perpendicular to each other
lines.
(b) out of phase and not perpendicular to each other
Answer (b) line absorption (c) in phase and not perpendicular to each other
11. Which of the following is an electromagnetic wave? (d) out of phase and perpendicular to each other
(a) α - rays (b) β - rays -Solution :-
(c) γ - rays (d) all of them  In electromagnetic waves electric
-Solution :- vector (⃗⃗⃗𝐸 ), magnetic vector (⃗⃗⃗𝐵 )
 α - rays and β - rays are charged rays. So they are not an electromagnetic waves. and propagation vector (⃗⃗𝑆) are all
 But γ - rays have no charge and have velocity equal to that of light, hence it is an mutually perpendicular to each
electromagnetic wave other.
Answer (c) γ - rays  Also both the ⃗⃗⃗𝐸 and ⃗⃗⃗𝐵 attains their
12. Which one of them is used to produce a propagating electromagnetic wave? maxima and minima at the same
(a) an accelerating charge (b) a stationary charge time.(i.e.)they are inphase
(c) a charge moving with constant velocity (d) an uncharged particle in phase and perpendicular to
-Solution :- Answer (a)
each other
 Electromagnetic waves are produced by any accelerated charge.
Answer (a) an accelerating charge

victory R. SARAVANAN. M.Sc., M.Phil., B.Ed PG ASST [PHYSICS], GBHSS, PARANGIPETTAI - 608 502
12 PHYSICS UNIT - 5 ELECTROMAGNETIC WAVES COMPLETE GUIDE AND MODEL QUESTION
8. Define emission spectra.
PART – II 2 MARK SHORT ANSWER QUESTIONS & ANSWERS
 The spectrum obtained from a self luminous source of light is called emission
1. Define displacement current. spectrum.
 The displacement current can be defined as the current which comes into play  Each sourch has its own characteristic emission spectrum.
in the region in which the electric field and the electric flux are changing with 9. Define absorbtion spectra.
time  When light is allowed to pass through an absorbing substance, then the
 That is when ever the change in electric field takes place, displacement current spectrum obtained is known as absorbtion spectrum.
is produced.  It is the characteristic of absorbing substance.
2. Define electro magnetic waves. 10. Define Fraunhofer lines.
 Electromagnetic waves are non-mechanical waves which move with speed  When the spectrum obtained from the Sun is examined, it consists of large
equals to the speed of light (in vacuum) number of dark lines (line absorbtion spectrum).
3. Give the modified form of Ampere’s circuital law.  These dark lines in the solar spectrum are known as Fraunhofer lines.
 If 𝐼𝐶 and 𝐼𝐷 are the conduction and displacement current, then the modified 11. What are the uses of Fraunhofer lines?
Ampere’s circuital law is givent by,  The absorption spectra for various materials are compared with the Fraunhofer
∮ ⃗⃗⃗𝐵 . ⃗⃗⃗⃗
𝑑𝑙 = 𝜇𝑜 (𝐼𝐶 + 𝐼𝐷 ) lines in the solar spectrum, which helps to identifying elements present in the
Sun’s atmosphere.
𝑑 12. Why electro magnetic waves are non mechanical?
(𝑜𝑟) ∮ ⃗⃗⃗𝐵 . ⃗⃗⃗⃗ ∫ ⃗⃗⃗𝐸 . ⃗⃗⃗⃗⃗⃗
𝑑𝑙 = 𝜇𝑜 𝐼𝐶 + 𝜇𝑜 𝜀𝑜 𝑑𝐴
𝑑𝑡  Electromagnetic waves do not require any medium for propagation. So
 This is also known as Ampere - Maxwell’s law. electromagnetic wave is a non-mechanical wave.
4. Write a note on Gauss’s law in magnetism.
 Maxwell’s second equation is called as Gauss’ s law for magnetism. It states that
the surface integral of magnetic field over a closed surface is zero.
⃗⃗⃗⃗⃗ = 𝟎
⃗⃗ . 𝒅𝑨
∮𝑩
 It implies that the magnetic lines of force form a continuous closed path. It
means that no isolated magnetic monopole exists.
5. Write a note on Ampere -Maxwell law.
 It is modified Ampere’s circuital law This law relates the magnetic field around
any closed path toe the conduction current and displacement current through
that path. Mathematically,
∮ ⃗⃗⃗𝐵 . ⃗⃗⃗⃗
𝑑𝑙 = 𝜇𝑜 (𝐼𝐶 + 𝐼𝐷 )
𝑑
(𝑜𝑟) ∮ ⃗⃗⃗𝐵 . ⃗⃗⃗⃗
𝑑𝑙 = 𝜇𝑜 𝐼𝐶 + 𝜇𝑜 𝜀𝑜 ∫ ⃗⃗⃗𝐸 . ⃗⃗⃗⃗⃗
𝑑𝐴
𝑑𝑡
Here, ⃗𝑩
⃗ → magnetic field
 It implies that both conduction and displacement current produces magnetic
field
6. Define electromagnetic spectrum.
 The orderly distribution of electromagnetic waves in terms of wavelength or
frequency is called electromagnetic spectrum.
7. Define dispersion.
 When white light is made to pass through the prism, it is split in to its seven
xonstituent colours. This phenomenon is known as dispersion of light.
 The patern of colours obtanined on the screen after dispersion is called
spectrum.
victory R. SARAVANAN. M.Sc., M.Phil., B.Ed PG ASST [PHYSICS], GBHSS, PARANGIPETTAI - 608 502
12 PHYSICS UNIT - 5 ELECTROMAGNETIC WAVES COMPLETE GUIDE AND MODEL QUESTION
3. Write a note on Radio waves.
PART – III 3 MARK SHORT ANSWER QUESTIONS & ANSWERS Radio waves :
1. Discuss briefly the experiment conducted by Hertz to produce and detect  It is produced by oscillators in electric circuits.
electromagnetic spectrum.  Wavelength range : 1 𝑋 10−4 𝑚 − 1 𝑋 104 𝑚
Hertz experiment :  Frequency range ∶ 3 𝑋 109 𝐻𝑧 − 3 𝑋 104 𝐻𝑧
 The theoritical prediction of existence of electromagnetic wave by Maxwell was  They obey reflection and diffraction
experimentally confirmed by Henrich Hertz.  It is used in,
 His experimental set up consists of two metal electrodes which are made of (i) radio and television communication systems
small spherical metals. (ii) cellular phones to transmit voice communication in the ultra high
 These are connected to larger spheres and the ends of them are connected to frequency band
induction coil which produce very high emf. 4. Write a note on infra microwaves.
 Due to this high voltage, the air between the electrodes gets ionized and spark is Microwaves :
produced.  It is produced by electromagnetic oscillators in electrical circuits
 A receiver (ring electrode) kept at a distance also gets spark which implies that  Wavelength range: 1 𝑋 10−3 𝑚 − 3 𝑋 10−4 𝑚
the energy is transmitted from electrode to the receiver as a wave known as  Frequency range : 3 𝑋 1011 𝐻𝑧 − 1 𝑋 109 𝐻𝑧
electromagnetic waves.  They obey reflection and polarization
 If the receiver is rotated by 90, then no spark is observed by the receiver.  It is used in,
 This confirms that electromagnetic waves are transverse waves as predicted by (i) radar system for aircraft navigation,
Maxwell. (ii) speed of the vehicle,
 Hertz detected radio waves and also computed the speed of radio waves which (iii) microwave oven for cooking
is equal to the speed of light (3 𝑋 108 𝑚 𝑠 −1 ). (iv) very long distance wireless communication through satellites
2. Explain the sources of electromagnetic waves. 5. Write a note on infra red rays.
Sources of electromagnetic waves : Infra red rays :
 Any stationary source charge produces only electric field. When the charge  It is produced from hot bodies and also when the molecules undergo rotational
moves with uniform velocity, it produces steady current which gives rise and vibrational transitions.
magnetic field around the conductor in which charge flows.  Wavelength range : 8 𝑋 10−7 𝑚 − 5 𝑋 10−3 𝑚
 If the charged particle accelerates, in addition to electric field, it also produces  Frequency range : 4 𝑋 1014 𝐻𝑧 − 6 𝑋 1010 𝐻𝑧
magnetic field. Here both electric and magnetic fields are time varying fields.  It provides electrical energy to satellites by means of solar cells
 The linked electric and magnetic fields have wave property which propagate in  It is used in,
the direction perpendicular to the plane containing electric and magnetic field (i) producing dehydrated fruits
vectors. (ii) green housed to keep the plants warm,
 This is known as electromagnetic waves and it is transverse in nature. (iii) heat therapy for muscular pain or sprain
 Any oscillatory motion is also an accelerating motion, so when the charge (iv) TV remote as a signal carrier, to look through haze fof or mist
oscillates about their mean position, it produces electromagnetic waves. (v) Night vision or infrared photography
 Let, electric and magnetic vectors are given by, 6. Write a note visible light.
𝐸𝑦 = 𝐸𝑜 sin(𝑘𝑧 − 𝜔𝑡) Visible light :
𝐵𝑥 = 𝐵𝑜 sin(𝑘𝑧 − 𝜔𝑡)  It is produced by incandescent bodies and also it is radiated by excited atoms in
then the direction of propagation of electromagnetic waves are along Z -axis gases.
 Here the frequency of the electromagnetic wave is equal to the frequency of the  Wavelength range : 4 𝑋 10−7 𝑚 − 7 𝑋 10−7 𝑚
source (oscillation charge)  Frequency range : 7 𝑋 1014 𝐻𝑧 − 4 𝑋 1014 𝐻𝑧
 In free space or vacuum, the ratio between 𝑬𝒐 and 𝑩𝒐 is equal to the speed of  It obeys the laws of reflection, refraction, interference, diffraction, polarization,
electromagnetic wave which is equal to speed of light (c) photo -electric effect and photographic action. It can be used to,
𝑬𝒐 (i) study the structure of molecules
𝒄 =
𝑩𝒐 (ii) arrangement of electrons in eternal shells of atoms and
(iii) sensation of our eyes
victory R. SARAVANAN. M.Sc., M.Phil., B.Ed PG ASST [PHYSICS], GBHSS, PARANGIPETTAI - 608 502
12 PHYSICS UNIT - 5 ELECTROMAGNETIC WAVES COMPLETE GUIDE AND MODEL QUESTION
7. Write a note on ultra violet rays.
Ultra violet rays :
 It is produced by Sun, arc and ionized gases.
 Wavelength range : 6 𝑋 10−10 𝑚 − 4 𝑋 10−7 𝑚
 Frequency range : 5 𝑋 1017 𝐻𝑧 − 7 𝑋 1014 𝐻𝑧
 It has less penetrating power
 It can be absorbed by atmospheric ozone and harmful to human body.
 It is used to,
(i) destroy bacteria
(ii) sterilizing the surgical instruments,
(iii) burglar alarm
(iv) detect the invisible writing, finger prints and
(v) study of molecular structure
8. Write a note on X - rays.
X - rays :
 It is produced when there is a sudden deceleration of high speed electrons at
high atomic number target.
 Also by electronic transitions among the innermost orbits of atoms.
 Wavelength range : 1 𝑋 10−13 𝑚 − 1 𝑋 10−8 𝑚
 Frequency range : 3 𝑋 1021 𝐻𝑧 − 1 𝑋 1016 𝐻𝑧
 It has more penetrating power than UV - rays.
 It is used in,
(i) studying structures of inner atomic electron shell and crystal structures.
(ii) detecting fracture, diseased organs, formation of bones and stones,
observing the progress of healing bones
(iii) detect faults, cracks, flaws and holes in a finished metal product
9. Write a note on gamma rays.
Gamma rays :
 It is produced by transitionsof atomic nuclei and decay of certain elementary
particles.
 Wavelength range : 1 𝑋 10−14 𝑚 − 1 𝑋 10−10 𝑚
 Frequency range : 3 𝑋 1022 𝐻𝑧 − 3 𝑋 1018 𝐻𝑧
 They produce chemical reactions on photographic plates, fluorescence,
ionization, diffraction.
 Its penetrating power is higher than X-rays and UV rays.
 It has no charge but harmful to human body.
 It is used in,
(i) providing information about the structure of atomic nuclei
(ii) radio therapy for the treatment of cancer and tumour
(iii) food industry to kill pathogenic micro organism

victory R. SARAVANAN. M.Sc., M.Phil., B.Ed PG ASST [PHYSICS], GBHSS, PARANGIPETTAI - 608 502
12 PHYSICS UNIT - 5 ELECTROMAGNETIC WAVES COMPLETE GUIDE AND MODEL QUESTION
Equation - 4 :
PART – IV 5 MARK LONG ANSWER QUESTIONS & ANSWERS  It is modified Ampere’s circuital law and also called as Ampere - Maxwell’s
1. Write down Maxwell equations in integral form. law.
Maxwel equations - Integral form :  This law relates the magnetic field around any closed path toe the conduction
 Electrodynamics can be summarized into four basic equations, known as current and displacement current through that path.
Maxwell’s equations.  Mathematically,
 Maxwell’s equations completely explain the behaviour of charges, currents and
∮ ⃗⃗⃗𝐵 . ⃗⃗⃗⃗
𝑑𝑙 = 𝜇𝑜 (𝐼𝐶 + 𝐼𝐷 )
properties of electric and magnetic fields.
 This equation ensures the existence of electromagnetic waves. 𝑑
(𝑜𝑟) ∮ ⃗⃗⃗𝐵 . ⃗⃗⃗⃗
𝑑𝑙 = 𝜇𝑜 𝐼𝐶 + 𝜇𝑜 𝜀𝑜 ∫ ⃗⃗⃗𝐸 . ⃗⃗⃗⃗⃗
𝑑𝐴
Eqution - 1 : 𝑑𝑡
 It is nothing but Gauss’s law Here, ⃗𝑩⃗ → magnetic field
 It relates the net electric flu to net electric charge enclosed in a surface.  It implies that both conduction and displacement current produces magnetic
 Mathematically, Gauss law is expressed as, field
𝑸 2. Explain the modification of Ampere’s circuital law.
∮𝑬 ⃗⃗⃗⃗⃗ = 𝒄𝒍𝒐𝒔𝒆𝒅
⃗ . 𝒅𝑨 − − − − (1)
𝜺𝒐 Maxwell’s corrections to Ampere’s circuital law :
 Here, 𝑬 ⃗ → electric field  According to Faraday’s law of electromagnetic induction, the change in
𝑸𝒄𝒍𝒐𝒔𝒆𝒅 → charge enclosed magnetic field produces an electric field. Mathematically
 This equation is true for both discreate or continuous distribution of charges 𝜕 𝜕
⃗⃗⃗ = −
∮ 𝐸⃗ . 𝑑𝑙 Φ𝐵 = − ∮𝐵 ⃗⃗⃗⃗⃗
⃗ . 𝑑𝐴
 It also indicates that the electric field lines start from positive change and 𝜕𝑡 𝜕𝑡
terminate at negative charge.  It implies that the electric field 𝐸⃗ is induced along a closed loop by the changing
 The electric field lines do not form a continuous closed path (i.e.) isolated magnetic flux Φ𝐵 in the region encircled by the loop.
positive or negative charges can exist.  The converse of this statement, that is change in electric flux produces magnetic
Equation - 2 : field is explained by Maxwell.
 It has no name. But this law os similar to Gauss law in electrostatics. Hence this 𝜕 𝜕
∮𝐵 ⃗⃗⃗ = −
⃗ . 𝑑𝑙 Φ =− ⃗⃗⃗⃗⃗
∮ 𝐸⃗ . 𝑑𝐴
law can be called as Gauss’s law in magnetism. 𝜕𝑡 𝐸 𝜕𝑡
 According to this law, the surface integral of magnetic field over a closed surface  This is known as Maxwell’s law of induction.
is zero.  To understand how the changing electric field produces magnetic field, let us
 Mathematically, this law can be expressed as, consider the situation of charging a parallel plate capacitor.
∮𝑩 ⃗⃗⃗⃗⃗ = 𝟎
⃗⃗ . 𝒅𝑨 − − − − (2)
Here, 𝑩⃗⃗ → magnetic field
 This equation implies that the magnetic field lines form a continuous closed
path. (i.e.) no isolated magnetic monopole exists
Equation - 3 :
 This is Faraday’s laws of electromagnetic induction.
 This law relates electric field with the changing magnetic flux.
 This equation implies that, the line integral of the electric field around any  The electric current passing through the wire is the conduction current ‘𝐼𝐶 ’
closed path is equal to the rate of change of magnetic flux through the closed  This current generates magnetic field around the wire connected across the
path bounded by the surface. capacitor.
 Mathematically it is expressed as,  To calculate the magnetic field at a point ‘P’ near the wire, let us consider an
𝒅𝚽𝑩 amperian loop which encloses the surface𝑆1 . Thus from Ampere circuital law,
∮ ⃗𝑬 . ⃗⃗⃗⃗
𝒅𝒍 = − − − − − (3)
𝒅𝒕 ⃗⃗⃗ = 𝜇𝑜 𝐼𝐶
⃗ . 𝑑𝑙
∮𝐵 − − − − (1)
Here, ⃗𝑬 → electric field
𝑆1

victory R. SARAVANAN. M.Sc., M.Phil., B.Ed PG ASST [PHYSICS], GBHSS, PARANGIPETTAI - 608 502
12 PHYSICS UNIT - 5 ELECTROMAGNETIC WAVES COMPLETE GUIDE AND MODEL QUESTION
 Suppose the same loop is enclosed by balloon shaped surface 𝑆2 , then the  They travel with speed of light in vacuum or free space and it is given by,
boundaries of two surfaces are same but shape of the enclosing surfaces are 𝟏
𝒄= = 𝟑 𝑿 𝟏𝟎𝟖 𝒎 𝒔−𝟏
different. √ 𝜺𝒐 𝝁𝟎
 Ampere’s law does not depend on shape of the enclosing surface and hence the  In a medium with permittivity ‘ 𝜀 ’ and permeability ‘ 𝜇 ’, the speed of
integrals will give the same answer. electromagnetic wave is less than speed in free space or vacuum. (i.e.) 𝒗 < 𝒄
 But there is no current in between the plates of the capacitor, the magnetic field Hence, refractive index of the medium is,
on the surface is zero. So the magnetic field at ‘P’ is zero. Hence 𝒄
𝝁 = = √ 𝜺𝒓 𝝁𝒓
⃗ . ⃗⃗⃗ 𝒗
∮𝐵 𝑑𝑙 = 0 − − − − (2)
 They are not deflected by electric or magnetic field.
𝑆2
 They show interference, diffraction and polarization.
 Here there is an inconsistency between equation (1) and (2). Maxwell resolved  Like other waves, electromagnetic waves also carry energy, linear momentum
this inconsistency as follows. and angular momentum.
 Due to external source, the capacitor gets charged up because of current 4. Explain in detail the emission spectra.
flowing through the capacitor. This produces an increasing electric field Emission spectra :
between the capacitor plates.  The lighe from self luminous source gives emission spectrum.
 This time varying electric field (or flux) existing between the plates of the  Each source has its own characteristic emission spectrum.
capacitor also produces a current known as displacement current.  The emission spectrum can be divided in to three types ;
 From Gauss ‘s law, (i) Continuous emission spectra :
𝑞
⃗⃗⃗⃗⃗ = 𝐸 𝐴 =
Φ𝐸 = ∮ 𝐸⃗ . 𝑑𝐴  Incandescent solids, liquids gives continuous spectra.
𝜀𝑜  It consists of wavelengths containing all the visible colours ranging
 The change in electric flux is, from violet to red.
𝑑Φ𝐵 1 𝑑𝑞 1
= = 𝐼 (e.g.) Spectrum obtained from carbon arc,
𝑑𝑡 𝜀𝑜 𝑑𝑡 𝜀𝑜 𝑑 incandescent filament lamp, etc
𝒅𝚽𝑩 (ii) Line emission spectra :
∴ 𝑰𝒅 = 𝜺𝒐
𝒅𝒕  Light from excited atoms gives line spectrum. They are also known as
𝑑𝑞
Where, = 𝑰𝒅 → Displacement current discontinuous spectra.
𝑑𝑡
 The displacement current can be defined as the current which comes in to play  The line spectr are sharp lines of definite wavelengths or frequencies.
in the region in which the electric field and the electric flux are changing with  It is different for different elements
time. (e.g.) spectra of atomic hydrogen, helium,
 So Maxwell modified Ampere’s law as etc
(iii) Band emission spectra :
⃗⃗ . ⃗⃗⃗⃗
∮𝑩 𝒅𝒍 = 𝝁𝒐 𝑰 = 𝝁𝒐 (𝑰𝑪 + 𝑰𝒅 ) − − − (3)  The light from excited molecules gives band spectrum.
 It consists of several number of very closely spaced spectral lines
 Where, 𝐼 = 𝐼𝐶 + 𝐼𝑑 → total current which overlapped together forming specific coloured bands.
3. Explain the properties of electromagnetic waves.  This spectrum has a sharp edge at one end and fades out at the other
Properties of electromagnetic waves : end.
 Electromagnetic waves are produced by any accelerated charge.  Band spectrum is the characteristic of the molecule.
 They do not require any medium for propagation. So electromagnetic waves are (e.g.) spectra of hydrogen gas, ammonia gas in the discharge tube, etc
non-mechanical wave.
 They are transverse in nature, (i.e) the oscillating electric field vector,
oscillation magnetic field vector and direction of propagation are mutually
perpendicular to each other.

victory R. SARAVANAN. M.Sc., M.Phil., B.Ed PG ASST [PHYSICS], GBHSS, PARANGIPETTAI - 608 502
12 PHYSICS UNIT - 5 ELECTROMAGNETIC WAVES COMPLETE GUIDE AND MODEL QUESTION
5. Explain in detail the absorption spectra.
Absorption spectra :
 When light is allowed to pass through an absorbing substance, then the
spectrum obtained is known as absorption spectrum.
 It is characteristic of the absorbing substance.
 Absorption spectrum is classified into three types;
(i) Continuous absorption spectrum :
 When the light is passed through a medium, it is dispersed by the
prism, we get continuous absorption spectrum.
 For instance, when we pass white light through a blue glass plate, it
absorbs every thing except blue. This is an example for continuous
absorption spectrum.
(ii) Line absorption spectrum :
 When light from incandescent lamp is passed through cold gas, the
spectrum obtained through the dispersion due to the prism is line
absorption spectrum.
 For example, when light from carbon arc is made to pass through
sodium vapour, a continuous spectrum of carbon arc with two dark
lines in the yellow rigion of sodium vapour is obtained.

(iii) Band absorption spectrum :


 When the white light is passed through the iodine vapour, dark bands
on continuous bright background is obtained. This is known as band
absorption spectra.
 It is also obtained when white light is passed through diluted solution
of blood or chlorophyll or through certain solutions of organic and
inorganic compounds.

victory R. SARAVANAN. M.Sc., M.Phil., B.Ed PG ASST [PHYSICS], GBHSS, PARANGIPETTAI - 608 502
12 PHYSICS UNIT - 5 ELECTROMAGNETIC WAVES COMPLETE GUIDE AND MODEL QUESTION
4. A magnetron in a microwave oven emits electromagnetic waves (em waves)
EXAMPLE PROBLEMS WITH SOLUTIONS with frequency f = 2450 MHz. What magnetic field strength is required for
1. Consider a parallel plate capacitor which is connected to an 230 V RMS value electrons to move in circular paths with this frequency?
and 50 Hz frequency. If the separation distance between the plates of the :Solution :- 𝑓 = 2450 𝑀𝐻𝑧 = 2450 𝑋 106 𝐻𝑧 ; 𝑞 = |𝑒| = 1`. 6 𝑋 10−9 𝐶 ;
capacitor and area of the plates are 1 mm and 20 cm 2 respectively. Calculate 𝑚 = 9.11 𝑋 10−31 𝑘𝑔 ; 𝐵 =?
the displacement current at t = 1 s.  Angular frequency of the circular motion,,
:Solution :- 𝑉𝑅𝑀𝑆 = 230 𝑉 ; 𝑑 = 1 𝑚𝑚 = 1 𝑋 10−3 𝑚 ; 𝐴 = 20 𝑐𝑚2 = 20 𝑋 10−4 𝑚2 𝐵𝑞
𝑓 = 50𝐻𝑧 ; 𝑡 = 1 𝑠 ; 𝐼𝑑 = ? 𝜔=
𝑚
 Potential difference between the plates ; 𝐵𝑞
(𝑜𝑟) 2𝜋 𝑓 =
𝑉 = 𝑉𝑚 sin 𝜔𝑡 = 𝑉𝑅𝑀𝑆 √2 sin 2𝜋𝑓𝑡 𝑚
𝑉 = 230 𝑋1.414 𝑋 sin(2𝜋 𝑋 50 𝑡)  Therefore, the magnetic field required is,
𝑉 = 325 sin(100 𝜋 𝑡) 2𝜋𝑓𝑚
𝐵=
 Hence the displacement current, 𝑞
𝑑Φ𝐸 𝑑 2 𝑋 3.14 𝑋 2450 𝑋 106 𝑋 9.11 𝑋 10−31
𝐼𝑑 = 𝜀𝑜 = 𝜀𝑜 (𝐸𝐴) [∵ Φ𝐸 = 𝐸𝐴] 𝐵=
𝑑𝑡 𝑑𝑡 1.6 𝑋 10−19
𝑑 𝑉 𝜀𝑜 𝐴 𝑑𝑉 2 𝑋 3.14 𝑋 2450 𝑋 9.11 𝑋 10−6
𝐼𝑑 = 𝜀𝑜 ( ) 𝐴= [ ] 𝐵=
𝑑𝑡 𝑑 𝑑 𝑑𝑡 1.6
𝜀𝑜 𝐴 𝑑 𝐵 = 8.76 𝑋 104 𝑋 10−6
𝐼𝑑 = [ (325 sin100 𝜋 𝑡)] 𝑩 = 𝟖. 𝟕𝟔 𝑿 𝟏𝟎−𝟐 𝑻 = 𝟎. 𝟎𝟖𝟕𝟔 𝑻
𝑑 𝑑𝑡
𝜀𝑜 𝐴
𝐼𝑑 = [325 𝑋100 𝜋 𝑋 cos100 𝜋 𝑡 ]
𝑑
8.85 𝑋 10−12 𝑋 20 𝑋 10−4
𝐼𝑑 = 𝑋 325 𝑋 100 𝑋 3.14 𝑋 cos100 𝜋(1)
1 𝑋 10−3
𝐼𝑑 = 825 𝑋 20 𝑋 325 𝑋 314 𝑋 10−13 𝑋 (1) [∵ cos100 𝜋 = 1]
𝑰𝒅 = 1. 806 𝑋 107 𝑋 10−13 = 1. 806 𝑋 10−6 𝐴 = 𝟏. 𝟖𝟎𝟔 𝝁 𝑨
2. The relative magnetic permeability of the medium is 2.5 and the relative
electrical permittivity of the medium is 2.25. Compute the refractive index of
the medium.
:Solution :- 𝜇𝑟 = 2.5 ; 𝜀𝑟 = 2.25 ; 𝑛=?
 The refractive index of the medium,
𝑛 = √𝜇𝑟 𝜀𝑟
𝑛 = √2.5 𝑋 2.25 = √5.625
𝒏 = 𝟐. 𝟑𝟕𝟐 (𝒏𝒐 𝒖𝒏𝒊𝒕)
3. Compute the speed of the electromagnetic wave in a medium if the amplitude
of electric and magnetic fields are 3 × 104 N C–1 and 2 × 10–4 T, respectively.
:Solution :- 𝐸𝑜 = 3 𝑋 104 𝑁 𝐶 −1 ; 𝐵𝑜 = 2 𝑋 10−4 𝑇 ; 𝑣 = ?
 The speed of the electromagnetic wave
𝐸𝑜 3 𝑋 104 3
𝑣= = −4
= 𝑋 108
𝐵𝑜 2 𝑋 10 2
𝒗 = 𝟏. 𝟓 𝑿 𝟏𝟎𝟖 𝒎 𝒔−𝟏

victory R. SARAVANAN. M.Sc., M.Phil., B.Ed PG ASST [PHYSICS], GBHSS, PARANGIPETTAI - 608 502
12 PHYSICS UNIT - 5 ELECTROMAGNETIC WAVES COMPLETE GUIDE AND MODEL QUESTION
4. Let an electromagnetic wave propagate along the x - direction, the magnetic
EXERCISE PROBLEMS WITH SOLUTIONS field oscillates at a frequency of 1010 Hz and has an amplitude of 10−5 T, acting
1. Consider a parallel plate capacitor whose plates are closely spaced. Let R be the along the y - direction. Then, compute the wavelength of the wave. Also write
radius of the plates and the current in the wire connected to the plates is 5 A, down the expression for electric field in this case.
calculate the displacement current through the surface passing between the :Solution :- 𝒇 = 1010 𝐻𝑧 ; 𝐵𝑜 = 10−5 𝑇 ; 𝜆 = ? ; ⃗⃗⃗𝐸 (𝑥, 𝑡) = ?
plates by directly calculating the rate of change of flux of electric field through  Wavelength of electromagnetic waves,
the surface. 𝐶 3 𝑋 108
:Solution :- 𝐼𝐶 = 5 𝐴 ; 𝐼𝑑 =? 𝝀= = = 𝟑 𝑿 𝟏𝟎−𝟐 𝒎
𝑓 1010
 Electric flux,  Expression for electric field,
Φ𝐸 = ∮ 𝐸 𝑑𝐴 cos 0° 𝐸 (𝑥, 𝑡) = 𝐸𝑜 sin(𝑘𝑥 − 𝜔𝑡) − − − − − − − − (1)
𝜎 𝑞  We know that, the velocity of electromagnetic waves,
Φ𝐸 = 𝐸 𝐴 = 𝐴= 𝐸𝑜
𝜀𝑜 𝜀𝑜 𝐶=
 Hence displacement current, 𝐵𝑜
𝑑Φ𝐸 𝑑 𝑞
(𝑜𝑟) 𝑬𝒐 = 𝐵𝑜 𝐶 = 10−5 𝑋 3 𝑋 108 = 𝟑 𝑿 𝟏𝟎𝟑 𝑵 𝑪−𝟏
𝐼𝑑 = 𝜀𝑜 = 𝜀𝑜 ( )  Angular frequency,
𝑑𝑡 𝑑𝑡 𝜀𝑜
1 𝑑𝑞 𝝎 = 2 𝜋 𝑓 = 2 𝑋 3.14 𝑋 1010 = 𝟔. 𝟐𝟖 𝑿 𝟏𝟎𝟏𝟎 𝒓𝒂𝒅 𝒔−𝟏
𝐼𝑑 = 𝜀𝑜  And wave number,
𝜀𝑜 𝑑𝑡 𝜔 6.28 𝑋 1010
𝑑𝑞 𝒌= = = 𝟐. 𝟎𝟗𝟑 𝑿 𝟏𝟎𝟐 𝒓𝒂𝒅 𝒎−𝟏
𝐼𝑑 = 𝐶 3 𝑋 108
𝑑𝑡  Put this in equation (1),
𝑰𝒅 = 𝑰𝑪 = 𝟓 𝑨
2. A transmitter consists of LC circuit with an inductance of 1 μH and a 𝑬 (𝒙, 𝒕) = 𝟑 𝑿 𝟏𝟎𝟑 𝐬𝐢𝐧(𝟐. 𝟎𝟗𝟑 𝑿 𝟏𝟎𝟐 𝒙 − 𝟔. 𝟐𝟖 𝑿 𝟏𝟎𝟏𝟎 𝒕) 𝑵 𝑪−𝟏
capacitance of 1 μF. What is the wavelength of the electromagnetic waves it  Since the electric field oscillates along z -axis,
emits? ⃗⃗⃗𝑬 (𝒙, 𝒕) = 𝟑 𝑿 𝟏𝟎𝟑 𝐬𝐢𝐧(𝟐. 𝟎𝟗𝟑 𝑿 𝟏𝟎𝟐 𝒙 − 𝟔. 𝟐𝟖 𝑿 𝟏𝟎𝟏𝟎 𝒕) 𝒌 ̂ 𝑵 𝑪−𝟏
:Solution :- 𝐿 = 1 𝜇 𝐻 = 1 𝑋 10−6 𝐻 ; 𝐶 = 1 𝜇 𝐹 = 1 𝑋 10−6 𝐹 ; 𝜆 = ? 5. If the relative permeability and relative permittivity of a medium are 1.0 and
 Wavelength of electromagnetic waves, 2.25 respectively, find the speed of the electromagnetic wave in this medium.
𝐶 𝐶 :Solution :- : 𝜇𝑟 = 1.0 ; 𝜀𝑟 = 2.25 ; 𝑣 = ?
𝜆= = = 𝐶 (2 𝜋 √𝐿 𝐶)  Refractive index of the medium,
𝑓 1
( )
2 𝜋 √𝐿 𝐶 𝜇 = √𝜇𝑟 𝜀𝑟
𝜆 = 3 𝑋 108 (2 𝑋 3.14 𝑋 √1 𝑋 10−6 𝑋 1 𝑋 10−6 ) 𝐶
(𝑜𝑟) = √𝜇𝑟 𝜀𝑟
𝜆 = 3 𝑋 108 𝑋 2 𝑋 3.14 𝑋 1 𝑋 10−6 𝑣
𝐶 3 𝑋 108
𝝀 = 𝟏𝟖. 𝟖𝟒 𝑿 𝟏𝟎𝟐 𝒎 = 𝟏𝟖𝟖𝟒 𝒎 ∴ 𝑣= =
3. A pulse of light of duration 10−6 s is absorbed completely by a small object √𝜇𝑟 𝜀𝑟 √1.0 𝑋 2.25
initially at rest. If the power of the pulse is 60 × 10 −3 W, calculate the final 3 𝑋 108
𝑣=
momentum of the object. 1.5
:Solution :- 𝑡 = 10−6 𝑠 ; 𝑃 = 60 𝑋 10−3 𝑊 ; 𝑝 = ? 𝒗 = 𝟐 𝑿 𝟏𝟎𝟖 𝒎 𝒔−𝟏
 Final momentum,
𝐸
𝑝= [ 𝐸 = 𝑚 𝐶 2 = 𝑝 𝐶]
𝐶
𝑃𝑋𝑡 𝑊 𝐸
𝑝= [𝑃 = = ]
𝐶 𝑡 𝑡
−3 −6
60 𝑋 10 𝑋 10
𝑝=
3 𝑋 108
𝒑 = 𝟐𝟎 𝑿 𝟏𝟎−𝟏𝟕 𝒌𝒈 𝒎 𝒔−𝟏

victory R. SARAVANAN. M.Sc., M.Phil., B.Ed PG ASST [PHYSICS], GBHSS, PARANGIPETTAI - 608 502
12 PHYSICS UNIT - 5 ELECTROMAGNETIC WAVES COMPLETE GUIDE AND MODEL QUESTION
11. How would a blue object appear under sodium lamp light?
CONCEPTUAL QUESTIONS AND ANSWERS  The blue object will look black.
1. Explaing how the displacement current maintains the continuity of current in a  We know white light consists all colours from violet to red in visible region.
circuit containing a capacitor? When white light falls on a blue object, it absorbs all colours except blue and
 The current into the capacitor to change the electric field between the plates is reflects blue.
equal to the displacement current between the plates.  But sodium vapour lamp is a mono chromatic source which emits only yellow
2. When you stand outdoors in the Sunlight, why can you feel the energy that the light. When sodium lamp yellow light falls on the blue object, it absorbs yellow
Sunlight carries, but not the momentum it carrirs? and no colour to reflect. So it looks black.
 The amount of energy carried by Sun light is about 𝟏𝟎𝟎 𝑾𝒎−𝟐 which can quickly 12. Do materials always have the same colour whether viewed by reflected light or
produce a considerable change in temperature. through transmitted light?
 But the light pressure of Sun’s light is about 𝟑 𝑿 𝟏𝟎−𝟕 𝑵 𝒎−𝟐 . This value is too  It is not necessary.
small to notice.  Usually substances have same colour in reflected and transmitted light. But some
3. What is the physical significance of the Poynting vector? lubricating oils appers to be green in reflected light, but red in transmitted light.
𝟏
 Poynting vector is given by, ⃗𝑺 = (𝑬 ⃗ 𝑿 ⃗𝑩
⃗ ) = 𝒄𝟐 𝜺𝒐 (𝑬
⃗ 𝑿 ⃗𝑩
⃗) 13. Why it is dangerous to view Solar eclipse with naked eye? (or)
𝝁𝒐
Why does a welder wearing a mask against eye during welding?
 Poynting vector at any point gives the direction of energy flow and the amount of
 Sun is the powerful natural source of UV rays. When we view Sun directly with
energy per unit area transported per second.
nacked eye, the UV rays from Sun damage the retina of our eye.
4. (i) If the electric field and magnetic field in a plane electromagnetic wave were
 Simillarly, UV light produced by the welding arc is dangerous to the eye and is
interchanged, in which direction relative to before would the energy
filtered out by the mask.
propagate? (ii) What if the electric and the magnetic fields were both changed
14. Eye is most sensitive to yellow, but danger signals are red. Why?
to their negatives?
(i) According to right hand rule, the direction of energy propagation would reverse.  According to Raleigh scattering law, the intensity of scattered light is inversely
⃗ ), and therefore the propagation direction proportional to the fourth power of wavelength. Scattering of red is less than
(ii)This would leave the Poynting vector (𝑺
yellow and it can penetrate through fog without much loss of instensity. So red is
is the same.
preferred in danger signals.
5. Long distance radio broadcasts use short wave bands. Why?
15. What are the factors upon which Maxwell’s displacement current depends?
 Because only short waves can be reflected by the atmosphere.
 It depends upon the rate of change of electric flux in that region.
6. It is necessary to use satellites for long distance TV transmission. Why?
16. A constant current flows through a wire of resistance ‘R’. What is the
 TV signals cannot be transmitted by reflecting them from the ionosphere. If we displacement current in it?
use a satellite, wider area can be covered.
 The displacement current is zero. The total charge in any volume of wire is
7. The ozone layer on top of the atmosphere is crusial for human survival. Why?
always constant. Therefore electric flux is also constant.
 Ultra-violet rays (UV) are harmful to living bodies and are absorbed by the ozone
 Since displacement current is proportional to rate of change of electric flux,
layer.
therefore it is zero.
8. If the earth did not have an atmosphere, would its average surface temperature
17. Why is electro magnetic wave so named?
be higher or lower than what it is now?
 Since the electromagnetic wave is composed of varying electric and magnetic
 Lower. Because there will be no greenhouse effect and all the infra-red
vectors at right angles to each other, it is called electro - mangnetic wave.
radiations (IR) will be radiated out at night
18. What is the evidence in support of the theory that light is electro-magnetic in
9. People usally prefer light coloured dresses during summer and dark coloured
nature?
dresses during winter. Why? 𝟏
 Light coloured dresses reflects most of the light falling on it. But a black body  According to theory, the velocity of electromagnetic wave is ; 𝒗 =
√𝝁𝒐 𝜺𝒐
absorbs all the colours and converts light into heat.  By substituting 𝝁𝒐 = 𝟒 𝝅 𝑿 𝟏𝟎−𝟕 𝑯/𝒎 and 𝜺𝒐 = 𝟖. 𝟖𝟓𝟒 𝑿 𝟏𝟎−𝟏𝟐 𝑪𝟐 𝑵−𝟏 𝒎−𝟐
 So by wearing dark dresses, body will remain warm in winter. we get 𝒗 = 𝟑 𝑿 𝟏𝟎𝟖 𝒎/𝒔 which is evidently the velocity of light.
10. Why one prefers even in summer a black umbrella?  So light can be considered to be an electromagnetic wave.
 Through white umbrella, a part of Sun light may penetrate, but a black one
absorbs all the radiation and hence it provides a better shade.

victory R. SARAVANAN. M.Sc., M.Phil., B.Ed PG ASST [PHYSICS], GBHSS, PARANGIPETTAI - 608 502
12 PHYSICS UNIT - 5 ELECTROMAGNETIC WAVES COMPLETE GUIDE AND MODEL QUESTION
19. Which is more dangerous to living things, gamma rays or X -rays. Explain. 23. How does the displacement current arise? What is the SI unit of displacement
 Gamma rays are more energetic since they have a shorter wavelength (hirgher current?
frequency), so they can potentially do more damage to biological organisms ;  Displacement current is arise due to varying electromotive force (i,e.) changing
however this does not imply that X-rays are necessarily safe - they must still be electric field and it is given by,
used to sparingly and cautiously. 𝒅 𝒅
𝒊𝒅 = 𝜺𝒐 𝚽𝑬 = 𝜺𝒐 ∮ 𝑬 ⃗⃗⃗⃗⃗
⃗ . 𝒅𝑨
20. Explain how we use spectral absorption and emission lines to determine the 𝒅𝒕 𝒅𝒕
composition of a gas. 𝒔
 As light passes through a gas, each specific element in a gas will leave a unique  The SI unit of displacement current is ampere (A)
pattern of absorption lines. Likewise, if a gas is heated, each specific element will 24. Distinguish between conduction current and displacement current.
emit a unique pattern of emission lines.  Conduction current is due to the flow of electrons in a circuit. It exists even if
 We know which patterns go with which elements and so we can use these electrons flow at a uniform rate.
spectral finger prints to deduce the composition of the gas.  Displacement currnt is due to the time varying electric field. It does not exist
21. Explain how emission lines and absorption lines are formed? under steady conduction.
 Emission lines are formed when an atom’s electron moves from a higher to a 25. How we measuring the speed of light using the microwave oven?
lower energy level and that atom then emits a photon with particular wavelength  It is interesting to note that the speed of light can be measured using micro wave
that corresponds to the energy difference between the two levels. oven.
 Absorption lines are caused when light passes through a cloud and some photons  Nowadays the microwave oven is very commonly used to heat the food items.
with energies corresponding to differences in energy levels in a given element Micro waves of wavelengths 1 mm to 30 cm are produced in these ovens. Such
are consumed by atoms to push their electrons to higher energy states. waves form the standing waves between the interior walls of the oven.
22. Give the differential form and integral form of Maxwell’s equations?  The standing waves have nodes and antinodes at fixed points. At node point, the
Name of the law Diffential Law Integral Law amplitude of the wave is zero and at antinodes point, the amplitude is maximum.
𝜌 𝑄 In other words, the maximal energy of microwaves is located at antinode points.
Gauss Law in ∇ . 𝐸⃗ = ⃗⃗⃗⃗⃗ =
∮ 𝐸⃗ . 𝑑𝐴  When we keep some food items like chappathi or choclate (after removing the
electrostatics 𝜀𝑜 𝜀𝑜
𝑠 rotating platform) inside the oven, we can notice that at antinode locations,
Gauss Law in chappathi will be burnt more than other locations.
⃗ = 0
∇ .𝐵 ∮𝐵 ⃗⃗⃗⃗⃗ = 0
⃗ . 𝑑𝐴
magnetism  The distance between two successive burnt spots will give the half wavelength of
𝑠 microwave. The frequency of microwave is printed in the panel of oven.

𝜕𝐵 𝜕Φ𝐵  By knowing wavelength and frequency of microwaves, using the formula,
Faraday’s Law ∇ 𝑋 𝐸⃗ = − ∮ 𝐸⃗ . ⃗⃗⃗
𝑑𝑙 = −
𝜕𝑡 𝜕𝑡 𝒗 𝝀 = 𝒄, we can calculate the speed of light c.
𝑙

Ampere- ⃗
𝜕𝐵 𝑑
⃗ = 𝜇𝑜 𝐽 + 𝜇𝑜 𝜀𝑜 ∮𝐵 ⃗⃗⃗ = 𝜇𝑜 𝑖𝐶 + 𝜇𝑜 𝜀𝑜
⃗ . 𝑑𝑙 ⃗⃗⃗⃗⃗
∮ 𝐸⃗ . 𝑑𝐴
Maxwell law ∇𝑋𝐵 𝑑𝑡
𝜕𝑡 𝑠 𝑠
Meaning :
Equation - 1
 The electric field leaving a volume is proportional to the charge inside
Equation - 2
 There are no magnetic monopoles, the total magnetic flux passing through a
closed surface is zero.
Equation - 3
 The voltage accumulated around a closed circuit is proportional to the time rate
of change of the magnetic flux it encloses.
Equation - 4
 Electric currents and changes in electric fields are proportional to the magnetic
field circulating about the area they pass through.

victory R. SARAVANAN. M.Sc., M.Phil., B.Ed PG ASST [PHYSICS], GBHSS, PARANGIPETTAI - 608 502
12 PHYSICS UNIT - 5 ELECTROMAGNETIC WAVES COMPLETE GUIDE AND MODEL QUESTION
EXAM NO 9. If the magnetic monopole exists, then which of the Maxwell’s equation to be
NAME : modified?.
UNIT - 5 ELECTROMAGNETIC WAVES 𝐸 . ⃗⃗⃗⃗⃗
(a) ∮ ⃗⃗⃗
𝑄
𝑑𝐴 = 𝑒𝑛𝑐𝑙𝑜𝑠𝑒𝑑
𝜀𝑜
Time - 2 : 30 hours Total - 60 marks ⃗ . ⃗⃗⃗⃗⃗
(b) ∮ 𝐵 𝑑𝐴 = 0
𝑑
PART - I 15 X 1 = 15 (c) ∮ 𝐵 ⃗⃗⃗ = 𝜇𝑜 𝑖𝐶 + 𝜇𝑜 𝜀𝑜
⃗ . 𝑑𝑙
𝑑𝑡
⃗⃗⃗⃗⃗
⃗⃗⃗ . 𝑑𝐴
∮𝐸
𝑑
Note : (i) Answer all the questions ⃗⃗⃗ = −
(d) ∮ 𝐸⃗ . 𝑑𝑙 Φ
𝑑𝑡 𝐵
(ii) Choose the best answer and write the option code and 10. Fraunhofer lines are an example of _______________________ spectrum.
corresponding answer (a) line emission
𝟏
1. The dimension of is (b) line absorption
𝜺𝒐 𝝁𝒐
(c) band emission
(a) [L T−1] (b) [L2 T−2]
(d) band absorption
(c) [L−1 T] (d) [L−2 T2]
11. Which of the following is an electromagnetic wave?
2. If the amplitude of the magnetic field is 3 × 10−6 T, then amplitude of the
(a) α - rays
electric field for a electromagnetic waves is
(b) β - rays
(a) 100 V m−1 (b) 300 V m−1
(c) γ - rays
(c) 600 V m−1 (d) 900 V m−1
(d) all of them
3. Which of the following electromagnetic radiations is used for viewing objects
12. Which one of them is used to produce a propagating electromagnetic wave?
through fog
(a) an accelerating charge
(a) microwave (b) gamma rays
(b) a stationary charge
(c) X- rays (d) infrared
(c) a charge moving with constant velocity
4. Which of the following is false for electromagnetic waves ?
(d) an uncharged particle
(a) transverse (b) non-mechanical waves
13. If E = Eo sin[106 x - ωt] be the electric field of a plane electromagnetic wave, the
(c) longitudinal (d) produced by accelerating charges
value of ω is
5. Consider an oscillator which has a charged particle oscillating about its mean
(a) 0.3 × 10−14 rad s−1
position with a frequency of 300 MHz. The wavelength of electromagnetic
(b) 3 × 10−14 rad s−1
waves produced by this oscillator is
(c) 0.3 × 1014 rad s−1
(a) 1 m (b) 10 m
(d) 3 × 1014 rad s−1
(c) 100 m (d) 1000 m
14. Which of the following is NOT true for electromagnetic waves?.
6. The electric and the magnetic fields, associated with an electromagnetic wave,
(a) it transports energy
propagating along negative X axis can be represented by
(b) it transports momentum
⃗ = 𝐵𝑜 𝑘̂
(a) ⃗⃗⃗𝐸 = 𝐸0 𝑖̂ and 𝐵 (b) ⃗⃗⃗𝐸 = 𝐸𝑜 𝑘̂ and ⃗⃗⃗𝐵 = 𝐵𝑜 𝑗̂ (c) it transports angular momentum
(c) 𝐸⃗ = 𝐸𝑜 𝑖̂ and 𝐵 ⃗ = 𝐵𝑜 𝑗̂ (d) ⃗⃗⃗𝐸 = 𝐸𝑜 𝑗̂ and ⃗⃗⃗𝐵 = 𝐵𝑜 𝑖̂ (d) in vacuum, it travels with different speeds which depend on their frequency
7. In an electromagnetic wave travelling in free space the rms value of the 15. The electric and magnetic fields of an electromagnetic wave are
electric field is 3 V m−1. The peak value of the magnetic field is (a) in phase and perpendicular to each other
(a) 1.414 × 10−8 T (b) 1.0 × 10−8 T (b) out of phase and not perpendicular to each other
(c) 2.828 × 10 T −8 (d) 2.0 × 10−8 T (c) in phase and not perpendicular to each other
8. An e.m. wave is propagating in a medium with a velocity ⃗⃗⃗𝒗 = 𝒗 𝒊̂ . The (d) out of phase and perpendicular to each other
instantaneous oscillating electric field of this e.m. wave is along +y-axis, then
the direction of oscillating magnetic field of the e.m. wave will be along:
(a) –y direction (b) –x direction
(c) +z direction (d) –z direction

victory R. SARAVANAN. M.Sc., M.Phil., B.Ed PG ASST [PHYSICS], GBHSS, PARANGIPETTAI - 608 502
12 PHYSICS UNIT - 5 ELECTROMAGNETIC WAVES COMPLETE GUIDE AND MODEL QUESTION
PART - II 6 X 2 = 12
Note : (i) Answer any 6 of the following questions .
(ii) Question No. 23 is compulsory
16. Define displacement current.
17. Give the modified form of Ampere’s circuital law.
18. What is the significance of Gauss’s law in magnetism.
19. Distinguish between emission spectra and absorption spectra.
20. Define Fraunhofer lines.
21. Define dispersion.
22. Why electro magnetic waves are non mechanical?
23. The relative magnetic permeability of the medium is 2.5 and the relative electrical
permittivity of the medium is 2.25. Compute the refractive index of the medium

PART - III 6 X 3 = 18
Note : (i) Answer any 6 of the following questions .
(ii) Question No. 30 is compulsory
24. Discuss briefly the experiment conducted by Hertz to produce and detect
electromagnetic spectrum.
25. Write a note on Radio waves.
26. Write a note on Micro waves. யாமறிந்த மமாழிகளிலே தமிழ்மமாழி லபால்
27. Write a note on IR rays (infra red rays). இனிதாவது எங்கும் கால ாம் !
28. Write a note on UVrays (ultra violet rays). பாமர ராய், விேங்குகளாய், உேகனைத்தும்
29. Compute the speed of the electromagnetic wave in a medium if the amplitude of
electric and magnetic fields are 3 × 104 N C–1 and 2 × 10–4 T, இகழ்ச்சிம ாேப் பான்னமமகட்டு
respectively. நாமமது தமிழமரைக் மகாண்டு இங்கு
30. If the relative permeability and relative permittivity of a medium are 1.0 and 2.25
respectively, find the speed of the electromagnetic wave in this medium
வாழ்ந்திடுதல் நன்ல ா? ம ால்லீர் !
PART - IV 3 X 5 = 15 லதமதுரத் தமிலழான உேகமமோம்
Note : (i) Answer all the questions பரவும்வனக ம ய்தல் லவண்டும்.
31. Write down Maxwell equations in integral form.
(OR) நான் அறிந்த மமாழிகளிலே தமிழ்மமாழி லபாே இனினமயாை
Explain the modification of Ampere’s circuital law. மமாழி எங்கும் இல்னே !
32. Explain the properties of electromagnetic waves. உேகனைத்தும் இகழும்படி சி ப்புகள் மகட்டுலபாக தமிழர் என்
(OR) மபயனர மட்டும் மகாண்டு வாழ்ந்திடுதல் நன்ல ா? ம ால்லுங்கள்.
(i) Write a note on X -rays எைலவ லதன் லபான் இனிய தமிலழான னய உேகமமோம்
(ii) Write a note on gamma rays பரவும்வனக ம ய்க.
33. Explain in detail the types of emission spectra.
(OR)
Explain in detail the types of absorption spectra. தமிழ் - மகாகவி சுப்பிரமண்ய பாரதியார்

victory R. SARAVANAN. M.Sc., M.Phil., B.Ed PG ASST [PHYSICS], GBHSS, PARANGIPETTAI - 608 502
பசித்திரு (Be hungry) தனித்திரு (Be individual) விழித்திரு (Be conscious)

HIGHER SECONDARY SECOND YEAR-PHYSICS

NAME :
STANDARD : 12 SECTION :
SCHOOL :
EXAM NO :

victory R. SARAVANAN. M.Sc, M.Phil, B.Ed.,


PG ASST (PHYSICS)
GBHSS, PARANGIPETTAI - 608 502
12 PHYSICS UNIT – 6 RAY OPTICS COMPLETE GUIDE AND MODEL QUESTION

PART – I 1 MARK MULTIPLE CHOICE QUESTIONS & ANSWERS 5. If the velocity and wavelength of light in air is Va and λa and that in water is
1. The speed of light in an isotropic medium depends on, Vw and λw, then the refractive index of water is,
(a) its intensity 𝑉𝑤 𝑉𝑎 𝜆𝑤 𝑉𝑎 𝜆𝑎
(a) (b) (c) (d)
(b) its wavelength 𝑉𝑎 𝑉𝑤 𝜆𝑎 𝑉𝑤 𝜆𝑤
(c) the nature of propagation Solution :
𝑣 𝜆
(d) the motion of the source w.r.t medium  Refractive index of water; 𝑛𝑤 = 𝑣 𝑎 = 𝜆 𝑎
Solution : 𝑤 𝑤
𝑽𝒂
 Velocity of light ; 𝑐 = 𝜆 𝜈 Answer (b) 𝑽𝒘
 Here frequency 𝜈 is always constant and hence velocity depends on wavelength 𝜆 6. Stars twinkle due to,
Answer (b) its wavelength (a) reflection (b) total internal reflection
2. A rod of length 10 cm lies along the principal axis of a concave mirror of focal (c) refraction (d) polarisation
length 10 cm in such a way that its end closer to the pole is 20 cm away from Solution :
the mirror. The length of the image is,  The change in intensity of light coming from the distance star is called twinkling
(a) 2.5 cm (b) 5 cm of stars. It is due to atmospheric refraction of star’s light when it passes
(c) 10 cm (d) 15 cm through different layers of a turbulent atmosphere and hence the star light
Solution : reaching our eyes change continuously and stars appear to twinkle.
 Here object distance . 𝑢𝐴 = − 20𝑐𝑚 ; 𝑢𝐵 = − 30 𝑐𝑚 ; 𝑓 = − 10 𝑐𝑚 . Then Answer (c) refraction
1 1 1 1 1 1 1 1 1
+ = (or) = − = (−10) − = − ⟹ 𝑣𝐴 = −20 𝑐𝑚 7. When a biconvex lens of glass having refractive index 1.47 is dipped in a
𝑣𝐴 𝑢𝐴 𝑓 𝑣𝐴 𝑓 𝑢𝐴 (−20) 20
1
+
1
=
1
(or)
1
=
1

1
=
1

1
=−
1
⟹ 𝑣𝐵 = −15 𝑐𝑚 liquid, it acts as a plane sheet of glass. This implies that the liquid must have
𝑣𝐵 𝑢𝐵 𝑓 𝑣𝑏 𝑓 𝑢𝐵 (−10) (−30) 15 refractive index,
 Hence image length, 𝑣𝐵 − 𝑣𝐴 = −15 − (−20) = 5 𝑐𝑚 (a) less than one (b) less than that of glass
Answer (b) 5 cm (c) greater than that of glass (d) equal to that of glass
3. An object is placed in front of a convex mirror of focal length of f and the Solution :
1 𝑛 1 1
maximum and minimum distance of an object from the mirror such that the  From Len’s makers formula, 𝑓 = ( 𝑛𝑔 − 1) [𝑅 − 𝑅 ]
image formed is real and magnified. 𝑙 1 2

(a) 2f and c (b) c and ∞  For plane sheet of glass, 𝑓 = ∞


1 𝑛 1 1 𝑛 1 1
(c) f and O (d) None of these  So, ∞ = ( 𝑛𝑔 − 1) [𝑅 − 𝑅 ] (or) ( 𝑛𝑔 − 1) [𝑅 − 𝑅 ] = 0
Solution : 1
𝑙
1
1 2
𝑛𝑔
𝑙 1 2
𝑛𝑔
 Convex mirror always forms virtual image irrespective of the position of the object  Here, [
𝑅1
− 𝑅2
] ≠0 Hence, (
𝑛𝑙
− 1) = 0 (or) 𝑛𝑙
= 1 (or) 𝑛𝑔 = 𝑛𝑙
Answer (d) None of these Answer (d) equal to that of glass
4. For light incident from air on a slab of refractive index 2, the maximum 8. The radius of curvature of curved surface at a thin planoconvex lens is 10 cm
possible angle of refraction is, and the refractive index is 1.5. If the plane surface is silvered, then the focal
(a) 30o (b) 45o length will be,
(c) 60 o (d) 90o (a) 5 cm (b) 10 cm (c) 15 cm (d) 20 cm
Solution : Solution :
 From the product form of law of refraction (Snell’s law), 𝑛𝑎𝑖𝑟 sin 𝑖 = 𝑛𝑠𝑙𝑎𝑏 sin 𝑟  Let 𝑓𝐿 be the focal length of plano convex lens, then
 Here angle of refraction is maximum, when angle of incidence will be 90 1 1 1
= (𝑛 − 1) [𝑅 − ∞] = (𝑛 − 1) 𝑅
1
(or) 𝑓𝐿 =
𝑅 10 10
= 1.5−1 = 0.5 = 20 𝑐𝑚
1 𝑓𝐿 𝑛−1
 Hence, (1) sin 90 = (2) sin 𝑟𝑚𝑎𝑥 (or) sin 𝑟𝑚𝑎𝑥 = (or) 𝑟𝑚𝑎𝑥 = 30 1
2  If the plane surface is silvered, then it acts as plane mirror of focal length, 𝑓𝑀 = ∞
Answer (a) 30o  So the total power of silvered plano convex lens, 𝑃 = 𝑃𝐿 + 𝑃𝑀 + 𝑃𝐿
1 1 1 1 2 2 2 1
(or)
𝐹
=𝑓 +𝑓 +𝑓 = 𝑓𝐿
+ 0 = 𝑓 = 20 = 10 (or) 𝐹 = 10 𝑐𝑚
𝐿 𝑀 𝐿 𝐿
Answer (b) 10 cm
victory R. SARAVANAN. M.Sc., M.Phil., B.Ed PG ASST [PHYSICS], GBHSS, PARANGIPETTAI - 608 502
12 PHYSICS UNIT – 6 RAY OPTICS COMPLETE GUIDE AND MODEL QUESTION
9. An air bubble in glass slab of refractive index 1.5 (near normal incidence) is 5
cm deep when viewed from one surface and 3 cm deep when viewed from the
PART – II & III 2 and 3 MARK SHORT ANSWER QUESTIONS & ANSWERS
opposite face. The thickness of the slab is, 1. Define reflection.
(a) 8 cm (b) 10 cm  The bouncing back of light in to the same medium when it encounters a
(c) 12 cm (d) 16 cm reflecting surface is called reflection of light.
Solution : 2. State the laws of reflection.
 We know that, Actual depth = refractive index X apparent depth (1) The incident ray, reflected ray and the normal to the surface all are
 Actual depth of drop from one surface= 1.5 𝑋 5 = 7.5 𝑐𝑚 coplanar.
 Actual depth of drop from opposite surface= 1.5 𝑋 3 = 4.5 𝑐𝑚 (2) The angle of incidence (𝑖) is equal to angle of reflection (𝑟). That is 𝒊 = 𝒓
3. What is the angle of deviation due to reflection?
 So the thickness of the slab = 7.5 + 4.5 = 12 𝑐𝑚
 The angle between the incident and deviated ray is called angle of deviation (d)
Answer (c) 12 cm of the light ray.
10. A ray of light travelling in a transparent medium of refractive index n falls, on a
surface separating the medium from air at an angle of incidents of 45o. The ray
can undergo total internal reflection for the following n,
(a) n = 1.25 (b) n = 1.33
(c) n = 1.4 (d) n = 1.5
Solution :
 By the product form of Snell’s law ; 𝑛1 sin 𝑖 = 𝑛2 sin 𝑟
 When 𝑖 = 𝑖𝐶 , then 𝑟 = 90°  From figure (a),
 For total internal reflection occur, 𝑖 > 𝑖𝐶 Hence, 𝑛 sin 𝑖 > (1) sin 90° 𝑑 = 180° − (𝑖 + 𝑟) [𝑖 = 𝑟]
1 1 1 𝒅 = 𝟏𝟖𝟎° − 𝟐 𝒊
𝑛> sin 𝑖
(or) 𝑛> sin 45°
(or) 𝑛 >
1/√2
(or) 𝑛 > √2 (1.414)  The angle between the incident ray and the reflecting surface is called glancing
Answer (d) n = 1.5 angle (𝛼).
 From figure (b), 𝑑 = ∠𝐵𝑂𝑌 + ∠𝑌𝑂𝐶 = 𝛼 + 𝛼 = 𝟐 𝜶
4. What are the characteristics of the image formed by the plane mirror?
Characteristics of the image of the plane mirror :
 Virtual, erect and laterally inverted.
 Size of image is equal to the size of the object.
 The distance of the image behind the mirror is equal to the distance of object in
front of it.
 If an object placed between two plane mirrors inclined at an angle 𝜃 , then the
number (n) of images formed is,
360° 360°
(1) If [ ] even, then ; 𝑛 = [ − 1 ] for objects placed symmentrically or
𝜃 𝜃
unsymmentrically.
360° 360°
(2) If [ ] odd, then ; 𝑛 = [ − 1 ] for objects placed symmentrically
𝜃 𝜃
360° 360°
(3) If [ ] odd, then ; 𝑛 = [ ] for objects placed unsymmentrically
𝜃 𝜃
5. Distinguish convex mirror and concanve mirror?
Convex mirror Concave mirror
It is a spherical mirror in which It is a spherical mirror in which
reflection takes place at the convex reflection takes place at the concave
surface and other surface is silvered surface and other surface is silvered

victory R. SARAVANAN. M.Sc., M.Phil., B.Ed PG ASST [PHYSICS], GBHSS, PARANGIPETTAI - 608 502
12 PHYSICS UNIT – 6 RAY OPTICS COMPLETE GUIDE AND MODEL QUESTION
6. Define (1) centre of curvature, (2) Radius of curvature (3) pole, (4) principal  The line ‘CM’ is the normal to the mirror at ‘M’
axis, (5) focus or focal point, (6) focal length, (7) focal plane  From the figure (a),
(1) Centre of curvature : angle of incidence ; 𝑖 = ∠𝐴𝑀𝐶
 The centre of the sphere of which the mirror is a part is called centre of angle of reflection ; 𝑟 = ∠𝐶𝑀𝐹
curvature (C)  By the law of reflection. we have, 𝒊 = 𝒓
(2) Radius of curvature :  Thus, , ∠𝑀𝐶𝑃 = 𝑖 & ∠𝑀𝐹𝑃 = 2 𝑖
 The radius of the sphere of which the spherical mirror is a part is called the  From ∆𝑀𝐶𝑃 and ∆𝑀𝐹𝑃
radius of curvature (R) of the mirror. 𝑃𝑀 𝑃𝑀
(3) Pole (or) Optic centre : tan 𝑖 = & tan 2 𝑖 =
𝑃𝐶 𝑃𝐹
 The middle point on the spherical surface of the mirror (or) the geometrical  As the angles are small, we have tan 𝑖 ≈ 𝑖 and tan 2 𝑖 ≈ 2 𝑖 . So
centre of the mirror is called the pole (P) of the mirror. 𝑃𝑀
𝑖 = − − − − − (1)
(4) Principal axis : 𝑃𝐶
 The line joining the pole (P) and the centre of curvature (C) is called the 𝑃𝑀
2𝑖 = − − − − − (2)
principal axis (or) optical axis of the mirror. 𝑃𝐹
(5) Focus or Focal point :  Put eqn (1) in eqn (2)
 Light rays travelling parallel and close to the principal axis when incident 𝑃𝑀 𝑃𝑀
2 =
on a spherical mirror, converge at a point for concave mirror or appears to 𝑃𝐶 𝑃𝐹
diverge from a point for convex mirror on the principal axis. This point is (𝑜𝑟) 2 𝑃𝐹 = 𝑃𝐶
called the focus or focal point (F) of the mirror (𝑜𝑟) 2 𝑓= 𝑅
(6) Focal length : 𝑹
(𝑜𝑟) 𝒇= − − − − − (3)
 The distance between the pole (P) and the Focus (F) is called the focal 𝟐
length (f) of the mirror. 9. How we locate the image formation in spherical mirrors?
(7) Focal plane : Image formation in spherical mirrors:
 The plane through the focus and perpendicular to the principal axis is
called the focal plane of the mirror.
7. Define paraxial rays and marginal rays.
Paraxial rays :
 The rays travelling very close to the principal axis and make small angle with it
are called paraxial rays. a) A ray parallel to the principal axis after reflection will pass through or appear to
Marginal rays : pass through the principal focus.
 The rays travelling far away from the principal axis and fall on the mirror far b) A ray passing through or appear to pass through the principal focus, after
away from the pole are called as marginal rays. reflection will travel parallel to the principal axis,
8. Obtain the relation between focal length (f) and radius of curvature (R) of the c) A ray passing through the centre of curvature retraces its path after reflection
spherical mirror. as it is a case of normal incidence.
Relation between f and R : d) A ray falling on the pole will get reflected as per law of reflection keeping
principal axis as the normal.
10. What are the Cartesian sign conventions for a spherical mirrors?
Cartesian sign convention :

 Let ‘C’ be the centre of curvature of the mirror.


 Consider a light ray parallel to the principal axis and incident at ‘M’ on the
mirror.
 After reflection, it will passes through principal focus ‘F’
victory R. SARAVANAN. M.Sc., M.Phil., B.Ed PG ASST [PHYSICS], GBHSS, PARANGIPETTAI - 608 502
12 PHYSICS UNIT – 6 RAY OPTICS COMPLETE GUIDE AND MODEL QUESTION
 The incident light is taken from left to right. 17. What is the principle of reversiability?
 All the distances are measured from the pole.  The principle of reversibility states that, light will be follow exactly the same
 The distance measured to the right of pole along the principal axis are taken as path if its direction of travel is reversed.
positive  This is true for both reflection and refraction.
 The distance measured to the left of pole along the principal axis are taken as 18. Define relative refractive index.
negative sin 𝑖 𝑛
 From Snell’s law, = 2
sin 𝑟 𝑛1
 Heights measured in the upward perpendicular direction to the principal axis 𝑛2
are taken as positive  Here the term [ ] is called relative refractive index of second medium with
𝑛1
 Heights measured in the downward perpendicular direction tothe principal axis respect to the first medium and it is denoted by 𝑛21 (i,e.) 𝒏𝟐𝟏 =
𝒏𝟐
are taken as negative 𝒏𝟏

11. Define refractive index. 19. Give the useful relations obtained from the concept of relative refractive
 Refractive index (n) of a transparent medium is defined as the ratio of speed of index.
𝒄 𝟏 𝒏𝟏 𝟏
light in vacuum (or air) to the speed of light on that medium. 𝒏 = (1) Inverse rule : 𝒏𝟏𝟐 = (𝒐𝒓) = 𝒏𝟐
𝒏𝟐𝟏 𝒏𝟐 [ ]
𝒗 𝒏𝟏
12. Define optical path. 𝒏𝟑 𝒏 𝒏𝟏
(2) Chain rule : 𝒏𝟑𝟐 = 𝒏𝟑𝟏 × 𝒏𝟏𝟐 (𝒐𝒓) = 𝒏𝟑 ×
 Optical path of a medium is defined as the distance (d) light travels in vacuum 𝒏𝟐 𝟏 𝒏𝟐
in the same time it travels a distance (d) in the medium. 20. Obtain the equation for apparent depth.
 If ‘n’ is the refractive index of the medium. then optical path is ; d = n d Apparent depth :
13. What is called refraction?
 Refraction is passing through of light from one optical medium to another
optical medium through a boundary.
14. State the laws of refraction (Snell’s law).
 The incident ray, refracted ray and normal are all coplanar.
 The ratio of angle of incident ‘i’ in the first medium to the angle of reflection ‘r’
in the second medium is equal to the ratio of refractive index of the second
medium ‘𝑛2 ’ to that of the refractive index of the first medium ‘𝑛1 ’
sin 𝑖 𝑛2  We observe that the bottom of a tank filled with water with water appears
= (𝒐𝒓) 𝒏𝟏 𝐬𝐢𝐧 𝒊 = 𝒏𝟐 𝐬𝐢𝐧 𝒓 raised as shown.
sin 𝑟 𝑛1
15. What is the angle of deviation due to refraction?  Light OB from the object ’O’ passes through water get refracted in air
Angle of deviation due to refraction :  The refracted ray BC appers to come from ‘I’ which is just above ‘O’ (i.e) the
 The angle between the incident and deviated ray is called angle of deviation. object is appears to be at ‘I’
 When light travels from rarer to denser medium it deviates towards normal.  Refractive index of water = 𝒏𝟏
Hence the angle of deviation ; 𝒅 = 𝒊 − 𝒓 Refractive index of air = 𝒏𝟐
 When light travels from denser to rarer medium it deviates away normal. Hence Angle of incidence in water = 𝒊
the angle of deviation ; 𝒅 = 𝒓 − 𝒊 Angle of refraction in air = 𝒓
Original depth of tank = 𝑫𝑶 = 𝒅
Apparent depth of tank = 𝑫𝑰 = 𝒅
 Here 𝒏𝟏 > 𝒏𝟐 . Hence , 𝒊 < 𝒓
 By Snell’s law in product form,
𝑛1 sin 𝑖 = 𝑛2 sin 𝑟
 As the angles aresmall, we can write
16. Define simultaneous reflection or simultaneous refraction. sin 𝑖 ≈ 𝑡𝑎𝑛 𝑖 & sin 𝑟 ≈ 𝑡𝑎𝑛𝑟
 The phenomenon in which a part of light from a source undergoing reflection Hence, 𝑛1 ta𝑛 𝑖 = 𝑛2 tan 𝑟 − − − − (1)
and other part of light from same source undergoing refraction at the same  In ∆𝐷𝑂𝐵 𝑎𝑛𝑑 ∆𝐷𝐼𝐵,
surface is called simultaneous reflection or simultaneous refraction. 𝐷𝐵 𝐷𝐵
ta𝑛 𝑖 = =
 Such surfaces are available as partially silvered glasses. 𝐷𝑂 𝑑
victory R. SARAVANAN. M.Sc., M.Phil., B.Ed PG ASST [PHYSICS], GBHSS, PARANGIPETTAI - 608 502
12 PHYSICS UNIT – 6 RAY OPTICS COMPLETE GUIDE AND MODEL QUESTION
𝐷𝐵 𝐷𝐵 25. Obtain the reason for glittering of diamond.
ta𝑛 𝑟 = = 
𝐷𝐼 𝒅 Glittering of diamond :
 Put this in eqn (1)  The glittering of diamond is due to the total internal reflection of light happens
𝐷𝐵 𝐷𝐵 inside the diamond.
𝑛1 [ ] = 𝑛2 [  ]
𝑑 𝒅  The refractive index of diamond is 2.417 and the critical angle is 24.4
1 1  Diamond has large number of cut planed faces.
𝑛1 = 𝑛2 
𝑑 𝒅  So light entering the diamond get total internally reflected from many cut faces

𝒏𝟐
∴ 𝒅 = 𝒅 before getting out.
𝒏𝟏  This gives a sparkling effect for diamond.
 For air ; 𝒏𝟐 = 𝟏 and let 𝒏𝟐 = 𝒏 , then apparent depth 26. What are mirage and looming?
𝒅
𝒅 = Mirrage :
𝒏  In hot places, air near the ground is hotter than air at a height. Hot air less
 Thus the bottom appears to be elevated by (𝑑 − 𝑑  ) dense.
𝒅 𝟏
𝒅 − 𝒅 = 𝒅 − = 𝒅 (𝟏 − )  The refractive index of air decreases with decrease in density.
𝒏 𝒏  Because of this, the air near hot ground acts as rarer medium than the air at
21. Define critical angle. height.
 The angle of incidence in the denser medium for which the refracted ray graces  When light from tall object like tree, passes through a medium whose refractive
the boundary is called critical angle 𝒊𝑪 index decreases towards the ground, it successively deviates away from the
22. Define total internal reflection. normal and undergoes total internal reflection when the angle of incidence near
 If the angle of incidence in the denser medium is greater than the critical angle, the ground exceeds the critical angle.
there is no refraction possible in the rarer medium.  This gives an illusion as if the light comes from somewhere below the ground.
 The entire light is reflected back in to the denser medium itself, This  For of the shaky nature of the layers of air,the observers feels as wet surface
phenomenon is called total internal reflection. beneath the object.
23. What are the conditions to achieve total internal reflection?  This phenomenon is called mirage.
 Light must travel from denser to rarer medium Looming :
 Angle of incidence must be greater than critical angle (𝑖 > 𝑖𝐶 )  In cold places, the refractive index increases towards the ground, because the
24. Obtain an expression for critical angle. temperature of air close to the ground is less than the air at height.
Critical angle:  So in cold regions like glaciers and frozen lakes and seas, the reverse effect of
 When light ray passes from denser medium to rarer medium, it bends away mirage will happen.
from normal. So 𝑖 < 𝑟  Hence an inverted image is formed little above the surface. This phenomenon
 As 𝑖 increases, 𝑟 also increases rapidly and at a certain stage it just gracing called looming.
the boundary (𝑟 = 90°). The corresponding anle of incidence is called 27. Write a note on the prisms making using of total internal reflection.
critical angle (𝑖𝐶 ) Prisms making using use of total internal reflection
 From Snell’s law of product form
𝒏𝟏 𝐬𝐢𝐧 𝒊 = 𝒏𝟐 𝐬𝐢𝐧 𝒓
 When 𝑖 = 𝑖𝐶 , then 𝑟 = 90°
𝒏𝟏 𝐬𝐢𝐧 𝒊𝑪 = 𝒏𝟐 𝐬𝐢𝐧 90°
𝒏𝟏 𝐬𝐢𝐧 𝒊𝑪 = 𝒏𝟐
𝒏𝟐
𝐬𝐢𝐧 𝒊𝑪 =
𝒏𝟏
 If the rarer medium is air, then 𝒏𝟐 = 𝟏 and let 𝒏𝟏 = 𝒏 , then
𝟏
𝐬𝐢𝐧 𝒊𝑪 =
𝒏  Prisms can be designed to reflect light by 90° or by 180° by making use of total
𝟏 internal reflection.
(or) 𝒊𝑪 = 𝐬𝐢𝐧−𝟏 ( )
𝒏  In both cases, the critical angle of material of the prism must be less than 45°
.This is true for both crown glass and flint glass
victory R. SARAVANAN. M.Sc., M.Phil., B.Ed PG ASST [PHYSICS], GBHSS, PARANGIPETTAI - 608 502
12 PHYSICS UNIT – 6 RAY OPTICS COMPLETE GUIDE AND MODEL QUESTION
28. What is Snell’s window (or) Radius of illumination? 31. Define primary and secondary focal points.
Snell’s window : Primary focal point (𝐅𝟏 ) :
 When a light source like electric bulb is kept inside a water tank, the light from
the source travels in all direction inside the water.
 The light that incident on water surface at an angle less than the critical angle
will undergo refraction and emerge out from the water.
 The light incident at an angle greater than the critical angle will undergo total
internal reflection.  It is defined as a point, where an object should be placed to give paraller
 But the light incident at critical angle graces the surface and hence the entire emergent ray to the principal axis
surface of water appears illuminated when seen from outside.  For convergent lens such an object is real and for divergent lens, the object is
 On the otherhand, when light entering water from outside is seen from inside virtual. Here, 𝑃𝐹1 = 𝑓1 → principal focal length
the water, the view is restricted to a particular angle equal to the critical angle Secondary focus point (𝐅𝟐 ) :
𝑖𝐶
 The restricted illuminated circular area is called Snell’s window.
29. Write a note on optical fibres.
Optical fibre:
 It is defined as a point, where all the parallal rays travelling close to the
principal axis converge to form an image on the principal axis.
 For convergent lens such an image is real and for divergent lens, the imagt is
virtual. Here, 𝑃𝐹2 = 𝑓2 → secondary focal length
32. What are the sign conventions for lens on focal length?
 The sign of focal length is not decided on the direction of measurement of the
focal length from the pole of the lens as they have two focal lengths on either
 Transmitting signals from one end to another end due to the phenomenon of side of the lens.
total internal reflection is called optical fibres.  The focal length of thin lens is taken as positive for a converging lens and
 It consists of inner part called core and outer part called cladding or sleeving negative for a diverging lens
 The refractive index of the core must be higher than that of the cladding. 33. Define power of a lens.
 Signal in the form of light is made to incident inside the core-cladding boundary  The power ‘P’ of a lens is defined as the reciprocal of its focal length (𝒇)
at an angle greater than the critical angle. 𝟏
𝑷 = = (𝒏 − 𝟏) [ − ]
𝟏 𝟏
𝒇 𝑹𝟏 𝑹𝟐
 Hence it undergoes repeated total internal reflections along the length of the
fibre without undergoing any refraction.  The unit of power is diopter (D)
 Even while bending the optic fibre, it is done in such a way that the condition  Power is positive for converging lens and negative for diverging lens.
for total internal reflection is ensured at every reflection. 34. Define the power of a mirror.
30. Write a note on an endoscope.  The power of a mirror is negative of the reciprocal of its focal length. (i. e.) 𝑷 =
𝟏
Endoscope : −𝒇
 An endoscope is an instrument used by doctors which has a bundle of optical  This is because, a concave mirror which has negative focal length is a
fibres that are used to see inside a patient’s body. converging mirror with positive power.
 It works on the phenomenon of total internal reflection. 35. Define silvered lenses.
 It is inserted in to the body through mouth or nose or a special hole made in the  If one of the surfaces of a lens is silvered from outside, then such a lens is said to
body. be a silvered lens. It is a combination of a lens and a mirror.
 The necessary instruments for operation is attached at their ends.  A silvered lens is basically a modified mirror and its power is given by
𝑷 = 𝟐 𝑷𝒍𝒆𝒏𝒔 + 𝑷𝒎𝒊𝒓𝒓𝒐𝒓
𝟏 𝟐 𝟏
(𝒐𝒓) [ ]= [ ]+ [ ]
−𝒇 𝒇𝒍𝒆𝒏𝒔 −𝒇𝒎𝒊𝒓𝒓𝒐𝒓

victory R. SARAVANAN. M.Sc., M.Phil., B.Ed PG ASST [PHYSICS], GBHSS, PARANGIPETTAI - 608 502
12 PHYSICS UNIT – 6 RAY OPTICS COMPLETE GUIDE AND MODEL QUESTION
36. Write a note on prism. 44. Why does cloud appears as white colour?
 A prism is a triangular block of glass or plastic which is bounded by the three  When size of particles or water drops are greater than the wavelength of light
plane faces not parallel to each other. (𝑎 ≫ 𝜆), the intensity of scattering is equal for all the wavelength.
 Its one face is grounded which is called base.  Since clouds contains large amount of dust and water droplets, all the colours
 The other two faces are polished which are called refracing faces of the prism. get equally scattered irrespective of wavelength. This is the reason for the
 The angle between the two refracting faces is called angle of prism (A) whitish appearance of cloud.
37. Define angle of minimum deviation.  But the rain clouds appear dark because of the condensation of water droplets
 The angle between incident ray and emergent ray is called angle of deviation (d). on dust paricles that make the cloud become opque.
 When the angle of incidence increases, the angle of deviation decreases, reaches 45. How are rainbows formed?
a minimum value and then continues to increase. Formation of rainbows :
 The minimum value of angle of deviation is called angle of minimum deviation (D).  Rainbows are formed due to dispersion of sunlight through droplets of water
38. What is called dispersion of light? during rainy days.
 The splitting of white light in to its constituent colours is called dispersion of  Rainbow is observed during rainfall or after rainfall or looking water fountain
light. provided the Sun is at the back of the observer.
 This band of colours of light is called its spectrum.  When sun light falls on the water drop suspended air, it splits in to its
 The spectrum consists seven colours in the order VIBGYOR constituent seven colours. Here waterdrops acts as a glass prism.
39. Define dispersive power.  Primary rainbow is formed when one total internal reflection takes place
 Dispersive power (𝜔) is the ability of the material of the prism to cause prism. inside the drop. The angle of view for violet to red in primary rainbow is 40
 It is defined as the ratio of the angular dispersion for the extreme colours to the to 42
deviation for any mean colour.  Secondary rainbow is formed when two total internal reflection takes place
40. What is Rayleigh’s scattering? inside the drop. The angle of view for violet to red in primary rainbow is 52
 The scattering of light by atoms and molecules which have size (𝒂) very less to 54
than that of the wavelength (𝜆) of light is called Rayleigh’s scattering.
(i.e) condition for Rayleigh’s scattering is 𝒂 << 
41. State Rayleigh’s scattering law.
 The intensity (I) of Rayleigh’s scattering is inversely proportional to fourth
power of wavelength (𝜆)
𝟏
𝑰 ∝ 𝟒
𝝀
42. Why does sky appears blue colour?
 According to Rayleigh’s scattering, shorter wavelenths (violet) scattered much
more than longer wavelengths (Red)
 As our eyes are more sensitive to blue colour than violet, the sky appears blue
during day time.
43. Why does sky and Sun looks reddish during sunset and sunrise?
 During sunset or sunrise, the light from Sun travels a greater distance through
atmosphere.
 Hence the blue light which has shorter wavelength is scattered away and less
scattered red light of longer wavelength reaches observer
 This is the reason for reddish appearance of sky and Sun during sunrise and
sunset.

victory R. SARAVANAN. M.Sc., M.Phil., B.Ed PG ASST [PHYSICS], GBHSS, PARANGIPETTAI - 608 502
12 PHYSICS UNIT – 6 RAY OPTICS COMPLETE GUIDE AND MODEL QUESTION
Lateral magnification:
PART – IV 5 MARK LONG ANSWER QUESTIONS & ANSWERS
 It is defined as the ratio of the height of the image (ℎ1 ) to the height of the object
1. Derive the mirror equation and the equation for lateral magnification. (h)..
Mirror equation :  From eqn (1)
 The equation which gives the 𝐴1 𝐵1 𝑃𝐴1
relation between object =
𝐴𝐵 𝑃𝐴
distance (𝑢), image distance − ℎ1 −𝑣
(𝑣) and focal length (𝑓) is of =
ℎ −𝑢
spherical mirror is called  Hence magnification,
mirror equation. 𝒉𝟏 𝒗
 Let an object AB is placed on 𝒎= =− − − − − (𝟓)
𝒉 𝒖
the principle axis of a concave  Using eqn (4)
mirror beyond the centre of 𝒉𝟏 𝒇−𝒗 𝒇
curvature ‘C’ 𝒎= = = − − (𝟔)
 The real and inverted image 𝐴1 𝐵1 is formed between C and F 𝒉 𝒇 𝒇−𝒖
2. Describe the Fizeau’s method to determine speed of light.
 By the laws of reflection,
angle of incidence (𝑖) = angle of reflection (𝑟) Fizeau’s method :
∠𝐵𝑃𝐴 = ∠𝐵1 𝑃𝐴1
 From figure, ∆ 𝐵𝑃𝐴 and ∆ 𝐵1 𝑃𝐴1 are similar triangles. So
𝐴1 𝐵1 𝑃𝐴1
= − − − − − (1)
𝐴𝐵 𝑃𝐴
1 1
 Also ∆ 𝐷𝑃𝐹 and ∆ 𝐵 𝐴 𝐹 are similar triangles. So
𝐴1 𝐵1 𝐴1 𝐹
= [𝑃𝐷 = 𝐴𝐵]
𝑃𝐷 𝑃𝐹
1 1 1
𝐴 𝐵 𝐴 𝐹
= − − − − − (2)
𝐴𝐵 𝑃𝐹
 From eqn (1) and (2),
𝑃𝐴1 𝐴1 𝐹
=
𝑃𝐴 𝑃𝐹
𝑃𝐴1 𝑃𝐴1 − 𝑃𝐹  The light from the source S was first allowed to fall on a partially silvered glass
= − − − − (3) plate G kept at an angle of 45 to the vertical.
𝑃𝐴 𝑃𝐹
 By applying sign conventions,  The light then allowed to pass through a rotating toothed-wheel with N -teeth
𝑃𝐴 = −𝑢 ; 𝑃𝐴1 = −𝑣 ; 𝑃𝐹 = −𝑓 and N -cuts.
−𝑣 − 𝑣 − ( −𝑓)  The speed of rotation of the wheel could be varied through an external
= mechanism.
−𝑢 −𝑓
𝑣 𝑣−𝑓  The light passing through one cut in the wheel get reflected by a mirror M kept
(𝑜𝑟) = at a long distance ‘d’ (about 8 km) from the toothed wheel.
𝑢 𝑓
𝑣 𝑣  If the toothed wheel was not rotating, the reflected light from the mirror would
(𝑜𝑟) = −1 again pass through the same cut and reach the observer through G.
𝑢 𝑓
Working :
 Dividing both sides by 𝑣
1 1 1  The angular speed of the rotation of the toothed wheel was increased until light
= − passing through one cut would completely be blocked by the adjacent tooth. Let
𝑢 𝑓 𝑣 that angular speed be 𝜔
𝟏 𝟏 𝟏
+ = − − − − − (𝟒)  The total distance traveled by the light from the toothed wheel to the mirror
𝒗 𝒖 𝒇 and back to the wheel is ‘2d’ and the time taken be ‘t’.
 This is called mirror equation. It is also valid for convex mirror.
victory R. SARAVANAN. M.Sc., M.Phil., B.Ed PG ASST [PHYSICS], GBHSS, PARANGIPETTAI - 608 502
12 PHYSICS UNIT – 6 RAY OPTICS COMPLETE GUIDE AND MODEL QUESTION
2
 Then the speed of light in air, 𝑑 𝑛12− 𝑛22
2𝑑 =
𝑣= 𝑅2 𝑛22
𝑡 𝑅2 𝑛22
 But the angular speed is, (𝑜𝑟) =
𝜃 𝑑2 𝑛12 − 𝑛22
𝜔= 𝑛22
𝑡 𝑅2 = 𝑑 2 [ 2 ]
 Here 𝜃 is the angle between the tooth and the slot which is rotated by the 𝑛1 − 𝑛22
toothed wheel within that time ‘t’ . Then, 𝒏𝟐𝟐
𝑡𝑜𝑡𝑎𝑙 𝑎𝑛𝑔𝑙𝑒 𝑜𝑓 𝑡ℎ𝑒 𝑐𝑖𝑟𝑐𝑙𝑒 𝑖𝑛 𝑟𝑎𝑑𝑖𝑎𝑛 2𝜋 𝜋 ∴ 𝑹=𝒅√
𝜃= = = 𝒏𝟏𝟐 − 𝒏𝟐𝟐
𝑛𝑢𝑚𝑏𝑒𝑟 𝑜𝑓 𝑡𝑒𝑒𝑡ℎ + 𝑛𝑢𝑚𝑏𝑒𝑟 𝑜𝑓 𝑐𝑢𝑡𝑠 2𝑁 𝑁
 Hence angular speed,  If the rarer medium outsideis air, then 𝑛2 = 1 and let 𝑛1 = 𝑛 , then
𝜋 𝟏
( ) 𝜋 𝑹=𝒅 [ ]
𝜔= 𝑁 = 𝟐
√𝒏 − 𝟏
𝑡 𝑁𝑡 4. Derive the equation for acceptance angle and numerical aperture of optical
𝜋
(𝑜𝑟) 𝑡 = fibe.
𝑁𝜔
 Therefore the speed of light in air, Acceptance angle :
2𝑑 2𝑑  To ensure the critical angle incidence in the core-cladding boundary inside the
𝑣= = 𝜋 optical fibre, the light should be incident at a certain angle at the ene of the
𝑡 ( )
𝑁𝜔 optical fibre while entering in to it. This angle is called acceptance angle.
𝟐𝒅𝑵𝝎
𝒗=
𝝅
 The speed of light in air was determined as, 𝒗 = 𝟐. 𝟗𝟗𝟕𝟗𝟐 𝑿 𝟏𝟎𝟖 𝒎 𝒔−𝟏
3. Obtain the equation for radius of illumination (or) Snell’s window.
Radius of Snell’s window :
 Light is seem from a point ‘A’ at a depth ‘d’
 Applying Snell’s law in product form at point ‘B,
𝑛1 sin 𝑖𝐶 = 𝑛2 sin 90°
𝑛1 sin 𝑖𝐶 = 𝑛2
𝑛2  Applying Snell’s law at point ‘A’,
(𝑜𝑟) sin 𝑖𝐶 = − − − (1)
𝑛1 sin 𝑖𝑎 𝑛1
 𝐼𝑛 ∆𝐴𝐵𝐶, = − − − − − (1)
sin 𝑟𝑎 𝑛3
𝐶𝐵 𝑅  To have total internal reflection inside optical fibre, the anle of incidentce at the
sin 𝑖𝐶 = = − − − (2)
𝐴𝐵 √𝑅 + 𝑑 2
2 core-cladding interface at B should be atleast critical angle (𝑖𝐶 )
 Compare eqn (1) and (2)  Appliying Snell’s law at point ‘B’
𝑅 𝑛2 sin 𝑖𝐶 𝑛2
= =
2
√𝑅 + 𝑑 2 𝑛1 sin 90° 𝑛1
𝑅2 𝑛2 2 𝑛2
(𝑜𝑟) = ( ) (𝑜𝑟) sin 𝑖𝐶 = − − − − − (2)
𝑅2 + 𝑑 2 𝑛1 𝑛1
𝑅2 + 𝑑 2 𝑛1 2  From ∆𝐴𝐵𝐶 , 𝑖𝐶 = 90° − 𝑟𝑎
(𝑜𝑟) 2
= ( )  Then eqn (2) becomes,
𝑅 𝑛2 𝑛2
𝑑2 𝑛1 2 sin(90° − 𝑟𝑎 ) =
1+ 2 = ( ) 𝑛1
𝑅 𝑛2 𝑛2
𝑑2 𝑛1 2 𝑛12 (𝑜𝑟) cos 𝑟𝑎 =
= ( ) −1= 2−1 𝑛1
𝑅 2 𝑛2 𝑛2

victory R. SARAVANAN. M.Sc., M.Phil., B.Ed PG ASST [PHYSICS], GBHSS, PARANGIPETTAI - 608 502
12 PHYSICS UNIT – 6 RAY OPTICS COMPLETE GUIDE AND MODEL QUESTION
 𝐼𝑛 ∆𝐵𝐶𝐸 ,
𝑛2 2 𝐿 𝐿
∴ s𝑖𝑛 𝑟𝑎 = √1 − cos 2 𝑟𝑎 = √1 − ( ) sin(𝑖 − 𝑟) = (𝑜𝑟) 𝐵𝐶 =
𝑛1 𝐵𝐶 sin(𝑖 − 𝑟)
𝑛12 − 𝑛22 √𝑛12 − 𝑛22  𝐼𝑛 ∆𝐵𝐶𝐹,
s𝑖𝑛 𝑟𝑎 = √ = 𝑡 𝑡
𝑛12 𝑛1 cos 𝑟 = (𝑜𝑟) 𝐵𝐶 =
𝐵𝐶 cos 𝑟
 Put this in eqn (1),  Hence,
sin 𝑖𝑎 𝑛1 𝐿 𝑡
= =
√𝑛12− 𝑛22 𝑛3 sin(𝑖 − 𝑟) cos 𝑟
( ) 𝐬𝐢𝐧(𝒊 − 𝒓)
𝑛1 𝑳 =𝒕 [ ]
sin 𝑖𝑎 1 𝐜𝐨𝐬 𝒓
=  Therfore lateral displacement depends on,
√𝑛12 − 𝑛22 𝑛3
(1) thickness of the glass slab
√𝑛12 − 𝑛22 𝑛12 − 𝑛22 (2) angle of incidence
sin 𝑖𝑎 = = √ 6. Derive equation for refraction at single spherical surface.
𝑛3 𝑛32
Refraction at single spherical surface :
𝒏𝟏𝟐 − 𝒏𝟐𝟐
𝒊𝒂 = 𝐬𝐢𝐧−𝟏 [√ ]
𝒏𝟑𝟐
 If the outer medium is air, then 𝑛3 = 1 . The acceptance angle becomes,

𝒊𝒂 = 𝐬𝐢𝐧−𝟏 (√𝒏𝟏𝟐 − 𝒏𝟐𝟐 )


 Light can have any angle of incidence from zero to 𝒊𝒂 with the normal at the end
of the optical fibre forming a conical shape called acceptance cone.
 The term (𝑛3 sin 𝑖𝑎 ) is called numerical aperture (NA) of optical fibre
𝑵𝑨 = 𝒏𝟑 𝐬𝐢𝐧 𝒊𝒂 = √𝒏𝟏𝟐 − 𝒏𝟐𝟐  Refractive index of rarer medium = 𝑛1
5. Derive the equation for lateal displacement of light passing through a glass Refractive index of spherical medium = 𝑛2
slab. Centre of curvature of spherical surface = 𝐶
Refraction through a glass slab : Point object in rarer medium = 𝑂
Point image formed in denser medium = 𝐼
 Apply Snell’s law of product form at the point N
𝑛1 sin 𝑖 = 𝑛2 sin 𝑟
 Since the angles are small, we have, sin 𝑖 ≈ 𝑖 and sin 𝑟 ≈ 𝑟
∴ 𝑛1 𝑖 = 𝑛2 𝑟 − − − − (1)
 𝐿𝑒𝑡, ∠𝑁𝑂𝑃 = 𝛼, ∠𝑁𝐶𝑃 = 𝛽, ∠𝑁𝐼𝑃 = 𝛾, then
𝑃𝑁 𝑃𝑁
tan 𝛼 = (𝑜𝑟) 𝛼 =
𝑃𝑂 𝑃𝑂
𝑃𝑁 𝑃𝑁
tan 𝛽 = (𝑜𝑟) 𝛽=
𝑃𝐶 𝑃𝐶
𝑃𝑁 𝑃𝑁
tan 𝛾 = (𝑜𝑟) 𝛾 =
 Thickness of glass slab = t 𝑃𝐼 𝑃𝐼
 From figure , 𝑖 = 𝛼 + 𝛽 and
Refractive index of glass = n
 The perpendicular distance ‘CE’between refracted ray and incident ray at C 𝛽 = 𝑟 + 𝛾 (or) 𝑟 = 𝛽 − 𝛾
gives the lateral displacement (L).
victory R. SARAVANAN. M.Sc., M.Phil., B.Ed PG ASST [PHYSICS], GBHSS, PARANGIPETTAI - 608 502
12 PHYSICS UNIT – 6 RAY OPTICS COMPLETE GUIDE AND MODEL QUESTION
 Put the values of 𝑖 and 𝑟 in eqn (1)  Adding equation (1) and (2), we get,
𝑛1 (𝛼 + 𝛽) = 𝑛2 (𝛽 − 𝛾) 𝑛2 𝑛1 𝑛1 𝑛2 𝑛2 − 𝑛1 𝑛1 − 𝑛2
− + − 1 = +
𝑛1 𝛼 + 𝑛1 𝛽 = 𝑛2 𝛽 − 𝑛2 𝛾 𝑣1 𝑢 𝑣 𝑣 𝑅1 𝑅2
(𝑜𝑟) 𝑛1 𝛼 + 𝑛2 𝛾 = 𝑛2 𝛽 − 𝑛1 𝛽 𝑛1 𝑛1 1 1
− = (𝑛2 − 𝑛1 ) [ − ]
(𝑜𝑟) 𝑛1 𝛼 + 𝑛2 𝛾 = (𝑛2 − 𝑛1 ) 𝛽 𝑣 𝑢 𝑅1 𝑅2
 Put 𝛼 , 𝛽 and 𝛾 , we have 1 1 (𝑛2 − 𝑛1 ) 1 1
𝑃𝑁 𝑃𝑁 𝑃𝑁 − = [ − ]
𝑣 𝑢 𝑛1 𝑅1 𝑅2
𝑛1 [ ] + 𝑛2 [ ] = (𝑛2 − 𝑛1 ) [ ] 𝟏 𝟏 𝒏𝟐 𝟏 𝟏
𝑃𝑂 𝑃𝐼 𝑃𝐶
𝑛1 𝑛2 𝑛2 − 𝑛1 − = ( − 𝟏) [ − ] − − − (𝟐)
(𝑜𝑟) + = 𝒗 𝒖 𝒏𝟏 𝑹𝟏 𝑹𝟐
𝑃𝑂 𝑃𝐼 𝑃𝐶  If the object is at infinity, the image is formed at the forcus of the lens. Thus, for
 Using Cartesian sign convension, we get
𝑢 = ∞ , 𝑣 = 𝑓 Then equation becomes,
𝑃𝑂 = −𝑢 ; 𝑃𝐼 = +𝑣 ; 𝑃𝐶 = +𝑅
𝑛1 𝑛2 𝑛2 − 𝑛1 1 1 𝑛2 1 1
∴ + = − = ( − 1) [ − ]
−𝑢 𝑣 𝑅 𝑓 ∞ 𝑛1 𝑅1 𝑅2
𝒏𝟐 𝒏𝟏 𝒏𝟐 − 𝒏𝟏 𝟏 𝒏𝟐 𝟏 𝟏
(𝒐𝒓) − = − − − (2) = ( − 𝟏) [ − ] − − − (𝟑)
𝒗 𝒖 𝑹 𝒇 𝒏𝟏 𝑹𝟏 𝑹𝟐
 Here rarer medium is air and hence 𝒏𝟏 = 𝟏 and let the refractive index of  Here first medium is air and hence 𝒏𝟏 = 𝟏 and let the refractive index of
second medium be 𝒏𝟐 = 𝒏 . Therefore second medium be 𝒏𝟐 = 𝒏 . Therefore
𝒏 𝟏 𝒏− 𝟏 𝟏 𝟏 𝟏
− = − − − (𝟑) = (𝒏 − 𝟏) [ − ] − −(𝟒)
𝒗 𝒖 𝑹 𝒇 𝑹𝟏 𝑹𝟐
7. Obtain Lens maker formula and metion its significance.  The above equation is called lens maker’s formula.
Lens maker’s formula :  By comparing eqn (2) and (3)
 A thin lens of refractive index 𝑛2 is placed in a medium of refractive index 𝑛1 𝟏 𝟏 𝟏
 Let 𝑅1 and 𝑅2 be radii of curvature of two spherical surfaces ① and ② − = − − − − − − (𝟓)
𝒗 𝒖 𝒇
respectively
 This equation is known as lens equation..
8. Derive the equation for thin lens and obtain its magnification.
Magnification of thin lens :

 Let P be pole of the lens and O be the Point object.


 Here 𝐼1 be the image to be formed due the refracton at the surface ① and 𝐼 be
the final image obtanined due the refracton at the surface ②  Let an object 𝑂𝑂1 is placed on the principal axis with its height perpendicular
 We know that, equation for single spherical surface to the principal axis.
𝑛2 𝑛1 𝑛2 − 𝑛1  The ray 𝑂1 𝑃 passing through the pole of the lens goes undeviated.
− =  But the ray parallel to principal axis, after refraction it passes through
𝑣 𝑢 𝑅
 For refracting surface ①, the light goes from 𝑛1 𝑡𝑜 𝑛2 . secondary focus ‘F’
 Hence  At the point of intersection of these two rays, an inverted, real image 𝐼𝐼1 is
𝑛2 𝑛1 𝑛2 − 𝑛1 formed.
− = − − − (1)
𝑣1 𝑢 𝑅1  Height of object ; 𝑂𝑂1 = ℎ
 For refracting surface ②, the light goes from 𝑛2 𝑡𝑜 1 . Hence Height of image ; 𝐼𝐼1 = 𝒉𝟏
𝑛1 𝑛2 𝑛1 − 𝑛2
− 1 = − − − (2)
𝑣 𝑣 𝑅2
victory R. SARAVANAN. M.Sc., M.Phil., B.Ed PG ASST [PHYSICS], GBHSS, PARANGIPETTAI - 608 502
12 PHYSICS UNIT – 6 RAY OPTICS COMPLETE GUIDE AND MODEL QUESTION
 The lateral magnification (m) is defined as the ration of the heiht of the image to  Adding equation (1) and (2)
that of the object. 1 1 1 1 1 1
− + − 1 = +
𝐼𝐼1 𝑣 1 𝑢 𝑣 𝑣 𝑓1 𝑓2
𝑚= − − − − (1) 𝟏 𝟏 𝟏 𝟏
𝑂𝑂1
 ∆𝑃𝑂𝑂1 and ∆𝑃𝐼𝐼1 are similar triangles . So , − = + − − − (𝟑)
𝒗 𝒖 𝒇𝟏 𝒇 𝟐
𝐼𝐼1 𝑃𝐼  If this combination acts as a single lens of focal lenth ‘F’, then, ,
= 𝟏 𝟏 𝟏
𝑂𝑂1 𝑃𝑂
 Using Cartesian sign convension, − = − − − (𝟒)
𝒗 𝒖 𝑭
−𝒉𝟏 𝑣  Compare eqn (3) and (4)
𝑚= = 𝟏 𝟏 𝟏
𝒉 −𝑢 = + − − − (𝟓)
𝒉𝟏 𝒗 𝑭 𝒇𝟏 𝒇𝟐
(𝑜𝑟) 𝒎= = − − − − (𝟐)
𝒉 𝒖  For any number of lenses,
 The magnification is negative for real image and positive for virtural image. 𝟏 𝟏 𝟏 𝟏 𝟏
= + + + +⋯
 Thus for convex lens, the magnification is negative, and for concave lens, the 𝑭 𝒇𝟏 𝒇𝟐 𝒇𝟑 𝒇𝟒
magnification is positive.  Let 𝑷𝟏 , 𝑷𝟐 , 𝑷𝟑 , 𝑷𝟒 … be the power of each lens, then the net power of the lens
 Combining the lens equation and magnification equation, we get combination,
𝒉𝟏 𝒇 𝑷 = 𝑷𝟏 + 𝑷𝟐 + 𝑷𝟑 + 𝑷𝟒 + ⋯
𝒎= =
𝒉 𝒇+ 𝒖  Let 𝒎𝟏 , 𝒎𝟐 , 𝒎𝟑 , 𝒎𝟒 … be the magnification of each lens, then the net
𝒉 𝟏
𝒇−𝒗 magnification of the lens combination,
(𝒐𝒓) 𝒎= = 𝒎 = 𝒎𝟏 × 𝒎𝟐 × 𝒎𝟑 × 𝒎𝟒 × …
𝒉 𝒇
10. Derive the equation for angle of deviation produced by af prism and thus
9. Derive the equation for effective forcal length for lenses in contact.
obtain the equation for refractive index of material of the prism.
Focal length of lenses in contact :
Angle of deviation (d) :
 Let ‘ABC’ be the section of
triangular prism.
 Here face ‘BC’ is grounded and it is
called base of the prism.
 The other two faces ‘AB’ and ‘AC’
are polished which are called
refracting faces.
 The angle between two refraction
faces is called angle of the prism ‘A’
 Let us consider two lenses ① and ② of focal lengths 𝑓1 and 𝑓2 placed co-  Here, ‘PQ’ be incident ray, ‘QR’ be
axially in contact with each other. refracted ray and ‘RS’ be emergent
 Let the object is placed at ‘O’ beyond the principal focus of ① on the principal ray.
axis.  The angle between incident ray and emergent ray is called angle of deviation
 It forms an image at 𝐼1 (d)
 This image 𝐼1 acts as an object for lens ② and hence the final image is formed  Let QN and RN be the normal drawn at the points Q and R
at ‘I’  The incident and emergent ray meet at a point M
 Writing the lens equation for lens ①  From figure, ∠ 𝑀𝑄𝑅 = 𝑑1 = 𝑖1 − 𝑟1
1 1 1 and ∠ 𝑀𝑅𝑄 = 𝑑2 = 𝑖2 − 𝑟2
− = − − − − (1)
𝑣1 𝑢 𝑓1  Then total angle of deviation,
 Writing the lens equation for lens ② 𝑑 = 𝑑1 + 𝑑2
1 1 1 𝑑 = (𝑖1 − 𝑟1 ) + (𝑖2 − 𝑟2 )
− = − − − − (2) 𝑑 = (𝑖1 + 𝑖2 ) − (𝑟1 + 𝑟2 ) − − − (1)
𝑣 𝑣1 𝑓2
victory R. SARAVANAN. M.Sc., M.Phil., B.Ed PG ASST [PHYSICS], GBHSS, PARANGIPETTAI - 608 502
12 PHYSICS UNIT – 6 RAY OPTICS COMPLETE GUIDE AND MODEL QUESTION
 In the quadrilateral AQNR, ∠𝑄 = ∠𝑅 = 90° . Hence 11. n the equation for dispersive power of a medium.
𝐴 + ∠𝑄𝑁𝑅 = 180° Dispersion :
(𝑜𝑟) 𝐴 = 180° − ∠𝑄𝑁𝑅 − − − (2)  Splitting of white light into its constituent colours is called dispersion.
 In QNR,  The coloured band obtained due to dispersion is called spectrum.
𝑟1 + 𝑟2 + ∠𝑄𝑁𝑅 = 180° Dispersive power :
𝑟1 + 𝑟2 = 180° − ∠𝑄𝑁𝑅 − − − (3)
 From eqn (2) and (3)
𝐴 = 𝑟1 + 𝑟2 − − − − (4)
 Put eqn (4) in eqn (1),
𝒅 = (𝒊𝟏 + 𝒊𝟐 ) − 𝑨 − − − −(5)
 Thus the angle of deviation depends on,
(1) the angle of incidence (𝑖1 )
(2) the angle of the prism (A)
(3) the material of the prism (n)
(4) the wavelength of the light ()  Dispersive power (𝝎) is the ability of the material of the prism to cause
Angle of minimum deviation (D) : dispersion.
 A graph is plotted between the angle of  It is defined as the ration of the angular dispersion for the extreme colours
incidence along x-axis and angle of to the deviation for any mean colour.
deviation along y-axis.  Let A be the angle of prism and D be the angle of minimum deviation, then the
 From the graph, as angle of incidence refractive index of the material of the prism is
increases, the angle of deviation decreases, 𝐴+𝐷
reaches a minimum value and then sin [ ]
n= 2
continues to increase. 𝐴
sin [ ]
 The minimum value of angled of deviation 2
is called angle of minimum deviation (D).  If the angle of the prism is small in the order of 10 then it is called small angle
 At minimum deviation, prism. In this prism, the angle of deviation also become small.
(1) 𝑖1 = 𝑖2  Let A be the angle of prism and 𝛿 be the angle of minimum deviation, then the
(2) 𝑟1 = 𝑟2 refractive index
(3) Refracted ray ‘QR’ is parallel to the base ‘BC’ of the prism. 𝐴+𝛿
sin [ ]
Refractive index of the material of the prism (n) : 2
n= − − − − (1)
𝐴
 At angle of minimum deviation, 𝑖1 = 𝑖2 = 𝑖 & 𝑟1 = 𝑟2 = 𝑟 sin [ ]
2
 Put this in equations (4) and (5)  Since A and 𝛿 are small, we may write,
𝑨 𝐴+𝛿 𝐴+𝛿
𝐴= 𝑟+ 𝑟 =2𝑟 (𝑜𝑟) 𝒓= − − − − (6) sin [ ] ≈ [ ]
𝟐 2 2
𝑨+𝑫 𝐴 𝐴
𝐷 = (𝑖 + 𝑖) − 𝐴 = 2 𝑖 − 𝐴 (𝑜𝑟) 𝒊= − − − − (7) sin [ ] ≈ [ ]
𝟐 2 2
 Then by Snell’s law ,  Put this in eqn (1),
sin 𝑖 𝐴+𝛿
n= [ ]
sin 𝑟 2 𝐴+𝛿
𝑨+𝑫 n= =
𝐬𝐢𝐧 [ ] 𝐴 𝐴
𝟐 [ ]
𝐧= − − − − (𝟖) 2
𝑨 n𝐴 = 𝐴+𝛿
𝐬𝐢𝐧 [ ]
𝟐 (or) 𝛿 = n 𝐴 − 𝐴
∴ 𝜹 = (𝐧 − 𝟏) 𝑨 − − − − − − (2)

victory R. SARAVANAN. M.Sc., M.Phil., B.Ed PG ASST [PHYSICS], GBHSS, PARANGIPETTAI - 608 502
12 PHYSICS UNIT – 6 RAY OPTICS COMPLETE GUIDE AND MODEL QUESTION
 Thus, angle of deviation for violet and red light,
𝜹𝑽 = (𝐧𝑽 − 𝟏) 𝑨 − − − − − − (3) EXAMPLE PROBLEMS WITH SOLUTIONS
𝜹𝑹 = (𝐧𝑹 − 𝟏) 𝑨 − − − − − − (4) 1. Prove that for the same incident light when a reflecting surface is tilted by an
 The angular dispersion is given by, angle θ, the reflected light will be tilted by an angle 2θ.
𝛿𝑉 − 𝛿𝑅 = (n𝑉 − 1) 𝐴 − (n𝑅 − 1) 𝐴 -Solution :-
𝛿𝑉 − 𝛿𝑅 = n𝑉 𝐴 − A − n𝑉 𝐴 + A  𝐴𝐵 − reflecting surface
𝜹𝑽 − 𝜹𝑹 = (𝐧𝑽 − 𝐧𝑹 ) 𝑨 − − − − − (5) 𝐼𝑂 − incident ray
 Let 𝜹 be the angle of deviation for mean ray (yellow) and n be the 𝑂𝑅1 − reflected ray
corresponding refractive index, then 𝑂𝑁 − normal
𝜹 = (𝐧 − 𝟏) 𝑨 − − − − − − (6) ∠𝐼𝑂𝑁 − angle of incidence (𝑖)
 By definition, dispersive power ∠𝑁𝑂𝑅1 − angle of reflection (r)
𝑎𝑛𝑔𝑢𝑙𝑎𝑟 𝑑𝑖𝑠𝑝𝑒𝑟𝑠𝑖𝑜𝑛 𝜹𝑽 − 𝜹𝑹  From law of refraction ;
𝜔= = ∠𝐼𝑂𝑁 = ∠𝑁𝑂𝑅1 = 𝑖
𝑚𝑒𝑎𝑛 𝑑𝑒𝑣𝑖𝑎𝑡𝑖𝑜𝑛 𝜹
(n𝑉 − n𝑅 ) 𝐴  When the surface AB is tilted to 𝐴1 𝐵1 by an angle θ, the normal N is also is tilted
𝜔=
(n − 1) 𝐴 to 𝑂𝑁 1 by the same angle θ
(𝐧𝑽 − 𝐧𝑹 )  Now, in the tilted system,
𝝎= − − − − − − − (𝟕) the angle of incidence ; ∠𝐼𝑂𝑁 1 = 𝑖 + 𝜃
(𝐧 − 𝟏)
 Dispersive power is a dimensionless quantity. It has no unit. It is always the angle of reflection ; ∠𝑁 1 𝑂𝑅2 = 𝑖 + 𝜃
positive.  The angle between 𝑂𝑁 1 and OR1 is ; ∠𝑁 1 𝑂𝑅1
 The angle tilted on the reflected light is the angle between OR1 and OR2 which is,
∠𝑅1 𝑂𝑅2 = ∠𝑁 1 𝑂𝑅2 − ∠𝑁 1 𝑂𝑅1
∠𝑅1 𝑂𝑅2 = (𝑖 + 𝜃) − (𝑖 − 𝜃) = 𝑖 + 𝜃 − 𝑖 + 𝜃
∠𝑹𝟏 𝑶𝑹𝟐 = 𝟐 𝜽
2. What is the height of the mirror needed for a person to see his/her image fully
on the mirror?
-Solution :-

 Let us assume a person of height h is standing in front of a vertical plane mirror.


 The person could see his/her head when light from the head falls on the mirror
and gets reflected to the eyes. Same way, light from the feet falls on the mirror
and gets reflected to the eyes.
 Let the distance between his head H and eye E is h1 and distance between his feet
F and eye E is h2. The person’s total height is ; h = h1 + h2

victory R. SARAVANAN. M.Sc., M.Phil., B.Ed PG ASST [PHYSICS], GBHSS, PARANGIPETTAI - 608 502
12 PHYSICS UNIT – 6 RAY OPTICS COMPLETE GUIDE AND MODEL QUESTION
By the law of reflection, the angle of incidence and angle of reflection are the  From figure,
same for the two extreme reflections. The normals are now the bisectors of the 𝑓
angles between the incident and the reflected rays at the two points. 𝑢1 = 𝑢 +
3
 By geometry, the height of the mirror needed is only half of the height of the 𝑓
2𝑓 = 𝑢+
person. (i.e.) 3
ℎ1 ℎ2 ℎ1 + ℎ2 𝒉 𝑓 5𝑓
+ = = (𝑜𝑟) 𝑢 = 2𝑓 − =
2 2 2 𝟐 3 3
3. An object is placed at a distance of 20.0 cm from a concave mirror of focal 𝑓
& 𝑣=𝑢+ + 𝑙1
length 15.0 cm. (a) What distance from the mirror a screen should be placed to 3
get a sharp image? (b) What is the nature of the image? 5𝑓 𝑓 𝑚𝑓 5𝑓+𝑓+𝑚𝑓 𝑓 𝑓
𝑣= + + = = (5 + 1 + 𝑚) = (6 + 𝑚)
-Solution :- 𝑓 = −15 𝑐𝑚 ; 𝑢 = −20 𝑐𝑚 ; 𝑣 = ? 3 3 3 3 3 3
1 1 1
(a) From the mirror equation,  From mirror ; = +
1 1 1 𝑓 𝑢 𝑣
= + 1 1 1
𝑓 𝑢 𝑣  For concave mirror ;
[− 𝑓]
= 5𝑓 + 𝑓
[− ] [− (6+𝑚)]
1 1 1 𝑢−𝑓 3 3
(𝑜𝑟) = − = 3 3
𝑣 𝑓 𝑢 𝑓𝑢 (𝑜𝑟) 1=+
𝑓𝑢 5 (6 + 𝑚)
(𝑜𝑟) 𝑣 = 3 3
𝑢−𝑓 (𝑜𝑟) 1− =
(−15)(−20) 300 300 5 6+𝑚
∴ 𝑣 = = = 2 3
(−20 ) − (−15) −20 + 15 −5 (𝑜𝑟) =
5 6+𝑚
𝒗 = −𝟔𝟎 𝒄𝒎 12 + 2 𝑚 = 15
 The screen is to be placed at distance 60.0 cm to the left of the concave 2 𝑚 = 15 − 12 = 3
mirror. 𝟑
(b) Magnification, 𝒎 = = 𝟏. 𝟓
𝟐
ℎ1 𝑣 (−60) 5. Pure water has refractive index 1.33. What is the speed of light through it?
𝑚= =− =− = −𝟑 -Solution : 𝑛 = 1.33 ; 𝑣 = ?
ℎ 𝑢 (−20)
𝑐
 As the sign of magnification is negative, the image is inverted. 𝑛=
𝑣
 As the magnitude of magnification is 3, the image is enlarged three times. 𝑐 3 𝑋 108 3 𝑋 108 9 𝑋 108
 As the image is formed to the left of the concave mirror, the image is real. (𝑜𝑟) 𝑣= = = =
𝑛 1.33 4 4
4. A thin rod of length f/3 is placed along the optical axis of a concave mirror of ( )
3
focal length f such that one end of image which is real and elongated just 𝒗 = 𝟐. 𝟐𝟓 𝑿𝟏𝟎𝟖 𝒎 𝒔−𝟏
touches the respective end of the rod. Calculate the longitudinal magnification.  Light travels with a speed of 2.26 × 108 m s-1 through pure water.
𝑓
-Solution :- Object length = 𝑙 = ; image length = 𝑙1 6. Light travels from air into a glass slab of thickness 50 cm and refractive index
3
 By definition, 1.5.
𝐼𝑚𝑎𝑔𝑒 𝑙𝑒𝑛𝑔𝑡ℎ (a) What is the speed of light in the glass slab?
𝐿𝑜𝑛𝑔𝑖𝑡𝑢𝑑𝑖𝑛𝑎𝑙 𝑚𝑎𝑔𝑛𝑖𝑓𝑖𝑐𝑎𝑡𝑖𝑜𝑛 = (b) What is the time taken by the light to travel through the glass slab?
𝑜𝑏𝑗𝑒𝑐𝑡 𝑙𝑒𝑛𝑔𝑡ℎ
𝑙1 𝑙1 3 𝑙1 (c) What is the optical path of the glass slab?
𝑚= = = -Solution : 𝑑 = 50 𝑐𝑚 = 50 𝑋 10−2 𝑚 ; 𝑛 = 1.5 ; 𝑣 = ? ; 𝑡 = ? ; 𝑑1 =?
𝑙 𝑓 𝑓 𝑐
3 (a) Refractive index of the medium ; 𝑛 =
𝑚𝑓 𝑣
(𝑜𝑟) 𝑙1 = −−−−−−−−− (1) 
3
 Image of one end coincides with the respective end of object. Thus, the coinciding Speed of light in the glass slab is
end must be at centre of curvature. Thus, 𝑢1 = 𝑅 = 2 𝑓 𝑐 3 𝑋 108
𝑣= = = 𝟐 𝑿𝟏𝟎𝟖 𝒎 𝒔−𝟏
𝑛 1.5

victory R. SARAVANAN. M.Sc., M.Phil., B.Ed PG ASST [PHYSICS], GBHSS, PARANGIPETTAI - 608 502
12 PHYSICS UNIT – 6 RAY OPTICS COMPLETE GUIDE AND MODEL QUESTION
(b) Let ‘t’ be the time taken by light to travel through the glass slab, (b) Critical angle,
𝑑 50 𝑋 10−2 1 1 3
𝑡= = = 25 𝑋 10−10 𝑠 = 𝟐. 𝟓 𝑿 𝟏𝟎−𝟗 𝒔 𝑖𝐶 = sin−1 [ ] = sin−1 [ ] = sin−1 [ ] = sin−1 [0.75] = 𝟒𝟖. 𝟔 °
𝑣 2 𝑋108 𝑛 4⁄ 4
(c) Optical path ; 𝑑1 = 𝑛 𝑑 = 1.5 𝑋 50 𝑋 10−2 = 𝟕𝟓 𝑿 𝟏𝟎−𝟐 𝒎 = 𝟕𝟓 𝒄𝒎 3
 Light would have travelled an additional 25 cm (75 cm – 50 cm) in vacuum (c) The total angle of view of the cone =2 𝑖𝐶 = 2 𝑋 48.6 ° = 𝟗𝟕. 𝟐 °
at the same time had there been no glass slab in its path. 10. A optical fibre is made up of a core material with refractive index 1.68 and a
7. Light travelling through transparent oil enters in to glass of refractive index cladding material of refractive index 1.44. What is the acceptance angle of the
1.5. If the refractive index of glass with respect to the oil is 1.25, what is the fibre if it is kept in air medium without any cladding?
refractive index of the oil? -Solution : 𝑛1 = 1.68 ; 𝑛2 = 1.44
-Solution : 𝑛𝑔 = 1.5 ; 𝑛𝑔𝑂 = 1.25 ; 𝑛𝑂 = ?  If there is cladding, then acceptance angle ;
𝑛𝑔
 Refractive index of glass with respect to oil ; 𝑛𝑔𝑂 =
𝑛𝑂
𝑖𝑎 = sin−1 √𝑛12 − 𝑛22
𝑛𝑔 1.5 150
(𝑜𝑟) 𝑛𝑂 = = = = 𝟏. 𝟐 𝑖𝑎 = sin−1 √1.682 − 1.442
𝑛𝑔𝑂 1.25 125 𝑖𝑎 = sin−1 √2.8224 − 2.0736
8. A coin is at the bottom of a trough containing three immiscible liquids of
𝑖𝑎 = sin−1 √0.7488
refractive indices 1.3, 1.4 and 1.5 poured one above the other of heights 30 cm,
𝑖𝑎 = sin−1 (0.8653)
16 cm, and 20 cm respectively. What is the apparent depth at which the coin
𝒊𝒂 ≈ 𝟔𝟎°
appears to be when seen from air medium outside? In which medium the coin
 If there is no cladding then, n2 = 1. Then acceptance angle
will appear?
-Solution : 𝑑1 = 30 𝑐𝑚 ; 𝑑2 = 16 𝑐𝑚 ∶ 𝑑3 = 20 𝑐𝑚 ; 𝑛1 = 1.3 ; 𝑛2 = 1.4 ∶ 𝑛3 = 1.5 𝑖𝑎 = sin−1 √𝑛12 − 1
 The equations for apparent depth for each medium is,,
𝑑1 30 𝑖𝑎 = sin−1 √1.682 − 1
𝑑11 = = = 23. 1 𝑐𝑚 𝑖𝑎 = sin−1 √2.8224 − 1 = sin−1 √1.8224
𝑛1 1.3
𝑑2 16 𝑖𝑎 = sin−1 (1.349)
𝑑21 = = = 11.4 𝑐𝑚 −1
Here sin (> 1) is not possible. But, this includes the range 0o to 90o. Hence, all the rays
𝑛2 1.4
𝑑3 20 entering the core from flat surface will undergo total internal reflection.
𝑑31 = = = 13.3 𝑐𝑚 11. The thickness of a glass slab is 0.25 m. It has a refractive index of 1.5. A ray of
𝑛3 1.5
light is incident on the surface of the slab at an angle of 60 o. Find the lateral
 Total depth of three medium,,
displacement of the light when it emerges from the other side of the glass slab.
𝑑 = 𝑑1 + 𝑑2 + 𝑑3 -Solution : 𝑡 = 0.25 𝑚 ; 𝑛 = 1.5 ; 𝑖 = 60°
𝑑 = 30 + 16 + 20 = 66 𝑐𝑚  By Snell’s law ,
 Total apparent depth of three medium,
𝑑1 = 𝑑11 + 𝑑21 + 𝑑31 = 23.1 + 11.4 + 13.3 = 𝟒𝟕. 𝟖 𝒄𝒎 √3
( )
sin 𝑖 sin 𝑖 sin 60° 2 √3 1
9. What is the radius of the illumination when seen above from inside a 𝑛= (𝑜𝑟) sin 𝑟 = = = = = = 0.58
swimming pool from a depth of 10 m on a sunny day? What is the total angle of sin 𝑟 𝑛 1.5 1.5 3 √3
view? [Given, refractive index of water is 4/3] 𝑟 = sin−1 (0.58) = 35.25°
-Solution : 𝑛=
4
; 𝑑 = 10 𝑚  Hence the lateral displacement,
3
sin(𝑖 − 𝑟)
(a) Radius of illumination, 𝐿=𝑡 [ ]
𝑑 10 10 cos 𝑟
𝑅= = = sin(60° − 35.25°) sin 24.75°
√𝑛2 −1 2 16
√(4) −1 √ −1 𝐿 = 0.25 𝑋 [ ] = 0.25 𝑋 [ ]
9
10
3
10 𝑋 3 30
cos 35.25° cos 35.25°
𝑅= = = 0.4187

16−9 √16−9 √7 𝐿 = 0.25 𝑋 [ ]
9 0.8166
𝑹 = 𝟏𝟏. 𝟑𝟒 𝒎 𝑳 = 𝟎. 𝟏𝟐𝟖𝟐 𝒎 = 𝟏𝟐. 𝟖𝟐 𝒄𝒎

victory R. SARAVANAN. M.Sc., M.Phil., B.Ed PG ASST [PHYSICS], GBHSS, PARANGIPETTAI - 608 502
12 PHYSICS UNIT – 6 RAY OPTICS COMPLETE GUIDE AND MODEL QUESTION
12. Locate the image of the point object O in the situation shown. The point C 14. Determine the focal length of the lens made up
denotes the centre of curvature of the separating surface. of a material of refractive index 1.52 as shown
-Solution : 𝑛1 = 1 ; 𝑛2 = 1.5 𝑢 = −15 𝑐𝑚 ; 𝑅 = 30 𝑐𝑚 in the diagram. (Points C1 and C2 are the centers
 Equation for single spherical surface of curvature of the first and second surfaces
is respectively.)
𝑛2 𝑛1 𝑛2 − 𝑛1 -Solution 𝑛 = 1.52 ; 𝑅1 = 10 𝑐𝑚 ; 𝑅2 = 20 𝑐𝑚
− =
𝑣 𝑢 𝑅  From lens makers formula,
1.5 1 1.5 − 1 1 1 1
− = = (𝑛 − 1) [ − ]
𝑣 − 15 30 𝑓 𝑅1 𝑅2
1.5 1 0.5
+ = 1 1 1
𝑣 15 30 = (1.52 − 1) [ − ]
1.5 0.5 1 0.5 − 2 1.5 𝑓 10 20
= − = =− 1 2−1 1
𝑣 30 15 30 30 = 0.52 𝑋 [ ] = 0.52 𝑋
1 1 𝑓 20 20
= − (𝑜𝑟) 𝒗 = − 𝟑𝟎 𝒄𝒎
𝑣 30 𝟐𝟎
 The image is a virtual image formed 30 cm to the left of the spherical surface. ∴ 𝒇= = 𝟑𝟖. 𝟒𝟔 𝒄𝒎
𝟎. 𝟓𝟐
13. A biconvex lens has radii of curvature 20 cm and 15 cm for the two curved  As the focal length is positive, the lens is a converging lens
surfaces. The refractive index of the material of the lens is 1.5. 15. If the focal length is 150 cm for a lens, what is the power of the lens?
(a) What is its focal length? -Solution : 𝑓 = 150 𝑐𝑚 = 1.5 𝑚
(b) Will the focal length change if the lens is flipped by the side?  Power of the lens,
-Solution : 𝑅1 = 20 𝑐𝑚 ; 𝑅2 = −15 𝑐𝑚 ; 𝑛 = 1.5 1 1 10
 From lens makers formula, 𝑃= = = = 𝟎. 𝟔𝟕 𝒅𝒊𝒐𝒑𝒕𝒆𝒓
𝑓 1.5 15
1 1 1
= (𝑛 − 1) [ − ]  As the power is positive, it is a converging lens.
𝑓 𝑅1 𝑅2 16. What is the focal length of the combination if the lenses of focal lengths –70 cm
1 1 1 and 150 cm are in contact? What is the power of the combination?
= (1.5 − 1) [ − ]
𝑓 20 − 15 -Solution : 𝑓1 = −70 𝑐𝑚 ; 𝑓2 = 150 𝑐𝑚
1 1 1 3+4 7 7  The focal length of the combination lens,
= 0.5 𝑋 [ + ] = 0.5 𝑋 [ ] = 0.5 𝑋 [ ] =
𝑓 20 15 60 60 120 1 1 1
𝟏𝟐𝟎 = +
𝐹 𝑓1 𝑓2
∴ 𝒇= = 𝟏𝟕. 𝟏𝟒 𝒄𝒎
𝟕 1 1 1 1 1
= + =− +
 As the focal length is positive the lens is a converging lens. 𝐹 − 70 150 70 150
 When the lens is flipped by the side;𝑅1 = 15 𝑐𝑚, 𝑅2 = −20 𝑐𝑚 ; 𝑛 = 1.5 1 − 150 + 70 80 8
= = − = −
1 1 1 𝐹 10500 10500 1050
= (𝑛 − 1) [ − ] 1050
𝑓 𝑅1 𝑅2 ∴ 𝐹=−
1 1 1 1 1 8
= (1.5 − 1) [ − ] = 0.5 𝑋 [ + ] 𝑭 = − 𝟏𝟑𝟏. 𝟐𝟓 𝒄𝒎 = −𝟏. 𝟑𝟏𝟐𝟓 𝒎
𝑓 15 − 20 20 15
 As the final focal length is negative, the combination of two lenses is a diverging
𝟏𝟐𝟎
∴ 𝒇= = 𝟏𝟕. 𝟏𝟒 𝒄𝒎 system of lenses.
𝟕  The power of the combination ,
 Thus, it is concluded that the focal length of the lens will not change if it is flipped 1
by the side. This is true for any lens. 𝑃=
𝑓
 The focal length is positive the lens is a converging lens. 1
𝑃=
− 1.3125
𝑷 = − 𝟎. 𝟕𝟔𝟏𝟔 𝑫

victory R. SARAVANAN. M.Sc., M.Phil., B.Ed PG ASST [PHYSICS], GBHSS, PARANGIPETTAI - 608 502
12 PHYSICS UNIT – 6 RAY OPTICS COMPLETE GUIDE AND MODEL QUESTION
17. An object of 5 mm height is placed at a 18. A thin biconvex lens is made up of a glass
distance of 15 cm from a convex lens of of refractive index 1.5. The two surfaces
focal length 10 cm. A second lens of focal have equal radii of curvature of 30 cm
length 5 cm is placed 40 cm from the first each. One of its surfaces is made reflecting
lens and 55 cm from the object. Find (a) by silvering it from outside. (a) What is
the position of the final image, (b) its the focal length and power of this silvered
nature and (c) its size. lens? (b) Where should an object be
-Solution :ℎ1 = 5 𝑚𝑚 = 0.5 𝑐𝑚 ; 𝑢1 = − 15 𝑐𝑚 ; 𝑓1 = 10 𝑐𝑚 ; 𝑓2 = 5 𝑐𝑚 ; 𝑑 = 40 𝑐𝑚 placed in front of this lens so that the
 For the first lens, the lens equation is, image is formed on the object itself?
1 1 1 -Solution : n = 1.5; R1 = 30 cm; R2 = –30 cm;
− = (a) By Lens makers formula,focal length of
𝑣1 𝑢1 𝑓1
1 1 1 1 1 1 1 lens ;
(𝑜𝑟) = + = + = − 1 1 1
𝑣1 𝑓1 𝑢1 10 − 15 10 15 = (𝑛 − 1) [ − ]
1 15 − 10 5 1 𝑓𝑙 𝑅1 𝑅2
= = == 1 1 1 2 1 2 1
𝑣1 150 150 30 = (1.5 − 1) [ − ] = (0.5) [ ] = [ ] =
∴ 𝒗𝟏 = 𝟑𝟎 𝒄𝒎 𝑓𝑙 30 (−30) 30 2 30 30
∴ 𝒇𝒍 = 𝟑𝟎 𝒄𝒎 = 𝟎. 𝟑 𝒎
 Equation for magnification of first lens,
ℎ2 𝑣1  And focal length of mirror ;
𝑚= = 𝑅2 −30
ℎ1 𝑢1 𝒇𝒎 = = = −15 𝑐𝑚 = −𝟎. 𝟏𝟓 𝒎
𝑣1 30 15 2 2
∴ ℎ2 = ℎ1 = 0.5 𝑋 = −  Now the focal length of the silvered lens is,
𝑢1 − 15 15 1 2 1 2 1 2 1 4 2 2
𝒉𝟐 = − 𝟏 𝒄𝒎 =[ + ]= [ + ]=[ + ]= = =
−𝑓 𝑓𝑙 − 𝑓𝑚 30 −(−15) 30 15 30 15 7.5
 As the height of the image is negative, the image is inverted and real. ∴ 𝒇 = −𝟕. 𝟓 𝒄𝒎 = −𝟎. 𝟎𝟕𝟓 𝒎
 This image acts as object for second lens. The object distance for second lenses  The silvered mirror behaves as a concave mirror with its focal length on left
𝑢2 = −(40 − 30) = −10 𝑐𝑚. For the second lens, the lens equation is side.
1 1 1  The power of the silvered lens,
− =
𝑣2 𝑢2 𝑓2 𝑃 = 2 𝑃1 + 𝑃𝑚
1 1 1 1 1 1 1 2 1 4 400 40
(𝑜𝑟) = + = + = − 𝑃= + = = = = 13.33𝐷
𝑣2 𝑓2 𝑢2 5 − 10 5 10 𝑓𝑙 − 𝑓𝑚 30 𝑋 10−2 30 3
1 10 − 5 5 1  As the power is positive it is a converging system.
= = =
𝑣2 50 50 10 Note:
∴ 𝒗𝟐 = 𝟏𝟎 𝒄𝒎  Here, we come across a silvered lens which has negative focal length and
 Let the height of the final image formed by the second lens is ℎ21 and we have positive power. Which implies that the focal length is to the left and the
height of the object for the second lens is ℎ2 . Then Equation for magnification 𝑚1 system is a converging one. Such situations are possible in silvered lenses
for the second lens is, because a silvered lens is basically a modified mirror.
ℎ21 𝑣2 (b) Here both 𝑢 and 𝑣 are sane (𝑣 = 𝑢) as the image coincides with the object.From
𝑚1 = = the mirror formula ;
ℎ2 𝑢2
𝑣2 10 1 1 1 1 1 2
𝟏
𝒉𝟐 = ℎ2 = (−1) 𝑋 = 𝟏 𝒄𝒎 = 𝟏𝟎 𝒎𝒎 = + = + =
𝑢2 (−10) 𝑓 𝑣 𝑢 𝑢 𝑢 𝑢
(a) Thus the final image is formed 10 cm to the right of the second lens. (𝑜𝑟) 𝑢 = 2𝑓 = 2(−7.5) = −15 𝑐𝑚 = −0.15𝑚
(b) As the height of the image is positive, the image is erect and real.  The object is to be placed to the left of the silvered lens.
(c) The size (i.e.) height of the final image is10 mm

victory R. SARAVANAN. M.Sc., M.Phil., B.Ed PG ASST [PHYSICS], GBHSS, PARANGIPETTAI - 608 502
12 PHYSICS UNIT – 6 RAY OPTICS COMPLETE GUIDE AND MODEL QUESTION
19. A monochromatic light is incident on an equilateral prism at an angle 30 oand
is emergent at an angle of 75o. What is the angle of deviation produced by the
EXERCISE PROBLEMS WITH SOLUTIONS
prism? 1. An object of 4 cm height is placed at 6 cm in front of a concave mirror of radius
-Solution : 𝐴 = 60° ; 𝑖1 = 30° ; 𝑖2 = 75° of curvature 24 cm. Find the position, height, magnification and nature of the
 Equation for angle of deviation, image.
𝑑 = 𝑖1 + 𝑖2 − 𝐴 -Solution : ℎ = 4 𝑐𝑚 ; 𝑅 = −24 𝑐𝑚 ; 𝑢 = −6 𝑐𝑚
𝑑 = (30° + 75°) − 60° = 105 − 60 = 𝟒𝟓 (i) Position of the image:
20. Light ray falls at normal incidence on the first  From the relation between focal length (f) and radius of curvature (R),
face and emerges gracing the second face for an 𝑅 − 24
equilateral prism. 𝑅 =2𝑓 (𝑜𝑟) 𝑓= = = −12 𝑐𝑚
2 2
(a) What is the angle of deviation produced? 1 1 1
 From mirror equation ; = +
(b) What is the refractive index of the material of 𝑓 𝑣 𝑢
the prism? 1 1 1 1 1 1 1 −1 + 2 1
∴ = − = − =− + = =
-Solution : 𝐴 = 60° ; 𝑖1 = 0° ; 𝑖2 = 90° 𝑣 𝑓 𝑢 (−12) (−6) 12 6 12 12
(a) Equation for angle of deviation, (𝑜𝑟) 𝒗 = +𝟏𝟐 𝒄𝒎
𝑑 = 𝑖1 + 𝑖2 − 𝐴 (ii) Magnificantion :
𝑑 = (0° + 90°) − 60° = 90 − 60 𝑣 12
 Magnification is given by ; 𝒎= − = − = +𝟐
𝒅 = 𝟑𝟎 𝑢 (−6)
(b) The light inside the prism must be falling on the second face at critical angle as it (iii) Height of the image:
graces the boundary. ic = 90° – 30° = 60° 𝒉𝟏
 Magnification; 𝒎 = Hence height of the image ; 𝒉𝟏 = 𝑚 ℎ = 2 𝑋 4 = 𝟖 𝒄𝒎
𝒉
 Critical angle and refractive index are related as
 Thus the imageis erect, virtual, twice the height of object formed on right
1 1
𝑛= = side of mirror
sin 𝑖𝐶 sin 60 2. An object is placed in front of a concave mirror of focal length 20 cm. The image
1 2 1
𝑛 = √3 = = 2 𝑋 0.577 [∵ = 0.577] formed is three times the size of the object. Calculate two possible distances of
( ) √3 √3
2
the object from the mirror.
𝒏 = 𝟏. 𝟏𝟓𝟒
-Solution : 𝑓 = − 20 𝑐𝑚 = − 20 𝑋 10−2 𝑚
21. The angle of minimum deviation for an equilateral prism is 37o. Find the
 From the equation of magnification,
refractive index of the material of the prism.
𝑓 𝑓
-Solution : 𝐴 = 60° ; 𝐷 = 37° 𝑚= (𝑜𝑟) 𝑢= 𝑓−
 Equation for refractive index is 𝑓−𝑢 𝑚
𝐴+𝐷  For real image, = −3 . Hence the distance of the object
sin ( ) (−20) 20 −60 − 20
𝑛= 2
𝐴 𝑢 = (−20) − = −20 − =
sin ( ) (−3) 3 3
2 𝟖𝟎
60 + 37 97 𝒖= − 𝒄𝒎
sin ( ) sin ( )
𝑛= 2 = 2 = sin(48.5) = 0.75 = 75 𝟑
60 60 sin(30) 0.5 5  For virtual image, = +3 . Hence the distance of the object
sin ( ) sin ( ) (−20) 20 −60 + 20
2 2
𝒏 = 𝟏. 𝟓 𝑢 = (−20) − = −20 + =
3 3 3
22. Find the dispersive power of a prism if the refractive indices of flint glass for 𝟒𝟎
red, green and violet colours are 1.613, 1.620 and 1.632 respectively. 𝒖= − 𝒄𝒎
𝟑
-Solution : 𝑛𝑉 = 1.632 ; 𝑛𝐺 = 1.620 ; 𝑛𝑅 = 1.613
 The dispersive power
𝑛𝑉 − 𝑛𝑅 1.632 − 1.613 0.019
𝜔= = = = 𝟎. 𝟎𝟑𝟎𝟔𝟓
𝑛𝐺 − 1 1.620 − 1 0.620

victory R. SARAVANAN. M.Sc., M.Phil., B.Ed PG ASST [PHYSICS], GBHSS, PARANGIPETTAI - 608 502
12 PHYSICS UNIT – 6 RAY OPTICS COMPLETE GUIDE AND MODEL QUESTION
6. A thin converging lens of refractive index 1.5 has a power of + 5.0 D. When this
3. A beam of light consisting of red, green and blue is lens is immersed in a liquid of refractive index n, it acts as a divergent lens of
incident on a right-angled prism as shown in figure. The focal length 100 cm. What must be the value of n?
refractive index of the material of the prism for the above -Solution : 𝑛𝑔 = 1.5 ; 𝑃𝑔 = + 5.0 𝐷 ; 𝑓 = − 100 𝑐𝑚 = − 1𝑚 ; 𝑛𝑎 = 1
red, green and blue colours are 1.39, 1.44 and 1.47  Power of lens placed in water,
respectively. What are the colours suffer total internal 1 1
reflection? 𝑃𝑙 = = = = − 1 .0 𝐷
𝑓 (− 1)
Solution : : 𝒊 = 45°; 𝑛𝑅 = 1.39 ; 𝑛𝐺 = 1.44 ; 𝑛𝐵 = 1.47  When glass lens place in air
 Condition for total internal reflection, 𝒊 > 𝒊𝒄 𝑛𝑔 1 1
𝑃𝑔 = ( − 1) [ − ] − − − − − (1)
 From Snell’s law, 𝑛1 sin 𝑖 = 𝑛2 sin 𝑟 𝑛𝑎 𝑅1 𝑅2
 When 𝒊 = 𝒊𝒄 𝒕𝒉𝒆𝒏 𝒓 = 𝟗𝟎° Hence ,  When glass lens immersed in liqiud
𝑛 𝑛𝑔 1 1
𝑛1 sin 𝑖𝑐 = 𝑛2 sin 90° (or) 𝑛1 sin 𝑖𝑐 = 𝑛2 (or) sin 𝑖𝑐 = 2 𝑃𝑙 = ( − 1) [ − ] − − − − − (2)
𝑛1 𝑛 𝑅1 𝑅2
1
 Here,, 𝑛1 = 𝑛 and 𝑛2 = 1 So, sin 𝑖𝑐 = (or) 𝑛𝑔 1 1 𝑛𝑔
𝑛 (1) 𝑃𝑔 ( − 1) [ − ] ( − 1)
1 1 𝑛𝑎 𝑅1 𝑅2 𝑛𝑎
𝑛= = = √2 = 1.414 ⇒ = 𝑛 = 𝑛
(2) 𝑃𝑙 𝑔 1 1 𝑔
sin 45° 1/√2 ( − 1) [ − ] ( − 1)
𝑛 𝑅1 𝑅2 𝑛
 Hence, 𝑛𝑅 < 𝑛 So red colour will emerge out of the prism 1.5
 But, 𝑛𝐺 > 𝑛 and 𝑛𝐵 > 𝑛 So green and blue undergo total internal reflection 5.0 ( − 1)
= 1
4. An object is placed at a certain distance from a convex lens of focal length 20 (−1.0) 1.5
( − 1)
cm. Find the object distance if the image obtained is magnified 4 times. 𝑛
-Solution : 𝑓 = 20 𝑐𝑚 = 20 𝑋 10−2 𝑚 ; 𝑚 = 4 (1.5 − 1) (0.5)
−5 = =
 If u be the object distance, then magnification, 1.5 1.5
( − 1) ( − 1)
ℎ2 𝑓 𝑓 𝑛 𝑛
𝑚= = (𝑜𝑟) 𝑓+𝑢 = 1.5 (0.5)
ℎ1 𝑓+𝑢 𝑚 −1 = = −0.1
𝑓 20 𝑛 −5
(𝑜𝑟) 𝑢= − 𝑓= − 20 = 5 − 20 = −𝟏𝟓 𝒄𝒎 1.5
𝑚 4 = −0.1 + 1 = 0.9
𝑛
5. Obtain the lens maker’s formula for a lens of 1 0.9 9 3
refractive index n2 which is separating two = = =
media of refractive indices n1 and n3 on the left 𝑛 1.5 15 5
𝟓
and right respectively. 𝒏 =
-Solution : 𝟑
7. If the distance D between an object and screen is greater than 4 times the focal
 For the refracting surface , the light goes length f of a convex lens, then there are two positions for which the lens forms
from n1 to n2, then an enlarged image and a diminished image respectively. This method is called
𝑛2 𝑛1 𝑛2 − 𝑛1
− = − − − − − (1) conjugate foci method. If d is the distance between the two positions of the lens,
𝑣1 𝑢 𝑅1 obtain the equation for focal length of the convex lens.
 For the refracting surface , the light goes from n2 to n3, then -Solution :
𝑛3 𝑛2 𝑛3 − 𝑛2
− 1= − − − − − (2)
𝑣 𝑣 𝑅1
 Adding equations (1) and (2)
𝑛2 𝑛1 𝑛3 𝑛2 𝑛2 − 𝑛1 𝑛3 − 𝑛2
− + − 1= +
𝑣1 𝑢 𝑣 𝑣 𝑅1 𝑅1
𝒏𝟑 𝒏𝟏 (𝒏𝟐 − 𝒏𝟏 ) (𝒏𝟑 − 𝒏𝟐 )
− = +
𝒗 𝒖 𝑹𝟏 𝑹𝟏

victory R. SARAVANAN. M.Sc., M.Phil., B.Ed PG ASST [PHYSICS], GBHSS, PARANGIPETTAI - 608 502
12 PHYSICS UNIT – 6 RAY OPTICS COMPLETE GUIDE AND MODEL QUESTION
 From figure, 9. A point object is placed at 20 cm from a thin plano-
𝐷 =𝑢+𝑣 − − − − − − (1) convex lens of focal length 15 cm whose plane
𝑑 =𝑣−𝑢 − − − − − − (2) surface is silvered. Locate the position and nature of
 (1) + (2) 𝐷+𝑑 =𝑢+𝑣+𝑣−𝑢 =2𝑣 the final image.
𝐷+𝑑 -Solution : : 𝑓𝑙𝑒𝑛𝑠 = 15 𝑐𝑚 ; 𝑢 = 20 𝑐𝑚
𝑣=
2  The light from’O’undergoes two refractions (1,3)
 (1) - (2) 𝐷−𝑑 = 𝑢+𝑣−𝑣+𝑢 = 2𝑢 and one reflection (2) and forms final image at ‘I’
𝐷−𝑑  Hence the equivalent power of this combination,
𝑢=
2 𝑃𝑒𝑞𝑢𝑖𝑣𝑎𝑙𝑒𝑛𝑡 = 2𝑃𝑙𝑒𝑛𝑠 + 𝑃𝑚𝑖𝑟𝑟𝑜𝑟
 If ‘f’ is the focal length of convex lens,, 1 2 1
1 1 1 1 1 𝑢+𝑣 − = +
= − = + = 𝑓𝑒𝑞𝑢𝑖𝑣𝑎𝑙𝑒𝑛𝑡 𝑓𝑐𝑜𝑛𝑣𝑒𝑥 𝑓𝑚𝑖𝑟𝑟𝑜𝑟
𝑓 𝑣 (−𝑢) 𝑣 𝑢 𝑢𝑣 1 2 1 2 1
𝑢𝑣 − = + =− [∵ = 0]
𝑓= 𝑓𝑒𝑞𝑢𝑖𝑣𝑎𝑙𝑒𝑛𝑡 15 ∞ 15 ∞
𝑢+𝑣 𝟏𝟓
 𝑃𝑢𝑡 𝑡ℎ𝑒 𝑣𝑎𝑙𝑢𝑒 𝑜𝑓 𝑢 𝑎𝑛𝑑 𝑣, ∴ 𝒇𝑒𝑞𝑢𝑖𝑣𝑎𝑙𝑒𝑛𝑡 = − 𝒄𝒎
(𝐷 + 𝑑)(𝐷 − 𝑑) 𝟐
𝐷−𝑑 𝐷+𝑑 [ ] 1 1 1
(
2
)(
2
) 4  From mirror equation; + =
𝑓= = 𝑣 𝑢 𝒇𝑒𝑞𝑢𝑖𝑣𝑎𝑙𝑒𝑛𝑡
𝐷+𝑑 𝐷−𝑑 𝐷+𝑑+𝐷−𝑑 1 1 1 2 1 2 1 −8 + 3 − 5 1
+ [ ] ∴ = − =− − =− + = = = −
2 2 2
(𝑫 + 𝒅)(𝑫 − 𝒅) 𝑣 𝒇𝑒𝑞𝑢𝑖𝑣𝑎𝑙𝑒𝑛𝑡 𝑢 15 (− 20) 15 20 60 60 12
𝒇= 𝒗 = −𝟏𝟐 𝒄𝒎
𝟒𝑫
𝑫𝟐 − 𝒅𝟐  Hence final image will form at 12 cm left side of the system.
𝒇= 10. Find the ratio of the intensities of lights with wavelengths 500 nm and 300 nm
𝟒𝑫
8. Prove that a convex mirror can only form a virtual, erect and diminished which undergo Rayleigh scattering.
image. -Solution : 𝜆1 = 500 𝑛𝑚 = 500 𝑋10−9 𝑚 ; 𝜆2 = 300 𝑛𝑚 = 300 𝑋10−9 𝑚
1
-Solution :  From Rayleigh’s scattering law, the intensity of scattered light ; 𝐼 ∝
𝜆4
1 1
 Hence, 𝐼1 ∝ and 𝐼2 ∝ 𝜆 4
𝜆14 2
 From this,
𝐼1 𝜆24
4
300 𝑋10−9 3 4 81
= 4=( ) = ( ) =
𝐼2 𝜆 1 500 𝑋10 −9 5 625
(𝑜𝑟) 𝑰𝟏 : 𝑰𝟐 = 𝟖𝟏: 𝟔𝟐𝟓
Figure (i): 11. Refractive index of material of the prism is 1.541. Find the critical angle?
-Solution : 𝒏 = 𝟏. 𝟓𝟒𝟏
 Position of object - At infinity  Let 𝑖𝐶 be the critical angle, then
 Position of image- At F, right side of convex mirror 1 1
 Size of the image - Point image sin 𝑖𝐶 = = = 0.6489
𝑛 1.541
 Nature of the image = Erect, diminished and virtual image ∴ 𝒊𝑪 = sin (0.6489) = 𝟒𝟐°𝟐𝟕
−1

Figure (ii):
 Position of object - Between pole (P) and infinity
 Position of image- Between Pole (P) and Focus (F) on right side of convex
mirror
 Size of the image - very small
 Nature of the image = Erect, diminished and virtual image
victory R. SARAVANAN. M.Sc., M.Phil., B.Ed PG ASST [PHYSICS], GBHSS, PARANGIPETTAI - 608 502
12 PHYSICS UNIT – 6 RAY OPTICS COMPLETE GUIDE AND MODEL QUESTION
5. Why is yellow light preferred to during fog?
CONCEPTUAL QUESTIONS AND ANSWERS  Yellow light is longer wavelength than green, blue or violet component of white
1. Why are dish antennas curved? light. As scattered intensity is inversely proportional to fourth power of the
 Dish antenna is curved so as it can receive parallel signal rays coming from same 𝟏
wavelength (𝑰 ∝ ), so yellow colour is least scattered and produces sufficient
direction. These parallel signal rays reflect from parabolic dish and gathered at 𝝀𝟒
its focus point where the antenna receiver is placed. illumination.
 This increases directivity of antenna and the strength of signal received is  Moreover the sensitivity of human eye is more for yellow light which improves
maximum. The parabolic dish antenna was invented by Henrich Hertz. the visibility.
2. What type of lens is formed by a bubble inside water? 6. An object placed between two plane mirrors inclined at angle 𝜽 with each
 Air bubble has spherical shape and is surrounded by medium (water) of higher other. What is the total number of images formed?
refractive index.  If angle 𝜽 is a submultiple of 180 , then the number of images formed is
𝟑𝟔𝟎
 When light passes from water to air it get diverged. So air bubble in water 𝒏= 𝜽
− 𝟏
behaves as a diverging lens (i.e.) concave lens  If angle 𝜽 is not a submultiple of 180 , then the number of images (𝒏) formed is
3. Is it possible for two lenses to produce zero power? 𝟑𝟔𝟎
 The power of the combination of two lenses is given by the sum of the individual next higher than ( − 𝟏)
𝜽
powers of the lenses. 7. Two concave mirrors have the same focal length but the aperture of one is
 If one lens is converging (convex) lens with focal length ‘𝑓’ , so that its power is larger than that of the other. Which mirror forms the sharper image and why?
𝑃1 = +𝑃 and the other lens is diverging (concave) lens with same focal length  The concanve mirror with smaller aperture forms the sharper image because it
but with a negative sign ,so that its power is 𝑃2 = − 𝑃 is free from spherical aberration.
 So the total power of this combination of these two lenses will be zero 8. How will you distinguish between a plane, a concave and a convex mirror
𝑃𝑡𝑜𝑡𝑎𝑙 = 𝑃1 + 𝑃2 = 𝑃 + (−𝑃) = 0 without touching its surface?
4. A biconvex lens has focal length f and intensity of  We see our own face in the mirror and note the magnification of the image.
light I passing through it. What will be the focal length  If magnification is 1, mirror is plane
and intensity for portions of lenses obtained by cutting If magnification is more than 1, mirror is concave
it vertically and horizontally as shown in figure? If magnification is less than 1, mirror is convex
1 1 1
 The lens maker’s formula is, = (𝑛 − 1) [ − ] 9. What is the advantage of using a parabolic concave mirror (as compared to a
𝑓 𝑅1 𝑅2
convex lens) as objective of a telescope?
(a) For biconvex lens shown in (a), the radii 𝑅1 = + 𝑅 and 𝑅2 = −𝑅
1 1 1 2  Parabolic concave mirror is free spherical and chromatic aberrations
= (𝑛 − 1) [ + ] = (𝑛 − 1) 10. Why is a concave mirror preferred to a plane mirror for shaving and dental
𝑓 𝑅 𝑅 𝑅
(b) If the lens is cut vertically as shown in (b), then 𝑅1 = ∞ and 𝑅2 = − 𝑅 doctors?
1 1 1 1 1
(𝑛 − 1) [ + ] = (𝑛 − 1) [0 + ] = (𝑛 − 1) = (𝑛 − 1)
2 1  Because concave mirror forms a magnified and erect image of face when it is
𝑣 = =
𝑓 ∞ 𝑅 𝑅 𝑅 2𝑅 2𝑓 held closer to the face or teeth.
∴ 𝒇𝒗 = 𝟐 𝒇 (𝒊. 𝒆. ) 𝒇𝒐𝒄𝒂𝒍 𝒍𝒆𝒏𝒈𝒕𝒉 𝒘𝒊𝒍𝒍 𝒃𝒆 𝒅𝒐𝒖𝒃𝒍𝒆𝒅 11. For the same angle of incidence the angle of refraction in three different
(c) If the lens is cut horizontally as shown in (c), then 𝑅1 = + 𝑅 and 𝑅2 = − 𝑅 media A, B and C are 15 , 25 and 35 respectively. In which medium will the
1 1 1 2 1
ℎ =
(𝑛 − 1) [ + ] = (𝑛 − 1) = (𝑜𝑟) 𝒇𝒉 = 𝒇 velocity of light be minimum?
𝑓 𝑅 𝑅 𝑅 𝑓
𝐬𝐢𝐧 𝒊 𝒄 𝐬𝐢𝐧 𝒓
(𝒊. 𝒆. ) 𝒏𝒐 𝒄𝒉𝒂𝒏𝒈𝒆 𝒊𝒏 𝒇𝒐𝒄𝒂𝒍 𝒍𝒆𝒏𝒈𝒕𝒉  By Snell’s law, 𝝁 = =𝒗 (or) 𝒗 = 𝒄
𝐬𝐢𝐧 𝒓 𝐬𝐢𝐧 𝒊
 We know that the intensity of image formed by the lens is proportional to area  Here ‘c’ is constant and for given angle of incidence( 𝒊), 𝒗 ∝ 𝒔𝒊𝒏 𝒓
exposed to incident light of object (i.e.) 𝒊𝒏𝒕𝒆𝒏𝒔𝒊𝒕𝒚 ∝ 𝒂𝒓𝒆𝒂 𝒐𝒇 𝒕𝒉𝒆 𝒂𝒑𝒆𝒓𝒕𝒖𝒓𝒆  Hence 𝑣𝐴 ∝ 𝑠𝑖𝑛 15 , 𝑣𝐵 ∝ 𝑠𝑖𝑛 25, 𝑣𝐶 ∝ 𝑠𝑖𝑛 35
(a) For biconvex lens shown in (a), 𝑰 ∝ 𝝅 𝒓𝟐  But 𝑠𝑖𝑛 15 < 𝑠𝑖𝑛 25 < 𝑠𝑖𝑛 35 . ∴ 𝒗𝑨 < 𝒗𝑩 < 𝒗𝑪
(b) If the lens is cut vertically as shown in (b), there is no change in area of the  (i.e.) Velocity of light is minimum in medium ‘A’
aperture and hence intensity remains the same (i.e.) 𝑰𝒗 = 𝑰 12. Do the frequency and wavelength change when light passes from a rarer to a
(c) If the lens is cut horizontally as shown in (c), then the area of aperture denser medium?
𝝅 𝒓𝟐 𝑰
becomes halved. So the intensity becomes, 𝑰𝒉 ∝ ∝  When light passes from a rarer to a denser medium, wavelength of light changes
𝟐 𝟐
(i.e.) intensity will be halved but frequency remains unchanged.

victory R. SARAVANAN. M.Sc., M.Phil., B.Ed PG ASST [PHYSICS], GBHSS, PARANGIPETTAI - 608 502
12 PHYSICS UNIT – 6 RAY OPTICS COMPLETE GUIDE AND MODEL QUESTION
13. The critical angle for glass - air interface is 𝒊𝑪 . Will the critical angle for glass - 22. A convex lens forms the image of the Sun at a distance of 20 cm. Where will be
water interface be greater than or less than 𝒊𝑪 ? the image be formed when another lens of the same aperture but double the
𝑛 1 power is used?
 For glass - air interface, sin 𝑖𝐶 = 𝑎 = 𝑎 1 1
𝑛𝑔 𝑛𝑔
𝑛𝑤 1
 Here the power of first lens is ; 𝑃 = = −2 = 5 𝑑𝑖𝑜𝑝𝑡𝑒𝑟
𝑓 20 𝑋 10
 Let 𝑖𝐶1 be the critical angle for glass - water interface, then sin 𝑖𝐶1 = = 𝑤𝑛  If this lens is replace by another lens having double the power (i.e.) 10 diopter,
𝑛𝑔 𝑔
1
 Here, 𝑤 𝑛𝑔 < 𝑎𝑛𝑔 . Therefore, sin 𝑖𝐶1 > sin 𝑖𝐶 (or) 𝑖𝐶1 > 𝑖𝐶 its focal length 𝑓 1 = = 0.1 𝑚 = 𝟏𝟎 𝒄𝒎
10
 (i.e.) The critical angle for glass-water interface is greater than for glass-air  Hence the image will be formed at 10 cm from the second lens.
interface. 23. What happens to a focal length and power of a convex lens, when it is
14. An air bubble in a jar of water shines brightly. Why? immersed in water?
 Light entering water is totally reflected from the air bubble. For the observer, 𝟏 𝟏 𝟏
 From lens makers formula, = (𝒏 − 𝟏) [ − ]
this light is appears to come from the bubble. So it shines brightly. 𝒇 𝑹𝟏 𝑹𝟐
𝟏
15. Why does a dimond sparkle?  Thus, 𝒇 ∝ As 𝑤
𝑛𝑔 < 𝑎 𝑤
𝑛𝑔 , we have 𝑓𝑔 > 𝑎
𝑓𝑔
(𝒏−𝟏)
 Refractive index of dimond is large (𝑛 = 2.42), so its critical angle is small
(𝑖𝐶 = 24.4°).  So focal length of the convex lens will increase when it is immersed in water
 The faces of diamond are cut suitably so that light entering it suffers total  Since power is reciprocal of the focal length, power of the glass lens will
internal reflections repeatedly and get collected inside, but it comes out through decrease when it is immersed in water.
only a few faces. 24. The focal length of an equi convex lens is equal to the radius of curvature of
 Diamond sparkles when seen in the direction of emerging light either face of the lens, then what is the refractive index of the lens material?
 Given that, 𝑅1 = +𝑅, 𝑅2 = − 𝑅, 𝑓 = 𝑅
16. During summer noon, why do the trees and houses on the other side of an open 𝟏 𝟏 𝟏
ground appear to be shaking?  From lens makers formula, = (𝒏 − 𝟏) [ − ]
𝒇 𝑹𝟏 𝑹𝟐
 Open ground becomes very hot during a summer noon. It heats up the air in 1 1 1 1 1 2
= (𝑛 − 1) [ − ] = (𝑛 − 1) [ + ] = (𝑛 − 1) 𝑅
contact. So convention currents are set up in air. 𝑅 𝑅 (−𝑅) 𝑅 𝑅
1 1 3
 Light rays passing through this air change their path due to reflection. This gives (𝑜𝑟) 2(𝑛 − 1) = 1 (𝑜𝑟) 𝑛−1= (𝑜𝑟) 𝒏= + 1 = = 𝟏. 𝟓
shaking appearance to the objects from which these light rays start. 2 2 2
25. A ray of light after refraction through a concave lens
17. If a plane glass slab is placed on letters of different colours, then red coloured
emerges parallel to the principal axis. Under what
letters appears more raise up. Why?
𝟏 condition can it happen?
 The apparent shift caused by a slab of thickness ‘t’ is given by ; 𝒅 = 𝒕 (𝟏 − )  This will happen when the incident ray is directed
𝒏
 As refractive index for red light is maximum , red coloured letters are more towards the focus of the concave lens as shown.
raised up. 26. In figure the line AB represents a lens. State whether
18. When does a convex lens behave as a concave lens? it is convex or concave?
 When a convex lens is placed inside a transparent medium of refractive index  The lens is concave, because the rays become more
greater than that of its own material, it behaves as a concave lens divergent after refraction through the lens.
19. A lens immersed in a transparent liquid is not visible. Under what condition 27. A ray of light is incident normally on one face of an equilateral prism. Trace
this can happens? the path of the ray through the prism and emerging from it?
 When the refractive index of the liquid is same as the lens material, no light will  When llight incident normally on first face, the anle of
be reflected by the lens and hence it will not be visible incidence will be, 𝑖 = 0°. So the angle of refraction at
20. What is the focal length and power of a rectangular glass slab? this face will be 𝑟 = 0°.
 Focal length of a glass slab = ∞  Then the refracted ray through the prism incident on
𝟏 second face at an angle of incidence 𝑖 = 60°
 Power of a glass slab = =𝟎
∞  The critical angle for glass is 𝑖𝐶 = 42°
21. Sun glasses have curved surfaces, but they do not have any power. Why?  Here light travels from denser (glass) to rarer (air)
 Both the surfaces of Sun glasses are equally curved (i.e.) 𝑅1 = 𝑅2 and hence its medium and the angle of indence at second face is
𝟏 𝟏 𝟏
power 𝑷 = = (𝒏 − 𝟏) [ − ] = 𝟎 greater than the critical angle. So total internal reflection occurs and the
𝒇 𝑹 𝑹
𝟏 𝟐 reflected light emerge from third face of the prism as shown.

victory R. SARAVANAN. M.Sc., M.Phil., B.Ed PG ASST [PHYSICS], GBHSS, PARANGIPETTAI - 608 502
12 PHYSICS UNIT – 6 RAY OPTICS COMPLETE GUIDE AND MODEL QUESTION
28. What will be the colour of the sky in the absence of atmosphere? 36. Distinguish real and virtual images?
 The Sun light will not scattered in the absence of atmosphere. So the sky will Real image Virtual image
appear dark. 1) Here the rays actually meet at the Here the rays appear to diverge from
 Moon has no atmosphere. So there is no scattering of Sun light. Thus the sky image point the image point
appears dark when we look on the moon. 2) It can be taken on a screen It cannot be taken on a screen
29. Eye is most sensitive to yellow colour. Why do then we use traffic light stop 3) It is always inverted It is always erect
signals of red colour? 37. A straight rod appears bent in water. Why?
 Accoring to Raleigh’s scattering law, the intensity of scattered light is inversely  AB is a straight rod which partially immersed
proportional to the fourth power of the wavelength. (i.e.) shortest wavelength in water.
scattered much more than the longest wavelength  Two rays starting from end B of the rod suffer
 In visible spectrum, red colour has largest wavelength and hence it is least refraction at the surface of water.
scattered. So it can be easily observed even in foggy and dust conditions.  Here these two refracted rays are appears to
30. Why do sometimes we observe holes (rings) round the Sun or the Moon? diverge from point 𝐵1 .So 𝐶𝐵1 is the virtual
 When the Sun or the Moon is seen through a thin veil of high clouds, holes are image of 𝐶𝐵
seen. These are formed due to reflection of light by the icy crystals present in the  Consequently, the rod appears to be bent at
atmosphere. point ‘C’
31. How is rainbow formed in the sky? 38. An empty test tube dipped into water in a beaker appears silvery, when viewed
 Rainbow is formed by dispersion of Sunlight into its constituent colours by rain from a suitable direction. Why?
drops which disperse Sunlight by refraction and deviate the colours by total  An empty test tube immersed in water appears
internal reflection. silvery white when viewed from top. It may happen
32. Why is the sequence of colours in the secondary rainbow reverse of that in the that for the light rays passing from water (denser
primary rainbow? Also why the secondary rainbow is fainter than the primary medium) to air inside the tube (rarer medium), the
rainbow? angle of incidence is greater than the critical angle.
 This is because a secondary rainbow is formed by two internal reflections of light  Such light rays will suffer total internal reflection.
in water droplets while a primary rainbow is formed by just one total internal Due to this, the tube will give silvery appearance.
reflection. 39. Explain the twinkling of stars. Why do the planets not show twinkling effect?
 Since Sun light suffers two total internal reflections in the formation of  The light from stars undergoes refraction continuously before it reaches earth.
secondary rainbow, more of light intensity is absorbed and hence it looks as So the apparent position of the star is slightly different than its actual
fainter position.Dut to variation in atmospheric conditions like change in temperature,
33. The size of the Moon is much smaller than the Stars,still it appears bigger.Why? density etc., this apparent position keeps on changing. So the amount of light
 The apparent size of an object as seen by our eyes depends on the visual angle. entering our eyes from particular star increases and decreases randomly with
 The Moon is much closer to the earth than the stars. Hence Moon subtends a time. (i.e.) sometimes the star appears brighter and other times it appears
great visual angle at our eyes and appears bigger than the stars. fainter. This gives rise to the twinkling effect of stars.
34. The Sun appears to be a small disc, even though its diameter is 𝟏𝟎𝟗 𝒎. Why?  As the planets are much closer to the earth, the amout of light received from
 The Sun is at a large distance of 𝟏𝟎𝟏𝟏 𝒎 from the earth. It subtends a very small them is much greater and the fluctuations caused in the amount of light due to
visual angle of about 𝟏𝟎−𝟐 𝒓𝒂𝒅 at our eyes. atmospheric refraction are negligible as compared to the amount of light
 An equal angle is subtended by the disc of 1 cm diameter placed at a distance of 1 received from them. So the planets do not show twinkling effect.
m from our eyes. So the Sun appears to be a small disc. 40. Only the stars near the horizon twinkle while those overhead do not twinkle.
35. Bees can see objects in the ultra-violet light while human beings cannot do so. Why?
Why?  Light from the star near the horizon reaches the earth obliquely through the
 Ultra-violet light has wavelength shorter than that of violet light. atmosphere. Its path changes due to refraction. Frequent atmospheric
 Bees have some retinal cones that are sensitive to ultra-violet light, so they can disturbances change the path of light and cause twinkling of star.
see objects in ultra-violet light  But light from the stars over head reaches the earth normally, it does not suffer
 But human eyes do not possess retinal cones sensitive to ultra-violet light and so refraction. There is no change in its path. Hence there is no twinkling effect.
human beings are ultra-violet blind

victory R. SARAVANAN. M.Sc., M.Phil., B.Ed PG ASST [PHYSICS], GBHSS, PARANGIPETTAI - 608 502
12 PHYSICS UNIT – 6 RAY OPTICS COMPLETE GUIDE AND MODEL QUESTION
41. What change in the focal length of a (a) concave mirror and (b) convex lens 46. The image of a candle is formed by a convex lens on a screen. The lower half of
occur, when the incident violet light is replaced with red light? the lens is painted black to make it completely opaque. How will this image be
𝑹 different from the one obtained when the lens is not painted black?
a) For concave mirror, focal length is 𝒇 = and it is independent of wavelength of
𝟐  When the lower half of the lens is painted black, the image formed is still of the
light. So focal length does not change when violet light (shorter wavelength) is same size as that with unpainted lens but it has now reduced intensity
replaced by red light (longer wavelength)
𝟏
b) But for a convex lens, the focal length, ∝ . Since 𝜆𝑣 < 𝜆𝑟 then 𝑛𝑣 > 𝑛𝑟 . So
(𝒏−𝟏)
𝑓𝑣 < 𝑓𝑟 (i.e.) focal length increases when violet light (shorter wavelength) is
replaced by red light (longer wavelength)
42. Give the reasons for the following observations on the surface of moon.
a) Sun rise and Sun set are abrupt
b) Sky appears as dark
c) A rainbow is never formed 47. When light travels from rarer to denser medium the speed decreases. Does
a) Moon has no atmosphere. There is no scattering of light. Sun light reaches moon this decrease in speed imply a decrease in energy carried by the light wave?
straight covering shortest distance. Hence Sunrise and Sunset are abrupt.  No. The energy carried by the light wave remains the same.
b) Moon has no atmosphere. So there is nothing to scatter Sunlight towards the  Because when light travels from one medium to another medium its speed and
moon and no skylight reaches moon surface. Thus sky appears black in the day wavelength are changed, but its frequency remains constant.
time as it does at night.  Since the frequency that does not change when light travel from one medium to
c) No water vapours are present in moon surface. No clouds are formed. There are another, the energy 𝑬 = 𝒉𝝂 remains the same
no rains on the moon. So rainbow is never observed. 48. How would a biconvex lens appear when placed in a trough of liquid having
43. A glass prism causes dispersion, while a glass plate does not. Why? the same refractive index as that of the lens?
 When white light passes through a prism, it splits up in to its constituent colours,  A biconvex lens appears plane glass sheet when placed in a trough of liquid
because refractive index of glass is different for different colours. Here having the same refractive index as that of the lens
Angular dispersion = (𝜇𝑉 − 𝜇𝑅 ) 𝐴 49. Two thin lenses of power −𝟐 𝑫 and +𝟐 𝑫 are placed in contact coaxially. What
and Angular deviation = (𝜇 − 1 )𝐴 is the focal length of the combination?
 But in case of a glass plate both faces are parallel and hence 𝐴 = 0  Total power of this combination ; 𝑷 = −𝟐 𝑫 + 𝟐𝑫 = 𝟎
𝟏 𝟏
So there will be neither dispersion nor deviation. All the emergent rays will be  Thus the focal length of this combination ; 𝒇 = = = ∞ (𝒊𝒏𝒇𝒊𝒏𝒊𝒕𝒚)
𝑷 𝟎
parallel to the incident ray.
50. A convex lens is placed in contact with a plane mirror. A point object at a
44. A beam of white light on passing through a hollow prism gives no spectrum.
distance 20 cm on the axis of this combination has its image coinciding with
Why?
itself. What is the focal length of the lens?
 In order that the reflected rays
from plane mirror trace the path of
incident ray, the rays incident on it
must be normal to its surface.
 Hence these reflected rays after
refraction through the convex lens
 A hollow prism contains air which does not cause dispersion. The two faces of forms image on the object itself,
the hollow prism behaves like parallel sides of glass plates. The beam is laterally thus object and image overlapping
deviated at each of the two refracting faces. However the rays of different colours each other at focus of convex lens. Thus focal length of lens = 20 cm
emerge parallel to each other .So thereis no dispersion.
45. Dispersion is caused by refraction not by reflection. Why?
 Because, for given angle of incidence, the angle of reflection is same for all the
wavelengths of white light while the angle of refraction is different for different
wavelengths.

victory R. SARAVANAN. M.Sc., M.Phil., B.Ed PG ASST [PHYSICS], GBHSS, PARANGIPETTAI - 608 502
12 PHYSICS UNIT – 6 RAY OPTICS COMPLETE GUIDE AND MODEL QUESTION
EXAM NO 9. An air bubble in glass slab of refractive index 1.5 (near normal incidence) is 5
NAME : cm deep when viewed from one surface and 3 cm deep when viewed from the
UNIT – 6 RAY OPTICS opposite face. The thickness of the slab is,
(a) 8 cm (b) 10 cm
Time - 2 : 15 hours Total - 50 marks (c) 12 cm (d) 16 cm
PART - I 10 X 1 = 10 10. A ray of light travelling in a transparent medium of refractive index n falls, on a
Note : (i) Answer all the questions surface separating the medium from air at an angle of incidents of 45 o. The ray
(ii) Choose the best answer and write the option code and can undergo total internal reflection for the following n,
corresponding answer (a) n = 1.25 (b) n = 1.33
1. The speed of light in an isotropic medium depends on, (c) n = 1.4 (d) n = 1.5
(a) its intensity (b) its wavelength
(c) the nature of propagation (d) the motion of the source w.r.t medium PART - II 5 X 2 = 10
2. A rod of length 10 cm lies along the principal axis of a concave mirror of focal
length 10 cm in such a way that its end closer to the pole is 20 cm away from Note : (i) Answer any 5 of the following questions .
the mirror. The length of the image is, (ii) Question No. 17 is compulsory
(a) 2.5 cm (b) 5 cm 11. State the laws of reflection.
(c) 10 cm (d) 15 cm 12. State the laws of refraction (snell’s law)
3. An object is placed in front of a convex mirror of focal length of f and the 13. Define optical path.
maximum and minimum distance of an object from the mirror such that the 14. What are the conditions to achieve total internal reflection.
image formed is real and magnified. 15. Define dispersive power.
(a) 2f and c (b) c and ∞ 16. Why does sky and Sun looks reddish during Sun set and Sun rise?
(c) f and O (d) None of these 17. Find the dispersive power of a prism if the refractive indices of flint glass for red,
4. For light incident from air on a slab of refractive index 2, the maximum green and violet colours are 1.613, 1.620 and 1.632 respectively.
possible angle of refraction is, PART - III 5 X 3 = 15
(a) 30o (b) 45o
(c) 60 o (d) 90o Note : (i) Answer any 5 of the following questions .
5. If the velocity and wavelength of light in air is Va and λa and that in water is (ii) Question No. 24 is compulsory
Vw and λw, then the refractive index of water is, 18. What are the characteristics of images formed by plane mirror?
𝑉𝑤 𝑉𝑎 19. Obtain the relation between focal length and the radius of curvature of the spherical
(a) (b) mirror?
𝑉𝑎 𝑉𝑤
(c)
𝜆𝑤 𝑉 𝜆
(d) 𝑎 𝑎 20. Obtain the equation for apparent depth.
𝜆𝑎 𝑉𝑤 𝜆𝑤 21. Obtain the expression for critical angle.
6. Stars twinkle due to, 22. How are rainbows are formed?
(a) reflection (b) total internal reflection 23. Prove that for the same incident light when a reflecting surface is tilted by an
(c) refraction (d) polarisation angle , the reflected light will be tilted by an angle 2
7. When a biconvex lens of glass having refractive index 1.47 is dipped in a 24. Light travels from air into a glass slab of thickness 50 cm and refractive
liquid, it acts as a plane sheet of glass. This implies that the liquid must have index 1.5.
refractive index, (a) What is the speed of light in the glass slab?
(a) less than one (b) less than that of glass (b) What is the time taken by the light to travel through the glass slab?
(c) greater than that of glass (d) equal to that of glass (c) What is the optical path of the glass slab?
8. The radius of curvature of curved surface at a thin planoconvex lens is 10 cm
and the refractive index is 1.5. If the plane surface is silvered, then the focal
length will be,
(a) 5 cm (b) 10 cm
(c) 15 cm (d) 20 cm

victory R. SARAVANAN. M.Sc., M.Phil., B.Ed PG ASST [PHYSICS], GBHSS, PARANGIPETTAI - 608 502
12 PHYSICS UNIT – 6 RAY OPTICS COMPLETE GUIDE AND MODEL QUESTION
PART - IV 3 X 5 = 15
Note : (i) Answer all the questions
25. Derive the mirror equation and the equation for lateral magnification.
(OR)
Describe the Fizeau’s method to determine the speed of light. அன்னையும் பிதாவும் முன்ைறி ததய்வம்
26. Derive equation for refraction at single spherical surface தாய், தந்னத கண்கண்ட ததய்வம்
(OR) ஈயார் ததட்னட தீயார் தகாள்வர்
Obtain Len’s makers formula and mention its significance பிறருக்கு உதவி தெய்யாததார் தபாருனைத் தீயவர் பறித்துக்
27. Derive the equation for thin lens and obtain its magnification தகாள்வார்
(OR) ஏவா மக்கள் மூவா மருந்து
What is dispersion? Obtain the equation for dispersive power of a medium குறிப்பறிந்து தெய்யும் பிள்னைகள் அமிர்தம் தபான்றவர்கள்
குற்றம் பார்க்கின் சுற்றம் இல்னை
பிறர் குற்றங்கனை ஆராய்ந்து தகாண்டிருந்தால், சுற்றத்தார்
எவரும் இருக்க மாட்டார்கள்
னகப்தபாருள் தன்னில் தமய்ப்தபாருள் கல்வி
னகயில் உள்ை தபாருனைவிட, உண்னமயாை தெல்வம்
கல்விதய ஆகும்
சீனரத்ததடின் ஏனரத் ததடு
புகத ாடு வா விரும்பிைால், பயிர்ததாழிலில் ஈடுபட
தவண்டும்
ததாழுது ஊண் சுனவயின் உழுது ஊண் இனிது
பிறரிடம் வணங்கி வரும் ஊதியத்தில் உண்பனத விட பயிர்
தெய்து உண்பது இனிது
முற்பகல் தெய்யின் பிற்பகல் வினையும்
பிறருக்கு தெய்யும் நன்னம, தீனமகள் பின்பு நமக்தக வந்து
தெரும்
தமழிச்தெல்வம் தகான ப் படாது
கைப்னபயால் உன த்து தெர்த்த தெல்வம், ஒருதபாதும்
வீண்தபாகாது
தமாழிவது மறுக்கின், அழிவது கருமம்
தபரிதயார் தொல் தகைாமல் மறுத்தால், அந்த காரியம் தகடும்
ஐயம் புகினும் தெய்வை தெய்
பிச்னெ எடுத்தாவது தெய்யதவண்டிய நல்ை காரியங்கனை
தெய்
ஊக்கம் உனடனம, ஆக்கத்திற்கு அ கு
உற்ொகமாை முயற்சியுடன் இருப்பதத முன்தைற்றத்திற்கு
அ கு
- தகான்னற தவந்தன் - ஔனவயார்

தமிழ் - மகாகவி சுப்பிரமண்ய பாரதியார்


victory R. SARAVANAN. M.Sc., M.Phil., B.Ed PG ASST [PHYSICS], GBHSS, PARANGIPETTAI - 608 502
பசித்திரு (Be hungry) தனித்திரு (Be individual) விழித்திரு (Be conscious)

HIGHER SECONDARY SECOND YEAR-PHYSICS

NAME :
STANDARD : 12 SECTION :
SCHOOL :
EXAM NO :

victory R. SARAVANAN. M.Sc, M.Phil, B.Ed.,


PG ASST (PHYSICS)
GBHSS, PARANGIPETTAI - 608 502
12 PHYSICS UNIT – 6 RAY OPTICS COMPLETE GUIDE AND MODEL QUESTION

PART – I 1 MARK MULTIPLE CHOICE QUESTIONS & ANSWERS 4. Two coherent monochromatic light beams of intensities I and 4I are
1. A plane glass is placed over a various coloured letters (violet, green, yellow, superposed. The maximum and minimum possible intensities in the resulting
red) The letter which appears to be raised more is, beam are
(a) red (a) 5I and I
(b) yellow (b) 5I and 3I
(c) green (c) 9I and I
(d) violet (d) 9I and 3I
Solution :- Solution :-
1  Maximum intensity ; 𝐼𝑚𝑎𝑥 = 𝐼1 + 𝐼2 + 2 √𝐼1 𝐼2
 Shift is given by ; 𝑠 = 𝑑 (1 − )
𝑛
 Thus, if refractive index ‘𝑛 ‘ increases, shift ‘s’ also increases. 𝐼𝑚𝑎𝑥 = 𝐼 + 4 𝐼 + 2 √𝐼 𝑋4 𝐼 = 𝐼 + 4 𝐼 + 4 𝐼 = 9𝐼
 Since , 𝑛𝑣 > 𝑛𝑔 > 𝑛𝑦 > 𝑛𝑟 we have 𝑠𝑣 > 𝑠𝑔 > 𝑠𝑦 > 𝑠𝑟  Minimum intensity ; 𝐼𝑚𝑖𝑛 = 𝐼1 + 𝐼2 − 2 √𝐼1 𝐼2
Answer (d) violet 𝐼𝑚𝑎𝑥 = 𝐼 + 4 𝐼 − 2 √𝐼 𝑋4 𝐼 = 𝐼 + 4 𝐼 − 4 𝐼 = 𝐼
2. Two point white dots are 1 mm apart on a black paper. They are viewed by eye Answer (c) 9I and I
of pupil diameter 3 mm approximately. The maximum distance at which these 5. When light is incident on a soap film of thickness 5×10–5 cm, the wavelength of
dots can be resolved by the eye is, [take wavelength of light, λ = 500 nm] light reflected maximum in the visible region is 5320 Å. Refractive index of the
(a) 1 m film will be,
(b) 5 m (a) 1.22
(c) 3 m (b) 1.33
(d) 6m (c) 1.51
Solution :- (d) 1.83
1.22 𝜆 𝑢 Solution :-
 Resolving power of eye lens; 𝑑𝑚𝑖𝑛 =
𝑎 𝜆
 Thus the maximum distance between the two points which can be just resolved,  Condition for maximum intensity in thin film ; 2 𝜇 𝑡 = (2𝑛 − 1)
2
𝑎 𝑑𝑚𝑖𝑛 3𝑋 10−3 𝑋 1 𝑋 10−3  For visible region, 𝑛 = 3
𝑢= = =5𝑚
1.22 𝜆 1.22 𝑋 500 𝑋 10−9 𝜆 5𝜆 5 𝑋 5320 𝑋 10−10 5320 𝑋 10−3
Answer (d) 5 m ∴ 𝜇 = (2𝑛 − 1) 4𝑡
= 4𝑡
= 4 𝑋 5 𝑋 10−5 𝑋10−2 == 4
= 1.33
3. In a Young’s double-slit experiment, the slit separation is doubled. To maintain Answer (b) 1.33
the same fringe spacing on the screen, the screen-to-slit distance D must be 6. First diffraction minimum due to a single slit of width 1.0×10 –5 cm is at 30o.
changed to, Then wavelength of light used is,
(a) 2D (a) 400 Å
𝐷 (b) 500 Å
(b)
2 (c) 600 Å
(c) √2 D (d) 700 Å
𝐷
(d) Solution :-
√2  Condition for first minimum in single slit ; 𝑎 sin 𝜃 = 𝜆
Solution :- 1
𝜆𝐷 𝜆 = 1 𝑋10−5 𝑋 10−2 𝑠𝑖𝑛30° = 10−7 𝑋 = 0.5 𝑋 10−7 𝑚 = 500 𝑋10−10 𝑚 = 500 Å
 Band width ; 𝛽 = 2
𝑑
𝜆 𝐷1 𝜆 𝐷1
Answer (b) 𝟓𝟎𝟎 Å
 If 𝑑1 → 2 𝑑, then the new band width ; 𝛽1 = =
𝑑1 2𝑑
𝜆 𝐷1 𝜆𝐷
 Given that ; 𝛽1 = 𝛽 (or) = (or) 𝐷1 = 2 𝐷
2𝑑 𝑑
Answer (a) 2 D

victory R. SARAVANAN. M.Sc., M.Phil., B.Ed PG ASST [PHYSICS], GBHSS, PARANGIPETTAI - 608 502
12 PHYSICS UNIT – 6 RAY OPTICS COMPLETE GUIDE AND MODEL QUESTION
7. A ray of light strikes a glass plate at an angle 60o.
If the reflected and refracted
rays are perpendicular to each other, the refractive index of the glass is, PART – II & III 2 AND 3 MARK SHORT ANSWER QUESTIONS & ANSWERS
(a) √3 1. What are the salient features of corpuscular theory of light?
(b)
3 Corpuscular theory :
2  Sir Isaac Newton proposed corpuscular theory of light.
3  According this theory, light is emitted as tiny, massless and perfectly elastic
(c) √
2 particles called corpuscles.
(d) 2  As the corpuscles are very small, the source of light does not suffer appreciable
Solution :- loss of mass even if it emits light for a long time.
 When reflected and refracted rays are perpendicular to each other, then the  They travel with high speed and they are unaffected by the force of gravity. So
angle of incidence is equal to angle of polarization. From Brewster’s law their path is a straight line.
𝑛 = tan 𝑖𝑃 = tan 60° = √3  The energy of light is the kinetic energy of these corpuscles.
 When they impinge on the retina of the eye, the vision is produced. The
Answer (a) √𝟑
different size of the corpuscles is the reason for different colours of light.
8. One of the of Young’s double slits is covered with a glass plate as shown in  The reflection of light is due to repulsion of the corpuscles by the medium and
figure. The position of central maximum will, refraction of light is due to the attraction of the courpuscles by the medium.
(a) get shifted downwards  This theory could not explain, why speed of light is lesser in denser medium
(b) get shifted upwards than rarer medium and also interference, diffraction and polarization.
(c) will remain the same 2. Write a note on wave theory of light.
(d) data insufficient to conclude Wave theory of light :
Solution :-  Christian Huygens proposed the wave theory of light.
 When the slit is covered with a glass plate, due to refraction the light ray bends  According to wave theory, light is a disturbance from a source that travels as
upwards. Hence central maximum get shifted upwards longitudinal mechanical wave through the ether medium that was presumed to
Answer (b) get shifted upwards pervade in all space.
9. Light transmitted by Nicol prism is,  This theory could successfully explain reflection, refraction, interference, and
(a) partially polarised diffraction. But polarization could not explain by this theory as it is the
(b) unpolarised property of only transverse waves.
(c) plane polarised  Later the existence of ether in all space was proved to be wrong.
(d) elliptically polarised 3. Write a note on electromagnetic wave theory .
Solution :- Electromagnetic wave theory of light :
 The ordinary ray is total internally reflected at the layer of canada balsam and is  Maxwell proposed electromagnetic theory of light.
prevented from emerging along with extraordinary ray. Where as, the  According to electromagnetic wave theory, light is an electromagnetic wave
extraordinary ray is transmitted through the crystal which is plane polarized. which is transverse in nature carrying electromagnetic energy.
Answer (c) plane polarised  No medium is necessary for the propagation of electromagnetic waves.
10. The transverse nature of light is shown in,  All the phenomenon of light could be successfully explained by electromagnetic
(a) interference theory. But the interaction of light with matter like photoelectric effect,
(b) diffraction Compton effect could not be explained by this theory.
(c) scattering 4. Write a short note on quantum theory of light.
(d) polarisation Quantum theory of light :
Solution :-  By extending Max Plank quantum ideas, Albert Einstein proposed quantum
 Both, longitudinal and transverse waves exhibit the phenomena of interference theory of light.
and diffraction. Since light is an electromagnetic wave, it is transverse in nature.  According to quantum theory, light is not continuous but it propagated in the
The transverse nature of light wave is proved in the phenomenon called form of discrete packets of energy called photon.
polarisation.  Each photon has energy of 𝑬 = 𝒉 𝝂
Answer (d) polarisation Here 𝒉 →Plank’s constant( 𝒉 = 𝟔. 𝟔𝟐𝟓𝑿 𝟏𝟎−𝟑𝟒 𝑱𝒔)

victory R. SARAVANAN. M.Sc., M.Phil., B.Ed PG ASST [PHYSICS], GBHSS, PARANGIPETTAI - 608 502
12 PHYSICS UNIT – 6 RAY OPTICS COMPLETE GUIDE AND MODEL QUESTION
5. What is Dual nature of light ? 15. Give the methods to obtain coherent light waves.
 A light has both wave as well as particle nature and it is said to have dual nature.  Coherent waves are obtained by following three techniques.
(1) Light propagated as a waves (1) Intensity or amplitude division
(2) Light interacts with matter as a particle (2) Wavefront division
6. Write a note on wave nature of light. (3) Source and images
Wave nature of light : 16. Write a note on intensity or amplitude division.
 Light is transverse electromagnetic wave. Intensity or amplitude division :
 The wave nature of light was confirmed by the experiments on interference and  If light is incident on a partially silvered mirror, both reflection and refraction
diffraction. takes place simultaneously.
 Like electromagnetic wave, light can travel through vacuum.  As the two light beams are obtained from the same light source, the two divided
 The transverse nature of light was proved by polarization. light beams will be coherent beams.
7. Define wave front.  They will be either in-phase or at constant phase difference. (e.g.) Michelson’s
 A wavefront is the locus of points which are in the same state or phase of interferometer
vibration. 17. Write a note on wavefront division.
8. What are the shapes of wavefront for (a) source at infinite, (b) point source and Wavefront division :
(c) line source?  It is the common method used for producing two coherent sources. We know all
(1) A point source located at a finite distance gives spherical wavefront. the points on the wavefront are at the same phase.
(2) A point source located at infinite distance gives plane wavefront.  If two points are chosen on the wavefront by using a doubl slit, the two points
(3) A line source gives cylindrical wavefront. will act as coherent sources. (e.g.) Young’s double slit method
9. State Huygen’s principle. 18. Write a note on Source and images method.
Huygen’s principle : Source and images :
 Each point of the wavefront is the source of secondary wavelets which  In this method, a source and its image will act as a set of coherent source,
spreading out in all directions with speed of the wave. because the source and its image will have waves in-phase or constant phase
 The envelope to all this wavelets gives the position and shape of the new difference. (e.g.) Lloyd’s mirror
wavefront at a later time. 19. What are called constructive and destructive interference?
10. Define interference. Constructive interference :
 The phenomenon of superposition of two light waves which produces increase  During superposition of two coherent waves, the points where the crest of one
in intensity at some points and decrease in intensity at some other points is wave meets the crest of other (or) the trough of one wave meets the trough of
called interference of light. the other wave, the waves are in-phase.
11. What is phase of a wave?  Hence the displacement is maximum and these points appear as bright.
 Phase is the angular position of a vibration.  This type of interference is said to be constructive interference.
12. Give the relation between phase difference and path difference. Destructive interference :
 In the path of the wave, one wavelength  corresponds to a phase of 2   During superposition of two coherent waves, the points where the crest of one
 Hence the path difference 𝛿 corresponds to a phase difference 𝜙 is wave meets the trough of other (or) vice versa, the waves are out-of-
𝝀 phase.
𝜹= 𝝓  Hence the displacement is minimum and these points appear as dark.
𝟐𝝅
13. Whar are called coherent sources?  This type of interference is said to be destructive interference.
 Two light sources are said to be coherent, if they produce waves which have 20. What is bandwidth of interference pattern?
same phase or constant phase difference, same frequency or wavelength, same  The band width () is defined as the distance between any two consecutive
waveform and preferably same amplitude. bright or dark fringes.
14. Can two independent monochromatic sources acts as coherent sources? 21. What are the conditions for obtaining clear and broad interference bands?
 Two independent monochromatic sources never be coherent, because they may (1) The screen should be as far away from the source as possible.
emit waves of same frequency and same amplitude, but not with same phase. (2) The wavelength of light used must be larger.
 Due to thermal vibrations, the atoms while emitting light undergoes this change (3) Two coherent sources must be as close as possible
in phase.

victory R. SARAVANAN. M.Sc., M.Phil., B.Ed PG ASST [PHYSICS], GBHSS, PARANGIPETTAI - 608 502
12 PHYSICS UNIT – 6 RAY OPTICS COMPLETE GUIDE AND MODEL QUESTION
22. Brilliant colours are exhibited by the surface of oil films and soap bubbles. 28. Distinguish between interference and diffraction.
Why? Interference Diffraction
 The colours exhibited by the surface of oil films and soap bubbles are due to Superposition of two waves Bending of waves around the edges
interference of white light undergoing multiple reflections from the top and Superposition of waves from two Superposition of wavefronts emitted
bottom surfaces of thin films. coherent sources from various points of the same
 The colourd depends upon, wavefront
(1) thickness of the film Equally spaced fringes Unequally spaced fringes
(2) refractive index of the film Intensity of all the bright fringes is Intensity falls rapidly for higher
(3) angle of incidence of the light almost same orders
23. What is diffraction? Large number of fringes are obtained Less number of fringes are obtained.
 Diffraction is bending of waves around sharp edges into the geometrically
29. What is Fresnel’s distance? Obtain an expression for it.
shadowed region.
Fresnel’s distance:
 We observe diffraction only when the size of the obstacle is comparable to the
 Fresnel’s distance is the distance upto which ray optics is obeyed and beyond
wavelength
which ray optics is not obeyed but wave optics becomes significant.
24. Distinguish between Fresnel and Fraunhofer diffraction.
Expression :
Fresnel diffraction Fraunhofer diffraction
 Let Fresnel distance = 𝑧
Spherical or cylindrical wave front Plane wavefront undergoes
 From the diffraction equation for first
undergoes diffracion diffraction
minimum,
The source of light is finite distance The source of light is infinite distance 𝜆 𝜆
from the obstacle from the obstacle sin 𝜃 = (𝑜𝑟) 𝜃 =
𝑎 𝑎
Convex lenses need not be used Convex lenses are to be used  From the definition of Fresnel’s distance,
Difficult to observe and analyse Easy to observe and analyse 𝑎 𝑎
sin 2𝜃 = (𝑜𝑟) 2𝜃 =
𝑧 𝑧
25. Discuss the special cases on first minimum in Fraunhofer diffraction.  Equating the above two equantion,
 The equation for first minimum in single slit diffraction is 𝒂 𝒔𝒊𝒏𝜽 = 𝝀 𝜆 𝑎
Case (1) :If 𝒂 < 𝝀 then 𝒔𝒊𝒏𝜽 > 𝟏 which is not possible. Hence diffraction does 2 =
𝑎 𝑧
not takes place. 𝒂𝟐
Case (2) : If 𝒂 = 𝝀 then 𝒔𝒊𝒏𝜽 = 𝟏 (or) 𝜽 = 𝟗𝟎° (i.e.) The first minimumis at 𝟗𝟎° 𝒛=
𝟏 𝟐𝝀
Case (3) : If 𝒂 > 𝝀 (i.e.) 𝒂 = 𝟐 𝝀 then, 𝒔𝒊𝒏𝜽 = or 𝜽 = 𝟑𝟎° The diffraction is 30. Give the reason for colourful appearance of the compact disc.
𝟐
observed with a measurable spread. Hence, it is concluded that for observing the  On the read or writable side of the disc, there are many narrow circular tracks
diffraction pattern, essentially the width of the slit a must be just few times whose width are comparable to the wavelength of visible light.
greater than the wavelength of light λ  Hence the diffraction takes place after reflection for incident white light to give
Case(4) : If 𝒂 >> 𝝀 then 𝒔𝒊𝒏𝜽 << 𝟏 . The first minimum falls within the width colourful appearance. Thus tracks act as reflecting grating.
space of the slit itself. Hence, the phenomenon of diffraction is not observed at all. 31. What are resolution and resolving power?
26. What is diffraction grating?  Two point sources must be imaged in such a way that their images are
 Grating is a plane sheet of transparent material on which opaque rulings are sufficiently far apart that their diffraction pattersn do not overlap. This is called
made with a fine diamond pointer. resolution.
 Thus gratting has multiple slits with equal widths of size comparable to the  The inverse of resolution is called resolving power. The ability of an optical
wavelength of light instrument to separate or distinguish small or closely adjacent objects through
 The modern commercial grating contains about 6000 lines per centimeter. the image formation is said to be resolving power of the instrument.
27. Define grating element and corresponding points. 32. What is Rayleigh’s criterion?
 The combined width of a ruling (b) and a slit (a) is called grating element  According to Rayleigh’s criterion, for tow point objects to be just resolved, the
(e = a + b) minimum distance between their diffraction images must be in such a way that
 Points on successive slits separated by a distance equal to the grating element the central maximum of one coincides with the first minimum of the other and
are called corresponding points. vice versa. The Rayleigh’s criterion is said to be limit of resolution.

victory R. SARAVANAN. M.Sc., M.Phil., B.Ed PG ASST [PHYSICS], GBHSS, PARANGIPETTAI - 608 502
12 PHYSICS UNIT – 6 RAY OPTICS COMPLETE GUIDE AND MODEL QUESTION
33. Define polarization. 38. What is polarizer and analyser?
 The phenomenon of restricting the vibrations of light to a particular direction Polariser :
perpendicular to the direction of wave propagation motion is called  The polaroid which plane polarizes the unpolarized light passing through it is
polarization. called a polarizer.
34. Distinguish between unpolarized and plane polarized light. Analyser :
Unpolarized light Plane polarized light  The polaroid which is used to examine whether a beam of light is polarized or
A transverse wave which has A transverse wave which has vibrations not is called analyser.
vibrations in all directions in a plane in only one direction in a plane 39. Discuss how a plane polarized and partially polarized light will be analysed
perpendicular to the direction of perpendicular to the direction of using analyser?
propagation is said to be unpolarized propagation is said to be planepolarized Plane polarized light :
light light  If the intensity of light varies from maximum to zero for every rotation of 90
Symmetrical about the ray direction Asymmetrical about the ray direction of the analyser, the light is said to be plane polarized
Partially polarized light :
Produced by conventional light It is obtained from unpolarized light
 If the intensity of light varies from maximum to mimimum for every rotation of
sources with help of polarizers
90 of the analyser, the light is said to be partially polarized.
35. Define plane of vibration and plane of polarization.
40. State and prove Malus’ law.
Plane of vibration:
Malus’s law :
 The plane containing the vibrations of the electric field vector is known as plane
 When a beam of plane polarized light of intensity 𝐼𝑜 is incident on an analyser,
of vibration.
the light transmitted of intensity I from the analyser varies directly as the
Plane of polarization:
square of the cosine of the angle  between the transmission axis of polarizer
 The plane perpendicular to the plane of vibration and containing the ray of light
and analyser. This is known as Malus’ law.
is known as the plane of polarization.
36. How an unpolarized light can be polarized? 𝑰 = 𝑰𝒐 𝐜𝐨𝐬 𝟐 𝜽
 The unpolarized light can be polarized by following techniques. Proof :
(1) Polarization by selective absorption  Let the angle between plane of polarizer and
(2) Polarization by reflection analyser = 
(3) Polarization by double refraction Intensity of electric vector transmitted by the
(4) Polarization by scattering polarizer = 𝐼𝑜
37. Discuss polarization by selective absorption. Amplitude of this electric vector = 𝑎
Polarization by selective absorption (Polaroids) :
 Selective absorption is the property of a material which transmits waves whose
electric field vibrate in a plane parallel to a certain direction of orientation and  The amplitude of the incident light was resolved in
absorbs all other waves. to two components,
 The polroids or polarizer using this property of selective absorption to (1) 𝒂 𝐜𝐨𝐬 𝜽 - parallel component to the axis of transmission of the analyser
produce intense plane polarized light. (2) 𝒂 𝐬𝐢𝐧 𝜽 - perpendicular component to the axis of transmission of the
 Selective absorption is also called as dichroism. analyser
 Edwin Land developed polarizer in the form of thin sheets.  Here only the parallel component (𝒂 𝐜𝐨𝐬 𝜽) will be transmitted by the analyser.
 Tourmaline is a natural polarizing material. But polaroids are made artificially.  Hence ht intensity of the transmitted light is,
 A number of needle shaped crystals of quinine iodosulphate with their axes 𝐼 ∝ (𝑎 cos 𝜃 )2
parallel to one another packed in between two transparent plastic sheets serve 𝐼 = 𝑘(𝑎 cos 𝜃 )2
as a good polaroid. 𝐼 = 𝑘 𝑎2 cos 2 𝜃
 Recently new type of polaroids are prepared in which thin film of polyvinyl 𝑰 = 𝑰𝒐 𝐜𝐨𝐬 𝟐 𝜽
alcohol (colour less crystals) is used. (1) When 𝜃 = 0° , 𝑰 = 𝑰𝒐
(2) When 𝜃 = 90° , 𝑰 = 𝟎

victory R. SARAVANAN. M.Sc., M.Phil., B.Ed PG ASST [PHYSICS], GBHSS, PARANGIPETTAI - 608 502
12 PHYSICS UNIT – 6 RAY OPTICS COMPLETE GUIDE AND MODEL QUESTION
41. List the uses of polaroids. 45. Write a note on pile of plates.
Uses of polaroids : Pile of plates :
 Used in goggles and cameras to avoid glare of light
 Used in holography (three dimensional motion pictrure)
 Used to improve contrast in old oil paintings
 Used as window glasses to control the intensity of incoming light
 Polaroid produce polarized lights to be used in liquid crystal display (LCD)
42. Defined angle of polarization.
 The angle of incidence at which the reflected beam is plane polarized is called
polarizing angle or Brewste’s angle (𝒊𝑷 )
 The polarizing angle for glass is ; 𝒊𝑷 = 𝟓𝟕. 𝟓°
43. Explain polarization by reflection.  It work on the principle of polarization by reflection.
Polarization by reflection :  It consists of a number of glass plates placed one over the other in a tube.
 It is the simplest method to produce plane  These plates are inclined at an angle (𝟗𝟎° − 𝒊𝑷 ) to the axis of the tube.
polarized light.It is discovered by Malus.  A beam of unpolarized light is allowed to fall on the pile of plates along the axis
 Here, XY - reflecting surface of the tube. So the angle of incidence of light will be 𝒊𝑷 , which is the polarizing
AB - incident unpolarized light beam angle for glass.
BC - reflecting light beam  The vibrations perpendicular to the plane of incidence are reflected at each
BD - refracted light beam surface and those parallel to it are transmitted.
 On examining the reflected beam ‘BC’ with an  The larger the number of surfaces, the greater the intensity of the reflected
analyser, it is found that the ray is is partially plane polarized light.
plane polarized.  The pile of plates is used as a polrizer and also as an analyser.
 When the light is allowed to be incident on particular angle, the reflected beam 46. Define double refraction.
is found to be plane polarized. That angle of incidence is called polarizing  When a ray of unpolrized light is incident on a calcite crystal, two refracted rays
angle ( 𝒊𝑷 ) are produced. Hence two images of a single object are formed. This
44. State and prove Brewster’s law phenomenon is called double refraction. The obtained images are called as,
Brewste’ s law : (1) Ordinary image
 Sir David Brewster found that, at polarizing angle, the reflected and (2) Extra ordinary image
transmitted rays are perpendicular to each other.  Double refraction is also called bi refringence.
 Let, incident polarizing angle = 𝑖𝑃 47. Distinguish between ordinary ray and extra ordinary ray.
Angle of refraction = 𝑟 Ordinary ray Extraordinary ray
 From the figure, They obey the laws of refraction They do not obey the laws of
𝑖𝑃 + 90° + 𝑟𝑃 = 180° refraction
𝒓𝑷 = 𝟗𝟎° − 𝒊𝑷 − − − −(1) Inside the crystal, they travel with Inside the crystal, they travel with
 From Snell’s law same velocity in all directions different velocities along different
sin 𝑖𝑃 directions
=𝑛
sin 𝑟𝑃 A point source inside the crystal A point source inside the crystal
sin 𝑖𝑃 produces spherical wavefront for produces elliptical wavefront for extra
=𝑛
sin(90° − 𝑖𝑃 ) ordinary ray ordinary ray
sin 𝑖𝑃 48. Define Optic axis.
=𝑛
cos 𝑖𝑃  Inside the double refracting crystal, there is a particular direction in which both
𝐭𝐚𝐧 𝒊𝑷 = 𝒏 the ordinary and extraordinary rays travel with same velocity. This direction is
 This relation is known as Brewster’s law. called optic axis.
 This law states that, the tangent of the polarizing angle for a transparent  Along optic axis, the refractive index is same for both rays.
medium is equal to its refractive index.

victory R. SARAVANAN. M.Sc., M.Phil., B.Ed PG ASST [PHYSICS], GBHSS, PARANGIPETTAI - 608 502
12 PHYSICS UNIT – 6 RAY OPTICS COMPLETE GUIDE AND MODEL QUESTION
49. Define uniaxial crystal and biaxial crystal.  The electric field of light interact
 Crystals like calcite, quartz, tourmaline and ice having only one optic axis are with the electrons present in the
called uniaxial crystals. air molecules.
 Crystals like mica, topaz, selenite and aragonite having two optic axes are  Under the influence of the electric
called biaxial crystals. field of the incident wave the
50. Discuss about Nicol prism. electrons in the molecules
Nicol prism : acquire components of motion in
 Nicol prism is made by calcite both these directions.
crystal which is double  We have shown an observer
refracting crystal. looking at 90 to the direction of
 ABCD is the principal section of the sun.
a calcite crystal with its length  Clearly, charges accelerating
is three times of its breadth. paralled do not radiate energy
 The face angles are 72 and towards this observer since their
108 acceleration has no transverse component.
 It is cut in to two halves along the diagonal AB and joined together by a layer of  The radiation scattered by the molecule is therefore polarized perpendicular to
canada balsam, a transparent cement. the plane of the paper.
 Let an unpolarized light from monochromatic source is incident on the face AC of  This explains the reason for polarization of sunlight by scattering.
the Nicol prism. 53. Distinguish between near point focusing and normal focusing.
 Here double refraction takes place, and the ray split in to ordinary ray and Near point focusing Normal focusing
extraordinary ray. The image is formed at near point The image is formed at infinity
 For this calcite crystal.
In this position, the eye feel little In this position, the eye is most relaxed
refractive index for the ordinary ray = 1.658
strain to view the image
refractive index for the extraordinary ray = 1.486
𝑫 𝑫
 The refractive index of canada balsam = 1.523 Magnification is high ; 𝒎 = 𝟏 + Magnification is low ; 𝒎=
𝒇 𝒇
Here canada balsam does not polarize light
 The ordinary ray is totally internally reflected at the layer of canada balsam. 54. Why is oil immersed objective preferred in a microscope?
 The extraordinaty ray alone is transmitted through the crystal which is plane  The ability of microscope depends not only in magnifying the object but also in
polarized. resolving two points on the object separated by a small distance . That is,
51. What are the uses and drawbacks of Nicol prism? smaller the value of ′𝑑𝑚𝑖𝑛 ′ better will be the resolving power of the microscope.
Uses :  To further reduce the value of ′𝑑𝑚𝑖𝑛 ′ , the optical path of the light is increased
 It produces plane polarized light and funcitons as a polarizer. by immersing the objective of the microscope in to a bath containg oil of
 It can also used as an analyser. refractive index ‘n’. 𝑖. 𝑒. (𝑑𝑚𝑖𝑛 =
1.22 𝜆
)
Drawbacks : 2 𝑛 sin 𝛽

 It cost is very high due to scarity of large and flawless calcite crystal.  Such an objective is called oil immersed objective.
 Due to extraordinary ray passing obliquely through it, the emergent ray is  The term ‘𝒏 𝐬𝐢𝐧 𝜷′ is called numerical aperture (NA)
always displaced a little to one side. 55. What are the merits and demerits of reflecting telescope?
 The effective field of view is quite limited. Merits :
 Light emerging out of it is not uniformly plane polarized.  Only one surface is to be polished and maintained.
52. Explain polarization by scattering.  Support can be given from the entire back of the mirror rather than only at the
Polarization by scattering: rim for lens.
 The light from a clear blue portion of the sky shows a rise and fall of intensity  Mirror weigh much less compared to lens.
when viewed through a polaroid which is rotated. Demerits :
 This is because of sunlight, which has changed its direction on encountering the  The objective mirror would focus the light inside the telescope tube. One must
molecules of the earth’s atmosphere. have an eye piece insided obstruction some light.

victory R. SARAVANAN. M.Sc., M.Phil., B.Ed PG ASST [PHYSICS], GBHSS, PARANGIPETTAI - 608 502
12 PHYSICS UNIT – 6 RAY OPTICS COMPLETE GUIDE AND MODEL QUESTION
56. What is the use of an erecting lens in a terrestrial telescope?
 A terrestrial telescope is used to see object at long distance on the surface of PART – IV 5 MARK LONG ANSWER QUESTIONS & ANSWERS
earth. Hence image should be erect. 1. Prove laws of reflection using Huygens principle.
 So an additional erecting lens is used to make the final image enlarged and Laws of reflection - Proof :
erect.  𝑋𝑌 − Reflecting surface
57. What is the use of collimator inspectrometer?  𝐴𝐵 −Incident plane wavefront.
 The collimator is an arrangement to produce a parallel beam of light.  The incident rays from L and M are
58. What are the uses of spectrometer? perpendicular to this incident
 Spectrometer is an optical instrument used to, wavefront.
(1) study the spectra of different sources of light  Initially the point ‘A’ reaches reflecting
(2) measure the refractive indices of materials surface.
59. What is myopia? What is its remedy?  Then the successive points between AB reaches the surface.
 A person suffering from myopia or nearsightedness cannot see distant objects  Finally, by the time B reaches 𝐵1 , the point A would have reached 𝐴1
clearly.  This is applicable to all the points on the wavefront AB. Thus the reflected
 It occurs when the eye lens has too short focal length due to thickening of the wavefront 𝐴1 𝐵1 emanates as a plane wavefront.
lens or larger diameter of the eyeball than usual.  The line from 𝐿1 𝑎𝑛𝑑 𝑀1 perpendiculars to 𝐴1 𝐵1 represent reflected rays.
 Using concave lens this defect can be rectified.  .As the reflection happens in the same medium, the speed of light is same before
60. What is hypermetopia? What is its remedy? and after reflection. Hence, 𝐴𝐴1 = 𝐵𝐵1
 A person suffering from hypermetopia or farsightedness cannot see objects Law (1) :
close to the eye.  The incident rays, the reflected rays and the normal are in the same plane.
 It occurs when the eye lens has too long focal length due to thinning of eye lens Law (2) :
or shortening of the eyeball than normal.  Angle of incidence,
 Using convex lens this defect can be rectified. ∠𝑖 = ∠𝑁𝐴𝐿 = 90° − ∠𝑁𝐴𝐵 = ∠𝐵𝐴𝐵1
61. What is presbyopia?  Angle of reflection,
 The least distance for clear vision for aged people is appreciably more than 25 ∠𝑟 = ∠𝑁 1 𝐵1 𝑀1 = 90° − ∠𝑁 1 𝐵1 𝐴1 = ∠𝐴1 𝐵1 𝐴
cm and the person has to keep the object inconveniently away from the eye.  In ∆𝐴𝐵𝐵1 and ∆𝐵1 𝐴1 𝐴,
 Thus reasing or viewing smaller things held in the hands is difficult for them. ∠𝐵 = ∠𝐴1 = 90°
 This kind of farsightedness arising due to aging is called presbyopia. 𝐴𝐴1 = 𝐵𝐵1 and
62. What is astigmatism? hypotenuse 𝐴𝐵1 𝑖𝑠 𝑐𝑜𝑚𝑚𝑜𝑛
 Astigmatism is the defect arising due to different curvatures along different  Thus the two triangles are congruent. (i.e) ∠𝐵𝐴𝐵1 = ∠𝐴1 𝐵1 𝐴
planes in the eye lens. ∴ ∠𝒊 = ∠𝒓
 Astigmatic person cannot see all the directions equally well.  Hence laws of reflection are proved.
 Lenses with different curvatures in different planes called cylindrical lens is 2. Prove laws of refraction using Huygen’ principle.
used to rectify astigmatism defect. Laws of refraction - Proof :
63. Whar are called Airy’s discs?  Let XY be the refracting surface
 When a circular aperture like a lens or the iris of eye forms an image of a point  The incident wavefront AB is
object, the image formed will not be a point, but a diffraction pattern of in rarer medium (1)
concentric circles that becomes fainter while moving away from the centre.  The incident rays from L and M
 These are known as Airy’s discs. are perpendicular to this
incident wavefront.
 Initially the point ‘A’ reaches
refracting surface.
 Then the successive points
between AB reaches the surface.

victory R. SARAVANAN. M.Sc., M.Phil., B.Ed PG ASST [PHYSICS], GBHSS, PARANGIPETTAI - 608 502
12 PHYSICS UNIT – 6 RAY OPTICS COMPLETE GUIDE AND MODEL QUESTION
1 1
 Finally, by the time B reaches 𝐵 , the point A would have reached 𝐴 in the  The intensity of light is directly proportional to the square of the amplitude.
other medium. 𝐼 ∝ 𝐴2
 This is applicable to all the points on the wavefront AB. Thus the refracted 𝐼 ∝ 𝑎12 + 𝑎22 + 2 𝑎1 𝑎2 cos 𝜙
wavefront 𝐴1 𝐵1 emanates as a plane wavefront. 𝑰 ∝ 𝑰𝟏 + 𝑰𝟐 + 𝟐 √𝑰𝟏 𝑰𝟐 𝒄𝒐𝒔𝝓 − − − (𝟒)
 The line from 𝐿1 𝑎𝑛𝑑 𝑀1 perpendiculars to 𝐴1 𝐵1 represent refracted rays. (1) When, 𝜙 = 0, ±2𝜋, ±4𝜋, … …. .the resultant intensity becomes maximum.
 Let 𝑣1 be the speed of light in medium (1) and 𝑣2 be the speed of light in This is called constructive interference.
medium (2). Here 𝑣1 > 𝑣2 𝑰𝒎𝒂𝒙 ∝ (𝒂𝟏 + 𝒂𝟐 )𝟐
 The time taken for the ray to travel from B to 𝐵1 is same as the time taken for
𝑰𝒎𝒂𝒙 ∝ 𝑰𝟏 + 𝑰𝟐 + 𝟐 √𝑰𝟏 𝑰𝟐 − − − (𝟓)
the ray to travel from A reaches 𝐴1 . So 𝐴𝐴1 = 𝑣2 𝑡 and 𝐵𝐵1 = 𝑣1 𝑡
𝐵𝐵1 𝑣1 (2) When,𝜙 = ±𝜋, ±3𝜋, ±5𝜋 … …. the resultant intensity becomes minimum.
∴ 1
= − − − − − (1) This is called destructive interference.
𝐴𝐴 𝑣2
𝑰𝒎𝒊𝒏 ∝ (𝒂𝟏 − 𝒂𝟐 )𝟐
Law (1) :
 The incident rays, refracted rays and the normal are in the same plane. 𝑰𝒎𝒂𝒙 ∝ 𝑰𝟏 + 𝑰𝟐 − 𝟐 √𝑰𝟏 𝑰𝟐 − − − (𝟔)
Law (2) : Special case :
 Angle of incidence,  If 𝒂𝟏 = 𝒂𝟐 = 𝒂 , then resultant amplitude,
∠𝑖 = ∠𝑁𝐴𝐿 = 90° − ∠𝑁𝐴𝐵 = ∠𝐵𝐴𝐵1 𝐴 = √𝑎2 + 𝑎2 + 2 𝑎2 𝑐𝑜𝑠𝜙
 Angle of refraction, 𝐴 = √2 𝑎2 + 2 𝑎2 𝑐𝑜𝑠𝜙
∠𝑟 = ∠𝑁 1 𝐵1 𝑀1 = 90° − ∠𝑁 1 𝐵1 𝐴1 = ∠𝐴1 𝐵1 𝐴 𝐴 = √2 𝑎2 (1 + 𝑐𝑜𝑠𝜙)
 From ∆𝐴𝐵𝐵1 and ∆𝐵1 𝐴1 𝐴 ,
𝐵𝐵1 𝑐 𝜙
sin 𝑖 (
𝐴𝐵1
) 𝐵𝐵1 𝑣1 (
𝑛1
) 𝑛2 𝐴 = √2 𝑎2 [2 𝑐𝑜𝑠 2 ( )]
= = = = = 2
sin 𝑟 𝐴𝐴1 𝐴𝐴1 𝑣2 𝑐
( ) 𝑛1
( 1) 𝑛2 𝝓
𝐴𝐵
𝑨 = 𝟐 𝒂 𝒄𝒐𝒔 ( ) − − − − − (𝟕)
 In product form, 𝟐
𝒏𝟏 𝐬𝐢𝐧 𝒊 = 𝒏𝟐 𝐬𝐢𝐧 𝒓  If 𝑰𝟏 = 𝑰𝟐 = 𝑰𝑶 , then the resultant intensity, 𝐼 ∝ 𝐴2
3. Obtain the equation for resultant intensity due to interference of light. 𝜙
𝐼 ∝ 4 𝑎2 𝑐𝑜𝑠 2 ( )
Resultant intensity due to interference : 2
 Let 𝑆1 and 𝑆2 are the two light waves 𝝓
𝑰 = 𝟒 𝑰𝑶 𝒄𝒐𝒔𝟐 ( ) − − − − − (8)
meeting at a point ‘P’ 𝟐
 At any instant ‘t’, the displacement equations, When, 𝜙 = 0, ±2𝜋, ±4𝜋, … …., 𝑰𝒎𝒂𝒙 = 𝟒 𝑰𝑶
𝑦1 = 𝑎1 sin 𝜔𝑡 − − − − (1) and 𝜙 = ±𝜋, ±3𝜋, ±5𝜋 … …., 𝑰𝒎𝒊𝒏 = 𝟎
𝑦2 = 𝑎2 sin (𝜔𝑡 + 𝜙) − − − − (2)  Thus the phase difference (𝜙) between the two waves decides the intensity of
where, 𝜙  phase difference between them light at the point, where the two waves meet.
 Then the resultant displacement, 4. Explain Young’s double slit experimental set up and obtain equation for path
𝑦 = 𝑦1 + 𝑦2 = 𝑎1 sin 𝜔𝑡 + 𝑎2 sin(𝜔𝑡 + 𝜙) difference.
 By solving this, we get, Young’ s double slit experiment :
𝒚 = 𝑨 𝒔𝒊 𝒏 (𝝎𝒕 + 𝜽) − − − − (3)  Thomas Young used an opaque
screen with two small openings
 where, 𝐴 = √𝑎12 + 𝑎22 + 2 𝑎1 𝑎2 𝑐𝑜𝑠𝜙 and called double slit 𝑆1 and 𝑆2 kept
𝑎2 sin 𝜙 equidistance from a source ‘S’
𝜃 = tan−1 [ ]
𝑎1 + 𝑎2 cos 𝜙  The width of each slit is about
(1) When, 𝜙 = 0, ±2𝜋, ±4𝜋, … …. .the resultant amplitude becomes maximum 0.03 mm and they are separated
𝑨𝒎𝒂𝒙 = √(𝒂𝟏 + 𝒂𝟐 )𝟐 by a distance of about 0.3 mm.
(2) When, 𝜙 = ±𝜋, ±3𝜋, ±5𝜋 … …. the resultant amplitude becomes minimum  As 𝑆1 and 𝑆2 are equidistant
𝑨𝒎𝒊𝒏 = √(𝒂𝟏 − 𝒂𝟐 )𝟐 from ‘S’, the light waves from ‘S’
reach 𝑆1 and 𝑆2 in phase.
victory R. SARAVANAN. M.Sc., M.Phil., B.Ed PG ASST [PHYSICS], GBHSS, PARANGIPETTAI - 608 502
12 PHYSICS UNIT – 6 RAY OPTICS COMPLETE GUIDE AND MODEL QUESTION
 So 𝑆1 and 𝑆2 act as coherent sources which are the requirement of obtaining 5. Obtain the equation for band width in young’s double slit method.
interference pattern. Theory :
 The wavefronts from 𝑆1 and 𝑆2 get superposed on the otherside of the double  Let distance between
slit. 𝑆1 𝑎𝑛𝑑 𝑆2 = 𝑑
 When screen is placed at a distance of about 1 m from double slit, equally Distance of the screen from
spaced alternate bright and dark fringes are appears on the screen. These are double slit = 𝐷
called interference fringes. Wavelength of coherent light
 At the point ‘O’ on the screen, the waves from 𝑆1 and 𝑆2 travels equal distances wave = 
and arrive in-phase. Due to constructive interference, bright fringe is  Hence path difference
formed at point ‘O’ . This is called central bright fringe. between the light waves from
 When one of the slit is covered, then the fringes disappear and there is uniform 𝑆1 𝑎𝑛𝑑 𝑆2 to the point ‘P’
illumination observed on the screen. This clearly shows that the fringes are due 𝒚
𝜹= 𝒅
to interference e. 𝑫
Path difference (𝜹) : Condition for bright fringe (maxima) :
 For constructive interference, the path difference will be,
𝜹= 𝒏𝝀 [ 𝑛 = 0, 1, 2, … ]
𝑦
𝑑 = 𝑛𝜆
𝐷
 Thus the distance of the n th brigt fringe from ‘O’ is
𝑫
𝒚𝒏 = 𝒏 𝝀 − − − − − (3)
𝒅
Condition for dark fringe (minima) :
 For destructive interference, the path difference will be,
𝝀
𝜹 = (𝟐 𝒏 − 𝟏) [ 𝑛 = 1, 2, … ]
𝟐
𝑦 𝜆
𝑑 = (2 𝑛 − 1)
 Let distance between 𝑆1 𝑎𝑛𝑑 𝑆2 =𝑑 𝐷 2
 Thus the distance of the n th darkt fringe from ‘O’ is
Distance of the screen from double slit =𝐷 𝑫 𝝀
Wavelength of coherent light wave = 𝒚𝒏 = (𝟐 𝒏 − 𝟏) − − − − − (4)
 Hence path difference between the light waves from 𝑆1 𝑎𝑛𝑑 𝑆2 to the point ‘P’ 𝒅 𝟐
Band width (𝜷)
is  The band width is defined as the distance between any two consecutive bright
𝛿 = 𝑆2 𝑃 − 𝑆1 𝑃 = 𝑆2 𝑃 − 𝑀𝑃 = 𝑆2 𝑀 or dark fringes.
 From the figure, ∠𝑂𝐶𝑃 = ∠𝑆2 𝑆1 𝑀 = 𝜃  The distance between (n+1)th and nth consecutive bright fringes from ‘O’ is
 𝐼𝑛 ∆𝑆2 𝑆1 𝑀 𝛽 = 𝑦𝑛+1 − 𝑦𝑛
𝑆2 𝑀 𝛿
sin 𝜃 = = 𝐷 𝐷
𝑆1 𝑆2 𝑑 𝛽= (𝑛 + 1) 𝜆 − 𝑛𝜆
𝑑 𝑑
∴ 𝛿 = sin 𝜃 . 𝑑 𝑫
 Here 𝜃 is small. Hence, sin 𝜃 ≈ tan 𝜃 ≈ 𝜃 𝜷= 𝝀 − − − − − − − (𝟓)
𝒅
𝛿= 𝜃. 𝑑 − − − − − (1)  Simillarly the distance between (n+1)th and nth consecutive dark fringes from ‘O’
 Also, in ∆𝑂𝐶𝑃, 𝑫
𝑂𝑃 𝑦 𝜷= 𝝀 − − − − − − − (𝟔)
𝜃 ≈ tan 𝜃 = = 𝒅
𝑂𝐶 𝐷  Eqn (5) and (6) shows that the bright and dark fringes are of same width
 Put this in eqn (1) equally spaced on either side of central bright fringe
𝒚
𝜹= 𝒅 − − − − − (2)
𝑫
 Point ‘P’ may be apper either bright or dark depending on the path differendce.
victory R. SARAVANAN. M.Sc., M.Phil., B.Ed PG ASST [PHYSICS], GBHSS, PARANGIPETTAI - 608 502
12 PHYSICS UNIT – 6 RAY OPTICS COMPLETE GUIDE AND MODEL QUESTION
6. Obtain the equations for constructive and destructive interference for 7. Discuss diffraction at single slit and obtain the condition for nth minimum.
transmitted and reflected waves in thin films. Diffraction at single slit :
Interference in thin films :

 Let a parallel beam of light fall normally on a single slit AB. The centre of the slit
is C
 A straight line through ‘C’ perpendicular to the plane of slit meets the centre of
 Consider a thin film of transparent material of refractive index ‘’ and thickness ‘t’ the screen at ‘O’
 A parallel beam of light is incident on the film at an angle ‘𝑖’  Let 𝑦 be the distance of of point ‘P’ from ‘O’
 At upper surface, the light wave is divided in to two parts. One part is reflected  The lines joining ‘P’ to the different points on the slit can be treated as parallel
and other part is refracted. lines, making and angle 𝜃 with the normal ‘CO’
 The refracted part which enters in to the film, again gets divided at the lower  All the parallel waves from different points on the slits get interfere at ‘P’ to give
surface in two parts. One is transmitted and the other is reflected back in to the resultant intensity.
film. Here interference is produced by both the reflected and transmitted light. Condition for minima :
Interference due to reflected light :  To explain minimum intensity, divide the slit in to even number of parts.
 When light travelling in a rarer medium and getting reflected by a denser (1) Condition for P to be first minimum :
𝑎
medium, undergoes a phase change of  . Hence an additional path difference of  Let us divide the slit AB in to two half’s each of width
𝝀 2
is introduced.  The various points on the slit which are separated by the same width
𝟐
𝑎
 Again for normal incidence (𝑖 = 0), the points ‘A’ and ‘C’ are very close to each ( ) called corresponding points
2
other.  The path difference of light waves from different corresponding points
 The extra distance travelled by the wave coming out from ‘C’ is (𝐴𝐵 + 𝐵𝐶) meeting at ‘P’
 Hence the path difference between the waves reflected at ‘A’ and ‘C’ is 𝑎
𝛿 = 𝜇 (𝐴𝐵 + 𝐵𝐶) = 𝜇 (𝑑 + 𝑑) = 2 𝜇 𝑑 𝛿 = sin 𝜃
2
𝜆
 Since additional path difference is introduced due to reflection at A, the the  The condition for ‘P’ to be first minimum,
2
𝑎 𝜆
total path difference, sin 𝜃 =
𝝀 2 2
𝜹 =𝟐𝝁𝒅 + − − − (4) (𝑜𝑟) 𝒂 𝐬𝐢𝐧 𝜽 = 𝝀
𝟐
(1) The condition for constructive interference in reflected ray is,, (2) Condition for P to be second minimum :
𝑎
𝛿 =𝑛𝜆  Let us divide the slit AB in to four equal parts of width
4
𝜆  Here various corresponding points on the slit which are separated by the
(𝑜𝑟) 2𝜇𝑑 + =𝑛𝜆 𝑎
2 same width ( )
𝝀 4
(𝑜𝑟) 𝟐 𝝁 𝒅 = (𝟐𝒏 − 𝟏) − − − (𝟓)  The path difference of light waves from different corresponding points
𝟐 meeting at ‘P’
(2) The condition for destructive interference in reflected ray is, 𝑎
𝜆 𝛿 = sin 𝜃
𝛿 = (2𝑛 + 1) 4
2  The condition for ‘P’ to be second minimum,
𝜆 𝜆 𝑎 𝜆
(𝑜𝑟) 2 𝜇 𝑑 + = (2𝑛 + 1) sin 𝜃 =
2 2 4 4
(𝑜𝑟) 𝟐𝝁𝒅 = 𝒏𝝀 − − − (6) (𝑜𝑟) 𝒂 𝐬𝐢𝐧 𝜽 = 𝟐 𝝀
victory R. SARAVANAN. M.Sc., M.Phil., B.Ed PG ASST [PHYSICS], GBHSS, PARANGIPETTAI - 608 502
12 PHYSICS UNIT – 6 RAY OPTICS COMPLETE GUIDE AND MODEL QUESTION
(3) Condition for P to be nthminimum :  The path difference (𝛿) between the diffracted waves from one pair of
𝑎
 Let us divide the slit AB in to 2n equal parts of width corresponding points is,
2𝑛
 The condition for ‘P’ to be nth minimum, 𝛿 = (𝑎 + 𝑏) sin 𝜃
𝑎 𝜆  The point ‘P’ will be bright, when 𝛿 = 𝑚 𝜆 [𝑚 = 0,1,2,3 … ]
sin 𝜃 =  Hence, (𝒂 + 𝒃) 𝐬𝐢𝐧 𝜽 = 𝒎 𝝀 − − − − (1)
2𝑛 2
(𝑜𝑟) 𝒂 𝐬𝐢𝐧 𝜽 = 𝒏 𝝀 where 𝑚  order of diffraction
Condition for maxima : (1) Condition for zero order maximum :
 To explain maximum intensity, divide the slit in to odd number of parts.  When, (𝑎 + 𝑏) sin 𝜃 = 0, then, 𝜃 = 0 ; 𝑚 = 0 It is zero order
𝑎 diffraction or central maximum
 For first maximum, the slit is divided in to three equal parts each of width ( ).
3 (2) Condition for first order maximum :
Hence  When, (𝑎 + 𝑏) sin 𝜃1 = 𝜆, then, 𝜃 = 𝜃1 ; 𝑚 = 1 It is first order
𝑎 𝜆 𝝀
sin 𝜃 = (𝑜𝑟) 𝒂 𝐬𝐢𝐧 𝜽 = 𝟑 diffraction
3 2 𝟐 (3) Condition for second order maximum :
𝑎
 For secod maximum, the slit is divided in to five equal parts each of width ( ).  When,(𝑎 + 𝑏) sin 𝜃2 = 2𝜆, then,𝜃 = 𝜃2 ; 𝑚 = 2 It is second order
5
Hence diffraction
𝑎 𝜆 𝝀 (4) Condition for higher order maxima :
sin 𝜃 = (𝑜𝑟) 𝒂 𝐬𝐢𝐧 𝜽 = 𝟓
5 2 𝟐 (𝑎 + 𝑏) sin 𝜃 = 𝑚 𝜆
 In general, for nth first maximum, the slit is divided in to (2n+1) equal parts each  If ‘N’ be the number of rulings drawn per unit width (1 m), then ,
𝑎
of width ( ). Hence 𝑁 𝑎 + 𝑁 𝑏 = 1 (𝑜𝑟)
2𝑛+1
𝑎 𝜆 𝝀 𝑁 (𝑎 + 𝑏) = 1
sin 𝜃 = (𝑜𝑟) 𝒂 𝐬𝐢𝐧 𝜽 = (𝟐𝒏 + 𝟏) 1
2𝑛 + 1 2 𝟐 𝑎+𝑏 =
8. Discuss the diffraction at a grating and obtain the condition for m th maximum. 𝑁
1
Diffraction in grating : ∴ sin 𝜃 = 𝑚 𝜆
𝑁
(𝑜𝑟) 𝐬𝐢𝐧 𝜽 = 𝑵 𝒎 𝝀 − − − − − (𝟐)
9. Discuss the experiment to determine the wavelength of monochromatic light
using diffraction grating.
Experiment to determine wavelength of light :

 Let ‘AB’ represent the plane transmission grating.


 It has number of slits of equal width (𝑎) and equal number of opaque rulings of
equal width (𝑏)
 Lte a plane wavefornt of monochromatic light of wavelength ‘𝜆’ be incident
 The wavelength of a spectral line can be very accurately determined with help
normally on the grating.
of a diffraction grating and a spectrometer.
 As the slit size is comparable to that of wavelength, the incident light diffracts at
 Let all the preliminary adjustments are made on the spectrometer.
the grating.  The slit of the spectrometer is illuminated by a monochromatic light, whose
 Using convex lens, the diffracted waves are focused on the screen.
wavelength to be determined.
 Consider a point ‘P’ on the screen, at an angle ‘𝜃 ‘ with the normal drawn from
 The telescope is brought in line with collimator to view the direct image of the
the centre of the grating to the screen.
slit.
victory R. SARAVANAN. M.Sc., M.Phil., B.Ed PG ASST [PHYSICS], GBHSS, PARANGIPETTAI - 608 502
12 PHYSICS UNIT – 6 RAY OPTICS COMPLETE GUIDE AND MODEL QUESTION
 The given transmission grating is then mounted on the prism table with its 11. Discuss about simple microscope and obtain the equations for magnification
plane perpendicular to the incident beam of light coming from collimator. for near point focusing and normal focusing.
 The telsescope is turn to one side until the first order diffraction image of the Simple microscope - Near point focussing :
slit coincides with the vertical cross wire of the eye piece.  A simple microscope is a single
 The reading of the position of the telescope is noted. magnifying lens of small focal
 Similarly the first order diffraction image on the other side is made to coincide length.
with vertical cross wire and corresponding reading is noted.  In near point focusing, object
 The difference between two positions gives 2 𝜃 distance ‘u’ is less than ‘f’
 Half of its value gives 𝜃, the diffraction angle for first order maximum.  The image is formed at near point
 The wavelength of light is calculated from, or least distance ‘D’ of distinct
𝐬𝐢𝐧 𝜽 vision.
𝝀=
𝑵𝒎  The magnification ‘m’ is given by,
10. Discuss the experiment to determine the wavelength of different colours 𝑣
𝑚=
using diffraction grating. 𝑢
Determination of wavelength of different colours :  Using lens equation,
𝑣
 White light is a composite light 𝑚 =1−
which contains all wavlengths 𝑓
from violet to red in visible  Substitute, 𝑣 = −𝐷
region. 𝑫
𝒎= 𝟏+
 When white light is used, the 𝒇
diffraction pattern consists of a Simple microscope - Normal focussing :
white central maximum and on  Here the image is formed at
both sides continuous coloured infinity.
diffraction patterns are  So we will not get direct
formed. practical relation for
magnification. Hence we can
 The central maximum is white practically use the angular
as all the colours meet here magnification.
constructively with no phase  The angular magnification is defined as the ratio of angle (𝜃𝑖 ) subtended by the
difference. image with aided eye to the angle (𝜃𝑂 ) subtended by the object with unaided
 It produces a spectrum of diffraction pattern from violet to red on either side of eye. That is,
central maximum. 𝜃𝑂
𝑚= − − − − − − (1)
 By measuring the angle (𝜽) at which these colours appear for various order (m) 𝜃𝑖
of diffraction, the wavelength of different colours could be calculated using the  For unaided eye,
formula, ℎ
𝐬𝐢𝐧 𝜽 tan 𝜃𝑂 ≈ 𝜃𝑂 =
𝐷
𝝀=  For aided eye,
𝑵𝒎
where, 𝑵  number of rulings drawn per unit width of grating ℎ
tan 𝜃𝑖 ≈ 𝜃𝑖 =
𝑓
 Thus eqn (1) becomes,

𝜃𝑂 ( )
𝑚= = 𝐷
𝜃𝑖 ℎ
( )
𝑓
𝑫
𝒎=
𝒇
victory R. SARAVANAN. M.Sc., M.Phil., B.Ed PG ASST [PHYSICS], GBHSS, PARANGIPETTAI - 608 502
12 PHYSICS UNIT – 6 RAY OPTICS COMPLETE GUIDE AND MODEL QUESTION
12. Explain about compound microscope and obtain the equation for 13. Obtain the equation for resolving power of microscope.
magnification. Resolving power of microscope:
Compound microscope :  A microscope is used to see the details of the object under observation.
 The lens near the object is called the objective, forms a real, inverted, magnified  Good microscope should not only magnify the object but also resolve the two
image of the object. points on an object which are separated by the smallest distance dmin.
 This serves as the object for the second lens which is the eyepiece.  Actually, dmin is the resolution and its reciprocal is the resolving power.
 Eye piece serves as a simple microscope that produces finally an enlarged and
virtual image.
 The first inverted image formed by the objective is to be adjusted close to, but
within the focal plane of the eyepiece, so that the final image is formed nearly at
infinity or at the near point.

 The spatial resolution (radius of central maximum) is


1.22 𝜆 𝑓
𝑟𝑜 =
𝑎
 In microscope, the object distance is just more than the focal length f and the
image is formed at v as shown in the Figure. Hence,.
1.22 𝜆 𝑣
𝑟𝑜 =
𝑎
 Here, in the place of focal length f we have the image distance v. If the difference
between the two points on the object to be resolved is dmin, then the
magnification m is,
𝑟𝑜 𝑟𝑜 1.22 𝜆 𝑣 1.22 𝜆 𝑣
 The final image is inverted with respect to the original object. 𝑚= (𝑜𝑟) 𝑑𝑚𝑖𝑛 = = = 𝑣
𝑑𝑚𝑖𝑛 𝑚 𝑚𝑎 ( )𝑎
Magnification (m) : 𝑢
 From the ray diagram, the linear magnification due to the objective is, 1.22 𝜆 𝑢 1.22 𝜆 𝑓
(𝑜𝑟) 𝑑𝑚𝑖𝑛 = = [∵ 𝑢 ≈ 𝑓]
ℎ1 𝐿 𝑎 𝑎
𝑚𝑜 = = − − − − − (1)  On the object side, 2 tan 𝛽 ≈ 2 sin 𝛽 =
𝑎
ℎ 𝑓𝑜 𝑓
Here ‘L’ is the distance between the first focal point of the eye piece to the 1.22 𝜆
second focal point of the objective. This is called the tube length. 𝑑𝑚𝑖𝑛 =
2 sin 𝛽
 The magnification of the eyepiece,  To further reduce the value of dmin the optical path of the light is increased by
𝐷
𝑚𝑒 = 1 + − − − − − (2) immersing the objective of the microscope into a bath containing oil of
𝑓𝑒 refractive index n.
 The total magnification ‘m’ in near point focusing , 1.22 𝜆
𝑳 𝑫 𝑑𝑚𝑖𝑛 =
𝒎 = 𝒎𝒐 𝒎𝒆 = [ ] [𝟏 + ] 2 𝑛 sin 𝛽
𝒇𝒐 𝒇𝒆  Such an objective is called oil immersed objective. The term n sin β is called
 If the final image is formed at infinity (normal focusing), the magnification if eye numerical aperture (NA). Hence,
piece is, 𝟏. 𝟐𝟐 𝝀
𝐷 𝒅𝒎𝒊𝒏 =
𝑚𝑒 = − − − − − (3) 𝟐 (𝑵𝑨)
𝑓𝑒  Then the resolving power of microscope is,
 The total magnification ‘m’ in normal focusing is, 𝟏 𝟐 (𝑵𝑨)
𝑳 𝑫 𝑹𝑴 = =
𝒎 = 𝒎𝒐 𝒎𝒆 = [ ] [ ] 𝒅𝒎𝒊𝒏 𝟏. 𝟐𝟐 𝝀
𝒇𝒐 𝒇 𝒆
victory R. SARAVANAN. M.Sc., M.Phil., B.Ed PG ASST [PHYSICS], GBHSS, PARANGIPETTAI - 608 502
12 PHYSICS UNIT – 6 RAY OPTICS COMPLETE GUIDE AND MODEL QUESTION
14. Discuss about astronomical telescope. (3) Telescope :
Astronomical telescope :  The telescope is an astronomical type.
 An astronomical telescope is used to  It consists of an eyepiece provided with cross wires at one end and an
get the magnification of distant
objective at its other end.
astronomical objects like stars,
 The distance between the objective and the eyepiece can be adjusted so
planets
 The image formed by this will be that the telescope forms a clear image at the cross wires.
inverted.  The telescope is attached to a circular scale and both can be rotated
 It has an objective of long focal together.
length and a much larger aperture  The telescope and prism table are provided with radial screws for fixing
than eye piece. them at a desired position and tangential screws for fine adjustments.
 Light from a distant object enters Preliminary adjustments of the spectrometer
the objective and a real image is (1) Adjustment of the eyepiece: The telescope is turned towards an illuminated
formed in the tube at its second
surface and the eyepiece is moved to and fro until the cross wires are clearly
focal point.
 The eye piece magnifies this image producing a final inverted image. seen.
Magnification (m) : (2) Adjustment of the telescope: The telescope is adjusted to receive parallel rays
 The magnification ‘m’ is the ratio of the angle 𝛽 subtended at the eye by the by focusing it to a distant object to get a clear image on the cross wire.
final image to the angle 𝛼 which the object subtends at the lens or the eye. (3) Adjustment of the collimator : The telescope is brought in line with the
𝛽 collimator. The distance between the illuminated slit and the lens of the
𝑚=
𝛼 collimator is adjusted until a clear image of the slit is seen at the cross wire.
 From figure, (4) Levelling of the prism table : The prism table is brought to the horizontal level
ℎ by adjusting the levelling screws and it is ensured by using sprit level.
[ ] 𝒇𝒐
𝑓𝑒 16. Explain the experimental determination of material of the prism using
𝑚= =
ℎ 𝒇𝒆 spectrometer.
[ ]
𝑓𝑜 Determination of refractive index :
 The length of the telescope is approximately, 𝑳 = 𝒇𝒐 + 𝒇𝒆
 The preliminary adjustments of the telescope, collimator and the prism table of
15. Mention different parts of spectrometer and explain the preliminary
the spectrometer are made.
adjustments.
 The refractive index () of the prism is determined by knowing the angle of the
Spectrometer :
prism (A) and the angle of minimum deviation (D)
 The spectrometer is an optical instrument used to analise the spectra of different
(1) Angle of the prism (A) :
sources of light, to measure the wavelength of different colours and to measure
the refractive indices of materials of prisms. It basically consists of three parts
(1) Collimator:
 The collimator is used for producing parallel beam of light.
 It has a convex lens and a vertical slit of adjustable width which faces the
source.
 The position of slit can be adjusted so that it is kept at the focus of the lens.
 The collimator is rigidly fixed to the base.
(2) Prism table:
 The prism table is used for mounting the prism, grating etc. It consists of
two circular discs provided with three levelling screws.
 It can be rotated and its position can be read from two verniers V1 and V2 .
 The prism table can be fixed at any desired height.

victory R. SARAVANAN. M.Sc., M.Phil., B.Ed PG ASST [PHYSICS], GBHSS, PARANGIPETTAI - 608 502
12 PHYSICS UNIT – 6 RAY OPTICS COMPLETE GUIDE AND MODEL QUESTION
 The prism is placed on the prism table with its refracting edge facing the
collimator. EXAMPLE PROBLEMS WITH SOLUTIONS
 The slit is illuminated by a sodium light. 1. The wavelength of light from sodium source in vacuum is 5893Å.What are its
 The parallel rays coming from the collimator fall on the two faces AB and AC. (a) wavelength, (b) speed and (c) frequency when this light travels in water
 The telescope is rotated to the position 𝑇1 until the image of the slit formed by which has a refractive index of 1.33.
the reflection at the face AB coincides with the vertical cross wire of the -Solution :- 𝑛1 = 1 ; 𝑛2 = 1.33 ; 1 = 5893 A ;
telescope.  Refractive index of water,
 The corresponding vernier readings are noted. 𝑐 1  1
 The telescope is then rotated to the position 𝑇2 where the image of the slit 𝑛= = =
𝑣 2  2
formed by the reflection at the face AC coincides with the vertical cross wire of 1 5893 𝑋 10−10
the telescope. The corresponding vernier readings are again noted. 2 = =
 The difference between these two readings gives the angle rotated by the 𝑛 1.33
𝟐 = 𝟒𝟒𝟑𝟎 𝑿 𝟏𝟎−𝟏𝟎 = 𝟒𝟒𝟑𝟎 𝐀
telescope, which is twice the angle of the prism. 𝑐
 Half of this value gives the angle of the prism (A)  Since, 𝑛 = , velocity of light in water,
𝑣
(2) Angle of minimum deviation (D) : 𝑐 3 𝑋 108 3 𝑋 108 9 𝑋 108
𝑣= = = = = 𝟐. 𝟐𝟓 𝑿 𝟏𝟎𝟖 𝒎 𝒔−𝟏
𝑛 1.33 4 4
( )
3
 Frequency is same in both air and water medium, then
𝑐 3 𝑋 108 3 𝑋 1018
 = = =
1 5893 𝑋 10−10 5893
 = 5. 090 𝑋 10−4 𝑋 1018 = 𝟓. 𝟎𝟗𝟎 𝑿 𝟏𝟎𝟏𝟒 𝑯𝒛
2. Two light sources with amplitudes 5 units and 3 units respectively interfere
with each other. Calculate the ratio of maximum and minimum intensities.
-Solution :- 𝑎1 = 5 ; 𝑎2 = 3
 Resultant amplitude,
 The prism is placed on the prism table, so that the light from the collimator falls
A = √𝑎12 + 𝑎22 + 2 𝑎1 𝑎2 cos 𝜙
on a refracting face and the refracted image is observed through the telescope.
 The prism table is now rotated, so that the angle of deviation decreases.  When 𝜙 = 0 (𝑜𝑟) cos 𝜙 = 1 , then resultant amplitude will be maximum.
 A stage comes when the image stops for a moment and if we rotate the prism
table further in the same direction, the image is seen to recede and the angle of A𝑚𝑎𝑥 = √𝑎12 + 𝑎22 + 2 𝑎1 𝑎2 (1)
deviation increases. A𝑚𝑎𝑥 = √( 𝑎1 + 𝑎2 )2 = 𝑎1 + 𝑎2 = 5 + 3
 The vertical cross wire of telescope is made to coincide with the image of the 𝐀𝒎𝒂𝒙 = 𝟖 units
slit, where it turns back. This gives the minimum deviation position.  When 𝜙 = 180° (𝑜𝑟) cos 𝜙 = − 1 then resultant amplitude will be
 The vernier readings corresponding to this position is noted. minimum.
 Now the prism is removed and the telescope is turned to receive the direct ray
 A𝑚𝑖𝑛 = √𝑎12 + 𝑎22 + 2 𝑎1 𝑎2 (−1)
and the vernier readings are again noted.
 The difference between the two readings gives the angle of minimum deviation A𝑚𝑖𝑛 = √( 𝑎1 − 𝑎2 )2 = 𝑎1 − 𝑎2 = 5 − 3
(D) 𝐀𝒎𝒊𝒏 = 𝟐 units
Refractive index (𝝁) of the prism :  Intensity is directly proportional to the square of the amplitude (𝐼 ∝ 𝐴2 )
2
 The refractive index of the material of the prism is calculated using the formula, 𝐼𝑚𝑎𝑥 𝐴𝑚𝑎𝑥 82 64 16
𝑨+𝑫 = 2 = 2= =
𝐬𝐢𝐧 ( ) 𝐼𝑚𝑖𝑛 𝐴𝑚𝑖𝑛 2 4 1
𝝁= 𝟐 𝑰𝒎𝒂𝒙 ∶ 𝑰𝒎𝒊𝒏 = 𝟏𝟔 ∶ 𝟏
𝑨
𝐬𝐢𝐧 ( )
𝟐

victory R. SARAVANAN. M.Sc., M.Phil., B.Ed PG ASST [PHYSICS], GBHSS, PARANGIPETTAI - 608 502
12 PHYSICS UNIT – 6 RAY OPTICS COMPLETE GUIDE AND MODEL QUESTION
3. Two light sources of equal amplitudes interfere with each other. Calculate the And distance of 3rd dark fringe,
ratio of maximum and minimum intensities. 𝐷 5𝜆 2 5 𝑋 450 𝑋 10−9
-Solution :- 𝑦3 = = 𝑋 = 5 𝑋 3000 𝑋 10−6
𝑑 2 0.15 𝑋 10−3 2
 Let ‘a’ be the amplitude ,then equation for intensity 𝒚𝟑 = 15000 𝑋 10−6 = 𝟏𝟓 𝑿 𝟏𝟎−𝟑 𝒎 = 𝟏𝟓 𝒎𝒎
𝜙 (b) Fringe width,
𝐼 ∝ 4 𝑎2 𝑐𝑜𝑠 2 [ ]
2 𝜆𝐷 450 𝑋 10−9 𝑋 2
𝜙 𝛽= = = 3000 𝑋 10−6 𝑋 2 = 𝟔 𝑿 𝟏𝟎−𝟑 𝒎 = 𝟔 𝒎𝒎
 When, 𝜙 = 0 (𝑜𝑟) cos ( ) = 1 then, intensity will be maximum. 𝑑 0.15 𝑋 10−3
2
(c) Since, 𝜷 ∝ 𝑫 , when distance (D) between slit and screen increases, the fringe
𝐼𝑚𝑎𝑥 ∝ 4 𝑎2
𝜙 width (𝜷) also increases.
 When 𝜙 = 180 (𝑜𝑟) cos ( ) = 0 then, intensity will be minimum. 4
2 (d) When, 𝑛 = ,then fringe width (𝛽1 ) in water
𝐼𝑚𝑖𝑛 = 0 3
𝜆1 𝐷 𝜆𝐷 𝛽 6 𝑋 10−3 3 𝑋 6 𝑋 10−3 18 𝑋 10−3 𝜆
 Hence the ratio of maximum and minimum , 1
𝛽 = = = = = = [∵ 𝜆1 = ]
𝑰𝒎𝒂𝒙 ∶ 𝑰𝒎𝒊𝒏 = 𝟒 𝒂𝟐 ∶ 𝟎 𝑑 𝑛𝑑 𝑛 4 4 4 𝑛
( )
3
4. Two light sources have intensity of light as I0. What is the resultant intensity at
𝜷𝟏 = 𝟒. 𝟓 𝑿 𝟏𝟎−𝟑 𝒎 = 𝟒. 𝟓 𝒎𝒎
a point where the two light waves have a phase difference of π/3?
𝝅 7. Lights of two wavelengths 560 nm and 420 nm are used in Young’s double slit
-Solution :- 𝜙= experiment. Find the least distance from the central fringe where the bright
𝟑
𝜙
 Resultant intensity, 𝐼 ∝ 4 𝑎2 𝑐𝑜𝑠 2 [ ] [∵ 𝐼𝑂 ∝ 𝑎2 ] fringes of the two wavelengths coincide.
2
𝜋 2
-Solution :- 𝝀𝟏 = 560 𝑛𝑚 = 560 𝑋 10−9 𝑚 ; 𝝀𝟐 = 420 𝑛𝑚 = 420 𝑋 10−9 𝑚
𝜙 (3) 𝜋 √3 3 𝟏
𝐼 = 4 𝐼𝑂 𝑐𝑜𝑠 2 [ ] = 4 𝐼𝑂 𝑐𝑜𝑠 2 [ ] = 4 𝐼𝑂 𝑐𝑜𝑠 2 [ ] = 4 𝐼𝑂 [ ] = 4 𝐼𝑂 [ ]  Here for given ‘y’, 𝝀 ∝ .Here, nth order bright fringe of longer wavelength λ1
2 2 6 2 4 𝒏
coincides with (n+1)th order bright fringe of shorter wavelength λ2,
𝑰 = 𝟑 𝑰𝑶 𝐷
5. The wavelength of a light is 450 nm. How much phase it will differ for a path of  Distance of nth bright fringe ; 𝑦𝑛 = 𝑛𝜆
𝑑
3 mm? 𝐷 𝐷
 Hence, 𝑛 𝜆1 = (𝑛 + 1) 𝜆2
-Solution :- 𝜆 = 450 𝑛𝑚 = 450 𝑋 10−9 𝑚 ; 𝛿 = 3 𝑚𝑚 = 3 𝑋 10−3 𝑚 𝑑 𝑑
𝑛 𝜆1 = (𝑛 + 1) 𝜆2
 Phase difference, (𝑛 + 1) 𝜆1
2𝜋 2𝜋 𝝅 (𝑜𝑟) =
𝜙= 𝛿= 𝑋 3 𝑋 10−3 = 𝑿𝟏𝟎𝟔 𝒓𝒂𝒅 = 𝟒. 𝟏𝟗 𝑿 𝟏𝟎𝟒 𝒓𝒂𝒅 𝑛 𝜆2
𝜆 450 𝑋 10−9 𝟕𝟓 1 𝜆1
6. In Young’s double slit experiment, the two slits are 0.15 mm apart. The light (𝑜𝑟) 1+ =
𝑛 𝜆2
source has a wavelength of 450 nm. The screen is 2 m away from the slits. 1 𝜆1
(a) Find the distance of the second bright fringe and also third dark fringe = −1
from the central maximum. 𝑛 𝜆2
(b) Find the fringe width. 1 560 𝑋 10−9
= −1
(c) How will the fringe pattern change if the screen is moved away from the 𝑛 420 𝑋 10−9
1 4 1
slits? = −1=
(d) What will happen to the fringe width if the whole setup is immersed in 𝑛 3 3
waterof refractive index 4/3. ∴ 𝒏=𝟑
-Solution :- 𝑑 = 0.15 𝑚𝑚 = 0.15 𝑋 10−3 𝑚 ; 𝐷 = 2 𝑚 ; 𝜆 = 450 𝑛𝑚 = 450 𝑋 10−9 𝑚  Thus, the 3rd bright fringe of λ1 and 4th bright fringe of λ2 coincide at the least
𝐷 distance y.
(a) Distance of nth bright fringe from centre, 𝑦𝑛 = 𝑛 𝜆
𝑑  The least distance from the central fringe where the bright fringes of the two
𝐷 𝜆
Distance of nth dark fringe centre, 𝑦𝑛 = (2𝑛 − 1) wavelengths coincides is
𝑑 2
Hencedistance of 2 bright fringe,
nd 𝐷 1
𝑦𝑛 = 𝑛𝜆= 𝑋 3 𝑋 560 𝑋 10−9
𝐷 2 𝑑 3 𝑋 10−3
𝑦2 = 𝑛𝜆= 𝑋 2 𝑋 450 𝑋 10−9 = 2 𝑋 2 𝑋 3000 𝑋 10−6 𝒚𝒏 = 𝟓𝟔𝟎 𝑿 𝟏𝟎−𝟔 𝒎 = 𝟎. 𝟓𝟔 𝑿 𝟏𝟎−𝟑 𝒎 = 𝟎. 𝟓𝟔 𝒎𝒎
𝑑 0.15 𝑋 10−3
𝒚𝟐 = 12000 𝑋 10−6 = 𝟏𝟐 𝑿 𝟏𝟎−𝟑 𝒎 = 𝟏𝟐 𝒎𝒎

victory R. SARAVANAN. M.Sc., M.Phil., B.Ed PG ASST [PHYSICS], GBHSS, PARANGIPETTAI - 608 502
12 PHYSICS UNIT – 6 RAY OPTICS COMPLETE GUIDE AND MODEL QUESTION
−𝟑
8. Find the minimum thickness of a film of refractive index 1.25, which will 𝒚𝟏 = 𝟏. 𝟓 𝑿𝟏𝟎 𝒎 = 𝟏. 𝟓 𝒎𝒎
strongly reflect the light of wavelength 589 nm. Also find the minimum For second minimum (𝑛 = 2)
thickness of the film to be anti-reflecting. 𝑎 sin 𝜃 = 2 𝜆
-Solution :- 𝜆 = 589 𝑛𝑚 = 589 𝑋 10−9 𝑚 ; 𝜇 = 1.25 𝑦2
𝑎 = 2𝜆
 For the film to have strong reflection, the reflected waves should interfere 𝐷
constructively. Hence the path difference, 2𝜆𝐷 2 𝑋 500 𝑋 10−9 𝑋60 𝑋 10−2
𝜆 ∴ 𝑦2 = =
𝑎 0.2 𝑋 10−3
2 𝜇 𝑡 = (2𝑛 − 1) 20 𝑋 500 𝑋10−9 𝑋 60 𝑋 10−2
2 𝑦2 = = 300000 𝑋10−8
 The least optical path difference, 𝑛 = 1, then 2 𝑋 10−3
𝜆 𝒚𝟐 = 𝟑. 𝟎 𝑿𝟏𝟎−𝟑 𝒎 = 𝟑 𝒎𝒎
2𝜇𝑡 = (b) The distance between the central maximum and second minimum is,
2
𝜆 589 𝑋 10−9 589 𝑋 10−9 𝒚𝟐 − 𝒚𝟏 = 𝟑 − 𝟏. 𝟓 = 𝟏. 𝟓 𝒎𝒎
∴ 𝑡= = = 10. A monochromatic light of wavelength 5000 Å passes through a single slit
4𝜇 4 𝑋 1.25 5
𝒕 = 𝟏𝟏𝟕. 𝟖 𝑿 𝟏𝟎−𝟗 𝒎 = 𝟏𝟏𝟕. 𝟖 𝒏𝒎 producing diffraction pattern for the central maximum as shown in the figure.
 For the film to be anti-reflecting, the reflected rays should interfere Determine the width of the slit.
destructively. -Solution :- 𝜆 = 5000 𝐀 = 5000 𝑋 10−10 𝑚 ;  = 30 ; 𝑛 = 1
Hence the path difference,  Equation for diffraction minimum is,
2𝜇𝑡 =𝑛𝜆 𝑎 sin 𝜃 = 𝑛 𝜆
 The least optical path difference, 𝑛 = 1, then  For first minimum (𝑛 = 1)
2𝜇𝑡 =𝜆 𝑎 sin 𝜃 = 𝜆
𝜆 589 𝑋 10−9 589 𝑋 10−9 5890 𝑋 10−9 𝜆
∴ 𝑡= = = = (𝑜𝑟) 𝑎=
2𝜇 2 𝑋 1.25 2.5 25 sin 𝜃
𝒕 = 𝟐𝟑𝟓. 𝟔 𝑿 𝟏𝟎−𝟗 𝒎 = 𝟐𝟑𝟓. 𝟔 𝒏𝒎 5000 𝑋 10−10 5000 𝑋 10−10
𝑎= =
9. Light of wavelength 500 nm passes through a slit of 0.2 mm wide. The sin 30° 1
( )
diffraction pattern is formed on a screen 60 cm away. Determine the, 2
(a) angular spread of central maximum 𝑎 = 10000 𝑋 10−10 = 1 𝑋 10−6 𝑚 = 0.001 𝑋 10−3
(b) the distance between the central maximum and the second minimum. 𝒂 = 𝟎. 𝟎𝟎𝟏 𝒎𝒎
-Solution :- 𝜆 = 500 𝑛𝑚 = 500 𝑋 10−9 𝑚 ; 𝑎 = 0.2 𝑚𝑚 = 0.2 𝑋 10−3 𝑚 11. Calculate the distance upto which ray optics is a good approximation for light
𝐷 = 60 𝑐𝑚 = 60 𝑋 10−2 𝑚 of wavelength 500 nm falls on an aperture of width 0.5 mm.
(a) Equation for diffraction minimum is, 𝑎 sin 𝜃 = 𝑛 𝜆 -Solution :- 𝜆 = 500 𝐧𝐦 = 500 𝑋 10−9 𝑚 ; 𝑎 = 0.5 𝑚𝑚 = 0.5 𝑋 10−3 𝑚
The central maximum is spread up to the first minimum. Hence, n = 1 𝑎2
 Fresnel distance, 𝑧 =
𝑎 sin 𝜃 = 𝜆 2𝜆
𝜆 500 𝑋 10−9 5000 𝑋 10−6 (0.5 𝑋 10−3 )2 0.25 𝑋 10−6 0.25 𝑋 10−6
(𝑜𝑟) sin 𝜃 = = = = 2500 𝑋10−6 𝑧= −9
= −9
=
𝑎 0.2 𝑋 10−3 2 2 𝑋 500 𝑋 10 1000 𝑋 10 10−6
(𝑜𝑟) sin 𝜃 = 0.0025 𝒛 = 𝟎. 𝟐𝟓 𝒎 = 𝟐𝟓 𝒄𝒎
(𝑜𝑟) 𝜽 = 𝐬𝐢𝐧−𝟏 𝟎. 𝟎𝟎𝟐𝟓 = 𝟎. 𝟎𝟎𝟐𝟓 𝒓𝒂𝒅 12. A diffraction grating consists of 4000 slits per centimeter. It is illuminated by a
From the figure, for first mininmum monochromatic light. The second order diffraction maximum is produced at an
𝑦1 angle of 30°. What is the wavelength of the light used?
tan 𝜃 = -Solution :- 𝑁 = 4000 𝑙𝑖𝑛𝑒𝑠 ⁄𝑐𝑚 = 400000 𝑙𝑖𝑛𝑒𝑠 ⁄𝑚 ;  = 30 ; 𝑚 = 2
𝐷
As 𝜃 is small, sin 𝜃 ≈ tan 𝜃 For first minimum  Equation for diffraction maximum for grating is,
𝑎 sin 𝜃 = 𝜆 sin 𝜃 = 𝑁 𝑚 𝜆
𝑦1 1
𝑎 =𝜆 sin 𝜃 sin 30° ( ) 1
𝐷 𝜆= = = 2 = 𝑋 10−5
𝜆𝐷 500 𝑋 10 𝑋60 𝑋 10−2
−9
5000 𝑋 10−9 𝑋 60 𝑋 10−2 𝑁𝑚 400000 𝑋 2 8 𝑋 105 16
∴ 𝑦1 = = −3
= = 150000 𝑋10−8 𝝀 = 𝟎. 𝟎𝟔𝟐𝟓 𝑿 𝟏𝟎−𝟓 = 𝟔𝟐𝟓𝟎 𝑿 𝟏𝟎−𝟏𝟎 𝒎 = 𝟔𝟐𝟓𝟎 𝑨°
𝑎 0.2 𝑋 10 2 𝑋 10−3
victory R. SARAVANAN. M.Sc., M.Phil., B.Ed PG ASST [PHYSICS], GBHSS, PARANGIPETTAI - 608 502
12 PHYSICS UNIT – 6 RAY OPTICS COMPLETE GUIDE AND MODEL QUESTION
13. A monochromatic light of wavelength of 500 nm strikes a grating and produces (b) If third polaroid 𝑃3 kept between 𝑃1 & 𝑃2 at 45o, the intensity of emergent
fourth order maximum at an angle of 30°. Find the number of slits per light from 𝑃3
centimeter. 𝐼 𝐼 1 2 𝐼 1 𝐈
-Solution :- 𝜆 = 500 𝑛𝑚 = 500 𝑋 10−9 𝑚 ;  = 30 ; 𝑚 = 4 𝐼1 = 𝐼𝑂 𝑐𝑜𝑠 2 𝜃 = 𝑐𝑜𝑠 2 45° = ( ) = 𝑋 =
2 2 √2 2 2 𝟒
 Equation for diffraction maximum for grating is ; sin 𝜃 = 𝑁 𝑚 𝜆 Finally, the intensity of emergent light from 𝑃2 ,
1 I I 1 2 I 1 𝐈
sin 𝜃 sin 30° ( ) 1 𝐼11 = 𝐼1 𝑐𝑜𝑠 2 𝜃 = 𝑐𝑜𝑠 2 45° = ( ) = 𝑋 =
∴ 𝑁= = = 2 = 𝑋 106 = 0.25 𝑋 106
−9 −9 4 4 √2 4 2 𝟖
𝜆𝑚 500 𝑋 10 𝑋 4 2000 𝑋 10 4
17. Find the polarizing angles for (i) glass of refractive index 1.5 and (ii) water of
𝑵 = 𝟐. 𝟓 𝑿 𝟏𝟎𝟓 𝒍𝒊𝒏𝒆𝒔⁄𝒎 = 𝟐𝟓𝟎𝟎 𝒍𝒊𝒏𝒆𝒔⁄𝒄𝒎
refractive index 1.33.
14. The optical telescope in the Vainu Bappu observatory at Kavalur has an -Solution :-
objective lens of diameter 2.3 m. What is its angular resolution if the (i) When, 𝑛 = 1.5 , from Brewster’s law ; tan 𝑖𝑃 = 𝑛 (𝑜𝑟) 𝑖𝑃 = tan−1 𝑛
wavelength of light used is 589 nm?
𝒊𝑷 = 𝐭𝐚𝐧−𝟏 𝟏. 𝟓 = 𝟓𝟔. 𝟑°
-Solution :- 𝜆 = 589 𝑛𝑚 = 589 𝑋 10−9 𝑚 ; 𝑎 = 2.3 𝑚
(ii) When, 𝑛 = 1.33 from Brewster’s law ;, tan 𝑖𝑃 = 𝑛 (𝑜𝑟) 𝑖𝑃 = tan−1 𝑛
 Equation for angular resolution,
1.22 𝜆 1.22 𝑋 589 𝑋 10−9 𝒊𝑷 = 𝐭𝐚𝐧−𝟏 𝟏. 𝟑𝟑 = 𝟓𝟑. 𝟏°
 = = 18. What is the angle at which a glass plate of refractive index 1.65 is to be kept
𝑎 2.3
 = 3. 125 𝑋 102 𝑋 10−9 = 𝟑. 𝟏𝟐𝟓 𝑿 𝟏𝟎−𝟕 𝒓𝒂𝒅𝒊𝒂𝒏 ≈ 𝟎. 𝟎𝟎𝟏𝟏 with respect to the horizontal surface so that an unpolarised light travelling
horizontal after reflection from the glass plate is found to be plane polarised?
15. Two polaroids are kept with their transmission axes inclined at 30 o.
-Solution :- 𝑛 = 1.65
Unpolarised light of intensity I falls on the first polaroid. Find out the intensity
of light emerging from the second polaroid.  From Brewster’s law ; tan 𝑖𝑃 = 𝑛
-Solution :- (𝑜𝑟) 𝑖𝑃 = tan−1 𝑛 = 𝐭𝐚𝐧−𝟏 𝟏. 𝟔𝟓 = 𝟓𝟖. 𝟖°
 As the intensity of the unpolarised light falling on the first polaroid is I, the  The angle at which a glass plate to be kept with respect to the horizontal surface
intensity of polarized light emerging from it will be, = 𝟗𝟎° − 𝟓𝟖. 𝟖° = 𝟑𝟏. 𝟐°
𝑰 19. A man with a near point of 25 cm reads a book which has small print using a
𝑰𝑶 =
𝟐 magnifying lens of focal length 5 cm.
 Let 𝐼1 be the intensity of light emerging from the second polaroid. (a) What are the closest and the farthest distances at which he should keep the
𝐼1 = 𝐼𝑂 𝑐𝑜𝑠 2 𝜃 lens from the book?
2
𝐼 𝐼 √3 𝐼 3 (b) What are the maximum and the minimum magnification possible?
𝐼 = 𝑐𝑜𝑠 2 30° =
1
( ) = 𝑋 -Solution :- 𝐷 = 25 𝑐𝑚 ; 𝑓 = 5 𝑐𝑚
2 2 2 2 4
𝟑  Closest distance of object =𝑢
𝟏
𝑰 = ( )𝑰 Image distance (near point focusing) = 𝑣 = −25 𝑐𝑚
𝟖
16. Two polaroids are kept crossed (transmission axes at 90o) to each other.  Farthest distance of object = 𝑢1
(a) What will be the intensity of the light coming out from the second polaroid Image distance (normal focusing) = 𝑣1 = ∞
when an unpolarised light of intensity I falls on the first polaroid? (a) Lens equation for near point focusing,
(b) What will be the intensity of light coming out from the second polaroid if a 1 1 1 1 1 1 1 1 1 1 −1 − 5 6
− = (𝑜𝑟) = − = − =− − = = −
third polaroid is kept in between at 45o inclination to both of them. 𝑣 𝑢 𝑓 𝑢 𝑣 𝑓 (−25) 5 25 5 25 25
-Solution :- 𝟐𝟓
𝒖= − = −𝟒. 𝟏𝟔𝟕 𝒄𝒎
(a) As the intensity of the unpolarised light falling on the first polaroid 𝑃1 is I, the 𝟔
𝑰 Lens equation for normal focusing,
intensity of polarized light emerging from it will be ; 𝑰𝑶 =
𝟐 1 1 1 1 1 1 1 1 1 1
Let 𝐼1 , be the intensity of light emerging from the second polaroid 𝑃2 . From 1
− 1 = 1 (𝑜𝑟) 1 = 1 − 1 = − = −0− =−
𝑣 𝑢 𝑓 𝑢 𝑣 𝑓 ∞ 5 5 5
Malus’ law 𝒖𝟏 = − 𝟓 𝒄𝒎
𝐼 The closest distance between the lens and the book is, u = −𝟒. 𝟏𝟔𝟕 𝒄𝒎
𝐼1 = 𝐼𝑂 𝑐𝑜𝑠 2 𝜃 = 𝑐𝑜𝑠 2 90° = 0
2 The farthest distance at which the person can keep the book is = −𝟓𝒄𝒎
No light comes out from the second polaroid.

victory R. SARAVANAN. M.Sc., M.Phil., B.Ed PG ASST [PHYSICS], GBHSS, PARANGIPETTAI - 608 502
12 PHYSICS UNIT – 6 RAY OPTICS COMPLETE GUIDE AND MODEL QUESTION
(b) Magnification in near point focusing, 22. Calculate the power of the lens of the spectacles needed to rectify the defect of
𝐷 25 nearsightedness for a person who could see clearly up to a distance of 1.8 m.
𝒎= 1+ = 1+ = 1+5= 𝟔
𝑓 5 -Solution :- 𝑥 = 1.8 𝑚
Magnification in normal focusing,  The lens should have a focal length of 𝒇 = −𝒙 = −𝟏. 𝟖 𝒎
𝐷 25  It is a concave (or) diverging lens.
𝒎= = =𝟓
𝑓 5  The power of the lens is,
20. A microscope has an objective and eyepiece of focal lengths 5 cm and 50 cm 1 1
𝑃 = =−
respectively with tube length 30 cm. Find the magnification of the microscope 𝑓 1.8
in the (a) near point and (b) normal focusing. 𝑷 = −𝟎. 𝟓𝟔 diopter
-Solution :- 𝑓𝑂 = 5 𝑐𝑚 = 5 𝑋 10−2 𝑚 ; 𝑓𝑒 = 50 𝑐𝑚 = 50 𝑋 10−2 𝑚 23. A person has farsightedness with the far distance he could see clearly is 75 cm.
𝐿 = 30 𝑐𝑚 = 30 𝑋 10−2 𝑚 ; 𝐷 = 25 𝑐𝑚 = 25 𝑋 10−2 𝑚 Calculate the power of the lens of the spectacles needed to rectify the defect.
(a) Magnification of the microscope in near point focusing, -Solution :- 𝑦 = 75 𝑐𝑚 = 75 𝑋 10−2 𝑚
𝐿 𝐷  The lens should have a focal length of 𝑓,
𝑚 = 𝑚𝑜 𝑚𝑒 = [ ] [1 + ] 𝑦 𝑋 25 𝑐𝑚
𝑓𝑂 𝑓𝑒
𝑓=
30 𝑋 10−2 25 𝑋 10−2 𝑦 − 25 𝑐𝑚
𝑚 = 𝑚𝑜 𝑚𝑒 = [ ] [1 + ] 75 𝑐𝑚 𝑋 25 𝑐𝑚 1875
5 𝑋 10−2 50 𝑋 10−2 𝑓= = 𝑐𝑚
1 3 18 75 𝑐𝑚 − 25 𝑐𝑚 50
𝑚 = 𝑚𝑜 𝑚𝑒 = [6] [1 + ] = 6 𝑋 = =𝟗 𝟏𝟖𝟕𝟓
2 2 2 𝒇= 𝒄𝒎 = 𝟑𝟕. 𝟓 𝒄𝒎
(b) Magnification of the microscope in normal focusing, 𝟓𝟎
𝐿 𝐷  It is a convex lens (or) converging lens.
𝑚 = 𝑚𝑜 𝑚𝑒 = [ ] [ ]  The power of the lens is,
𝑓𝑂 𝑓𝑒
30 𝑋 10−2 25 𝑋 10−2 1 1 10−2 100
𝑚 = 𝑚𝑜 𝑚𝑒 = [ ][ ] 𝑃 = =− −2
= =
5 𝑋 10−2 50 𝑋 10−2 𝑓 37.5 𝑋 10 37.5 37.5
1 6 𝑷 = 𝟐. 𝟔𝟕 diopter
𝑚 = 𝑚𝑜 𝑚𝑒 = [6] [ ] = =𝟑
2 2
21. A small telescope has an objective lens of focal length 125 cm and an eyepiece
of focal length 2 cm. (a) What is the magnification of the telescope? (b) What is
the separation between the objective and the eyepiece?(c) What is the angular
separation between two stars when viewed through this telescope if they
subtend 1 ' for bare eye?
-Solution :- 𝑓𝑂 = 125 𝑐𝑚 = 125 𝑋 10−2 𝑚 ; 𝑓𝑒 = 2 𝑐𝑚 = 2 𝑋 10−2 𝑚
(a) Magnification of telescope,
𝑓𝑂 125 𝑋 10−2 125
𝑚= = −2
=
𝑓𝑒 2 𝑋 10 2
𝒎 = 𝟔𝟐. 𝟓
(b) Equation for approximate length of telescope,
𝐿 = 𝑓𝑂 + 𝑓𝑒
𝐿 = (125 𝑋 10−2 ) + (2 𝑋 10−2 ) = (125 + 2) 𝑋 10−2
𝑳 = 𝟏𝟐𝟕 𝑿 𝟏𝟎−𝟐 𝒎 = 𝟏. 𝟐𝟕 𝒎
(c) The angular separation,
𝜃𝑖
𝑚= (𝑜𝑟) 𝜃𝑖 = 𝑚 𝜃𝑜
𝜃𝑜
𝟔𝟐. 𝟓
𝜽𝒊 = 𝟔𝟐. 𝟓 𝑿 𝟏 = 𝟔𝟐. 𝟓 = = 𝟏. 𝟎𝟒°
𝟔𝟎
victory R. SARAVANAN. M.Sc., M.Phil., B.Ed PG ASST [PHYSICS], GBHSS, PARANGIPETTAI - 608 502
12 PHYSICS UNIT – 6 RAY OPTICS COMPLETE GUIDE AND MODEL QUESTION
4. A beam of light of wavelength 600 nm from a distant source falls on a single slit
EXERCISE PROBLEMS WITH SOLUTIONS 1 mm wide and the resulting diffraction pattern is observed on a screen 2 m
1. The ratio of maximum and minimum intensities in an interference pattern is away. What is the distance between the first dark fringes on either side of the
36 : 1. What is the ratio of the amplitudes of the two interfering waves? central bright fringe?
-Solution :- 𝐼𝑚𝑎𝑥 ∶ 𝐼𝑚𝑖𝑛 = 36 ∶ 1 -Solution :- 𝑎 = 1𝑚𝑚 = 1 𝑋 10−3 𝑚 ;
𝐼𝑚𝑎𝑥 (𝑎1 +𝑎2 )2 𝑎1 +𝑎2 𝐼 36 𝜆 = 600 𝑛𝑚 = 600 𝑋 10−9 𝑚 ; 𝐷 = 2 𝑚
 We have, = (𝑎1 −𝑎2 )2
(𝑜𝑟) = √ 𝐼𝑚𝑎𝑥 = √ 1 = 6  Equation for diffraction minimum,
𝐼𝑚𝑖𝑛 𝑎1 −𝑎2 𝑚𝑖𝑛
𝑦
 Hence, 𝑎1 + 𝑎2 = 6 (𝑎1 − 𝑎2 ) 𝑎 𝑠𝑖𝑛 𝜃 = 𝑛 𝜆 (𝑜𝑟) 𝑎 = 𝑛 𝜆
𝐷
𝑎1 + 𝑎2 = 6 𝑎1 − 6 𝑎2  Condition for 1st minimum (n=1)
𝑎2 + 6 𝑎2 = 6 𝑎1 − 𝑎1 𝑦
𝑎 =𝜆
7 𝑎2 = 5 𝑎1 𝐷
𝑎1 7 𝐷𝜆 2 𝑋 600 𝑋 10−9
= ∴ 𝑦 = = = 1200 𝑋 10−6 𝑚 = 1.2 𝑋10−3 𝑚 = 1.2 𝑚𝑚
𝑎2 5 𝑎 1 𝑋 10−3
𝒂𝟏 ∶ 𝒂𝟐 = 𝟕 ∶ 𝟓  Hence distance between the first fringe on either side of the central bright fringe
2. In Young’s double slit experiment, 62 fringes are seen on a screen for sodium 𝒀 = 2 𝑦 = 2 𝑋1.2 = 𝟐. 𝟒 𝒎𝒎
light of wavelength 5893 Å. If violet light of wavelength 4359 Å is used in place 5. Light of wavelength of 5000 Å produces diffraction pattern of the single slit of
of sodium light, how many fringes will be seen? width 2.5 μm. What is the maximum order of diffraction possible?
-Solution :- 𝜆1 = 5893 A ; 𝜆2 = 4359 A ; 𝑛1 = 62 -Solution :- 𝑎 = 2.5 𝜇𝑚 = 2.5 𝑋 10−6 𝑚 ; 𝜆 = 5000 Å = 5000 𝑋 10−10 𝑚
 Equation of fringe width ; 𝛽 =
𝜆𝐷  Equation for diffraction minimum : 𝒂 𝒔𝒊𝒏 𝜽 = 𝒏 𝝀
𝑑  For maximum order ; 𝜃 = 90° (𝑜𝑟) 𝑠𝑖𝑛 𝜃 = 1 . Hence
𝜆𝐷
 Hence fringe width of n-fringes , 𝑛 𝛽 = 𝑛 .Thus 𝒂 𝒔𝒊𝒏 𝜽 2.5 𝑋 10−6 𝑋 1
𝑑
𝜆1 𝐷 𝜆2 𝐷 𝒏= = = 0.5 𝑋 101 = 𝟓
𝑛1 = 𝑛2 𝝀 5000 𝑋 10−10
𝑑 𝑑 6. I0 is the intensity of light existing between two cross Polaroids kept with their
𝑛1 𝜆1 = 𝑛2 𝜆2 axes perpendicular to each other. A third polaroid is introduced between
𝑛1 𝜆1 them. What must be the angle between the axes of first and the newly
𝑛2 =
𝜆2 introduced polaroid to get the maximum light from the whole arrangement?
62 𝑋 5893 𝑋 10−10 62 𝑋 5893 -Solution :-
𝑛2 = =
4359 𝑋 10−10 4359  If the intensity of the unpolarised light is I then the intensity of polarised light
𝒏𝟐 = 𝟖𝟑. 𝟖𝟏 ≈ 𝟖𝟒
will be I/2. The other half of intensity is restricted by the polariser.
3. Light of wavelength 600 nm that falls on a pair of slits producing interference
pattern on a screen in which the bright fringes are separated by 7.2 mm. What
must be the wavelength of another light which produces bright fringes
separated by 8.1 mm with the same apparatus?
-Solution :- 𝜆1 = 600 𝑛𝑚 = 600 𝑋 10−9 𝑚 ; 𝛽1 = 7.2𝑚𝑚 = 7.2 𝑋 10−3 𝑚 ;
𝛽2 = 8.1 𝑚𝑚 = 8.1 𝑋 10−3 𝑚
𝜆𝐷
 Equation of fringe width ; 𝛽= 𝑑
 Hence,
𝛽1 𝜆1 𝛽2  Intensity of incident light on 𝑃1 = 𝐼𝑜
= (or) 𝜆2 = 𝜆1 𝐼𝑜
𝛽2 𝜆2 𝛽1  Intensity of emergent light from 𝑃1 ; 𝐼1 =
−3 2
8.1 𝑋 10 4860 𝐼
𝜆2 = 600 𝑋 10−9 𝑋 −3
= 𝑋 10−9 = 675 𝑋 10−9 𝑚 = 675 𝑛𝑚  Intensity of emergent light from 𝑃2 ; 𝐼2 = 𝐼1 cos 2 𝜃 = 𝑜 cos 2 𝜃
7.2 𝑋 10 7.2 2
 Intensity of emergent light from 𝑃3 ; 𝐼3 = 𝐼2 cos 2 (90 − 𝜃)
𝐼𝑜
𝐼3 = cos 2 𝜃 cos 2 (90 − 𝜃)
2
victory R. SARAVANAN. M.Sc., M.Phil., B.Ed PG ASST [PHYSICS], GBHSS, PARANGIPETTAI - 608 502
12 PHYSICS UNIT – 6 RAY OPTICS COMPLETE GUIDE AND MODEL QUESTION
2
𝐼𝑜 𝐼𝑜 𝐼𝑜 𝑠𝑖𝑛 2 𝜃 8. The reflected light is found to be plane polarised when an unpolarized light
𝐼3 = cos 2 𝜃 sin2 𝜃 = [𝑠𝑖𝑛 𝜃 𝑐𝑜𝑠 𝜃]2 = [ ] falls on a denser medium at 60° with the normal. Find the angle of refraction
2 2 2 2
2
𝐼𝑜 sin 2𝜃 𝐼𝑜 and critical angle of incidence for total internal reflection in the denser to
𝐼3 = [ ]= sin2 2𝜃
2 4 8 rarer medium reflection.
 When sin2 2𝜃 = 𝑚𝑎𝑥𝑖𝑚𝑢𝑚 , then 𝐼3 will bemaximum. (i.e.) Solution :-
sin2 2𝜃 = 1 (𝑜𝑟) 𝑠𝑖𝑛 2𝜃 = 1 (𝑜𝑟) 2𝜃 = 90°  The angle of incidence at which the reflected ray get completely plane polarized
∴ 𝜽 = 𝟒𝟓° is called angle of polarization (𝑖𝑃 ). Hence 𝒊𝑷 = 𝟔𝟎°
7. An unpolarised light of intensity 32 Wm–2 passes through three Polaroids such  At polarizing angle, the angle of refraction ,
that the axes of the first and the last Polaroids are at 90°. What is the angle 𝐫 = 90° − 𝑖𝑃 = 90° − 60° = 𝟑𝟎°
between the axes of the first and middle Polaroids so that the emerging light  From Brewster’s law, 𝑛 = tan 𝑖𝑃 = tan 60° = √3
has an intensity of only 3 Wm–2?  Let 𝑖𝐶 be the critical angle, then
-Solution :- : 𝐼𝑜 = 32 𝑊𝑚−2 ; 𝐼3 = 3 𝑊𝑚−2 1 1
 If the intensity of the unpolarised light is I then the intensity of polarised light sin 𝑖𝐶 = = = 0.5774
𝑛 √3
will be I/2. The other half of intensity is restricted by the polariser. 𝒊𝑪 = sin (0.5774) = 𝟑𝟓. 𝟐𝟔° = 𝟑𝟓°𝟏𝟔𝐈
−1

9. The near point and the far point for a person are 50 cm and 500 cm,
respectively. Calculate the power of the lens the person should wear to read a
book held in hand at 25 cm. What maximum distance is clearly visible for the
person with this lens on the eye?
Solution :-𝑢 = −25 𝑐𝑚 = −0.25 𝑚 ; 𝑣𝑛 = −50𝑐𝑚 = −0.5 𝑚 ; 𝑣𝑓 = −500𝑐𝑚 = −5𝑚
 Power of lens ;
1 1 1
𝑷= = −
 Intensity of incident light on 𝑃1 = 𝐼𝑜 𝑓 𝑣𝑛 𝑢
𝐼 1 1 1 1
 Intensity of emergent light from 𝑃1 ; 𝐼1 = 𝑜 𝑷= − =− + = −2 + 4 = 𝟐 𝑫
2 (−0.5) (−0.25) 0.5 0.25
𝐼𝑜
 Intensity of emergent light from 𝑃2 ; 𝐼2 = 𝐼1 cos 2 𝜃 = cos 2 𝜃  Let 𝑢𝑚𝑎𝑥 be the maximum distance, then
2
 Intensity of emergent light from 𝑃3 ; 𝐼3 = 𝐼2 cos 2 (90 − 𝜃) 1 1 1
𝑷= = −
𝐼𝑜 𝑓 𝑣𝑓 𝑢𝑚𝑎𝑥
𝐼3 = cos 2 𝜃 cos 2 (90 − 𝜃) 1 1 1 1
2 𝟐= − =− −
𝐼𝑜 𝐼𝑜 𝐼𝑜 𝑠𝑖𝑛 2 𝜃 2 (−5) 𝑢𝑚𝑎𝑥 5 𝑢𝑚𝑎𝑥
𝐼3 = cos 2 𝜃 sin2 𝜃 = [𝑠𝑖𝑛 𝜃 𝑐𝑜𝑠 𝜃]2 = [ ] 1 1 11
2 2 2 2 =− −2=−
2
𝐼𝑜 sin 2𝜃 𝐼𝑜 𝑢𝑚𝑎𝑥 5 5
𝐼3 = [ ]= sin2 2𝜃 5
2 4 8
𝑢𝑚𝑎𝑥 = − = − 0.454 𝑚 = − 45.45 𝑐𝑚
32 11
3= sin2 2𝜃  The maximum distance that clearly visible for the person with this lens on the
8
24 3 eye will be = 45.45 𝑐𝑚
(𝑜𝑟) sin2 2𝜃 = =
32 4
√3
(𝑜𝑟) 𝑠𝑖𝑛 2𝜃 =
2
√3
(𝑜𝑟) 2𝜃 = sin−1 [ ] = 60°
2
∴ 𝜽 = 𝟑𝟎°

victory R. SARAVANAN. M.Sc., M.Phil., B.Ed PG ASST [PHYSICS], GBHSS, PARANGIPETTAI - 608 502
12 PHYSICS UNIT – 6 RAY OPTICS COMPLETE GUIDE AND MODEL QUESTION
10. A compound microscope has a magnifying power of 100 when the image is
formed at infinity. The objective has a focal length of 0.5 cm and the tube CONCEPTUAL QUESTIONS AND ANSWERS
length is 6.5 cm. What is the focal length of the eyepiece. 1. What are the reasons to believe that light is a wave motion?
Solution :- 𝑚 = 100 ; 𝑓𝑜 = 0.5 𝑐𝑚 ; 𝐿 = 6.5 𝑐𝑚 ; 𝐷 = 25𝑐𝑚  Light undergoes interference, diffraction and polarization. These phenomena
 When the image is formed at infinity, the real image produced by objective lens establish that light is a wavemotion.
is formed at the focus of the eyepiece, then 𝑣𝑜 + 𝑓𝑒 = 𝐿 2. ‘A’ and ‘B’ are two points on a water surface where waves are generated. What
𝑣𝑜 + 𝑓𝑒 = 6.5 − − − − − (1) is the phase difference if (a) A and B are on the same wavefront but separated
in distance by the wavelength ‘’ (b) If A and B are on successive crests but
linearly separated by the distance 2.5  and (c) A and B are on successive
 Magnification of compound microscope ; troughs?
𝑣𝑜 𝐷 𝑣𝑜 𝐷 𝑣𝑜 25 a) Since A and B are on the same wavefront, the phase difference between them
𝑚= 𝑋 = − [1 − ] = − [1 − ] will be zero
𝑢𝑜 𝑓𝑒 𝑓𝑜 𝑓𝑒 𝑓𝑜 𝑓𝑒
𝑣𝑜 25 b) Given path difference ; 𝛿 = 2.5 , then
100 = − [1 − ] the phase diffence ; 𝜑 =
2𝜋
𝛿=
2𝜋
(2.5 ) = 𝟓𝝅
0.5 𝑓𝑒  
𝑣𝑜 1 c) Since A and B are at successive troughs, the path difference ; 𝛿 =  , then
4 = − [1 − ] 2𝜋 2𝜋
0.5 𝑓𝑒 the phase diffence ; 𝜑 = 𝛿= () = 𝟐𝝅
𝑣𝑜  
4 𝑓𝑒 = − 1 + = −1 + 2 𝑣𝑜 3. Can two wavefronts cross one another? Give reason.
0.5
(𝑜𝑟) 2 𝑣𝑜 − 4 𝑓𝑒 = 1 − − − − − (2)  No. If they intersect, then there will be two directions of propagation of energy
(1)𝑋 2 ⟹ 2 𝑣𝑜 + 2 𝑓𝑒 = 13 − − − − − (3) at the point of intersection which is not possible.
(3) − (2) ⟹ 6 𝑓𝑒 = 12 4. When a wave undergoes reflection at a denser medium, what happens to its
12 phase and path? Does any change in phase occurs during refraction?
𝒇𝒆 = = 𝟐 𝒄𝒎  When a wave reflected in to rarer medium from the surface of a denser medium,
6 
it undergoes a phase change of 𝝅 𝒓𝒂𝒅𝒊𝒂𝒏 and path change of
𝟐
 No phase change occurs during refraction.
5. What are the essential condition for two light waves to be coherent?
 The two waves must be continuous
 The two waves should be of same frequency or wavelength
 They should have a constant or zero phase difference
 Preferably they should have equal amplitude
6. Why are coherent sources necessary to produce a sustained interference
pattern?
 Coherent sources have a constant phase difference. This ensures that the
positions of maxima and minima do not change with time (i.e.) a sustained
interference pattern is obtained.
7. Two slits in Young’s double slit experiment are illuminated by two different
lamps emitting light of the same wavelength. Will you observe the
interference pattern? Justify your answer.
 No. The light waves emitted by two different lamps cannot be coherent since the
phase difference between them varies continuously with time. So the positions
of bright and dark fringes will change rapidly on the screen and such rapid
changes cannot be detected by our eyes thus by producing uniform illumination
on the screen.

victory R. SARAVANAN. M.Sc., M.Phil., B.Ed PG ASST [PHYSICS], GBHSS, PARANGIPETTAI - 608 502
12 PHYSICS UNIT – 6 RAY OPTICS COMPLETE GUIDE AND MODEL QUESTION
8. Can two independent light sources act as coherent sources. Why? 14. When a thin transparent film is placed just in front of one of the slits in the
 Two independent sources of light cannot be coherent. Young’s double slit experiment using white light, what change results in the
 This is because light is emitted by individual atoms, when they return to ground fringe system?
state. Since the source of light contains billions of atoms which obviously cannot  If ‘’ is the refractive index and ‘t’ is the thickness of the film, then the entire
emit light waves in the same phase. 𝑫 (𝝁−𝟏) 𝒕
interference pattern gets displaced by distance ; ∆𝒙 =
9. How does the fringe width in Young’s double slit experiment change, when the 𝒅
distance between the two slits is halved and the distance between the slits and  As ‘’ increases with decreasing in wavelength (), the violet fringe is shiffeted
the screen is doubled? farther than the red fringe. So there is a kind of dispersion in the central white
𝑫 fringe.
 Fringe width is given by ; 𝜷 =  But the fringe width remains the same.
𝒅
 When the distance ‘d’ between the slits is halved and the distance ‘D’ between 15. Explain the statement ‘light added to light can produce darkness’.
the slits and the screen is doubled, then  When two light waves of equal amplitude meet at a point in opposite phase, the
 (𝟐𝑫) 𝑫 resultant amplitude and hence intensity becomes zero at that point. (i.e.) When
𝜷𝟏 = 𝒅/𝟐
= 𝟒 𝒅
= 𝟒𝜷
light added to light undergoes destructive interference, it produce darkness.
(i.e.) fringe width increases four times 16. What happens to the light energy when light waves interfere destructively at a
10. Why should we have a narrow source to produce good interference fringes? point?
 Because, a broad source is equivalent to a large number of narrow sources lying  Energy gets transferred from the regions of destructive interference to the
close to each other. regions of constructive interference.
 Different pairs on narrow sources will produce their own interference patterns 17. What will be the effect on the fringes formed in Young’s double slit
which will overlap each other. So the fringe system is lost experiment, if the whole apparatus is immersed in water?
11. No interference pattern is detected when two coherent sources are infinitely  Let ‘’ be the wavelength of light in air and ‘’ be the refractive index of the
close to one another and also far apart. Why? 𝟏 
𝑫 water, then the wavelength of light in water is ;  = (i.e.) 𝟏 < 
 Fringe width is given by ; 𝜷 = 𝝁
𝒅 𝑫 𝟏 𝑫
 When 𝒅 = 𝟎, then 𝜷 = ∞ (i.e.) fringe width is very large. Even a single fringe  Fringe width in air ; 𝜷 = and in water ; 𝜷𝟏 =
𝒅 𝒅
occupy the entire screen. So the interference pattern cannot be observed Since D and d are constants, 𝜷 ∝  and 𝜷𝟏 ∝ 𝟏
𝟏
 Since 𝜷 ∝ , When the distance between the coherence sources is large, the  As 𝟏 <  , then 𝜷𝟏 < 𝜷 (i.e) fringe width decreases, when the apparatus is
𝒅
fringe width becomes too small to be detected. So the interference pattern immersed in water
cannot be observed. 18. Is there any difference between the colours emerging from a prism and the
12. Two identical coherent waves each of intensity 𝑰𝒐 are producing an colours of a soap film seen in sunlight?
interference pattern. Write the value of the resultant intensity at a point of (a)  Yes. In prism, colours are produced due to dispersion of white light. But in the
constructive interference and (b) destrictive interference case of soap film, the colours are produced due to interference of light waves
a) Condition for maximum intensity; 𝜙 = 0, ±2𝜋, ±4𝜋, … Hence Resultant intensity reflected or transmitted by the film
at point of constructive interference ; 𝑰𝒎𝒂𝒙 = 𝑰𝒐 + 𝑰𝒐 + 𝟐 √𝑰𝒐 𝑰𝒐 𝒄𝒐𝒔 𝝓 = 𝟒 𝑰𝒐 19. How an excessively thin film appears in reflected light?
b) Condition for minimum intensity; 𝜙 = ±𝜋, ±3𝜋, … HenceResultant intensity at  It appears as black. Because for an excessively thin film (t < < ), the factor
𝟐𝝁𝒕 𝐜𝐨𝐬 𝒓 is negligibly small. The effective path difference between any two
point of destructive interference ; 𝑰𝒎𝒊𝒏 = 𝑰𝒐 + 𝑰𝒐 + 𝟐 √𝑰𝒐 𝑰𝒐 𝒄𝒐𝒔 𝝓 = 𝟎
successive rays in reflected system is ⁄2 . This is the condition for minimum
13. In Young’s double slit experiment, lights of green, yellow and orange colours
are successively used. Write the fringe widths for three colours in increasing intensity and hence the film will appear dark.
order.  But the film appears bright in transmitted light.
𝑫 20. Why do thick films not show interference effects?
 Fringe width is given by ; 𝜷 = . Since D and d are constants, 𝜷 ∝ 
𝒅  When the film is thick (𝑡 ≅ 20), the path difference 𝟐𝝁𝒕 𝐜𝐨𝐬 𝒓 will be so large
 As, 𝑮 < 𝒀 < 𝑶 , therefore 𝜷𝑮 < 𝜷𝒀 < 𝜷𝑶 that the conditions of both maxima and minima for different wavelengths of
white light will be essentially satisfied at the same value of thickness ‘t’
 Different colours will overlap each other at all places of the film producing
general illumination of the film. No separate colour or fringes are seen.
victory R. SARAVANAN. M.Sc., M.Phil., B.Ed PG ASST [PHYSICS], GBHSS, PARANGIPETTAI - 608 502
12 PHYSICS UNIT – 6 RAY OPTICS COMPLETE GUIDE AND MODEL QUESTION
21. The central fringe obtained with a Lloyd’s mirror set up for interference is 28. A single slit diffraction pattern is completely immersed in water without
found to be dark, whereas it is bright with a biprism. Why? changing any other parameter. How is the width of central maximum affected?
 The central fringe obtained with Lloyd’s mirror is dark because the reflection of 𝟏 
 Wavelength of light in water ( = ) decreases, so width of central maximum
light from the denser mirror surface introduces a phase difference of 𝝅 and 𝝁
also decreases.
equivalent path difference of ⁄2 which is the condition for minimum intensity.
29. Coloured spectrum is seen when we look through a muslin cloth. Why?
 In case of biprism, the waves from the coherent sources reach the central point  Muslin cloth is made of very fine threads and as such fine slits are formed.
in the same phase.So the central fringe is bright due to constructive interference  White light passing through these slits gets diffracted giving rise to coloured
22. State the essential condition for diffraction of light to occur?
spectrum. The central maximum is white, while the secondary maxima are
 Diffraction of light occurs, when size of the obstacle or the aperture should be of coloured. This is because the position of secondary maxima depend on the
the order of the wavelength of light used. wavelength of light.
23. In diffraction at single slit, why does the intensity of the secondary maximum 30. How does the resolving power of a telescope change when the aperture of the
becomes less as compared to the central maximum? objective is increased?
 The central maximum is due to the constructive interference of wavelets from 𝒂
 The resolving power of telescope is ; 𝑹𝑻 =
all parts of the slit. 𝟏.𝟐𝟐  𝐟
 With the increase in the value of order ‘n’, the wavelets from lesser and lesser 
When the aperture (a) of the objective is increased, the resolving power of the
partsof the slit produce constructive interference to form secondary maximum. telescope also increases.
Hence the intensity of secondary maximum decreases with the increase in the 31. Why oil immersion objective is used in microscope?
𝟐 𝒏 𝒔𝒊𝒏 𝜷 𝟐 (𝑵𝑨)
value of ‘n’  Resolving power of microscope is ; 𝑹𝑴 = =
𝟏.𝟐𝟐  𝟏.𝟐𝟐 
24. Yellow light is used in a single slit diffraction experiment with slit width of
 Since such a microscope uses oil of high refractive index (n) between the object
0.6 mm. If yellow light is replaced by X -rays, how will the diffraction pattern
and the objective so it has a high resolving power.
be affected?
32. Which special characteristic of light is demonstrated only by the phenomenon
 The wavelength of X -rays are of the order of nanometer and it is much smaller
of polarization?
than that of yellow light. To diffract X -rays, the size of the obstacle or aperture
 The phenomenon of polarization demonstrates that light waves has transverse
must be the order of nanometer.
wave nature.
 So diffraction pattern is lost when the yellow light is replaced by X -rays.
33. Light waves can be polarized while sound waves cannot. Why?
25. What is the basic difference between interference and diffraction of light?
 Only transverse waves can be polarized.
 Interference is the result of superposition of light waves from two different
 Light waves are transverse in nature, so they can be polarized.
coherent sources, whereas diffraction is due to interference of secondary
 But sound waves have longitudinal in nature, so they cannot be polarized.
wavelets starting from different part of the same wavefront.
26. A small circular disc is placed in the path of light from a distant source. Will 34. Will ultrasonic waves show any polarization? Give the reason for your answer.
the centre of shadow be bright or dark?  No. ultrasonic waves are longitudinal in nature.
 Waves from distance source are diffracted by the edge of the disc. These  Only transverse waves can be polarized and so ultrasonic waves cannot be
diffracted waves interfere constructively at the centre of the shadow and polarized.
produce a bright fringe. 35. Why longitudinal waves cannot be polarized?
27. Diffraction is common in sound but not common in light waves. Why?  In polarization, vibrations perpendicular to the direction of propagation are
 Diffraction effect is more pronounced, if the size of the obstacle or aperture is of restricted to just one direction. This is possible in transverse waves which have
the order of the wavelength of the waves. such vibrations.
 Sound waves have larger wavelength and is more or less comparable to the size  In longitudinal waves, vibrations occur along the direction of propagation. So
of the objects around us and hence diffraction of sound waves are common. their polarization is not possible.
 But the wavelength of light waves is of the order of 𝟏𝟎−𝟕 𝒎 and it is much 36. Which plane is defined as the plane of polarization in a plane polarized
smaller than the size of the objects around us. So diffraction of light is not easily electromagnetic wave?
seen.  The plane containing the direction of propagation of light and perpendicular to
the plane of vibration is called plane of polarization.
 This is the reason that we are able to hear a person standing behind a wall but
not see him though both sound and light are waves.  It contains no vibrations.

victory R. SARAVANAN. M.Sc., M.Phil., B.Ed PG ASST [PHYSICS], GBHSS, PARANGIPETTAI - 608 502
12 PHYSICS UNIT – 6 RAY OPTICS COMPLETE GUIDE AND MODEL QUESTION
37. Why does the electric field vector of electro magnetic wave determine the 43. A region is illuminated by two sources of light. The intensity ‘I’ at each point is
state of polarization and not its magnetic field vector? found to be equal to 𝑰𝟏 + 𝑰𝟐 where 𝑰𝟏 is the intensity of light at the point when
 An electromagnetic wave exerts a much larger electric force on a slowly moving source 2 is absent and 𝑰𝟐 is the intensity of light at the same point when
charged particle than the magnetic force. source 1 is absent. Are the two sources are coherent or incoherent? Explain.
 Optical pheneomena can be explained by considering the interaction between  When two light waves of intensities 𝑰𝟏 and 𝑰𝟐 , having phase difference 𝜙 meet
the electric field vector of light and the matter through which it passes. So one at a point, the resultant intensity is ; 𝑰 = 𝑰𝟏 + 𝑰𝟐 + 𝟐 √𝑰𝟏 𝑰𝟐 𝒄𝒐𝒔 𝜙
can specify the state of polarization by electric field vector only.  Given intensity is ; 𝑰 = 𝑰𝟏 + 𝑰𝟐
38. Does the value of polarizing angle of incidence depend on the colour of light?  Hence the interference term ; 2 √𝐼1 𝐼2 𝑐𝑜𝑠 𝜙 = 0 (or) 𝑐𝑜𝑠 𝜙 = 0 (or) 𝜙 = 90°
 The refractive index (n) of a material depends on the colour or wavelength () of
 It implies that the two sources have a phase diffence which is not stable (i.e.)
light.
varies from 0 to 2𝜋. Such sources are called incoherent sources.
 From Brewster’s law ; 𝒏 = 𝐭𝐚𝐧 𝒊𝑷 , As the polarizing angle (𝒊𝑷 ) depends on
44. What will be the effect on the interference fringes in Young’s double slit
refractive index (n), so it also depends on wavelength of light.
experiment when, (a) the monochromatic light is replaced by another
39. At what angle of incidence should a light beam strike a glass slab of refractive
monochromatic light of shorter wavelength? (b) the monochromatic light is
index √𝟑, such that the reflected and the refracted rays are perpendicular to replaced by a source of white light. Justify your answer in each case.
each other? 𝐷
 When the reflected and refracted rays are perpendicular to each other, then the a) Fringe width; 𝛽 = , Since D and d are constants, 𝜷 ∝  . Hence with
𝑑
corresponding angle of incidence is called polarizing angle(𝒊𝑷 ). monochromatic source of shorter wavelength (), fringe width (𝜷) decreases.
 According to Brewster’s law, at polarizing angle, ; 𝐭𝐚𝐧 𝒊𝑷 = 𝒏 (or) 𝐭𝐚𝐧 𝒊𝑷 = √𝟑 b) At the centre of the screen, the path difference is zero for all the wavelengths. So
(or) 𝒊𝑷 = 𝐭𝐚𝐧−𝟏 √𝟑 (or) 𝒊𝑷 = 𝟔𝟎° the central fringe is white. As violet colour has lowest wavelength, the closest
40. Sun glasses are made of polaroids and not of coloured glasses. Why? fringe or either side of central fringe is violet, while the farthest fringe is red.
 Polaroids absorb only that part of light which produces dazzling or glaring After a few fringes, the fringe pattern becomes indistinct due to large
effect in the eye. overlapping.
45. One of the two slits in Young’s double slit experiment is so painted that it
 But coloured glasses absorb more light incident on them and so the image
transmits half the intensity of the other, what is the effect on interference
appears dim.
fringes.
41. Compare Maxwell’s electromagnetic theory with Huygen’s theory of light.
 Let 𝑰𝟎 be the intensity of each slit. When the slit is not painted,
 In both theories, light is regarded as a wave motion.
 In the electromagnetic theory, light is regarded as electromagnetic waves which 𝑰𝒎𝒂𝒙 = 𝑰𝒐 + 𝑰𝒐 + 𝟐 √𝑰𝒐 𝑰𝒐 𝒄𝒐𝒔 𝟎° = 𝟒 𝑰𝒐
are transverse in nature and do not require any material medium for their 𝑰𝒎𝒊𝒏 = 𝑰𝒐 + 𝑰𝒐 + 𝟐 √𝑰𝒐 𝑰𝒐 𝒄𝒐𝒔 𝟏𝟖𝟎° = 𝟎
propagation.  When one of the slits is painted, it transmits half of the original intensity, then
 In Huygen’s theory, light waves are mechanical wave which are considered as 𝑰𝒐 𝑰 𝑰𝒐
longitudinal in nature and require a material medium for their propagation. 𝑰𝒎𝒂𝒙 = 𝑰𝒐 + + 𝟐 √𝑰𝒐 [ 𝒐 ] 𝒄𝒐𝒔 𝟎° = 𝑰𝒐 + + 𝑰𝒐 √𝟐 = 𝟐. 𝟗𝟏𝟒 𝑰𝒐
𝟐 𝟐 𝟐
That is why Huygens assumed the existence of an all pervading hypothetical 𝑰𝒐 𝑰 𝑰𝒐
medium called ether. 𝑰𝒎𝒊𝒏 = 𝑰𝒐 + + 𝟐 √𝑰𝒐 [ 𝒐 ] 𝒄𝒐𝒔 𝟏𝟖𝟎° = 𝑰𝒐 + − 𝑰𝒐 √𝟐 = 𝟎. 𝟎𝟖𝟔 𝑰𝒐
𝟐 𝟐 𝟐
42. If 𝝁𝒐 and 𝜺𝟎 are the permeability and permittivity of free space and 𝝁 and 𝜺 are  Hence on painting one of the two slits, the intensity of maxima decreases from
the corresponding quantities for a medium, then find the refractive index of 𝟒 𝑰𝒐 to 𝟐. 𝟗𝟏𝟒 𝑰𝒐 and that of minima increases from 0 to 𝟎. 𝟎𝟖𝟔 𝑰𝒐 (i.e.) the
the medium? contrast between the bright and dark fringes decreases.
𝟏
 Velocity of light in vacuum or free space ; 𝒄 = 46. Two sources of intensity 𝑰𝟏 and 𝑰𝟐 undergo interference in Young’s double slit
√𝝁𝒐 𝜺𝒐
𝟏
experiment, then find the ratio of intensities of maxima and minima.
 Velocity of light in medium ; 𝒗 =  When two light waves of amplitude 𝒂𝟏 and 𝒂𝟐 having phase difference of 𝝓
√ 𝜺
𝝁

 Then the refractive index of the medium ; 𝑛 =


𝑐
= √𝝁
𝝁𝜺 interfere, the resultant amplitude is ; 𝑨 = √𝒂𝟏𝟐 + 𝒂𝟐𝟐 + 𝟐 𝒂𝟏 𝒂𝟐 𝒄𝒐𝒔 𝝓
𝑣 𝒐 𝜺𝒐
 We know that ; 𝑰 ∝ 𝑨𝟐 (or) 𝑰 = 𝒌 𝑨𝟐 = 𝒌 [𝒂𝟏𝟐 + 𝒂𝟐𝟐 + 𝟐 𝒂𝟏 𝒂𝟐 𝒄𝒐𝒔 𝝓]
 When 𝝓 = 𝟎° ; 𝒄𝒐𝒔 𝝓 = 𝟏, then the intensity is maximum and it is given by,
𝑰𝒎𝒂𝒙 = 𝒌 [𝒂𝟏𝟐 + 𝒂𝟐𝟐 + 𝟐 𝒂𝟏 𝒂𝟐 (𝟏)] = 𝒌 (𝒂𝟏 + 𝒂𝟐 )𝟐
victory R. SARAVANAN. M.Sc., M.Phil., B.Ed PG ASST [PHYSICS], GBHSS, PARANGIPETTAI - 608 502
12 PHYSICS UNIT – 6 RAY OPTICS COMPLETE GUIDE AND MODEL QUESTION
 When 𝝓 = 𝟏𝟖𝟎° ; 𝒄𝒐𝒔 𝝓 = −𝟏, then the intensity is minimum and it is given by, 50. How would the linear width of central maximum change if (a) monochromatic
𝑰𝒎𝒊𝒏 = 𝒌 [𝒂𝟏𝟐 + 𝒂𝟐𝟐 + 𝟐 𝒂𝟏 𝒂𝟐 (−𝟏)] = 𝒌 (𝒂𝟏 − 𝒂𝟐 )𝟐 yellow light is replaced with red light and (b) if the distance between the slit
𝑰𝒎𝒂𝒙 𝒌 (𝒂𝟏 +𝒂𝟐 )𝟐 (𝒂𝟏 +𝒂𝟐 )𝟐 and the screen is increased?
 Therfore the ratio of maxima and minima is ; = = 𝑫 𝑫 𝟐𝑫
𝑰𝒎𝒊𝒏 𝒌 (𝒂𝟏 −𝒂𝟐 )𝟐 (𝒂𝟏 −𝒂𝟐 )𝟐  The linear width of central maximum is ; 𝒚𝒐 = − (− ) =
𝒂 𝒂 𝒂
47. Interference of sound can be observed with two independent tuning forks but a) We know that 𝒓𝒆𝒅 > 𝒚𝒆𝒍𝒍𝒐𝒘 . As 𝒚𝒐 ∝  , when monochromatic yellow
Interference of light cannot be observed with two independent bulbs. Why?
light is replaced with red light, the linear width of the central maximum
 When two tuning forks struck simultaneously they produce sound waves almost
increases.
in the same phase. Their phase difference if any, varies slowly with time.
b) As 𝒚𝒐 ∝ 𝑫 , If the distance (D) between the slit and screen is increased, the
Interference pattern also varies slowly with time. Such variations can be
linear width of the central maximum increases.
detected easily by the human ear. So interference pattern is easily observable.
51. In a single slit diffraction pattern, how does the angular width of central
 The phase difference between two independent light bulbs changes 108 times
maximum change, when (a) slit width is decreased, (b) distance between the
per second. The interference pattern also changes 108 times per second. Such
slit and the screen is increased and (c) light of smaller visible wavelength is
rapid variations cannot be detected by our eyes. So this interference pattern is used? Justify your answer in each case.
not observable. 𝑫 𝑫 𝟐𝑫
48. What change will occur in diffraction pattern if (a) light of smaller wavelength  The linear width of central maximum is ; 𝒚𝒐 = − (− ) =
𝒂 𝒂 𝒂
is used (b) slit is made narrower and (c) another slit is placed near and 𝒚𝒐 𝟐
 Hence the angular width (angular spread) of central maximum 𝜽𝒐 = =
parallel to the first slit? 𝑫 𝒂
𝟏
 Condition for minima in diffracton at single slit is ; 𝒂 𝐬𝐢𝐧 𝜽𝒏 = 𝒏  a) As 𝜽𝒐 ∝ , If slit width (a) decreases, then, the angular width of the central
𝒂
 If 𝜽𝒏 is small, then 𝐬𝐢𝐧 𝜽𝒏 ≈ 𝜽𝒏 Hence the directions of various minima in a maximum increases
𝒏 b) Since 𝒚𝒐 ∝ 𝑫, if the distance (D) between the slit and the screen increased,
diffraction pattern are given by ; 𝜽𝒏 =
𝒂 the linear width (𝒚𝒐 ) of central maximum also increases. So the angular
a) When light of smaller wavelength () is used, then 𝜃𝑛 also small. Hence the width does not change.
diffraction pattern becomes narrower. c) As 𝜽𝒐 ∝  , when light of small wavelength is used, angular width of the
b) When width of slit (a) decreases, 𝜃𝑛 increases and the diffraction pattern central maximum decreases.
spreads out. 52. State the condition for diffraction of light to occur. In the diffraction at a single
c) If another slit is placed near and parallel to the first slit, then it becomes slit experiment, how would the width and intensity of the central maximum
double slit and the inrerference pattern replaces the diffraction pattern. change, if (a) slit width is halved and (b) visible light of longer wavelength is
49. How is the width of central maxima affected if (a) width of slit is doubled, used?
(b) the wavelength of the light used is increased and (c) what happens to the  Diffraction of light is highly pronounced if the size of the obstacle (or) aperture
width of the central maxima if the whole apparatus is immersed in water and is comparable to the wavelength of light used
why? 𝑫 𝑫 𝟐𝑫
 The linear width of central maximum is ; 𝒚𝒐 = − (− ) =
 The diffraction pattern produced by a 𝒂 𝒂 𝒂
𝑎 𝟐𝑫 𝟐𝑫
single slit, has equal widths of maxima. a) If 𝑎 → , then width of central maximum ; 𝒚𝟎𝟏 = = 𝟐( ) = 𝟐 𝒚𝒐
2 𝒂/𝟐 𝒂
But, the intensity falls rapidly for (i.e.) if slit width is halved, then the width of central maximum is doubled.
higher order diffraction fringes. So its area becomes four times and hence intensity becomes one-fourth of
 Width of central maximum is the initial intensity.
𝑫 𝑫 𝟐𝑫
𝒚𝒐 =
𝒂
− (− ) =
𝒂 𝒂
b) As 𝒚𝒐 ∝  , if visible light of longer wavelength () is used, the width of
a) As 𝒚𝒐 ∝  , when wavelength () central maximum increases and hence intensity decreases.
of light used is increased, the width of central maximum increases. 53. What are the main changes in diffraction pattern of single slit will observe
𝟏
b) As 𝒚𝒐 ∝ , when width (a) of the slit increased, the width of central when the monochromatic source of light is replaced by a source of white light?
𝒂  When the monochromatic source is replaced by a source of white light, the
maximum decreases. diffraction pattern shows following changes,

c) As 𝑤 = 𝑎, the wavelength of light in water decreases and so the width of a) In each diffraction order, the diffracted image of the slit gets dispersed in to
𝜇
central maximum also decreases. component colours of white light. As 𝒚 ∝ , the red fringe is with higher
wavelength is wider than the violet fringe with smaller wavelength.
victory R. SARAVANAN. M.Sc., M.Phil., B.Ed PG ASST [PHYSICS], GBHSS, PARANGIPETTAI - 608 502
12 PHYSICS UNIT – 6 RAY OPTICS COMPLETE GUIDE AND MODEL QUESTION
b) In higher order spectra, the dispersion is more and it causes overlapping of 58. Distinguish between magnifying power and resolving power of a telescope.
different colours. Magnifying power of telescope Resolving power of telescope
54. Give the reason for the Astronomers prefer to use telescopes with large In normal adjustment it is defined as It is defined as the reciprocal of the
objective diameters to observe astronomical objects? the ratio of the angle subtended by the smallest angular separation between
 The objective of large aperture has a large light gathering capacity and it forms image at the eye and the angle two distant objects, whose images
bright images of even distant faint stars. It also increases the resolving power of subtended by the object seen directly, are just be seen in the microscope as
the telescope. when both lie at infinity separate.
55. Give the reason for the value of the Brewsters angle for a transparent medium (a) In normal adjustment, 𝒎=
𝒇𝒐 The resolving power of telescope is,
is different for lights of different colours. 𝒇𝒆 𝟏 𝒂 𝟏
(b) when the final image formed at the 𝑹𝑻 = = (or) 𝑹𝑻 = =
𝜽 𝟏.𝟐𝟐 𝒓𝒐
 According to Brewster’s law, 𝐭𝐚𝐧 𝒊𝑷 = 𝒏 𝒂
 As refractive index (n) of a transparent medium is different for light of different least distance of distinct vision,
𝒇 𝒇 𝟏.𝟐𝟐 𝒇
colours, so Brewster’s angle (i.e.) polarizing angle (𝒊𝑷 ) is different for light of 𝒎 = 𝒐 [𝟏 + 𝒆 ] 𝜽 →angular resolution
𝒇𝒆 𝑫
different colours. 𝐷 → least distance of distinct vision 𝒓𝒐 →spatial resolution
56. How wil the angular separation and visibility of the fringes in Young’s double (25cm) 𝒂 → diameter of the objective
slit experiment change when (a) screen is moved away from the plane of the 59. How does the resolving power of a compound microscope change, when
slits and (b) width of the source slit is increases? (a) refractive index of the medium between the object and the objective lens
𝑫
 Interference fringe width ; 𝜷 = increase and (b) wavelength of the radiation used is increases?
𝒅 𝟐 𝒏 𝐬𝐢𝐧 𝜽
𝜷   The resolving power of a compound microscope is, 𝑹𝑴 =
& Angular separation 𝜽 = = 𝟏.𝟐𝟐
𝑫 𝒅 a) Since 𝑹𝑴 ∝ 𝒏, when the refractive index (n) of the medium between the
a) Since 𝜷 ∝ 𝑫, when the screen is moved away from the slit, the distance ‘D’
object and the objective lens increases (oil immersed objective) the
increases and hence fringe width ‘𝜷’ also increases. But angular separation
resolving power increases.
‘’ remains unchanged. 𝟏
b) If the width of the slit increases, the interference pattern becomes less and b) Since 𝑹𝑴 ∝ , when the wavelength () of the radiation uses is increased,

less sharp and finally interference pattern disappears. But the angular the resolving power of microscope decreases.
width remains unchanged. 60. Explain with reason,how the resolving power of a compound microscope will
57. Distinguish between magnifying power and resolving power of a microscope. change, when (a) frequency of the incident light is increased, (b) focal length
Magnifying power of microscope Resolving power of microscope of the objective lens is increased and (c) aperture of the objective lens is
It is defined as the ratio of the angle It is defined as the reciprocal of the increased?
𝟐 (𝒏 𝐬𝐢𝐧 𝜽) 𝟐 (𝒏 𝐬𝐢𝐧 𝜽) 𝝂
subtended by the image at the eye and minimum distance between two point  The resolving power of a compound microscope is, 𝑹𝑴 = =
𝟏.𝟐𝟐 𝟏.𝟐𝟐 𝒄
the angle subtended by the object seen objects, which can just be seen a) Since 𝑹𝑴 ∝ 𝝂, When the frequency (𝝂) of incident light increases, the
directly, when both lie at the least through the microscope as separate. resolving power also increases.
distance of distinct vision. b) Since 𝑹𝑴 is independent of focal length (f), Resolving power does not
(a) Total magnification for near point The resolving power of microscope is, change with change in focal length of the objective lens.
focussing 𝟏 𝟐 𝒏 𝐬𝐢𝐧 𝜽 𝟐(𝑵𝑨)
𝑹𝑴 = = = c) Since 𝒂 = 𝒇 (𝟐 𝐬𝐢𝐧 𝜽), when the aperture (a) of the objective lens increases,
𝑳 𝑫 𝒅𝒎𝒊𝒏 𝟏.𝟐𝟐 𝟏.𝟐𝟐
𝒎 = 𝒎𝒐 𝒎𝒆 = [ ] [𝟏 + ] 𝑛 →refractive index of the medium the semi vertical angle (𝜽) increases and hence the resolving power of the
𝒇𝒐 𝒇𝒆
(b) Total magnification for mormal between object and objective lens microscope increases.
focussing  →half the angle of cone on light from 61. How does the resolving power of a telescope change, if (a) the size of the
𝑳 𝑫 the point object aperture of the objective lens is increased, (b) the focal length of the objective
𝒎 = 𝒎𝒐 𝒎𝒆 = [ ] [ ] lens is decreased?
𝒇𝒐 𝒇𝒆 𝑛 sin 𝜃 = 𝑁𝐴 → Numerical aperture 𝟏 𝒂
𝐷 → least distance of distinct vision  The resolving power of telescope is, 𝑹𝑻 = =
𝜽 𝟏.𝟐𝟐
(25cm)
𝐿 → distance between focal point eye a) Since 𝑹𝑻 ∝ 𝒂, When the aperture (a) of the objective lens increases, the
piece and objective(tube length) resolving power of telescope also increases
b) If the focal length (f) of objective decreased, the resolving power ( 𝑹𝑻 ) of
the telescope is not altered.
victory R. SARAVANAN. M.Sc., M.Phil., B.Ed PG ASST [PHYSICS], GBHSS, PARANGIPETTAI - 608 502
12 PHYSICS UNIT – 6 RAY OPTICS COMPLETE GUIDE AND MODEL QUESTION
62. When a sheet of transparent plastic is placed between two crossed polarizer,
no light is transmitted. When the sheet is stretched in one direction, some
light passes through the crossed polarizers. What is happening?
 A transparent plastic sheet is not a polaroid. So when two polarizers are placed
with crossed axes, no light is transmitted whether the plastic sheet is placed
between them or not.
 But when the sheet is stretched, the polymer molecules in it make it a polaroid
with its own polaroid axis, which make some angle with the axes of th two
polaroids. Now it becomes a case of three polaroids with the middle polaroid
having its axis between the axes of the two fixed polaroids. That is why some
light is transmitted in this case.
63. Show that when a ray of light is incident on the surface of a transparent
medium at the polarizing angle, the reflected and transmitted rays are
perpendicular to each other?
 Let 𝒊𝑷 be the angle of polarization, 𝒓𝑷 be the angle of refration and  be the angle
between relected and transmitted (refracted) rays.
sin 𝑖𝑃
 From Snell’s law ;
sin 𝑟𝑃
=𝑛
sin 𝑖𝑃
 From Brewster’s law ; tan 𝑖𝑃 = 𝑛 (or)
cos 𝑖𝑃
=𝑛
sin 𝑖𝑃 sin 𝑖
 Combining these two law, we have ;
sin 𝑟𝑃
= cos 𝑖𝑃 (or) sin 𝑟𝑃 = cos 𝑖𝑃
𝑃
(or) sin 𝑟𝑃 = sin(90° − 𝑖𝑃 ) (or) 𝑟𝑃 = 90° − 𝑖𝑃 (or) 𝑖𝑃 + 𝑟𝑃 = 90°
 But, 𝑖𝑃 + 𝜃 + 𝑟𝑃 = 180° (or) 𝜃 = 180° − (𝑖𝑃 + 𝑟𝑃 ) = 180° − 90° = 𝟗𝟎°
 Hence the reflected and transmitted rays are perpendicular to each other.
64. The critical angle between a given transparent medium and air is denoted by
𝒊𝑪 A ray of light in air medium at an angle of incidence equal to the polarizing
angle 𝒊𝑷 Deduce a relation for the angle of refraction (𝒓𝑷 ) in terms of 𝒊𝑪
 When a ray of light incident on a transparent medium at polarizing angle, then
𝑖𝑃 + 𝑟𝑃 = 90° (or) 𝑖𝑃 = 90° − 𝑟𝑃
 From Brewster’s law ; 𝑛 = tan 𝑖𝑃 (or) 𝑛 = tan(90° − 𝑟𝑃 ) (or) 𝑛 = cot 𝑟𝑃
1
 As 𝒊𝑪 is the critical angle, then 𝑛 =
sin 𝒊𝑪
1
 Hence, cot 𝑟𝑃 = (or) tan 𝑟𝑃 = sin 𝒊𝑪 (or) 𝒓𝑷 = 𝐭𝐚𝐧−𝟏 [𝐬𝐢𝐧 𝒊𝑪 ]
sin 𝒊𝑪
65. Give the relation between critical angle (𝒊𝑪 ) and polarizing angle (𝒊𝑷 ).
1
 From Brewster’s law ; 𝑛 = tan 𝑖𝑃 ; As 𝒊𝑪 is the critical angle, then 𝑛 =
sin 𝒊𝑪
𝟏
 Hence,
𝐬𝐢𝐧 𝒊𝑪
= 𝐭𝐚𝐧 𝒊𝑷 (or) 𝐬𝐢𝐧 𝒊𝑪 = 𝐜𝐨𝐭 𝒊𝑷 (or) 𝒊𝑪 = 𝐬𝐢𝐧−𝟏 (𝐜𝐨𝐭 𝒊𝑷 )
66. Why are the telescopes fitted with a large aperture objectives and the
microscopes fitted with small aperture objectives?
 Greater the aperture, greater is the number of rays entering it. Thus the image is
brighter in the lens of wide aperture and hence the telescopes are fitted with
larger aperture objectives.
 In microscopes, smaller aperture used because, light rays from near by tiny
objects spread over small aperture and the final image formed is very small.

victory R. SARAVANAN. M.Sc., M.Phil., B.Ed PG ASST [PHYSICS], GBHSS, PARANGIPETTAI - 608 502
12 PHYSICS UNIT – 6 RAY OPTICS COMPLETE GUIDE AND MODEL QUESTION
EXAM NO 6. First diffraction minimum due to a single slit of width 1.0×10 –5 cm is at 30o.
NAME : Then wavelength of light used is,
UNIT – 7 WAVE OPTICS (a) 400 Å
(b) 500 Å
Time - 2 : 15 hours Total - 50 marks (c) 600 Å
PART - I 10 X 1 = 10 (d) 700 Å
Note : (i) Answer all the questions 7. A ray of light strikes a glass plate at an angle 60o. If the reflected and refracted
(ii) Choose the best answer and write the option code and rays are perpendicular to each other, the refractive index of the glass is,
corresponding answer (a) √3
3
1. A plane glass is placed over a various coloured letters (violet, green, yellow, (b)
2
red) The letter which appears to be raised more is,
3
(a) red (c) √
2
(b) yellow
(c) green (d) 2
(d) violet 8. One of the of Young’s double slits is covered with
2. Two point white dots are 1 mm apart on a black paper. They are viewed by eye a glass plate as shown in figure. The position of
of pupil diameter 3 mm approximately. The maximum distance at which these central maximum will,
dots can be resolved by the eye is, [take wavelength of light, λ = 500 nm] (a) get shifted downwards
(a) 1 m (b) get shifted upwards
(b) 5 m (c) will remain the same
(c) 3 m (d) data insufficient to conclude
(d) 6m 9. Light transmitted by Nicol prism is,
3. In a Young’s double-slit experiment, the slit separation is doubled. To maintain (a) partially polarised
the same fringe spacing on the screen, the screen-to-slit distance D must be (b) unpolarised
changed to, (c) plane polarised
(a) 2D (d) elliptically polarised
𝐷 10. The transverse nature of light is shown in,
(b) (a) interference
2
(c) √2 D (b) diffraction
𝐷 (c) scattering
(d) (d) polarisation
√2
4. Two coherent monochromatic light beams of intensities I and 4I are PART - II 5 X 2 = 10
superposed. The maximum and minimum possible intensities in the resulting
beam are
Note : (i) Answer any 5of the following questions .
(ii) Question No. 17 is compulsory
(a) 5I and I
11. Define interference
(b) 5I and 3I
(c) 9I and I 12. What are called coherent sources
(d) 9I and 3I 13. What are the conditions for obtaining clear and broad interference bands?
5. When light is incident on a soap film of thickness 5×10 –5 cm, the wavelength of 14. What is diffraction?
light reflected maximum in the visible region is 5320 Å. Refractive index of the 15. Define angle of polarization.
film will be, 16. Distinguish between near point focusing and normal focusing.
(a) 1.22 17. The wavelength of a light is 450 nm. How much phase it will differ for a path of 3 mm?
(b) 1.33
(c) 1.51
(d) 1.83

victory R. SARAVANAN. M.Sc., M.Phil., B.Ed PG ASST [PHYSICS], GBHSS, PARANGIPETTAI - 608 502
12 PHYSICS UNIT – 6 RAY OPTICS COMPLETE GUIDE AND MODEL QUESTION
PART - III 5 X 3 = 15
Note : (i) Answer any5 of the following questions .
(ii) Question No. 24 is compulsory
18. Write a note on wave theory of light.
19. State Huygen’s principle.
20. Distinguish between interference and diffraction.
21. What is Fresnel’s distance? Obtain an expression for it.
22. State Brewster’s law
23. Write a note on pile of plates.
24. The ratio of maximum and minimum intensites in an interference pattern is 36 : 1
What is the ratio of the amplitudes of the two interfering waves? அறம் செய்ய விரும்பு – தருமத்தத செய்ய விருப்பம் ச ொள்
ஆறுவது சினம் – அடக் வவண்டியது வ ொபவம ஆகும்ஃ
PART - IV 3 X 5 = 15 இயல்வது ரவவல் – ச ொடுக் முடிந்த சபொருதை யொசிப்பவர்க்கு
Note : (i) Answer all the questions ச ொடு
25. Prove laws of refraction using Huygen’s principle
ஈவது விலக்வ ல் – பிறருக்கு ச ொடுப்பதத தடுக் க்கூடொது
உதடயது விைம்வபல் – உள்ை சபொருதை (அ) இர சியங் தை
(OR)
Obtain the equation for resultant intensity due to interference of light
பிறரிடம் செொல்லொவத
26. Obtain the equation for band width in Young’s double slit method
ஊக் மது த விவடல் – எப்வபொதும் முயற்சிதய த விடொவத
எண் எழுத்து இ வேல் – ணிதம், இலக் ணம் நூல் தை அலட்சியம்
(OR)
செய்யொவத
Obtain the equation for constructive and destructive interference for transmitted ஏற்பது இ ழ்ச்சி – இரந்து (யொசித்து) வொழ்வது இழிவொனது
and reflected waves in thin films
ஐயம் இட்டு உண் – வததவபடுவவொருக்கு உணவிட்டு பின் உண்
27. Discuss the diffraction at single slit and obtain the condition for nth minimum
ஒப்புரவு ஒழுகு – உல நதடமுதற அறிந்து அதன்படி நடந்து
(OR) ச ொள்
Obtain the equation for resolving power of microscope ஓதுவது ஒழிவயல் – நல்ல தூல் தை படிப்பதத விட்டுவிடொவத
ஓைவியம் வபவெல் – யொரிடமும் சபொறொதம ச ொண்டு வபெொவத
அஃ ஞ் சுருக்வ ல் – தொனியங் தை குதறத்து அைந்து விற் ொவத

ஆத்திச்சூடி - ஔதவயொர்

victory R. SARAVANAN. M.Sc., M.Phil., B.Ed PG ASST [PHYSICS], GBHSS, PARANGIPETTAI - 608 502
பசித்திரு (Be hungry) தனித்திரு (Be individual) விழித்திரு (Be conscious)

HIGHER SECONDARY SECOND YEAR-PHYSICS

NAME :
STANDARD : 12 SECTION :
SCHOOL :
EXAM NO :

victory R. SARAVANAN. M.Sc, M.Phil, B.Ed.,


PG ASST (PHYSICS)
GBHSS, PARANGIPETTAI - 608 502
12 PHYSICS UNIT – 8 DUAL NATURE OF RADIATION AND MATTER COMPLETE GUIDE AND MODEL QUESTION

PART – I 1 MARK MULTIPLE CHOICE QUESTIONS & ANSWERS 𝑽


wavelength 2λ, the stopping potential is . The threshold wavelength for the
1. The wavelength λe of an electron and λp of a photon of same energy E are 𝟒
related by metallic surface is
5
(a) 𝜆𝑝 ∝ 𝜆𝑒 (a) 4 λ (b) 5 λ (c) λ (d) 3 λ
2
(b) 𝜆𝑝 ∝ √𝜆𝑒 Solution :-
1 ℎ𝑐 ℎ𝑐 1 1 λ𝑜 − λ
(c) 𝜆𝑝 ∝  For stopping potential ‘V’ ; 𝑒𝑉 = − = ℎ𝑐 [ − ]=ℎ𝑐 [ ]
√𝜆𝑒 λ λ𝑜 λ λ𝑜 λ λ𝑜
𝑉 𝑉 ℎ𝑐 ℎ𝑐 1 1 λ𝑜 − 2λ
(d) 𝜆𝑝 ∝ 𝜆𝑒2  For stopping potential
4
; 𝑒 =
4 2λ

λ𝑜
= ℎ𝑐 [


λ𝑜
]=ℎ𝑐 [
2 λ λ𝑜
]
Solution :- λ −λ
ℎ𝑐[ 𝑜 ]
𝑒𝑉 λ λ𝑜 2 [λ𝑜 − λ]
ℎ ℎ2  Hence, = (or) 4 = (or) 4 λ𝑜 − 8 λ = 2 λ𝑜 − 2 λ
 For electron, 𝜆𝑒 = (or) 𝐸 = 𝑒𝑉/4 λ − 2λ
ℎ𝑐[ 𝑜 ] [λ𝑜 − 2 λ]
√2 𝑚𝑒 𝐸 2 𝑚𝑒 𝜆𝑒2 2 λ λ𝑜
ℎ𝑐 4 λ𝑜 − 2 λ𝑜 = 8 λ − 2 λ (or) 2 λ𝑜 = 6 λ (or) λ𝑜 = 3 λ
 For photon, 𝐸 = ℎ𝜈 =
𝜆𝑝 Answer (d) 𝟑𝛌
ℎ2 ℎ𝑐 2𝑚 𝑐 5. If a light of wavelength 330 nm is incident on a metal with work function
 Since the energy is same,
2 𝑚𝑒 𝜆𝑒2
= 𝜆𝑝
(or) 𝜆𝑝 = [ ℎ𝑒 ] 𝜆𝑒2
3.55 eV, the electrons are emitted. Then the wavelength of the wave associated
Answer (d) 𝝀𝒑 ∝ 𝝀𝒆𝟐 with the emitted electron is (Take h = 6.6 × 10–34 Js)
2. In an electron microscope, the electrons are accelerated by a voltage of 14 kV. (a) < 2.75 𝑋 10−9 𝑚 (b) ≥ 2.75 𝑋 10−9 𝑚
−12
If the voltage is changed to 224 kV, then the de Broglie wavelength associated (c) ≤ 2.75 𝑋 10 𝑚 (d) < 2.5 𝑋 10−10 𝑚
with the electrons would Solution :-
(a) increase by 2 times  By Einstein’s equation ; [note : ℎ 𝑐 = 1240 nm eV]
ℎ𝑐 1240 nmeV
(b) decrease by 2 times [𝐾𝐸] = − Φ𝑜 = − 3.55 𝑒𝑉 = 3.757 − 3.55 = 0.207 𝑒 𝑉
λ 330 𝑛𝑚
(c) decrease by 4 times  Hence wavelength of electron,
(d) increase by 4 times ℎ 6.6 𝑋 10−34
λ𝑚𝑖𝑛 = = = 2.75 𝑋 10−9 𝑚
Solution :- √2𝑚[𝐾𝐸] √2 𝑋 9.1 𝑋 10−31 𝑋 0.207𝑋 1.6 𝑋 10−19
12.27 𝑜 12.27 𝑜 Answer (d) ≥ 𝟐. 𝟕𝟓 𝑿 𝟏𝟎−𝟗 𝒎
 The de Broglei wavelength of electron ; 𝜆1 = 𝐴 and 𝜆2 = 𝐴
√𝑉1 √𝑉2 6. A photoelectric surface is illuminated successively by monochromatic light of
𝜆1 𝑉2 224 𝑋 103 𝜆1 𝛌
 Hence the ratio,
𝜆2
= √ = √
𝑉 14 𝑋 103
= √16 = 4 (or) 𝜆2 = 4
wavelength 𝛌 and
𝟐
. If the maximum kinetic energy of the emitted
1
photoelectrons in the second case is 3 times that in the first case, the work
Answer (c) decrease by 4 times function of the material is
3. The wave associated with a moving particle of mass 3 X 10–6 g has the same ℎ𝑐 2ℎ𝑐 ℎ𝑐 ℎ𝑐
wavelength as an electron moving with a velocity 6 X 10 6 m s-1 .The velocity of (a) (b) (c) (d)
λ λ 3λ 2λ
the particle is Solution :-
ℎ𝑐
(a) 1.82 X 10-18 m s-1  For case (i) ; 𝐾1 = ℎ 𝜈1 − Φ𝑜 = − Φ𝑜
λ
(b) 9 X 10-2 m s-1 ℎ𝑐 2ℎ𝑐
 For case (ii) ; 𝐾2 = ℎ 𝜈2 − Φ𝑜 = − Φ𝑜 = − Φ𝑜
(c) 3 X 10-31 m s-1 λ
2
λ
(d) 1.82 X 10-15 m s-1  But given that, 𝐾2 = 3 𝐾1 . Hence,
Solution :- 2ℎ𝑐 ℎ𝑐 2ℎ𝑐 ℎ𝑐
− Φ𝑜 = 3 ( − Φ𝑜 ) (or) − Φ𝑜 = 3 − 3 Φ𝑜
 If wavelength is same, then their momentum also same. Hence, 𝑚𝑝 𝑣𝑝 = 𝑚𝑒 𝑣𝑒 λ λ λ λ
ℎ𝑐 ℎ𝑐 ℎ𝑐 ℎ𝑐
𝑚𝑒 𝑣𝑒 9.1 𝑋 10−31 𝑋 𝟔 𝐗 𝟏𝟎𝟔 3 Φ𝑜 − Φ𝑜 = 3 −2 (or) 2 Φ𝑜 = (or) Φ𝑜 =
∴ 𝑣𝑝 = 𝑚𝑝
= 𝟑 𝐗 𝟏𝟎−𝟔 𝑿 𝟏𝟎−𝟑
= 1.82 𝑋 10−15 𝑚 𝑠 −1 λ λ λ 2λ
𝒉𝒄
Answer (d)
Answer (d) 𝟏. 𝟖𝟐 𝑿 𝟏𝟎−𝟏𝟓 𝒎 𝒔−𝟏 𝟐𝛌
4. When a metallic surface is illuminated with radiation of wavelength λ, the
stopping potential is V. If the same surface is illuminated with radiation of

victory R. SARAVANAN. M.Sc., M.Phil., B.Ed PG ASST [PHYSICS], GBHSS, PARANGIPETTAI - 608 502
12 PHYSICS UNIT – 8 DUAL NATURE OF RADIATION AND MATTER COMPLETE GUIDE AND MODEL QUESTION
7. In photoelectric emission, a radiation whose frequency is 4 times threshold 10. If the mean wavelength of light from sun is taken as 550 nm and its mean
frequency of a certain metal is incident on the metal. Then the maximum power as 3.8 × 1026 W, then the average number of photons received by the
possible velocity of the emitted electron will be human eye per second from sunlight is of the order of
ℎ𝜈𝑜 6 ℎ𝜈𝑜 (a) 1045
(a) √ (b) √ (b) 1042
𝑚 𝑚
(c) 1054
ℎ 𝜈𝑜 ℎ 𝜈𝑜
(c) 2√ (d) √ (d) 1051
𝑚 2𝑚
Solution :-
Solution :- 𝑛ℎ𝑐
1 2  Total power (i.e.) energy per second ; 𝐸𝑛 = 𝑛 ℎ 𝜈 =
 According to Einstein’ s photo electric equation, ℎ 𝜈 = ℎ 𝜈𝑜 + 𝑚𝑣𝑚𝑎𝑥 𝜆
2 𝐸𝑛 𝜆 3.8 𝑋 1026 𝑋 550 𝑋 10−9 2090 𝑋 1043
2 1  Hence, 𝑛 = = = = 1.055𝑋 1045
 But given that, 𝜈 = 4 𝜈𝑜 . Hence, ℎ (4 𝜈𝑜 ) = ℎ 𝜈𝑜 + 𝑚𝑣𝑚𝑎𝑥 ℎ𝑐 6.6 𝑋 10−34 𝑋 3 𝑋 108 19.8
2
1 6 ℎ 𝜈𝑜 6 ℎ 𝜈𝑜 Answer (a) 𝟏. 𝟎𝟓𝟓𝑿 𝟏𝟎𝟒𝟓
2 2
(or) 𝑚𝑣𝑚𝑎𝑥 = 3 ℎ 𝜈𝑜 (or) 𝑣𝑚𝑎𝑥 = (or) 𝑣𝑚𝑎𝑥 = √ 11. The threshold wavelength for a metal surface whose photoelectric work
2 𝑚 𝑚
function is 3.313 eV is
𝟔 𝒉 𝝂𝒐
Answer (b) √ (a) 4125 Å
𝒎
(b) 3750 Å
8. Two radiations with photon energies 0.9 eV and 3.3 eV respectively are falling (c) 6000 Å
on a metallic surface successively. If the work function of the metal is 0.6 eV, (d) 20625.Å
then the ratio of maximum speeds of emitted electrons in the two cases will be Solution :-
(a) 1:4 (b) 1:3  By definition, work function ; Φ𝑜 = ℎ 𝜈𝑜 =
ℎ𝑐
𝜆𝑜 =
ℎ𝑐
(c) 1:1 (d) 1:9 𝜆𝑜 Φ𝑜
Solution :-  Thus threshold wavelength ;
1 ℎ𝑐 1240 𝑛𝑚 𝑒𝑉
 For photon -1 ; 𝑚𝑣12 = ℎ 𝜈 − ℎ 𝜈𝑜 = 0.9 𝑒𝑉 − 0.6 𝑒𝑉 = 0.3 𝑒𝑉 𝜆𝑜 = Φ𝑜
= 3.313 𝑒𝑉
= 374.8 𝑛𝑚 ≈ 3750 Å
2
1
 For photon -2 ; 𝑚𝑣22 = ℎ 𝜈 − ℎ 𝜈𝑜 = 3.3 𝑒𝑉 − 0.6 𝑒𝑉 = 2.7 𝑒𝑉 Answer (b) 3750 Å
2
𝑣21 0.3 𝑒𝑉 1 𝑣1 1 12. A light of wavelength 500 nm is incident on a sensitive metal plate of
 Hence the ratio ; = = (or) = photoelectric work function 1.235 eV. The kinetic energy of the photo
𝑣22 2.7 𝑒𝑉 9 𝑣2 3
electrons emitted is (Take h = 6.6 × 10–34 Js)
Answer (b) 1:3
(a) 0.58 eV
9. A light source of wavelength 520 nm emits 1.04 × 1015 photons per second (b) 2.48 eV
while the second source of 460 nm produces 1.38 × 10 15 photons per second. (c) 1.24 eV
Then the ratio of power of second source to that of first source is (d) 1.16 eV
(a) 1.00 (b) 1.02 Solution :-
(c) 1.5 (d) 0.98  By Einstein’s equation ; [note : ℎ 𝑐 = 1240 nm eV]
Solution :- ℎ𝑐
𝑛 ℎ𝑐 [𝐾𝐸]𝑚𝑎𝑥 = ℎ 𝜈 − Φ𝑜 = − Φ𝑜
 For source - 1 ; 𝐸𝑛1 = 𝑛1 ℎ 𝜈1 = 1𝜆 1249 𝑛𝑚 𝑒𝑉
λ
1
𝑛 ℎ𝑐
[𝐾𝐸]𝑚𝑎𝑥 = − 1.235𝑒𝑉 = 2.48 − 1.35 = 1.245 𝑒𝑉
500 nm
 For source - 2 ; 𝐸𝑛2 = 𝑛2 ℎ 𝜈2 = 2𝜆
2 Answer (c) 1.24 eV
𝐸𝑛2 𝑛2 𝜆1 1.38 𝑋 1015 𝑋 520 𝑋 10−9
 Hence the ratio ; = = = 1.5
𝐸𝑛1 𝑛2 𝜆2 1.04 𝑋 1015 𝑋 460 𝑋 10−9
Answer (c) 1.5

victory R. SARAVANAN. M.Sc., M.Phil., B.Ed PG ASST [PHYSICS], GBHSS, PARANGIPETTAI - 608 502
12 PHYSICS UNIT – 8 DUAL NATURE OF RADIATION AND MATTER COMPLETE GUIDE AND MODEL QUESTION
13. Photons of wavelength λ are incident on a metal. The most energetic electrons
ejected from the metal are bent into a circular arc of radius R by a
PART – II & III
2 AND 3 MARK SHORT ANSWER QUESTIONS &
perpendicular magnetic field having magnitude B. The work function of the ANSWERS
1. Why do metals have a large number of free electrons?
metal is  In metals, the electrons in the outer most shells are loosely bound to the
ℎ𝑐 𝑒 2 𝐵2 𝑅2 ℎ𝑐 𝑒𝐵𝑅 2 nucleus.
(a) − 𝑚𝑒 + 2 𝑚 (b) + 2 𝑚𝑒 [2 𝑚 ]
𝜆 𝑒 𝜆 𝑒  Even at room temperature, due to thermal agitation the loosely bounded
ℎ𝑐 2 𝑒 2 𝐵2 𝑅2 ℎ𝑐 𝑒𝐵𝑅 2 electrons are detached from their orbit and free to move inside the metal in a
(c) − 𝑚𝑒 𝑐 − 2 𝑚 (d) − 2 𝑚𝑒 [2 𝑚 ]
𝜆 𝑒 𝜆 𝑒 random manner. This is the reason for large number of free electrons in the
Solution :- metal.
1 2. Define surface barrier.
 According Einstien’s equation ; ℎ 𝜈 = Φ𝑜 + 𝑚𝑒 𝑣 2
2
1 ℎ𝑐 1  The potential barrier which prevents free electrons from leaving the metallic
(or) Φ𝑜 = ℎ 𝜈 −
2
𝑚𝑒 𝑣2𝑚𝑎𝑥 (or) Φ𝑜 =
𝜆
− 2 𝑚𝑒 𝑣2𝑚𝑎𝑥 surface is called surface barrier. It is created by the positive nuclei of the metal
𝑚𝑒 𝑣2 𝐵𝑒𝑅 3. Define electron emission.
 Magnetic Lorentz force is given by ; 𝐵 𝑒 𝑣 = (or) 𝑣 =
𝑅 𝑚𝑒  The liberation of electrons from any surface of a substance is called electron
 Thus from two equations, emission.
ℎ𝑐 1 𝐵𝑒𝑅 2 ℎ𝑐 𝐵𝑒𝑅 2  The material with small work function is more effective in electron emission.
Φ𝑜 =
𝜆
− 2 𝑚𝑒 ( 𝑚 ) = 𝜆
− 2 𝑚𝑒 ( 2 𝑚 )
𝑒 𝑒 4. Define work function of a metal. Give its unit.
𝒉𝒄 𝑩𝒆𝑹 𝟐  The minimum energy needed for an electron to escape from the metal surface is
Answer (c) − 𝟐 𝒎𝒆 ( 𝟐 𝒎 )
𝝀 𝒆 called work function of that metal. It is denoted by 𝜙𝑂
14. The work functions for metals A, B and C are 1.92 eV, 2.0 eV and 5.0 eV  Its unit is electron volt (eV).
respectively. The metal/metals which will emit photoelectrons for a radiation 5. Define electron volt (eV)
of wavelength 4100Å is/are  One electron volt is defined as the kinetic energy gained by a electron when
(a) A only accelerated by a potential difference of 1 volt. 𝟏 𝒆𝑽 = 𝟏. 𝟔𝟎𝟐 𝑿 𝟏𝟎−𝟏𝟗 𝑱
(b) both A and B 6. What is photo electric effect?
(c) all these metals  The ejection of electrons from a metal plate when illuminated by light or any
(d) none other electromagnetic radiation of suitable wavelength or frequency is called
Solution :- photo electric effect.
ℎ𝑐 12400 Å 𝐞𝐕
 Energy of photon ; 𝐸 = ℎ𝜈 = = ≅ 3 𝑒𝑉  The ejected electrons are called as photo electrons and the corresponding
𝜆 4100 Å
current is called photo electric current
 Thus both the metals A and B emits photo electrons, but not C
7. What are called photo sensitive materials?
Answer (b) both A and B  The materials which eject photoelectrons upon irradiation of electromagnetic
15. Emission of electrons by the absorption of heat energy is called wave of suitable wavelength are called photosensitive materials.
……………emission. (e.g.) Metals like cadmium, zinc, magnesium etc and Alkali metals like lithium,
(a) photoelectric sodium, caesium
(b) field 8. How does photo electric current vary with the intensity of the incident light?
(c) thermionic Variation of photo current with intensity :
(d) secondary  Keeping the frequency ( 𝜈 ) and acceleration
Solution :- potential (V) as constant, the intensity of incident
 When a metal is heated to a high temperature, the free electrons on the surface light is varied and the corresponding photo eletric
of the metal get sufficient energy in the form of thermal energy so that they are current is measured
emitted from the metallic surface .This type of emission is known as  A graph is drawn between intensity along X-axis
thermionic emission. and the photo current along Y-axis.
Answer (c) thermionic  From the graph, the photo current (i.e) the number
of electrons emitted per second is directly
proportional to the intensity of incident light.
victory R. SARAVANAN. M.Sc., M.Phil., B.Ed PG ASST [PHYSICS], GBHSS, PARANGIPETTAI - 608 502
12 PHYSICS UNIT – 8 DUAL NATURE OF RADIATION AND MATTER COMPLETE GUIDE AND MODEL QUESTION
9. Define stopping potential.  Each atomic oscillator which vibrates with its characteristic frequency emits or
 The negative or retarding potential given to collecting electrode which is just absorbs electromagnetic radiation of the same frequency.
sufficient to stop the most energetic photoelectrons emitted and make the (i) If an oscillator vibrates with frequency v, its energy can have only certain
photo current zero is called stopping potential or cut - off potential. discrete values,
10. Define threshold frequency. 𝑬𝒏 = 𝒏 𝒉 𝝂 [𝑛 = 1,2,3, … . ]
 For a giver surface, the emission of photo electrons takes place only if the where h  Planck’s constant.
frequency of incident light is greater than a certain minimum frequency called (ii) The oscillators emit or absorb energy in small packets or quanta and the
threshold frequency. energy of each quantum is E = h ν
11. State the laws of photo electric effect.  This implies that the energy of the oscillator is quantized and not continuous
Laws of photo electric effect : This is called quantization of energy.
 For a given frequency of incident light, the number of photoelectrons emitted is 14. Explain Eienstein’s explanation for the particle nature (quanta ) of light
directly proportional to the intensity of the incident light. The saturation Particle nature of light - Eienstein’s explanation :
current is also directly proportional to the intensity of incident light.  According to Einstein, the energy in light is not spread out over wavefronts but
 Maximum kinetic energy of the photo electrons is independent of intensity of is concentrated in small packets or energy quanta. Therefore, light of frequency
the incident light. v from any source can be considered as a stream of quanta
 Maximum kinetic energy of the photo electrons from a given metal is directly  The energy of each light quantum ; E = h ν
proportional to the frequency of incident light. 𝒉𝝂
 The linear momentum of quanta is ; 𝒑 =
 For a given surface, the emission of photoelectrons takes place only if the 𝒄
frequency of incident light is greater than a certain minimum frequency called  The individual light quantum of definite energy and momentum can be
the threshold frequency. associated with a particle. The light quantum can behave as a particle and this is
called photon.
 There is no time lag between incidence of light and ejection of photoelectrons.
15. Define intensity of light according to the quantum concept.
12. Explain why photoelectric effect cannot be explained on the basis of wave
nature of light  According to quantum concept, the intensity of light of given wavelength is
Failures of classical wave theory : defined as the number of energy quanta or photons incident per unit area per
unit time with photon having same energy.
 According to wave theory, light of greater intensity should impart greater
kinetic energy to the liberated electrons.  The unit is 𝑾 𝒎−𝟐
But the experiments show that maximum kinetic energy of the photoelectrons 16. What is the nature of light?
does not depend on the intensity of the incident light.  The light possesses dual nature that of both wave and particle.
 According to wave theory, if a sufficiently intense beam of light is incident on  Light behaves as a wave during its propagation and behaves as a particle during
the surface, electrons will be liberated from the surface of the target, however its interaction with matter.
low the frequency of the radiation is. 17. What is photo electric cell? Give its type.
But photoelectric emission is not possible below a certain minimum frequency  The device which converts light energy into electrical energy is called photo
called threshold frequency. electric cell or simply photo cell.
 Since the energy of light is spread across the wavefront, each electron needs  It works on the principle of photo electric cell
considerable amount of time (a few hours) to get energy sufficient to overcome  Photo cells are classified in to three types.
the work function and to get liberated from the surface. (1) Photo emissive cell
But experiments show that photoelectric emission is almost instantaneous (2) Photo voltaic cell
process (3) Photo conductive cell
13. Explain the concept of quantization of energy. 18. What is De Broglie hypothesis?
Quantization of energy :  Due to the symmetry in nature, de Broglie suggested that if radiation like light
 Max Planck proposed quantum concept in 1900 in order to explain the block can act as particles at times, then matter particles like electrons should also act
body radiations. as waves at times.
 According to Planck, matter is composed of a large number of oscillating  According to de Broglie hypothesis, all matter particles like electrons, protons,
particles (atoms) which vibrate with different frequencies. neutrons in motion are associated with waves. These waves are called de
Broglie waves or matter waves.

victory R. SARAVANAN. M.Sc., M.Phil., B.Ed PG ASST [PHYSICS], GBHSS, PARANGIPETTAI - 608 502
12 PHYSICS UNIT – 8 DUAL NATURE OF RADIATION AND MATTER COMPLETE GUIDE AND MODEL QUESTION
19. What is called matter waves or de Broglie waves? 25. Name an experiment which shows wave nature of the electron. Which
 The waves assoiated with matter particles like electrons in motion is called phenomenon was observed in this experiment using an electron beam?
matter waves or de Broglei waves.  The wave nature of electron (i.e) de Broglie hypothesis of matter waves was
20. Derive the expression of de Broglie wavelength. experimentally confirmed by Davisson and Germer experiment.
De Broglei wavelength :  Diffraction is the important property of waves. So in this experiment, diffraction
 The momentum of photon of frequency ′𝜈′ is, of electron beam was observed when they fall on crystalline solids.
𝐸 ℎ𝜈 ℎ 26. An electron and an alpha particle have same kinetic energy. How are the
𝑝= = = [𝑐 = 𝜆𝜈]
𝑐 𝑐 𝜆 deBroglie wavelengths associated with them related?

 The wavelength of a photon is, 𝜆 =  De Broglei wavelength of electron beam,
𝑝

 According to de Broglie, this equation is applicable to matter particle also. 𝜆𝑒 =
√2 𝑚𝑒 𝐾
 Let ‘m’ be the mass and ‘𝒗’ be the velocity of the particle, then the wavelength
𝒉 𝒉  De Broglei wavelength of alpha particle,
𝝀= = ℎ
𝒎𝒗 𝒑 𝜆𝛼 =
√2 𝑚𝛼 𝐾
 This wavelength of the matter waves is known as de Broglie wavelength. 𝝀𝒆 𝒎
21. Write the expression for the de Broglie wavelength associated with a charged ∴ 𝝀𝜶
= √ 𝒎𝜶
𝒆
particle of charge ‘q’ and mass ‘m’, when it is accelerated through a potential
V. 27. What are called X - rays? Why are they so called?
 De Broglie wavelength in terms of potential ‘V’ ,  X - rays are invisible, electromagnetic waves of very short wavelength ranging
𝒉 𝒉 from 0.1 A to 100 A
𝝀= =  When a fast moving electrons strike a metal target of high atomic weight, X -
𝒎 𝒗 √𝟐 𝒎 𝒒 𝑽
rays are produced.
22. Why we do not see the wave properties of a baseball?
𝒉
 At the time of discovery, the orgin of this highly penetrating rays were not
 The de Broglie wavelength of matter is ; 𝝀 = known. So they were called as X - rays.
𝒎𝒗
 Thus the de Broglie wavelength is inversely proportional to the mass 28. List the properties of X - rays.
 Since the mass of base ball is too large as compared with the electron, the de Properties of X - rays :
Broglie wavelength of base ball is negligibly small  They travel in straight line with the velocity of light
 So we do not see the wave property of the baseball  They are not deflected both by electric and magnetic field
23. A proton and an electron have same kinetic energy. Which one has greater de  X - ray photons are highly energetic
Broglie wavelength. Justify.  They pass through materials which are opaque to visible light.
𝒉 29. What factor does the quality and intensity of X - rays were depends?
 De Broglei wavelength of proton ; 𝝀𝒑 =
√𝟐 𝒎𝒑 𝑲
𝒉
 The quality of X - rays is measured in terms of its penetrating power which
 De Broglei wavelength of electron ; 𝝀𝒆 = depends on the velocity of the striking electron and the atomic number of target
√𝟐 𝒎𝒆 𝑲
 Here the mass of the proton is greater than the mass of the electron (𝒎𝑷 > 𝒎𝒆 ) material.
 Hence the de Broglei wavelength of electron is greater than that of proton (𝝀𝒆 >  The intensity of X - rays is depends on the number of electrons striking the
𝝀𝑷 ) target
24. Write the relationship of de Broglie wavelength λ associated with a particle of 30. Write a note on the production of X - rays.
mass m in terms of its kinetic energy K. Production of X - rays :
 De Broglie wavelength in terms of potential ‘V’ ,  X - rays are produced in a Coolidge
𝒉 𝒉 tube which is a discharge tube.
𝝀 = 𝒎𝒗 =  Here a tungsten filament ‘F’ is
√𝟐 𝒎 𝒒 𝑽
 Since, q V = K (kinetic energy), we have heated by L.T, so that electrons are
𝒉 emitted from it by thermionic
𝝀= emission.
√𝟐 𝒎 𝑲
 These electrons are accelerated to
very high speeds by H.T
victory R. SARAVANAN. M.Sc., M.Phil., B.Ed PG ASST [PHYSICS], GBHSS, PARANGIPETTAI - 608 502
12 PHYSICS UNIT – 8 DUAL NATURE OF RADIATION AND MATTER COMPLETE GUIDE AND MODEL QUESTION
 The target material like tungsten is embedded in the face of solid copper anode.  During the downward transition, the
 When high speed electrons strike the target, they are decelarated suddenly and energy difference between the levels
lose their kinetic energy. is given out in the form of X - ray
 As a result, X -ray photons are produced. photon of definite wavelength.
 The face of target is inclined at particular angle, so that the X - rays can leave the  Such wavelengths, characteristic of
tube through its side. the target, consitute the line spectrum.
 Since most of the kinetic energy of electrons get converted in to heat, the target  It is evident that K - series of lines in
made of high melting point and a cooling system are usally employed. the X - ray spectrum arise due to the
31. What is X -ray spectra? Give its types. electronic transistions from L, M. N, O,
 The intensity of the X-rays when plotted against its wavelength gives a curve ……… shells to K - shell.
called X - ray spectrum.  Similarly L - series originates due to
 X - ray spectrum consists of two parts, namely electronic transition from M, N, O, …….
(1) Continuous X -ray spectrum shells to L - shell.
(2) Characteristic X - ray spectrum 34. Explain the applications of X -rays.
32. Write a note on continuous X - ray spectrum. (1) Medical diagnosis :
Continuous X - ray spectrum :  X - rays can pass through flesh more easily than through bones. Thus X -ray
 When a fast moving electron penetrates and approaches a target nucleus, it get radiograph containing a deep shadow of the bones and a light shadow of
accelerates or decelerates flesh. So X -rays radiographs ae used to detect fractures, foreign bodies,
 It may results in a change of path of the electron. diseased organs etc.,
 The radiation produced from such decelerating electron is called (2) Medical therapy :
Bremsstrhlung or braking radiation.  X - ray can kill diseased tissues. So they are employed to cure skin diseases,
 The energy of the emitted photon (radiation) is equal to the loss of kinetic malignant tumours etc.,
energy of the electron. (3) Industry :
 So the photons are emitted with all possible energies or frequencies.  They are used to check for flaws in welded joints, motor tyres, tennis balls
 The continuous X -ray spectrum is due to such radiations. and wood,
 When an electron gives up all its energy, then the photon is emitted with  At the custom post, they are used for detection of contraband goods.
highest frequency (𝜈0 ) or lowest wavelength (𝜆0 ) (4) Scientific Research :
 The intial kinetic energy of an electron = eV where, V  accelerating voltage  X - ray diffraction is important tool to study the structure of the crystalline
 Thus, materials (i.e) the arrangement of atoms and molecules in crystals.
𝑐 35. Mention the two features of x-ray spectra, not explained by classical
𝑒 𝑉 = ℎ 𝜈0 = ℎ
𝜆0 electromagnetic theory.
𝒉𝒄 𝟏𝟐𝟒𝟎𝟎  Though classical electromagnetic theory suggests the emission of radiations
(𝑜𝑟) 𝝀𝟎 = = 𝑨° from accelerating electrons, it could not explain two features exhibited by x-ray
𝒆𝑽 𝑽
 This relation is known as Duane - Hunt formula. spectra. These features are given below.
33. Write a note on characteristic X - ray spectra. (1) For a given accelerating voltage, the lower limit for the wavelength of
Characteristic X - ray spectra : continuous x-ray spectra is same for all targets. This minimum wavelength
 When the target is hit by fast electrons, the obtained X - ray spectra shows some is called cut-off wavelength.
narrow peaks at some well-defined wavelength. (2) The intensity of x-rays is significantly increased at certain well-defined
 The line spectrum showing these peaks is called characteristic X - ray spectrum. wavelengths
 This X -ray spectrum is due to the electronic transitions within the atoms. 36. What is Bremsstralung?
 For example, when an energetic electron penetrates in to the target atom and  When a fast moving electron penetrates and approaches a target nucleus, it get
removes the electrons in K - shell and create a vacancy in it. accelerates or decelerates. It may results in a change of path of the electron.
 So the electrons from outer orbits jump to fill up the vacancy in K - shell.  The radiation produced from such decelerating electron is called
Bremsstrhlung or braking radiation.

victory R. SARAVANAN. M.Sc., M.Phil., B.Ed PG ASST [PHYSICS], GBHSS, PARANGIPETTAI - 608 502
12 PHYSICS UNIT – 8 DUAL NATURE OF RADIATION AND MATTER COMPLETE GUIDE AND MODEL QUESTION
 When spark is formed, the charges will oscillate back and forth rapidly and the
PART – IV 5 MARK LONG ANSWER QUESTIONS & ANSWERS electromagnetic waves are produced.
1. What do you mean by electron emission? Explain briefly various methods of  To detect this electromagnetic waves, a copper wire bent in the shape of a circle
electron emission. is used as detector.
Electron emission : Hallwachs’s experiment :
 The liberation of electrons from any surface of a substance is called electron  In Hertz experiment, in order to
improve the visibility of the
emission.
spark, it is exposed to ultra
 The minimum energy needed to liberate the electorns from the metal surface is violet rays which makes the
called work function of that metal. spark as more vigorous.
 Depending upon the energy source, the electron emission is classified as four  Wilhelm Hallwachs confirmed
types which are explained below. that the strange behaviour of
(1) Thermionic emission : the spark is due to the photo
 When a metal is heated to a high temperature, the free electrons on the electric emission under the
surface get sufficient energy in the form of heat, so that they are emitted action of ultra violet light.
from the metallic surface. This type of emission is known as thermionic  In Hallwachs experiment, a clean circular plate of zinc is mounted in insulating
emission. stand and is attached to a gold leaf electroscope by a wire.
 The intensity of the thermionic emission depends on the metal used and its  When uncharged zinc plate is irradiated by ultraviolet light, it becomes
temperature. positively charged and the leaves are open as shown in figure (a)
(e.g.) electron microscopes, X-ray tubes  If negatively charged zinc plate is exposed to ultraviolet light, the leaves will
(2) Field emission : close as the charges leaked away quickly as shown in figure (b)
 When a very strong electric field is applied across the metal, this strong  If positively charged plate is exposed to uv-light, it becomes more positive and
field pulls the free electrons and helps to overcome the surface barrier of the leaves are open further as shown in figure (c)
the metal. This type of emission of electron is called field emission.  From these observations, it was concluded that negatively charged electrons
(e.g.) Field emission display were emitted from the zinc plate under the action of uv - light.
(3) Photo electric emission : Lenard experiment :
 When an electromagnetic radiation of suitable frequency is incident on the  A and C are two metallic plates
surface of the metal, the energy is transferred from the radiation to free placed in an evacuated quartz
electrons. bulb.
 Hence the free electrons gets sufficient energy to cross the surface barrier  Galvanometer G and battery B
and this type is called photo electric emission. are connected in the circuit.
(e.g.) photo electric cells, phot diodes  When uv - light is incident on
(4) Secondary emission : plate C, and electric current
 When a beam of fast moving electrons strikes the surface of the metal, the flows in a circuit which is
kinetic energy is transferred to the free electrons on the metal surface. indicated by the deflection in
 Thus free eletrons get sufficient kinetic energy and emitted from the the galvanometer.
surface. This type is called seconday emission.  But if the plate A is irradiated
(e.g.) photo multiplier tube by uv - light, no current is
2. Briefly discuss the observations of Hertz, Hallwachs and Lenard. observed in the circuit.
Hertz experiment :  From these observations, it is concluded that when uv- light falls on the
 Heinrich Hertz successfully generating and detecting the existence of negative plate C, electrons are ejected from it, which are attracted by the
electromagnetic waves. positive plate A.
 He used high voltage induction coil to cause a spark discharge between two  Hence the circuit is completed and the current flows in it.
metallic spheres.  Thus the uv - light falling on the negative plate causes the electron emission
from the surface of the plate.
victory R. SARAVANAN. M.Sc., M.Phil., B.Ed PG ASST [PHYSICS], GBHSS, PARANGIPETTAI - 608 502
12 PHYSICS UNIT – 8 DUAL NATURE OF RADIATION AND MATTER COMPLETE GUIDE AND MODEL QUESTION
3. Explain the experimental set up for study of photo electric effect  From the graph,
Experiment for study of photo electric effect : (1) When the potential of A increases, the photo current also increases and
reaches a saturation value called saturation current.
(2) When a negative potential is applied to A, the photo current does not
immediately drop to zero, because the photo electrons are emitted wity
some definite and different kinetic energies.
(3) If the negative or retarding potential of A gradually increased, the photo
current starts decreasing and becomes zero at one particular negative
potential 𝑉𝑂
(4) The value of negative or retarding potential give to anode A which is just
sufficient to stop the most energetic photo electrons emitted and make the
 S is the source of electromagnetic wave of frequency ‘𝜈’ and intensity ‘I’
photo current zero is called stopping potential or cut - off potential (𝑉𝑂 )
 C is the cathode made up of photo sensitive material and is used to emit
(5) Here the initial kinetic energy of the fastest electron (0 is equal to the work
electrons.
done by the stopping potential to stop it. (i.e.)
 A is the anode which collects the emitted electrons 1 2
 A and C are placed in an evacuated glass envelope with a quartz window that 𝑒 𝑉𝑂 = 𝑚 𝑣𝑚𝑎𝑥
2
permits uv -light and visible light.
 PQ is a potential divider arrangement which is connected through a key K and 𝟐 𝒆 𝑽𝑶
(𝑜𝑟) 𝒗𝒎𝒂𝒙 = √ = 𝟓. 𝟗𝟑 𝑿 𝟏𝟎𝟓 √𝑽𝑶
battery B .The voltmeter ‘V’ and micro ammeter ‘A’ also included in this circuit. 𝒎
 If there is no light incident on the cathode C, no photoelectrons are emitted and 5. Explain how frequency of incident light varies with stopping potential.
the micro ammeter reads zero. Effect of frequency on photoelectric current :
 When uv - light or visible light is allowed to fall on C, the photo electrons are  Let the intensity of incident light is
emitted and are attracted towards anode. kept constant.
 As a result, the photo electric current is set up in the circuit which is measured  The variation of photo current with
using micro ammeter. The photo electric current depends following quantities, the Anode potential is studied for
(1) the intensity of incident light different incident frequencies.
(2) the potential difference between the electrodes  A graph is plotted by taking anode
(3) the nature of the material potential along x - axis and photo
(4) frequency of incident light current along y - axis
4. Explain the effect of potential difference on photo electric current.  From the graph,
Effect of potential difference on photoelectric current : (1) Stopping potential vary over different frequencies of incident light. (i.e)
 Let the frequency and Greater the frequency, larger the stopping potential
intensity of the incident light (2) Thus as the frequency is increased, the photoelectrons are emitted with
are kept constant. greater kinetic energies so that the retarding potential needed to stop
 Now, the potential of A is thephotoelectrons is also greater.
increased and the Variation of stopping potential with frequency :
corresponding photocurrent  From the graph,
is noted. (1) The stopping potential varies
 Simillarly, a negative linealy with frequency.
(retarding) potential is (2) Below a particular frequency
applied to A and again the called threshold frequency (𝝂𝑶 ),
photocurrent is noted. no electrons are emitted.
 Plot a graph by taking anode (3) Hence at threshold frequency
potential along X -axis and stopping potential is zero for that
photo current along Y - axis reason.

victory R. SARAVANAN. M.Sc., M.Phil., B.Ed PG ASST [PHYSICS], GBHSS, PARANGIPETTAI - 608 502
12 PHYSICS UNIT – 8 DUAL NATURE OF RADIATION AND MATTER COMPLETE GUIDE AND MODEL QUESTION
6. List out the laws of photoelectric effect. where m  mass of the electron and
Laws of photoelectric effect : υ  velocity
 For a given frequency of incident light, the number of photoelectrons emitted is  At threshold frequency, the kinetic energy of ejeced electrons will be zero. (i.e.)
directly proportional to the intensity of the incident light. The saturation current when. 𝜈 = 𝜈0 then 𝐾 = 0 Thus eqn (1) becomes
is also directly proportional to the intensity of incident light. ℎ 𝜈0 = 𝜙0 − − − − (2)
 Maximum kinetic energy of the photo electrons is independent of intensity of the  Put eqn (2) in (1)
incident light. 1
ℎ 𝜈 = ℎ 𝜈0 + 𝑚 𝑣 2 − − − − (3)
 Maximum kinetic energy of the photo electrons from a given metal is directly 2
proportional to the frequency of incident light.  The equation (3) is known as Einstein’s photoelectric equation.
 For a given surface, the emission of photo electrons takes place only if the  If the electron does not lose energy by
frequency of incident light is greater than a certain minimum frequency called internal collisions, then it is emitted with
the threshold frequency. maximum kinetic energy Kmax. Then
 There is no time lag between incidence of light and ejection of photoelectrons. 1
ℎ 𝜈 = ℎ 𝜈0 + [ 𝑚 𝑣 2 ]
(i.e.) phote electric effect is an instantaneous process 2 𝑚𝑎𝑥
7. Explain the particle nature of light. List the characteristics of photons. 1 2
(𝑜𝑟) 𝑚 𝑣𝑚𝑎𝑥 = ℎ 𝜈 − ℎ 𝜈0
Particle nature of light : 2
 According to Eienstein, the energy in light is not spread out over wavefronts, (𝑜𝑟) 𝑲𝒎𝒂𝒙 = 𝒉 𝝂 − 𝝓𝟎 − − − − (𝟒)
but is concentrated in small packets or energy quanta.  A graph between maximum kinetic energy
 The energy of each light quantum is ; 𝑬 = 𝒉 𝝂 Kmax of the photoelectron and frequency ν
 The individual light quantum of definite energy and momentum can be of the incident light is a straight line
associated with a particle and this is called photon. 9. Explain experimentally observed facts of photoelectric effect with the help of
Characteristics of photons : Einstein’s explanation.
𝒉𝒄 Explanation for photo electric effect :
 Each photon will have energy given by 𝑬 = 𝒉 𝝂 =
𝝀  As each photon liberates one electron, then the increase of intensity of the light
 The energy of a photon is determined by the frequency of the radiation and not increases the number of electrons emitted there by increasing the photo
by its intensity. current.
 The photons travel with the velocity of light and its momentum is given by,  From, 𝑲𝒎𝒂𝒙 = 𝒉 𝝂 − 𝝓𝟎 , it is evident that Kmax is proportional to the
𝒉 𝒉𝝂 frequency of the light and is independent of intensity of the light.
𝒑= =
𝝀 𝒄 𝟏
 From, 𝒉 𝝂 = 𝒉 𝝂𝟎 + 𝒎 𝒗𝟐 , there must be minimum energy (equal to the work
 Photons are electrically neutral, and hency they are not deflected ny electric and 𝟐
magnetic fields. function of the metal) for incident photons to liberate electrons from the metal
 When photon interacts with matter, the total energy, total linear momentum surface. Below which, emission of electrons is not possible. Correspondingly,
and angular momentum are conserved. there exists minimum frequency called threshold frequency below which there
8. Obtain Einstein’s photoelectric equation with necessary explanation. is no photoelectric emission.
Einstein’s explanation of photoelectric equation :  According to quantum concept, the transfer of photon energy to the electrons is
 When a photon of energy ‘hν’ is incident on a metal surface, it is completely instantaneous so that there is no time lag between incidence of photons and
absorbed by a single electron and the electron is ejected. ejection of electrons.
 In this process, the energy of incident photom is utilized in two ways. 10. Explain photo electric cells and its types.
(1) Part of the photon energy is used for the ejection of the electrons from the Phot electric cell :
metal surface and it is called work function (𝝓𝟎 )  Photo electric cell or photo cell is a device which converts light energy into
(2) Remaining energy as the kinetic energy (K) of the ejected electron. electrical energy.
 From the law of conservation of energy,  It works on the principle of photo electric effect.
ℎ 𝜈 = 𝜙0 + 𝐾  When light is incident on the photosensitive materials, their electric properties
1 will get affected, based on which photo cells are classified into three types. They
(𝑜𝑟) ℎ 𝜈 = 𝜙0 + 𝑚 𝑣 2 − − − − (1) are Phote emissive cell, Phot voltaic cell and Photo conductive cell
2

victory R. SARAVANAN. M.Sc., M.Phil., B.Ed PG ASST [PHYSICS], GBHSS, PARANGIPETTAI - 608 502
12 PHYSICS UNIT – 8 DUAL NATURE OF RADIATION AND MATTER COMPLETE GUIDE AND MODEL QUESTION
(1) Photo emissive cell :  Hence the speed of the electron is.
 Its working depends on the electron emission from a metal cathode due to 2𝑒𝑉
𝑣2 =
irradiation of light or other radiations. 𝑚
(2) Photo voltaic cell : 2𝑒𝑉
 Here sensitive element made of semiconductor is used which generates 𝑣= √ − − − − (1)
voltage proportional tothe intensity of light or other radiations. 𝑚
(3) Photo conductive cell :  The de Broglie wavelength of electron is
 In this, the resistance of the semiconductor changes in accordance with the ℎ ℎ
𝜆= =
radiant energy incident on it. 𝑚𝑣 2𝑒𝑉
11. Give the construction and working of photo emissive cell. 𝑚√
𝑚
Photo emissive cell : 𝒉
 It consists of an evacuated glass or quartz bulb in 𝝀= − − − (2)
√𝟐 𝒎 𝒆 𝑽
which two metallic electrodes a cathode and an
anode are fixed. where, ℎ = 6.626 𝑋 10−34 𝐽 𝑠
 The cathode C is semi-cylindrical in shape and is 𝑒 = 1.6 𝑋 10−19 𝐶
coated with a photo sensitive material. 𝑚 = 9.11 𝑋 10−31 𝑘𝑔
 The anode A is a thin rod or wire kept along the 𝟏𝟐. 𝟐𝟕 𝑿 𝟏𝟎−𝟏𝟎 𝟏𝟐. 𝟐𝟕
axis of the semi-cylindrical cathode. ∴ 𝝀= = 𝑨𝒐
 A potential difference is applied between the √𝑽 √𝑽
14. Describe briefly Davisson – Germer experiment which demonstrated the wave
anode and the cathode through a galvanometer G.
nature of electrons.
Working :
Davisson - Gerner experiment :
 When cathode is illuminated, electrons are
 De Broglie hypothesis of matter
emitted from it.
waves was experimentally
 These electrons are attracted by anode and hence confirmed by Clinton Davisson and
a current is produced which is measured by the Lester Germer in 1927.
galvanometer.
 They demonstrated that electron
 For a given cathode, the magnitude of the current depends on beams are diffracted when they fall
(1) the intensity to incident radiation and on crystalline solids.
(2) the potential difference between anode and cathode.
 Since crystal can act as a three-
12. Give the application of photo cells .
dimensional diffraction grating for
Applications of photo cells :
matter waves, the electron waves
 Photo cells have many applications especially as switches and sensors. incident on crystals are diffracted
 Automatic lights that turn on when it gets dark use photocells, as well as street off in certain specific directions.
lights that switch on and off according to whether it is night or day.  The filament F is heated by a low
 Photo cells are used for reproduction of sound in motion pictures tension (L.T.) battery so that
 They are used as timers to measure the speeds of athletes during a race. electrons are emitted from the hot
 Photo cells of exposure meters in photography are used to measure the filament by thermionic emission.
intensity of the given light and to calculate the exact time of exposure.  They are then accelerated due to the potential difference between the filament
13. Derive an expression for de Broglie wavelength of electrons. and the anode aluminium cylinder by a high tension (H.T.) battery.
De Boglie wavelength of electrons :  Electron beam is collimated by using two thin aluminium diaphragms and is
 An electron of mass m is accelerated through a potential difference of V volt. allowed to strike a single crystal of Nickel.
 The kinetic energy acquired by the electron is given by  The electrons scattered by Ni atoms in different directions are received by the
1
𝑚 𝑣2 = 𝑒 𝑉 electron detector which measures the intensity of scattered electron beam.
2

victory R. SARAVANAN. M.Sc., M.Phil., B.Ed PG ASST [PHYSICS], GBHSS, PARANGIPETTAI - 608 502
12 PHYSICS UNIT – 8 DUAL NATURE OF RADIATION AND MATTER COMPLETE GUIDE AND MODEL QUESTION
 The detector is rotatable in the Principle :
plane of the paper so that the  The wave nature of the electron is used in the construction of microscope called
angle 𝜃 between the incident electron microscope.
beam and the scattered beam can  The resolving power of a microscope is inversely proportional to the
be changed at our will. wavelength of the radiation used.
 The intensity of the scattered  Thus higher resolving power can be obtained by employing the waves of
electron beam is measured as a shorter wavelengths.
function of the angle θ.  De Broglie wavelength of electron is very much less than (a few thousands less)
 The graph shows the variation of that of the visible light.
intensity of the scattered  As a result, the microscopes employing de Broglie waves of electrons have very
electrons with the angle θ for the accelerating voltage of 54V. much higher resolving power than optical microscope.
 For a given accelerating voltage V, the scattered wave shows a peak or  Electron microscopes giving magnification more than 2,00,000 times are
maximum at an angle of 50° to the incident electron beam. common in research laboratories.
 This peak in intensity is attributed to the constructive interference of electrons Working :
diffracted from various atomic layers of the target material.  The construction and working of an electron microscope is similar to that of an
 From the known value of interplanar spacing of Nickel, the wavelength of the optical microscope except that in electron microscope focussing of electron
electron wave has been experimentally calculated as 1.65Å. beam is done by the electrostatic or magnetic lenses.
 The wavelength can also be calculated from de Broglie relation for V = 54 V as  The electron beam passing across a suitably arranged either electric or
12.27 𝑜 12.27 𝑜 magnetic fields undergoes divergence or convergence thereby focussing of the
𝝀= 𝐴 = 𝐴 = 𝟏. 𝟔𝟕 𝐀
√𝑉 √54 beam is done
 This value agrees well with the experimentally observed wavelength of 1.65Å.  The electrons emitted from the source are accelerated by high potentials.
 Thus this experiment directly verifies de Broglie’s hypothesis of the wave  The beam is made parallel by magnetic condenser lens.
nature of moving particles.  When the beam passes through the sample whose magnified image is needed,
15. Briefly explain the principle and working of electron microscope. the beam carries the image of the sample.
Electron microscope :  With the help of magnetic objective lens and magnetic projector lens system,
the magnified image is obtained on the screen.
 These electron microscopes are being used in almost all branches of science.

victory R. SARAVANAN. M.Sc., M.Phil., B.Ed PG ASST [PHYSICS], GBHSS, PARANGIPETTAI - 608 502
12 PHYSICS UNIT – 8 DUAL NATURE OF RADIATION AND MATTER COMPLETE GUIDE AND MODEL QUESTION
2. A radiation of wavelength 300 nm is incident on a silver surface. Will
EXAMPLE PROBLEMS WITH SOLUTIONS photoelectrons be observed? [work function of silver = 4.7 eV]
1. For the photoelectric emission from cesium, show that wave theory predicts -Solution :- : 𝜆 = 300 𝑛𝑚 = 300 𝑋 10−9 𝑚
that (i) maximum kinetic energy of the photoelectrons (Kmax) depends on the  Energy of the incident photon is
intensity I of the incident light (ii) Kmax does not depend on the frequency of the ℎ𝑐
incident light and (iii) the time interval between the incidence of light and the 𝐸 =ℎ𝜈= (𝑗𝑜𝑢𝑙𝑒)
𝜆
ejection of photoelectrons is very long. (Given : The work function for cesium is ℎ𝑐
2.14 eV and the power absorbed per unit area is 𝟏. 𝟔𝟎 𝑿 𝟏𝟎−𝟔 𝑾 𝒎−𝟐 which 𝐸= (𝑒𝑉)
𝜆𝑒
produces a measurable photocurrent in cesium.) 6.626 𝑋 10−34 𝑋 3 𝑋 108
-Solution :- 𝐸=
300 𝑋 10−9 𝑋 1.6 𝑋 10−19
For the sake of simplicity, the following standard assumptions can be made when 6.626 𝑋 10−26 6.626 66.26
light is incident on the given material. 𝐸= = =
100 𝑋 10−28 𝑋 1.6 1.6 16
(a) Light is absorbed in the top atomic layer of the metal 𝑬 = 𝟒. 𝟏𝟒 𝒆𝑽
(b) For a given element, each atom absorbs an equal amount of energy and this
 The work function of silver = 4.7 eV. Since the energy of the incident photon is
energy is proportional to its cross-sectional area A
less than the work function of silver, photoelectrons are not observed in this case.
(c) Each atom gives this energy to one of the electrons.
3. When light of wavelength 2200Å falls on Cu, photo electrons are emitted from
(i) According to wave theory, the energy in a light wave is spread out uniformly and
it. Find (i) the threshold wavelength and (ii) the stopping potential. Given: the
continuously over the wavefront.
work function for Cu is ϕ0 = 4.65 eV.
 The energy absorbed by each electron in time t is given by E = I A t -Solution :- 𝜆 = 2200 𝐴° = 2200 𝑋 10−10 𝑚 ; 𝜙𝑂 = 4.65 𝑒𝑉 = 4.65 𝑋 1.6 𝑋 10−19 𝐽
 With this energy absorbed, the most energetic electron is released with Kmax ℎ𝑐
(i) Work function ; 𝜙𝑂 = ℎ 𝜈𝑂 =
by overcoming the surface energy barrier or work function ϕ0 and this is 𝜆𝑂
expressed as ; 𝑲𝒎𝒂𝒙 = 𝑰 𝑨 𝒕 − 𝝓𝑶 Hence threshold wavelength,
 Thus, wave theory predicts that for a unit time, at low light intensities when ℎ𝑐 6.626 𝑋 10−34 𝑋 3 𝑋 108 19.878 𝑋 10−26
𝜆𝑂 = = =
𝑰 𝑨 < 𝝓𝑶 , no electrons are emitted. At higher intensities, when 𝑰 𝑨 ≥ 𝝓𝑶 , 𝜙𝑂 4.65 𝑋 1.6 𝑋 10−19 7.44 𝑋 10−19
electrons are emitted. This implies that higher the light intensity, greater 19.878 𝑋 10 −7

will be Kmax. 𝜆𝑂 =
7.44
 Kmax is dependent only on the intensity under given conditions – that is, by 𝝀𝑶 = 𝟐. 𝟔𝟕𝟐 𝑿 𝟏𝟎−𝟕 𝒎 = 𝟐𝟔𝟕𝟐 𝑿 𝟏𝟎−𝟏𝟎 𝒎 = 𝟐𝟔𝟕𝟐 𝑨°
suitably increasing the intensity, one can produce photoelectric effect even (ii) By Einstein’s photo electric equation; 𝐾𝑚𝑎𝑥 = ℎ𝜈 − 𝜙𝑂 (𝑜𝑟) 𝑒 𝑉𝑂 = ℎ𝜈 − 𝜙𝑂
if the frequency is less than the threshold frequency. So the concept of ℎ𝑐
 Energy of incident photon; 𝐸 = ℎ 𝜈 =
threshold frequency does not even exist in wave theory. 𝜆
(ii) According to wave theory, the intensity of a light wave is proportional to the 6.626 𝑋 10−34 𝑋 3 𝑋 108 19.878 𝑋 10−16
𝐸= =
square of the amplitude of the electric field (𝐸 2𝑂 ). The amplitude of this electric 2200 𝑋 10−10 2200
field increases with increasing intensity and imparts an increasing acceleration 𝐸 = 9. 036 𝑋 10−3 𝑋 10−16 = 9. 036 𝑋 10−19 𝐽
and kinetic energy to an electron. This means that Kmax should not depend at all  and working function,
on the frequency of the classical light wave which again contradicts the 𝜙𝑂 = 4. 65 𝑒𝑉 = 4.65 𝑋 1.6 𝑋 10−19 𝐽 = 7.44 𝑋10−19 𝐽
experimental results.  Hence, 𝑒 𝑉𝑂 = ℎ𝜈 − 𝜙𝑂
(iii) If an electron accumulates light energy just enough to overcome the work 𝑒 𝑉𝑂 = 9. 036 𝑋 10−19 − 7.44 𝑋10−19 = 1.596 𝑋 10−19 𝐽
function, then it is ejected out of the atom with zero kinetic energy. Therefore, (𝑜𝑟) 𝑒 𝑉𝑂 = 1. 6 𝑋 10−19 𝐽
0 = 𝐼 𝐴 𝑡 − 𝜙𝑂 (𝑜𝑟) 𝐼 𝐴 𝑡 = 𝜙𝑂  Then stopping potential,
𝜙𝑂 𝜙𝑂 2.14 𝑋 1.6 𝑋 10−19
∴ 𝑡= = = 2 = 𝟔. 𝟖 𝑿 𝟏𝟎𝟔 𝒔 ≈ 𝟕𝟗 𝒅𝒂𝒚𝒔 1. 6 𝑋 10−19 1. 6 𝑋 10−19
𝐼𝐴 𝐼 𝜋 𝑟2 1.60 𝑋 10−6 𝑋 3.14 𝑋 (1 𝑋 10−10 ) 𝑉𝑂 = =
𝑒 1. 6 𝑋 10−19
 Thus, wave theory predicts that there is a large time gap between the 𝑽𝑶 = 𝟏 𝑽
incidence of light and the ejection of photoelectrons but the experiments
show that photo emission is an instantaneous process

victory R. SARAVANAN. M.Sc., M.Phil., B.Ed PG ASST [PHYSICS], GBHSS, PARANGIPETTAI - 608 502
12 PHYSICS UNIT – 8 DUAL NATURE OF RADIATION AND MATTER COMPLETE GUIDE AND MODEL QUESTION
4. The work function of potassium is 2.30 eV. UV light of wavelength 3000 Å and (ii) Threshold wavelength ;
intensity 2 Wm–2 is incident on the potassium surface. (i) Determine the ℎ𝑐 6.626 𝑋 10−34 𝑋 3 𝑋 108
maximum kinetic energy of the photo electrons (ii) If 40% of incident photons 𝜆𝑂 = =
𝜙𝑂 3.337 𝑋 10−19
produce photo electrons, how many electrons are emitted per second if the 19.878 𝑋 10−26 19.878 𝑋 10−7
area of the potassium surface is 2 cm2 ? 𝜆𝑂 = =
3.337 𝑋 10−19 3.337
-Solutio:𝜆 = 3000 𝐴° = 3000 𝑋 10−10 𝑚 ; 𝜙𝑂 = 2.30 𝑒𝑉 ; 𝐴 = 2 𝑐𝑚2 = 2 𝑋 10−4 𝑚2 𝝀𝑶 = 𝟓. 𝟗𝟓𝟕 𝑿 𝟏𝟎−𝟕 𝒎 = 𝟓𝟗𝟓𝟕 𝑿 𝟏𝟎−𝟏𝟎 𝒎 = 𝟓𝟗𝟓𝟕 𝑨°
ℎ𝑐
(i) Energy of incident photon; 𝐸 =ℎ𝜈= 6. Calculate the momentum and the de Broglie wavelength in the following cases:
𝜆
6.626 𝑋 10−34 𝑋 3 𝑋 108 6.626 𝑋 10−36 (i) an electron with kinetic energy 2 eV.
𝐸= = = 𝟔. 𝟔𝟐𝟔 𝑿 𝟏𝟎−𝟏𝟗 𝑱 (ii) a bullet of 50 g fired from rifle with a speed of 200 m/s
3000 𝑋 10−10 1000 𝑋 10−10
6.626 𝑋 10−19 6.626 66.26 (iii) a 4000 kg car moving along the highways at 50 m/s
𝐸= −19
𝑒𝑉 = 𝑒𝑉 = 𝑒𝑉 = 𝟒. 𝟏𝟒 𝒆𝑽 Hence show that the wave nature of matter is important at the atomic level but
1.6 𝑋 10 1.6 16
By Einstien’s photo electric equation, the maximum kinetic energy is, is not really relevant at macroscopic level.
𝐾𝑚𝑎𝑥 = ℎ𝜈 − 𝜙𝑂 -Solution :-
𝐾𝑚𝑎𝑥 = 4.14 − 2.30 = 𝟏. 𝟖𝟒 𝒆 𝑽 [∵ 1 𝑒𝑉 = 1.6 𝑋 10−19 𝐽] (i) 𝐾 = 2 𝑒𝑉 = 2 𝑋 1.6 𝑋 10−19 𝐽 ; 𝑚 = 9.1` 𝑋 10−31 𝑘𝑔
(𝒐𝒓) 𝑲𝒎𝒂𝒙 = 𝟐. 𝟗𝟒𝟔 𝑿 𝟏𝟎 −𝟏𝟗 Momentum of electron,
𝑃 𝑝 = √2 𝑚 𝐾
(ii) Number of photons reaches the surface per second; 𝑛𝑃 = 𝑋 𝐴
𝐸
2 −4
4 15
𝑝 = √2 𝑋 9.1` 𝑋 10−31 𝑋 2 𝑋 1.6 𝑋 10−19
𝑛𝑃 = 𝑋 2 𝑋 10 = 𝑋 10
6.626 𝑋 10−19 6.626 𝑝 = √58.24 𝑋 10−50 = 𝟕. 𝟔𝟑𝟏 𝑿 𝟏𝟎−𝟐𝟓 𝒌𝒈 𝒎 𝒔−𝟏
−1 15 𝟏𝟒
𝑛𝑃 = 6.037 𝑋 10 𝑋 10 = 𝟔. 𝟎𝟑𝟕 𝑿 𝟏𝟎 𝒑𝒉𝒐𝒕𝒐𝒏𝒔/𝒔𝒆𝒄 Hence de Broglie wavelength of electron,
Hence rate of emission of photons, ℎ 6.626 𝑋 10−34
𝑛 = 40% 𝑛𝑃 = 0.40 𝑋 𝑛𝑃 = 0.4 𝑋 6.037 𝑋 1014 𝜆= =
𝑝 7. 631 𝑋 10−25
𝒏 = 𝟐. 𝟒𝟏𝟓 𝑿 𝟏𝟎𝟏𝟒 𝒑𝒉𝒐𝒕𝒐𝒏𝒔/𝒔𝒆𝒄
6.626 𝑋 10−9
5. Light of wavelength 390 nm is directed at a metal electrode. To find the energy 𝜆=
of electrons ejected, an opposing potential difference is established between it 7. 631
𝜆 = 8. 684 𝑋 10−1 𝑋 10−9 = 8. 684 𝑋 10−10 𝑚
and another electrode. The current of photoelectrons from one to the other is
𝝀 = 𝟖. 𝟔𝟖𝟒 𝑨
stopped completely when the potential difference is 1.10 V. Determine (i) the
(ii) 𝑚 = 50 𝑔 = 50 𝑋 10−3 𝑘𝑔 ; 𝑣 = 200 𝑚𝑠 −1 , then momentum of bullet,
work function of the metal and (ii) the maximum wavelength of light that can
eject electrons from this metal. 𝑝 = 𝑚 𝑣 = 50 𝑋 10−3 𝑋 200 = 10000 𝑋 10−3
-Solution :- 𝜆 = 390 𝑛𝑚 = 390 𝑋 10−9 𝑚 ; 𝑉𝑂 = 1.10 𝑉 𝒑 = 𝟏𝟎 𝒌𝒈 𝒎 𝒔−𝟏
(i) By Einstien’s photo electric equation,, Hence de Broglie wavelength of bullet,
ℎ 6.626 𝑋 10−34
𝐾𝑚𝑎𝑥 = ℎ𝜈 − 𝜙𝑂 (𝑜𝑟) 𝑒 𝑉𝑂 = ℎ𝜈 − 𝜙𝑂 𝜆= = = 𝟔. 𝟔𝟐𝟔 𝑿 𝟏𝟎−𝟑𝟓 𝒌𝒈 𝒎 𝒔−𝟏
 Hence working function, 𝑝 10
ℎ𝑐 (iii) 𝑚 = 4000 𝑘𝑔 ; 𝑣 = 50 𝑚𝑠 −1 then momentum of car,
𝜙𝑂 = ℎ𝜈 − 𝑒𝑉𝑂 = − 𝑒𝑉𝑂 𝑝 = 𝑚 𝑣 = 4000 𝑋 50 = 200000 = 𝟐 𝑿 𝟏𝟎𝟓 𝒌𝒈 𝒎 𝒔−𝟏
𝜆
6.626 𝑋 10−34 𝑋 3 𝑋 108 Hence de Broglie wavelength of car,
𝜙𝑂 = [ ] − [1.6 𝑋 10−19 𝑋 1.10] ℎ 6.626 𝑋 10−34
390 𝑋 10−9 𝜆= = = 𝟑. 𝟑𝟏𝟑 𝑿 𝟏𝟎−𝟑𝟗 𝒌𝒈 𝒎 𝒔−𝟏
19.878 𝑋 10−17 𝑝 2 𝑋 105
𝜙𝑂 = [ ] − [1.76 𝑋 10−19 ]  From these calculations, we notice that electron has significant value of de Broglie
390
wavelength (≈10–9m which can be measured from diffraction studies) but moving
𝜙𝑂 = [5.097 𝑋 10−19 ] − [1.76 𝑋 10−19 ] = [5.097 − 1.76] 𝑋 10−19
bullet and car have negligibly small de Broglie wavelengths associated with them
𝝓𝑶 = 𝟑. 𝟑𝟑𝟕 𝑿 𝟏𝟎−𝟏𝟗 𝑱 (≈10–33m and 10–39m respectively, which are not measurable by any experiment).
3.337 𝑋 10−19 33.37
(𝒐𝒓) 𝝓𝑶 = 𝑒𝑉 =  This implies that the wave nature of matter is important at the atomic level but it
1.6 𝑋 10−19 16 is not really relevant at the macroscopic level.
𝝓𝑶 = 𝟐. 𝟎𝟖𝟓 𝒆𝑽

victory R. SARAVANAN. M.Sc., M.Phil., B.Ed PG ASST [PHYSICS], GBHSS, PARANGIPETTAI - 608 502
12 PHYSICS UNIT – 8 DUAL NATURE OF RADIATION AND MATTER COMPLETE GUIDE AND MODEL QUESTION
7. Find the de Broglie wavelength associated with an alpha particle which is 9. Calculate the cut-off wavelength and cut-off frequency of x-rays from an x –ray
accelerated through a potential difference of 400 V. Given that the mass of the tube of accelerating potential 20,000 V.
proton is 1.67 × 10–27 kg. -Solution :- 𝑉 = 20000 𝑉
-Solution :- 𝑉 = 400 𝑉 ; 𝑚𝑃 = 1.67 𝑋 10−27 𝑘𝑔  The cut-off wavelength of the x-rays in the continuous spectrum is given by,
 An alpha particle contains 2 protons and 2 neutrons. It is represented by 42𝐻𝑒 12400
𝜆𝑂 = 𝐴°
 Hence ; 𝑞 = 2𝑒 = 2 𝑋 1.6 𝑋 10−19 = 3.2 𝑋 10−19 𝐶 𝑉
𝑀 = 4 𝑚𝑃 = 4 𝑋 1.67 𝑋 10−27 = 6.68 𝑋 10−27 𝑘𝑔 12400 12400
𝜆𝑂 = 𝐴° = 𝐴° = 6200 𝑋 10−4 𝐴°
 The de Broglie wavelength associated with it is, 20000 2 𝑋 104
ℎ 𝝀𝑶 = 𝟎. 𝟔𝟐 𝑨°
𝜆=  The corresponding frequency is
√2 𝑀 𝑞 𝑉 𝑐
6.626 𝑋 10−34 𝜈𝑂 =
𝜆= 𝜆𝑂
√2 𝑋 6.68 𝑋 10−27 𝑋 3.2 𝑋 10−19 𝑋 400 3 𝑋 108 3 𝑋 1018
6.626 𝑋 10−34 6.626 𝑋 10−11 𝜈𝑂 = =
𝜆= = 0.62 𝑋 10−10 0.62
√17100.8 𝑋 10−46 √17100.8 𝝂𝑶 = 𝟒. 𝟖𝟑𝟖 𝑿 𝟏𝟎𝟏𝟖 𝑯𝒛
𝜆 = 5. 068 𝑋 10−2 𝑋10−11
𝝀 = 𝟓. 𝟎𝟔𝟖 𝑿 𝟏𝟎−𝟏𝟑 𝒎 = 𝟎. 𝟎𝟎𝟓𝟎𝟔𝟖 𝑿 𝟏𝟎−𝟏𝟎 𝒎 = 𝟎. 𝟎𝟎𝟓𝟎𝟔𝟖 𝑨°
8. A proton and an electron have same de Broglie wavelength. Which of them moves
faster and which possesses more kinetic energy?
Solution :-

 de Broglie wavelength of proton ; 𝜆𝑃 =
√2 𝑚𝑃 𝐾𝑃

 de Broglie wavelength of electron ; 𝜆𝑒 =
√2 𝑚𝑒 𝐾𝑒
 Since proton and electron have same de Broglie wavelength (𝜆𝑃 = 𝜆𝑒 ), we get,
ℎ ℎ
=
√2 𝑚𝑃 𝐾𝑃 √2 𝑚𝑒 𝐾𝑒
𝐾𝑃 𝑚𝑒
(𝑜𝑟) = − − − − − − − − (1)
𝐾𝑒 𝑚𝑃
Since 𝒎𝒆 < 𝒎𝑷 ; 𝑲𝑷 < 𝑲𝒆 (𝑖. 𝑒. ) the electron has more kinetic energy than
the proton.
1
 Also kinetic energy of proton ; 𝐾𝑃 = 𝑚𝑃 𝑣𝑃2
2
1
 And kinetic energy of eletron ; 𝐾𝑒 = 𝑚𝑒 𝑣𝑒2
2
1
𝐾𝑃 𝑚 𝑣2
2 𝑃 𝑃
 Then ratio of the kinetic energies ; = 1
𝐾𝑒 𝑚 𝑣2
2 𝑒 𝑒

𝑣𝑃2 𝐾𝑃 𝑚𝑒 𝑣𝑃 𝐾𝑃 𝑚𝑒
= (𝑜𝑟) = √
𝑣𝑒2 𝐾𝑒 𝑚𝑃 𝑣𝑒 𝐾𝑒 𝑚𝑃
𝑣𝑃 𝑚 𝑚𝑒 𝑚2 𝒎𝒆
 Put equation (1),
𝑣𝑒
= √𝑚 𝑒 = √𝑚𝑒2 =
𝑃 𝑚𝑃 𝑃 𝒎𝑷
Since 𝒎𝒆 < 𝒎𝑷 ; 𝒗𝑷 < 𝒗𝒆 (𝑖. 𝑒. ) the electron moves faster than the proton.

victory R. SARAVANAN. M.Sc., M.Phil., B.Ed PG ASST [PHYSICS], GBHSS, PARANGIPETTAI - 608 502
12 PHYSICS UNIT – 8 DUAL NATURE OF RADIATION AND MATTER COMPLETE GUIDE AND MODEL QUESTION
−9
(ii) If 𝜆 = 0.1 𝑛𝑚 = 0.1 𝑋 10 𝑚 then, energy of X-ray photon,
EXERCISE PROBLEMS WITH SOLUTIONS ℎ𝑐
𝐸 =ℎ𝜈=
1. How many photons per second emanate from a 50 mW laser of 640 nm? 𝜆
-Solution :- 𝑃 = 50 𝑚𝑊 = 50 𝑋 10−3 𝑊 ; 𝜆 = 640 𝑛𝑚 = 640 𝑋 10−9 𝑚 6.626 𝑋 10−34 𝑋 3 𝑋 108 19.878 𝑋 10−36
𝐸= =
 Number of photons per second, 0.1 𝑋 10−9 1 𝑋 10−10
−16
𝑃 𝑃 𝑃 𝑃𝜆 𝐸 = 19.878 𝑋 10 𝐽
𝑛𝑃 = = = = 19.878 𝑋 10−17 19.878 𝑋 102 198.78 𝑋 102
𝐸 ℎ𝜈 (ℎ 𝑐⁄𝜆) ℎ𝑐 (𝑜𝑟) 𝐸= 𝑒𝑉 = 𝑒𝑉 = 𝑒𝑉
1.6 𝑋 10−19 1.6 16
50 𝑋 10−3 𝑋 640 𝑋 10−9 32000 𝑋 10−12 2
𝐸 = 12.42 𝑋 10 𝒆𝑽 = 𝟏𝟐𝟒𝟐 𝒆𝑽
𝑛𝑃 = =
6.626 𝑋 10 −34 𝑋 3 𝑋 10 8 19.878 𝑋 10−26 (iii) If 𝜆 = 10 𝑚 then, energy of radio waves,
32000 𝑋 10 14 ℎ𝑐
𝑛𝑃 = = 1.610 𝑋 103 𝑋 1014 = 𝟏. 𝟔𝟏 𝑿 𝟏𝟎𝟏𝟕 𝐸 =ℎ𝜈=
19.878 𝜆
2. Calculate the maximum kinetic energy and maximum velocity of the 6.626 𝑋 10−34 𝑋 3 𝑋 108 19.878 𝑋 10−36
𝐸= =
photoelectrons emitted when the stopping potential is 81V for the 10 10
−27
photoelectric emission experiment. 𝐸 = 19.878 𝑋 10 𝐽
-Solution :- 𝑉𝑂 = 81 𝑉 19.878 𝑋 10−27 19.878 𝑋 10−8 198.78 𝑋 10−8
(𝑜𝑟) 𝐸= 𝑒𝑉 = 𝑒𝑉 = 𝑒𝑉
 The maximum kinetic energy of photo electrons is equal to stopping potential 1.6 𝑋 10 −19 1.6 16
−8 −𝟕
energy. (i.e.) 𝐸 = 12.42 𝑋 10 = 𝟏. 𝟐𝟒𝟐 𝑿 𝟏𝟎 𝒆𝑽
𝐾𝑚𝑎𝑥 = 𝑒 𝑉𝑂 4. A 150 W lamp emits light of mean wavelength of 5500 Å . If the efficiency is
𝐾𝑚𝑎𝑥 = 1.6 𝑋 10−19 𝑋 81 = 129.6 𝑋 10−19 12%, find out the number of photons emitted by the lamp in one second.
12
𝑲𝒎𝒂𝒙 = 𝟏. 𝟐𝟗𝟔 𝑿 𝟏𝟎−𝟏𝟕 𝑱 -Solution :- 𝑃 = 150 𝑊 ; 𝜆 = 5500 𝐴° = 5500 𝑋 10−10 𝑚 ; 𝜂 = 12% =
100
 But kinetic energy is given by  Number of photons emitted per second,
1 2 𝑃 𝑃 𝑃 𝑃𝜆
𝐾𝑚𝑎𝑥 = 𝑚𝑣𝑚𝑎𝑥 𝑛𝑃 = = = =
2 𝐸 ℎ𝜈 ℎ 𝑐 ℎ𝑐
1 ( ⁄𝜆)
1.296 𝑋 10−17 = 𝑋 9.1 𝑋 10−31 𝑋 𝑣𝑚𝑎𝑥 2
2 150 𝑋 5500 𝑋 10−10
2 𝑋 1.296 𝑋 10−17 2. 592 𝑋 1014 𝑛𝑃 =
2
𝑣𝑚𝑎𝑥 = = 6.626 𝑋 10−34 𝑋 3 𝑋 108
9.1 𝑋 10−31 9.1 825000 𝑋 10−10 825 𝑋 1019
𝑛𝑃 = = = 4.150 𝑋 101 𝑋 1019
2. 592 𝑋 1014 259.2 𝑋 1012 19.878 𝑋 10−26 19.878
∴ 𝑣𝑚𝑎𝑥 = √ = √ 𝒏𝑷 = 𝟒. 𝟏𝟓𝟎 𝑿 𝟏𝟎𝟐𝟎 𝒑𝒉𝒐𝒕𝒐𝒏𝒔/𝒔𝒆𝒄
9.1 9.1
 The number of photons emitted by the lamp in one second,
𝒗𝒎𝒂𝒙 = 𝟓. 𝟑𝟑𝟕 𝑿𝟏𝟎𝟔 𝒎 𝒔−𝟏 12
3. Calculate the energies of the photons associated with the following radiation: 𝑛 = 𝜂 𝑛𝑃 = 𝑋 4.150 𝑋 1020 = 12 𝑋 4.150 𝑋 1018 = 49.8 𝑋 1018
100
(i) violet light of 413 nm (ii) X-rays of 0.1 nm (iii) radio waves of 10 m. 𝒏 = 𝟒. 𝟗𝟖 𝑿 𝟏𝟎𝟏𝟗 𝒑𝒉𝒐𝒕𝒐𝒏𝒔/𝒔𝒆𝒄
-Solution :- 5. How many photons of frequency 1014 Hz will make up 19.86 J of energy?
(i) If 𝜆 = 413 𝑛𝑚 = 413 𝑋 10−9 𝑚 , then energy of violet light photon, 𝑈
-Solution :- 𝜈 = 1014 𝐻𝑧 ; 𝑃 = = 19.86 𝐽
ℎ𝑐 𝑡
𝐸 =ℎ𝜈=  Number of photons emitted per second,
𝜆
6.626 𝑋 10−34 𝑋 3 𝑋 108 19.878 𝑋 10−36 𝑃 𝑃
𝐸= = 𝑛𝑃 = =
413 𝑋 10−9 413 𝑋 10−9 𝐸 ℎ𝜈
−17 19.86
19.878 𝑋 10 𝑛𝑃 =
𝐸= 𝐽 6.626 𝑋10−34 𝑋 1014
413
19.878 𝑋 10 −17
19.878 𝑋 102 19.86 𝑋1020
(𝑜𝑟) 𝐸= 𝑒𝑉 = 𝑒𝑉 𝑛𝑃 =
413 𝑋 1.6 𝑋 10−19 660.8 6.626
−2 2
𝐸 = 3.008 𝑋 10 𝑋 10 = 𝟑. 𝟎𝟎𝟖 𝒆𝑽 ≈ 𝟑 𝒆𝑽 𝒏𝑷 = 𝟐. 𝟗𝟗𝟕 𝑿 𝟏𝟎𝟐𝟎 ≈ 𝟑 𝑿 𝟏𝟎𝟐𝟎

victory R. SARAVANAN. M.Sc., M.Phil., B.Ed PG ASST [PHYSICS], GBHSS, PARANGIPETTAI - 608 502
12 PHYSICS UNIT – 8 DUAL NATURE OF RADIATION AND MATTER COMPLETE GUIDE AND MODEL QUESTION
6. What should be the velocity of the electron so that its momentum equals that (iii) By Einstein’s photo electric equation,
of 4000 Å wavelength photon. 1 2
ℎ 𝜈 = 𝜙𝑂 + 𝑚 𝑣𝑚𝑎𝑥
-Solution :- 𝑝𝑒 = 𝑝𝑃 ; 𝜆𝑃 = 4000 𝐴° = 4000 𝑋 10−10 𝑚 2
 de Broglie wavelength of photon, 1 2
(𝑜𝑟) 𝜙𝑂 = ℎ 𝜈 − 𝑚 𝑣𝑚𝑎𝑥
ℎ ℎ ℎ 2
𝜆𝑃 = = = (𝑜𝑟) 𝜙𝑂 = ℎ 𝜈 − 𝑒 𝑉𝑂
𝑝𝑃 𝑝𝑒 𝑚 𝑣𝑒
𝜙𝑂 = (33.13 𝑋 10−20 ) − (1.6 𝑋 10−19 𝑋 0.8)
ℎ 6.626 𝑋10−34
∴ 𝑣𝑒 = = 𝜙𝑂 = (33.13 𝑋 10−20 ) − (1.28 𝑋 10−19 )
𝑚 𝜆𝑃 9.1 𝑋 10−31 𝑋 4000 𝑋 10−10 𝜙𝑂 = (3.313 𝑋 10−19 ) − (1.28 𝑋 10−19 )
6.626 𝑋107 6.626 𝑋104 𝜙𝑂 = (3.313 − 1.28 ) 𝑋 10−19
𝑣𝑒 = = = 1.821 𝑋 10−1 𝑋 104
9.1 𝑋 4000 36.4 𝝓𝑶 = 𝟐. 𝟎𝟑𝟑 𝑿 𝟏𝟎−𝟏𝟗 𝑱
𝒗𝒆 = 𝟏𝟖𝟐𝟏 𝒎 𝒔−𝟏 2.033 𝑋 10−19 20.33
7. When a light of frequency 9 X 1014 Hz is incident on a metal surface, (𝑜𝑟) 𝜙𝑂 = 𝑒𝑉 = 𝑒𝑉
1.6 𝑋 10−19 16
photoelectrons are emitted with a maximum speed of 8 X10 5 ms-1. Determine (𝑜𝑟) 𝝓𝑶 = 𝟏. 𝟐𝟕𝟎 𝒆𝑽
the threshold frequency of the surface. (iv) Work function,
-Solution :- : 𝜈 = 9 𝑋 1014 𝐻𝑧 ; 𝑣𝑚𝑎𝑥 = 8 𝑋 105 𝑚 𝑠 −1 𝜙𝑂 = ℎ 𝜈𝑂
 By Einstein’s photo electric equation, 𝜙𝑂 2.033 𝑋 10−19 2.033 𝑋 1015
1 2 (𝑜𝑟) 𝜈𝑂 = = =
ℎ 𝜈 = ℎ 𝜈𝑂 + 𝑚 𝑣𝑚𝑎𝑥 ℎ 6.626 𝑋10−34 6.626
2 𝝂𝑶 = 3. 068 𝑋 10−1 𝑋 1015 = 𝟑. 𝟎𝟔𝟖 𝑿 𝟏𝟎𝟏𝟒 𝑯
1 2
(𝑜𝑟) ℎ 𝜈𝑂 = ℎ 𝜈 − 𝑚 𝑣𝑚𝑎𝑥 (v) Net energy of the electron after it leaves the surface is nothing but its kinetic
2
1 energy which is given by,
ℎ 𝜈𝑂 = [6.626 𝑋10−34 𝑋9 𝑋 1014 ] − [ 𝑋 9.1 𝑋 10−31 𝑋 64 𝑋 1010 ] 𝐾𝑚𝑎𝑥 = ℎ𝜈 − 𝜙𝑂
2
𝐾𝑚𝑎𝑥 = 2.071 − 1.270
ℎ 𝜈𝑂 = [59.634 𝑋10−20 ] − [291.2 𝑋 10−21 ]
𝑲𝒎𝒂𝒙 = 𝟎. 𝟖𝟎𝟏 𝒆𝑽
(𝑜𝑟) ℎ 𝜈𝑂 = [59.634 𝑋10−20 ] − [29.12 𝑋 10−20 ]
9. A 3310 Å photon liberates an electron from a material with energy 3 X 10 -19 J
ℎ 𝜈𝑂 = [59.634 − 29.12] 𝑋10−20 = 30.514 𝑋10−20 while another 5000 Å photon ejects an electron with energy 0.972 X 10 -19 J
30.514 𝑋10−20 30.514 𝑋10−20 30.514 𝑋1014 from the same material. Determine the value of Planck’s constant and the
∴ 𝜈𝑂 = = =
ℎ 6.626 𝑋10 −34 6.626 threshold wavelength of the material.
𝝂𝑶 = 𝟒. 𝟔𝟎𝟑 𝑿𝟏𝟎𝟏𝟒 𝑯𝒛 -Solution :- 𝜆1 = 3310 𝐴° = 3310 𝑋 10−10 𝑚 ; 𝐾1 = 3 X 10−19 J
−10
8. When a 6000Å light falls on the cathode of a photo cell, photoemission takes 𝜆2 = 5000 𝐴° = 5000 𝑋 10 𝑚 ; 𝐾2 = 0.972 X 10−19 J
place. If a potential of 0.8 V is required to stop emission of electron, then  By Einstein’s photo electric equation,
determine the (i) frequency of the light (ii) energy of the incident photon (iii) ℎ 𝜈 = 𝜙𝑂 + 𝐾𝑚𝑎𝑥
work function of the cathode material (iv) threshold frequency and (v) net ℎ𝑐
(𝑜𝑟) = 𝜙𝑂 + 𝐾𝑚𝑎𝑥
energy of the electron after it leaves the surface. 𝜆
-Solution :- 𝜆 = 6000 𝐴° = 6000 𝑋 10−10 𝑚 ; 𝑉𝑂 = 0.8 𝑉  For given material, work function is constant ,
(i) Frequency of light , ℎ𝑐
= 𝜙𝑂 + 𝐾1 − − − − − − (1)
𝑐 3 𝑋 108 1 𝑋 1015 𝜆1
𝜈= = = = 0.5 𝑋 1015 ℎ𝑐
𝜆 6000 𝑋 10−10 2 = 𝜙𝑂 + 𝐾2 − − − − − − (2)
𝝂 = 𝟓 𝑿 𝟏𝟎𝟏𝟒 𝑯𝒛 𝜆2
(ii) Energy of incident photon, ℎ𝑐 ℎ𝑐
(1) − (2) ⟹ − = 𝜙𝑂 + 𝐾1 − 𝜙𝑂 − 𝐾2
𝐸 = ℎ 𝜈 = 6.626 𝑋10−34 𝑋 5 𝑋 1014 = 33.13 𝑋 10−20 𝐽 𝜆1 𝜆2
33.13 𝑋 10−20 331.3 𝑋 10−1 1 1
(𝑜𝑟) 𝐸= 𝑒𝑉 = 𝑒𝑉 = 20. 71 𝑋 10−1 𝑒𝑉 ℎ 𝑐 [ − ] = 𝐾1 − 𝐾2
1.6 𝑋 10−19 16 𝜆1 𝜆2
𝑬 = 𝟐. 𝟎𝟕𝟏 𝒆𝑽

victory R. SARAVANAN. M.Sc., M.Phil., B.Ed PG ASST [PHYSICS], GBHSS, PARANGIPETTAI - 608 502
12 PHYSICS UNIT – 8 DUAL NATURE OF RADIATION AND MATTER COMPLETE GUIDE AND MODEL QUESTION
𝜆2 − 𝜆1 11. UV light of wavelength 1800Å is incident on a lithium surface whose threshold
ℎ𝑐 [ ] = 𝐾1 − 𝐾2
𝜆1 𝜆2 wavelength is 4965Å. Determine the maximum energy of the electron emitted.
5000 − 3310 1 -Solution :- 𝜆 = 1800 𝐴° = 1800 𝑋 10−10 𝑚 ; 𝜆𝑂 = 4965 𝐴° = 4965 𝑋 10−10 𝑚
ℎ (3𝑋108 ) [ ] −10 = (3 − 0.972) 𝑋 10−19  By Einstein’s photo electric equation,
3310 𝑋 5000 10
1690 ℎ 𝜈 = 𝜙𝑂 + 𝐾𝑚𝑎𝑥
ℎ (3𝑋108 ) [ ] = 2.028 𝑋10−19
16550 𝑋 10−7 (𝑜𝑟) 𝐾𝑚𝑎𝑥 = ℎ 𝜈 − 𝜙𝑂
2.028 𝑋10−19 𝑋 16550 𝑋 10−7 (𝑜𝑟) 𝐾𝑚𝑎𝑥 = ℎ 𝜈 − ℎ 𝜈 𝑂
ℎ = ℎ 𝑐 ℎ𝑐 1 1
1690 𝑋 3𝑋108
2.028 𝑋16550 𝑋 10−34 (𝑜𝑟) 𝐾𝑚𝑎𝑥 = − =ℎ𝑐 [ − ]
𝜆 𝜆𝑂 𝜆 𝜆𝑂
ℎ =
5070 𝜆𝑂 − 𝜆
𝒉 = 𝟔. 𝟔𝟐𝟏 𝑿 𝟏𝟎−𝟑𝟒 𝑱 𝒔 𝐾𝑚𝑎𝑥 = ℎ 𝑐 [ ]
𝜆 𝜆𝑂
 From equation (2), Work function is, 4965 − 1800 1
ℎ𝑐 𝐾𝑚𝑎𝑥 = 6.626 𝑋 10−34 𝑋 3 𝑋 108 [ ] 𝑋 −10
𝜙𝑂 = − 𝐾2 4965 𝑋 1800 10
𝜆2 3165
6.621 𝑋 10−34 𝑋 3𝑋108 𝐾𝑚𝑎𝑥 = 19.878 𝑋 10−19 [ ] 𝐽
𝜙𝑂 = [ ] − [0.972 𝑋 10−19 ] 8937
−19
5000 𝑋 10−10 19.878 𝑋 10 3165
(𝑜𝑟) 𝐾𝑚𝑎𝑥 = [ ] 𝑒𝑉
19.863 𝑋 10−19 1.6 𝑋 10 −19 8937
𝜙𝑂 = [ ] − [ 0.972 𝑋 10−19 ] 198.78 3165
5 (𝑜𝑟) 𝐾𝑚𝑎𝑥 = [ ] [ ] 𝑒𝑉
𝜙𝑂 = [3.972 𝑋 10−19 ] − [0.972 𝑋 10−19 ] 16 8937
12.42 𝑋 3165
𝜙𝑂 = [3.972 − 0.972 ] 𝑋 10−19 (𝑜𝑟) 𝐾𝑚𝑎𝑥 = 𝑒𝑉
𝜙𝑂 = 3 𝑋 10−19 8937
ℎ𝑐 𝑲𝒎𝒂𝒙 = 𝟒. 𝟑𝟗𝟖 𝒆𝑽 ≈ 𝟒. 𝟒 𝒆𝑽
(𝑜𝑟) = 3 𝑋 10−19 12. Calculate the de Broglie wavelength of a proton whose kinetic energy is equal
𝜆𝑂
to 81.9 × 10–15 J. (Given: mass of proton is 1836 times that of electron).
ℎ𝑐 6.621 𝑋 10−34 𝑋 3 𝑋 108
∴ 𝜆𝑂 = = -Solution :- 𝐾𝑃 = 81.9 X 10−15 J ; 𝑚𝑃 = 1836 𝑚𝑒
3 𝑋 10−19 3 𝑋 10−19  de Broglie wavelength of proton,
𝝀𝑶 = 𝟔. 𝟔𝟐𝟏 𝑿 𝟏𝟎−𝟕 𝒎 = 𝟔𝟔𝟐𝟏 𝑿 𝟏𝟎−𝟏𝟎 𝒎 = 𝟔𝟔𝟐𝟏 𝑨°
ℎ ℎ
10. At the given point of time,the earth receives energy from sun at 4 cal cm–2 min– 𝜆𝑃 = =
1 Determine the number of photons received on the surface of the Earth per √2 𝑚𝑃 𝐾𝑃 √2 (1836 𝑚𝑒 ) 𝐾𝑃
cm2 per minute. (Given : Mean wavelength of sun light = 5500 Å ) 6.621 𝑋 10−34
-Solution :- 𝑃 = 4 𝑐𝑎𝑙 𝑐𝑚−2 𝑚𝑖𝑛 −1 = 4 𝑋 4.2 = 16.8 𝐽 𝑐𝑚−2 𝑚𝑖𝑛−1 𝜆𝑃 =
√2 𝑋 1836 𝑋 9.1 𝑋 10−31 𝑋 81.9 X 10−15
𝜆 = 5500 𝐴° = 5500 𝑋 10−10 𝑚 6.621 𝑋 10−11
 The number of photons received on the surface of the Earth per cm 2 per minute 𝜆𝑃 =
√2 𝑋 1836 𝑋 9.1 𝑋 81.9
, 𝜆𝑃 = 4. 005 𝑋 10−3 𝑋 10−11
𝑃 𝑃 𝑃 𝑃𝜆 𝝀𝑷 ≈ 𝟒 𝑿 𝟏𝟎−𝟏𝟒 𝒎
𝑛𝑃 = = = =
𝐸 ℎ𝜈 ℎ 𝑐
( ⁄𝜆) ℎ𝑐 13. A deuteron and an alpha particle are accelerated with the same potential.
16.8 𝑋 5500 𝑋 10−10 Which one of the two has (i) greater value of de Broglie wavelength associated
𝑛𝑃 = with it and (ii) less kinetic energy? Explain.
6.626 𝑋 10−34 𝑋 3 𝑋 108
924 𝑋 1018 -Solution :- 𝑚𝑁 = 1.67 𝑋 10−27 𝑘𝑔 ; 𝑒 = 1.6 𝑋 10−19 𝐶
𝑛𝑃 = = 4.648 𝑋 101 𝑋1018  For deuteron ; 𝑚𝑑 = 2 𝑚𝑁 , 𝑞𝑑 = 𝑒
19.878
𝒏𝑷 = 𝟒. 𝟔𝟒𝟖 𝑿 𝟏𝟎𝟏𝟗 For alpha particle ; 𝑚𝛼 = 4 𝑚𝑁 , 𝑞𝛼 = 2 𝑒
(i) de Broglie wavelength of deuteron,
ℎ ℎ ℎ ℎ
𝜆𝑑 = = = =
√2 𝑚𝑑 𝑞𝑑 𝑉 √2 (2 𝑚𝑁 ) 𝑒 𝑉 √4 𝑚𝑁 𝑒 𝑉 2 √𝑚𝑁 𝑒 𝑉
victory R. SARAVANAN. M.Sc., M.Phil., B.Ed PG ASST [PHYSICS], GBHSS, PARANGIPETTAI - 608 502
12 PHYSICS UNIT – 8 DUAL NATURE OF RADIATION AND MATTER COMPLETE GUIDE AND MODEL QUESTION
de Broglie wavelength of alpha particle, 15. The ratio between the de Broglie wavelength associated with proton,
ℎ ℎ ℎ ℎ accelerated through a potential of 512 V and that of alpha particle accelerated
𝜆𝛼 = = = =
√2 𝑚𝛼 𝑞𝛼 𝑉 √2 (4 𝑚𝑁 ) (2 𝑒) 𝑉 √16 𝑚𝑁 𝑒 𝑉 4 √𝑚𝑁 𝑒 𝑉 through a potential of X volts is found to be one. Find the value of X.
-Solution :- 𝑉𝑃 = 512 𝑉 ; 𝜆𝑃 ∶ 𝜆𝛼 = 1 ; 𝑉𝛼 = 𝑋

[ ] 1  For proton ; 𝑚𝑃 = 𝑚𝑁 , 𝑞𝑃 = 𝑒
𝜆𝑑 2 √𝑚𝑁 𝑒 𝑉 ( ) 4
= = 2 = =2 For alpha particle ; 𝑚𝛼 = 4 𝑚𝑁 , 𝑞𝛼 = 2 𝑒
𝜆𝛼 ℎ 1 2
[ ] ( )  de Broglie wavelength of proton,
4
4 √𝑚𝑁 𝑒 𝑉 ℎ ℎ
𝝀𝒅 = 𝟐 𝝀𝜶 𝜆𝑃 = =
(ii) de Broglie wavelength of deuteron , √2 𝑚𝑃 𝑞𝑃 𝑉𝑃 √2 𝑚𝑁 𝑒 𝑉𝑃
ℎ ℎ ℎ de Broglie wavelength of alpha particle,
𝜆𝑑 = = = ℎ ℎ ℎ
√2 𝑚𝑑 𝐾𝑑 √2 (2 𝑚𝑁 ) 𝐾𝑑 √4 𝑚𝑁 𝐾𝑑 𝜆𝛼 = = =
ℎ 2 √2 𝑚𝛼 𝑞𝛼 𝑉𝛼 √2 (4 𝑚𝑁 ) (2 𝑒) 𝑋 √16 𝑚𝑁 𝑒 𝑋
(𝑜𝑟) 𝜆𝑑2 = ℎ
4 𝑚𝑁 𝐾𝑑 [ ]
ℎ2 𝜆𝑃 √2 𝑚𝑁 𝑒 𝑉𝑃 ℎ √16 𝑚𝑁 𝑒 𝑋 8𝑋
(𝑜𝑟) 𝐾𝑑 = = = 𝑋 = √
4 𝑚𝑁 𝜆𝑑2 𝜆𝛼 ℎ √2 𝑚𝑁 𝑒 𝑉𝑃 ℎ 𝑉𝑃
[ ]
de Broglie wavelength of alpha particle, √16 𝑚𝑁 𝑒 𝑋
ℎ ℎ ℎ 𝜆𝑃 2 8𝑋
𝜆𝛼 = = = (𝑜𝑟) ( ) =
√2 𝑚𝛼 𝐾𝛼 √2 (4 𝑚𝑁 ) 𝐾𝛼 √8 𝑚𝑁 𝐾𝛼 𝜆𝛼 𝑉𝑃
ℎ 2
𝜆𝑃 2 𝑉𝑃
(𝑜𝑟) 𝜆𝛼2 = (𝑜𝑟) 𝑋 = ( )
8 𝑚𝑁 𝐾𝛼 𝜆𝛼 8
ℎ2 512
(𝑜𝑟) 𝐾𝛼 = 𝑋 = (1)2
8 𝑚𝑁 𝜆𝛼2 8
ℎ2 𝑿 = 𝟔𝟒 𝑽
[ ]
𝐾𝑑 4 𝑚𝑁 𝜆𝑑2 ℎ2 8 𝑚𝑁 𝜆𝛼2 𝜆𝛼2 𝜆𝛼 2
∴ = 2 = 2 𝑋 = 2 2 = 2 ( )
𝐾𝛼 ℎ 4 𝑚 𝑁 𝜆𝑑 ℎ2 𝜆𝑑 𝜆𝑑
[ 2 ]
8 𝑚 𝑁 𝜆𝛼
𝐾𝑑 𝜆𝛼 2 1 1
∴ = 2( ) = 2𝑋 =
𝐾𝛼 2 𝜆𝛼 4 2
𝑲𝜶
𝑲𝒅 =
𝟐
14. An electron is accelerated through a potential difference of 81V. What is the
de Broglie wavelength associated with it? To which part of electromagnetic
spectrum does this wavelength correspond?
-Solution :- 𝑉 = 81 𝑉
 de Broglie wavelength of electron,
12. 27
𝜆= 𝐴
√𝑉
12. 27 12. 27
𝜆= 𝐴 = 𝐴
√81 9
𝝀 = 𝟏. 𝟑𝟔 𝑨
 It lies in X -ray region of electromagnetic spectrum..

victory R. SARAVANAN. M.Sc., M.Phil., B.Ed PG ASST [PHYSICS], GBHSS, PARANGIPETTAI - 608 502
12 PHYSICS UNIT – 8 DUAL NATURE OF RADIATION AND MATTER COMPLETE GUIDE AND MODEL QUESTION
 Hence 𝑷 = √2 𝑫 (i.e.) de Broglie wavelength of proton is 1.414 times greater
UNIT - 8 CONCEPTUAL QUESTIONS AND ANSWERS than the de Broglie wavelength of deuteron.
1. If the momentum of electron is changed by ‘𝑷’, then the de Broglie wavelength 𝒉 𝒉 ℎ
 Also we have  = = ⟹ 𝑝=
associated with it changes 0.5%. Find the initial momentum of electron? 𝒎𝒗 𝒑 

 The de Broglie wavelength associated with the moving electron having  Hence momentum of proton ; 𝑝𝑃 =
𝒉 𝒉 𝑷
momentum ‘𝑝’ will be ;  = =  Momentum of duetron ; 𝑝𝐷 =

=

= √2

= √2 𝑝𝑃
𝒎𝒗 𝒑 
𝑫 [ 𝑷] 𝑷
 Now according to problem we have, 𝑝
√2
𝒅 𝒅𝒑 0.5 𝑃 𝟏𝟎𝟎 𝑷  Thus 𝑝𝑃 = 𝐷 = 0.707 𝑝𝐷 (i.e.) Momentum of proton is 0.707 times less than
= − (or) = ⇒ 𝑷𝒊 = = 𝟐𝟎𝟎 𝑷 √2
 𝒑 100 𝑃𝑖 𝟎.𝟓
that of the momentum of duetron.
2. Electrons are accelerated through a potential difference ‘V’ and protons are 5. A particle of mass ‘M’ at rest decays into two particles of masses 𝒎𝟏 and 𝒎𝟐
accelerated through a potential difference ‘4V’. Then find the ratio of having non-zero velocities. Find the ratio of the de Broglie wavelengths of the
de Broglie wavelengths of electron to the proton? two particles?
 The de Broglie wavelength associated with the moving electron accelerated by  The initial momentum of the pariticle at rest = 0
𝒉 𝒉
the potential difference of ‘V’ is ; 𝒆 = =  Let 𝑣1 and 𝑣2 be the velocities of the two particles after decay, then the total
𝒎𝒆 𝒗𝒆 √𝟐 𝒎𝒆 𝒆 𝑽
final momentum of the two particles = 𝑚1 𝑣1 + 𝑚2 𝑣2
 The de Broglie wavelength associated with the moving proton accelerated by the
𝒉 𝒉  According to conservation of linear momentum,
potential difference of ‘4V’ is ; 𝑷 = = Initial total linera momentum = Final total linear momentum
𝒎𝑷 𝒗𝑷 √𝟐 𝒎𝑷 𝒆 𝟒 𝑽
𝒆 𝒉 √𝟐 𝒎𝑷 𝒆 𝟒 𝑽 √𝒎 0 = 𝑚1 𝑣1 + 𝑚2 𝑣2
 Hence ;
𝑷
=
√𝟐 𝒎𝒆 𝒆 𝑽
𝑿 𝒉
= 𝟐 √𝒎 𝑷 (𝑜𝑟) 𝑚1 𝑣1 = 𝑚2 𝑣2
𝒆
𝒉 𝒉
3. Let an electron of mass ‘m’ and charge ‘e’ initially at rest gets accelerated by a  The de Broglie wavlengths of the two particles ; 𝟏 = and 𝟐 =
𝒎𝟏 𝒗𝟏 𝒎𝟐 𝒗𝟐
constant electric field ‘E’.Then find the rate of change of de Broglie wavelength 𝒉
(𝒎 𝒗 )
of this electron at time ‘t’? 𝟏
 Now the ratio of the de Broglie wavelengths ; = 𝟏𝒉 𝟏 = 1
 The force experienced by the electron ; 𝐹 = 𝑒 𝐸 𝟐 ( )
𝒎 𝒗𝟐 𝟐
𝐹 𝑒𝐸
 The acceleration ; 𝑎 = = 6. A proton has kinetic energy ‘K’ which is equal to that of a photon. The
𝑚 𝑚
 The velocity of the electron ; 𝑣 = 𝑎 𝑡 = (
𝑒𝐸
)𝑡 wavelength of proton is 𝟏 and wavelength of photon is 𝟐. Find the ratio of
𝑚
𝒉 𝒉 𝒉
wavelengths?
 Hence the de Broglie wavelength of electron ;  = = 𝑒𝐸 = 𝒆𝑬𝒕  Kinetic energy of proton ; 𝐾 =
1
𝑚 𝑣2 =
1
𝑚2 𝑣 2 =
𝑝2
𝒎𝒗 𝒎( )𝑡 2 2𝑚 2𝑚
𝑚
 Therefore the rate of change of de Broglie wavelength at time ‘t’ will be, Hence momentum of proton is ; 𝑝 = √2 𝑚 𝐾
𝒅 𝒉 𝟏 𝒉 ℎ ℎ
= 𝒆 𝑬 (− 𝒕𝟐) = − Hence its de Broglie wavelength is ; 1 = =
𝒅𝒕 𝒆 𝑬 𝒕𝟐 𝑝 √2 𝑚 𝐾
4. A proton and a deuteron are accelerated through the same accelerating  The energy of photon is ; 𝐸 = ℎ 𝜈 =
ℎ𝑐
= 𝐾 (or) 2 =
ℎ𝑐

potential ‘V’. Which one of the two has greater value of de Broglie wavelength 2 𝐾
associated with it and less momentum?  Hence the ratio of the de Broglie wavelengths of proton to that of wavelength of
 The de Broglie wavelength associated with the moving proton of mass 𝒎𝑷 1 ℎ 𝐾 1 𝐾 𝟏
photon will be ;
2
= √
2𝑚𝐾
𝑋 ℎ𝑐
=
𝑐

2𝑚
(or)
𝟐
∝ √𝑲
𝒉 𝒉
accelerated by the potential difference of ‘V’ is ; 𝑷 = = 7. If ‘𝒎𝒏 ’ is the mass of a neutron, then what is the de Broglie wavelength of a
𝒎𝑷 𝒗𝑷 √𝟐 𝒎𝑷 𝒒𝑷 𝑽
 The de Broglie wavelength associated with the moving deuteron of mass 𝒎𝑫 neutron in thermal equilibrium with heavy water at a temperature T K?
𝒉 𝒉
accelerated by the potential difference of ‘V’ is ; 𝑫 = =
𝒉 𝒉  The de Broglie wavelength of neutron is ; 𝒏 = =
𝒎𝑫 𝒗𝑫 √𝟐 𝒎𝑫 𝒒𝑫 𝑽 𝒎𝒏 𝒗𝒏 √𝟐 𝒎𝒏 (𝑲𝑬)
3
√𝟐 𝒎𝑫 𝒒𝑫 𝑽  But kinetic energy (KE) of thermal neutron is ; 𝐾𝐸 = 𝑘𝑇
 √𝟐 (𝟐 𝒎𝑷 ) 𝒆 𝑽 2
 Since 𝒎𝑫 = 𝟐𝒎𝑷 and 𝒒𝑫 = 𝒒𝑷 = 𝒆,then 𝑷 = = = √𝟐 𝒉 𝒉
𝑫 √𝟐 𝒎𝑷 𝒒𝑷 𝑽 √𝟐 𝒎𝑷 𝒆 𝑽  Hence ; 𝒏 = =
3
√𝟐 𝒎𝒏 ( 𝑘 𝑇) √𝟑𝒎𝒏 𝒌 𝑻
2

victory R. SARAVANAN. M.Sc., M.Phil., B.Ed PG ASST [PHYSICS], GBHSS, PARANGIPETTAI - 608 502
12 PHYSICS UNIT – 8 DUAL NATURE OF RADIATION AND MATTER COMPLETE GUIDE AND MODEL QUESTION
𝒐
8. Radiation of wavelength ‘’ is incident on a photo cell. The fastest emitted 11. A 5 watt source emits monochromatic light of wavelength 𝟓𝟎𝟎𝟎 𝐀 when
𝟑
photo electron has speed ‘𝐯’. If the wavelength is changed to , then find the placed 𝟎. 𝟓 𝒎 away, it liberates photoelectrons from a photosensitive metallic
𝟒
speed of the fastest photo electron emitted? surface. When the source is moved to a distance of 𝟏. 𝟎 𝒎, what is the change in
𝑐
 For  , the frequency will be 𝜈 = . According to Eienstein’s photo electric number of photoelectrons emitted?
𝜆 𝟏
1  Number of photo electrons emitted ; 𝑵𝒆 ∝ 𝒊𝒏𝒕𝒆𝒏𝒔𝒊𝒕𝒚 ∝
equation, ; 𝑚v 2 = ℎ (𝜈 − 𝜈𝑜 ) (𝒅𝒊𝒔𝒕𝒂𝒏𝒄𝒆)𝟐
2
𝟑 4𝑐 4  As the distance is doubled, the number of photo electrons emitted decreases by
 For , the frequency will be 𝜈1 = = 𝜈 . According to Eienstein’s photo 𝟏
𝟒 3𝜆 3 times
1 12 1 12 4 𝟒
electric equation, ; 𝑚v =ℎ (𝜈1 − 𝜈𝑜 ) (or) 𝑚v = ℎ ( 𝜈 − 𝜈𝑜 )
2 2 3 12. The work function of metal ‘A’ and ‘B’ are in the ratio 1 : 2. If light of
1 2 4 2 4 4 frequencies 𝝂 and 𝟐 𝝊 are incident on the surfaces of A and B respectively, then
𝑚v1 ℎ( 𝜈−𝜈𝑜 ) v1 ( 𝜈−𝜈𝑜 ) v1 ( 𝜈−𝜈𝑜 )
∴ 2
1 = 3
(or) = 3
(or) =√ 3
find the ratio of the maximum kinetic energies of photo electrons emitted?
𝑚v2 ℎ(𝜈−𝜈𝑜 ) v2 (𝜈−𝜈𝑜 ) v (𝜈−𝜈𝑜 )
2 (here 𝝂 is greater than threshold frequency of A and 𝟐 𝝂 is greater than
𝟒 𝟒 threshold frequency of B)
( 𝝂−𝝂𝒐 ) ( 𝝀𝟎 − 𝝀)
(or) 𝐯 𝟏 = 𝐯√ 𝟑
(or) 𝐯 𝟏 = 𝐯√ 𝟑  According to Einstein’s photo electric equation,
(𝝂−𝝂𝒐 ) (𝝀𝟎 − 𝝀) 𝐸𝐴 = ℎ 𝜈 − 𝝓𝐴 and 𝐸𝐵 = ℎ (2𝜈) − 𝜙𝐵
9. A particle ‘A’ of mass ‘𝒎’ and initial velocity ‘𝒗’, collides with a particle ‘B’ of  Since, 𝜙𝐴 ∶ 𝜙𝐵 = 1 ∶ 2 we have, 𝜙𝐵 = 2 𝜙𝐴 then
mass ‘𝒎/𝟐’ which is at rest.The collision is head on and elastic. What is the 𝐸𝐵 = 2 ℎ 𝜈 − 2 𝜙𝐴 = 2 (ℎ 𝜈 − 𝜙𝐴 ) = 2 𝐸𝐴
𝑬𝑨 1
ratio of the de Broglie wavelengths of particle A to B after collision?  Hence, =2 (or) 𝑬𝑨 : 𝑬𝑩 = 𝟏 ∶ 𝟐
𝑬𝑩
 By law of conservation of linear momentum,
𝑚𝐴 𝑣𝐴 + 𝑚𝐵 𝑣𝐵 = 𝑚𝐴 𝑢𝐴 + 𝑚𝐵 𝑢𝐵 13. A source of light is placed at a distance of 50 cm from a photocell and the
𝑚 𝑚 stopping potential is found to be 𝑽𝟎 . If the distance between the light source
𝑚𝑣𝐴 + 𝑣𝐵 = 𝑚 𝑣 + (0) and photo cell is made 25 cm, then what will be new stopping potential?
2 2
1
𝑣𝐴 + 𝑣𝐵 = 𝑣 (or) 2𝑣𝐴 + 𝑣𝐵 = 2 𝑣 ------------ (1)  Stopping potential is independent of distance and depends on the frequency.
2  Hence new stopping potential will remain unchanged (i.e.) 𝑽𝟎
𝑣𝐵 −𝑣𝐴 𝑣𝐵 −𝑣𝐴 14. The work function of cathode in a photo electric cell is 𝝓𝟏 Now this cathode is
 By law of collision, 𝑒 = = (or) 𝑣𝐵 − 𝑣𝐴 = 𝑒 𝑣 replaced by another cathode of work function 𝝓𝟐 such that 𝝓𝟐 > 𝝓𝟏 . Let the
𝑢𝐴 −𝑢𝐵 𝑣−0
For elastic collision, 𝑒 = 1 Hence, 𝑣𝐵 − 𝑣𝐴 = 𝑣 ------------------- (2) current before and after this change will be 𝑰𝟏 and 𝑰𝟐 . Then which current is
 Solving equation (1) and (2), we get ; 𝑣𝐴 =
𝑣
and 𝑣𝐵 =
4𝑣 greater? (Assuming 𝒉 𝝂 > 𝝓𝟐 )
3 3
ℎ ℎ 3ℎ  The work function has no effect on photoelectric current so long as the energy
 de Broglie wavlenth of particle ‘A’ after collision ; 𝜆𝐴 = = 𝑣 = of incident photon is greater than the work function. (i.e.) 𝒉 𝝂 > 𝝓𝒐
𝑚𝐴 𝑣𝐴 𝑚( ) 𝑚𝑣
3
ℎ ℎ 6ℎ  The photo electric current is proportional to the intensity of incident light. Since
de Broglie wavlenth of particle ‘B’ after collision ; 𝜆𝐵 = = 𝑚 4𝑣 = there is no change in intensity of light, hence 𝑰𝟏 = 𝑰𝟐
𝑚𝐵 𝑣𝐵 ( )( ) 4𝑚𝑣
2 3
𝝀𝑨 3ℎ 4𝑚𝑣 15. When the X -ray tube is operated at 1 kV, then X -rays of minimum wavelength
 Hence,
𝝀𝑩
= 𝑚𝑣
𝑋 6ℎ
=𝟐 𝒐
𝟔. 𝟐𝟐 𝐀 are produced. If the tube is operated at 10 kV, then what is the
10. A photon of frequency ‘𝝂’ causes the emission of a photo electron of maximum minimum wavelength of X -rays?
kinetic energy ’𝑬𝑲 ’ from a metal. If a photon of frequency ‘𝟑 𝝂’ is inident on the 𝒐
𝟏𝟐𝟒𝟎𝟎 𝐀
same metal, then find the maximum kinetic energy of the emitted photo  The minimum wavelength of X -rays will be ; 𝝀𝒎𝒊𝒏 =
𝑽
electron? 𝝀𝟐 𝑽𝟏 𝑽𝟏 𝟏𝟎𝟎𝟎 𝒐
−𝟏𝟎
 Let ‘𝝓𝒐 ’ be the work function of the metal.  Then = ∴ 𝝀𝟐 = 𝝀𝟏 = 𝟏𝟎𝟎𝟎𝟎 𝑿 𝟔. 𝟐𝟐 𝑿 𝟏𝟎 = 𝟎. 𝟔𝟐𝟐 𝐀
𝝀𝟏 𝑽𝟐 𝑽𝟐
 According to Einstein’s photo electric equation, 𝒐 𝒐

𝑬𝑲 = 𝒉 𝝂 − 𝝓𝒐 and 𝑬𝑲𝟏 = 𝒉 (𝟑𝝂) − 𝝓𝒐 16. What is the ratio of energies of X -rays of the wavelength 𝟎. 𝟎𝟏 𝐀 and . 𝟓 𝐀 ?
𝒉𝒄 𝟏
 Thus we may write, 3 𝐸𝐾 = 3 ℎ 𝜈 − 3𝝓𝒐 and 𝐸𝐾1 = 3 ℎ 𝜈 − 𝝓𝒐  The energy of X -ray photon will be ; 𝑬 = 𝒉 𝝂 = (i.e.) 𝑬∝𝝀
𝝀
 From above equations, 𝐸𝐾1 − 3 𝐸𝐾 = 2 𝝓𝒐 (or) 𝑬𝑲𝟏 = 𝟑 𝑬𝑲 + 𝟐 𝝓𝒐 𝑬𝟏 𝝀 𝟎.𝟓 𝑿 𝟏𝟎−𝟏𝟎
(or) 𝑬𝑲𝟏 > 𝟑 𝑬𝑲  Hence,
𝑬𝟐
= 𝝀𝟐 = 𝟎.𝟎𝟏 𝑿 𝟏𝟎−𝟏𝟎 = 𝟓𝟎 (or) 𝑬𝟏 ∶ 𝑬𝟐 = 𝟓𝟎 ∶ 𝟏
𝟏

victory R. SARAVANAN. M.Sc., M.Phil., B.Ed PG ASST [PHYSICS], GBHSS, PARANGIPETTAI - 608 502
12 PHYSICS UNIT – 8 DUAL NATURE OF RADIATION AND MATTER COMPLETE GUIDE AND MODEL QUESTION
17. When a metallic surface is illuminated with radiation of wavelength , the 22. The work function of metallic body is 𝟐 𝒆𝑽. When an electromagnetic radiation
stopping potential is ‘V’. If the same surface is illuminated with radiation of of frequency 𝝂 impinge on this surface, the maximum kinetic energy of
𝑽 ejected photo electron is found to be 𝟒 𝒆𝑽. What would be the maximum
wavelength 𝝀 , the stopping potential is
𝟒
. Find the threshold wavelength for
kinetic energy of photo electron for the radiation of frequency 𝟓 𝝂/𝟑 ?
the metallic surface?  According to Einstein’s photo electric equation, [𝑬𝑲 ]𝒎𝒂𝒙 = 𝒉 𝝂 − 𝝓𝒐
 According to Einstein’s photo electric equation, [𝑬𝑲 ]𝒎𝒂𝒙 = 𝒉 𝝂 − 𝝓𝒐  For 𝜈 ; 4 𝑒𝑉 = ℎ 𝜈 − 2 𝑒𝑉 (or) ℎ 𝜈 = 6 𝑒𝑉
 But, [𝑬𝑲 ]𝒎𝒂𝒙 = 𝒆 𝑽𝒐 and 𝝓𝒐 = 𝒉 𝝂𝟎 . 5𝜈 5
𝒉𝒄 𝒉𝒄  For 5 𝜈/3 ; [𝑬𝑲 ]𝒎𝒂𝒙 = ℎ [ ] − 2 𝑒𝑉 = (6 𝑒𝑉) − 2 𝑒𝑉 = 𝟖 𝒆𝑽
 Therefore, 𝒆 𝑽𝒐 = 𝒉 𝝂 − 𝒉 𝝂𝟎 = − 3 3
𝝀 𝝀𝟎 23. The maximum kinetic energy of photo electrons emitted from a surface, when
Here 𝑽𝒐 is the stopping potential 𝝂𝟎 is the threshold frequency and 𝝀𝟎 is the photons of energy 6 eV falls on it is 4 eV. What would be the stopping
threshold wavelength. potential?
𝒉𝒄 𝒉𝒄 𝟏 𝟏 𝝀 −𝝀
 Thus, 𝒆𝑽= − = 𝒉𝒄[ − ] = 𝒉𝒄 [ 𝟎 ] and  Stopping potential is the negative potential applied to the anode, to stop the
𝝀 𝝀𝟎 𝝀 𝝀𝟎 𝝀 𝝀𝟎
𝑽 𝒉𝒄 𝒉𝒄 𝟏 𝟏 𝝀 −𝟐𝝀
emission of photo electrons having maximum kinetic energy.
𝒆 (𝟒 ) = 𝟐 𝝀 − 𝝀 = 𝒉 𝒄 [𝟐𝝀 − 𝝀 ] = 𝒉 𝒄 [ 𝟐𝟎𝝀 𝝀 ]  Since the maximum kinetic energy is 4 eV, the sotpping potential will be 4 V
𝟎 𝟎 𝟎
𝒆𝑽 𝝀𝟎 − 𝝀 𝟏 𝟐 𝝀 𝝀𝟎 𝟐 [𝝀𝟎 − 𝝀] 𝟐 𝝀𝟎 − 𝟐 𝝀 24. A photo electric surface is illuminated successively by monochromatic light of
∴ 𝑽 = 𝒉 𝒄 [ 𝝀 𝝀 ] 𝑿 𝒉 𝒄 [𝝀 −𝟐𝝀] (or) 4 = = 𝝀
𝒆( )
𝟒
𝟎 𝟎 𝝀 𝟎 −𝟐𝝀 𝝀𝟎 −𝟐 𝝀 wavelengths 𝝀 and . If the maximum kinetic energy of the emitted photo
𝟐
(𝒐𝒓) 4 𝜆0 − 8 𝜆 = 2 𝜆0 − 2 𝜆 (or) 2 𝜆0 = 6 𝜆 (or) 𝝀𝟎 = 𝟑 𝝀 electrons in the second case is three times that in the first case. Find the work
18. The work function for sodium surface is 2.0 eV and that of aluminium surface function of the surface?
is 4.2 eV. The two metal surfaces are illuminated with appropriate radiation  For 1st case ; 𝑬𝑲𝟏 = 𝒉 𝝂𝟏 − 𝝓𝒐 (or) 𝑬𝑲𝟏 = − 𝝓𝒐
𝒉𝒄
so as to cause photo emission. which surface has least threshold frequency? 𝝀
𝒉𝒄
 Work fuction is given by ; 𝝓𝒐 = 𝒉 𝝂𝟎 (𝜈0 → 𝑡ℎ𝑟𝑒𝑠ℎ𝑜𝑙𝑑 𝑓𝑟𝑒𝑞𝑢𝑒𝑛𝑐𝑦)  For 2 nd case ; 𝑬𝑲𝟐 = 𝒉 𝝂𝟐 − 𝝓𝒐 (or) 𝑬𝑲𝟐 = − 𝝓𝒐
𝝀/𝟐
 Thus greater the work function, greater is the threshold frequency. Therefore ℎ𝑐 ℎ𝑐 2ℎ𝑐 3ℎ𝑐
the threshold frequency of sodium will be lesser than that for aluminium
 Given, 𝐸𝐾2 = 3 𝐸𝐾1 (or)
𝜆/2
− 𝜙𝑜 = 3 [ 𝜆 − 𝜙𝑜 ] (or) 𝜆
− 𝜙𝑜 = 𝜆
− 3 𝜙𝑜
19. A metal surface is illuminated by a light of given intensity and frequency to ℎ𝑐 𝒉𝒄
(or) 2 𝜙𝑜 = (or) 𝝓𝒐 =
cause photo emission. If the intensity of illumination is reduced to one-fourth 𝜆 𝟐𝝀
of its original value. How the value of maximum kinetic energy of emitted 25. Discuss the statement that “photo electric effect establishes the quantum
photo electrons changed? nature of electromagnetic radiation”
 The maximum kinetic energy of emitted photo electrons depends only the  Photon which is considered to be the elementary particle of light contains a
frequency of incident light and independ of the intensity of the light. quantum of energy ‘𝒉 𝝂’ . On incidence, this whole of the energy is acquired by
20. The surface of a metal is illuminated with a light of 𝟒𝟎𝟎 𝒏𝒎. The kinetic energy the electron which consequently, is ejected.
of the ejected photo electrons was found to be 𝟏. 𝟔𝟖 𝒆𝑽. What is the work  Had the radiation been in the form of a continuous wave, it would not have been
function of the metal? possible to eject the electron.
 Keep in mind ; ℎ 𝑐 = 1240 𝑒𝑉 𝑛𝑚 26. Would it be more convenient to demonstrate phot-electric effect with ultra-
 According to Einstein’s photo electric equation, [𝑬𝑲 ]𝒎𝒂𝒙 = 𝒉 𝝂 − 𝝓𝒐 violet light or visible light? Sunstantiate your answer.
𝒉𝒄  Ultra violet light is of shorter wavelength and hence of greater frequency as
 There fore work function ; 𝝓𝒐 = 𝒉 𝝂 − [𝑬𝑲 ]𝒎𝒂𝒙 = − [𝑬𝑲 ]𝒎𝒂𝒙
𝝀 compared to visible light. Therefore a photon of ultra violet light contains
𝟏𝟐𝟒𝟎 𝒆𝑽 𝒏𝒎
𝝓𝒐 = − 𝟏. 𝟔𝟖 𝒆𝑽 = 𝟑. 𝟏 − 𝟏. 𝟔𝟖 = 𝟏. 𝟒𝟐 𝒆𝑽 greater amount of energy. Thus it is convenient to demonstrate the
𝟒𝟎𝟎 𝒏𝒎
21. The threshold frequency of a metal is 𝝂𝟎 . If radiation of frequency 𝟐 𝝂𝟎 falls phenomenon of photo electric effect with ultra violet light.
on that metal, what will the maximum possible velocity of the emitted photo 27. If E and p are the energy and momentum of a photon, calculate velocity of light?
𝒉𝒄
electron?  The energy of a photon is ; 𝑬 = 𝒉 𝝂 =
𝝀
 According to Einstein’s photo electric equation, [𝑬𝑲 ]𝒎𝒂𝒙 = 𝒉 𝝂 − 𝝓𝒐 𝒉
1 2 2 2 ℎ 𝜈𝑜  The de Broglie wavelength is ; 𝝀 =
(or) 𝑚𝑣𝑚𝑎𝑥 = ℎ 𝜈 − ℎ 𝜈𝑜 = ℎ(2 𝜈𝑜 ) − ℎ 𝜈𝑜 = ℎ 𝜈𝑜 (or) 𝑣𝑚𝑎𝑥 = 𝒑
2 𝑚 𝒉𝒄
 From the above two equations, 𝑬 = =𝒑𝒄
𝟐 𝒉 𝝂𝒐 𝒉/𝒑
(or) 𝒗𝒎𝒂𝒙 = √ 𝒎
(𝒎 → 𝒎𝒂𝒔𝒔 𝒐𝒇 𝒑𝒉𝒐𝒕𝒐 𝒆𝒍𝒆𝒄𝒕𝒓𝒐𝒏) 𝑬
 So the velocity of light ; 𝒄 =
𝒑
victory R. SARAVANAN. M.Sc., M.Phil., B.Ed PG ASST [PHYSICS], GBHSS, PARANGIPETTAI - 608 502
12 PHYSICS UNIT – 8 DUAL NATURE OF RADIATION AND MATTER COMPLETE GUIDE AND MODEL QUESTION
28. Alkali metals are most suitable for photo electric emission. Why? 36. Why a fast neutron beam needs to be thermalized with the environment,
 Because the work function of alkali metals are low. They emit photo electrons before it can be used for neutron diffraction experiments?
even when visible light falls on them.  For diffraction to takes place, the wavelength of the particle used should be
29. It is difficult to eject a free electron from copper than from sodium. Why? comparable with interatomic distance.
 Copper has greater work function than sodium.  The wavelength of fast neutrons is very small.
 For copper work function is 4.65 eV and that of sodium is 2.75 eV  But when they are slowed down their energy is reduced and they become
30. Is photo electric emission possible at all frequencies? Give reason. thermal neutron. Their wavelength becomes comparable with the interatomic
 No. Because for each metal, there is minimum frequency of the incident spacing. So fast neutrons are thermalized.
radiation below which there is no photo electric emission. That minimum 37. Distinguish between electromagnetic waves and matter waves.
frequency is called threshold frequency. Electromagnetic waves Matter waves
31. Every metal has a definite work function. Why not photo electrons comes out 1) Electromagnetic waves are 1) The waves associated with matter
all with the same energy, if incident radiation is monochromatic? Why is there produced by accelerating in motion is called matter waves.
an energy distribution of photo electrons? chanrges They are probability waves.
 All the free electrons in a metal are not of the same energy. Work function tells 2) In vacuum, electromagnetic waves 2) The velocity of matter waves of
us about the minimum energy that must be supplied to an electron in the of different wavelengths all have different wavelength are different
highest level of the conduction band, to come out ; other electrons in different the same velocity
energy levels will come out with different energies. 38. A proton and an electron have same de Broglie wavelength. Which possesses
32. When light from a lamp falls on a wooden table, no photoelectrons are emitted more kinetic energy?
from it. Why? 𝒉 𝒉
 The de Broglie wavelength is ;  = = (ℎ → 𝑃𝑙𝑎𝑛𝑘 ′ 𝑠 𝑐𝑜𝑛𝑠𝑡𝑎𝑛𝑡)
 Because the work function of wood (the minimum energy required to eject the 𝒎𝒗 √𝟐 𝒎 𝑬
electron from it) is greater than that of the energy of the incident photon. 𝒉𝟐
 Thus kinetic energy may be written as, 𝑬 =
33. No photo electric emission takes place if the frequency of incident radiation is 𝟐 𝒎 𝝀𝟐
𝟏
less than the threshold frequency. Why?  Given that  is same for both proton and electron. So 𝑬 ∝
𝒎
 By Einstein’s photo electric equation, [𝑬𝑲 ]𝒎𝒂𝒙 = 𝒉 𝝂 − 𝒉 𝝂𝒐 = 𝒉 [𝝂 − 𝝂𝒐 ]  Since mass of proton is 1836 times greater than that of electron, the kinetic
 When frequency (𝜈) is less than the threshold frequency (𝝂𝒐 ), the maximum energy of proton is less than the kinetic energy of electron. So electron have
kinetic energy will become negative which is impossible. So photo electric more kinetic energy.
emission will not take place. 39. How will you control the intensity and quality of X -rays?
34. Why cannot we experience the existence of matter waves in our day-to-day  The intensity of X -rays depends upon the number of eletrons striking the target
life? and depends upon the filament temperature, which can be controlled by
𝒉 𝒉
 The de Broglie wavelength associated with the moving particle ;  = = adjusting the filament current.
𝒎𝒗 𝒑
 The quality of X -rays depend upon the penetrating power and can be controlled
 If ‘m’ is large,  will be small. Thus the wavelength will be very very small, by adjusting the anode potential (operating voltage)
compared to the size of the particle. Hence it is not observable. 40. Which are called “bremsstrahlung” and why?
35. An electron and a proton are prossessing same amount of kinetic energy.  The continuous X -rays are called bremsstrahlung or braking radiation.
Which of the two has greater de Broglie wavelength? Justify your answer.  Because they are produced by the slowing or braking down of the incident
𝒉 𝒉
 The de Broglie wavelength ;  = = charged particles (i.e.) decelerated electrons
𝒎𝒗 √𝟐 𝒎 𝑬
𝟏 41. Define one electron volt (eV).
 Since kinetic energy (E) is same for both electron and proton,  ∝  1 eV is defined as the kinetic energy gained by an electron when accelerated by
√𝒎
 Hence wavelength () will be greater for smaller mass (m). a potential difference of e1 V
 The mass of electron is 1836 times less than that of proton and so de Broglie  1 𝑒 𝑉 = 1.602 𝑋 10−19 𝐽
wavelength is greater for an electron than proton.

victory R. SARAVANAN. M.Sc., M.Phil., B.Ed PG ASST [PHYSICS], GBHSS, PARANGIPETTAI - 608 502
12 PHYSICS UNIT – 8 DUAL NATURE OF RADIATION AND MATTER COMPLETE GUIDE AND MODEL QUESTION
EXAM NO 7. In photoelectric emission, a radiation whose frequency is 4 times threshold
NAME : frequency of a certain metal is incident on the metal. Then the maximum
UNIT - 8 DUAL NATURE OF RADIATION AND MATTER possible velocity of the emitted electron will be
ℎ𝜈𝑜 6 ℎ𝜈𝑜 ℎ 𝜈𝑜
Time - 2 : 30 hours Total - 60 marks (a) √ (b) √ (c) 2√
ℎ 𝜈𝑜
(d) √
𝑚 𝑚 𝑚 2𝑚
PART - I 15 X 1 = 15 8. Two radiations with photon energies 0.9 eV and 3.3 eV respectively are falling
Note : (i) Answer all the questions on a metallic surface successively. If the work function of the metal is 0.6 eV,
(ii) Choose the best answer and write the option code and then the ratio of maximum speeds of emitted electrons in the two cases will be
corresponding answer (a) 1:4 (b) 1:3
1. The wavelength λe of an electron and λp of a photon of same energy E are (c) 1:1 (d) 1:9
related by 9. A light source of wavelength 520 nm emits 1.04 × 1015 photons per second
(a) 𝜆𝑝 ∝ 𝜆𝑒 (b) 𝜆𝑝 ∝ √𝜆𝑒 while the second source of 460 nm produces 1.38 × 1015 photons per second.
1 Then the ratio of power of second source to that of first source is
(c) 𝜆𝑝 ∝ (d) 𝜆𝑝 ∝ 𝜆𝑒2
√𝜆𝑒 (a) 1.00 (b) 1.02
2. In an electron microscope, the electrons are accelerated by a voltage of 14 kV. (c) 1.5 (d) 0.98
If the voltage is changed to 224 kV, then the de Broglie wavelength associated 10. If the mean wavelength of light from sun is taken as 550 nm and its mean
with the electrons would power as 3.8 × 1026 W, then the average number of photons received by the
(a) increase by 2 times (b) decrease by 2 times human eye per second from sunlight is of the order of
(c) decrease by 4 times (d) increase by 4 times (a) 1045 (b) 1042
3. The wave associated with a moving particle of mass 3 X 10–6 g has the same (c) 1054 (d) 1051
wavelength as an electron moving with a velocity 6 X 10 6 m s-1 .The velocity of 11. The threshold wavelength for a metal surface whose photoelectric work
the particle is function is 3.313 eV is
(a) 1.82 X 10-18 m s-1 (b) 9 X 10-2 m s-1 (a) 4125 Å (b) 3750 Å
(c) 3 X 10 m s
-31 -1 (d) 1.82 X 10-15 m s-1 (c) 6000 Å (d) 20625.Å
4. When a metallic surface is illuminated with radiation of wavelength λ, the 12. A light of wavelength 500 nm is incident on a sensitive metal plate of
stopping potential is V. If the same surface is illuminated with radiation of photoelectric work function 1.235 eV. The kinetic energy of the photo
𝑽 electrons emitted is (Take h = 6.6 × 10–34 Js)
wavelength 2λ, the stopping potential is . The threshold wavelength for the
𝟒 (a) 0.58 eV (b) 2.48 eV
metallic surface is (c) 1.24 eV (d) 1.16 eV
(a) 4 λ (b) 5 λ
5 13. Photons of wavelength λ are incident on a metal. The most energetic electrons
(c) λ (d) 3 λ ejected from the metal are bent into a circular arc of radius R by a
2
5. If a light of wavelength 330 nm is incident on a metal with work function perpendicular magnetic field having magnitude B. The work function of the
3.55 eV, the electrons are emitted. Then the wavelength of the wave associated metal is
with the emitted electron is (Take h = 6.6 × 10–34 Js) ℎ𝑐 𝑒 2 𝐵2 𝑅2 ℎ𝑐 𝑒𝐵𝑅 2
(a) < 2.75 𝑋 10−9 𝑚 (b) ≥ 2.75 𝑋 10−9 𝑚
(a)
𝜆
− 𝑚𝑒 + 2 𝑚𝑒
(b)
𝜆
+ 2 𝑚𝑒 [2 𝑚 ]
𝑒
(c) ≤ 2.75 𝑋 10 −12
𝑚 (d) < 2.5 𝑋 10−10 𝑚 ℎ𝑐 𝑒 2 𝐵2 𝑅2 ℎ𝑐 𝑒𝐵𝑅 2
6. A photoelectric surface is illuminated successively by monochromatic light of
(c)
𝜆
− 𝑚𝑒 𝑐 2 − 2 𝑚 (d)
𝜆
− 2 𝑚𝑒 [2 𝑚 ]
𝑒 𝑒
𝛌 14. The work functions for metals A, B and C are 1.92 eV, 2.0 eV and 5.0 eV
wavelength 𝛌 and . If the maximum kinetic energy of the emitted
𝟐 respectively. The metal/metals which will emit photoelectrons for a radiation
photoelectrons in the second case is 3 times that in the first case, the work of wavelength 4100Å is/are
function of the material is (a) A only (b) both A and B
ℎ𝑐 2ℎ𝑐
(a) (b) (c) all these metals (d) none
λ λ
(c)
ℎ𝑐
(d)
ℎ𝑐 15. Emission of electrons by the absorption of heat energy is called
3λ 2λ ……………emission.
(a) photoelectric (b) field (c) thermionic (d) secondary
victory R. SARAVANAN. M.Sc., M.Phil., B.Ed PG ASST [PHYSICS], GBHSS, PARANGIPETTAI - 608 502
12 PHYSICS UNIT – 8 DUAL NATURE OF RADIATION AND MATTER COMPLETE GUIDE AND MODEL QUESTION
PART - II 6 X 2 = 12
Note : (i) Answer any 6 of the following questions .
(ii) Question No. 23 is compulsory
16. Define work function of a metal. Give its unit
17. Define stopping potential.
18. Define threshold frequency.
19. What is called matter waves? அச்சம் தவிர் - பயம் க ொள்ளொதத
20. A proton and an electron have same kinetic energy. Which one has greater de Broglie
wavelength. Justify?
ஆண்மை தவதேல் - ைனவலிமை இழக் ொதத
21. What are called X – rays? Whyu are they so called?
இமளத்தல் இ ழ்ச்சி - பின்வொங்குதல் இ ழ்வதற்று உரியது
22. What is Bremsstralung? ஈம த் திேன் - பிேர்க்கு க ொடுத்தமை ைனதில் க ொள்
23. An electron is accelerated through a potential difference of 81V. What is the de உடலிமன உறுதிகசய் - உடம்மப திடைொ மவத்துக்க ொள்
Broglie wavelength associated with it? To which part of electromagnetic spectrum ஊண் மி விரும்பு - உணவு உண்ண விருப்பம் க ொள்
does this wavelength correspond? எண்ணுவது உயர்வு - எண்ணம் உயர்வொ க ொள்
PART - III 6 X 3 = 18 ஏறுதபொல் நட - நிமிர்ந்து நில்
Note : (i) Answer any 6 of the following questions . ஐம்கபொறி ஆட்சிக ொள் - ஐம்புைமனயும் அடக்கி ஆள்
(ii) Question No. 30 is compulsory ஒற்றுமை வலியைொம் - ஓற்றுமைதய வலிமையொனது
24. State the laws of photo electric effect. ஓய்தல் ஒழி - தசொர்வு மள நீக்கு
25. How does photo electric current vary with the intensity of the incident light?
26. Derive the expression for de Broglie wavelength. ஓளடதம் குமே - ைருந்மத குமே
27. Write a note on characteristic X –ray spectra. ற்ேகதொழுகு - ற்ேமத பின்பற்று
28. An electron and an alpha particle have same kinetic energy. How are the deBroglie ம த்கதொழில் தபொற்று - ம த்கதொழிமை விரும்பிச் கசய்
wavelength associated with them related? தீதயொர்க்கு அஞ்தசல் - தீயவர் ளிடம் அச்சம் க ொள்ளொதத
29. Explain the application of X – rays. ததொல்வியில் ைங்த ல் - ததொல்வியில் ைக் ம் க ொள்ளொதத
30. When light of wavelength 2200Å falls on Cu, photo electrons are emitted from it. Find
(i) the threshold wavelength and (ii) the stopping potential. Given: the work function தநர்பட தபசு - சுற்றி வமளத்து தபசொதத
for Cu is ϕ0 = 4.65 eV. மநயப்புமட - தீயவற்மே அடித்து கநொறுக்கு
PART - IV 3 X 5 = 15 கபரிதினும் கபரிதுத ள் - இைக்ம கபரியதொ க ொள்
Note : (i) Answer all the questions கரௌத்திரம் பழகு - நல்ைவற்றிக் ொ த ொபப்படு
31. Give the construction and working of photo emissive cell.
(OR)
Explain the effect of potential difference on photo electric current புதிய ஆத்திசூடி - ை ொ வி சுப்பிரைண்ய பொரதியொர்
32. Briefly explain the principle and working of electron microscope.
(OR)
Explain how frequency of incident light varies with stopping potential.
33. Obtain Einstein’s photoelectric equation with necessary explanations.
(OR)
Describe briefly Davisson – Germer experiment which demonstrated the wave
nature of electrons.

victory R. SARAVANAN. M.Sc., M.Phil., B.Ed PG ASST [PHYSICS], GBHSS, PARANGIPETTAI - 608 502
பசித்திரு (Be hungry) தனித்திரு (Be individual) விழித்திரு (Be conscious)

HIGHER SECONDARY SECOND YEAR-PHYSICS

NAME :
STANDARD : 12 SECTION :
SCHOOL :
EXAM NO :

victory R. SARAVANAN. M.Sc, M.Phil, B.Ed.,


PG ASST (PHYSICS)
GBHSS, PARANGIPETTAI - 608 502
12 PHYSICS UNIT –9 ATOMIC PHYSICS AND NUCLEAR PHYSICS COMPLETE GUIDE AND MODEL QUESTION

PART – I 1 MARK MULTIPLE CHOICE QUESTIONS & ANSWERS 6. In J.J. Thomson e/m experiment, a beam of electron is replaced by that of
1. Suppose an alpha particle accelerated by a potential of V volt is allowed to muons (particle with same charge as that of electrons but mass 208 times that
collide with a nucleus of atomic number Z, then the distance of closest ofelectrons). No deflection condition is achieved only if
approach of alpha particle to the nucleus is (a) B is increased by 208 times (b) B is decreased by 208 times
𝑍 𝑉 𝑍 𝑉 (c) B is increased by 14.4 times (d) B is decreased by 14.4 times
(a) 14.4 Å (b) 14.4 Å (c) 1.44 Å (d) 1.44 Å
𝑉 𝑍 𝑉 𝑍 Solution :-
Solution :- 𝑒 2𝑦𝐸
1 2 𝑍 𝑒2 1 2 𝑍 𝑒2 1 𝑍𝑒  By J.J. Thomson e/m experiment, specific charge of electron ;
𝑚
= 𝐶 𝑙 2 𝐵2
 Distance of closest approach ; 𝑟𝑜 = = =
4𝜋𝜀𝑜 𝐸𝐾 4𝜋𝜀𝑜 (2 𝑒) 𝑉 4𝜋𝜀𝑜 𝑉 2𝑦𝐸𝑚 2𝑦𝐸𝑚
𝑍 𝑍 𝑍  Hence the magnetic field ; 𝐵2 = (or) 𝐵=√
𝑟𝑜 = 9 𝑋 109 𝑋 1.6 𝑋 10−19 𝑋 𝑉
= 14.4 𝑋 10 −10
𝑋 𝑉 = 14.4 𝑉 Å 𝐶𝑙 𝑒
2 2
𝐶𝑙 𝑒
𝒁 2 𝑦 𝐸 (208 𝑚)
Answer (a) 𝟏𝟒. 𝟒 Å  Simillarly for muons ; 𝐵𝑚𝑢𝑜𝑛𝑠 = √
𝑽 𝐶 𝑙2 𝑒
2. In a hydrogen atom, the electron revolving in the fourth orbit, has angular 𝐵𝑚𝑢𝑜𝑛𝑠
 Hence the ratio ;
𝐵
= √208 = 14.4 (or) 𝐵𝑚𝑢𝑜𝑛𝑠 = 14.4 𝐵
momentum equal to
ℎ 4ℎ 2ℎ Answer (c) B is increased by 14.4 times
(a) h (b) (c) (d)
𝜋 𝜋 𝜋 7. The ratio of the wavelengths for the transition from n =2 to n = 1 in Li++, He+
Solution :- and H is
 According to Bohr’s postulate ; (a) 1 : 2: 3 (b) 1: 4: 9 (c) 3 : 2 : 1 (d) 4: 9: 36
𝑛ℎ 4ℎ 2ℎ
angular momentum = = = Solution :-
2𝜋 2𝜋 𝜋 1 1 1
𝟐𝒉  Wave number of spectral line, ̅𝜈 = = 𝑍 2 𝑅 [𝑛 2 − ]
Answer (d) 𝝅
𝜆 𝑚2
1 1 1 1 3
3. Atomic number of H-like atom with ionization potential 122.4 V for n = 1 is  Hence, ̅𝜈 = = 𝑍2𝑅 [ 2 − ] 2
= 𝑍 𝑅 [1 − ] = 𝑍2𝑅 [ ]
𝜆 1 22 4 4
(a) 1 (b) 2 (c) 3 (d) 4  Therefore, 𝜆 = (
4
)
1
(or) 𝜆∝
1
Solution :- 3𝑅 𝑍2 𝑍2
13.6 𝑍2  Here for lithium (Li++) Z =3 ; for helium (He+) Z =2 ; for hydrogen (H) Z =1
 Ionisation potential ; 𝑉𝑖𝑜𝑛𝑖𝑠𝑎𝑡𝑖𝑜𝑛 = 𝑣𝑜𝑙𝑡 1 1 4 9 36
𝑛2  Thus, 𝜆𝐿𝑖++ ∶ 𝜆𝐻𝑒+ ∶ 𝜆𝐻 = ∶ ∶ 1 = ∶ ∶ = 4 ∶ 9 ∶ 36
𝑉𝑖𝑜𝑛𝑖𝑠𝑎𝑡𝑖𝑜𝑛 𝑛2 122.4 𝑋 12 9 4 36 36 36
 Hence, 𝑍 2 = = =9 (or) 𝑍=3 Answer (d) 𝟒 ∶ 𝟗 ∶ 𝟑𝟔
13.6 13.6
Answer (c) 𝟑 8. The electric potential between a proton and an electron is given by 𝑽=
𝒓
4. The ratio between the fi rst three orbits of hydrogen atom is 𝑽𝒐 𝐥𝐧 [ ], where r0 is a constant. Assume that Bohr atom model is applicable to
𝒓𝒐
(a) 1:2:3 (b) 2:4:6 (c) 1:4:9 (d) 1:3:5
potential, then variation of radius of nth orbit rn with the principal quantum
Solution :-
number n is
 Radius of nth orbit hydrogen atom ; 𝑟𝑛 = 𝑎𝑜 𝑛2 (or) 𝑟𝑛 ∝ 𝑛2 1 1
 Hence, 𝑟1 ∶ 𝑟2 ∶ 𝑟3 = 1 ∶ 4 ∶ 9 (a) 𝑟𝑛 ∝ (b) 𝑟𝑛 ∝ 𝑛 (c) 𝑟𝑛 ∝ (d) 𝑟𝑛 ∝ 𝑛2
𝑛 𝑛2
Answer (c) 𝟏∶𝟒∶ 𝟗 Solution :-
𝑟 𝜕𝑈 𝑒 𝑉𝑜
5. The charge of cathode rays is  The potential energy ; 𝑈 = 𝑒 𝑉 = 𝑒 𝑉𝑜 ln [ ] ; ∴ 𝐹= − =
𝑟𝑜 𝜕𝑟 𝑟
(a) positive (b) negative
 This attractive force gives necessary centripetal force and hence,
(c) neutral (d) not defined 𝑒 𝑉𝑜 𝑚 𝑣2 𝑒 𝑉𝑜
Solution :- = (or) 𝑣2 =
𝑟 𝑟 𝑚
 Cathode rays are deflected both by electric and magnetic field. The direction of 𝑛ℎ
 From Bohr’s postulate ; 𝑚 𝑣 𝑟𝑛 = = 𝑛ℏ
deflection indicates that cathode rays (electron beam) are negatively charged. 2𝜋
𝑚 𝑣 𝑟𝑛 𝑛ℏ 𝑚 𝑟𝑛 𝑛ℏ𝑚 𝑛ℏ𝑣
Answer (b) negative  Thus,
𝑣2
= 𝑒 𝑉𝑜
[ ]
(or)
𝑣
= 𝑒𝑉
𝑜
(or) 𝑟𝑛 =
𝑒 𝑉𝑜
(or) 𝑟𝑛 ∝ 𝑛
𝑚
Answer (b) 𝒓𝒏 ∝ 𝒏
victory R. SARAVANAN. M.Sc., M.Phil., B.Ed PG ASST [PHYSICS], GBHSS, PARANGIPETTAI - 608 502
12 PHYSICS UNIT –9 ATOMIC PHYSICS AND NUCLEAR PHYSICS COMPLETE GUIDE AND MODEL QUESTION
9. If the nuclear radius of 27Al
is 3.6 fermi, the approximate nuclear radius of 64Cu is 13. A radioactive nucleus (initial mass number A and atomic number Z emits
(a) 2.4 2α and 2 positrons. The ratio number of neutrons to that of proton in the final
(b) 1.2 nucleus will be
(c) 4.8 𝐴−𝑍−4 𝐴−𝑍−2
(a) (b)
(d)3.6 𝑍−2 𝑍−6
𝐴−𝑍−4 𝐴−𝑍−12
Solution :- (c)
𝑍−6
(d) 𝑍−4
1/3 1/3
 Nuclear radius ; 𝑅𝐶𝑢 = 𝑅𝑜 𝐴𝐶𝑢 and 𝑅𝐴𝑙 = 𝑅𝑜 𝐴𝐴𝑙 . Hence Solution :-
1/3 1/3
𝑅𝐶𝑢 𝑅𝑜 𝐴𝐶𝑢 𝐴𝐶𝑢 64 1/3 4 4 4  Let ‘z’ be the atomic number and ‘a’ be the mass number of the final nucleus then
= = = = (or) 𝑅𝐶𝑢 = 𝑋𝑅𝐴𝑙 = 𝑋 3.6 = 4.8 𝐹
𝑅𝐴𝑙 1/3
𝑅𝑜 𝐴𝐴𝑙
1/3
𝐴𝐴𝑙 271/3 3 3 3 ; 𝐴 𝑎
𝑍𝑋 ⟶ 𝑧𝑌 + 2 𝛼 + 2 𝑒
+
(or) 𝐴 𝑎 4
𝑍𝑋 ⟶ 𝑧𝑌 + 2 2𝐻𝑒 + 2 1𝑒
0

Answer (c) 𝟒. 𝟖 𝑭  By conservation of charge and mass, we have


10. The nucleus is approximately spherical in shape. Then the surface area of 𝑍 =𝑧+4+2 =𝑧+6 ⟹ 𝑧 =𝑍−6
nucleus having mass number A varies as 𝐴 = 𝑎+8+0= 𝑎+8 ⟹ 𝑎 =𝐴−8
(a) A2/3  Hence number of neutrons in final nucleus ; 𝑛 = 𝑎 − 𝑧 = 𝐴 − 𝑍 − 2
𝑛 𝐴−𝑍−2
(b) A4/3  So for the final nucleus, the ratio ; =
𝑧 𝑍−6
(c) A1/3
𝑨 − 𝒁 − 𝟐
(d) A5/3 Answer (b)
Solution :- 𝒁−𝟔
 If R be the radius of nucleus, then, its surface area 14. The half-life period of a radioactive element A is same as the mean life time of
2
S.A = 4 𝜋 𝑅2 = 4 𝜋 (𝑅𝑜 𝐴1/3 ) = 4 𝜋 𝑅𝑜2 𝐴2/3 (or) S.A ∝ 𝐴2/3 another radioactive element B. Initially both have the same number of atoms.
Then
Answer (a) A2/3 (a) A and B have the same decay rate initially
11. The mass of a 𝟕𝟑𝑳𝒊 nucleus is 0.042 u less than the sum of the masses of all its (b) A and B decay at the same rate always
nucleons. The binding energy per nucleon of 𝟕𝟑𝑳𝒊 nucleus is nearly (c) B will decay at faster rate than A
(a) 46 MeV (d) A will decay at faster rate than B.
(b) 5.6 MeV Solution :-
(c) 3.9MeV 0.6931 1
 Half life of nucleus A ; 𝑇𝐴 = and mean life of nucleus B ; 𝜏𝐵 =
(d)23 MeV 𝜆𝐴 𝜆𝐵
0.6931 1
Solution :-  Given that, 𝑇𝐴 = 𝜏𝐵 (or) = 𝜆 (or) 𝜆𝐴 = 0.6931 𝜆𝐵
𝜆𝐴
 Here mass defect ; ∆𝑚 = 0.042 𝑢 𝐵
 Since 𝜆𝐴 > 𝜆𝐵 , nucleus B will decay at faster rate than A
 So binding energy ; 𝐵𝐸 = ∆𝑚 𝑋 931 𝑀𝑒𝑉 = 0.042 𝑋 931 𝑀𝑒𝑉 = 39.1𝑀𝑒𝑉
 Then binding energy per nucleon ;
𝐵𝐸
=
39.1
= 5.58 ≈ 5.6 𝑀𝑒𝑉 Answer (c) B will decay at faster rate than A
𝐴 7
15. A radiative element has No number of nuclei at t = 0. The number of nuclei
Answer (b) 𝟓. 𝟔 𝑴𝒆𝑽 𝟏
remaining after half of a half-life (that is, at time 𝒕 = 𝑻𝟏/𝟐 )
12. Mp denotes the mass of the protonand Mn denotes mass of a neutron. A given 𝟐
𝑁 𝑁
nucleus of binding energy B, contains Z protons and N neutrons. The mass (a) 𝑜 (b) 𝑜
2 √2
M (N,Z) of the nucleus is given by (where c is the speed of light) 𝑁𝑜 𝑁𝑜
(a) M (N, Z) = N Mn + Z Mp − B c2 (c) (d)
4 8
(b) M (N, Z) = N Mn + Z Mp + B c 2 Solution :-
1
(c) M (N, Z) = N Mn + Z Mp − B/ c2 𝑡 𝑇1/2 1
(d) M (N, Z) = N Mn + Z Mp + B/ c2  Number of half life periods ; 𝑛 = = 2𝑇 = 2
𝑇1/2 1/2
Solution :- 1
2 𝐵 1 𝑛 1 2 𝑁
 Binding energy ; 𝐵 = ∆𝑚 𝑐 (or) ∆𝑚 = 2
𝑐
 So the number of nuclei remaining ; 𝑁 = 𝑁𝑜 ( ) = 𝑁𝑜 ( ) = 𝑜
2 2 √2
𝐵 𝐵
(or) 𝑁 𝑀𝑛 + 𝑍 𝑀𝑝 − 𝑀 (𝑁, 𝑍) = (or) 𝑀 (𝑁, 𝑍) = 𝑁 𝑀𝑛 + 𝑍 𝑀𝑝 − 𝑵𝒐
𝑐2 𝑐2 Answer (b) √𝟐
Answer (c) M (N, Z) = N Mn + Z Mp − B/ c2
victory R. SARAVANAN. M.Sc., M.Phil., B.Ed PG ASST [PHYSICS], GBHSS, PARANGIPETTAI - 608 502
12 PHYSICS UNIT –9 ATOMIC PHYSICS AND NUCLEAR PHYSICS COMPLETE GUIDE AND MODEL QUESTION
6. What are the conclusion made by Rutherford from the results of alpha
PART – II & III 2 AND 3 MARK SHORT ANSWER QUESTIONS & ANSWERS scattering experiments.
1. What are called cathode rays? Conclusion made in alpha scattering experiment : (Rutherford atom model)
 When the pressure is about 0.01 mm of Hg, positive column disappears and a  Rutherford proposed that an atom has a lot of empty space and contains a tiny
dark space is formed between anode and cathode which is called Crooke’s dark matter known as nucleus whose size is of the order of 10-14 m.
space.  The nucleus is positively charged and most of the mass of the atom is
 At this time the walls of the tube appear with green colour and some invisible concentrated in nucleus.
rays emanate from cathode called cathode rays, which are later found be a beam  The nucleus is surrounded by negatively charged electrons.
of electrons.  Since static charge distribution cannot be in a stable equilibrium, he suggested
2. Give the properties of cathode rays. that the electrons are not at rest and they revolve around the nucleus in circular
Properties of cathode rays : orbits like planets revolving around the sun.
 Cathode rays possess energy and momentum 7. What is distance of closest approach? Obtain expression for it.
 They travel in a straight line with high speed of the order of 107m s-1. Definition :
 It can be deflected by both electric and magnetic fields.  The minimum distance between the centre of the nucleus and the alpha particle
 The direction of deflection indicates that they are negatively charged particles. just before it gets reflected back through 180° is defined as the distance of closest
 When the cathode rays are allowed to fall on matter, they produce heat. approach (or) contact distance 𝑟𝑜
 They affect the photographic plates
 They produce fluorescence
 When the cathode rays fall on a material of high atomic weight, x-rays are
produced. Expression :
 Cathode rays ionize the gas through which they pass.  At this closest distance, all the kinetic energy of the alpha particle will be
1 converted into electrostatic potential energy
 The speed of cathode rays is up to ( ) th of the speed of light.
10 1 1 (2𝑒)(𝑍𝑒)
3. Define specific charge. 𝑚 𝑣𝑜 2 =
2 4 𝜋 𝜀𝑜 𝑟𝑜
 Charge per unit mass is called specific charge (or) mass-normalized charge.
1 2 𝑍 𝑒2
 Its unit is 𝑪 𝒌𝒈−𝟏 𝑟𝑜 =
4. Write a note on Thomson’s atom model. 4 𝜋 𝜀𝑜 ( 1 𝑚 𝑣 2 )
2 𝑜
Thomson’s atom model : 𝟏 𝟐 𝒁 𝒆𝟐
 According to J. J. Thomson’s atom model, the atoms are considered as 𝒓𝒐 = − − − − (1)
homogeneous spheres which contain uniform distribution of positively charged 𝟒 𝝅 𝜺𝒐 𝑬𝑲
particles.  where 𝑬𝑲 → kinetic energy of alpha particle
 The negatively charged electrons are embedded in it like seeds in water melon 8. Define impact parameter.
fruit.  The impact parameter (b) is defined as the perpendicular distance between the
 The atoms are electrically neutral, this implies that the total positive charge in an centre of the gold nucleus and the direction of velocity vector of alpha particle
atom is equal to the total negative charge. when it is at a large distance.
 This model could not expalain stability of atom and further, it fails to explain the 9. What are the drawbacks of Rutherford atom model?
origin of spectral lines observed in the spectrum of hydrogen atom and other (1) Stability of atom cannot be explained :
atoms.  According to classical electrodynamics, any accelerated charge emits
5. Give the results of Rutherford alpha scattering experiment. electromagnetic radiations which results loses in its energy.
Results of alpha scattering experiment :  Hence, it can no longer sustain the circular motion and the radius of the orbit
 Most of the alpha particles are undeflected through the gold foil and went straight. becomes smaller and smaller (undergoes spiral motion) and finally the
 Some of the alpha particles are deflected through a small angle. electron should fall into the nucleus and hence the atoms should
 A few alpha particles (one in thousand) are deflected through the angle more disintegrate.
than 90°  But this does not happen. Hence, Rutherford model could not account for the
 Very few alpha particles returned back (back scattered) –that is, deflected back stability of atoms.
by 180°
victory R. SARAVANAN. M.Sc., M.Phil., B.Ed PG ASST [PHYSICS], GBHSS, PARANGIPETTAI - 608 502
12 PHYSICS UNIT –9 ATOMIC PHYSICS AND NUCLEAR PHYSICS COMPLETE GUIDE AND MODEL QUESTION
(2) Line spectrum of atom could not explained : 15. What are the drawbacks in Bohr atom model?
 According to this model, emission of radiation must be continuous and must Drawbacks of Bohr atom model :
give continuous emission spectrum.  Bohr atom model is valid only for hydrogen atom or hydrogen like-atoms but not
 But experimentally we observe only line (discrete) emission spectrum for for complex atoms
atoms.  When the spectral lines are closely examined, individual lines of hydrogen
10. State the postulates of Bohr’s atom model. spectrum is accompanied by a number of faint lines. These closed packed lines
Postulate (1) : are called fine structure. This is not explained by Bohr atom model.
 The electron in an atom moves around nucleus in circular orbits under the  Bohr atom model fails to explain the intensity variations in the spectral lines.
influence of Coulomb electrostatic force of attraction.  The distribution of electrons in atoms is not completely explained by Bohr atom
 This Coulomb force gives necessary centripetal force for the electron to undergo model.
circular motion. 16. Define atomic number and mass number.
Postulate (2) :  The number of protons in the nucleus is called the atomic number (Z)
 Electrons in an atom revolve around the nucleus only in certain discrete orbits  The number of neutrons in the nucleus is called neutron number (N).
called stationary orbits where it does not radiate electromagnetic energy.  The total number of neutrons and protons in the nucleus is called the mass
 The angular momentum (𝑙) of the electron in these stationary orbits are number(A) [A = Z+N]
ℎ 17. Write a general notation of nucleus of element X. What each term denotes?
quantized (i.e.) integral multiple of
2𝜋  The general notation of nucleus is ; 𝑨𝒁𝑿
𝒉
𝒍=𝒏 =𝒏ℏ  where X is the chemical symbol of the element, A is the mass number and Z is
𝟐𝝅 the atomic number. (e.g.) Nitrogen nucleus is represented by 𝟏𝟓𝟕𝑵
where n  principal quantum number
18. What is the charge of nucleus?
 This condition is known as angular momentum quantization condition.
 The nucleus is made up of positively charged protons and electrically neutral
Postulate (3) :
neutrons.
 Energy of orbits are not continuous but discrete. This is called the quantization
 So the net charge of the nucleus is positive and it has the value = + Z e
of energy.
19. What is isotope? Give an example.
 An electron can jump from one orbit to another orbit by absorbing or emitting a
 The atoms of the same element having same atomic number Z, but different mass
photon whose energy is equal to the difference in energy (ΔE) between the two
number A are called isotopes.
orbital levels
𝒄  The isotopes of any element have same electronic structure and same chemical
∆𝑬 = 𝑬𝒇 − 𝑬𝒊 = 𝒉 𝝂 = 𝒉 properties. (e.g) Hydrogen isotopes : 𝟏𝟏𝑯 , 𝟐𝟏𝑯 , 𝟑𝟏𝑯
𝝀
where c  speed of light 20. What is isobars? Give an example.
λ  wavelength of the radiation used and  The atoms of different elements having the same mass number A, but different
𝝂  frequency of the radiation atomic number Z are called isobars.
11. Define excitation energy.  They have different physical and chemical properties
 The energy required to excite an electron from lower energy state to any higher (e.g) 𝟒𝟎 𝟒𝟎 𝟒𝟎 𝟒𝟎 𝟒𝟎
𝟏𝟔 𝑺, 𝟏𝟕 𝑪𝒍, 𝟏𝟖 𝑨𝒓, 𝟏𝟗 𝑲 and 𝟐𝟎 𝑪𝒂
energy state is known as excitation energy. 21. What is isotones? Give an example.
 Its unit is electron volt (eV)  The atoms of different elements having same number of neutrons are called
12. Define excitation potential. isotones. (e.g.) 𝟏𝟐𝟓 𝑩 and 𝟏𝟑𝟔 𝑪
 Excitation potential is defined as excitation energy per unit charge. 22. Define atomic mass unit.
 Its unit is volt (V)  One atomic mass unit (u) is defined as the 1/12th of the mass of the isotope of
13. Define ionization energy. carbon ( 𝟏𝟐𝟔 𝑪)
 The minimum energy required to remove an electron from an atom in the  𝟏 𝒖 = 𝟏. 𝟔𝟔 𝑿 𝟏𝟎−𝟏𝟗 𝒌𝒈
ground state is known as binding energy or ionization energy. 23. Give the empirical formula for nuclear radius.
 Ionization energy of hydrogen atom is 𝑬𝒊𝒐𝒏𝒊𝒛𝒂𝒔𝒊𝒐𝒏 = 𝟏𝟑. 𝟔 𝒆𝑽  The nuclear radius is given by,
14. Define ionization potential. 𝟏
 Ionization potential is defined as ionization energy per unit charge. 𝑹 = 𝑹 𝑶 𝑨 𝟑

 The ionization potential of hydrogen atom is, 𝑽𝒊𝒐𝒏𝒊𝒛𝒂𝒔𝒊𝒐𝒏 = 𝟏𝟑. 𝟔 𝑽 where 𝑅𝑂 = 1.2 𝐹 [1 𝐹 = 10−15 𝑚]

victory R. SARAVANAN. M.Sc., M.Phil., B.Ed PG ASST [PHYSICS], GBHSS, PARANGIPETTAI - 608 502
12 PHYSICS UNIT –9 ATOMIC PHYSICS AND NUCLEAR PHYSICS COMPLETE GUIDE AND MODEL QUESTION
24. Difine nuclear density. 32. Give the symbolic representation of alpha decay, beta decay and gamma decay.
 Nuclear density is defined as the ratio of mass of the nucleus to its volume. (1) Alpha decay :
𝒎  When unstable nuclei decay by emitting an α-particle ( 42𝐻𝑒 ), its atomic
𝝆= = 𝟐. 𝟑 𝑿 𝟏𝟎𝟏𝟕 𝒌𝒈 𝒎−𝟑
𝟒 number (Z) decreases by 2, the mass number (A) decreases by 4.
𝝅 𝑹𝑶𝟑
𝟑  The α- decay process symbolically written as
25. What is mass defect? 𝐴 𝐴−4 4
𝑍𝑋 ⟶ 𝑍−2𝑌 + 2𝐻𝑒
 The experimental mass of a nucleus is less than the total mass of its individual 238 234 4
constituents. (e.g.) 92𝑈 ⟶ 90𝑇ℎ + 2𝐻𝑒

 The mass difference between total mass of the nucleons and the real mass of the (2) Beta decay :
nucleus is called mass defect (Δm)  In 𝜷− - decay, the atomic number of the nucleus increases by one but mass
number remains the same.
∆𝒎 = (𝒁 𝒎𝒑 + 𝑵 𝒎𝒏 ) − 𝑴 𝐴 𝐴 0
𝑍𝑋 ⟶ 𝑍+1𝑌 + −1𝑒 + 𝜈̅
26. Define binding energy. 14 14 0
 When Z protons and N neutrons are combine to form a nucleus, the mass (e.g.) 6𝐶 ⟶ 7𝑁 + −1𝑒 + 𝜈̅

diappear equivalent to mass defect (Δm) is converted in to energy which is used  In 𝜷+ - decay, the atomic number of the nucleus decreases by one but mass
to bind the nucleons in the nucleus. This is known as binding energy (BE) number remains the same.
𝐴 𝐴 0
𝑩𝑬 = ∆𝒎 𝒄𝟐 = [(𝒁 𝒎𝒑 + 𝑵 𝒎𝒏 ) − 𝑴 ] 𝒄𝟐 𝑍𝑋 ⟶ 𝑍−1𝑌 + 1𝑒 + 𝜈
22 22 0
27. Calculate the energy equivalent to one atomic mass unit (1 u). Give the answer (e.g.) 11𝑁𝑎 ⟶ 10𝑁𝑒 + 1𝑒 + 𝜈

in eV unit. (3) Gamma decay :


 According to Eienstein’s mass - energy relation  In α and β decay, the daughter nucleus is in the excited state most of the
𝐸 = 𝑚 𝑐 2 = (1 𝑢 ) 𝑋 (3 𝑋 108 )2 time.
𝐸 = 1.66 𝑋 10−27 𝑋 9 𝑋 1016  So this excited state nucleus immediately returns to the ground state or
𝐸 = 14.94 𝑋 10−11 𝐽 lower energy state by emitting highly energetic photons called γ rays.
 But we have, 1 𝑒𝑉 = 1.602 𝑋 10−19 𝐽  During gamma decay there is no change in atomic number and mass number
𝐴 ∗ 𝐴
∴ 𝑬 = 𝟗𝟑𝟏 𝑴𝒆𝑽 𝑍 𝑿 ⟶ 𝑍𝑋 + gamma rays (𝛾)
12 12 ∗ 0
28. Define average binding energy per nucleon? (e.g) 5𝐵 ⟶ 6𝐶 + −1𝑒 + 𝜈̅
 The average binding energy per nucleon is the energy required to separate single 12 ∗ 12
6𝐶 ⟶ 6𝐶 + 𝛾
nucleon from the particular nucleus. (̅̅̅̅̅
𝐵𝐸 ). 33. Define disintegration energy.
 It measures the stability of the nucleus.  In decay process, the total mass of the daughter nucleus and product nucleus is
29. What is nuclear force? always less than that of the parent nucleus. The difference in mass ( Δm) is
 It was concluded that there must be a strong attractive force between protons to released as energy called disintegration energy Q
overcome the repulsive Coulomb’s force. This strong attractive force which holds  If Q > 0, the decay is spontaneous (natural radioactivity) If Q < 0, the decay
the nucleus together is called nuclear force. process cannot occur spontaneously and energy must be supplied to induce the
30. Give the properties of nuclear forces? decay.
Properties of Nuclear forces : 34. In alpha decay, why the unstable nucleus emits 𝟒𝟐𝑯𝒆 nucleus? Why it does not
 The strong nuclear force is of very short range, acting only up to a distance of a emit four separate nucleons?
few Fermi.  For example, if 238 234
92𝑈 nucleus decays into 90𝑇ℎ by emitting four separate
 Nuclear force is the strongest force in nature. nucleons (two protons and two neutrons), then the disintegration energy Q for
 The strong nuclear force is attractive and acts with an equal strength between this process turns out to be negative.
proton-proton, proton-neutron, and neutron – neutron.  It implies that the total mass of products is greater than that of parent ( 238
92𝑈 )
 Strong nuclear force does not act on the electrons. So it does not alter the nucleus.
chemical properties of the atom.  This kind of process cannot occur in nature because it would violate
31. Define radioactivity. conservation of energy.
 The phenomenon of spontaneous emission of highly penetrating radiations such 35. Write a note on positron?
as α, β and γ rays by an element is called radioactivity and the substances which  The positron is an anti-particle of an electron whose mass is same as that of
emit these radiations are called radioactive elements. electron and charge is opposite to that of electron (i.e.) +e.

victory R. SARAVANAN. M.Sc., M.Phil., B.Ed PG ASST [PHYSICS], GBHSS, PARANGIPETTAI - 608 502
12 PHYSICS UNIT –9 ATOMIC PHYSICS AND NUCLEAR PHYSICS COMPLETE GUIDE AND MODEL QUESTION
36. State the properties of neutrino. 44. List the properties of neutrons.
Properties of neutrino : Properties of neutrons :
 It has zero charge  Mass of the neutron is little greater than the mass of the proton and had no
 It has an antiparticle called anti-neutrino. charge.
 Recent experiments showed that the neutrino has very tiny mass.  Neutrons are stable inside the nucleus. But freeneutron has half life of
 It interacts very weakly with the matter. Therefore, it is very difficult to detect. 13 minutes. Then it decays with emission of proton, electron and anti neutrino.
37. State the law of radioactive decay.  Neutrons are classified according to their kinetic energy as
 At any instant t, the number of decays per unit time, called rate of decay is (1) slow neutrons (0 to 1000 eV)
proportional to the number of nuclei ( N ) at the same instant. (2) fast neutrons (0.5 MeV to 10 MeV).
38. Define activity. Give its unit.  The neutrons with average energy of about 0.025 eV in thermal equilibrium are
 Activity or decay rate which is the number of nuclei decayed per second and it is called thermal neutron.
denoted as R 45. What is meant by nuclear fission?
𝒅𝑵  The process of breaking up of the nucleus of a heavier atom into two smaller
𝑹=
𝒅𝒕 nuclei with the release of a large amount of energy is called nuclear fission.
 Its unit is becquerel (Bq) and curie (Ci) 46. Calculate the energy released per fission.
39. Define one bequerel. Energy released in one fission:
 one Becquerel (Bq) is equal to one decay per second. 𝟏 𝑩𝒒 = 𝟏 𝒅𝒆𝒄𝒂𝒚/𝒔𝒆𝒄𝒐𝒏𝒅  Consider the following fission reaction.
40. Define one curie. 𝟐𝟑𝟓 𝟏 𝟏𝟒𝟏 𝟗𝟐
𝟗𝟐𝑼 + 𝟎𝒏 ⟶ 𝟓𝟔𝑩𝒂 + 𝟑𝟔𝑲𝒓 + 𝟑 𝟎𝒏 + 𝑸
𝟏

 one curie was defined as number of decays per second in 1 g of radium  Total mass before fission ;
𝟏 𝑪𝒊 = 𝟑. 𝟕 𝑿 𝟏𝟎𝟏𝟎 𝒅𝒆𝒄𝒂𝒚/𝒔𝒆𝒄𝒐𝒏𝒅 𝑚𝑎𝑠𝑠 𝑜𝑓 𝟐𝟑𝟓 𝟗𝟐𝑼 = 235.045733 𝑢
41. What is half life of nucleus. Give the expression. 𝑚𝑎𝑠𝑠 𝑜𝑓 𝟏𝟎𝒏 = 1.008665 𝑢
 The half life (𝑻𝟏/𝟐 ) is the time required for the number of atoms initially present
to reduce to one half of the initial amount . = 236.054398 𝑢
𝟎. 𝟔𝟗𝟑𝟏  Total mass after fision ;
𝑻𝟏 = 𝑚𝑎𝑠𝑠 𝑜𝑓 𝟏𝟒𝟏
𝟓𝟔𝑩𝒂 = 140.9177 𝑢
𝟐 𝝀
42. What is mean life of nucleus? Give the expression. 𝑚𝑎𝑠𝑠 𝑜𝑓 𝟗𝟐
𝟑𝟔 𝑲𝒓 = 91.8854 𝑢
𝟏
 The mean life time (𝝉) of the nucleus is the ratio of sum or integration of life 𝑚𝑎𝑠𝑠 𝑜𝑓 𝟑 𝟎𝒏 = 3.025995 𝑢
times of all nuclei to the total number nuclei present initially. = 235.829095 𝑢
𝟏
𝝉= mass defect ; ∆𝑚 = 236.054398 𝑢
𝝀 (−) 235.829095 𝑢
43. Write a note on the discovery of neutron.
Discovery of neutron : = 0.225303 𝑢
 In 1930, Bothe and Becker found that when beryllium was bombarded with α
 Then energy released during this fission reaction,
particles, highly penetrating radiation was emitted.
𝑄 = ∆𝑚 𝑋 931 𝑀𝑒𝑉
 This radiation was capable of penetrating the thick layer of lead and was
𝑄 = 0.225303 𝑋 931 𝑀𝑒𝑉
unaffected by the electric and magnetic fields.
𝑸 = 𝟐𝟎𝟎 𝑴𝒆𝑽
 Initially, it was thought wrongly as γ - radiation.
47. What is called chain reaction. Give its types.
 But in 1932, James Chadwick discovered that those radiations are particles of
 During every fission reaction, three neutrons are released along with products.
mass little greater than the mass of the proton and had no charge.
 These three neutrons cause further fission produces nine neutrons and this
 He called them as neutrons ( 𝟏𝟎𝒏). The above reaction was written as
𝟗 𝟒 𝟏𝟐 𝟏 process goes on.
𝟒𝑩𝒆 + 𝟐𝑯𝒆 ⟶ 𝟔𝑪 + 𝟎𝒏  Thus the number of neutrons goes on increasing almost in geometric progression
and this is called a chain reaction
 There are two kinds of chain reactions:
(1) Uncontrolled chain reaction
(2) Controlled chain reaction.
victory R. SARAVANAN. M.Sc., M.Phil., B.Ed PG ASST [PHYSICS], GBHSS, PARANGIPETTAI - 608 502
12 PHYSICS UNIT –9 ATOMIC PHYSICS AND NUCLEAR PHYSICS COMPLETE GUIDE AND MODEL QUESTION
48. What is called nuclear reactor? 54. What is radio carbon dating?
 Nuclear reactor is a system in which the nuclear fission takes place in a self-  Radioactive dating or carbon dating is the technique to estimate the age of
sustained controlled manner. ancient object by using radio carbon isotope ( 𝟏𝟒𝟔𝑪)
 The energy produced is used either for research purpose or for power 55. Write a note on smoke detector.
generation. Smoke detecter :
 The first nuclear reactor was built in the year 1942 at Chicago, USA  An important application of alpha decay is smoke detector which prevent us from
49. What is nuclear fusion? any hazardous fire.
 When two or more light nuclei (A<20) combine to form a heavier nucleus, then it  It uses around 0.2 mg of man-made weak radioactive isotope called americium
is called nuclear fusion. ( 𝟐𝟒𝟏
𝟗𝟓𝑨𝒎)
50. What is mean by thermo nuclear reactions?  This radioactive source is placed between two oppositely charged metal plates
 When two light nuclei come closer to combine, it is strongly repelled by the and α radiations from 𝟐𝟒𝟏𝟗𝟓𝑨𝒎 continuously ionize the nitrogen, oxygen molecules
coulomb repulsive force in the air space between the plates.
 To overcome this repulsion, the two light nuclei must have enough kinetic energy  As a result, there will be a continuous flow of small steady current in the circuit.
to move closer to each other such that the nuclear force becomes effective.  If smoke enters, the radiation is being absorbed by the smoke particles rather
 This can be achieved if the temperature is very much greater than the value 107 than air molecules.
K.  As a result, the ionization and along with it the current is reduced. This drop in
 When the surrounding temperature reaches around 107 K, lighter nuclei start current is detected by the circuit and alarm starts.
fusing to form heavier nuclei and this resulting reaction is called thermonuclear  The radiation dosage emitted by americium is very much less than safe level, so
fusion reaction. it can be considered harmless.
51. What is the source of stellar energy?
 The energy generation in every star is only through thermonuclear fusion
because its temperature is of the order of 107 K
 Most of the stars including our Sun fuse hydrogen into helium and some stars
even fuse helium into heavier elements.
52. Write a note on proton - proton cycle.
 The sun’s interior temperature is around 1.5 𝑋 107 𝐾 .
 At this temperature, fusion reaction takes place and the sun is converting
6 𝑋 1011 𝑘𝑔 hydrogen into helium every second.
 According to Hans Bethe, the sun is powered by proton-proton cycle of fusion
reaction.
 This cycle consists of three steps :
Step - 1 : 𝟏𝟏𝑯 + 𝟏𝟏𝑯 ⟶ 𝟐𝟏𝑯 + 𝟎𝟏𝒆 + 𝝂
𝟏 𝟐 𝟑
Step - 2 : 𝟏𝑯 + 𝟏𝑯 ⟶ 𝟐𝑯𝒆 + 𝜸
𝟑 𝟑 𝟒 𝟏
Step - 3 : 𝟐𝑯𝒆 + 𝟐𝑯𝒆 ⟶ 𝟐𝑯𝒆 + 𝟏𝑯 + 𝟏𝟏𝑯
 In general, the above three steps can be written as.
𝟒 𝟏𝟏𝑯 ⟶ 𝟒𝟐𝑯𝒆 + 𝟐 𝟏𝟏𝑯 + 𝟐 𝟎𝟏𝒆 + 𝟐 𝝂 + 𝟐𝟕 𝑴𝒆𝑽
53. What are the constituent particles of neutron and proton?
 Protons and neutrons are made up of quarks which are now considered as
elementary particles
 According to quark model,
2 1
(1) Proton is made up of two up quarks (+ 𝑒) and one down quark (− 𝑒) and
3 3
2 1
(2) Neutron is made up of one up quark (+ 𝑒) and two down quarks (− 𝑒)
3 3

victory R. SARAVANAN. M.Sc., M.Phil., B.Ed PG ASST [PHYSICS], GBHSS, PARANGIPETTAI - 608 502
12 PHYSICS UNIT –9 ATOMIC PHYSICS AND NUCLEAR PHYSICS COMPLETE GUIDE AND MODEL QUESTION
Method (2) - Deflection due to electric field :
PART – IV 5 MARK LONG ANSWER QUESTIONS & ANSWERS  When the magnetic field is turned off (𝐵 = 0),
1. Explain the J.J. Thomson experiment to determine the specific charge of the deflection is only due to electric field.
electron.  Let ‘m’ be the mass of the electron, the upward
Specific charge of elctron - J J Thomson Experiment acceleration due to electric field ‘E’ is
 Charge per unit mass of an electron is called specific charge (e/m) 𝐹𝐸 𝑒𝐸
𝑎𝐸 = =
Principle : 𝑚 𝑚
 Cathode rays (electron beam) deflects by both electric and magnetic fields is the  Upward initial velocity ; 𝑢 = 0
principle involved in this method.  Let ‘𝑙’ be the length of the deflecting plate, then time taken to travel in electric
Set up : field is,
 It is ahighly evacuated discharge 𝑙
𝑡=
tube 𝑣
 Cathode rays (electron beam)  Hence the deflection at the end of the electric field
produced at cathode ‘C’ are attracted 1 1
𝑦  = 𝑢 𝑡 + 𝑎 𝑡 2 = 0 + 𝑎𝐸 𝑡 2
towards anode disc A which allow 2 2
only a narrow beam of cathode rays. 
1 𝑒𝐸 𝑙 2 1 𝑒 𝐸 𝑙2
𝑦 = ( ) =
 These cathode rays are now allowed 2 𝑚 𝑣 2 𝑚 𝑣2
2 2
to pass through the parallel plates 1 𝑒 𝐸 𝑙 𝐵
𝑦 =
and strike the screen coated with 2 𝑚 𝐸2
ZnS, a light spot is observed at O 𝟏 𝒆 𝒍𝟐 𝑩𝟐
 The metal plates are maintained at high voltage. 𝒚 = − − − − (𝟐)
𝟐𝒎 𝑬
 Further, this gas discharge tube is kept in between pole pieces of magnet such  Then the deflection on the screen, 𝑦 ∝ 𝑦  (𝑜𝑟) 𝑦 = 𝐶 𝑦
that both electric and magnetic fields are perpendicular to each other.  𝐶  Proportionality constant . Using equation (2),
Velocity of cathode rays (𝒗) 1 𝑒 𝑙 2 𝐵2
 Let ‘e’ be the charge of cathode ray particle. 𝑦=𝐶
2𝑚 𝐸
 The upward force acting on cathode rays due to 𝒆 𝟐𝒚𝑬
electric field ‘E’ is; 𝑭𝑬 = 𝒆 𝑬 = − − − − − (𝟑)
𝒎 𝑪 𝒍𝟐 𝑩𝟐
 The downward force acting on cathode rays due  By substituting the known values, we get
to magnetic field is ; 𝑭𝑩 = 𝒆 𝑩 𝒗 𝒆
 In undeflected equilibrium position, = 𝟏. 𝟕 𝑿 𝟏𝟎𝟏𝟏 𝑪 𝒌𝒈−𝟏
𝒎
𝐹𝐸 = 𝐹𝐵 Method (3) - Deflection due to magnetic field :
𝑒𝐸 = 𝑒𝐵𝑣  When the electric field is turned off (𝐸 = 0), the deflection is only due to
𝑬 magnetic field.
𝒗 = − − − − − −(𝟏)
𝑩  The magnetic force provides the centripetal force, the electron beam undergoes
Method (1) - To find specific charge : semi-circular path . Hence.
 Let ‘V’ be the potential difference between anode and cathode. 𝑚 𝑣2
 Since the cathode rays (electron beam) are accelerated from cathode to anode, 𝑒𝑣𝐵=
𝑅
the potential energy ‘eV’ of the electron beam at the cathode is converted into 𝐸
kinetic energy of the electron beam at the anode. Hence, 𝑚𝑣 𝑚 ( ) 𝑚𝐸
𝑒𝐵= = 𝐵 =
1 𝑅 𝑅 𝐵𝑅
𝑒𝑉 = 𝑚 𝑣 2 𝒆 𝑬
2
𝒆 𝟏 𝒗𝟐 𝟏 𝑬𝟐 = 𝟐 − − − − − (𝟒)
𝒎 𝑩 𝑹
∴ = =  The specific charge is independent of
𝒎 𝟐𝑽 𝟐 𝑽 𝑩𝟐
 The value of specific charge is , (1) Gas used
𝒆 (2) Nature of the electrodes
= 𝟏. 𝟕 𝑿 𝟏𝟎𝟏𝟏 𝑪 𝒌𝒈−𝟏
𝒎
victory R. SARAVANAN. M.Sc., M.Phil., B.Ed PG ASST [PHYSICS], GBHSS, PARANGIPETTAI - 608 502
12 PHYSICS UNIT –9 ATOMIC PHYSICS AND NUCLEAR PHYSICS COMPLETE GUIDE AND MODEL QUESTION
2. Discuss the Millikan’s oil drop experiment to determine the charge of an  From the free body diagram,
electron. 𝐹𝑔 = 𝐹𝑏 + 𝐹𝑣
Millikan’s oil drop method : 4 4
 It consists of two horizontal 𝜌 [ 𝜋 𝑟 ] 𝑔 = 𝜎 [ 𝜋 𝑟3] 𝑔 + 6 𝜋 𝑟 𝜂 𝑣
3
3 3
circular metal plates A and B each 4 3
4 3
with diameter around 20 cm and 𝜌 [ 𝜋 𝑟 ]𝑔 − 𝜎 [ 𝜋 𝑟 ]𝑔 = 6 𝜋 𝑟 𝜂 𝑣
3 3
are separated by a small distance 4 3 (𝜌
𝜋𝑟 − 𝜎 )𝑔 = 6 𝜋 𝑟 𝜂 𝑣
1.5 cm. 3
 These two parallel plates are 𝑟3 18 𝜋𝜂𝑣
=
enclosed in a chamber with glass 𝑟 4 𝜋 (𝜌 − 𝜎 )𝑔
walls. 9𝜂𝑣
 A high potential difference around 𝑟2 =
2 − 𝜎 )𝑔
(𝜌
10 kV applied across the metal 𝟏
plates, such that electric field acts 𝟗𝜼𝒗 𝟐
𝒓 = [ ] − −(𝟏)
vertically downward. 𝟐 (𝝆 − 𝝈 )𝒈
 A small hole is made at the centre of the upper plate A and atomizer is kept (2) Determination of electric charge :
exactly above the hole to spray the liquid.  When the electric field is switched on, charged oil
 When a fine droplet of highly viscous liquid (like glycerine) is sprayed using drops experience an upward electric force (qE).
atomizer, it falls freely downward through the hole of the top plate only under  Strength of the electric field is adjusted to make
the influence of gravity. that particular drop to be stationary.
 Few oil drops in the chamber can acquire electric charge (negative charge)  Under these circumstances, there will be no viscous
because of friction with air or passage of x-rays in between the parallel plates. force acting on the oil drop.
 Further the chamber is illuminated by light which is passed horizontally and oil  From the free body diagram,
drops can be seen clearly using microscope placed perpendicular to the light 𝐹𝑔 = 𝐹𝑏 + 𝐹𝐸
beam. 4 4
𝜌 [ 𝜋 𝑟3] 𝑔 = 𝜎 [ 𝜋 𝑟3] 𝑔 + 𝑞 𝐸
 These drops can move either upwards or downward. 3 3
(1) Radius of oil drop : 4 4
(𝑜𝑟) 𝑞 𝐸 = 𝜌 [ 𝜋 𝑟3] 𝑔 − 𝜎 [ 𝜋 𝑟3] 𝑔
 When the electric field is switched off, the oil drop accelerates downwards. 3 3
 Due to the presence of air drag forces, the oil drops easily attain its terminal 4 3 (𝜌
𝑞𝐸= 𝜋𝑟 − 𝜎 )𝑔
velocity and moves with constant velocity. Let it be ‘𝑣’ 3
 Radius of the oil drop =𝑟 4
𝑞= 𝜋 𝑟 3 (𝜌 − 𝜎 )𝑔
Density of the the oil =𝜌 3𝐸
Density of the air =𝜎  Put equation (1), we get
1
 The downward gravitational force acting on the oil 4 9𝜂𝑣 9𝜂𝑣 2
drop is. 𝑞= 𝜋 [ ][ ] (𝜌 − 𝜎 )𝑔
3𝐸 2 (𝜌 − 𝜎 )𝑔 2 (𝜌 − 𝜎 )𝑔
4 1
𝐹𝑔 = 𝑚 𝑔 = 𝜌 𝑉 𝑔 = 𝜌 [ 𝜋 𝑟 3 ] 𝑔 18 𝜂𝑣 2
3 𝑞= 𝜋 [𝜂 𝑣] [ ] (𝜌 − 𝜎 )𝑔
 The upthrust force experienced by the oil drop due 𝐸 2 (𝜌 − 𝜎 )𝑔
𝟏
to displaced air is
4 𝟏𝟖 𝜼𝟑 𝒗 𝟑 𝟐

𝐹𝑏 = 𝑚 𝑔 = 𝜎 𝑉 𝑔 = 𝜎 [ 𝜋 𝑟 3 ] 𝑔 𝒒= 𝝅[ ] − − − −(𝟐)
3 𝑬 𝟐 (𝝆 − 𝝈 )𝒈
 Once the oil drop attains a terminal velocity υ, the net downward force acting on  Millikan repeated this experiment several times and computed the charges on oil
the oil drop is equal to the viscous force acting opposite to the direction of drops.
motion of the oil drop.  He found that the charge of any oil drop can be written as integral multiple of a
 From Stokes law, the viscous force on the oil drop is ; 𝐹𝑣 = 6 𝜋 𝑟 𝜂 𝑣 basic value, (−1.6 X 10−19 C), which is nothing but the charge of an electron.
Hence, 𝒆 = − 𝟏. 𝟔 𝑿 𝟏𝟎−𝟗 𝑪
victory R. SARAVANAN. M.Sc., M.Phil., B.Ed PG ASST [PHYSICS], GBHSS, PARANGIPETTAI - 608 502
12 PHYSICS UNIT –9 ATOMIC PHYSICS AND NUCLEAR PHYSICS COMPLETE GUIDE AND MODEL QUESTION
3. Derive the expression for radius and energy of the n th orbit of hydrogen atom  For hydrogen, (Z = 1), So radius of 𝑛 orbit, 𝒓𝒏 = 𝒂𝑶 𝒏𝟐
𝑡ℎ
− − − − (𝟒)
using Bohr atom model.  For first orbit, n = 1, (ground level) 𝑟1 = 𝑎𝑂 = 0.529 𝐴
Radius of nth orbit :  For second orbit, n = 2, (first excited level)
 Consider an atom which contains the nucleus at 𝑟2 = 4 𝑎𝑂 = 4 𝑋 0.529 𝐴 = 2.116 𝐴
rest which is made up of of protons and neutrons.  For third orbit, n = 3, (second excited level)
 Let an electron revolving around the stabe nucleus 𝑟3 = 9 𝑎𝑂 = 9 𝑋 0.529 𝐴 = 4.761 𝐴
 Atomic number =𝑍  Thus, radius of the orbit, 𝒓𝒏 ∝ 𝒏𝟐
Total charge of th nucleus =+𝑍𝑒 Velocity of electron in nth orbit :
Charge of an electron =−𝑒  According to Bohr’s quantization condition,
Mass of the electron =𝑚 ℎ
 From Coulomb’s law, the force of attraction 𝑚 𝑣𝑛 𝑟𝑛 = 𝑛
2𝜋
between the nucleus and the electron is 𝑛2 ℎ
1 (+ 𝑍 𝑒) (−𝑒) 1 𝑍 𝑒2 𝑚 𝑣𝑛 𝑎𝑂 =𝑛
⃗⃗⃗𝐹𝑐𝑜𝑢𝑙𝑜𝑚𝑏 = 𝑟̂ = − 𝑟̂ 𝑍 2𝜋
4 𝜋 𝜀𝑂 𝑟𝑛2 4 𝜋 𝜀𝑂 𝑟𝑛2 𝒉 𝒁
𝒗𝒏 = − − − (𝟓)
 This force provides necessary centripetal force given by. 𝟐 𝝅 𝒎 𝒂𝑶 𝒏
𝟏
𝑚 𝑣𝑛2  Hence, 𝒗𝒏 ∝ (i.e.) the velocity of the electron decreases as the principal
⃗⃗⃗𝐹𝑐𝑒𝑛𝑡𝑟𝑖𝑝𝑒𝑡𝑎𝑙 = − 𝑟̂ 𝒏
𝑟𝑛 quantum number increases
 At equilibrium, ⃗⃗⃗𝐹𝑐𝑜𝑢𝑙𝑜𝑚𝑏 = ⃗⃗⃗𝐹𝑐𝑒𝑛𝑡𝑟𝑖𝑝𝑒𝑡𝑎𝑙 Total Energy of electron in nth orbit :
1 𝑍 𝑒2 𝑚 𝑣𝑛2  Electrostatic force is a conservative force.
− 𝑟̂ = − 𝑟̂  So potential energy of the electron in nth orbit,
4 𝜋 𝜀𝑂 𝑟𝑛2 𝑟𝑛
1 𝑍𝑒 2
𝑚 𝑣𝑛 2 1 (+ 𝑍 𝑒) (−𝑒) 1 𝑍 𝑒2
= − − − − − (1) 𝑈𝑛 = = −
4 𝜋 𝜀𝑂 𝑟𝑛 2 𝑟𝑛 4 𝜋 𝜀𝑂 𝑟𝑛 4 𝜋 𝜀𝑂 𝑟𝑛
(4 𝜋 𝜀𝑂 ) 𝑚 𝑣𝑛2 𝑟𝑛2 (4 𝜋 𝜀𝑂 ) 𝑚2 𝑣𝑛2 𝑟𝑛2  Kinetic energy of the electron in nth orbit,
𝑟𝑛 = = 1 1 1 𝑍 𝑒2
𝑍 𝑒2 𝑍 𝑒 2𝑚 𝐾𝐸𝑛 = 𝑚 𝑣𝑛2 = [ ] [𝑏𝑦 𝑒𝑞𝑛(1)]
(4 𝜋 𝜀𝑂 ) [𝑚 𝑣𝑛 𝑟𝑛 ]2 2 2 4 𝜋 𝜀𝑂 𝑟𝑛
𝑟𝑛 =  Thus , 𝑼𝒏 = −𝟐 𝑲𝑬𝒏
𝑍 𝑒 2𝑚
 From Bohr’s postulate,  Therefore, total energy of the electron in nth orbit,
ℎ 1 𝑍 𝑒2
𝑙𝑛 = 𝑚 𝑣𝑛 𝑟𝑛 = 𝑛 =𝑛ℏ 𝐸𝑛 = 𝑈𝑛 + 𝐾𝐸𝑛 = −2 𝐾𝐸𝑛 + 𝐾𝐸𝑛 = − 𝐾𝐸𝑛 = −
2𝜋 8 𝜋 𝜀𝑂 𝑟𝑛
 Hence, ℎ2 𝜀 𝑂𝑛2
 From equation (2), 𝑟𝑛 = [ ] .Hence
(4 𝜋 𝜀𝑂 ) [𝑙𝑛 ]2 𝜋 𝑚 𝑒2 𝑍
𝑟𝑛 = 1 𝑍𝑒 2
𝑍 𝑒 2𝑚 𝐸𝑛 = −
𝑛ℎ 2 8 𝜋 𝜀𝑂 ℎ2 𝜀𝑂 𝑛2
(4 𝜋 𝜀𝑂 ) [ ] [ ]
2𝜋 𝜋 𝑚 𝑒2 𝑍
𝑟𝑛 =
𝑍 𝑒 2𝑚 𝒎 𝒆𝟒 𝒁𝟐
(4 𝜋 𝜀𝑂 ) 𝑛2 ℎ2 𝑬𝒏 = − − − − − (𝟔)
𝑟𝑛 = 𝟖 𝜺𝑶𝟐 𝒉𝟐 𝒏𝟐
𝑍 𝑒 2𝑚 𝑋 4 𝜋 2  For hydrogen, (Z = 1), then
𝒉𝟐 𝜺𝑶 𝒏𝟐 𝒎 𝒆𝟒 𝟏
𝒓𝒏 = [ ] − − − − (𝟐) 𝑬𝒏 = − − − − − (𝟕)
𝝅 𝒎 𝒆𝟐 𝒁 𝟖 𝜺𝑶𝟐 𝒉𝟐 𝒏𝟐
 Here 𝜺𝑶 , 𝒉, 𝒎, 𝒆 and 𝝅 are constant.  The negative sign in equation (7) indicates that the electron is bound to the
𝒏𝟐 nucleus.
𝒓𝒏 = 𝒂𝑶 − − − − (𝟑)
𝒁  Put the values of 𝜺𝑶 , 𝒉, 𝒎, 𝒆 and using ‘eV’ unit we have,
𝒉 𝟐 𝜺𝑶 𝟏𝟑. 𝟔
where, 𝑎𝑂 =
𝝅𝒎𝒆 𝟐 = 0.529 𝐴 → Bohr radius 𝑬𝒏 = − 𝒆𝑽
𝒏𝟐
victory R. SARAVANAN. M.Sc., M.Phil., B.Ed PG ASST [PHYSICS], GBHSS, PARANGIPETTAI - 608 502
12 PHYSICS UNIT –9 ATOMIC PHYSICS AND NUCLEAR PHYSICS COMPLETE GUIDE AND MODEL QUESTION
 when, n = 1, 𝑬𝟏 = −𝟏𝟑. 𝟔 𝒆𝑽 5. Explain the variation of average binding energy with the mass number by
when, n = 2, 𝑬𝟐 = − 𝟑. 𝟒 𝒆𝑽 graph and discuss its features.
when, n = 3, 𝑬𝟑 = − 𝟏. 𝟓𝟏 𝒆𝑽 Binding energy curve :
 Thus, as ‘n’ increases, energy also increases. (i.e.) the orbit which is closest to the
nucleus has lowest energy. So it is often called ground state  The average binding energy per
nucleon ( 𝐵𝐸 ̅̅̅̅ ) is the energy
 The ground state energy of hydrogen (- 13.6 eV) is used as a unit of energy called
Rydberg. (i.e) 1 Rydberg = − 13.6 eV required to separate single nucleon
4. Explain the spectral series of hydrogen atom. from the particular nucleus.
Spectral series of hydrogen atom : 𝐵𝐸 [(𝑍 𝑚𝑃 + 𝑁 𝑚𝑛 ) − 𝑀𝐴 ] 𝑐 2
̅̅̅̅ =
𝐵𝐸 =
 When an electron jumps from mth orbit to nth orbit, a spectral line was obtained 𝐴 𝐴
 𝐵𝐸̅̅̅̅ is plotted against A of all known
whose wave number (i.e.) reciprocal of wave length is,
1 1 1 nuclei and the graph obtained is
𝜈̅ = = 𝑅 [ 2 − 2 ] called binding energy curve.
𝜆 𝑛 𝑚
 here, R  Ryderg constant(𝑅 = 1.097 𝑋 10 𝑚 ) 7 −1  From the graph,
 For m > n, various spectral series are obtained. (1) The value of ̅̅̅̅𝐵𝐸 rises as the mass number A increases until it reaches a
(1) Lyman series : maximum value of 8.8 MeV for A = 56 (iron) and then it slowly decreases.
 n = 1 and m = 2, 3, 4, ….. (2) The average binding energy per nucleon is about 8.5 MeV for nuclei having
 Hence the wave number, mass number between A = 40 and 120. These elements are comparatively
1 1 1 more stable and not radioactive.
𝜈̅ = = 𝑅 [ 2 − 2 ] (3) For higher mass numbers, the curve reduces slowly and ̅̅̅̅ 𝐵𝐸 for uranium is about
𝜆 1 𝑚
 They lie in ultra violet region 7.6 MeV. They are unstable and radioactive.
(2) Balmer series : (4) If two light nuclei with A<28 combine to form heavier nucleus, the binding
 𝑛 = 2 and 𝑚 = 3, 4, 5, . …. energy per nucleon is more for final nucleus than initial nuclei. Thus, if the
 Hence the wave number, lighter elements combine to produce a nucleus of medium value A, a large
1 1 1 amount of energy will be released. This is the basis of nuclear fusion and is the
𝜈̅ = = 𝑅 [ 2 − 2 ] principle of the hydrogen bomb.
𝜆 2 𝑚
 They lie in visible region (5) If a nucleus of heavy element is split (fission) into two or more nuclei of medium
(3) Paschen series : value A, the energy released would again be large. The atom bomb is based on
 𝑛 = 3 and 𝑚 = 4, 5, 6, …. this principle.
 Hence the wave number 6. Obtain the law of radioactivity (radioactive decay)
1 1 1 Law of radioactivity :
𝜈̅ = = 𝑅 [ 2 − 2 ]  At any instant t, the number of decays per unit time, called rate of decay (dN/ dt)
𝜆 3 𝑚
 They lie in infra red region is proportional to the number of nuclei (N) at the same instant. This is called law
(4) Brackett series : of radioactive decay.
 𝑛 = 4 and 𝑚 = 5, 6, 7, …. Expression :
 Hence the wave number,  Let 𝑁𝑂 be the numer of nuclei at initial time (t = 0)
1 1 1  Let ‘N’ be the number of undecayed nuclei at any time ‘t’
𝜈̅ = = 𝑅 [ 2 − 2 ] 𝑑𝑁
𝜆 4 𝑚  If ‘dN’ be the number of nuclei decayed in time ‘dt’ then, rate of decay =
𝑑𝑡
 They lie in middle infra red region,
 From law of radioactivity,
(5) Pfund series : 𝑑𝑁
 𝑛 = 5 and 𝑚 = 6, 7,8, …. ∝𝑁
𝑑𝑡
 Hence the wave number 𝑑𝑁
1 1 1 (𝑜𝑟) = −𝜆𝑁 − − − − − (1)
𝜈̅ = = 𝑅 [ 2 − 2 ] 𝑑𝑡
𝜆 5 𝑚 Here, 𝜆 → decay constant
 They lie in far infra red region.

victory R. SARAVANAN. M.Sc., M.Phil., B.Ed PG ASST [PHYSICS], GBHSS, PARANGIPETTAI - 608 502
12 PHYSICS UNIT –9 ATOMIC PHYSICS AND NUCLEAR PHYSICS COMPLETE GUIDE AND MODEL QUESTION
 Decay constant (𝜆 ) is different for different radioactive sample and the negative Mean life time (𝝉) :
sign in the equation implies that the N is decreasing with time.  The mean life time of the nucleus is the ratio of sum or integration of life times of
 By rewriting the equation (1), we get all nuclei to the total number nuclei present initially.
𝑑𝑁  Let 𝜆 be the decay constant of the radioactive substance, then
= − 𝜆 𝑑𝑡 𝟏
𝑁 𝝉=
Integrating on both sides, 𝝀
𝑁 𝑡  Thus mean life and decay constant is inversely proportional to each other.
𝑑𝑁
∫ = − 𝜆 ∫ 𝑑𝑡 Half life and mean life - Relation :
𝑁𝑂 𝑁 0
𝑁
[ln 𝑁] 𝑁𝑂 = − 𝜆 𝑡  Half life time is given by,
ln 2 0.6931
[ln 𝑁 − ln 𝑁𝑂 ] = − 𝜆 𝑡 𝑇1 = =
2 𝜆 𝜆
𝑁
ln [ ] = − 𝜆 𝑡  Mean life period is given by,
𝑁𝑂 1
Taking exponential on both sides,
𝜏= 𝜆
𝑁  From the above two equations,
= 𝑒− 𝜆 𝑡 𝑻𝟏 = 𝝉 𝒍𝒏 𝟐 = 𝟎. 𝟔𝟗𝟑𝟏 𝝉
𝑁𝑂
−𝝀𝒕 𝟐
𝑵 = 𝑵𝑶 𝒆 − − − − (𝟐) 8. Explain radio carbon dating.
 Equation (2) is called the law of radioactive decay. Radio carbon dating :
 Here the number of atoms is decreasing exponentially over the time.  The important application of beta ecay is radioactive dating or carbon dating.
 This implies that the time taken for all the radioactive nuclei to decay will be Using this technique, the age of an ancient object can be calculated.
infinite.  All living organisms absorb carbon dioxide (CO2) from air to synthesize organic
7. Obtain an expression for half life time and mean life time. molecules.
Half life time (𝑻𝟏 ) : In this absorbed CO2, the major part is 12
𝟐
 6 𝐶 and very small
14
 Half-life T1/2 is thetime required for the number of atoms initially present to fraction 6 𝐶 whose half-life is 5730 years.
reduce to one half of the initial amount.  Carbon-14 in the atmosphere is always decaying but at the same time, cosmic
 From the law of radioactive decay, 𝑁 = 𝑁𝑂 𝑒 − 𝜆 𝑡 rays from outer space are continuously bombarding the atoms in the atmosphere
𝑁𝑂 which produces 146 𝐶 .
 If 𝑡 = 𝑇1 then, 𝑁 = . Hence
2 2  So the continuous production and decay of 146 𝐶 in the atmosphere keep the ratio
𝑁𝑂 − 𝜆 𝑇1 of 146 𝐶 to 126 𝐶 always constant.
= 𝑁𝑂 𝑒 2
2  Since our human body, tree or any living organism continuously absorb CO 2 from
1 − 𝜆 𝑇1 𝜆 𝑇1
the atmosphere, the ratio of 146 𝐶 to 126 𝐶 in the living organism is also nearly
= 𝑒 2 (𝑜𝑟) 𝑒 2 = 2
2 constant.
 Taking log on both sides,  But when the organim dies, it stops absorbing CO2. Since 146 𝐶 starts to decay, the
𝜆 𝑇1 = ln 2 ratio of 146 𝐶 to 126 𝐶 in a dead organism or specimen decreases over the years.
2
𝐥𝐧 𝟐 𝟎. 𝟔𝟗𝟑𝟏  Suppose the ratio of 146 𝐶 to 126 𝐶 in the ancient tree pieces excavated is known,
𝑻𝟏 = = then the age of the tree pieces can be calculated.
𝟐 𝝀 𝝀
 If the number of atoms present at t = 0 is N0 , then 9. Describe the working of nuclear reactor with a block diagram.
𝑵 Nucleaar reactor :
(1) Number of atoms remais undecayed after 1st half life = 𝑶  Nuclear reactor is a system in which the nuclear fission takes place in a self-
𝟐
𝑵𝑶
(2) Number of atoms remais undecayed after 2nd half life = sustained controlled manner
𝟒
𝑵𝑶  The energy produced is used either for research purpose or for power
(3) Number of atoms remais undecayed after 3rd half life =
𝟖 generation.
 In general, after n half-lives, the number of nuclei remaining undecayed is given  The first nuclear reactor was built in the year 1942 at Chicago.
by
𝑵𝑶
𝑁 = 𝒏
𝟐
victory R. SARAVANAN. M.Sc., M.Phil., B.Ed PG ASST [PHYSICS], GBHSS, PARANGIPETTAI - 608 502
12 PHYSICS UNIT –9 ATOMIC PHYSICS AND NUCLEAR PHYSICS COMPLETE GUIDE AND MODEL QUESTION
 This coolant passes through the fuel block and carries away the heat to the
steam generator through heat exchanger
 The steam runs the turbines which produces electricity in power reactors.
(6) Shielding :
 For a protection against harmful radiations, the nuclear reactor is
surrounded by a concrete wall of thickness of about 2 to 2.5 m.
10. Briefly explain the elementary particles of nature.
Elementary particles :
 An atom has a nucleus surrounded by electrons and nuclei is made up of protons
and neutrons.
 Initially, protons, neutrons and electrons are considered as fundamental building
blocks of matter.
 But in 1964, Murray Gellman and George Zweig theoretically proposed that
Main parts of Nuclear reactor : protons and neutrons are not fundamental particles, but they are made up of
(1) Fuel : quarks.
 The commonly used fuels are 235 239
92𝑈 and 94 𝑃𝑢  These quarks are now considered elementary particles of nature.
 Naturally occurring uranium contains only 0.7% of 235 92𝑈 and 99.3% are only  Electrons are fundamental or elementary particles because they are not made up
238
92𝑈 . of anything.
 So the 238 235
92𝑈 must be enriched such that it contains at least 2 to 4% of 92𝑈  In the year 1968, the quarks were discovered experimentally by Stanford.
(2) Neutron source :  There are six quarks namely,
 A neutron source is required to initiate the chain reaction for the first time. (1) Up quark
 A mixture of beryllium with plutonium or polonium is used as the neutron (2) Down quark
source (3) Charm quark
(3) Moderators : (4) Strange quark
 The probability of initiating fission by fast neutron in another nucleus is very (5) Top quark and
low. Therefore, slow neutrons are preferred for sustained nuclear reactions (6) Bottom quark
 The moderator is a material used to convert fast neutrons into slow  There exist their anti particle also.
neutrons.  All these quarks have fractional charges. For example,
 Usually the moderators having mass comparable to that of neutrons. Hence, 2
Charge of up quark is = + 𝑒
these light nuclei undergo collision with fast neutrons and the speed of the 3
1
neutron is reduced Charge of down quark is = − 𝑒
3
 Most of the reactors use water, heavy water (D2O) and graphite as  According to quark model,
moderators. (1) Proton is made up of two up quarks and one down quark
(4) Control rods : (2) Neutron is made up of one up quark and two down quarks
 The control rods are used to adjust the reaction rate.
 During each fission, on an average 2.5 neutrons are emitted
 In order to have the controlled chain reactions, only one neutron is allowed
to cause another fission and the remaining neutrons are absorbed by the
control rods.
 Usually cadmium or boron acts as control rod material
(5) Coolants :
 The cooling system removes the heat generated in the reactor core.
 Ordinary water, heavy water and liquid sodium are used as coolant since
they have very high specific heat capacity and have large boiling point under
high pressure.

victory R. SARAVANAN. M.Sc., M.Phil., B.Ed PG ASST [PHYSICS], GBHSS, PARANGIPETTAI - 608 502
12 PHYSICS UNIT –9 ATOMIC PHYSICS AND NUCLEAR PHYSICS COMPLETE GUIDE AND MODEL QUESTION
11. Explain in detail the four fundamental forces.
Fundamental forces in nature : EXAMPLE PROBLEMS WITH SOLUTIONS
 Gravitational, electromagnetic, strong and weak forces are called fundamental 1. The radius of the 5th orbit of hydrogen atom is 13.25 Å. Calculate the de Broglie
forces of nature. wavelength of the electron orbiting in the 5th orbit.
𝒐
(1) Gravitational forces : -Solution :- 𝒓 = 𝟏𝟑. 𝟐𝟒 𝑨 = 𝟏𝟑. 𝟐𝟒 𝑿 𝟏𝟎−𝟏𝟎 𝒎 ; 𝒏 = 𝟓
 The attractive force between two masses is called gravitational force and it is
 From quantum mechanics ; 2 𝜋 𝑟 = 𝑛 𝜆
universal in nature.
2𝜋𝑟 2 𝑋 3.14 𝑋 13.24 𝑋 10−10
 Our planets are bound to the sun through gravitational force of the sun. (𝑜𝑟) 𝜆 = = = 2 𝑋 3.14 𝑋 2.648 𝑋 10−10
 We are in the Earth because of Earth’s gravitational attraction on our body. 𝑛 5 𝑜
(2) Electromagnetic force : 𝜆 = 16.63 𝑋 10−10 𝑚 = 16.63 𝐴
 Between two charges there exists electromagnetic force and it plays major 2. Find the (i) angular momentum (ii) velocity of the electron revolving in the 5 th
role in most of our day-today events. orbit of hydrogen atom. (h = 6.6 × 10–34 Js, m = 9.1 × 10–31 kg)
 We are standing on the surface of the earth because of the electromagnetic -Solution :- 𝒏 = 𝟓 ; 𝒉 = 𝟔. 𝟔 𝑿 𝟏𝟎−𝟑𝟒 𝑱𝒔, 𝒎 = 𝟗. 𝟏 𝑿 𝟏𝟎−𝟑𝟏 𝒌𝒈
force between atoms of the surface of the earth with atoms in our foot (i) According to Bohr’s atom model, the angular momentum
−34
 It is stronger than gravitational force. 𝑛 ℎ 5 𝑋 6.6 𝑋 10 5 𝑋 6.6 𝑋 10−34 𝑋 7
𝑙 =𝑛ℏ= = =
(3) Strong Nuclear force : 2𝜋 22 2 𝑋 22
2𝑋
 Between two nucleons, there exists a strong nuclear force and this force is 7
responsible for stability of the nucleus. 𝑙 = 2.5 𝑋 0.3 𝑋 10−34 𝑋 7
 The atoms in our body are stable because of strong nuclear force. 𝒍 = 𝟓. 𝟐𝟓 𝑿 𝟏𝟎−𝟑𝟒 𝒌𝒈 𝒎𝟐 𝒔−𝟏
(4) Weak Nuclear force : (ii) By definition, angular momentum is given by; = 𝑚 𝑣 𝑟𝑛 .
 In addition to these three forces, there exists another fundamental force of Hence velocity,
nature called the weak force. 𝑙 𝑙
𝑣= =
 This weak force is even shorter in range than nuclear force. 𝑚 𝑟𝑛 𝑚 𝑛2 𝑟1
 This force plays an important role in beta decay and energy production of 5.25 𝑋 10−34
stars. 𝑣 =
9.1 𝑋 10−31 𝑋 25 𝑋 0.53 𝑋 10−10
 During the fusion of hydrogen into helium in sun, neutrinos and enormous 5.25 𝑋 107
radiations are produced through weak force. 𝑣 =
9.1 𝑋 25 𝑋 0.53
 The lives of species in the earth depend on the solar energy from the sun and 𝒗 = 4.355 𝑋 10−2 𝑋 107 = 𝟒. 𝟑𝟓𝟓 𝑿 𝟏𝟎𝟓 𝒎 𝒔−𝟏
it is due to weak force which plays vital role during nuclear fusion reactions 3. (a) Show that the ratio of velocity of an electron in the first Bohr orbit to the
going on in the core of the sun speed of light c is a dimensionless number. (b) Compute the velocity of electrons
in ground state, first excited state and second excited state in Bohr atom model
for hydrogen atom.
-Solution :-
ℎ 𝑍
(a) Velocity of electron in nth orbit ; 𝑣𝑛 = 2𝜋𝑚𝑎𝑜
[𝑛]
𝜀𝑜 ℎ2 𝑜
 Where, 𝑎𝑜 = = 0.53 𝐴 → Bohr radius
𝜋𝑚𝑒2
ℎ 𝑍 ℎ 𝜋𝑚𝑒 2 𝑍 𝑒2 𝑍
 Hence ; 𝑣𝑛 = 𝜀 ℎ2
[𝑛 ] = [𝑛 ] = [𝑛 ]
2𝜋𝑚( 𝑜 2 ) 2𝜋𝑚𝜀𝑜 ℎ 2 2𝜀𝑜 ℎ
𝜋𝑚𝑒
 If ‘c’ be the velocity of light, then
𝑣𝑛 𝑒2 𝑍 𝑍
= [ ] = 𝛼 [ ] − − − − (1)
𝑐 2𝜀𝑜 ℎ 𝑐 𝑛 𝑛
𝑒2
 Where, = 𝛼 → fine structure constant
2𝜀𝑜 ℎ 𝑐

victory R. SARAVANAN. M.Sc., M.Phil., B.Ed PG ASST [PHYSICS], GBHSS, PARANGIPETTAI - 608 502
12 PHYSICS UNIT –9 ATOMIC PHYSICS AND NUCLEAR PHYSICS COMPLETE GUIDE AND MODEL QUESTION
 For hydrogen atom ; 𝑍 = 1 , and for first orbit ; 𝑛 = 1. Hence  Hence kinetic energy of the system,
𝑣𝑛 𝑒2 (1.6 𝑋 10−19 )2 𝑝𝑒2 𝑝𝑛2 𝑝2 1 1 𝑝2 𝑚 + 𝑀
= 𝛼= = 𝐾𝐸 = + = [ + ]= [ ]
𝑐 2𝜀𝑜 ℎ 𝑐 2 𝑋 8.85 𝑋 10−12 𝑋 6.6 𝑋 10−34 𝑋 3 𝑋 108 2𝑚 2𝑀 2 𝑚 𝑀 2 𝑚𝑀
2 4
𝑣𝑛 1.6 𝑋 1.6 1 𝑝 1 𝜇𝑚 𝑒 1
= 𝛼= = 𝐾𝐸 = [ ]=
𝑐 2 𝑋 8.85 𝑋 6.6 𝑋 3 136.9 2 𝜇 𝑚 8 𝜀𝑜2 ℎ2 𝑛2
𝑣𝑛 1 𝑚𝑀
= 𝛼≈ Where, 𝜇𝑚 = → reduced mass
𝑚+ 𝑀
𝑐 137  Potential energy of the system,
 It is dimensionless number.
𝜇𝑚 𝑒 4 1
(b) From equation (1) , 𝑈 = −
𝑍 4 𝜀𝑜2 ℎ2 𝑛2
𝑣𝑛 = 𝛼 [ ] 𝑐  Hence total energy of the system,
𝑛
 For hydrogen atom ; 𝑍 = 1 Hence. 𝜇𝑚 𝑒 4 1
𝐸𝑛 = −
1 8 𝜀𝑜2 ℎ2 𝑛2
𝑣𝑛 = 𝛼 𝑐 [ ]  Since the nucleus is very heavy compared to the electron, the reduced mass is
𝑛
 For first orbit (i.e.) ground state ; 𝒏 = 𝟏 closer to the mass of the electron
1 5. Suppose the energy of an electron in hydrogen–like atom is given as
𝑣1 = 𝛼 𝑐 = 𝑋 3 𝑋 108 = 2.19 𝑋 10−2 𝑋 108 𝟓𝟒.𝟒
137 𝑬𝒏 = − 𝟐 𝒆𝑽 where ∈ ℕ . Calculate the following:
𝒗𝟏 = 𝟐. 𝟏𝟗 𝑿 𝟏𝟎𝟔 𝒎 𝒔−𝟏 𝒏
(a) Sketch the energy levels for this atom and compute its atomic number.
 For second orbit (i.e.) first excited state ; 𝒏 = 𝟐
1 1 1 1 (b) If the atom is in ground state, compute its first excitation potential and also
𝑣2 = 𝛼 𝑐 [ ] = 𝑋 3 𝑋 108 [ ] = 2.19 𝑋 106 𝑋 [ ] its ionization potential
2 137 2 2 (c) When a photon with energy 42 eV and another photon with energy 56 eV are
𝒗𝟐 = 𝟏. 𝟎𝟗𝟓 𝑿 𝟏𝟎𝟔 𝒎 𝒔−𝟏
made to collide with this atom, does this atom absorb these photons?
 For third orbit (i.e.) second excited state ; 𝒏 = 𝟑
1 1 1 1 (d) Determine the radius of its first Bohr orbit.
𝑣3 = 𝛼 𝑐 [ ] = 𝑋 3 𝑋 108 [ ] = 2.19 𝑋 106 𝑋 [ ] (e) Calculate the kinetic and potential energies of electron in the ground state.
3 137 3 3 -Solution :-
𝒗𝟑 = 𝟎. 𝟕𝟑 𝑿 𝟏𝟎𝟔 𝒎 𝒔−𝟏
(a) Given energy ;
 Here ; 𝒗𝟏 > 𝒗𝟐 > 𝒗𝟑 𝟓𝟒. 𝟒
4. The Bohr atom model is derived with the assumption that the nucleus of the 𝑬𝒏 = − 𝒆𝑽
𝒏𝟐
atom is stationary and only electrons revolve around the nucleus. Suppose the  If 𝒏 = 𝟏 then, 𝐸1 = − 54.4 𝑒𝑉
nucleus is also in motion, then calculate the energy of this new system.
 If 𝒏 = 𝟐 then, 𝐸2 = − 13.6 𝑒𝑉
-Solution :-
 Let the mass of the electron be m and mass of the nucleus be M. Since there is no  If 𝒏 = 𝟑 then, 𝐸3 = − 6.04 𝑒𝑉
external force acting on the system, the  If 𝒏 = 𝟒 then, 𝐸4 = − 3.4 𝑒𝑉
centre of mass of hydrogen atom  If 𝒏 = ∞ then, 𝐸∞ = 0 𝑒𝑉
remains at rest. Hence, both nucleus (b) From Bohr’s atom model, total energy of an atom ;
and electron move about the centre of 𝟏𝟑. 𝟔 𝒁𝟐
mass as shown in figure. 𝑬𝒏 = − 𝒆𝑽
 Let V be the velocity of the nuclear 𝒏𝟐
Compare it with the given equation,
motion and υ be the velocity of electron
13.6 𝑍 2 = 54.4
motion. Since the total linear 54.4
momentum of the system is zero ; 𝑍2 = = 4
13.6
⃗𝑝𝑒 + ⃗𝑝𝑛 = ⃗0 (or) |⃗𝑝𝑒 | = |⃗⃗𝑝𝑛 | = 𝑝 𝑍 = ±2
Since, atomic number cannot be negative number, Z = 2

victory R. SARAVANAN. M.Sc., M.Phil., B.Ed PG ASST [PHYSICS], GBHSS, PARANGIPETTAI - 608 502
12 PHYSICS UNIT –9 ATOMIC PHYSICS AND NUCLEAR PHYSICS COMPLETE GUIDE AND MODEL QUESTION
First excitation energy, 𝟏𝟗𝟕
7. Calculate the radius of 𝟕𝟗 𝑨𝒖 nucleus.
𝑬𝒆𝒙𝒄𝒊𝒕𝒂𝒕𝒊𝒐𝒏 = 𝐸2 − 𝐸1 = −13.6 − (−54.4) = −13.6 + 54.4 = 𝟒𝟎. 𝟖 𝒆𝑽 -Solution : 𝑍 = 79 ; 𝐴 = 197
1
Hence first excitation potential,
𝑬 40.8 𝑒𝑉 𝑅 = 𝑅𝑜 𝐴3
1
𝑽𝒆𝒙𝒄𝒊𝒕𝒂𝒕𝒊𝒐𝒏 = = = 𝟒𝟎. 𝟖 𝑽 𝑅 = 1.2 𝑋 10−15 𝑋 (197)3
𝑒 𝑒
Similarly, first ionization energy 𝑅 = 1.2 𝑋 10−15 𝑋 5.818
𝑬𝒊𝒐𝒏𝒊𝒔𝒂𝒕𝒊𝒐𝒏 = 𝐸∞ − 𝐸1 = 0 − (−54.4) = 𝟓𝟒. 𝟒 𝒆𝑽 𝑅 = 6.9816 𝑋 10−15 𝑚 = 6.9816 𝐹
and first ionization potential, 8. Calculate the density of the nucleus with mass number A.
𝑬𝒊𝒐𝒏𝒊𝒔𝒂𝒕𝒊𝒐𝒏 54.4 𝑒𝑉 -Solution :
𝑽𝒊𝒐𝒏𝒊𝒔𝒂𝒕𝒊𝒐𝒏 = = = 𝟓𝟒. 𝟒 𝑽
𝑒 𝑒  The total mass of the nucleus having mass number A is equal to 𝐴 𝑚 where m is
(c) Consider two photons to be A and B. Given that photon A with energy 42 eV and mass of the proton and is equal to 1.6726 x 10-27 kg
photon B with energy 51 eV From Bohr assumption, difference in energy levels is  Hence the nuclear density,
equal to the energy photon absorbed, then atom will absorb energy, otherwise, Nuclear mass 𝐴𝑚 𝐴𝑚
𝜌= = 4 3 = 1 3
not. Nuclear volume
3
𝜋𝑅 4
𝜋 (𝑅0 𝐴3 )
𝐸2 − 𝐸1 = −13.6 − (−54.4) = −13.6 + 54.4 = 𝟒𝟎. 𝟖 𝒆𝑽 3

𝐸3 − 𝐸1 = −6.04 − (−54.4) = −6.04 + 54.4 = 𝟒𝟖. 𝟑𝟔 𝒆𝑽 𝐴 𝑚 𝑚 3 𝑋 1.67 𝑋 10−27


𝜌= = =
𝐸4 − 𝐸1 = −3.4 − (−54.4) = −3.4 + 54.4 = 𝟓𝟏 𝒆𝑽 4 4 4 𝑋 3.14 𝑋 (1.2 𝑋 10−15 )3
𝜋 𝑅𝑜3 𝐴 𝜋 𝑅𝑜3
𝐸3 − 𝐸2 = −6.04 − (−13.6) = −6.04 + 13.6 = 𝟕. 𝟓𝟔 𝒆𝑽 3 3
3 𝑋 1.67 𝑋 1018 5.01 𝑋 1018
 For all possibilities, no difference in energy is an integer multiple of photon 𝜌= = = 2.308 𝑋 10−1 𝑋 1018
energy. Hence, photon A is not absorbed by this atom. But for Photon B, 4 𝑋 3.14 𝑋 1.728 21.70368
E4 – E1 = 51 eV, which means, Photon B can be absorbed by this atom. 𝝆 = 𝟐. 𝟑𝟎𝟖 𝑿 𝟏𝟎𝟏𝟕 𝒌𝒈 𝒎−𝟑
𝑎 𝑛 2  It implies that nucleons are extremely tightly packed or compressed state in the
(d) The radius of Bohr orbit is ; 𝑟𝑛 = 𝑜 nucleus and compare this density with the density of water which is 103 kg m-3.
𝑧
Here, 𝑛 = 1 , 𝑧 = 2 9. Compute the binding energy of 𝟒𝟐 𝑯𝒆 nucleus using the following data: Atomic
𝑎𝑜 0.529 𝑋 10−10 mass of Helium atom, 𝑴𝑨(𝑯𝒆) = 𝟒. 𝟎𝟎𝟐𝟔𝟎 𝒖 and that of hydrogen atom,
Hence, ; 𝑟1 =
2
= 2
= 0.264𝑋10−10 𝑚 = 0.2648 Å
(e) Since, total energy is equal to negative of kinetic energy in Bohr atom model, we 𝒎𝑯 = 𝟏. 𝟎𝟎𝟕𝟖𝟓 𝒖 .
get -Solution : 𝑍 = 2 ; 𝐴 = 4 ; 𝑁 = 𝐴 − 𝑍 = 2 ; 𝑀𝐴(𝐻𝑒) = 4.00260 𝑢 ; 𝑚𝐻 = 1.00785 𝑢
54.4 54.4  Mass defect , ∆𝑚 = 𝑍 𝑚𝐻 + 𝑁 𝑚𝑛 − 𝑀𝐴(𝐻𝑒)
𝐾𝐸𝑛 = − 𝐸𝑛 = − (− 2 𝑒𝑉) = 2
𝑒𝑉 ∆𝑚 = (2 𝑋 1.00785) + (2 𝑋 1.008665) − 4.00260
𝑛 𝑛
( )
For ground state 𝑛 = 1 ; 𝑲𝑬𝟏 = 𝟓𝟒. 𝟒 𝒆𝑽 ∆𝑚 = 2.0157 + 2.01733 − 4.00260
And potential energy, ∆𝑚 = 4.03303 − 4.00260
54.4 108.8 ∆𝒎 = 𝟎. 𝟎𝟑𝟎𝟒𝟑 𝒖
𝑈𝑛 = −2 𝐾𝐸𝑛 = − 2 ( 2 𝑒𝑉) = − 2
𝑒𝑉  The energy equivalent to this mass defect is called binding energy.
𝑛 𝑛
For ground state (𝑛 = 1) ; 𝑼𝟏 = − 𝟏𝟎𝟖. 𝟖 𝒆𝑽  Energy equivalent to 𝟏 𝒖 is 𝟗𝟑𝟏 𝑴𝒆𝑽. Hence the binding energy is,
6. Calculate the average atomic mass of chlorine if no distinction is made between 𝑩𝑬 = ∆𝒎 𝑿 𝟗𝟑𝟏 𝑴𝒆𝑽 = 0.03043 𝑋 931 = 𝟐𝟖. 𝟑𝟑 𝑴𝒆𝑽
its different isotopes? 10. Compute the binding energy per nucleon of 𝟒𝟐 𝑯𝒆 nucleus.
-Solution : -Solution : 𝑍 = 2 ; 𝐴 = 4
35 37
 The element chlorine is a mixture of 75.77% of 17𝑐𝑙 and 24.23% of 17𝑐𝑙. So the  Mass defect of helium nucleus, ∆𝒎 = 𝟎. 𝟎𝟑𝟎𝟒𝟑 𝒖
average atomic mass will be  Binding energy of helium nucleus,
75.77 24.23 𝑩𝑬 = ∆𝒎 𝑿 𝟗𝟑𝟏 𝑴𝒆𝑽 = 0.03043 𝑋 931 = 𝟐𝟖 𝑴𝒆𝑽
= 𝑋 34.96885 𝑢 + 𝑋 36.96593 𝑢 = 𝟑𝟓. 𝟒𝟓 𝒖
100 100  Hence Binding energy per nucleon,
𝐵𝐸 28
̅̅̅̅ =
𝑩𝑬 = = 𝟕 𝑴𝒆𝑽
𝐴 4

victory R. SARAVANAN. M.Sc., M.Phil., B.Ed PG ASST [PHYSICS], GBHSS, PARANGIPETTAI - 608 502
12 PHYSICS UNIT –9 ATOMIC PHYSICS AND NUCLEAR PHYSICS COMPLETE GUIDE AND MODEL QUESTION
𝟐𝟑𝟐
11. (a) Calculate the disintegration energy when stationary 𝟗𝟐 𝑼 nucleus decays to 12. Calculate the number of nuclei of carbon-14 un decayed after 22,920 years if the
thorium 𝟐𝟐𝟖
𝟗𝟎 𝑻𝒉 with the emission of α particle. The atomic masses are of initial number of carbon-14 atoms is 10,000. The half-life of carbon-14 is 5730
𝟐𝟑𝟐 𝟐𝟐𝟖 𝟒 years.
𝟗𝟐 𝑼 = 𝟐𝟑𝟐. 𝟎𝟑𝟕𝟏𝟓𝟔 𝒖, 𝟗𝟎 𝑻𝒉 = 𝟐𝟐𝟖. 𝟎𝟐𝟖𝟕𝟒𝟏 𝒖, 𝟐 𝑯𝒆 = 𝟒. 𝟎𝟎𝟐𝟔𝟎𝟑 𝒖
(b) Calculate kinetic energies of 𝟐𝟐𝟖 -Solution : 𝑁𝑜 = 10000 ; 𝑡 = 22920 𝑦𝑒𝑎𝑟𝑠 ; 𝑇1/2 = 5730 𝑦𝑒𝑎𝑟𝑠
𝟗𝟎 𝑻𝒉 and α-particle and their ratio.
-Solution :  Number of half life periods,
𝟐𝟑𝟐 𝟐𝟐𝟖 𝑡 22920
(a) The equation is given by ; 𝟗𝟐 𝑼 → 𝟗𝟎 𝑻𝒉 + 𝜶 𝑛= = =4
𝑇1/2 5730
 Hence the mass defect,
∆𝑚 = Total mass before decay - Total mass after decay  The number of nuclei remaining un decayed after 22,920 years (i.e.) 4 half life
periods,
∆𝑚 = 𝑚𝑈 − (𝑚 𝑇ℎ + 𝑚𝛼 ) = 232.037156 − (228.028741 + 4.002603)
1 𝑛 1 4 10000
∆𝑚 = 232.037156 − 232.031344 = 0.005812 𝑵 = ( ) 𝑁𝑜 = ( ) 𝑋 10000 = = 𝟔𝟐𝟓
 Energy equivalent to 𝟏 𝒖 is 𝟗𝟑𝟏 𝑴𝒆𝑽. Hence the binding energy is, 2 2 16
𝟏𝟑
13. A radioactive sample has 26.μg of pure 𝟕 𝑵 which has a half-life of 10 minutes.
𝑸 = ∆𝒎 𝑿 𝟗𝟑𝟏 𝑴𝒆𝑽 = 0.005812 𝑋 931 = 𝟓. 𝟒𝟏 𝑴𝒆𝑽
(a) How many nuclei are present initially? (b) What is the activity initially?
 This disintegration energy Q appears as the kinetic energy of α particle and
(c) What is the activity after 2 hours? (d) Calculate mean life of this sample.
the daughter nucleus.
(b) In any decay, the total linear momentum must be conserved. (i.e.) -Solution : : 𝑇1/2 = 10 min = 600 𝑠 ; 𝑚 = 2.6 𝜇𝑔 = 2.6 𝑋 10−6 𝑔 ; 𝐴 = 13
 Total linear momentum of the parent nucleus = total linear momentum of the (a) The atomic mass of nitrogen is 13. Therefore, 13 g of 𝟏𝟑𝟕 𝑵 contains Avogadro
daughter nucleus and α particle number (6.02 𝑋 1023 ) of atoms.
6.02 𝑋 1023
𝑚𝑈 (0) = 𝑚𝜶 ⃗⃗𝑣𝜶 + 𝑚𝑇ℎ ⃗⃗𝑣𝑇ℎ  In 1 g, the number of 𝟏𝟑𝟕 𝑵 atoms present =
13
0 = 𝑚𝜶 ⃗⃗⃗𝑣𝜶 + 𝑚 𝑇ℎ ⃗⃗⃗𝑣 𝑇ℎ
 So the number of 𝟏𝟑𝟕 𝑵 atoms present in 2.6 μg is
𝑚𝜶 ⃗⃗⃗𝑣𝜶 = − 𝑚 𝑇ℎ ⃗⃗⃗𝑣 𝑇ℎ
6.02 𝑋 1023 15.652
 It implies that the alpha particle and daughter nucleus move in opposite 𝑁𝑜 = 𝑋 2.6 𝑋 10−6 = 𝑋 10−17 = 𝟏. 𝟐𝟎𝟒 𝑿 𝟏𝟎𝟏𝟕 𝒂𝒕𝒐𝒎𝒔
directions. 13 13
(b) The initial activity R0
 In magnitude, 𝑚𝜶 𝑣𝜶 = 𝑚𝑻𝒉 𝑣𝑻𝒉 . Hence the velocity of 𝛼-particle, 0.6931 0.6931
𝑚 𝑇ℎ 𝑅𝑜 = 𝜆 𝑁𝑜 = 𝑁𝑜 = 𝑋 1.204 𝑋 1017
𝑣𝜶 = 𝑣𝑻𝒉 − − − − − (1) 𝑇1 600
𝑚𝜶 2
 The ratio of the kinetic energy of α particle to that of the daughter nucleus 𝑅𝑜 = 0.6931𝑋 0.2006 𝑋1017 𝑋1017 = 0.1390 𝑋 1017 𝑋 10−2
1 𝑹𝒐 = 𝟏. 𝟑𝟗 𝑿 𝟏𝟎𝟏𝟒 𝒅𝒊𝒔𝒊𝒏𝒕𝒆𝒈𝒓𝒂𝒕𝒊𝒐𝒏/ 𝒔 = 𝟏. 𝟑𝟗 𝑿 𝟏𝟎𝟏𝟒 𝑩𝒒
𝐾𝐸𝛼 𝑚𝛼 𝑣𝛼2 𝑚𝛼 𝑣𝛼2
= 2 = Since , 1 𝐶𝑖 = 3.7 𝑋 1010 𝐵𝑞 we have,
𝐾𝐸𝑇ℎ 1 2 𝑚 𝑣 2
𝑚 𝑣 𝑇ℎ 𝑇ℎ 1.39 𝑋 1014 13900 139000
2 𝑇ℎ 𝑇ℎ 𝑅𝑜 = 𝐶𝑖 = = = 𝟑. 𝟕𝟓𝟔 𝑿 𝟏𝟎𝟑 𝑪𝒊
3.7 𝑋 10 10 3.7 37
 Put equation (1) , we get,
𝐾𝐸𝛼 𝑚𝛼 𝑚𝑇ℎ 2 (c) Activity after 2 hours
= 2
( 𝑣 𝑻𝒉 ) 1 𝑛
𝑡
1 𝑇1/2
2𝑋3600
1 600 1 12
𝐾𝐸𝑇ℎ 𝑚𝑇ℎ 𝑣𝑇ℎ 𝑚𝜶
𝑅 = ( ) 𝑅𝑜 = ( ) 𝑋 𝑅𝑜 = ( ) 𝑋 3.756 𝑋 103 = ( ) 𝑋 3.756 𝑋 103
𝐾𝐸𝛼 𝑚 𝑇ℎ 228.02871 2 2 2 2
= = = 57 1
𝐾𝐸𝑇ℎ 𝑚𝜶 4.002603 𝑹= 𝑋 3756 = 𝟎. 𝟗𝟏𝟔𝟗 𝑪𝒊
𝐾𝐸𝛼 = 57 𝐾𝐸𝑇ℎ − − − − (2) 4096
 The kinetic energy of products (d) Mean life,
𝑇1
𝐾𝐸𝛼 + 𝐾𝐸𝑇ℎ = 5.41 𝑀𝑒𝑉 2
57 𝐾𝐸𝑇ℎ + 𝐾𝐸𝑇ℎ = 5.41 𝑀𝑒𝑉 𝜏 =
0.6931
58 𝐾𝐸𝑇ℎ = 5.41 𝑀𝑒𝑉 600
5.41 𝑀𝑒𝑉 𝜏= = 865.8 𝑠
0.6931
𝑲𝑬𝑻𝒉 = = 𝟎. 𝟎𝟗𝟑 𝑴𝒆𝑽
58
𝑲𝑬𝜶 = 57 𝑋 0.093 = 𝟓. 𝟑𝟎𝟏 𝑴𝒆𝑽

victory R. SARAVANAN. M.Sc., M.Phil., B.Ed PG ASST [PHYSICS], GBHSS, PARANGIPETTAI - 608 502
12 PHYSICS UNIT –9 ATOMIC PHYSICS AND NUCLEAR PHYSICS COMPLETE GUIDE AND MODEL QUESTION
14. Keezhadi (கீழடி), a small hamlet, has become one of the very important 1 50
𝑡= 2.303 log10 ( )
archeological places of Tamilnadu. It is located in Sivagangai district. A lot of 3.832 𝑋 10 −12 38
artefacts (gold coins, pottery, beads, iron tools, jewellery and charcoal, etc.) 1
𝑡= 2.303 log10 (1.316)
have been unearthed in Keezhadi which have given substantial evidence that an 3.832 𝑋 10−12
ancient urban civilization had thrived on the banks of river Vaigai. To determine 1
𝑡= 𝑋 2.303 𝑋 0.1193
the age of those materials, the charcoal of 200 g sent for carbon dating is given 3.832 𝑋 10−12
in the following figure (b). The activity of 𝟏𝟒𝟔 𝑪 is found to be 38 decays/s. 2.303 𝑋 0.1193 𝑋 1012
𝑡=
Calculate the age of charcoal. 3.832
-Solution : 𝑅 = 38 𝑑𝑒𝑐𝑎𝑦𝑠/𝑠 𝑡 = 7.170 𝑋 10−2 𝑋1012 = 7.170 𝑋 1010 𝑠
 In years,
7.170 𝑋 1010
𝑡= 𝑦𝑒𝑎𝑟𝑠
365.24 𝑋 24 𝑋 60 𝑋 60
10
7.170 𝑋 10
𝑡= 𝑦𝑒𝑎𝑟𝑠
3.156 𝑋 107
3
7.170 𝑋 10
𝑡= 𝑦𝑒𝑎𝑟𝑠
3.156
3
𝑡 = 2.272 𝑋 10 𝑦𝑒𝑎𝑟𝑠
 The atomic mass of carbon is 12. Therefore, 12 g of 𝟏𝟐𝟔 𝑪 contains Avogadro 𝒕 = 𝟐𝟐𝟕𝟐 𝒚𝒆𝒂𝒓𝒔 ≈ 𝟐𝟐𝟎𝟎 𝒚𝒆𝒂𝒓𝒔
number (6.02 𝑋 1023 ) of atoms. 15. Calculate the amount of energy released when 1 kg of 𝟐𝟑𝟓
6.02 𝑋 1023 𝟗𝟐 𝑼 undergoes fission
𝟏𝟐
 In 1 g, the number of 𝟔 𝑪 atoms present = reaction.
12
 So the number of 𝟏𝟐𝟔 𝑪 atoms present in 200 𝑔 is -Solution :
6.02 𝑋 1023 12.04  The atomic mass of 𝟐𝟑𝟓 𝟐𝟑𝟓
𝟗𝟐 𝑼 is 235. Therefore, 235 g of 𝟗𝟐 𝑼 contains Avogadro
𝑁𝑜 = 𝑋 200 = 𝑋 1025 = 𝟏 𝑿 𝟏𝟎𝟐𝟓 𝒂𝒕𝒐𝒎𝒔 23
number (6.02 𝑋 10 ) of atoms.
12 12 6.02 𝑋 1023
14 12 −12
 When the tree(sample) was alive, 6 𝐶 : 6 𝐶 = 1.3 𝑋 10 ∶1  In 1 g, the number of 𝟐𝟑𝟓 𝟗𝟐 𝑼 atoms present = 235
 So the total number of carbon-14 atoms is given by  So the number of 𝟐𝟑𝟓
𝑼 atoms present in 235 𝑔 is
𝟗𝟐
𝑵𝒐 = 1 𝑋 1025 𝑋 1.3 𝑋 10−12 = 𝟏. 𝟑 𝑿 𝟏𝟎𝟏𝟑 𝒂𝒕𝒐𝒎𝒔 6.02 𝑋 1023 6.02 𝑋 1026
 Decay constant, 𝑁= 𝑋 1000 = 𝑎𝑡𝑜𝑚𝑠
0.6931 0.6931 235 235
𝟐𝟑𝟓
𝜆 = =  Each 𝟗𝟐 𝑼 nucleus releases 200 MeV of energy during the fission. The total
𝑇1 5730 𝑋 365.24 𝑋 24 𝑋 60 𝑋 60
2 energy released by 1kg of 𝟐𝟑𝟓 𝟗𝟐 𝑼 is
𝜆 = 𝟑. 𝟖𝟑𝟐 𝑿 𝟏𝟎 −𝟏𝟐 −𝟏
𝒔 6.02 𝑋 1026 1204 𝑋 1026
𝑄= 𝑋 200 𝑀𝑒𝑉 = 𝑀𝑒𝑉
 The initial activity , 235 235
𝑅𝑜 = 𝜆 𝑁𝑜 = 3.832 𝑋 10−12 𝑋 1.3 𝑋 1013 𝑸 = 𝟓. 𝟏𝟐𝟑 𝑿 𝟏𝟎𝟐𝟔 𝑴𝒆𝑽
𝑅𝑜 = 4.9816 𝑋 10 = 49.816 ≈ 50 𝒅𝒊𝒔𝒊𝒏𝒕𝒆𝒈𝒓𝒂𝒕𝒊𝒐𝒏/ 𝒔  In terms of joule, [1 𝑒𝑉 = 1.6 𝑋 10−19 𝐽 ]
𝑹𝒐 = 𝟓𝟎 𝑩𝒒 𝑄 = 5.123 𝑋 1026 𝑋 106 𝑋 1.6 𝑋 10−19 𝐽
 Activity of the sample ; 𝑅 = 𝑅𝑜 𝑒 −𝜆𝑡 𝑸 = 𝟖. 𝟏𝟗𝟕 𝑿 𝟏𝟎𝟏𝟑 𝑱
𝑅 𝑅𝑜  In terms of joules [1 𝑘𝑊ℎ = 3.6 𝑋 106 𝐽 ]
= 𝑒− 𝜆 𝑡 (𝑜𝑟) = 𝑒𝜆𝑡 8. 197 𝑋 1013 81.97
𝑅𝑜 𝑅
𝑄 = 6
= 𝑋 107
 Taking log on both sides, we get 3.6 𝑋 10 36
𝑅𝑜 𝑅𝑜 𝑸 = 𝟐. 𝟐𝟕𝟕 𝑿 𝟏𝟎𝟕 𝒌𝑾𝒉
ln ( ) = 𝜆 𝑡 (𝑜𝑟) 2.303 log10 ( ) = 𝜆 𝑡
𝑅 𝑅
1 𝑅𝑜
(𝑜𝑟) 𝑡= 2.303 log10 ( )
𝜆 𝑅

victory R. SARAVANAN. M.Sc., M.Phil., B.Ed PG ASST [PHYSICS], GBHSS, PARANGIPETTAI - 608 502
12 PHYSICS UNIT –9 ATOMIC PHYSICS AND NUCLEAR PHYSICS COMPLETE GUIDE AND MODEL QUESTION
 From additive law,
EXERCISE PROBLEMS WITH SOLUTIONS
5𝑅𝑐 3𝑅𝑐
1. Consider two hydrogen atoms HA and HB in ground state. Assume that hydrogen 𝜈3 →2 + 𝜈2 →1 = +
36 4
atom HA is at rest and hydrogen atom HB is moving with a speed and make head- 5 𝑅 𝑐 + 27 𝑅 𝑐 32 𝑅 𝑐
on collision with the stationary hydrogen atom H A. After the collision, both of 𝜈3 →2 + 𝜈2 →1 = =
36 36
them move together. What is minimum value of the kinetic energy of the moving 8 𝑅𝑐
hydrogen atom HB, such that any one of the hydrogen atoms reaches first 𝜈3 →2 + 𝜈2 →1 =
9
excitation state. 𝝂𝟑 →𝟐 + 𝝂𝟐 →𝟏 = 𝝂𝟑 →𝟏
-Solution : 3. (a) A hydrogen atom is excited by radiation of wavelength 97.5 nm. Find the
 The collision between atom is elastic and during elastic collision, there is no loss principal quantum number of the excited state. (b) Show that the total number
𝒏(𝒏−𝟏)
in kinetic energy. (i.e.) of lines in emission spectrum is Compute the total number of possible
𝟐
Total K. E before collision = Total K.E after collision
1 1 1 1 lines in emission spectrum as given in (a).
𝑚𝐴 𝑢𝐴2 + 𝑚𝐵 𝑢𝐵2 = 𝑚𝐴 𝑣𝐴2 + 𝑚𝐵 𝑣𝐵2 -Solution : 𝜆 = 97.5 𝑛𝑚 = 97.5 𝑋 10−9 𝑚
2 2 2 2 (a) Energy required to excite the atom to nth level,
 Here, 𝑚𝐴 = 𝑚𝐵 = 𝑚 ; 𝑢𝐴 = 0 ; 𝑢𝐵 = 𝑢𝑖 ; 𝑣𝐴 = 𝑣𝐵 = 𝑣𝑓 . Then ℎ𝑐 6.626 𝑋 10−34 𝑋 3 𝑋 108
1 1 1 𝐸 = ℎ 𝜈 = = 𝐽
0 + 𝑚𝑢𝑖2 = 𝑚𝑣𝑓2 + 𝑚𝑣𝑓2 𝜆 97.5 𝑋 10−9
2 2 2 6.626 𝑋 10 −34
𝑋 3 𝑋 10 8
1 2
1 2 (𝑜𝑟) 𝐸= 𝑒𝑉
𝑚𝑢𝑖 = 2 ( 𝑚𝑣𝑓 ) 97.5 𝑋 10 𝑋 1.6 𝑋 10−19
−9
2 2 19.878 𝑋 10 2
1987.8
𝐸𝑖 = 2 𝐸𝑓 − − − − − − − − − (1) 𝐸= 𝑒𝑉 = 𝑒𝑉
156 156
 We know that, for hydrogen atom 𝐸 = 12.74 𝑒𝑉
Energy of electron in Ground state (n=1) = −13.6 𝑒𝑉
 For hydrogen atom, the ground state energy ; 𝐸1 = − 13.6 𝑒𝑉
Energy of electron in First excited state (n=2) = −3.4 𝑒𝑉
 If 𝐸𝑛 be the energy of nth state, then the excitation energy
Hence first excitation energy ; 𝐸𝑓 = −3.4 − (−13.6) = 10.2 𝑒𝑉
𝐸 = 𝐸𝑛 − 𝐸1
 The minimum value of the kinetic energy of the moving hydrogen atom HB (𝑜𝑟) 𝐸𝑛 = 𝐸 + 𝐸1 = 12.74 + (−13.6) = 12.74 − 13.6
𝑬𝒊 = 2 𝐸𝑓 = 2 𝑋 10.2 = 𝟐𝟎. 𝟒 𝒆𝑽 𝐸𝑛 = − 0.86 𝑒𝑉
2. In the Bohr atom model, the frequency of transitions is given by the following  From Bohr’s postulate, the energy of hydrogen atom in nth state,
𝟏 𝟏
expression 𝝂 = 𝑹 𝒄 [ 𝟐 − 𝟐] 𝐰𝐡𝐞𝐫𝐞 𝒏 < 𝒎. 13.6
𝒏 𝒎 𝐸𝑛 = − 2 𝑒𝑉
Consider the following transitions: Show that the 𝑛
13.6 13.6
frequency of these transitions obey sum rule 2
(𝑜𝑟) 𝑛 = − 𝑒𝑉 = − 𝑒𝑉
(which is known as Ritz combination principle) 𝐸𝑛 (− 0.86 𝑒𝑉)
-Solution : 13.6
𝑛2 = = 15.81 ≈ 16
 For transition 3 → 2 ; 𝒎 = 𝟑 ; 𝒏 = 𝟐 . Hence 0.86
∴ 𝒏=𝟒
the frequency.
1 1 1 1 5𝑅𝑐 (b) Total number of possible transitions,
𝜈3 →2 = 𝑅 𝑐 [ 2 − 2 ] = 𝑅 𝑐 [ − ] = 𝑛(𝑛 − 1) 4(4 − 1) 4 𝑋 3 12
2 3 4 9 36 = = = =6
 For transition 2 → 1 ; 𝒎 = 𝟐 ; 𝒏 = 𝟏 . Hence the 2 2 2 2
Thus the possible transitions,
frequency 1) 4 → 3
1 1 1 3𝑅𝑐
𝜈2 →1 = 𝑅 𝑐 [ 2 − 2 ] = 𝑅 𝑐 [1 − ] = 2) 4 → 2
1 2 4 4 3) 4 → 1
 For transition 3 → 1 ; 𝒎 = 𝟑 ; 𝒏 = 𝟏 . Hence the frequency,
4) 3 → 2
1 1 1 8𝑅𝑐
𝜈3 →1 = 𝑅 𝑐 [ 2 − 2 ] = 𝑅 𝑐 [1 − ] = 5) 3 → 1
1 3 9 9 6) 2 → 1

victory R. SARAVANAN. M.Sc., M.Phil., B.Ed PG ASST [PHYSICS], GBHSS, PARANGIPETTAI - 608 502
12 PHYSICS UNIT –9 ATOMIC PHYSICS AND NUCLEAR PHYSICS COMPLETE GUIDE AND MODEL QUESTION
4. Calculate the radius of the earth if density of the earth is equal to the density of 6. Half lives of two radioactive elements A and B are 20 minutes and 40 minutes
the nucleus. [mass of earth 𝟓. 𝟗𝟕 𝑿 𝟏𝟎𝟐𝟒 𝒌𝒈]. respectively. Initially, the samples have equal number of nuclei. Calculate the
-Solution :- 𝑀𝐸 = 5.97 𝑋 1024 𝑘𝑔 ratio of decayed numbers of A and B nuclei after 80 minutes.
𝐴 𝐵
 Given, 𝜌𝐸 = 𝜌𝑁 = 2.3 𝑋 10 𝑘𝑔 𝑚 17 −3 -Solution :- 𝑇1/2 = 20 min ; 𝑇1/2 = 40 min ; 𝑡 = 80 𝑚𝑖𝑛
 Density of earth,  Let the initial number of nuclei in sample A and B be = 𝑁𝑜
𝑀𝐸 𝑀𝐸  Number of half lives for sample A ; 𝑛𝐴 = 𝐴 =
𝑡 80
=4
𝜌𝐸 = = 𝑇 20
𝑉𝐸 4 1/2
𝜋 𝑅𝐸3 𝑡 80
3 Number of half lives for sample B ; 𝑛𝐵 = 𝐵 = =2
3
𝑀 𝐸 𝑀 𝐸 𝑇1/2 40
∴ 𝑅𝐸 = =
4 4  Then the number of nuclei remains undecayed in the samples after 80 minutes,
𝜋 𝜌𝐸 𝜋 𝜌𝑁
3 3
24 24
1 𝑛𝐴 1 4 𝑁𝑜
5.97 𝑋 10 3 𝑋 5.97 𝑋 10 𝑁 𝐴 = ( ) 𝑁 𝑜 = ( ) 𝑁𝑜 =
3
𝑅𝐸 = = 2 2 16
4 4 𝑋 3.14 𝑋 2.3 𝑋 1017 1 𝑛𝐵 1 2 𝑁𝑜
𝑋 3.14 𝑋 2.3 𝑋 1017
3 𝑁𝐵 = ( ) 𝑁𝑜 = ( ) 𝑁𝑜 =
3 𝑋 5.97 𝑋 10 24
17.91 𝑋10 7
179.1 𝑋10 6 2 2 4
𝑅𝐸3 = 17
= =  Thus the number of nuclei decayed in the samples after 80 minutes,
4 𝑋 3.14 𝑋 2.3 𝑋 10 28.888 28.888 𝑁𝑜 16 𝑁𝑜 − 𝑁𝑜 15 𝑁𝑜
1 1
179.1 𝑋10 6 3
179.1 3 𝑁𝑜 − 𝑁𝐴 = 𝑁𝑜 − = =
∴ 𝑅𝐸 = [ ] = [ ] 𝑋 102 16 16 16
21.888 21.888 𝑁𝑜 4 𝑁𝑜 − 𝑁𝑜 3 𝑁𝑜
𝑁𝑜 − 𝑁𝐵 = 𝑁𝑜 − = =
𝑅𝐸 = 1. 837𝑋 102 = 183.7 𝑚 4 4 4
𝑹𝑬 = 𝟏𝟖𝟒 𝒎  Hence the ratio of the number of nuclei decayed in the samples A and B ,
5. Calculate the mass defect and the binding energy per nucleon of the 𝟒𝟕𝑨𝒈 𝟏𝟎𝟖 15 𝑁 𝑜
𝑁𝑜 − 𝑁𝐴 ( ) 15 𝑁𝑜 4 5
16
nucleus. [atomic mass of Ag =107.905949 u] = = 𝑋 =
𝑁𝑜 − 𝑁𝐵 3𝑁 16 3 𝑁𝑜 4
-Solution :- 𝑍 = 47 ; 𝐴 = 108 ; 𝑁 = 𝐴 − 𝑍 = 61 ; 𝑀𝐻𝑔 = 107.905949 ( 𝑜)
4
 In terms of this atomic mass unit, (𝑵𝒐 − 𝑵𝑨 ) ∶ (𝑵𝒐 − 𝑵𝑩 ) = 𝟓 ∶ 𝟒
the mass of the neutron = 1.008665 u, 7. On your birthday, you measure the activity of the sample 210Bi which has a half-
the mass of the proton = 1.007276 u, life of 5.01 days. The initial activity that you measure is 1μCi . (a) What is the
the mass of the hydrogen atom = 1.007825 u approximate activity of the sample on your next birthday? Calculate (b) the
 Mass defect , decay constant (c) the mean life (d) initial number of atoms.
∆𝑚 = 𝑍 𝑚𝐻 + 𝑁𝑚𝑛 − 𝑀𝐻𝑔 -Solution :- 𝑇1/2 = 5.01 𝑑𝑎𝑦𝑠 ; 𝑅𝑜 = 1 𝜇 𝐶𝑖 ; 𝑡 = 1 𝑦𝑒𝑎𝑟 = 365 𝑑𝑎𝑦𝑠
∆𝑚 = (47 𝑋 1.007825) + (61𝑋1.008665) − 107.905949  Number of half-life periods,
∆𝑚 = 47.367775 + 61.528565 − 107.905949 𝑡 365
𝑛= = ≈ 73
∆𝑚 = 108.89634 − 107.905949 𝑇1/2 5.01
∆𝒎 = 𝟎. 𝟗𝟗𝟎𝟑𝟗𝟏 𝒖 (a) The activity of given sample after 73 half-life periods (i.e.) after 1 years
 Binding energy, 1 𝑛 1 73 1
𝐵𝐸 = ∆𝑚 𝑋 931 𝑀𝑒𝑉 = 0.990391 𝑋 931 𝑀𝑒𝑉 𝑅 = ( ) 𝑅 𝑜 = ( ) 1 𝜇 𝐶𝑖 = 73 𝜇 𝐶𝑖
2 2 2
𝑩𝑬 = 𝟗𝟐𝟐. 𝟎𝟓𝟒𝟎𝟐𝟏 𝑴𝒆𝑽 𝑅 = 1.064 𝑋 10−22
 Binding energy per nucleon, 𝑹 ≈ 𝟏𝟎−𝟐𝟐 𝝁 𝑪𝒊
𝐵𝐸 922.054021 (b) Decay constant,
̅̅̅̅
𝐵𝐸 = =
𝐴 108 0.6931 0.6931 0.6931
̅̅̅̅ = 𝟖. 𝟓𝟑𝟗 𝑴𝒆𝑽
𝑩𝑬 𝜆 = = 𝑑𝑎𝑦 −1 = 𝑠 −1
𝑇1/2 5.01 5.01 𝑋 24 𝑋 60 𝑋 60
0.6931 −1
𝜆 = 𝑠
432864
𝝀 = 𝟏. 𝟔𝟎𝟏 𝑿 𝟏𝟎−𝟔 𝒔−𝟏

victory R. SARAVANAN. M.Sc., M.Phil., B.Ed PG ASST [PHYSICS], GBHSS, PARANGIPETTAI - 608 502
12 PHYSICS UNIT –9 ATOMIC PHYSICS AND NUCLEAR PHYSICS COMPLETE GUIDE AND MODEL QUESTION
(c) Mean life period,  Hence.
𝑇1/2 5.01 Energy released per fission X N = Energy produced per second (power)
𝜏= =
0.6931 0.6931 200 X 106 X 1.6 X 10−19 X N = 1
𝝉 = 𝟕. 𝟐𝟐𝟖 𝒅𝒂𝒚𝒔 1 1
N = = 𝑋 1013
(d) If 𝑁𝑜 be the number of atoms present in the sample initially, 6
200 X 10 X 1.6 X 10 −19 320
𝑅𝑜 = 𝜆 𝑁𝑜 N = 3.125 𝑋10−3 𝑋1013
𝑅𝑜 1 𝜇 𝐶𝑖 𝑵 = 𝟑. 𝟏𝟐𝟓 𝑿𝟏𝟎𝟏𝟎 𝒂𝒕𝒐𝒎𝒔
∴ 𝑁𝑜 = =
𝜆 1.601 𝑋 10−6 10. Show that the mass of radium ( 𝟐𝟐𝟔 𝟖𝟖𝑹𝒂 ) with an activity of 1 curie is almost a
−6 10
1𝑋 10 𝑋 3.7 𝑋10 gram. Given T1/2 =1600 years.
𝑁𝑜 = -Solution :- 𝑅 = 1 𝐶𝑖 = 3.7 𝑋 1010 disintegration/s
1.601 𝑋 10−6
3.7 𝑋1010  From law of disintegration,
𝑁𝑜 = dN
1.601 = λN (or) 𝑅 = λ N
𝟏𝟎
𝑵𝒐 = 𝟐. 𝟑𝟏𝟏 𝑿 𝟏𝟎 𝒂𝒕𝒐𝒎𝒔 dt
8. Calculate the time required for 60% of a sample of radon undergo decay. R R T1/2
N = = = 𝑋𝑅
Given T1/2 of radon =3.8 days λ 0.6931 0.6931
( )
-Solution : 𝑁𝑜 = 100 % ; 𝑁 = 100 − 60 = 40 % ; 𝑇1/2 = 3.8 𝑑𝑎𝑦𝑠 T1/2
 From law of disintegration, the amount of sample left un decayed, 1600 X 365 X 24 X 60 X 60
N= X 3.7 X 1010
𝑁 = 𝑁𝑜 𝑒 −𝜆𝑡 0.6931
40 = 100 𝑒 − 𝜆 𝑡 N = 2.694 X 1011 X 1010
4 N = 2.694 X 1021 atoms
= 𝑒− 𝜆 𝑡
10  The atomic mass of 𝑟𝑎𝑑𝑖𝑢𝑚 is 226. Therefore, 226 g of 𝟐𝟐𝟔 𝟖𝟖𝑹𝒂 contains Avogadro
10 number (6.02 𝑋 10 23
) of atoms.
(𝑜𝑟) 𝑒𝜆𝑡 = = 2.5
4 (i.e.) mass of 6.02 𝑋 1023 atoms = 226 g
 Taking log on both sides, 226
Hence, mass of one atom = 𝑔
𝜆 𝑡 = log 𝑒 2.5 6.023 X 1023
23
(𝑜𝑟) 6.02 𝑋 10
𝜆 𝑡 = 2.303 𝑋 log10 2.5  In 1 g, the number of 𝟐𝟐𝟔 𝟖𝟖𝑹𝒂 atoms present =
1 235
𝑡 = 𝑋 2.303 𝑋 0.3979  Hence, total mass of 2.694 X 1021 atoms with an activity of 1 𝐶𝑖 is,
𝜆 226
1 𝑚 = 𝑋 2.694 𝑋 1021
𝑡= 𝑋 2.303 𝑋 0.3979 6.023 X 1023
0.6931
(
𝑇1/2
) 226 𝑋 2.694 𝑋 10−2
𝑚 =
𝑇1 6.023
2 𝑚 = 1.012 𝑋 102 𝑋 10−2 = 1.012 𝑔
𝑡= 𝑋 2.303 𝑋 0.3979
0.6931 𝒎 ≈𝟏𝒈
3.8 11. Charcoal pieces of tree is found from an archeological site. The carbon-14
𝑡= 𝑋 2.303 𝑋 0.3979
0.6931 content of this charcoal is only 17.5% that of equivalent sample of carbon from a
𝒕 = 𝟓. 𝟎𝟐𝟓 𝒅𝒂𝒚𝒔 living tree. What is the age of tree?
9. Assuming that energy released by the fission of a single 𝟐𝟑𝟓 𝟗𝟐 𝑼 nucleus is 200MeV, -Solution :- 𝑇1 = 5730 years, 𝑁𝑜 = 100 % ; 𝑁 = 17.5 %
calculate the number of fissions per second required to produce 1 watt power. 2

-Solution :-  From law of disintegration,


 Energy released per fission = 200 MeV = 200 X 106 X 1.6 X 10−19 J 𝑁 = 𝑁𝑂 𝑒 −𝜆𝑡
 Energy produced per second (i.e.) power = 1W = 1 J/s 17.5 = 100 𝑒 −𝜆𝑡
17.5
 Let Number of fissions per second f = N (say) = 𝑒− 𝜆 𝑡
100
100
(𝑜𝑟) = 𝑒 𝜆𝑡
17.5
victory R. SARAVANAN. M.Sc., M.Phil., B.Ed PG ASST [PHYSICS], GBHSS, PARANGIPETTAI - 608 502
12 PHYSICS UNIT –9 ATOMIC PHYSICS AND NUCLEAR PHYSICS COMPLETE GUIDE AND MODEL QUESTION
 Taking log on both sides,
100 CONCEPTUAL QUESTIONS AND ANSWERS
log 𝑒 ( ) = 𝜆𝑡
17.5 1. Which observation of Rutherford’s  -rays scattering experiments leads to the
100 conclusion that the atom contains a lot of empty space?
2.303 𝑋 log10 ( )= 𝜆𝑡
17.5  A large number positively charged  -particles were found to pass through gold
1 100 foil undeviated. It is possible only if the atom contains a lot of empty space.
∴ 𝑡= 𝑋 2.303 𝑋 log10 ( )
𝜆 17.5  Some of them scattered through large angles, which conclude that the positive
1 charge of the atom is concentrated at the centre of the atom called the nucleus
𝑡= 𝑋 2.303 𝑋 ( log10 100 − log10 17.5)
0.6931 2. What do you mean by a stationary orbits as postulated by Bohr?
( )  A stationary orbit or a non- radiating orbit or the permited orbit is one in which
𝑇1
2 the electron does not radiate any energy inspite of the fact that it undergoes an
𝑇1/2
𝑡 = 𝑋 2.303 𝑋(2.000 − 1.2430) accelerated motion,.
0.6931 3. How many electronic orbits are there in a hydrogen atom?
5730
𝑡 = 𝑋 2.303 𝑋 0.757  Theoritically there are infinite number of orbits.
0.6931  Since there is only one electron in the hydrogen atom, only one of these will be
𝒕 = 𝟏. 𝟒𝟒𝟏 𝑿 𝟏𝟎𝟒 years
occupied at one time. Rest of them will be vacant.
4. The total energy of an electron while revolving in a stationary orbit is negative.
What is the concept of this negative energy?
 A particle possessing negative energy is said to be in the bound state. That means
some energy has to be supplied to it in order to liberate it from that bond
 Hence the negative energy of electron shows the nucleus and electron are bound
by electrostatic force of attraction and that energy must be supplied to detach an
electron and it is called the binding energy of the electron.
5. If ‘𝒙’ is the radius of first orbit of an atom, what will be the radius of second
orbit of same atom?
𝟎 𝜺 𝒉𝟐 𝒏𝟐 𝒏𝟐
 The radius of nth orbit of an atom; 𝒓𝒏 = 𝝅𝒎𝒆 𝟐 [ 𝒁 ] = 𝒂𝟎 [ 𝒁 ]

𝟏 𝟎
 For first orbit, 𝑛 = 1 ; 𝒓𝟏 = 𝒂𝟎 [𝒁] = 𝒙 (ℎ𝑒𝑟𝑒 𝒂𝟎 = 𝟎. 𝟓𝟐𝟗 𝑨)
𝟐𝟐 𝟒
 For second orbit, 𝑛 = 2 ; 𝒓𝟐 = 𝒂𝟎 [ 𝒁 ] = 𝒂𝟎 [𝒁] = 𝟒 𝒓𝟏 = 𝟒𝒙
6. Will the energy of electron in hydrogen atom be more in first orbit or second
orbit? Obtain the ratio of two energies?
 The energy of electron in nth orbit hydrogen atom ;
𝒎𝒆𝟒 𝟏 𝟏
𝑬𝒏 = − [ ] 𝒋𝒐𝒖𝒍𝒆 (or) 𝑬𝒏 = −𝟏𝟑. 𝟔 [ 𝟐] 𝒆𝑽
𝟖 𝜺𝟎𝟐 𝒉𝟐 𝒏𝟐 𝒏
 For first orbit, 𝑛 = 1 ; 𝑬𝟏 = −𝟏𝟑. 𝟔 𝒆𝑽
𝟏 𝟏
 For second orbit, 𝑛 = 2 ; 𝑬𝟐 = −𝟏𝟑. 𝟔 [ 𝟐 ] 𝒆𝑽 = −𝟏𝟑. 𝟔 𝑿 = −𝟑. 𝟒𝒆𝑽
𝟐 𝟒
Hence 𝑬𝟐 > 𝑬𝟏 (i.e.) energy of electron in second orbit is more than first orbit
Thus the ratio is, 𝑬𝟏 ∶ 𝑬𝟐 = 𝟏 ∶ 𝟒
7. What is the frequency condition?
 The frequency of radiation emitted by an electron when it jumps from the orbit
𝑬 −𝑬
of higher enegy to lower energy is; 𝝂 = 𝟐 𝟏
𝒉

victory R. SARAVANAN. M.Sc., M.Phil., B.Ed PG ASST [PHYSICS], GBHSS, PARANGIPETTAI - 608 502
12 PHYSICS UNIT –9 ATOMIC PHYSICS AND NUCLEAR PHYSICS COMPLETE GUIDE AND MODEL QUESTION
8. How much energy in joule, is require to shift an electron of hydrogen atom 14. Energy levels A, B and C of a certain atom corresponding to increasing values of
from 3rd orbit to 4th orbit? energy (i.e.) 𝑬𝑨 < 𝑬𝑩 < 𝑬𝑪 . If 𝝀𝟏 , 𝝀𝟐 𝐚𝐧𝐝 𝝀𝟑 are the wavelengths of radiation
𝟏 corresponding to the transisions C to B, B to A and C to A respectively, then find
 The energy of electron in nth orbit hydrogen atom ; 𝑬𝒏 = −𝟏𝟑. 𝟔 [𝒏𝟐] 𝒆𝑽
the relation between the wavelengths?
𝟏 𝟏
 For 3rd orbit, 𝑛 = 3 ; 𝑬𝟑 = −𝟏𝟑. 𝟔 [𝟑𝟐] 𝒆𝑽 = −𝟏𝟑. 𝟔 𝑿 𝟗 = − 𝟏. 𝟓𝟏𝒆𝑽  According to Bohr’s second postulate,
𝟏 𝟏 𝒉𝒄
 For 4th orbit, 𝑛 = 4 ; 𝑬𝟒 = −𝟏𝟑. 𝟔 [𝟒𝟐] 𝒆𝑽 = −𝟏𝟑. 𝟔 𝑿 𝟏𝟔 = − 𝟎. 𝟖𝟓𝒆𝑽 𝑬𝑪 − 𝑬𝑩 = 𝒉𝝂𝟏 =
𝝀𝟏
 Since 𝑬𝟑 < 𝑬𝟒 , energy must require to shift electron from 3rd to 4th orbit and it 𝒉𝒄
𝑬𝑩 − 𝑬𝑨 = 𝒉𝝂𝟐 =
is given by ; 𝐸1 = 𝐸4 − 𝐸3 = −0.85 − (−1.51) = 0.66 𝑒𝑉 𝝀𝟐
 But, 1 𝑒𝑉 = 1.6 𝑋 10−19 𝐽 ∴ 𝑬𝟏 = 𝟎. 𝟔𝟔 𝑿 𝟏. 𝟔 𝑿 𝟏𝟎−𝟏𝟗 = 𝟏. 𝟎𝟓𝟔 𝑿 𝟏𝟎−𝟏𝟗 𝑱 𝒉𝒄
9. There is only one electron in hydrogen atom. When we observe a number of 𝑬𝑪 − 𝑬𝑨 = 𝒉𝝂𝟑 =
𝝀𝟑
series each consisting of a number of lines while observing the hydrogen atom.  From the figure, (𝑬𝑪 − 𝑬𝑩 ) + (𝑬𝑩 − 𝑬𝑨 ) = (𝑬𝑪 − 𝑬𝑨 )
How would you explain this? 𝒉𝒄 𝒉𝒄 𝒉𝒄 𝟏 𝟏 𝟏 𝝀 𝝀
 Hydrogen spectrum is observed by examining the light emitted when a discharge (or)
𝝀𝟏
+ 𝝀 = 𝝀 (or)
𝝀
+ 𝝀 = 𝝀 (or) 𝝀𝟑 = 𝝀 𝟏+ 𝝀𝟐
𝟐 𝟑 𝟏 𝟐 𝟑 𝟏 𝟐
pass through a tube containg hydrogen gas. The tube contains millions and 15. Find the ratio of energies of the hydrogen atom in its first to second excited
trillions of atoms in different states of excitation. These atoms unergo transition state?
from a higher orbit to lower one in a random manner, thus giving rise to a 𝟏
 The energy of electron in nth orbit hydrogen atom ; 𝑬𝒏 = −𝟏𝟑. 𝟔 [𝒏𝟐] 𝒆𝑽
number of lines.
𝟏 𝟏
10. Balmer series of hydrogen atom is photographed in the form a number of lines  For first excited state, 𝑛 = 2 ; 𝑬𝟐 = − 𝟏𝟑. 𝟔 [𝟐𝟐] 𝒆𝑽 = − 𝟏𝟑. 𝟔 [𝟒] 𝒆𝑽
whose wavelength increase from left to right. Which of two ; extreme right or 𝟏 𝟏
extreme left line will be the brightest?  For second excited state, 𝑛 = 3 ; 𝑬𝟑 = − 𝟏𝟑. 𝟔 [𝟑𝟐] 𝒆𝑽 = − 𝟏𝟑. 𝟔 [𝟗] 𝒆𝑽
 In case of Balmer series, that line is brightest for which the probability of 𝑬 9
 Therefore, 𝑬𝟐 = 4 (or) 𝑬𝟐 ∶ 𝑬𝟑 = 𝟗 ∶ 𝟒
transition is maximum. This is maximum for electrons going form 3 to 2 orbit.
rd nd 𝟑
𝟎
This radiation possesses the minimum energy (i.e.) minimum frequency and 16. Wavelength of the first line of the Balmer series of hydrogen is 𝟔𝟏 𝐀 . Find the
hence maximum wavelength. wavelength of the second line of this series?
 Therefore the extreme right line will be brightest, if the wavelength increases  The wave number 𝜈̅ (i.e..) reciprocal of wavelength 𝜆 of spectral line in Balmer
from left to right. 1 1 1 1 1
series (𝑛 = 2) is ; 𝜈̅ = = 𝑅 [ 2 − 2 ] = 𝑅 [ 2 − 2 ]
11. What is the minimum possible wavelength of a line in hydrogen spectrum? 𝜆 𝑛 𝑚 2 𝑚
1 1 1 5𝑅 36
 Wavelength of that line will be minimum for which the energy of trasition is  For 1st line of Balmer series, 𝑚 = 3 ; 𝜆 = 𝑅 [22 − 32] = 36 (or) 𝜆1 = 5 𝑅
maximum and this is for an electron undergoing transition from infinity distant 1
1
1 1 3𝑅 16
orbit to 1st orbit, thereby emitting energy 13.6 𝑒𝑉 (i.e.)  For 2nd line of Balmer series, 𝑚 = 4 ; = 𝑅 [ 2 − 2] =
𝜆2 2 4 16
(or) 𝜆2 =
3𝑅
ℎ𝑐
𝐸∞ − 𝐸1 = ℎ𝜈𝑚𝑎𝑥 = = 13.6 𝑒𝑉 𝜆 16 5𝑅 20 𝟐𝟎 𝟐𝟎 𝟎 𝟎
𝒉𝒄
𝜆𝑚𝑖𝑛
𝟏𝟐𝟓𝟎 𝒆𝑽 𝒏𝒎
 Hence, 2 = 𝑋 = (or) 𝝀𝟐 = 𝝀𝟏 = 𝑿 𝟔𝟓𝟔𝟏 𝐀 = 𝟒𝟖𝟔𝟎𝐀
𝜆1 3𝑅 36 27 𝟐𝟕 𝟐𝟕
(𝑜𝑟) 𝝀𝒎𝒊𝒏 = = = 𝟗𝟏. 𝟗 𝒏𝒎
𝟏𝟑.𝟔 𝒆𝑽 𝟏𝟑.𝟔 𝒆𝑽 17. From the diagram, when the system moves from ‘𝟐𝑬’ level to’𝑬’ level, a photon
12. Why is the chamber of Millikan’s oil drop method made up of good conductor? of wavelength ‘𝝀’ is emitted. Find the wavelength of photon produced during its
 Collisions of any stray charge in the atmosphere with the experimental oil drop is 𝟒𝑬
likely to affect the result. transistion from level to ‘E’ level.
𝟑
 The good conductor chamber is earthed. Any stray charge striking the chamber  According to Bohr’s second postulate,
ℎ𝑐 ℎ𝑐
goes to earth. Thus the result is not affected. 2𝐸 − 𝐸 = ℎ𝜈 = 𝜆 (or) 𝐸 = 𝜆
13. What is the difference between Rutherford’s and the Bohr’s atom model? 4𝐸 ℎ𝑐 𝐸 ℎ𝑐
 According to Rutherford atom model, electrons can revolve in any orbit and can − 𝐸 = ℎ𝜈 1 = 1 (or) = 1
3 𝜆 3 𝜆
emit radiations of all frequencies 𝐸 ℎ𝑐 𝜆
1
𝜆1 𝟏
 According to Bohr atom model, electron revolve only in allowed stationary orbits ∴
𝐸/3
= 𝜆 𝑋 ℎ 𝑐 (or) 3 = (or) 𝝀 = 𝟑𝝀
𝜆
and it can emit radiations when electrons jumps from stationary orbit of higher
energy to another stationary orbit of lower energy.

victory R. SARAVANAN. M.Sc., M.Phil., B.Ed PG ASST [PHYSICS], GBHSS, PARANGIPETTAI - 608 502
12 PHYSICS UNIT –9 ATOMIC PHYSICS AND NUCLEAR PHYSICS COMPLETE GUIDE AND MODEL QUESTION
18. In any Bohr orbit of the hydrogen atom, what is the ratio of kinetic, potential 23. The velocity of an electron in the second orbit of sodium atom is 𝒗, then find the
and total energy of the electron? velocity of an electron in its fifth orbit?
 According to Bohr’s postulates, the eletron in nth orbit have,  Accoring to Bohr’s postulate, the velocity of electron in nth orbit of an atom is
𝒎𝒆𝟒 𝒁𝟐 𝒉 𝒁 𝒆𝟐 𝒁 𝒁
1) Kinetic energy ; 𝑲𝒏 = [ ] (for Hydrogen, Z =1) 𝒗𝒏 = ( )= ( ) = 𝜶𝒄( )
𝟖 𝜺𝟎𝟐 𝒉𝟐 𝒏𝟐 𝟐 𝝅 𝒎 𝒂𝟎 𝒏 𝟐 𝜺𝟎 𝒉 𝒏 𝒏
𝒎𝒆𝟒 𝒁𝟐 𝜺 𝒉𝟐 𝒐
2) Potentall energy ; 𝑼𝒏 = − 𝟐 𝟐 [ 𝟐] Here, 𝒂 = 𝟎
= 𝟎. 𝟓𝟐𝟗 𝑨 → Bohr’s radius
𝟒 𝜺𝟎 𝒉 𝒏 𝟎 𝝅 𝒎 𝒆𝟐
𝒎𝒆𝟒 𝒁𝟐 𝒆𝟐 𝟏
3) Total energy ; 𝑬𝒏 = 𝑲𝒏 + 𝑼𝒏 = − 𝟐 𝟐 [ ] and 𝜶= ≈ 𝟏𝟑𝟕 → Fine structure constant
𝟖 𝜺𝟎 𝒉 𝒏𝟐 𝟐 𝜺𝟎 𝒉 𝒄
𝟏 𝟏 𝟏 𝟏 𝒗𝟓 𝟐 𝟐 𝟐
 Hence the ratio is ; 𝑲𝒏 ∶ 𝑼𝒏 ∶ 𝑬𝒏 = (𝟖) ∶ (− 𝟒) ∶ (− 𝟖) = 𝟏 ∶ −𝟐 ∶ −𝟏  From the above equation, 𝒗𝒏 ∝ 𝒏
Hence ,
𝒗𝟐
= 𝟓
(or) 𝒗𝟓 = 𝒗𝟐 =
𝟓 𝟓
𝒗
19. What happens to the energy, when a hydrogen atom is raised from the ground 24. Hydrogen atoms are excited from ground state to the excited state of principal
state to an excited state? quantum number 4. Then how many number of spectral lines are observed?
 For ground state, 𝑛 = 1 and for excited states, 𝑛 > 1  According to Bohr’s 2nd postulate, whenever electrons
𝟏 𝟏 𝟏 jumps from orbit of higher energy state to orbit of lower
 Since, 𝑲𝒏 ∝ 𝒏𝟐 ; 𝑼𝒏 ∝ − 𝒏𝟐 and 𝑬𝒏 ∝ − 𝒏𝟐 , when hydrogen atom raised
energy state, a spectral line are emitted.
from ground state to excited state (i.e.) as 𝑛 increases, kinetic energy (𝑲𝒏 )
 The number of spectral lines emitted is
decreases but potential energy (𝑼𝒏 ) increases and total energy (𝑬𝒏 ) increases. 𝒏 (𝒏−𝟏) 𝟒 (𝟒−𝟏) 𝟒 (𝟑)
20. Find the ratio of the frequencies of the long wavelength limits of Lyman and 𝑵𝑬 = 𝟐 = 𝟐 = 𝟐 = 𝟔
Balmer series of hydrogen spectrum? 25. The splitting of spectral line into group under the effect of electric field is called
 In any spectral series, the first transition corresponds to line of longest ________(1)________ effect and magnetic field is called _______(2)__________
wavelength and last transition corresponds to line of shortest wavelength.  (1) Stark effect (2) Zeeman effect
 The wave number 𝜈̅ (i.e..) reciprocal of wavelength 𝜆 of spectral line ; 𝒐
𝝂 𝟏 𝟏 𝟏 𝒄 𝟏 𝟏 26. The wavelength of spectral line of hydrogen atom is found to be 𝟒𝟖𝟔𝟎 𝑨 . Name
̅ = = = 𝑹 [ 𝟐 − 𝟐]
𝝂 (or) 𝝂 = 𝝀 = 𝑹 𝒄 [𝒏𝟐 − 𝒎𝟐] the corresponding spectral series.
𝒄 𝝀 𝒏 𝒎
𝒐
 Hence for Lyman series (𝑛 = 1) for longest wavelength (𝑚 = 2);  The given wavelength 𝟒𝟖𝟔𝟎 𝑨 is in visible range. So this wavelength corresponds
𝒄 𝟏 𝟏 𝟑
𝝂𝑳𝒚𝒎𝒂𝒏 = 𝝀𝒎𝒂𝒙
= 𝑹 𝒄 [𝟏𝟐 − 𝟐𝟐] = 𝟒
𝑹𝒄 to Balmer series.
27. Find the ratio of areas within the electron orbits for the first exicited state to
 Simillarly for Balmer series (𝑛 = 2) for longest wavelength (𝑚 = 3);
𝒄 𝟏 𝟏 𝟓 the ground state for hydrogen atom?
𝝂𝑩𝒂𝒍𝒎𝒆𝒓 = 𝝀𝒎𝒂𝒙
= 𝑹 𝒄 [𝟐𝟐 − 𝟑𝟐] = 𝟑𝟔
𝑹𝒄  According to Bohr’s atom model, 𝑟𝑛 ∝ 𝑛2 and hence 𝐴𝑛 ∝ (𝑟𝑛 )2 ∝ (𝑛2 )2
𝝂𝑳𝒚𝒎𝒂𝒏 3𝑅𝑐 36 𝟐𝟕  For ground state, n = 1 ; 𝐴𝑔𝑟𝑜𝑢𝑛𝑑 ∝ 1 & for first excited state, n = 2 ; 𝐴𝑓𝑖𝑟𝑠𝑡 ∝ 16
 Then the ratio ; 𝝂𝑩𝒂𝒍𝒎𝒆𝒓
= 4
𝑋 5𝑅𝑐 = 𝟓 𝑨𝒇𝒊𝒓𝒔𝒕 𝟏𝟔
 Hence the ratio , 𝑨 = 𝟏 (or) 𝑨𝒇𝒊𝒓𝒔𝒕 ∶ 𝑨𝒈𝒓𝒐𝒖𝒏𝒅 = 𝟏𝟔 ∶ 𝟏
21. The energy required to ionize the helium atom will be 𝟐𝟒. 𝟔 𝒆𝑽, then what is its 𝒈𝒓𝒐𝒖𝒏𝒅
ioninsation potential? 28. An electron in a hydrogen atom makes a trasistion from first excited state to
 Ionisation potential of helium atom will be = 𝟐𝟒. 𝟔 𝑽 ground state. Then how many times the current due to circulating electron
22. Find the ratio of angular momentum of electron revolving in first orbit to changes?
second orbit of hydrogen atom? 𝒆 𝟏
 Current due to circulating electron in nth orbit ; 𝒊𝒏 = 𝑻 (or) 𝒊𝒏 ∝ 𝟑
 According to Bohr’s first postulate, angular momentum is quantized. (i.e.) 𝒏 𝒏
𝒉  For ground state, 𝑛 = 1 ; 𝒊𝟏 ∝ 𝟏
𝑳 = 𝒎 𝒗𝒏 𝒓 𝒏 = 𝒏 ℏ = 𝒏 ( )
𝟐𝝅  For first excited state, 𝑛 = 2 ; 𝒊𝟐 ∝ 𝟏𝟑 ∝ 𝟏𝟖
𝒉 𝟐
 For 1st orbit , (𝑛 = 1) ; 𝐿1 = 𝟐𝝅 𝒊𝟏 1
 Hence, = = 8 (or) 𝒊𝟏 = 𝟖 𝒊𝟐 (i.e) increases 8 times
𝒊𝟐 1/8
 For 2nd orbit , (𝑛 = 2) ; 𝐿2 = 2 ( 𝒉 ) = 𝒉 𝟐𝝅 𝝅
𝐿1 𝒉 𝝅 𝟏
 Hence the ratio ; 𝐿2
=
𝟐𝝅
X
𝒉
=
𝟐
(or) 𝑳𝟏 ∶; 𝑳𝟐 = 𝟏 ∶ 𝟐

victory R. SARAVANAN. M.Sc., M.Phil., B.Ed PG ASST [PHYSICS], GBHSS, PARANGIPETTAI - 608 502
12 PHYSICS UNIT –9 ATOMIC PHYSICS AND NUCLEAR PHYSICS COMPLETE GUIDE AND MODEL QUESTION
29. In Hydrogen atom, an electron makes a transition from 𝒏 = 𝟐 to 𝒏 = 𝟏. What 33. In a hydrogen like atom electron make transition from an energy level with
is the change in magnetic field produced by the circulating electron at the quantum number 𝒏 to another with quantum number (𝒏 − 𝟏). If 𝒏 >> 𝟏, find
nucleus? the relation between frequency of radiation emitted and 𝒏?
𝑒
𝜇 0 ( )  According to Bohr’s postulate, the frequency of radiation emitted is,
 Magnetic field due to orbiting electron ;𝐵𝑛 = 𝜇0𝐼 = (𝑎 𝑇𝑛2) (or) 𝐵𝑛 ∝ 15
2 𝑟𝑛 2 1 1 𝑛2 − (𝑛−1)2 (2 𝑛−1) 2𝑛 𝟏
0𝑛 𝑛
𝜈 = 𝑅 𝑐 [(𝑛−1)2 − 𝑛2 ] = 𝑅 𝑐 [ ] = 𝑅 𝑐 [𝑛2 (𝑛−1)2 ] = 𝑅𝑐 𝑛4 = 𝟐𝑹𝒄 𝒏𝟑
 For ground state, 𝑛 = 1 ; 𝑩𝟏 ∝ 𝟏 𝑛2 (𝑛−1)2
𝟏
 For first excited state, 𝑛 = 2 ; 𝑩𝟐 ∝ 215 ∝ 1
32
 Hence, 𝝂 ∝ 𝒏𝟑
𝑩𝟏 1 34. Ultraviolet light of wavelength 𝝀𝟏 and 𝝀𝟐 , when allowed to fall on hydrogen
 Hence, = = 32 (or) 𝑩𝟏 = 𝟑𝟐 𝑩𝟐 (i.e) increases 32 times
𝑩𝟐 1/32 atoms in their ground state is found to liberate electrons with kinetic energy
30. The wavelengths of X-ray corresponding to 𝑲𝜶 , 𝑲𝜷 , 𝑳𝜶 are 𝝀𝑲𝜶 , 𝝀𝑲𝜷 , 𝝀𝑳𝜶 . Find the 𝑲𝑬𝟏 and 𝑲𝑬𝟐 respectively. Find the value of Plank’s constant?
relation between the wavelengths.  From Einstein’s equation,
𝒉𝒄 𝒉𝒄 𝒉𝒄 𝒉𝒄
 This lines corresponds to characteristic X-ray spectrum and the energy level 𝑲𝑬𝟏 = 𝒉 𝝂𝟏 − 𝒉 𝝂𝟎 = − and 𝑲𝑬𝟐 = 𝒉 𝝂𝟐 − 𝒉 𝝂𝟎 = −
𝝀𝟏 𝝀𝟎 𝝀𝟐 𝝀𝟎
diagram of the atom is shown in the figure. 𝒉𝒄 𝒉𝒄 𝟏 𝟏 𝝀𝟐 − 𝝀𝟏
 According to Bohr’s second postulate, ∴ 𝑲𝑬 𝟏 − 𝑲𝑬𝟐 = − = 𝒉𝒄 [ − ] = 𝒉𝒄 [ ]
𝝀𝟏 𝝀𝟐 𝝀𝟏 𝝀𝟐 𝝀𝟏 𝝀𝟐
𝒉𝒄
𝑬𝑳 − 𝑬𝑲 = 𝒉𝝂𝑲𝜶 = (𝑲𝜶 − 𝒍𝒊𝒏𝒆) (𝑲𝑬𝟏 − 𝑲𝑬𝟐 ) 𝝀𝟏 𝝀𝟐
𝝀𝑲𝜶  Hence Plank’s constant ; 𝒉 = 𝒄 (𝝀𝟐 − 𝝀𝟏 )
𝒉𝒄
𝑬𝑴 − 𝑬𝑲 = 𝒉𝝂𝑲𝜷 = (𝑲𝜷 − 𝒍𝒊𝒏𝒆) 35. Why X -rays are not preferred but electrons are preferred to construct high
𝝀𝑲𝜷
𝒉𝒄 resolving power microscopes?
𝑬𝑴 − 𝑬𝑳 = 𝒉𝝂𝑳𝜶 = (𝑳𝜶 − 𝒍𝒊𝒏𝒆)  The resolving power of a microscope is inversely proportional to the wavelength
𝝀𝑳𝜶
 From the figure, (𝑬𝑴 − 𝑬𝑳 ) + (𝑬𝑳 − 𝑬𝑲 ) = (𝑬𝑴 − 𝑬𝑲 ) of radiation used for illuminating the object under study. Higher magnification as
𝒉𝒄 𝒉𝒄 𝒉𝒄 𝟏 𝟏 𝟏 well as higher resolving power can be obtained by employing the waves of
(or)
𝝀𝑲𝜶
+ 𝝀 = 𝝀 (or)
𝝀𝑲𝜶
+ 𝝀 = 𝝀 shorter wavelengths.
𝑳𝜶 𝑲𝜷 𝑳𝜶 𝑲𝜷
𝒐  The wavelength of X-rays is smaller than that of the visible light, one can think of
31. The wavelength of 𝑲𝜶 line of X-rays produced by X-ray tube is 0.75 𝑨. Find the having X-ray microscope of high resolving power. However X -rays cannot be
atomic number of the anode material of the tube? focussed as visible radiations are focusssed using lenses.
 When electron jumps from L -shell (𝑛 = 2) to K- shell (𝑛 = 1), 𝑲𝜶 line is  On the other hand, electrons having deBroglie wavelength of the order of X-rays
1 1 1 1 1 3
obtained.Hence ; = 𝑅 𝑍2 [ 2 − 2 ] = 𝑅 𝑍2 [ 2 − 2] = 𝑅 𝑍2 [ ] can be focussed easily using electric and magnetic fields and one can build a high
𝜆 𝑛 𝑚 1 2 4
2 1 1 1 16 𝑋 103 resolving powermicroscope using electrons.
(or) 𝑍 = 3 = 3 == 3 3 =
𝜆𝑅[ ]
4
0.75 𝑋 10−10 𝑋 1.097 𝑋 107 𝑋[ ]
4
[ ] 𝑋 10−3 𝑋 1.097 𝑋[ ]
4 4
9 𝑋 1.097 36. Why did it take such a long time for the discovery of neutron?
16 𝑋 10 3 16 𝑋 10 3  Charged particles can easily interact with matter. Neutron being an uncharged
𝑍2 = ≈ ≈ 1600 (or) 𝒁 = 𝟒𝟎
9.873 10 particle, cannot interact with matter so easily. That is why discovery had taken
32. In Rutherford scattering experiment, the number of particles scattered at 𝟔𝟎° is such a long time.
𝟓 𝑿 𝟏𝟎𝟔 . Find the number of particles scattered at 𝟎° ? 37. The density of a nucleus is more than that of the atom containing nucleus. Why?
𝟏
 The number of particles scattered at an angle 𝜃 is, 𝑵 ∝ 𝟒 𝜽  The size of the atom is of the order of 10−10 𝑚 , whereas the size of the nucleus is
𝒔𝒊𝒏 ( ) of the order of 10−14 𝑚 . (i.e.) Size of the atom is 10,000 times larger than the
𝟐
𝟏 𝟏 𝟏 nucleus.
 At an angle 60° ; 𝑵𝟏 ∝ 𝟒 𝟔𝟎° ∝ 𝟒 ∝ 𝟏 𝟒 ∝ (𝟐)𝟒
𝒔𝒊𝒏 ( 𝟐 ) 𝒔𝒊𝒏 𝟑𝟎° ( )  The nucleus at the centre of the atom is packed with heavy particles such as
𝟐
𝟏 𝟏 𝟏 𝟐 𝟒 neutrons and protons, where as the atom contains large amount of empty space
 At an angle 30° ; 𝑵𝟐 ∝ 𝟒 𝟏𝟐𝟎° ∝ 𝟒 ∝ 𝟒 ∝ (√𝟑) around the nucleus in which negligible mass is distributed.
𝒔𝒊𝒏 ( 𝟐 ) 𝒔𝒊𝒏 𝟔𝟎° √𝟑
( )
𝟐  So the density of nucleus is more than that of the atom.
𝟐 𝟒 𝟏 𝟓 38. How it is possible that large number of protons exist in a very small space
 Hence, 𝑵 𝑵𝟏
𝟐
𝟑
𝟏
= (√ ) 𝑿 ( ) 𝟒 =
𝟐
𝟏
𝟗
(or) 𝑵𝟐 = 𝑵𝟏 = 𝑿 𝟏𝟎𝟔
𝟗 𝟗 inside the nucleus?
 It is due to strongest attractive force called nuclear force, which can over come
the electrostatic force of repulsion among protons.

victory R. SARAVANAN. M.Sc., M.Phil., B.Ed PG ASST [PHYSICS], GBHSS, PARANGIPETTAI - 608 502
12 PHYSICS UNIT –9 ATOMIC PHYSICS AND NUCLEAR PHYSICS COMPLETE GUIDE AND MODEL QUESTION
39. Beam of charged particles comprising of two isotopes of same element, moving 46. While explaining fission a nucleus is considered to be analogous to a liquid
with same speed was subjected to a uniform magnetic field at right angles to drop. Why is it so?
the direction of motion. Consequently, the beam get split into two beams  A liquid drop has two tendencies, one due to surface tension which tends to
following trajectories of different radii of curvatures. Which particles (heavier decrease its size and other due to excess pressure which tends to enlarge it.
or lighter) followed curve of lesser radius of curvatures?  Similarly a nucleus has two tendencies, one due to close packing, short range
 Here magnetic Lorentz force provides necessary centripetal force and hence, nuclear forces which tends to decrease its size and other due to electrostatic
𝒎 𝒗𝟐 𝒎𝒗 force of repulsion between protons which tends to enlarge it.
𝑩𝒒𝒗= 𝒓 (or) 𝒓 = 𝑩𝒒 (or) 𝒓 ∝𝒎
47. What is the importance of cadmium rod in a nuclear reactor?
 Therefore lighter particles follow the trajectory of lesser radius of curvature.  Cadmium rods have a high absorbing power of neutrons. Thus they can be used
40. Electrostatic force of repulsion between protons, in a nucleus tends to make to control the speed of the chain reaction.
the nucleus unstable. Inspite of this the nucleus is a highly stable entity? Why is  Greater the area of cadmium rods inside the reactor, greater will be absorption
it so? on neutrons. This will result in a decrease in the activity of chain reaction.
 This is due to attractive nature of nuclear forces between different nucleons 48. A fission reaction of 𝑼𝟐𝟑𝟓 , by a neutron liberates 200 MeV of energy. While a
whose magnitude is much larger than that of electrostatic force of repulsion fusion of 4 hydrogen atoms liberates about 6 MeV of energy. What do you think
between protons. is more exothermic; a reaction involving fission of 𝑼𝟐𝟑𝟓 or a reaction involving
41. Why the phenomenon of natural radioactivity generally found in elements of fussion of hydrogen atoms, when same amount of material is used in both of
high atomic number? them?
 Since the average binding energy per nucleon for the elements having high  Fusion of hydrogen is much more dangerous.
atomic number is very small. This energy is not sufficient to hold the larger  In fission reaction, 236 𝑎𝑚𝑢 of matter liberates 200 𝑀𝑒𝑉 of energy, thus giving
number of nucleons together. Therefore it breaks to emit certain radiation thus energy liberated per 𝑎𝑚𝑢 less than 1. (i.e.) 0.85 𝑀𝑒𝑉
producing natural radioactivity.  In fusion reaction 4 𝑎𝑚𝑢 of matter liberates 6 𝑀𝑒𝑉 of energy, thus giving energy
42. Why do  -rays have the highest ionizing power? liberated per 𝑎𝑚𝑢 is more than 1 (i.e.) 1.5 𝑀𝑒𝑉
  - particles have highest ionizing power due to following two reasons.  If same amount of 𝑈235 and hydrogen is used in these reactions, energy liberated
1) It is the heaviest particle of all the three radioactive radiations. Therefore it in second case (fusion) will be much larger than in the first (fission).
possesses greatest momentum and hence can knockout an orbital electron 49. Which has greatest rest mass a neutron or a proton?
easily.  Rest mass of proton ; 𝒎𝑷 = 𝟏. 𝟎𝟎𝟕𝟖𝟐𝟓 𝒂𝒎𝒖 and
2) It is most heavily charged. Therefore it is capable of exerting maximum Rest mass of neutron ; 𝒎𝒏 = 𝟏. 𝟎𝟎𝟖𝟔𝟔𝟓 𝒂𝒎𝒖
electrostatic interaction on the orbital electron.  So 𝒎𝒏 > 𝒎𝑷 (i.e.) rest mass of neutron is greater than rest mass of proton.
43. Two radioactive samples contain equal number of particles initially. Sample ‘A’ 50. The number of neutrons is more than the number of protons in heavy nuclei.
disintegrates at the rate 107 particles per second while ‘B’ disintegrates at the Why?
rate of 1000 particles per second. Which of the two will be finished earlier?  When the number of protons become very large, the long range electrostatic
 Life time of every radioactive substance is infinite, whatever its rate of force of repulsion will also be greater.
disintegration may be.  For nuclear stability, the short range nuclear force which is attractive must
 So the two samples will continue disintergrating for all times to come while their dominate. For this the number of neutrons must be more than that of number of
rate of disintegration exponentially decreases with time. protons
44. Which particle can be added to the nucleus without affecting its chemical 51. The atomic masses are not whole numbers for most of the atoms. Why?
properties?  The atomic mass is the average mass of the different stable isotopes of the
 Addition of neutrons to the nucleus results in the formation of isotopes having element weighed over their natural abundance, which is not a whole number.
similar chemical properties. 52. Give examples of a nucler disintegration, in which radioactivity is due to
45. Fission and fusion processes both are highly exothermic in nature. positron emission?
Conceptually what is common in them?  𝟏𝟑𝟕𝑵∗ −→ 𝟏𝟑𝟔𝑪 + 𝟎𝟏𝒆 (positron)
 Products of both fission and fusion belong to higher region of average binding  𝟑𝟎 ∗
−→ 𝟑𝟎 𝟎
energy per nucleon as compared to the reactants. 𝟏𝟓𝑷 𝟏𝟒𝑺𝒊 + 𝟏𝒆 (positron)
 Thus we can say that both these processes involve conversion from lower BE/A  𝟔𝟒 ∗ 𝟔𝟒 𝟎
𝟐𝟗𝑪𝒖 −→ 𝟐𝟖𝑵𝒊 + 𝟏𝒆 (positron)
to higher BE/A

victory R. SARAVANAN. M.Sc., M.Phil., B.Ed PG ASST [PHYSICS], GBHSS, PARANGIPETTAI - 608 502
12 PHYSICS UNIT –9 ATOMIC PHYSICS AND NUCLEAR PHYSICS COMPLETE GUIDE AND MODEL QUESTION
53. How are 𝜷 - rays emitted from a nucleus when it does not contain electrons? 60. Atomic power station at Tarapore has generating capacity of 200 MW. How
 A neutron ( 𝟏𝟎𝒏)in the nucleus is converted into a proton ( 𝟏𝟏𝒑), a electron ( −𝟏𝟎𝒆) much energy wil be generated in a day by this station?
and an antineutrino (𝝂 ̅) (i.e.) 𝟏𝟎𝒏 −→ 𝟏𝟏𝒑 + −𝟏𝟎𝒆 + 𝝂̅  The generating capacity = 𝟐𝟎𝟎 𝑴 𝑾 = 𝟐𝟎𝟎 𝑿 𝟏𝟎𝟔 𝑱 𝒔−𝟏
 This electron is emitted as 𝜷 − particle. To show that this electron is coming out ∴ 𝐸nergy generated in one day = 𝟐𝟎𝟎 𝑿 𝟏𝟎𝟔 𝑿 𝟔𝟎𝑿𝟔𝟎 𝑿𝟐𝟒 = 𝟏𝟕𝟐𝟖 𝑿 𝟏𝟎𝟏𝟎 𝑱
from the nucleus, it is termed as 𝜷 - rays. 61. Is it possible that a nucleus has negative mass defect?
54. Is there any difference between 𝜷 - particle emitted by radioactivity and  No. Negative mass defect is not possible.
electrons emitted by photoelectric emission and the thermonic emission?  If it is so, the sum of the masses of the nucleons is less than the real mass of the
 Yes. In photoelectroc emission and thermonic emission, orbital electrons are nucleus and this would mean creation of mass as well as binding energy which
emitted. Their kinetic energy is low. would violate the principal of conservation of mass and energy.
 In case of radioactivity, the electrons are emitted from the nucleus. During the 62. In radioactive dating, we use half life to determine the age of a sample, but not
process a proton converted in to neutron and antineutrino or neutron converted average life. Why?
in to proton and antineutrino, the emitted 𝜷 -particle travel with high velocity  It is a quantitative measure in which we compare the quantity of a radioactive
and hence they have high kinetic energy. substance in the sample to that in the atmoshphere or the fresh substance.
55. In general, an 𝜶 - emission or 𝜷 - emission is followed by the emission of a 63. Two nuclei have mass number in the ratio 𝟖 ∶ 𝟏𝟐𝟓. What is the ratio of their
gamma ray (𝜸). Why? nuclear radii?
𝟏
 After the emission of 𝜶 or 𝜷 particle, the nucleus becomes a new element and it  The nuclear radius is given by, 𝑹 = 𝑹𝟎 𝑨𝟑 (𝑅0 = 1.3 𝑓𝑒𝑟𝑚𝑖)
remains in the excited state. 𝑹𝟏
𝟏/𝟑
𝑨𝟏 𝟖𝟏/𝟑 𝟐
 When the nucleus make transition from the excited state to ground state, a  Hence, 𝑹𝟐
= 𝟏/𝟑 = 𝟏𝟐𝟓𝟏/𝟑
= 𝟓
(or) 𝑹𝟏 ∶ 𝑹𝟐 = 𝟐 ∶ 𝟓
𝑨𝟐
gamma ray photon is emitted. 64. Why it is found experimentally difficult to detect neutrinos in nucler 𝜷 −decay?
56. The penetrating power of 𝜷 -particle is greater than that of 𝜶 -particle, but its  Mass of the neutrino is extreamly small
ionizing power is less than that of 𝜶 -particle. Why?  Charge of the neutrino is negligibly small
 The speed of the 𝜷 - particle is much greater than that of the 𝛼 - particle and it  Neutrino interact very weakly with matter.
will move through a medium quickly and will not find time to ionize the medium. 65. If both number of protons and neutrons in a nuclear reaction is conserved. In
Since the energy is not spend for ionization, they move through a greater what way is mass converted in to energy or vice versa?
distance compared to 𝛼 - particles. Thus its penetrating power is greater, but
 The proton number and neutron number are conserved in a nuclear reaction, but
ionizing power is less compared to 𝛼 −rays. the total mass is not conserved. The total mass of the free protons and neutrons
57. What are thermal neutrons? is more than their total mass within the nucleus. The lost mass (i.e.) mass defect
 Neutrons which are in thermal equilibrium with the medium through which they (∆𝑚) gets converted in to binding energy as per the relation 𝐸 = ∆𝑚𝑐 2
pass are called thermal neutrons. They have energy approximately 0.025 eV.  So the total binding energy of the nuclei before reaction is not be the same as that
 They are considered as ideal particles for nuclear fission, because they are nuclei after reaction. The difference in binding energy causes a release of energy
neutral and as such they do not experience any Coulombian force of attraction or in the reaction.
repulsion. 𝟐 𝟐 𝟑 𝟏
58. Why is heavy water used as moderator?
(e.g) 𝟏𝑯 + 𝟏𝑯−→ 𝟐𝑯𝒆 + 𝟎𝒏 + 𝒆𝒏𝒆𝒓𝒈𝒚
𝑩𝑬
 Moderators are used to slow down the neutrons. 66. A nucleus with mass number 𝑨 = 𝟐𝟒𝟎 and = 𝟕. 𝟔 𝑴𝒆𝑽 breaks up into two
𝑨
 For slowing down the fast neutrons produced during nuclear fission, it must 𝑩𝑬
fragments each of 𝑨 = 𝟏𝟐𝟎 with = 𝟖. 𝟓 𝑴𝒆𝑽 . Calculate the released energy?
𝑨
collide with a nucleus of the same mass. During collision, the neutrons must not
be absorbed by the other nuclei. If heavy water is used, both these requirements
 Total binding energy of the nucleus before breakup; 𝐵𝐸𝑖𝑛𝑖𝑡𝑖𝑎𝑙 = 240 𝑋 7.6 𝑀𝑒𝑉
are satisfied because heavy water has negligible cross-section for neutron
 Total binding energy of the fragments; 𝐵𝐸𝑓𝑖𝑛𝑎𝑙 = 2 𝑋 120 𝑋 8.5 𝑀𝑒𝑉
absorption. Energy released ; Q = 𝐵𝐸𝑓𝑖𝑛𝑎𝑙 − 𝐵𝐸𝑖𝑛𝑖𝑡𝑖𝑎𝑙 = (2 𝑋 120 𝑋 8.5) − (240 𝑋 7.6 )
59. Obtain the energy equivalent of 1 kg of matter and 1 amu of matter? 𝑄 = (240 𝑋 8.5) − (240 𝑋 7.6 ) = 240 (8.5 − 7.6)
 Einstein’s mass - energy equivalence ; 𝑬 = 𝒎 𝒄𝟐 = 𝟏 𝑿 (𝟑 𝑿 𝟏𝟎𝟖 )𝟐 = 𝟗 𝑿 𝟏𝟎𝟏𝟔 𝑱 𝑄 = 240 𝑋 0.9 = 𝟐𝟏𝟔 𝑴𝒆𝑽
 But, 𝟏 𝒂𝒎𝒖 = 𝟏. 𝟔𝟕 𝑿 𝟏𝟎−𝟐𝟕 𝒌𝒈 and 𝟏𝒆𝑽 = 𝟏. 𝟔 𝑿 𝟏𝟎𝟏𝟗 𝑱 .
 Therefore energy equivalent of 1 amu is,
𝑬 = 𝟏. 𝟔𝟕 𝑿 𝟏𝟎−𝟐𝟕 𝑿(𝟑 𝑿 𝟏𝟎𝟖 )𝟐 = 𝟏𝟓. 𝟎𝟑 𝑿 𝟏𝟎−𝟏𝟏 𝑱 = 𝟗𝟑𝟏 𝑴𝒆𝑽

victory R. SARAVANAN. M.Sc., M.Phil., B.Ed PG ASST [PHYSICS], GBHSS, PARANGIPETTAI - 608 502
12 PHYSICS UNIT –9 ATOMIC PHYSICS AND NUCLEAR PHYSICS COMPLETE GUIDE AND MODEL QUESTION
67. Two nuclei have mass number in the ratio 𝟐 ∶ 𝟓. What is the ratio of their
nuclear density?
 Nuclear density; 𝜌𝑁 = 𝑚𝑎𝑠𝑠 = 𝐴4 𝑚𝑁3 = 𝐴 𝑚𝑁𝟏 3 = 4 𝑚𝑁3 = 1.618 𝑋1017 𝑘𝑔𝑚−3
𝑣𝑎𝑙𝑢𝑚𝑒 𝜋𝑅 𝜋𝑅0
3 4 3
𝜋(𝑹𝟎 𝑨𝟑 )
3

 Thus nuclear density is independent of mass number(A). So the ratio will be 𝟏 ∶ 𝟏


68. The isotope 𝟐𝟑𝟖𝟗𝟐𝑼 successively undergoes three α-decays and two β-decays. What
is the resulting isotope?
 The given reaction ; 𝟐𝟑𝟖 𝟒 𝟎
𝟗𝟐𝑼 −→ 𝟑 𝟐𝑯𝒆 + 𝟐 −𝟏𝒆 + 𝒛𝑿
𝑨

 By conservation of charge ; 3(2) + 2(−1) + 𝑍 = 92 (𝑜𝑟) 𝒁 = 𝟖𝟖


 By conservation of mass ; 3(4) + 2(0) + 𝐴 = 238 (𝑜𝑟) 𝑨 = 𝟐𝟐𝟔
 Thus the resulting isotope is radium (i.e.) 𝑨𝒛𝑿 = 𝟐𝟐𝟖
𝟖𝟖𝑹𝒂
69. How long will a radioactive isotope whose half life is 𝑻𝟏/𝟐 , take for its activity to
𝟏
reduce to ( ) th of its initial value?
𝟖
𝒕
𝑵 𝟏 𝒏 𝟏 𝑻
 The amount of isotope undecayed ; 𝑵𝟎
= (𝟐) = (𝟐) 𝟏/𝟐
𝑡 𝑡
1 1 𝑻𝟏/𝟐 1 3 1 𝑻𝟏/𝟐 𝑡
 Hence, 8
= (2) (or) (2) = (2) (or) 3 =
𝑻𝟏/𝟐
∴ 𝒕 = 𝟑 𝑻𝟏/𝟐
70. Two radioactive samples X and Y have the same number of atoms at t= 0. Their
half lives are 3 hours and 4 hours respectively. Compare the rates of
disintegration of the two nuclei after 12 hours.
𝒕 𝟏𝟐
𝟏 𝑻𝑿 𝟏 𝟑 𝟏 𝟒 𝑵
 Number of atoms in X after 12 hours; 𝑵𝑿 = 𝑵𝟎 (𝟐) = 𝑵𝟎 (𝟐) = 𝑵𝟎 (𝟐) = 𝟏𝟔𝟎
𝒕 𝟏𝟐
𝟏 𝟏 𝟏 𝟑 𝑵𝟎
 Number of atoms in Y after 12 hours; 𝑵𝒀 = 𝑵𝟎 (𝟐)𝑻𝒀 = 𝑵𝟎 (𝟐) 𝟒 = 𝑵𝟎 (𝟐) = 𝟖
𝒅𝑵 𝟎.𝟔𝟗𝟑𝟏 𝟎.𝟔𝟗𝟑𝟏 𝑵
 Rate of decay of X ; 𝑹𝑿 = [ ] = − 𝝀𝑿 𝑵𝑿 = −
𝒅𝒕 𝑿 𝑻𝑿
𝑵𝑿 = − 𝟑 [ 𝟏𝟔𝟎]
𝒅𝑵 𝟎.𝟔𝟗𝟑𝟏 𝟎.𝟔𝟗𝟑𝟏 𝑵𝟎
 Rate of decay of Y ; 𝑹𝒀 = [ ] = − 𝝀𝒀 𝑵𝒀 = − 𝑵𝒀 = − [𝟖]
𝒅𝒕 𝒀 𝑻𝒀 𝟒
𝑹 𝟑𝟐 𝟐
 Ratio of rate of decay ; 𝑿 = = (or) 𝑹𝑿 ∶ 𝑹𝒀 = 𝟐 ∶ 𝟑
𝑹𝒀 𝟒𝟖 𝟑
`Keezhadi (கீழடி) - Tamilians pride`
 Keezhadi, a small hamlet, has become one of the very important archeological places
of Tamilandu. It is located in Sivagangai district. A lot of artefacts (gold coins, pottery,
beads, iron tools, jewellery and charcoal, etc.) have been unearthed in Keezhadi
which have given substantial evidence that an ancient urban civilization had thrived
on the banks of river Vaigai.In broken pieces of pottery from the excavation site, the
carbon nanotubes found in keezhadi pottery pushes the oldest known use of nano
technology over 2400 years ago. (i.e.) 400BC
 The Tamil’s used clay pot which had nanotechnology coating (i.e.) They had heat the
wood materials with clay about 1200C. So imagine the technological achievements
of the Tamils in that ancient priod which is extremely remarkable

victory R. SARAVANAN. M.Sc., M.Phil., B.Ed PG ASST [PHYSICS], GBHSS, PARANGIPETTAI - 608 502
12 PHYSICS UNIT –9 ATOMIC PHYSICS AND NUCLEAR PHYSICS COMPLETE GUIDE AND MODEL QUESTION
EXAM NO 9. If the nuclear radius of 27Al
is 3.6 fermi, the approximate nuclear radius of 64Cu is
NAME : (a) 2.4 (b) 1.2
UNIT – 8 ATOMIC PHYSICS AND NUCLEAR PHYSICS (c) 4.8 (d)3.6
10. The nucleus is approximately spherical in shape. Then the surface area of
Time - 2 : 30 hours Total - 60 marks nucleus having mass number A varies as
PART - I 15 X 1 = 15 (a) A2/3 (b) A4/3 (c) A1/3 (d) A5/3
𝟕
Note : (i) Answer all the questions 11. The mass of a 𝟑 𝑳𝒊 nucleus is 0.042 u less than the sum of the masses of all its
𝟕
(ii) Choose the best answer and write the option code and nucleons. The binding energy per nucleon of 𝟑 𝑳𝒊 nucleus is nearly
corresponding answer (a) 46 MeV (b) 5.6 MeV
1. Suppose an alpha particle accelerated by a potential of V volt is allowed to (c) 3.9MeV (d)23 MeV
collide with a nucleus of atomic number Z, then the distance of closest 12. M p denotes the mass of the protonand M n denotes mass of a neutron. A given
approach of alpha particle to the nucleus is nucleus of binding energy B, contains Z protons and N neutrons. The mass
𝑍 𝑉 𝑍 𝑉 M (N,Z) of the nucleus is given by (where c is the speed of light)
(a) 14.4 Å (b) 14.4 Å (c) 1.44 Å (d) 1.44 Å
𝑉 𝑍 𝑉 𝑍 (a) M (N, Z) = N Mn + Z Mp − B c2 (b) M (N, Z) = N Mn + Z Mp + B c2
2. In a hydrogen atom, the electron revolving in the fourth orbit, has angular (c) M (N, Z) = N Mn + Z Mp − B/ c 2 (d) M (N, Z) = N Mn + Z Mp + B/ c2
momentum equal to 13. A radioactive nucleus (initial mass number A and atomic number Z emits

(a) h (b) 2α and 2 positrons. The ratio number of neutrons to that of proton in the final
𝜋
4ℎ 2ℎ
nucleus will be
(c) (d) 𝐴−𝑍−4 𝐴−𝑍−2 𝐴−𝑍−4 𝐴−𝑍−12
𝜋 𝜋 (a)
𝑍−2
(b)
𝑍−6
(c)
𝑍−6
(d) 𝑍−4
3. Atomic number of H-like atom with ionization potential 122.4 V for n = 1 is
14. The half-life period of a radioactive element A is same as the mean life time of
(a) 1 (b) 2
another radioactive element B. Initially both have the same number of atoms.
(c) 3 (d) 4
Then
4. The ratio between the fi rst three orbits of hydrogen atom is
(a) A and B have the same decay rate initially
(a) 1:2:3 (b) 2:4:6
(b) A and B decay at the same rate always
(c) 1:4:9 (d) 1:3:5
(c) B will decay at faster rate than A
5. The charge of cathode rays is
(d) A will decay at faster rate than B.
(a) positive (b) negative
15. A radiative element has No number of nuclei at t = 0. The number of nuclei
(c) neutral (d) not defined 𝟏
6. In J.J. Thomson e/m experiment, a beam of electron is replaced by that of remaining after half of a half-life (that is, at time 𝒕 = 𝑻𝟏/𝟐 )
𝟐
muons (particle with same charge as that of electrons but mass 208 times that 𝑁𝑜 𝑁𝑜 𝑁𝑜 𝑁𝑜
(a) (b) (c) (d)
ofelectrons). No deflection condition is achieved only if 2 √2 4 8
(a) B is increased by 208 times (b) B is decreased by 208 times PART - II 6 X 2 = 12
(c) B is increased by 14.4 times (d) B is decreased by 14.4 times Note : (i) Answer any 6 of the following questions .
7. The ratio of the wavelengths for the transition from n =2 to n = 1 in Li++, He+ (ii) Question No. 23 is compulsory
and H is 16. Define impact parameter.
(a) 1 : 2: 3 (b) 1: 4: 9 17. Define ionization energy.
(c) 3 : 2 : 1 (d) 4: 9: 36 18. Calculate the energy equivalent to one atomic mass unit (1u). Givt the answer in eV
8. The electric potential between a proton and an electron is given by
𝒓 19. What is called isotopes. Give one example.
𝑽 = 𝑽𝒐 𝐥𝐧 [ ], where r0 is a constant. Assume that Bohr atom model is applicable 20. Define one curie.
𝒓𝒐
to potential, then variation of radius of nth orbit rn with the principal quantum 21. What is half life of nucleus. Give the expression.
number n is 22. State the properties of neutrino
1
(a) 𝑟𝑛 ∝ (b) 𝑟𝑛 ∝ 𝑛 23. Calculate the number of nuclei of carbon – 14 undecayed after22,920 years if the
𝑛
1 initial number of carbon – 14 atoms is 10,000. The half life of carbon -= 14 is 5730
(c) 𝑟𝑛 ∝ (d) 𝑟𝑛 ∝ 𝑛2 years
𝑛2

victory R. SARAVANAN. M.Sc., M.Phil., B.Ed PG ASST [PHYSICS], GBHSS, PARANGIPETTAI - 608 502
12 PHYSICS UNIT –9 ATOMIC PHYSICS AND NUCLEAR PHYSICS COMPLETE GUIDE AND MODEL QUESTION
PART - III 6 X 3 = 18
Note : (i) Answer any 6 of the following questions .
(ii) Question No. 30 is compulsory
24. Give the properties of cathode rays.
25. What is the distance of closest approach? Obtain expression for it
26. State the postulates of Bohr’s atom model.
27. Explain radio carbon dating.
28. Give the symbolic representation of alpha decay and beta decay
29. Write a note on proton – proton cycle.
30. Calculate the amount of energy released when 1 kg of 235 92𝑈 undergoes fission
reaction.Give your answer in kwh மனதில் உறுதி வேண்டும்
PART - IV 3 X 5 = 15 ோக்கினிவே இனிமம வேண்டும்
Note : (i) Answer all the questions நிமனவு நல்ேது வேண்டும்
31. Explain the J.J.Thomson experiment to determine the specific charge of electron (Any
one method)
நநருங்கின நபாருள் மைப்பட வேண்டும்
(OR) ைனவு நமய்ப்பட வேண்டும்
Explain the spectral series of hydrogen atom மைேசமாேது விமைவில் வேண்டும்
32. Derive the expression for radius of the nth orbit of hydrogen atom using Bohr atom
model
தனமும் இன்பமும் வேண்டும்
(OR) தைணியிவே நபருமம வேண்டும்
Explain the variation of average binding energy with the mass number by graph and ைண்திறந்திட வேண்டும்
discuss its features. ைாரியத்திலுறுதி வேண்டும்
33. Obtain the law of radioactivity (radio active decay)
(OR) நபண் விடுதமே வேண்டும்
Describe the working of nuclear reactor with a block diagram நபரிய ைடவுள் ைாக்ை வேண்டும்
மண்பயனுற வேண்டும்
ோனமிங்கு நதன்படவேண்டும்
உண்மம நின்றிட வேண்டும்
ஓம்! ஓம்!! ஓம்!!!
- மைாைவி சுப்பிைமண்ய பாைதியார்

victory R. SARAVANAN. M.Sc., M.Phil., B.Ed PG ASST [PHYSICS], GBHSS, PARANGIPETTAI - 608 502
பசித்திரு (Be hungry) தனித்திரு (Be individual) விழித்திரு (Be conscious)

HIGHER SECONDARY SECOND YEAR-PHYSICS

NAME :
STANDARD : 12 SECTION :
SCHOOL :
EXAM NO :

victory R. SARAVANAN. M.Sc, M.Phil, B.Ed.,


PG ASST (PHYSICS)
GBHSS, PARANGIPETTAI - 608 502
12 PHYSICS UNIT – 10 ELECTRONICS AND COMMUNICATION COMPLETE GUIDE AND MODEL QUESTION

PART – I 1 MARK MULTIPLE CHOICE QUESTIONS & ANSWERS 5. The zener diode is primarily used as
1. The barrier potential of a silicon diode is approximately, (a) Rectifier (b) Amplifier
(a) 0.7 V (b) 0.3V (c) 2.0 V (d) 2.2V (c) Oscillator (d) Voltage regulator
Solution:- Solution:-
 A thin region formed near the PN-junction which is free from charge carriers  Zener diode working in the breakdown region can serve as a voltage regulator
(free electrons and holes) is called depletion region. The difference in potential Answer (d) Voltage regulator
across the depletion layer is called the barrier potential (𝑉𝑏 ) . 6. The principle based on which a solar cell operates is
 At 250C, this barrier potential is approximately 0.7 V for silicon and 0.3 V for (a) Diffusion (b) Recombination
germanium (c) Photovoltaic action (d) Carrier flow
Answer (a) 0.7 V Solution:-
2. Doping a semiconductor results in  A solar cell, also known as photovoltaic cell, works on the principle of
(a) The decrease in mobile charge carriers (b) The change in chemical properties photovoltaic effect. Accordingly, the p-n junction of the solar cell generates
(c) The change in the crystal structure (d) The breaking of the covalent bond emf when solar radiation falls on it.
Solution:- Answer (c) Photovoltaic action
 The process of adding impurities to the intrinsic semiconductor is called doping. 7. The light emitted in an LED is due to
It increases the concentration of charge carriers (electrons and holes) in the (a) Recombination of charge carriers
semiconductor and in turn, its electrical conductivity. (b) Reflection of light due to lens action
Answer (c) The change in the crystal structure (c) Amplification of light falling at the junction
3. In an unbiased p-n junction, the majority charge carriers (that is, holes) in the (d) Large current capacity.
p -region diffuse into n-region because of Solution:-
(a) the potential difference across the p-n junction  The excess minority carriers formed due to diffusion recombine with oppositely
(b) the higher hole concentration in p-region than that in n-region charged majority carriers in the respective regions. During recombination
(c) the attraction of free electrons of n-region process, energy is released in the form of light (radiative) or heat (non-
(d) the higher concentration of electrons in the n-region than that in the p-region radiative).
Solution:- Answer (a) Recombination of charge carriers
 Whenever p-n junction is formed, some of the free electrons diffuse from the 8. The barrier potential of a p-n junction depends on (i) type of semiconductor
n-side to the p-side while the holes from the p-side to the n-side. The diffusion of material (ii) amount of doping (iii) temperature. Which one of the following is
charge carriers happens due to the fact that the n-side has higher electron correct?
concentration and the p-side has higher hole concentration. (a) (i) and (ii) only (b) (ii) only
(b) the higher hole concentration in p-region than (c) (ii) and (iii) only (d) (i) (ii) and (iii)
that in n-region Solution:-
Answer the higher concentration of electrons in the n-  Barrier potential (𝑉𝑏 ) is 0.7 V for Si and 0.3 V for Ge
(d) region than that in the p-region  If concentration of dopping is increased, number of charged carrier also
4. If a positive half –wave rectified voltage is fed to a load resistor, for which part increases. Due to this width of depletion region decreases and hence potential
of a cycle there will be current flow through the load? barrier increases.
(a) 00–900 (b) 900–1800  As temperature increases, the value of barrier potential decreases, because the
(c) 00–1800 (d) 00–3600 temperature affects the kinetic energy of the charge carriers.
Solution:- Answer (d) (i) (ii) and (iii)

Answer (c) 00–1800

victory R. SARAVANAN. M.Sc., M.Phil., B.Ed PG ASST [PHYSICS], GBHSS, PARANGIPETTAI - 608 502
12 PHYSICS UNIT – 10 ELECTRONICS AND COMMUNICATION COMPLETE GUIDE AND MODEL QUESTION
9. To obtain sustained oscillation in an oscillator, 13. The output of the following circuit is 1 when the input ABC is
(a) Feedback should be positive (b) Feedback factor must be unity
(c) Phase shift must be 0 or 2π (d) All the above
Solution:-
 The following conditions called Barkhausen conditions should be satisfied for
sustained oscillations in the oscillator.
i. There should be positive feedback.
ii. The loop phase shift must be 00 or integral multiples of 2π. (a) 101 (b) 100
iii. The loop gain must be unity. That is, A =1. (c) 110 (d) 010
Answer (d) All the above Solution:-
10. If the input to the NOT gate is A = 1011, its output is  Output ; 𝑌 = (𝐴 + 𝐵) . 𝐶
(a) 0100 (b) 1000 (i) If ABC = 1 0 1, then 𝑌 = (1 + 0) . 1 = 1
(c) 1100 (d) 0011 (ii) If ABC = 1 0 0, then 𝑌 = (1 + 0) . 0 = 0
Solution:- (iii) If ABC = 1 1 0, then 𝑌 = (1 + 1). 1 = 0
(iv) If ABC = 0 1 0, then 𝑌 = (0 + 1) . 0 = 0
 Output of NOT gate ; 𝑌 = 𝐴̅ = ̅̅̅̅̅̅̅
1011 = 0100
Answer (a) 0100 Answer (a) 101
14. The variation of frequency of carrier wave with respect to the amplitude of
11. Which one of the following represents forward bias diode?
the modulating signal is called
(a) Amplitude modulation (b) Frequency modulation
(a) (b) (c) Phase modulation (d) Pulse width modulation
Solution:-
(c) (d)  If the amplitude of the carrier signal is modified in accordance with amplitude
Solution:- of the baseband signal, then it is called amplitude modulation (AM).
 If the positive terminal of the external voltage source is connected to the p-side  If the frequency of the carrier signal is modified in proportion to the amplitude
and the negative terminal to the n-side, it is called forward bias of the baseband signal, then it is called frequency modulation (FM).
 If the phase of the carrier signal is modified in proportion to the amplitude of
Answer (a) the baseband signal, then it is called phase modulation (PM).
12. The given electrical network is equivalent to Answer (b) Frequency modulation
15. The frequency range of 3 MHz to 30 MHz is used for
(a) Ground wave propagation (b) Space wave propagation
(c) Sky wave propagation (d) Satellite communication
Solution:-
(a) AND gate (b) OR gate
 Ground wave (or) surface wave propagation (nearly 2 kHz to 2 MHz)
(c) NOR gate (d) NOT gate
 Sky wave propagation (or) ionospheric propagation (nearly 3 MHz to 30 MHz)
Solution:-
 Space wave propagation (nearly 30 MHz to 400 GHz)
Answer (c) Sky wave propagation

Answer (c) NOR gate

victory R. SARAVANAN. M.Sc., M.Phil., B.Ed PG ASST [PHYSICS], GBHSS, PARANGIPETTAI - 608 502
12 PHYSICS UNIT – 10 ELECTRONICS AND COMMUNICATION COMPLETE GUIDE AND MODEL QUESTION
9. What is called P-type semiconductor?
PART – II & III 2 AND 3 MARK SHORT ANSWER QUESTIONS & ANSWERS  A P - type semiconductor is obtained by doping a pure Germanium (or Silicon)
1. What is called electronics? crystal with a dopant of trivalent elements (acceptor impurity) like Boron,
 Electronics is the branch of physics incorporated with technology towards the Aluminium, Gallium and Indium. In P-type semicondutors,
design of circuits using transistors and microchips. Holes are majority charge carriers
 It depicts the behaviour and movement of electrons in a semiconductor, Electrons are minority charge carriers
vacuum, or gas. 10. What is N-type semiconductor?
2. What are passive components and active components?  A N - type semiconductor is obtained by doping a pure Germanium (or Silicon)
 Components that cannot generate power in a circuit are called passive crystal with a dopant of pentavalent elements (donor impurity) like
components Phosphorus, Arsenic and Antimony. In N-type semicondutors,
(e.g.) Resistors, inductors, capacitors Electrons are majority charge carriers
 Components that can generate power in a circuit are called active components. Holes are minority charge carriers
(e.g.) transistors 11. Define junction potential or barrier potential.
3. What is energy band?  When P - type and N - type semiconductors combine to form PN junction, due to
 When millions of atoms are brought close to each other, the valence orbitals diffusion of majority charge carriers a depletion region is formed near the
and the unoccupied orbitals are split according to the number of atoms. Their junction.
energy levels will be closely spaced and will be difficult to differentiate the  It prevents the charge carriers to further diffusion across the junction. Because
orbitals of one atom from the other and they look like a band a potential difference is set up by the immobile ions in this depletion region.
 This band of very large number of closely spaced energy levels in a very small  This difference in potential across the depletion layer is called the barrier
energy range is known as energy band. potential or junction potential. This barrier potential approximately equals
4. What is valance band, conduction band and forbidden energy gap? 0.7 V for Silicon and 0.3 V for Germanium.
 The energy band formed due to the valence orbitals is called valence band. 12. What is P-N juction diode? Give its symbol.
 The energy band that formed due to the unoccupied orbitals is called the  A P-N junction diode is formed when a P -type semiconductor is fused with a N-
conduction band type semiconductor. It is a device with single P-N junction
 The energy gap between the valence band and the conduction band is called  The Schematic representation and circuit symbol is given below.
forbidden energy gap.
5. What is called intrinsic semiconductor?
 A semiconductor in its pure form without impurity is called an intrinsic
semiconductor.
 Its conduction is low. (e.g.) Silicon, Germanium
6. Define dopping. 13. What is called biasing? Give its types.
 The process of adding impurities to the intrinsic semiconductor is called  Biasing is the process of giving external energy to charge carriers to overcome
doping. the barrier potential and make them move in a particular direction.
 It increases the concentration of charge carriers (electrons and holes) in the  The external voltage applied to the p-n junction is called bias voltage.
semiconductor and in turn, its electrical conductivity. (1) Forward bias
 The impurity atoms are called dopants. (2) Reverse bias
7. What is extrinsic semiconductors? 14. Differentiate forward bias and reverse bias.
 The semiconductor obtained by dopping either pentavalent impurity or Forward bias :
trivalent impurity is called extrinsic semiconductor.  If the positive terminal of the external voltage source is connected to the P-side
(e.g.) P - type and N-type semicondutor and the negative terminal to the N-side, it is called forward biased
8. Define hole.  It reduces width of the depletion region.
 When an electron is excited, covalent bond is broken. Now octet rule will not be Reverse bias :
satisfied.  If the positive terminal of the battery is connected to the N-side and the
 Thus each excited electron leaves a vacancy to complete bonding. negative potential to the P-side, the junction is said to be reverse biased
 This ‘deficiency’ of electron is termed as a ‘hole’  It increases width of the depletion region.

victory R. SARAVANAN. M.Sc., M.Phil., B.Ed PG ASST [PHYSICS], GBHSS, PARANGIPETTAI - 608 502
12 PHYSICS UNIT – 10 ELECTRONICS AND COMMUNICATION COMPLETE GUIDE AND MODEL QUESTION
15. Define reverse saturation current.  Thisleads to the breaking of covalent bonds and in turn generates electron-hole
 Under reverse bias, a small current flows across the junction due to the pairs.
minority charge carriers in both regions.  The newly generated charge carriers are also accelerated by the electric field
 Because the reverse bias for majority charge carriers serves as the forward bias resulting in more collisions and further production of charge carriers.
for minority charge carriers.  This cumulative process leads to an avalanche of charge carriers across the
 The current that flows under a reverse bias is called the reverse saturation junction and consequently reduces the reverse resistance.
current or leakage current (IS).  This is known as avanlanche breakdown.
 It depends on temperature.  Here the diode current increases sharply.
16. What is meant by rectification? 20. What is called Zener diode? Give its circuit symbol.
 The process of converting alternating current into direct current is called  Zener diode is a reverse biased heavily doped
rectification. Silicon diode which is specially designed to be
 The device used for rectification is called rectifier. operated in the breakdown region.
 A P-N junction diode is used as rectifier. 21. Give the applications of Zener diode.
17. What is mean by break down voltage?  Voltage regulators
 The reverse saturation current due to the minority charge carriers is small.  Peak clippers
 If the reverse bias applied to a P-N junction is increased beyond a point, the  Calibrating voltages
junction breaks down and the reverse current rises sharply.  Provide fixed reference voltage in a network for biasing
 The voltage at which this breakdown happens is called the breakdown voltage  Meter protection against damage from accidental application of excessive
 It depends on the width of the depletion region, which in turn depends on the voltage.
doping level. 22. What is opto electronic devices?
18. Write a note on Zener breakdown.  The devices which convert electrical energy into light and light into electrical
Zener breakdown : energy through semiconductors are called opto electronic devices.
 It wil occur in heavily doped P-N junction which have narrow depletion layers  Optoelectronic device is an electronic device which utilizes light for useful
(< 10-6 m) applications.
 When a reverse voltage across this junction is increased to the breakdown limit, (e.g.) LEDs, photo diodes and solar cells.
a very strong electric field of strength 3 X 107 V m–1 is set up across the narrow 23. What is light emitting diode (LED)?
layer.  LED is a P-N junction diode which emits
 This electric field is strong enough to break or rupture the covalent bonds in the visible or invisible light when it is forward
lattice and thereby generating electron-hole pairs. This effect is called Zener biased.
effect.  Here electrical energy is converted into
 Even a small further increase in reverse voltage produces a large number of light energy, this process is also called
charge carriers. electroluminescence.
 Hence the junction has very low resistance in the breakdown region. 24. Give the applications of LEDs.
 This process of emission of electrons due to the rupture of bands in from the  Indicator lamps on the front panel of the scientific and laboratory equipments.
lattice due to strong electric field is known as internal field emission or field  Seven-segment displays.
ionization.  Traffic signals, exit signs, emergency vehicle lighting etc.
 The electric field required for this is of the order of 106 V m–1  Industrial process control, position encoders, bar graph readers.
19. Write a note on avalanche break down. 25. What is photo diode? Give its circuit symbol.
Avalanche breakdown :  A P-N junction diode which converts an optical signal into electric current is
 It will occurs in lightly doped junctions which have wide depletion layers. known as photodiode
 Here the electric field is not strong enough to produce breakdown.  Its operation exactly opposite to that of
 But the minority charge carriers accelerated by the electric field gains sufficient an LED.
kinetic energy, collide with the semiconductor atoms while passing through the  Photo diode words in reverse bias.
depletion region.  The circuit symbol of photo diode is given
below.

victory R. SARAVANAN. M.Sc., M.Phil., B.Ed PG ASST [PHYSICS], GBHSS, PARANGIPETTAI - 608 502
12 PHYSICS UNIT – 10 ELECTRONICS AND COMMUNICATION COMPLETE GUIDE AND MODEL QUESTION
26. Give the applications of photo diode. 31. Draw the circuit diagram of common base configurations of NPN transistor.
 Alarm system Common base configuration :
 Count items on a conveyer belt  Input termial - Emitter
 Photoconductors Output terminal - Collector
 Compact disc players, smoke detectors Common terminal - Base
 Medical applications such as detectors for computed tomography etc.  Input current = 𝐼𝐸
27. What are called solar cells? Output current = 𝐼𝐶
 A solar cell, also known as photovoltaic cell, converts light energy directly into  The input signal (VBE) is applied across
electricity or electric potential difference by photovoltaic effect. emitter - base junction
 It is basically a P-N junction which generates emf when solar radiation falls on  The output signal (VCB) is measured across collector - base junction.
the P-N junction. 32. Draw the circuit diagram of common emitter configurations of NPN transistor.
28. Give the applications of solar cells. Common emitter configuration :
 Solar cells are widely used in calculators, watches, toys, portable power  Input termial - Base
supplies, etc. Output terminal - Collector
 Solar cells are used in satellites and space applications Common terminal - Emitter
 Solar panels are used to generate electricity.  Input current = 𝐼𝐵
29. Write a note on bipolar junction transistor(BJT). Output current = 𝐼𝐶
 The bipolar junction transistor  The input signal (VBE) is applied across
(BJT) consists of a semiconductor base - emitter junction
(Silicon or Germanium) crystal in  The output signal (VCE) is measured across collector - emitter junction.
which an N-type material is 33. Draw the circuit diagram of common emitter configurations of NPN transistor.
sandwiched between two P-type Common collector configuration :
materials called PNP transistor or  Input termial - Base
a P -type material sandwiched Output terminal - Emitter
between two N-type materials Common terminal - Collector
called NPN transistor.  Input current = 𝐼𝐵
 The three regions formed are called Output current = 𝐼𝐸
emitter (E), base (B) and collector  The input signal (VBC) is applied across
(C) base - collector junction
30. Discuss the different modes of transistor biasing.  The output signal (VEC) is measured across emitter - collector junction.
(1) Forward Active : 34. Define input resistance of transistor.
 In this bias the emitter-base junction (JEB) is forward biased and the  The ratio of the change in base-emitter voltage (ΔVBE) to the change in base
collector-base junction (JCB) is reverse biased. current (ΔIB) at a constant collector-emitter voltage (VCE) is called the input
 The transistor is in the active mode and in this mode, the transistor resistance (𝒓𝒊 ).
functions as an amplifier. ∆ 𝑽𝑩𝑬
𝒓𝒊 = [ ]
(2) Saturation : ∆ 𝑰𝑩 𝑽
𝑪𝑬
 Here, both the emitter-base junction (JEB) and collector-base junction (JCB)  The input resistance is high for a transistor in common emitter configuration.
are forward biased. 35. Define output resistance of transistor.
 The transistor has a very large flow of currents across the junctions and in  The ratio of the change in collector-emitter voltage (ΔVCE) to the change in
this mode, transistor is used as a closed switch. collector current (ΔIC) at a constant base current (IB) is called the output
(3) Cut-off : resistance (𝒓𝑶 ).
 In this bias, both the emitter-base junction (JEB) and collector-base junction ∆ 𝑽𝑪𝑬
(JCB) are reverse biased. 𝒓𝑶 = [ ]
∆ 𝑰𝑪 𝑰
 Transistor in this mode is an open switch. 𝑩
 The output resistance is very low for a transistor in common emitter
configuration.
victory R. SARAVANAN. M.Sc., M.Phil., B.Ed PG ASST [PHYSICS], GBHSS, PARANGIPETTAI - 608 502
12 PHYSICS UNIT – 10 ELECTRONICS AND COMMUNICATION COMPLETE GUIDE AND MODEL QUESTION
36. Define forward current gain. (2) Amplifier :
 The ratio of the change in collector current (ΔI C) to the change in base current  Amplifier amplifies the input ac signal.
(ΔIB) at constant collector-emitter voltage (VCE) is called forward current gain (β). (3) Feed back network :
∆ 𝑰𝑪  The feedback circuit provides a portion of the output to the tank circuit
𝜷= [ ]
∆ 𝑰𝑩 𝑽 to sustain the
𝑪𝑬
 Its value is very high and it generally ranges from 50 to 200.  oscillations without energy loss.
37. Give the relation between 𝜶 and 𝜷  Hence, an oscillator does not require an external input signal.
 Forward current gain in common base mode,  The output is said to be self-sustained.
∆ 𝐼𝐶 42. Give the Barkhausen conditions for sustained oscillations.
𝛼= [ ]  The loop phase shift must be 00 or integral multiples of 2π.
∆ 𝐼𝐸 𝑉
𝐶𝐸  The loop gain must be unity. |𝑨𝜷| = 𝟏
 Forward current gain in common emitter mode, Here, A → Voltage gain of the amplifier,
∆ 𝐼𝐶 β → Feedback ratio
𝛽= [ ]
∆ 𝐼𝐵 𝑉 43. Give the applications of oscillator.
𝐶𝐸
 From the above two equations, we have  To generate a periodic sinusoidal or non sinusoidal wave forms.
𝜷 𝜶  To generate RF carriers.
𝜶= (𝒐𝒓) 𝜷=  To generate audio tones
𝟏+ 𝜷 𝟏− 𝜶
38. What is called transistor amplifier?  To generate clock signal in digital circuits.
 A transistor operating in the active region has the capability to amplify weak  As sweep circuits in TV sets and CRO.
signals. 44. Distinguish between analog and digital signal.
 Amplification is the process of increasing the signal strength (increase in the Analog signal Digital signal
amplitude). It is continuously varying voltage or It contains only two discreate values
39. What is called transistor oscillator? current with respect to time of voltages (i.e.) low (OFF) and high
 An electronic oscillator basically converts DC energy into AC energy of high (ON)
frequency ranging from a few Hz to several MHz. Hence, it is a source of These signals are employed in These signals are employed in signal
alternating current or voltage. rectifying circuits and transistor processing. communication etc.,
 Unlike an amplifier, oscillator does not require any external signal source. amplifier circuits
40. Give the types of an oscillator. 45. Distinguish between positive and negative logic.
 There are two types of oscillators: Positive logic Negative logic
(1) Sinusoidal and Binary 1 stands for +5 V Binary 0 Binary 1 stands for 0V Binary 0 stands
(2) Non-sinusoidal stands for 0 V for +5 V
 Sinusoidal oscillators generate oscillations in the form of sine waves at constant
amplitude and frequency
 Non-sinusoidal oscillators generate complex non-sinusoidal
waveforms like square wave, triangular wave or saw-toothed wave
41. Draw the block diagram of an oscillator
46. Why digital signals are preferred than analog signals?
Block diagram of oscillator :
 Because of their better performance, accuracy, speed, flexibility and immunity
 Oscillator essensially consists
to noice.
three main parts,
47. What are called logic gates?
(1) Tank circuit :
 A logic gate is an electronic circuit which functions based on digital signals.
 The tank circuit generates
 They are considered as the basic building blocks of most of the digital systems.
electrical oscillations and
 It has one output with one or more inputs.
acts as the AC input source
to the transistor amplifier.

victory R. SARAVANAN. M.Sc., M.Phil., B.Ed PG ASST [PHYSICS], GBHSS, PARANGIPETTAI - 608 502
12 PHYSICS UNIT – 10 ELECTRONICS AND COMMUNICATION COMPLETE GUIDE AND MODEL QUESTION
48. Give the circuit symbol, Boolean expression, logical operation and truth table 51. Give the circuit symbol, Boolean expression, logical operation and truth table
of AND gate . of NAND gate .
AND gate - circuit symbol : Inputs Output NAND gate - circuit symbol :
A B Y=A.B
0 0 0
0 1 0
1 0 0
1 1 1 Boolean expression :
Boolean expression :  Ley A and B are the inputs and Y be the output, then 𝒀 = ̅̅̅̅̅̅
𝑨 .𝑩
 Ley A and B are the inputs and Y be the output, then 𝑌 = 𝐴 . 𝐵 Logical operation :
Logical operation :  The output Y equals the complement of AND operation.
 The output of AND gate is high (1) only when all the inputs are high (1).  The circuit is an AND gate followed by a NOT gate. Therefore, it is summarized
 The rest of the cases the output is low (0) as NAND.
49. Give the circuit symbol, Boolean expression, logical operation and truth table  The output is at low (0) only when all the inputs are high (1).
of OR gate .  The rest of the cases, the output is high (1)
OR gate - circuit symbol : Truth table :
Inputs Inputs
Output Output
A B 𝒀=𝑨+𝑩 Input
(AND) (NAND)
0 0 0 A B Z=A.B 𝒀 = ̅̅̅̅̅̅
𝑨 .𝑩
0 1 1
0 0 0 1
1 0 1
0 1 0 1
1 1 1
1 0 0 1
Boolean expression : 1 1 1 0
 Ley A and B are the inputs and Y be the output, then 𝑌 = 𝐴 + 𝐵 52. Give the circuit symbol, Boolean expression, logical operation and truth table
Logical operation : of NOR gate .
 The output of OR gate is high (1) when either of the inputs or both are high (1) NOR gate - circuit symbol :
50. Give the circuit symbol, Boolean expression, logical operation and truth table
of NOT gate .
NOT gate - circuit symbol :
Input Output
A 𝒀=𝑨 ̅ Boolean expression :
0 1  Ley A and B are the inputs and Y be the output, then 𝒀 = ̅̅̅̅̅̅̅̅
𝑨+𝑩
1 0 Logical operation :
Boolean expression :  The output Y equals the complement of OR operation
 If A be the input and Y be the output, then  The circuit is an OR gate followed by a NOT gate and is summarized as NOR
𝒀=𝑨 ̅  The output is high (1) when all the inputs are low (0).
Logical operation :  The rest of the cases, the output is low (0)
 The output is the complement of the input. It is represented with an overbar. Truth table :
 It is also called as inverter. Output Output
Input
 The output Y is high (1), when input is low (0) and vice versa. (OR) (NOR)
A B Z = A + B 𝒀 = ̅̅̅̅̅̅̅̅
𝑨+𝑩
0 0 0 1
0 1 1 0
1 0 1 0
1 1 1 0
victory R. SARAVANAN. M.Sc., M.Phil., B.Ed PG ASST [PHYSICS], GBHSS, PARANGIPETTAI - 608 502
12 PHYSICS UNIT – 10 ELECTRONICS AND COMMUNICATION COMPLETE GUIDE AND MODEL QUESTION
53. Give the circuit symbol, Boolean expression, logical operation and truth table 58. How electron-hole pairs are created in a semiconductor material?
of EX-OR gate .  A small increase in temperature is sufficient enough to break some of the
EX-OR gate - circuit symbol : covalent bonds and release the electrons free from the lattice.
 Hence a vacant site is created in the valanceband and this vacancies are called
holes which are treated to possess positive charges.
 Thus electrons and holes are the two charge carriers in semiconductors.
59. A diode is called as a unidirectional device. Explain
Boolean expression :  An ideal diode behaves as conductor when it is forward biased and behaves as an
 Ley A and B are the inputs and Y be the output, then 𝒀 = 𝑨 . 𝑩 ̅+ 𝑨 ̅ .𝑩 = 𝑨⨁ 𝑩 insulator when it is reverse biased.
Logical operation :  Thus diode coducts current only from P -type to N -type through the
 The output Y is high (1) only when either of the two inputs is high (1). junction when it is forward biased.
 In the case of an Ex-OR gate with more than two inputs, the output will be high  Hence Diode is a unidirectional device.
(1) when odd number of inputs are high (1) 60. What is called modulation? Give its types.
Truth table :  For long distance transmission, the low frequency base band signal (input
Input Output signal) is superimposed on to a high frequency carrier signal (radio signal) by a
A B 𝒀= 𝑨⨁𝑩 process called modulation.
0 0 0 (1) Amplitude Modulation (AM)
0 1 1 (2) Frequency Modulation (FM)
1 0 1 (3) Phase Modulation (PM)
61. What is the necessity of modulation?
1 1 0
 When the information signal of low frequency is transmitted over a long
54. State Demorgan’s theorems. distances, there will be information loss occurs.
Theorem - 1 :
 As the frequency of the carrier signal is very high, it can be transmitted to long
 The complement of the sum of two logical inputs is equal to the product of its
distances with less attenuation.
complements. ̅̅̅̅̅̅̅̅
𝑨 + 𝑩 = 𝑨̅ . 𝑩
̅
 Thus in the modulation process, carrier signal of very high frequency signal
Theorem - 2 :
(radio signal) is used to carry the baseband signal(information)
 The complement of the product of two logical inputs is equal to the sum of its
62. Define amplitude modulation (AM)
complements. ̅̅̅̅̅̅
𝑨 .𝑩 = 𝑨̅+𝑩 ̅
 If the amplitude of the carrier signal is modified according to the instantaneous
55. What is an integrated circuit?
amplitude of the baseband signal, then it is called amplitude modulation (AM)
 An integrated circuit (IC) or a chip or a microchip is an electronic circuit, which
63. Give the advantages and limitations of amplitude modulation (AM)
consists of thousands to millions of transistors, resistors, capacitors, etc.
Advantages of AM :
integrated on a small flat piece of Silicon.
 Easy transmission and reception
56. What are the application of integrated circuits (ICs)
 Low cost  Lesser bandwidth requirements
 great performance.  Low cost
 Very small in size Limitations of AM :
 High reliability  Noise level is high
 They can function as an amplifier, oscillator, timer, microprocessor and  Low efficiency
computer memory.  Small operating range
57. Distinguish between digital IC and analog IC 64. Define frequency modulation (FM)
Digital IC :  If the frequency of the carrier signal is modified according to the instantaneous
 Digital ICs uses digital signals (logical 0 and 1). They usually find their amplitude of the baseband signal then it is called frequency modulation (FM)
applications in computers, networking equipment, and most consumer electronics. 65. Give the advantages and limitations of frequency modulation (FM)
Analog IC : Advantages of FM :
 Analog (or) linear ICs work with continuous values. Linear ICs are typically  Large decrease in noise. This leads to an increase in signal-noise ratio.
used in audio and radio frequency amplification.  The operating range is quite large.
victory R. SARAVANAN. M.Sc., M.Phil., B.Ed PG ASST [PHYSICS], GBHSS, PARANGIPETTAI - 608 502
12 PHYSICS UNIT – 10 ELECTRONICS AND COMMUNICATION COMPLETE GUIDE AND MODEL QUESTION
 The transmission efficiency is very high as all the transmitted power is useful. 72. What are the three modes of propagation of electromagnetic waves through
 FM bandwidth covers the entire frequency range which humans can hear. Due space.
to this, FM radio has better quality compared to AM radio.  Ground wave propagation (or) surface wave propagation (nearly 2 kHz to 2
Limitations of FM : MHz)
 FM requires a much wider channel.  Sky wave propagation (or) ionospheric propagation (nearly 3 MHz to 30 MHz)
 FM transmitters and receivers are more complex and costly.  Space wave propagation (nearly 30MHz to 400GHz)
 In FM reception, less area is covered compared to AM. 73. Write a note on ground wave propagation.
66. Define phase modulation (PM)  If the electromagnetic waves transmitted by the transmitter glide over the
 The instantaneous amplitude of the baseband signal modifies the phase of the surface of the earth to reach the receiver, then the propagation is called ground
carrier signal keeping the amplitude and frequency constant is called phase wave propagation.
modulation  The corresponding waves are called ground waves or surface waves.
67. What is called centre frequency (resting frequency)?  It is mainly used in local broadcasting, radio navigation, for ship-to-ship, shipto-
 When the frequency of the baseband signal is zero (no input signal), there is no shore communication and mobile communication.
change in the frequency of the carrier wave. 74. Give the factors that are responsible for transmission impairments.
 It is at its normal frequency and is called as centre frequency or resting Increasing distance :
frequency.  The attenuation of the signal depends on
 Practically 75 kHz is the allotted frequency of the FM transmitter. (1) power of the transmitter
68. Compare FM and PM ? (2) frequency of the transmitter, and
Comparison between FM and PM : (3) condition of the earth surface.
 PM wave is similar to FM wave. Absorption of energy by the Earth :
 PM generally uses a smaller bandwidth than FM. In other words, in PM, more  When the transmitted signal in the form of EM wave is in contact with the Earth,
information can be sent in a given bandwidth. it induces charges in the Earth and constitutes a current.
 Hence, phase modulation provides high transmission speed on a given  Due to this, the earth behaves like a leaky capacitorwhich leads to the
bandwidth. attenuation of the wave.
69. What is called base band signals? Tilting of the wave: As the wave :
 Information can be in the form of a sound signal like speech, music, pictures, or  As the wave progresses, the wavefront starts gradually tilting according to the
computer data. curvature of the Earth.
 The electrical equivalent of the original information is called the baseband  This increase in the tilt decreases the electric field strength of the wave.
signal.  Finally, at some distance, the surface wave dies out due to energy loss.
70. Define band width. 75. Define sky wave propagation.
 The frequency range over which the baseband signals or the information signals  The mode of propagation in which the electromagnetic waves radiated from an
such as voice, music, picture, etc. is transmitted is known as bandwidth. antenna, directed upwards at large angles gets reflected by the ionosphere back
 Bandwidth gives the difference between the upper and lower frequency limits to earth is called sky wave propagation or ionospheric propagation.
of the signal.  The corresponding waves are called sky waves
 If ν1 and ν2 are the lower and upper-frequency limits of a signal, then the 76. Define skip distance.
bandwidth,  The shortest distance between the transmitter and the point of reception of the
𝑩𝑾 = 𝝂𝟐 − 𝝂𝟏 sky wave along the surface is called as the skip distance
71. Define the size of the antenna. 77. Define skip zone.
 Antenna is used at both transmitter and receiver end.  There is a zone in between where there is no reception of electromagnetic
 Antenna height is an important parameter to be discussed. The height of the waves neither ground nor sky, called as skip zone or skip area.
𝝀 78. What is space wave propagation?
antenna must be a multiple of . (i.e.)
𝟒  The process of sending and receiving information signal through space is alled
𝝀 𝒄 space wave communication
𝒉= =
𝟒 𝟒𝝂  The electromagnetic waves of very high frequencies above 30 MHz are called as
space waves.
victory R. SARAVANAN. M.Sc., M.Phil., B.Ed PG ASST [PHYSICS], GBHSS, PARANGIPETTAI - 608 502
12 PHYSICS UNIT – 10 ELECTRONICS AND COMMUNICATION COMPLETE GUIDE AND MODEL QUESTION
79. Define fibre optical communication. 84. Distinguish between wireline and wireless communication? Specify the range
 The method of transmitting information from one place to another in terms of of electromagnetic waves in which it is used.
light pulses through an optical fiber is called fiber optic communication. Wirelinecommunication
 It works on the principle of total internal reflection.  It is apoint-point communication) uses mediums like wires, cables and optical
80. What is mean by RADAR? fibers.
 Radar basically stands for RAdio Detection And Ranging System.  These systems cannot be used for long distance transmission as they are
 It is one of the important applications of communication systems and is mainly connected physically. Examples are telephone, intercom and cable TV.
used to sense, detect, and locate distant objects like aircraft, ships, spacecraft, etc. Wireless communication
81. Write a note on mobile communication and give its applictions.  It uses free space as a communication medium.
 Mobile communication is used to communicate with others in different  The signals are transmitted in the form of electromagnetic waves with the help
locations without the use of any physical connection like wires or cables of a transmitting antenna.
 It enables the people to communicate with each other regardless of a particular  Hence wireless communication is used for long distance transmission.
location like office, house, etc. Examples are mobile, radio or TV broadcasting, and satellite communication.
 It also provides communication access to remote areas. 85. What are called noises?
Applictions :  It is the undesirable electrical signal that interfaces with the transmitted signal.
 It is used for personal communication and cellular phones offer voice and data  Noise attenuates or reduces the quality of the transmitted signal.
connectivity with high speed.  It may be man-made (automobiles, welding machines, electric motors etc.) or
 Transmission of news across the globe is done within a few seconds. natural (lightning, radiation from sun and stars and environmental effects).
 Using Internet of Things (IoT), it is made possible to control various devices 86. What are repeaters?
from a single device. Example: home automation using a mobile phone.  Repeaters are used to increase the range or distance through which the signals
 It enables smart classrooms, online availability of notes, monitoring student are sent.
activities etc. in the field of education.  It is a combination of transmitter and receiver.
82. Write a note on internet and give itsapplications.  The signals are received, amplified, and retransmitted with a carrier signal of
 Internet is a fast growing technology in the field of communication system with different frequency to the destination.
multifaceted tools.  The best example is the communication satellite in space.
 Internet is the largest computer network recognized globally that connects 87. Define attenuation.
millions of people through computers.  The loss of strength of a signal while propagating through a medium is known as
 It finds extensive applications in all walks of life. attenuation.
Applications : 88. Define Range.
Search engine :  It is the maximum distance between the source and the destination up to which
 The search engine is basically a web-based service tool used to search for the signal is received with sufficient strength.
information on World Wide Web.
Communication :
 It helps millions of people to connect with the use of social networking: emails,
instant messaging services and social networking tools.
E-Commerce :
 Buying and selling of goods and services, transfer of funds are done over an
electronic network.
83. What do you mean by Internet of Things?
 Using Internet of Things (IoT), it is made possible to control various devices
from a single device. (e.g.) Home automation using a mobile phone.

victory R. SARAVANAN. M.Sc., M.Phil., B.Ed PG ASST [PHYSICS], GBHSS, PARANGIPETTAI - 608 502
12 PHYSICS UNIT – 10 ELECTRONICS AND COMMUNICATION COMPLETE GUIDE AND MODEL QUESTION
 Thus, the electrical conduction increases with the increase in temperature. (i.e.)
PART – IV 5 MARK LONG ANSWER QUESTIONS & ANSWERS resistance decreases with increase in temperature.
1. Explain the classification of solids on the basis of energy band theory.  Hence, semiconductors are said to have negative temperature coefficient of
Classification of solids : resistance.
 Based on the energy band theory, solids are classified in to three types, namely  The most important elemental semiconductor materials are Silicon (Si) and
(1) Insulators Germanium (Ge).
(2) Metals (Condutors)  At room temperature,
(3) Semiconductors forbidden energy gap for Si ; 𝑬𝒈 = 𝟏. 𝟏 𝒆𝑽 and
forbidden energy gap for Ge ; 𝑬𝒈 = 𝟎. 𝟕 𝒆𝑽
2. Explain in detail the intrinsic semiconductor.
Intrinsic semiconductor :
 A semiconductor in its pure form without impurity is called an intrinsic
semiconductor.
(e.g) silicon, germanium
 Consider Silicon lattice. Each Silicon atom is covalently bonded with the
neighbouring four atoms to form the lattice.
Insulators :  At absolute zero (0 K), this will behaves as insulator, because there is no free
 In insulator the valence band (VB) and the conduction band (CB) are separated charges.
by a large energy gap.
 The forbidden energy gap (𝐸𝑔 ) is approximately 6 eV in insulators.
 The gap is very large that electrons from valence band cannot move into
conduction band even on the application of strong external electric field or the
increase in temperature.
 Therefore, the electrical conduction is not possible as the free electrons are
almost nil and hence these materials are called insulators.
 Its resistivity is in the range of 𝟏𝟎𝟏𝟏 − 𝟏𝟎𝟏𝟗 𝛀 𝒎
Metals (Conductors) :  But at room temperature, some of the covalent bonds are brakes and releases
 In metals, the valence band and onduction band overlap the electrons free from the lattice.
 Hence, electrons can move freely into the conduction band which results in a  As a result, some states in the valence band become empty and the same
large number of free electrons in the conduction band. number of states in the conduction band will be occupied.
 Therefore, conduction becomes possible even at low temperatures.  The vacancies produced in the valence band are called holes which are treates
 The application of electric field provides sufficient energy to the electrons to as positive charges.
drift in a particular direction to constitute a current.  Hence, electrons and holes are the two charge carriers in semiconductors.
 For metals, the resistivity value lies between 𝟏𝟎−𝟐 − 𝟏𝟎−𝟖 𝛀 𝒎  In intrinsic semiconductors, the number of electrons in the conduction band is
Semiconductors : equal to the number of holes in the valence band.
 In semiconductors, there exists a narrow forbidden energy gap (𝐸𝑔 < 3 𝑒𝑉)  The conduction is due to the electrons in the conduction band and holes in the
between the valence band and the conduction band. valence band
 At a finite temperature, thermal agitations in the solid can break the covalent  The total current (I) is always the sum of the electron current (𝑰𝒆 ) and the hole
bond between the atoms. current (𝑰𝒉 )
 This releases some electrons from valence band to conduction band. 𝑰 = 𝑰𝒆 + 𝑰𝒉
 Since free electrons are small in number, the conductivity of the  The increase in temperature increases the number of charge carriers (electrons
semiconductors is not as high as that of the conductors. and holes).
 The resistivity value of semiconductors is from 𝟏𝟎−𝟓 − 𝟏𝟎𝟔 𝛀 𝒎.
 When the temperature is increased further, more number of electrons is
promoted to the conduction band and increases the conduction.
victory R. SARAVANAN. M.Sc., M.Phil., B.Ed PG ASST [PHYSICS], GBHSS, PARANGIPETTAI - 608 502
12 PHYSICS UNIT – 10 ELECTRONICS AND COMMUNICATION COMPLETE GUIDE AND MODEL QUESTION
3. Elucidate the formation of a N-type and P-type semiconductors.  As Silicon atom has four valence electrons, one electron position of the dopant
N - type semiconductor : in the crystal lattice will remain vacant.
 A n-type semiconductor  The missing electron position in the covalent bond is denoted as a hole.
is obtained by doping a  To make complete covalent, the dopant is in need of one more electron.
pure Silicon (or  These dopants can accept electrons from the neighbouring atoms. Therefore,
Germanium) crystal this impurity is called an acceptor impurity.
with a dopant from  The energy level of the hole created by each impurity atom is just above the
pentavalent elements valence band and is called the acceptor energy level.
like Phosphorus,  For each acceptor atom, there will be a hole in the valence band in addition to
Arsenic, and Antimony the thermally generated holes.
 The dopant has five  Hence, in an P - type semiconductor, the
valence electrons while majority carriers - Holes
the Silicon atom has four valence electrons. minority carriers - Electrons
 During the process of doping, four of the five valence electrons of the impurity 4. Explain the formation of PN junction diode. Discuss its V–I characteristics.
atom are bound with the 4 valence electrons of the neighbouring replaced Formation of PN junction diode :
Silicon atom.  A P-N junction is formed by joining N
 The fifth valence electron of the impurity atom will be loosely attached with the -type and P-type semiconductor
nucleus as it has not formed the covalent bond. materials.
 The energy level of the loosely attached fifth electron is found just below the  Here the N-region has a high electron
conduction band edge and is called the donor energy level concentration and the P-region a high
 The energy required to set free a donor electron is only 0.01 eV for Ge and 0.05 hole concentration.
eV for Si.  So the electrons diffuse from the N-
 At room temperature, these electrons can easily move to the conduction band side to the P-side. Simillarly
with the absorption of thermal energy. holes also diffuse from P - side to the
 The pentavalent impurity atoms donate electrons to the conduction band and N- side. This causes diffusion current.
are called donor impurities.  In a P-N junction, when the electrons and holes move to the other side of the
 Therefore,each impurity atom provides one extra junction, they leave behind exposed charges on dopant atom sites, which are
 electron to the conduction band in addition to the thermally generated fixed in the crystal lattice and are unable to move.
electrons  On the n-side, positive ion cores are exposed and on the p- side, negative ion
 Hence, in an N - type semiconductor, the cores are exposed
majority carriers - Electrons  An electric field E forms between the positive ion cores in the n-type material
minority carriers - Holes and negative ion cores in the p-type material.
P - type semiconductor :  The electric field sweeps free carriers out of this region and hence it is called
 A n-type semiconductor is obtained by doping a pure Silicon (or Germanium) depletion region as it is depleted of free carriers.
crystal with a dopant from trivalent elements like Boron, Aluminium, Gallium  A barrier potential (Vb) due to the electric field E is formed at the junction.
and Indium  As this diffusion of charge carriers from both sides continues, the negative ions
 The dopant has three form a layer of negative space charge region along the p-side.
valence electrons while the  Similarly, a positive space charge region is formed by positive ions on the n-
Silicon atom has four side.
valence electrons.  The positive space charge region attracts electrons from P-side to n-side and
 During the process of the negative space charge region attracts holes from N-side to P -side.
doping, the dopant with  This moment of carriers happen in this region due to the formed electric field
three valence electrons are and it constitutes a current called drift current.
bound with the neighbouring  The diffusion current and drift current flow in the opposite direction and at one
three Silicon atoms. instant they both become equal. Thus, a P-N junction is formed.

victory R. SARAVANAN. M.Sc., M.Phil., B.Ed PG ASST [PHYSICS], GBHSS, PARANGIPETTAI - 608 502
12 PHYSICS UNIT – 10 ELECTRONICS AND COMMUNICATION COMPLETE GUIDE AND MODEL QUESTION
V -I characteristics : For Silicon ; 𝑰𝒍𝒆𝒂𝒌𝒂𝒈𝒆 = 𝟐𝟎 𝝁 𝑨
 It is the study of the variation in current through the diode with respect to the For Germanium ; 𝑰𝒍𝒆𝒂𝒌𝒂𝒈𝒆 = 𝟓𝟎 𝝁 𝑨
applied voltage across the diode when it is forward or reverse biased.  Besides, the current is almost independent of the voltage.
 The reverse bias voltage can be increased only up to the rated value otherwise
the diode will enter into the breakdown region.
5. Draw the circuit diagram of a half wave rectifier and explain its working.
Half wave rectifier :
 In a half wave rectifier circuit,
either a positive half or the
negative half of the AC input is
passed through while the other
half is blocked.
 Only one half of the input wave
reaches the output. Therefore, it
is called half wave rectifier.
 This circuit consists of a transformer, a P-N junction diode and a resistor (𝑅𝐿 )
 Here, a P-N junction diode acts as a rectifying diode.
Forward bias characteristics : During positive half cycle of input During negative half cycle of
 If the positive terminal of the external voltage source is connected to the P-side AC input AC
and the negative terminal to the N-side, it is called forward biased. Terminal A becomes positive with Terminal B becomes positive with
 A graph is plotted by taking the forward bias voltage (V) along the x-axis and respect to terminal B. respect to terminal A.
the current (I) through the diode along the y-axis. This graph is called the The diode is forward biased and The diode is reverse biased and
forward V-I characteristics. hence it conducts hence it does not conducts
 From the graph, The current flows through the load No current passes through RL and
(1) At room temperature, a potential difference equal to the barrier potential is resistor RL and the AC voltage there is no voltage drop across RL
required before a reasonable forward current starts flowing across the developed across RL constitutes the (The reverse saturation current in a
diode. This voltage is known as threshold voltage or cut-in voltage or output voltage V0 diode is negligible)
knee voltage (Vth).  The output waveform is shown below.
For Silicon ; 𝑽𝒕𝒉 = 𝟎. 𝟕 𝑽
For Germanium ; 𝑽𝒕𝒉 = 𝟎. 𝟑 𝑽
(2) The graph clearly infers that the current flow is not linear and is
exponential. Hence it does not obey Ohm’s law.
(3) The forward resistance (𝒓𝒇 ) of the diode is the ratio of the small change in
voltage (ΔV)to the small change in current(ΔI),
∆𝑽
𝒓𝒇 =
∆𝑰
(4) Thus the diode behaves as a conductor when it is forward biased.
Reverse bias characteristics :
 If the positive terminal of the battery is connected to the n-side and the negative
potential to the p-side, the junction is said to be reverse biased.  The output of the half wave rectifier is not a steady dc voltage but a pulsating
 A graph is drawn between the reverse bias voltage and the current across the wave.
junction, which is called the reverse V - I characteristics.  A constant ora steady voltage is required which can be obtained with the help of
 Under this bias, a very small current in μA, flows across the junction. This is due filter circuits and voltage regulator circuits.
to the flow of the minority charge carriers called the leakage current or  Efficiency (𝜼) is the ratio of the output dc power to the ac input power supplied
reversesaturation current. to the circuit. Its value for half wave rectifier is 40.6 %
victory R. SARAVANAN. M.Sc., M.Phil., B.Ed PG ASST [PHYSICS], GBHSS, PARANGIPETTAI - 608 502
12 PHYSICS UNIT – 10 ELECTRONICS AND COMMUNICATION COMPLETE GUIDE AND MODEL QUESTION
6. Explain the construction and working of a full wave rectifier. 7. Write a note on Zener diode. Explain the V - I characteristics of Zener diode.
Full wave rectifier : Zener diode :
 The positive and negative  Zener diode is a reverse biased heavily doped
half cycles of the AC input Silicon diode, designed to be operated in the
signal pass through this breakdown region.
circuit and hence it is called  Zener breakdown occurs due to the breaking of
the full wave rectifier. covalent bonds by the strong electric field set up
 It consists of two P-N in the depletion region by the reverse voltage.
junction diodes, a center  It produces an extremely large number of electrons and holes which constitute
tapped transformer, and a the reverse saturation current.
load resistor (RL). V-I Characteristics of Zener diode :
 The centre (C) is usually  The circuit to study the forward and reverse characteristic s of a Zener diode is
taken as the groundor zero shown below.
voltage reference point.
 Due to the centre tap transformer, the output voltage rectified by each diode is
only one half of the total secondary voltage.
During positive half cycle of input During negative half cycle of input
AC AC
Terminal M is positive, G is at Terminal M is negative, G is at
zero potential and N is at negative zero potential and N is at positive
potential potential  The V-I characteristics of a Zener diode is shown below.
Diode D1 is forward biased Diode D1 is reverse biased  The forward characteristic of a
Diode D2 is reverse biased Diode D2 is forward biased Zener diode is similar to that of
D1 conducts and current flows along D2 conducts and current flows along an ordinary P-N junction diode.
the path MD1AGC the path ND2 BGC  It starts conducting
The voltage appears across RL in the The voltage appears across RL in the approximately around 0.7 V.
direction G to C same direction G to C  However, the reverse
 Hence in a full wave rectifier both postive and negative half cycles of the input characteristics is highly
signal pass through the circuit in the same direction significant in Zener diode.
 The output waveform is shown below.  The increase in reverse voltage
normally generates very small
reverse current.
 While in Zener diode, when the
reverse voltage is increased to
the breakdown voltage (VZ), the
increase in current is very sharp.
 The voltage remains almost
constant throughout the breakdown region.
 Here, IZ(max) represents the maximum reverse current.
 If the reverse current is increased further, the diode will be damaged.
 The important parameters on the reverse characteristics are
 Though both positive and negative half cycles of ac input are rectified, the output VZ→Zener breakdown voltage
is still pulsating in nature. IZ(min)→minimum current to sustain breakdown
 The efficiency (η) of full wave rectifier is twice that of a half wave rectifier and is IZ(max)→maximum current limited by maximum
found to be 81.2 %. power dissipation.
victory R. SARAVANAN. M.Sc., M.Phil., B.Ed PG ASST [PHYSICS], GBHSS, PARANGIPETTAI - 608 502
12 PHYSICS UNIT – 10 ELECTRONICS AND COMMUNICATION COMPLETE GUIDE AND MODEL QUESTION
 The Zener diode is operated in the reverse bias having the voltage greater than  A transparent window is used to allow light to travel in the desired direction.
VZ and current less than IZ(max).  In addition, it has two leads; anode and cathode.
 The reverse characteristic is not exactly vertical which means that the diode  When the P-N junction is forward biased, the conduction band electrons on N-
possesses some small resistance called Zener dynamic impedance. side and valence band holes on P-side diffuse across the junction.
 Zener resistance is the inverse of the slope in the breakdown region. It means  When they cross the junction, they become excess minority carriers (electrons
an increase in the Zener current produces only a very small increase in the in P-side and holes in N-side).
reverse voltage which can be neglected.  These excess minority carriers recombine with oppositely charged majority
 Thus the voltage of an ideal Zener diode does not change once it goes into carriers in the respective regions, i.e. the electrons in the conduction band
breakdown. recombine with holes in the valence band
 It means that VZ remains almost constant even when IZ increases considerably.  During recombination process, energy is released in the form of light (radiative)
8. Explain the working of Zener diode as a voltage regulator. or heat (non-radiative).
Zener diode as a voltage regulator :  For radiative recombination, a photon of energy hv isemitted. For non-radiative
 A Zener diode working in the recombination, energy is liberated in the form of heat.
breakdown region can serve  The colour of the light is determined by the energy band gap of the material.
as a voltage regulator.  Therefore, LEDs are available in a wide range of colours such as blue (SiC),
 It maintains a constant output green (AlGaP) and red (GaAsP). Now a days, LED which emits white light
voltage even when input (GaInN) is also available.
voltage (𝑉𝑖 ) or load current 10. Explain in detail about the photo diode.
(𝐼𝐿 ) varies. Photo diode :
 Here, in this circuit the input  A P-N junction diode which converts an
voltage 𝑉𝑖 is regulated at a optical signal into electric current is known
constant voltage Vz (Zener as photodiode.
voltage) at the output represented as V0 using a Zener diode.  Thus, the operation of photodiode is exactly
 The output voltage is maintained constant as long as the input voltage does not opposite to that of an LED.
fall below Vz .  Photo diode works in reverse bias. The direction of arrows indicates that the
 When the potential developed across the diode is greater than V Z, the diode light is incident on the photo diode.
moves into the Zener breakdown region.  The device consists of a P-N junction semiconductor made of photosensitive
 It conducts and draws relatively large current through the series resistance 𝑹𝑺 material kept safely inside a plastic case . It has a small transparent window
 The total current I passing through 𝑹𝑺 equals the sum of diode current IZ and that allows light to be incident on the P-N junction.
load current IL (i.e.) 𝑰 = 𝑰𝒁 + 𝑰𝑳  Photodiodes can generate current when the P-N junction is exposed to light and
 It is to be noted that the total current is always less than the maximum Zener hence are called as light sensors.
diode current. Under all conditions 𝑽𝑶 = 𝑽𝒁 .  When a photon of sufficient energy (h) strikes the depletion region of the
 Thus, output voltage is regulated. diode, some of the valence band electrons are elevated into conduction band, in
9. What is meant by light emitting diode? Explain its working principle with turn holes are developed in the valence band. This creates electron-hole pairs.
diagram.  The amount of electronhole pairs generated depends on the intensity of light
Light Emitting Diode (LED) : incident on the P-N junction.
 LED is a p-n junction diode which  These electrons and holes are swept across the P-N junction by the electric
emits visible or invisible light field created by reverse voltage before recombination takes place.
when it is forward biased.  Thus, holes move towards the N-side and electrons towards the P-side.
 Since, electrical energy is  When the external circuit is made, the electrons flow through the external
converted into light energy, this circuit and constitute the photocurrent.
process is also called electro  When the incident light is zero, there exists a reverse current which is
luminescence negligible.
 It consists of a P-layer, N-layer  This reverse current in the absence of any incident light is called dark current
and a substrate. and is due to the thermally generated minority carriers.

victory R. SARAVANAN. M.Sc., M.Phil., B.Ed PG ASST [PHYSICS], GBHSS, PARANGIPETTAI - 608 502
12 PHYSICS UNIT – 10 ELECTRONICS AND COMMUNICATION COMPLETE GUIDE AND MODEL QUESTION
11. Explain the working principle of Solar cell. Mention its applications. 12. Explain transistor action in common base configuration.
Solar cell : Working of NPN transistor in CB mode :
 A solar cell, also known as  Basically, a BJT can be considered
photovoltaic cell, converts as two P-N junction
light energy directly into diodes connected back to- back.
electricity or electric potential  In the forward active bias of the
difference by photovoltaic transistor, the emitter-base
effect. junction is forward biased by VEB
 It is basically a P-N junction and the collector-base junction is
which generates emf when reverse biased by VCB.
solar radiation falls on the P-  The forward bias decreases the depletion region across the emitter-base
N junction. junction and the reverse bias increases the depletion region across the
 A solar cell is of two types : P-type and N-type. collector-base junction.
 Both types use a combination of P-type and N-type Silicon which together forms  Hence, the barrier potential across the emitter-base junction is decreased and
the P-N junction of the solar cell. the collector-base junction is increased.
 The difference is that P-type solar cells use P-type Silicon as the base with an  In an NPN transistor, the majority charge carriers in the emitter are electrons.
ultra-thin layer of N-type Silicon, while N-type solar cell uses the opposite As it is heavily doped, it has a large number of electrons.
combination.  The forward bias across the emitter-base junction causes the electrons in the
 The other side of the P-Silicon is coated with metal which forms the back emitter region to flow towards the base region and constitutes the emitter
electrical contact. current (IE).
 On top of the N-type Silicon, metal grid is deposited which acts as the front  The electrons after reaching the base region recombine with the holes in the
electrical contact. base region. Since the base region is very narrow and lightly doped, all the
 The top of the solar cell is coated with anti-reflection coating and toughened electrons will not have sufficient holes to recombine and hence most of the
glass. electrons reach the collector region.
 In a solar cell, electron–hole pairs are generated due to the absorption of light  Eventually, the electrons that reach the collector region will be attracted by the
near the junction. collector terminal as it has positive potential and flows through the external
 Then the charge carriers are separated due to the electric field of the depletion circuit. This constitutes the collector current (IC).
region.  The holes that are lost due to recombination in the base region are replaced by
 Electrons move towards N–type Silicon and holes move towards P-type Silicon the positive potential of the bias voltage VEB and constitute the
layer. base current (IB ).
 The electrons reaching the N-side are collected by the front contact and holes  The magnitude of the base current will be in micoamperes as against
reaching P-side are collected by the back electrical contact. milliamperes for emitter and collector currents.
 Thus a potential difference is developed across solar cell.  It is to be noted that if the emitter current is zero, then the collector current is
 When an external load is connected to the solar cell, photocurrent flows almost zero.
through the load.  It is therefore imperative that a BJT is called a current controlled device.
 Many solar cells are connected together either in series or in parallel  Applying Kirchoff ’s law, we can write the emitter current as the sum of the
combination to form solar panel or module. collector current and the base current. 𝑰𝑬 = 𝑰𝑩 + 𝑰𝑪
 Many solar panels are connected with each other to form solar arrays. For high  Since the base current is very small, we can write, 𝑰𝑬 ≈ 𝑰𝑪
power applications, solar panels and solar arrays are used.  There is another component of collector current due to the thermally generated
Applications : electrons called reverse saturation current, denoted as I CO.
(1) Solar cells are widely used in calculators, watches, toys, portable power  This factor is temperature sensitive.
supplies, etc.  The ratio of the collector current to the emitter current is called the forward
(2) Solar cells are used in satellites and space applications current gain (𝜶𝒅𝒄 ) of a transistor.
(3) Solar panels are used to generate electricity. 𝑰𝑪
𝜶𝒅𝒄 =
𝑰𝑬

victory R. SARAVANAN. M.Sc., M.Phil., B.Ed PG ASST [PHYSICS], GBHSS, PARANGIPETTAI - 608 502
12 PHYSICS UNIT – 10 ELECTRONICS AND COMMUNICATION COMPLETE GUIDE AND MODEL QUESTION
 The α of a transistor is a measure of the quality of a transistor. Higher the value  The input resistance is high for a transistor in common emitter
of α better is the transistor. configuration.
 The value of α is less than unity and ranges from 0.95 to 0.99. (2) Output characteristics :
13. Sketch the static characteristics of a common emitter transistor and bring out  The output characteristics give the
the essence of input and output characteristics. relationship between the variation
Static characteristics of NPN transistor in CE mode : in the collector current (ΔIC) with
respect to the variation in collector-
emitter voltage (ΔVCE) at constant
input current (IB)
 There are four important regions in
the curve
(i) Saturation region
(ii) Cut-off region
(iii) Active region
𝑽𝑩𝑬 − Base - emitter voltage (iv) Break down region
𝑽𝑪𝑬 − Collector - emitter voltage  The ratio of the change in the collectoremitter voltage (ΔVCE) to the
𝑰𝑩 − Base current corresponding change in the collector current (ΔI C) at constant base current
𝑰𝑪 − Collector current (IB) is called output resistance (𝒓𝑶 ).
𝑽𝑩𝑩 & 𝑽𝑪𝑪 −Biasing voltages ∆𝑽𝑪𝑬
𝒓𝑶 = [ ]
𝑹𝟏 & 𝑹𝟐 − Variable resistors ∆𝑰𝑪 𝐼
𝐵
(1) Input characteristics :  The output resistance for transistor in common emitter configuration is
 Input Characteristics curves give the relationship between the base current very low.
(IB) and base to emitter voltage (VBE) at constant collector to emitter (3) Current transfer characteristics :
voltage (VCE)  This gives the variation of collector current (IC) with changes in
 The curve looks like the forward base current (IB) at constant collector-emitter voltage (VCE)
characteristics of an ordinary P-N  It is seen that a small IC flows even
junction diode. when IB is zero.
 There exists a threshold voltage or knee  This current is called the common
voltage (Vk) below which the base emitter leakage current (ICEO), which
current is very small. is due to the flow of minority charge
 Beyond the knee voltage, the base current carriers.
increases with the increase in base-  The ratio of the change in collector
emitter voltage. current (ΔIC) to the change in base
 It is also noted that the increase in the current (ΔIB) at constant collector-
collector-emitter voltage decreases the emitter voltage (VCE) is called
base current. This shifts the curve forward current gain (β).
outward. ∆𝑰𝑪
 This is because the increase in collector-emitter voltage increases the width 𝜷= [ ]
∆𝑰𝑩 𝑉
of the depletion region in turn, reduces the effective base width and thereby 𝐶𝐸

the base current.  Its value is very high and it generally ranges from 50 to 200.
 The ratio of the change in base-emitter voltage (ΔVBE) to the change in base
current (ΔIB) at a constant collector-emitter voltage (VCE) is called the input
resistance (𝒓𝒊 ).
∆𝑽𝑩𝑬
𝒓𝒊 = [ ]
∆𝑰𝑩 𝑽
𝑪𝑬

victory R. SARAVANAN. M.Sc., M.Phil., B.Ed PG ASST [PHYSICS], GBHSS, PARANGIPETTAI - 608 502
12 PHYSICS UNIT – 10 ELECTRONICS AND COMMUNICATION COMPLETE GUIDE AND MODEL QUESTION
14. Transistor functions as a switch. Explain.
Transistor as a switch :  An NPN transistor is connected in the common emitter configuration.
 A load resistance, RC is connected in series with the collector circuit to measure
the output voltage.
 The capacitor C1 allows only the ac signal to pass through.
 The emitter bypass capacitor CE provides a low reactance path to the amplified
ac signal.
 The coupling capacitor CC is used to couple one stage of the amplifier with the
next stage while constructing multistage amplifiers.
 VS is the sinusoidal input signal source applied across the base-emitter.
 The output is taken across the collector-emitter.
 The transistor in saturation and cut-off regions functions like an electronic  Collector current, 𝑰𝑪 = 𝜷 𝑰𝑩
switch that helps to turn ON or OFF a given circuit by a small control signal.  Applying Kirchhoff ’s voltage law in the output loop, the collector-emitter
Presence of dc source at the input (saturation region) : voltage is 𝑽𝑪𝑬 = 𝑽𝑪𝑪 − 𝑰𝑪 𝑹𝑪
 When a high input voltage (Vin = +5 V) is applied, the base current (IB) increases Working of the amplifier :
and in turn increases the collector current. (1) During the positive half cycle :
 The transistor will move into the saturation region (turned ON).  Input signal (Vs) increases the forward voltage across the emitter-base.
 The increase in collector current (IC) increases the voltage drop across RC,  As a result, the base current (IB) increases.
thereby lowering the output voltage, close to zero.  Consequently, the collector current (IC) increases β times.
 The transistor acts like a closed switch and is equivalent to ON condition.  This increases the voltage drop across RC which in turn decreases the
Absence of dc source at the input (cutoff region) : collector-emitter voltage (VCE).
 A low input voltage (Vin = 0 V ), decreases the base current (IB) and in turn  Therefore, the input signal in the positive direction produces an amplified
decreases the collector current (IC). signal in the negative direction at the output. Hence, the output signal is
 The transistor will move into the cut-off region (turned OFF). reversed by 180°
 The decrease in collector current (IC) decreases the drop across R C, thereby (2) During the negative half cycle :
increasing the output voltage, close to +5 V.  Input signal (Vs) decreases the forward voltage across the emitter-base.
 The transistor acts as an open switch which is considered as the OFF condition.  As a result, base current (IB) decreases and in turn increases the collector
 It is manifested that, a high input gives a low output and a low input gives a high current (IC).
output.  The increase in collector current (IC) decreases the potential drop across RC
 Therefore, a transistor can be used as an inverter in computer logic circuitry. and increases the collector-emitter voltage (VCE).
15. Describe the function of a transistor as an amplifier with the neat circuit  Thus, the input signal in the negative direction produces an amplified signal
diagram. Sketch the input and output wave form. in the positive direction at the output.
Transisitor as an amplifier :  Therefore, 1800 phase reversal is observed during the negative half cycle of
 Amplification is the process of the input signal
increasing the signal strength
(increase in the amplitude).
 If a large amplification is
required, multistage amplifier is
used.
 Here, the amplification of an
electrical signal is explained with
a single stage transistor amplifier
 Single stage indicates that the
circuit consists of one transistor
with the allied components.

victory R. SARAVANAN. M.Sc., M.Phil., B.Ed PG ASST [PHYSICS], GBHSS, PARANGIPETTAI - 608 502
12 PHYSICS UNIT – 10 ELECTRONICS AND COMMUNICATION COMPLETE GUIDE AND MODEL QUESTION
16. Explain the action transistor as an oscillator. 17. State and prove De Morgan’s First and Second theorems.
Transistor as an oscillator : De Morgan’s First Theorem :
 An electronic oscillator basically  The complement of the sum of two logical inputs is equal to the product of its
converts dc energy into ac energy complements.
of high frequency ranging from a ̅̅̅̅̅̅̅̅
𝑨+𝑩 = 𝑨 ̅ .𝑩
̅
few Hz to several MHz. Proof :
 Hence, it is a source of alternating A B A+B ̅̅̅̅̅̅̅̅
𝑨+𝑩 ̅
𝑨 ̅
𝑩 ̅ .𝑩
𝑨 ̅
current or voltage. 0 0 0 1 1 1 1
 Unlike an amplifier, oscillator 0 1 1 0 1 0 0
does not require any external
1 0 1 0 0 1 0
signal source.
 Basically, there are two types of 1 1 1 0 0 0 0
oscillators: Sinusoidal and non-sinusoidal.  From the above truth table, we can conclude
Amplifier : ̅̅̅̅̅̅̅̅
𝑨+𝑩 = 𝑨 ̅ .𝑩̅
 Amplification is the process of increasing amplitude of weak signals (i.e)  It also says that a NOR gate is equal to a bubbled AND gate.
Amplifier amplifies the input ac signal  The corresponding logic circuit diagram
Feedback network :
 The circuit used to feedback a portion of the output to the input is called the
feedback network.
 If the portion of the output fed to the input is in phase with the input, then the
magnitude of the input signal increases. De Morgan’s First Theorem :
 It is necessary for sustained oscillations.  The complement of the products of two logical inputs is equal to the sum of its
Tank circuit : complements.
 The LC tank circuit consists of an inductance and a capacitor connected in ̅̅̅̅̅̅
𝑨 .𝑩 = 𝑨 ̅+𝑩̅
parallel Proof :
 Whenever energy is supplied to the tank ̅̅̅̅̅ ̅ ̅ ̅+𝑩 ̅
A B A .B 𝑨. 𝑩 𝑨 𝑩 𝑨
circuit from a DC source, the energy is
stored in inductor and capacitor 0 0 0 1 1 1 1
alternatively. 0 1 0 1 1 0 1
 This produces electrical oscillations of 1 0 0 1 0 1 1
definite frequency. 1 1 1 0 0 0 0
 But in practical oscillator circuits there  From the above truth table, we can conclude
will be loss of energy across resistors, ̅̅̅̅̅̅
𝑨 .𝑩 = 𝑨 ̅+𝑩 ̅
inductor coils and capacitors.  It also says that a NAND gate is equal to a bubbled OR gate.
 Due to this, the amplitude of the  The corresponding logic circuit diagram
oscillations decreases gradually.
 Hence, the tank circuit produces damped
electrical oscillations.
 Therefore, in order to produce undamped oscillations, a positive feedback is
provided from the output circuit to the input circuit.
 The frequency of oscillations is determined by the values of L and C using the
equation.
𝟏
𝒇=
𝟐 𝝅 √𝑳 𝑪

victory R. SARAVANAN. M.Sc., M.Phil., B.Ed PG ASST [PHYSICS], GBHSS, PARANGIPETTAI - 608 502
12 PHYSICS UNIT – 10 ELECTRONICS AND COMMUNICATION COMPLETE GUIDE AND MODEL QUESTION
18. State Boolean laws. Elucidate how they are used to simplify Boolean 19. What is called modulation? Explain the types of modulation with help of
expressions with suitable example. necessary diagrams.
Boolean laws : Modulation :
(1) Complement law :  For long distance transmission, the low frequency baseband signal (input
(i) 𝐴̿ = 𝐴 signal) is superimposed onto a high frequency radio signal by a process called
(2) OR -Laws: modulation.
(i) 𝐴 + 0 = 𝐴  In the modulation process, a very high frequency signal called carrier signal
(ii) 𝐴 + 1 = 1 (radio signal) is used to carry the baseband signal.
(iii) 𝐴 + 𝐴 = 𝐴 Types of modulation :
(iv) 𝐴 + 𝐴̅ = 1 (1) Amplitude modulation (AM)
(3) AND -Laws: (2) Frequency modulation (FM)
(i) 𝐴 .0 = 0 (3) Phase modulation (PM)
(ii) 𝐴 .1 = 𝐴 Amplitude modulation (AM) :
(iii) 𝐴 . 𝐴 = 𝐴  If the amplitude of the carrier signal is modified according to the instantaneous
(iv) 𝐴 . 𝐴̅ = 0 amplitude of the baseband signal, then it is called amplitude modulation.
(4) Commutative Laws :  Here the frequency and the phase of the carrier signal remain constant.
(i) 𝐴 + 𝐵 = 𝐵 + 𝐴
(ii) 𝐴 . 𝐵 = 𝐵 . 𝐴
(5) Associative Laws :
(i) 𝐴 + (𝐵 + 𝐶) = (𝐴 + 𝐵) + 𝐶
(ii) 𝐴 . (𝐵. 𝐶) = (𝐴 . 𝐵). 𝐶
(6) Distributive Laws :
(i) 𝐴 (𝐵 + 𝐶) = 𝐴 𝐵 + 𝐴 𝐶
(ii) 𝐴 + (𝐵 𝐶) = (𝐴 + 𝐵)(𝐴 + 𝐶)
Example :
Simplify the following Boolean expression.
𝑨𝑪 + 𝑨𝑩𝑪
Solution :
𝐴𝐶 + 𝐴𝐵𝐶 = 𝐴𝐶 (1 + 𝐵)
𝐴𝐶 + 𝐴𝐵𝐶 = 𝐴𝐶 .1 [𝑂𝑅 −law (2) ]
𝑨𝑪 + 𝑨𝑩𝑪 = 𝑨𝑪 [𝐴𝑁𝐷 −law (2) ] Circuit description

 We can see clearly that the carrier wave is modified in proportion to the
amplitude of the baseband signal.
 Amplitude modulation is used in radio and TV broadcasting.
Advantages of AM :
 Easy transmission and reception
 Lesser bandwidth requirements
 Low cost
Limitations of AM :
 Noise level is high
 Low efficiency
 Small operating range

victory R. SARAVANAN. M.Sc., M.Phil., B.Ed PG ASST [PHYSICS], GBHSS, PARANGIPETTAI - 608 502
12 PHYSICS UNIT – 10 ELECTRONICS AND COMMUNICATION COMPLETE GUIDE AND MODEL QUESTION
Frequency modulation (FM) : 20. Elaborate on the basic elements of communication system with the necessary
 If the frequency of the carrier signal is modified according to the instantaneous block diagram.
amplitude of the baseband signal, then it is called frequency modulation. Communication system :
 Here the amplitude and the phase of the carrier signal remain constant.  Electronic communication is nothing but the transmission of sound, text,
pictures, or data through a medium.
Basic elements of communication system :
(1) Input transducer :
 A transducer is a device that converts variations in a physical quantity
(pressure, temperature, sound) into an equivalent electrical signal or vice
versa.
 In communication system, the transducer converts the information which is
in the form of sound, music, pictures or computer data into corresponding
electrical signals.
 The electrical equivalent of the original information is called the baseband
signal. (e.g.) microphone
(2) Transmitter :
 It feeds the electrical signal from the transducer to the communication
channel.
 When the amplitude of the baseband signal is zero, the frequency of the  The transmitter is located at the broadcasting station.
modulated signal is the same as the carrier signal.
 The frequency of the modulated wave increases when the amplitude of the
baseband signal increases in the positive direction (A, C).
 The increase in amplitude in the negative half cycle (B, D) reduces the
frequency of the modulated wave
 When the frequency of the baseband signal is zero (no input signal), there is no
change in the frequency of the carrier wave.
 It is at its normal frequency and is called as centre frequency or resting
frequency.
 It consists,
 Practically 75 kHz is the allotted frequency of the FM transmitter.
(i) Amplifier : The transducer output is very weak and is amplified by the
Advantages of FM :
amplifier.
 Large decrease in noise. This leads to an increase in signal-noise ratio.
(ii) Oscillator : It generates high-frequency carrier wave (a sinusoidal
 The operating range is quite large. wave) for long distance transmission into space.
 The transmission efficiency is very high as all the transmitted power is useful. (iii) Modulator : It superimposes the baseband signal onto the carrier
 FM bandwidth covers the entire frequency range which humans can hear. Due signal and generates the modulated signal.
to this, FM radio has better quality compared to AM radio. (iv) Power amplifier : It increases the power level of the electrical signal in
Limitations of FM : order to cover a large distance.
 FM requires a much wider channel. (3) Transmitting antenna :
 FM transmitters and receivers are more complex and costly.  It radiates the radio signal into space in all directions.
 In FM reception, less area is covered compared to AM.  It travels in the form of electromagnetic waves with the velocity of light (3 ×
Phase modulation (PM) : 108 m s–1)
 The instantaneous amplitude of the baseband signal modifies the phase of the (4) Communication channel :
carrier signal keeping the amplitude and frequency constant is called phase  Communication channel is used to carry the electrical signal from transmitter
modulation to receiver with less noise or distortion.
 This modulation is used to generate frequency modulated signals.  The communication medium is basically of two types: wireline
communication and wireless communication.
victory R. SARAVANAN. M.Sc., M.Phil., B.Ed PG ASST [PHYSICS], GBHSS, PARANGIPETTAI - 608 502
12 PHYSICS UNIT – 10 ELECTRONICS AND COMMUNICATION COMPLETE GUIDE AND MODEL QUESTION
(5) Receiver : Sky wave propagation :
 The signals that are transmitted through the communication medium are  The mode of propagation in which the electromagnetic waves radiated from an
received with the help of a receiving antenna and are fed into the receiver. antenna, directed upwards at large angles gets reflected by the ionosphere back
 The receiver consists of electronic circuits like demodulator, amplifier, to earth is called sky wave propagation or ionospheric propagation.
detector etc.  Extremely long distance communication is possible as the radio waves can
undergo multiple reflections between the earth and the ionosphere.
 The phenomenon of bending the radio waves back to earth is due to the total
internal reflection.
 This is the reason why the EM waves are transmitted at a critical angle to
ensure that the waves undergo total reflection and reaches the ground without
escaping into space.
 The shortest distance between the transmitter and the point of reception of the
sky wave along the surface is called as the skip distance
 There is a zone in between where there is no reception of electromagnetic
waves neither ground nor sky, called as skip zone or skip area.
 The demodulator extracts the baseband signal from the carrier signal. Space wave propagation :
 Then the baseband signal is detected and amplified using amplifiers. Finally, it  The process of sending and receiving information signal through space is called
is fed to the output transducer. space wave communication
(6) Output transducer :
 The electromagnetic waves of very high frequencies above 30 MHz are called as
 It converts the electrical signal back to its original form such as sound, space waves.
music, pictures or data. (e.g.) loudspeakers, picture tubes, computer monitor,
 These waves travel in a straight line from the transmitter to the receiver. Hence,
etc.
it is used for a line of sight communication (LOS).
21. Explain the three modes of propagation of electromagnetic waves through
 For high frequencies, the transmission towers must be high enough so that the
space.
transmitted and received signals (direct waves) will not encounter the
Modes of propagation of electromagnetic waves :
curvature of the earth and hence travel with less attenuation and loss of signal
 The electromagnetic wave transmitted by :
strength.
(1) Ground wave propagation (or) surface wave propagation (2 kHz to 2 MHz)
 Certain waves reach the receiver after getting reflected from the ground.
(2) Sky wave propagation (or) ionospheric propagation (3 MHz to 30 MHz)
 The communication systems like television broadcast, satellite communication,
(3) Space wave propagation (30 MHz to 400 GHz)
and RADAR are based on space wave propagation.
Ground wave propagation :
 The range or distance (d) of coverage of the propagation depends on the height
 If the electromagnetic waves transmitted by the transmitter glide over the
surface of the earth to reach the receiver, then the propagation is called ground (h) of the antenna given by the equation, 𝒉 = √𝟐 𝑹 𝒉
wave propagation. where, 𝑅 → Radius of earth (6400 km)
 The corresponding waves are called ground waves or surface waves. 22. Explain satellite communication.
 Both transmitting and receiving antennas must be close to the earth. Satellite communication :
 The size of the antenna plays a major role in deciding the efficiency of the
radiation of signals.
 During transmission, the electrical signals are attenuated over a distance.
 Some reasons for attenuation are as follows:
• Increasing distance
• Absorption of energy by the Earth
• Tilting of the wave
 It is mainly used in local broadcasting, radio navigation, for ship-to-ship, shipto-
shore communication and mobile communication.  The satellite communication is a mode of communication of signal between
transmitter and receiver via satellite.

victory R. SARAVANAN. M.Sc., M.Phil., B.Ed PG ASST [PHYSICS], GBHSS, PARANGIPETTAI - 608 502
12 PHYSICS UNIT – 10 ELECTRONICS AND COMMUNICATION COMPLETE GUIDE AND MODEL QUESTION
 The message signal from the Earth station is transmitted to the satellite on board 24. Fiber optic communication is gaining popularity among the various
via an uplink (frequency band 6 GHz), amplified by a transponder and then transmission media -justify.
retransmitted to another earth station via a downlink (frequency band Fiber optic communication :
4 GHz)  The method of transmitting information from one place to another in terms of
 The high-frequency radio wave signals travel in a straight line (line of sight) light pulses through an optical fiber is called fiber optic communication.
may come across tall buildings or mountains or even encounter the curvature of  It is in the process of replacing wire transmission in communication systems.
the earth.  Light has very high frequency (400THz –790 THz) than microwave radio
 A communication satellite relays and amplifies such radio signals via systems.
transponder to reach distant and far off places using uplinks and downlinks.  The fibers are made up of silica glass or silicon dioxide which is highly abundant
 It is also called as a radio repeater in sky. on Earth.
Applications :  Now it has been replaced with materials such as chalcogenide glasses,
(1) Weather Satellites: fluoroaluminate crystalline materials because they provide larger infrared
 They are used to monitor the weather and climate of Earth. wavelength and better transmission capability.
 By measuring cloud mass, these satellites enable us to predict rain and  As fibers are not electrically conductive, it is preferred in places where multiple
dangerous storms like hurricanes, cyclones etc. channels are to be laid and isolation is required from electrical and
(2) Communication satellites: electromagnetic interference.
 They are used to transmit television, radio, internet signals etc. Multiple Applications :
satellites are used for long distances.  Optical fiber system has a number of applications namely, international
(3) Navigation satellites: communication, inter-city communication, data links, plant and traffic control
 These are employed to determine the geographic location of ships, aircrafts and defense applications.
or any other object. Merits :
23. Explain the function of RADAR. Give its applications.  Fiber cables are very thin and weigh lesser than copper cables.
RADAR :  This system has much larger band width. This means that its information
 Radar basically stands for RAdioDetection And Ranging System. carrying capacity is larger.
 It is one of the important applications of communication systems and is mainly  Fiber optic system is immune to electrical interferences.
used to sense, detect, and locate distant objects like aircraft, ships, spacecraft, etc.  Fiber optic cables are cheaper than copper cables.
 The angle, range, or velocity of the objects that are invisible to the human eye Demerits :
can be determined.  Fiber optic cables are more fragile when compared to copper wires.
 Radar uses electromagnetic waves for communication.  It is an expensive technology.
 The electromagnetic signal is initially radiated into space by an antenna in all directions . Importance :
 When this signal strikes the targeted object, it gets reflected or reradiated in  Fiber optic cables provide the fastest transmission rate compared to any other
many directions. form of transmission.
 This reflected (echo) signal is received by the radar antenna which in turn is  It can provide data speed of 1 Gbps for homes and business.
delivered to the receiver.  Multimode fibers operate at the speed of 10 Mbps.
 Then, it is processed and amplified to determine the geographical statistics of  Recent developments in optical communication provide the data speed at the
the object. rate of 25 Gbps
 The range is determined by calculating the time taken by the signal to travel
from RADAR to the target and back.
Applications :
 In military, it is used for locating and detecting the targets.
 It is used in navigation systems such as ship borne surface search, air search
and weapons guidance systems.
 To measure precipitation rate and wind speed in meteorological observations, Radars
are used.
 It is employed to locate and rescue people in emergency situations.
victory R. SARAVANAN. M.Sc., M.Phil., B.Ed PG ASST [PHYSICS], GBHSS, PARANGIPETTAI - 608 502
12 PHYSICS UNIT – 10 ELECTRONICS AND COMMUNICATION COMPLETE GUIDE AND MODEL QUESTION
4. Determine the wavelength of light emitted from LED which is made up of GaAsP
EXAMPLE PROBLEMS WITH SOLUTIONS semiconductor whose forbidden energy gap is 1.875 eV. Mention the colour of
1. An ideal diode and a 5 Ω resistor are the light emitted (Take h = 6.6 × 10–34 Js).
connected in series with a 15 V power -Solution :- 𝐸𝑔 = 1.875 𝑒𝑉 = 1.875 𝑋 1.6 𝑋 10−19 𝑱 ; ℎ = 6.6 𝑋 10−34 𝐽 𝑠
supply as shown in figure below.  By definition,
Calculate the current that flows through ℎ𝑐
the diode. 𝐸𝑔 = ℎ 𝜈 =
𝜆
-Solution :- ℎ𝑐 6.6 𝑋 10−34 𝑋 3 𝑋 108 19.8 𝑋 10−7
 The diode is forward biased and it is an ideal one. Hence, it acts like a closed ∴ 𝜆 = = =
𝐸𝑔 1.875 𝑋 1.6 𝑋 10−19 3
switch with no barrier voltage. Therefore, current that flows through the diode −7 −9
𝜆 = 6.6 𝑋 10 𝑚 = 660 𝑋 10 𝑚 = 𝟔𝟔𝟎 𝒏𝒎
can be calculated using Ohm’s law.
𝑉=𝐼𝑅  The wavelength 660 nm corresponds to red colour light.
𝑉 15 5. In a transistor connected in the common base configuration
(𝑜𝑟) 𝑰= = =𝟑𝑨 𝜶 = 𝟎. 𝟗𝟓 , 𝑰𝑬 = 𝟏 𝒎𝑨. Calculate the values of IC and IB.
𝑅 5
2. A silicon diode is connected with 1kΩ -Solution :- 𝛼 = 0.95 , 𝐼𝐸 = 1 𝑚𝐴
𝐼𝐶
resistor as shown. Find the value of  Current gain in common base mode ; 𝛼= 𝐼𝐸
current flowing through AB.  Hence, 𝑰𝑪 = 𝛼 𝐼𝐸 = 0.95 𝑋 1 = 𝟎. 𝟗𝟓 𝒎𝑨
-Solution :- 𝑅 = 1 𝑘 Ω = 1000 Ω
 Also, 𝐼𝐸 = 𝐼𝐵 + 𝐼𝐶 (𝑜𝑟) 𝑰𝑩 = 𝐼𝐸 − 𝐼𝐶 = 1 − 0.95 = 𝟎. 𝟎𝟓 𝒎𝑨
 The barrier voltage for silicon diode ; 𝑉𝑏 (𝑆𝑖) = 0.7 𝑉
6. In the circuit shown in the figure, the input voltage Vi is 20 V, VBE = 0 V and
 The P.D. between A and B is given by ; VCE = 0 V. What are the values of IB , IC , β?
𝑉 = [𝑉𝐴 − 𝑉𝐵 ] − 𝑉𝑏 (𝑆𝑖) = [3.3 − (−7.4)] − 0.7 = 10.7 − 0.7 = 𝟏𝟎 𝑽
-Solution :- 𝑅𝐵 = 500 𝑘 Ω ; 𝑅𝐶 = 4 𝑘 Ω
 The value of current flowing through AB can be obtained using Ohm’s law.
 Voltage across 𝑅𝐵 ; 𝑉𝐵 = 𝑉𝑖 − 𝑉𝐵𝐸
𝑉=𝐼𝑅
𝑉 10 1 Since, 𝑉𝐵𝐸 = 0 𝑉 we have, 𝑉𝐵 = 𝑉𝑖 Hence
(𝑜𝑟) 𝑰= = = = 10−2 𝐴 = 𝟏𝟎 𝒎 𝑨 𝑉𝐵 𝑉𝑖 20
𝑅 1000 100 𝑰𝑩 = = = = 0.04 𝑋 10−3
3. Find the current through the Zener diode when the load resistance is 2 kΩ. Use 𝑅𝐵 𝑅𝐵 500 𝑋 103
diode approximation. 𝑰𝑩 = 40 𝑋 10−6 𝐴 = 𝟒𝟎 𝝁 𝑨
-Solution :- 𝑉 = 15 𝑉 ; 𝑅𝑆 = 1 𝑘 Ω = 1000 Ω  Similarly, voltage across 𝑅𝐶 ; 𝑉𝐶 = 𝑉𝐶𝐶 − 𝑉𝐶𝐸
𝑉𝑍 = 9 𝑉 ; 𝑅𝐿 = 2 𝑘 Ω = 2000 Ω
Since, 𝑉𝐶𝐸 = 0 𝑉 we have, 𝑉𝐶 = 𝑉𝐶𝐶 Hence
 From figure, 𝑉𝑅 = 𝑉 − 𝑉𝑍 = 15 − 9 = 6 𝑉 𝑉𝐶 𝑉𝐶𝐶 20
 From Ohm’s law, current through ‘𝑅𝑆 ’ 𝑰𝑪 = = = = 5 𝑋 10−3 = 𝟓 𝒎 𝑨
𝑅𝐶 𝑅𝐶 4 𝑋 103
𝑉𝑅 6
𝑰= = = 6 𝑋 10−3 𝐴 = 𝟔 𝒎 𝑨  And current gain,
𝑅𝑆 1000 𝑰𝑪 5 𝑋 10−3
 And current through load resistance 𝑅𝐿 𝜷= = = 0.125 𝑋 103 = 𝟏𝟐𝟓
𝑰𝑩 40 𝑋 10−6
𝑉𝐿 𝑉𝑍 9 4.5
𝑰𝑳 = = = = = 4.5 𝑋 10−3 𝐴 = 𝟒. 𝟓 𝒎 𝑨 7. The current gain of a common emitter transistor
𝑅𝐿 𝑅𝐿 2000 1000 circuit shown in figure is 120. Draw the DC load line
 If 𝑰𝒁 be the current through Zener diode, then using Kirchoff’s current law at and mark the Q point on it. (VBE to be ignored).
junction A gives, -Solution :- 𝛽 = 120 ; 𝑉𝐶𝐶 = 25 𝑉 ; 𝑅𝐵 = 1 𝑀Ω ;
𝐼 = 𝐼𝐿 + 𝐼𝑍 𝑅𝐶 = 5 𝑘Ω
(𝑜𝑟) 𝐼𝑍 = 𝐼 − 𝐼𝐿 = 6 − 4.5  Voltage across 𝑅𝐵 ; 𝑉𝐵 = 𝑉𝐶𝐶 − 𝑉𝐵𝐸
𝑰𝒁 = 𝟏. 𝟓 𝒎𝑨 Since, 𝑉𝐵𝐸 = 0 𝑉 we have, 𝑉𝐵 = 𝑉𝐶𝐶 Hence
𝑉𝐵 𝑉𝐶𝐶 25
𝑰𝑩 = = = = 25 𝑋 10−6 𝐴 = 𝟐𝟓 𝝁 𝑨
𝑅𝐵 𝑅𝐵 1 𝑋 106

victory R. SARAVANAN. M.Sc., M.Phil., B.Ed PG ASST [PHYSICS], GBHSS, PARANGIPETTAI - 608 502
12 PHYSICS UNIT – 10 ELECTRONICS AND COMMUNICATION COMPLETE GUIDE AND MODEL QUESTION
𝐼𝐶 10. In the combination of the following gates,
 By definition, current gain ; 𝛽 =
𝐼𝐵 write the Boolean equation for output Y in
 Hence collector current, terms of inputs A and B.
𝐼𝐶 = 𝛽 𝐼𝐵 = 120 𝑋 25 𝑋 10−6 = 3000 𝑋 10−6 𝐴 -Solution :-
𝑰𝑪 = 3 𝑋 10−3 𝐴 = 𝟑 𝒎 𝑨  The output at the 1st AND gate = 𝑨 . 𝑩 ̅
 From figure, 𝑉𝐶𝐸 = 𝑉𝐶𝐶 − 𝑉𝐶 = 𝑉𝐶𝐶 − 𝐼𝐶 𝑅𝐶 ̅
 The output at the 2nd AND gate = 𝑨 . 𝑩
𝑉𝐶𝐸 = 25 − (3 𝑋 10−3 𝑋 5𝑋 103 )  The output at the OR gate ; 𝒀 = 𝑨 . 𝑩 ̅+ 𝑨
̅ .𝑩
𝑽𝑪𝑬 = 25 − 15 = 𝟏𝟎 𝑽 11. Prove the Boolean identity AC + ABC = AC and give its circuit description.
8. Calculate the range of the variable capacitor that is to be used in a tuned- -Solution :-
collector oscillator which has a fixed inductance of 150 μH. The frequency band 𝐴 𝐶 + 𝐴 𝐵 𝐶 = 𝐴 𝐶. (1 + 𝐵) [OR law-2]
is from 500 kHz to 1500 kHz. = 𝐴 𝐶 .1 [AND law – 2]
-Solution :- 𝐿 = 150 𝜇 𝐻 𝑨𝑪+𝑨𝑩𝑪= 𝑨𝑪
1 1
 Resonance frequency ; 𝑓𝑜 = (𝑜𝑟) 𝑓𝑜2 =  Thus the Boolean identity is proved.
2 𝜋 √𝐿 𝐶 4 𝜋2 𝐿 𝐶
1 Circuit description:
 Hence capacitance ; 𝐶 =
4 𝜋2 𝐿 𝑓𝑜2
 When, 𝑓𝑜 = 500 𝑘𝐻𝑧 ,
1
𝐶 =
4 𝑋 (3.14 𝑋 150 𝑋 10−6 𝑋 (500 𝑋 103 )2
)2
1
𝐶 =
4 𝑋 9.8596 𝑋 150 𝑋 250000
𝐶 = 6.761 𝑋 10−10 𝐹 = 676.1 𝑋 10−12 𝐹 12. A transmitting antenna has a height of 40 m and the height of the receiving
𝑪 ≈ 𝟔𝟕𝟔 𝒑𝑭 antenna is 30 m. What is the maximum distance between them for line-of-sight
 When, 𝑓𝑜 = 1500 𝑘𝐻𝑧 communication? The radius of the earth is 6.4×106 m.
1
𝐶 =
4 𝑋 (3.14 )2 𝑋 150 𝑋 10−6 𝑋 (1500 𝑋 103 )2
1
𝐶 =
4 𝑋 9.8596 𝑋 150 𝑋 2250000
𝐶 = 7.511 𝑋 10−11 𝐹 = 75.11 𝑋 10−12 𝐹
𝑪 ≈ 𝟕𝟓 𝒑𝑭
 Therefore, the capacitor range is 𝟕𝟓 𝒑𝑭 - 𝟔𝟕𝟔 𝒑𝑭
9. What is the output Y in the following -Solution :-
circuit, when all the three inputs A, B, and  The total distance d between the transmitting and receiving antennas will be the
C are first 0 and then 1? sum of the individual distances of coverage.
-Solution :-
𝑑 = 𝑑1 + 𝑑2 = √2 𝑅 ℎ1 + √2 𝑅 ℎ2 = √2 𝑅 (√ℎ1 + √ℎ2 )
 Out put of 𝐴𝑁𝐷 gate P : 𝑿 = 𝑨 . 𝑩
 Out put of 𝑁𝐴𝑁𝐷 gate Q : 𝒀 = ̅̅̅̅̅̅ 𝑿 .𝑩 𝑑 = √2 𝑋 6.4 𝑋 106 (√40 + √30)
A B C 𝑿 = 𝑨.𝑩 𝒀 = ̅̅̅̅̅̅
𝑿 .𝑩 𝑑 = √2 𝑋 6.4 𝑋 106 𝑋 √10 (√4 + √3)
0 0 0 0 1 𝑑 = √2 𝑋 6.4 𝑋 107 (√4 + √3)
1 1 1 1 0 √2 𝑋 64 𝑋 106 (√4 + √3)
𝑑=
𝑑= 1.414 𝑋 8 𝑋 103 (2 + 1.732)
𝑑= 1.414 𝑋 8 𝑋 103 𝑋 3.732
𝒅= 𝟒𝟐. 𝟐𝟏 𝑿 𝟏𝟎𝟑 𝒎 = 𝟒𝟐. 𝟐𝟏 𝒌𝒎

victory R. SARAVANAN. M.Sc., M.Phil., B.Ed PG ASST [PHYSICS], GBHSS, PARANGIPETTAI - 608 502
12 PHYSICS UNIT – 10 ELECTRONICS AND COMMUNICATION COMPLETE GUIDE AND MODEL QUESTION
3. Assuming VCEsat = 0.2 V and β = 50, find the
EXERCISE PROBLEMS WITH SOLUTIONS minimum base current (IB) required to drive the
1. The given circuit has two ideal diodes connected as shown in figure below. transistor given in the figure to saturation.
Calculate the current flowing through the resistance R1. -Solution :- 𝑉𝐶𝐶 = 3 𝑉 ; 𝑅𝐶 = 1 𝑘Ω = 1000 Ω
 From figure, 𝑉𝐶𝐶 = 𝐼𝐶 𝑅𝐶 + 𝑉𝐶𝐸
(𝑜𝑟) 𝐼𝐶 𝑅𝐶 = 𝑉𝐶𝐶 − 𝑉𝐶𝐸
𝑉𝐶𝐶 − 𝑉𝐶𝐸 3 − 0.2
(𝑜𝑟) 𝑰𝑪 = =
𝑅𝐶 1000
𝑰𝑪 = 𝟐. 𝟖 𝑿 𝟏𝟎−𝟑 𝑨
𝐼
 Then current gain ; 𝛽 = 𝐶 . Hence,
𝐼𝐵
𝐼𝐶 2.8 𝑋 10−3
-Solution :- 𝑰𝑩 = = = 0.056 𝑋 10−3 𝐴 = 56 𝑋 10−6 = 𝟓𝟔 𝝁 𝑨
𝛽 50
 Here diode 𝑫𝟏 is reverse biased. So it acts as open switch (OFF) and hence it does
4. A transistor of α = 0.99 and VBE = 0.7 V is connected in the common emitter
not allows current to pass through it.
configuration as shown in the figure. If the transistor is in saturation region,
 But diode 𝑫𝟐 is forward biased. So it acts as closed switch (ON) and hence it find the value of collector current.
allows current to pass through it. -Solution :-
 From Ohm’s law, 𝑉 = 𝐼 𝑅𝑆  If 𝛼 = 0.99 then,
𝑉 10 10 𝛼 0.99 0.99
(𝑜𝑟) 𝑰= = = = 𝟐. 𝟓 𝑨 𝛽= = = = 99
𝑅𝑆 (2 + 2) 4 1− 𝛼 1−0.99 0.01
2. Four silicon diodes and a 10 Ω resistor are connected as shown in figure below.  By definition. current gain,
Each diode has a resistance of 1Ω. Find the current flows through the 10Ω 𝐼𝐶 𝐼𝐶 𝐼𝐶
resistor. 𝛽= (𝑜𝑟) 𝑰𝑩 = =
𝐼𝐵 𝛽 90
-Solution :-  Here one thing must be remember that,
 Here diode 𝑫𝟏 & 𝑫𝟒 is reverse biased. So it transistor in saturation region have,
acts as open switch (OFF) and hence it does 𝑉𝐵𝐸−𝑠𝑎𝑡 = 0.8 𝑉 and 𝑉𝐶𝐸−𝑠𝑎𝑡 = 0.2 𝑉
not allows current to pass through it.  From figure, for input applying Kirchoff’s voltage law
 But diode 𝑫𝟐 & 𝑫𝟑 is forward biased. So it 𝑉1 + 𝑉2 + 𝑉3 = 𝑉𝐶𝐶 − 𝑉𝐵𝐸−𝑠𝑎𝑡
acts as closed switch (ON) and hence it 1000 (𝐼𝐶 + 𝐼𝐵 ) + 10000 𝐼𝐵 + 1000 (𝐼𝐶 + 𝐼𝐵 ) = 12 − 0.8
allows current to pass through it. 2000 𝐼𝐶 + 12000 𝐼𝐵 = 11.2 − − − − − − (1)
 Hence the given circuit is simplified as shown.  Similarly for output applying Kirchoff’s voltage law
 Since the barrier voltage of silicon is 0.7 V and 𝑉1 + 𝑉2 + 𝑉3 = 𝑉𝐶𝐶 − 𝑉𝐶𝐸−𝑠𝑎𝑡
hence voltage across 𝟏𝟎 𝛀 resistor, 1000 (𝐼𝐶 + 𝐼𝐵 ) + 10000 𝐼𝐵 + 1000 (𝐼𝐶 + 𝐼𝐵 ) = 12 − 0.2
𝑉10 = 3 − 0.7 − 0.7 = 𝟏. 𝟔 𝐕 3000 𝐼𝐶 + 2000 𝐼𝐵 = 11.8 − − − − − − (2)
 Total resistance of the circuit,  (2) X 6  18000 𝐼𝐶 + 12000 𝐼𝐵 = 70.8 − − − − − − (3)
𝑅𝑆 = 1 + 10 + 1 = 𝟏𝟐 𝛀  (3) - (1)  16000 𝐼𝐶 = 59.6
 Then the current through 𝟏𝟎 𝛀 resistor, 59.6
𝑉10 1.6 𝐼𝐶 = = 3. 724 𝑋 10−3 𝐴
𝐼= = = 𝟎. 𝟏𝟑𝟑 𝑨 16000
𝑅𝑆 12 −𝟑
𝑰𝑪 = 𝟑. 𝟕𝟐𝟒 𝑿 𝟏𝟎 𝑨 = 𝟑. 𝟕𝟐𝟒 𝒎𝑨

victory R. SARAVANAN. M.Sc., M.Phil., B.Ed PG ASST [PHYSICS], GBHSS, PARANGIPETTAI - 608 502
12 PHYSICS UNIT – 10 ELECTRONICS AND COMMUNICATION COMPLETE GUIDE AND MODEL QUESTION
5. In the circuit shown in the figure, the BJT has a (𝑨 + 𝑩)( 𝑨 + 𝑪) = 𝑨 + 𝑩 𝑪
current gain (β) of 50. For an emitter – base ̅ B = A + B using truth table.
8. Verify the given Boolean equation A + 𝑨
voltage VEB = 600 mV, calculate the emitter – Solution :-
collector voltage VEC (in volts). ̅ ̅𝑩 ̅𝑩
A B 𝑨 𝑨 𝑨+𝑨 A+B
Solution :- 𝑉𝐸 = 3 𝑉 ; 𝑅𝐵 = 60𝐾Ω ; 𝑅𝐶 =
500 Ω 0 0 1 0 0 0
 From figure, , 𝑉𝐸 = 𝑉𝐸𝐵 + 𝑉𝐵 0 1 1 1 1 1
(𝑜𝑟) 𝑉𝐵 = 𝑉𝐸 − 𝑉𝐸𝐵 1 0 0 0 1 1
60000 𝐼𝐵 = 3 − 600 𝑋 10−3 1 1 0 0 1 1
60 𝑋 103 𝐼𝐵 = 3 − 0.6 = 2.4 9. In the given figure of a voltage
2.4
𝐼𝐵 = = 0.04 𝑋 10−3 𝐴 = 40 𝑋 10−6 𝐴 = 𝟒𝟎 𝝁 𝑨 regulator, a Zener diode of breakdown
60 𝑋 103 voltage 15V is employed. Determine
𝐼
 By definition, current gain 𝛽 = 𝐶 the current through the load
𝐼𝐵
(𝑜𝑟) 𝑰𝑪 = 𝛽 𝐼𝐵 = 50 𝑋 40 𝑋 10−6 = 2000 𝑋 10−6 𝐴 = 2 𝑋 10−3 = 𝟐 𝒎 𝑨 resistance, the total current and the
 Hence, 𝑉𝐸𝐶 = 𝑉𝐸 − 𝑉𝐶 = 𝑉𝐸 − 𝐼𝐶 𝑅𝐶 = 3 − (2 𝑋 10−3 𝑋 0.5 𝑋 103 ) current through the diode. Use diode
𝑽𝑬𝑪 = 𝟑 − 𝟏 = 𝟐 𝑽 approximation.
Solution :- 𝑉 = 25 𝑉 ; 𝑅𝑆 = 500 Ω ; 𝑉𝑍 = 15 𝑉 ; 𝑅𝐿 = 3 𝑘 Ω = 3000 Ω
6. Determine the current flowing through 3Ω and 4Ω
resistors of the circuit given below. Assume that diodes  From the figure, 𝑽𝑹𝑺 = 𝑉 − 𝑉𝑍 = 25 − 15 = 10 𝑉
D1 and D2 are ideal diodes.  From Ohm’s law Current through load resistance 𝑅𝐿 ,
Solution :- 𝑉 𝑉 15
𝑰𝑳 = 𝐿 = 𝑍 = = 5 𝑋 10−3 𝐴 = 𝟓 𝒎 𝑨
 Here diode D1 is forward biased (closed switch) and D2 𝑅 𝐿 𝑅 𝐿 3000
is reverse biased (open switch)  And, current through 𝑅𝑆 (i.e.) total current
𝑽𝑹 10 1
 So D1 conducts while D2 do not conduct the current. 𝑰 = 𝑅 𝑺 = 500 = 50 = 0.02 𝐴 = 20 𝑋 10−3 𝐴 = 𝟐𝟎 𝒎 𝑨
 For ideal diode, there is no barrier voltage (i.e.) VB = 0 𝑆

 Let ‘I’ be the current through D1,then by Ohm’s Kirchoff’s voltage law,  If 𝑰𝒁 be the current through Zener diode, then from Kirchoff’s current law,
𝟐 𝑰 + 𝟒 𝑰 = 𝟏𝟐 (𝒐𝒓) 𝟔 𝑰 = 𝟏𝟐 (𝒐𝒓) 𝑰 = 𝟐 𝑨 𝐼 = 𝐼𝐿 + 𝐼𝑍
(𝑜𝑟) −3 −3 −3
 Since D2 will not conduct, no current flows through diode D2 𝐼𝑍 = 𝐼 − 𝐼𝐿 = (20 𝑋 10 ) − (5 𝑋 10 ) = 15 𝑋 10 𝐴
 Thus current flowing through 3Ω and 4Ω resistors of the circuit are 0 and 2 A 𝑰𝒁 = 𝟏𝟓 𝒎𝑨
Respectively. 10. Write down Boolean equation for the
7. Prove the following Boolean expressions using the laws and theorems of output Y of the given circuit and give its
̅ ̅
Boolean algebra. (i) (A+B) (A+𝑩) = A (ii) A(𝑨+B) = AB (iii) (A+B) (A+C) = A+BC truth table.
Solution :- Solution :-
̅ ̅
(i) (𝐴 + 𝐵)(𝐴 + 𝐵 ) = 𝐴 𝐴 + 𝐴 𝐵 + 𝐵 𝐴 + 𝐵 𝐵 ̅ ̅
[ By AND laws; 𝐴 𝐴 = 𝐴 & 𝐵 𝐵 = 0]  Output of AND gate= 𝐴. 𝐵
̅
= 𝐴 + 𝐴 ( 𝐵 + 𝐵) + 0 ̅
[ By OR laws ; 𝐵 + 𝐵 = 1 & A +0  Output of NOR gate = ̅̅̅̅̅̅̅̅
𝐴+𝐵
=0]  Thus the final output of OR gate ;
𝒀 ̅̅̅̅̅̅̅̅
= (𝑨. 𝑩) + (𝑨 + 𝑩)
= 𝐴 + 𝐴 (1) [ By AND laws ; 𝐴. 1 = 𝐴 ]
̅
(𝑨 + 𝑩)(𝑨 + 𝑩) = 𝑨 [ By OR laws ; 𝐴 + 𝐴 = 𝐴 ] A B A.B A+B ̅̅̅̅̅̅̅̅
𝑨+𝑩 ̅̅̅̅̅̅̅̅
𝒀 = (𝑨. 𝑩) + (𝑨 + 𝑩)
̅ ̅
(ii) 𝐴 ( 𝐴 + 𝐵) = 𝐴 𝐴 + 𝐴 𝐵 ̅
[ By AND laws ; 𝐴 𝐴 = 0]
0 0 0 0 1 1
= 0+𝐴𝐵 [ By OR laws ; 0 + 𝐴 = 𝐴 ]
𝑨 (̅𝑨 + 𝑩) = 𝑨 𝑩 0 1 0 1 0 1
(iii) (𝐴 + 𝐵)( 𝐴 + 𝐶) = 𝐴𝐴 + 𝐴 𝐶 + 𝐵 𝐴 + 𝐵 𝐶 1 0 0 1 0 1
=𝐴+𝐴𝐶+𝐵𝐴+𝐵𝐶 1 1 1 1 0 1
= 𝐴(1 + 𝐶 + 𝐵) + 𝐵 𝐶 [ By OR laws ; 1 + 𝐴 = 1]
= 𝐴(1) + 𝐵 𝐶
victory R. SARAVANAN. M.Sc., M.Phil., B.Ed PG ASST [PHYSICS], GBHSS, PARANGIPETTAI - 608 502
12 PHYSICS UNIT – 10 ELECTRONICS AND COMMUNICATION COMPLETE GUIDE AND MODEL QUESTION
8. In the figure, which of the diodes are forward biased and which are reverse
CONCEPTUAL QUESTIONS AND ANSWERS biased?
1. Why is it that crystals having ionic bonds do not conduct electricity easily?
 Conduction of electricity takes place through free electrons.
 In case of ionic bonds, the orbital electrons are strongly bound to each other.
Thus there are no free electrons in them. Hence they do not conduct electricity
easily.
2. Explain the reason for metallic bonds possessing high thermal and electrical
conductivity?
 In case of metallic bonding, positively charged heavier constituents of atom are
supposed to be submerged in a sea of free electrons. The presence of this large
number of free electrons called electron gas is responsible for high thermal and
electrical conductivity.  If P-region of the diode be at a positive potential with respect to N -region, then it
3. Sugar get readily dissolved in water, while oil cannot do so. Give the reason. is said to be in forward biased. If P-region of the diode be at a negative (or) low
 Water and sugar consists of polar molecules. When the mixed together, they positive potential with respect to N -region, then it is said to be in reverse biased.
come closer to each other forming closed chains.  When the terminal is earthed, it is at zero potential.
 Oil molecules are non-polar in nature. Therefore they cannot form closed chains (i) Reverse biased, because N -region is at positive potential
with water molecule. Thus oil remains separate from water. (ii) Forward biased, because N-region is at low positive potential w.r.to P- region
4. Is a P-type semiconductors positively charged? Why is it so called? (iii) Reverse biased, because P -region is connected to negative potential
 A P-type semiconductor or an N-type semiconductor, both are neutral. (iv) Forward biased, because P-region is at least negative potential than N-region
 The P-type semiconductors contains positively charged holes as majority charge (v) Forward biased,because N- region is connected negative potential
carriers. That is why they are named as P-type semiconductors. 9. What is the difference between an elemental semiconductor and a compound
5. Are the two components N-type and P-type semiconductor of a PN -junction semiconductor?
diode electrically neutral?  An elemental semiconductor consists of single species of atoms (e.g) Si and Ge
 No. They are not electrically neutral.  But in a compound semiconductor,there are more than one element (e.g) Ga As P
 As N-type and P-type crystals lie close to each other to form PN -junction, a cloud 10. How will you distinguish between a pure semiconductor and a semiconductor
of free electrons in N-type semiconductor diffuses across the junction, and hence made from metals?
N-type (which was previously neutral) aquires positive potential. The electrons  The resistance of a pure semiconductor made of Si and Ge decreases with
on reaching P-type semiconductor neutralize equal number of the holes and increase in temperature. They have negative temperature coefficient.
hence P-type (which was also previously neutral) now contains a lesser number  But the resistance of a semiconductor made from metals increases with rise in
of holes aquires a negative potential. temperature. They have positive temperature coefficient.
6. What is the barrier potential of Si and Ge semiconductor? How does it change 11. The forbidden band energy of silicon is 1.1 eV. What does it mean?
with increase in temperature?  The gap between valence band and conduction band is called forbidden gap.
 The barrier potential of Si is 0.7 V and that of Ge 0.3 V  Hence the forbidden band energy of silicon is 1.1eV means , if 1.1eV is given to an
 The change in barrier potential is ∆𝑽 = − 𝟎. 𝟎𝟎𝟐𝟓 ∆𝑻 electron in the valence band, it will jump to the conduction band.
(i.e.) the barrier potential decreases by 2.5 mV for each ℃ rise 12. The conductivity of intrinsic semiconductor is very low. Why? How it can be
7. Can two separate PN -junction diodes placed back to back be used to form raised?
PNP-transistor?  In a pure semiconductor, at room temperature only few electrons in valence
 No. When we join two PN - junctions the N-region will form the base. For a band comes out due to absorbtion of thermal energy, occupies the conduction
transistor the base must be very thin and it must be lightly dopped. These two band and leaving equal number of free holes in valence band.
conditions will not be satisfied when we make a transistor by joined two  Thus there are only few electrons in conduction band, its conductivity is very low
PN-junctions. The thickness will be large and dopping will be heavy.  The conductivity of semiconductor can be increased by dopping impurities like
 So PNP transistor cannot be formed by joining two PN-junction diodes. trivalent or pentavalent atoms and the impurity added semiconductor is called
extrinsic semiconductor.

victory R. SARAVANAN. M.Sc., M.Phil., B.Ed PG ASST [PHYSICS], GBHSS, PARANGIPETTAI - 608 502
12 PHYSICS UNIT – 10 ELECTRONICS AND COMMUNICATION COMPLETE GUIDE AND MODEL QUESTION
13. What is hole? Is there any hole in a N-type semiconductor? 21. The gain of a common emitter amplifier is given by = − 𝒈𝒎 𝑹𝑳 . Does it mean
 When an electron in valence band jumps to conduction band, a vacancy is that if we keep on increasing indefinitely 𝑹𝑳 , the gain of the amplifier will also
created in the valency band. This electron missing site in the valance band is increase indefinitely? Explain your answer?
called a hole. It has no charge and it behaves as if it has positive charge.  The gain will not increase.
 In N -type semiconductor, holes are produced by breaking the covalent bonds.  We know the expression ; 𝑽𝑪𝑬 = 𝑽𝑪𝑪 − 𝑰𝑪 𝑹𝑳 . When 𝑹𝑳 increses, the value of
They are the minority carriers. 𝑽𝑪𝑬 decreases. This will affect the zero signal collector current and zero signal
14. What is an ideal diode? collector-emitter voltage. Soon a stage will be reached, when both the junctions
 An ideal diode is one which behaves as a perfect conductor when forward biased get forward biased and the collector current (𝑰𝑪 ) gets saturated.
and as a perfect insulator when reverse biased. Under this situation, the forward  Once 𝑰𝑪 is saturated, there will be no increase in the gain of the amplifier.
resistance of the diode is assumed to be zero and the potential barrier is 22. Suppose two amplifiers having individual voltage gains 𝑨𝟏 and 𝑨𝟐 respectively
neglected. are connected in series. What is the overall gain of the amplifier?
𝑉
15. Why is semiconductor damaged by a strong current?  For the first stage, the gain, 𝐴1 = 1
𝑉𝑖
 A semi conductor is a low power device. When the current is strong, too much 𝑉𝑜
heat will be produced. This may damage the semiconductor.  For the second stage, the gain, 𝐴2 =
𝑉1
16. Can you exchange the emitter and collector of a transistor?  Hence the overall gain be; 𝐴 =
𝑉𝑜
=
𝑉1
𝑋
𝑉𝑜
= 𝐴1 𝑋 𝐴2
 No. Because, emitter is heavily dopped, where as collector is moderately dopped. 𝑉𝑖 𝑉𝑖 𝑉1

Also the size of the collector is larger than that of emitter.  If a number of amplifiers are cascaded then the overall gain; 𝐴 = 𝐴1 𝑋 𝐴2 𝑋 𝐴3 𝑋 ….
17. If the emitter and base of NPN-transistor have same doping concentration, 23. Which gates are known as universal gates? Why?
explain how will the base and collector current be affected?  NAND gates and NOR gates are called universal gates. Because by a suitable
 Base current will increase, collector current will decrease, because the collecting combination of NAND or NOR gates, we can produce all the basic gates such as
effect of collector will be very much reduced. OR, AND and NOT gates. In digital circuits, the NAND and NOR gates serve as
 The purpose of small width of the base region is to reduce the distance between building blocks and hence they named so.
the two potential barriers in the base-emitter and collector-base regions, so that 24. What is a transducer?
the collector just draws the majority carriers coming from the emitter.  A device which converts one form of energy to another is called transducer.
 This purpose is lost when the base and emitter have the same doping Example - microphone
concentration. 25. What is modulation factor or modulation depth? Give its significance.
18. A transistor is being used as a common emitter amplifier.What is the value of  The ratio of change in amplitude of the carrier wave to the amplitude of the
phase difference, if any, between the collector-emitter voltage and the imput original carrier wave is called modulation factor.
signal?  It determines strength and quality of the transmitted signal
 The phase difference between collector emitter voltage 𝑉𝐶𝐸 (output voltage) and 26. What is remote sensing?
the input signal voltage is 𝟏𝟖𝟎°  Various techniques used to get or collect information about an object, area or
 But the phase difference between the input signal current and the output phenomenon using a sensor that is not in direct contact contact with the target
collector current is zero. under study is called remote sensing.
19. What is the effect of temperature on a transistor? 27. What is meant by band width? Give the bandwidth in AM and FM transmission.
 With increase in temperature, the leakage current (i.e.) the current due to the  The frequency range used in a transmitting system for transmission is called
minority carriers will increase. Hence the collector current will also be band width.
increased.  In AM transmission; Band width = 2 𝑋 𝑚𝑎𝑥𝑖𝑚𝑢𝑚 𝑓𝑟𝑒𝑞𝑢𝑒𝑛𝑐𝑦 𝑜𝑓 𝑎𝑢𝑑𝑖𝑜 𝑠𝑖𝑔𝑛𝑎𝑙
 We can show that, the collector current ; 𝑰𝑪 = 𝜷 𝑰𝑩 + (𝟏 + 𝜷) 𝑰𝒍𝒆𝒌𝒂  In FM transmission ;Band width = 2𝑛 𝑋 𝑓𝑟𝑒𝑞𝑢𝑒𝑛𝑐𝑦 𝑜𝑓 𝑡ℎ𝑒 𝑚𝑜𝑑𝑢𝑙𝑎𝑡𝑖𝑛𝑔 𝑠𝑖𝑔𝑛𝑎𝑙
(where 𝑛 → the number of significant sidebands)
 Here leakage current increases rapidly with temperature. If temperature is
28. It is necessary to use satellites for long distance transmission. Why?
constant, then 𝑰𝑪 will depend on 𝑰𝑩 . But if temperature is increased, then 𝑰𝑪 is
 Because TV signals are high frequency signals and are not reflected by this
controlled by 𝑰𝒍𝒆𝒌𝒂 and not by 𝑰𝑩
ionosphere.
20. A transistor is called a current operated device. Why?
 For reflecting the transmitting TV signals back to earth, sattelites are used.
 The output current (i.e.) the collector current in a transistor is controlled by the
input current (i.e.) the base current. Hence the transistor is called a current
operated device.

victory R. SARAVANAN. M.Sc., M.Phil., B.Ed PG ASST [PHYSICS], GBHSS, PARANGIPETTAI - 608 502
12 PHYSICS UNIT – 10 ELECTRONICS AND COMMUNICATION COMPLETE GUIDE AND MODEL QUESTION
29. Greater the height of the TV transmitting antenna, greater is its coverage. 37. What is positive logic and negative logic?
Explain?  In positive logic, the high level voltage is represented by binary 1 or true and the
 The range or distance (d) of coverage of the propagation depends on the height low level voltage is represented by binary 0 or false
(h) of the antenna given by the equation, 𝑑 = √2 𝑅 ℎ (𝑅 →radius of earth)  In negative logic, the high level voltage is represented by binary 0 or false and
 Thus when the height (h) of the antenna increases, the distance (d) that can be the low level voltage is represented by binary 1 or true
covered by the programme will also increase 38. Where does the Fermi level lie in a conductor, insulator and pure
30. Why sky waves are not used in the transmission of TV signals? semiconductor?
 TV-signals have frequencies in the range from 100 MHz to 200 MHz.  The Fermi level in conductors lies in the conduction band, in insulator it lies in
 Ionosphere will reflect radio waves having frequencies upto 40 MHz. So sky the valence band and in pure semiconductors it lies in the gap between the
waves are not used in the transmission of TV signals. conduction band and valence band.
31. What are active and passive components? 39. Why does diamond behave like an insulator?
 Active components are those electrical components which require an external  There is a large forbidden band of 6 eV in diamond. So it is difficult to exite the
source for its full operation such as Diode, Transistor, Thyristor etc., electrons from valence band to the conduction band.
 Passive components are those components which do not need any external  Due to the abdence of free charge carriers, dimond behaves as an insulator.
source for its function such as resistors, capacitors and inductors 40. Carbon and silicon are known to have similar lattice structures. However the
32. How does the transistor act as a switch? four bonding electrons of carbon are present in second orbit while those of
 There are three useful regions of operation of a transistor namely saturation, cut silicon are present in its third orbit. How does this difference result in a
off and active regions. difference in their electrical conductivities?
 In the active region, the transistor acts as an amplifier  The energy required to take out an electron from Silicon atom is much smaller
In the saturation region, the transistor acts as a closed switch. than that in case of Carbon atom.
In the cut off region, the transistor acts as an open switch.  Hence number of free electrons for conduction in Si atom is quite significant but
 So to use a transistor as a switch, it needs to be operated in the saturation and negligibly small for C atom. Consequently the conductivity of silicon is much
cut off region. greater than that of carbon.
33. Why silicone diodes are preferred than germanium diodes? 41. How is a sample of an N-type semiconductor electrically neutral though it has
 Silicone crystals are more resistant to heat than germanium an excess of negative charge carriers?
 Silicone diode has a high voltage rating than the germanium diode  In an N-type semiconductor, the pentavalent impurity atom shares four of its
 Silicone is available in abundant compared to germanium. valence electrons with four tetravalent host atoms, while its fifth electron
34. What is Oscillator? remains free.
 An oscillator is an electronic circuit which generates a periodic AC signal from a  This impurity atom as a whole is electrically neutral. So the semiconductor is also
DC source. neutral.
 An oscillator has no input. The oscillor can be sinusoidal or square or atriangle 42. Which has greater conductivity either P -type or N -type semiconductor? Why?
wave.  Holes are majority charge carriers in P -type semiconductors and electrons are
35. Why do we need modulation? majority charge carriers in N-type semiconductor. Under a given electric field,
 Modulation converts a baseband signals (frequency upto10kHz) in to a pass free electrons have higher mobility than holes. So the conductivity of N-type
band signals (frequency higher than 100kHz) making it suitable for long distance semiconductor is greater than that of P -type semiconductor.
communication using an antenna. 43. Why is the base region of a transistor made very thin and lightly doped?
 Antenna size also depends on the frequency of the transmitting signal, so the  A thin and lightly doped base region contains a smaller number of majority
modulation allows us to use a small size antenna charge carriers. This reduce the recombination rate of electrons and holes at the
 Using modulation we can assign different frequencies to different signals which base - emitter junction. Most of the majority charge carriers coming from emitter
allow us to send multiple signals using the same medium without interference. to base immediately get collected by the collector. This reduces base current and
36. What is the difference between anolog, digital and discrete signal? increases both collector current and current gain of the transistor.
 Analog signal has continuous time and continuous amplitude.
 Digital signal has discrete time and discreate amplitude.
 Discrete signal has discrete time but continuous amplitude.

victory R. SARAVANAN. M.Sc., M.Phil., B.Ed PG ASST [PHYSICS], GBHSS, PARANGIPETTAI - 608 502
12 PHYSICS UNIT – 10 ELECTRONICS AND COMMUNICATION COMPLETE GUIDE AND MODEL QUESTION
44. In the working of a transistor, the emitter-base (EB) junction is forward biased 52. In the figure (i) and (ii), the switch which is open represents the logic state 0
while collector-base (CB) junction is reverese biased. Why? and the switch is closed represents the logic state 1. The lamp ‘L’is lit, when
 Only forward biased emitter-base junction can send the majority charge carriers output is logic state 1. What types of gate are represented by the given circuits?
from emitter to base and only reverse biased collector can collect these majority
charge carriers from the base region.
 If the emitter is reverse biased, no charge carriers will flow towards the collector
and hence no current will flow through the transistor and the transistor is said to
be in cut-off state.
45. Why is common emitter amplifier preferred over a common base amplifier?  The circuit in fig (i) represents an AND gate, because the lamp L will glow only
When is a common base amplifier preferred over a common emitter amplifier? when both the switches A and B are closed.
 Because current gain of CE- amplifier is more than that of a CB- amplifier.  The circuit in fig (ii) represents an OR gate, because the lamp L will glow when
 But when voltage amplification of the given signal is required without any phase switches A or switch B or both are closed.
change of signal voltage, CB- amplifier is preferred over CE- amplifier. 53. Can a transistor amplifier generate power?
46. Why a transistor cannot be used as a rectifier?  An amplifier is a circuit consisting of atleast one transistor which can be used to
 To use a transistor as a rectifier, either its EB portion or CB portion has to be increase current, voltage or power of alternating form. But it cannot generate
used. As base is thin and lightly dopped, either of the two portions will not work power. The energy for the higher AC power at the output is obtained from the DC
as a PN -junction. So a transistor cannot be use as a rectifier. battery.
47. How would you test in a simple way whether the transistor is spoiled or in 54. What will be the input of A and B for the Boolean expression, (𝑨 ̅̅̅̅̅̅̅̅
+ 𝑩) . (𝑨.̅̅̅̅̅
𝑩) = 𝟏
working order?  The given Boolean expression can be simplified as,
 For a transistor in working order, the forward biased EB-junction has a low ̅̅̅̅̅̅̅̅
𝑌 = (𝐴 ̅̅̅̅̅
+ 𝐵) . (𝐴. 𝐵) = (𝐴̅ . 𝐵̅ ) . (𝐴̅ + 𝐵̅ ) = (𝐴̅ . 𝐴̅ )𝐵̅ + 𝐴̅ (𝐵̅ . 𝐵̅ ) = 𝐴̅ . 𝐵̅ + 𝐴̅ . 𝐵̅ = 𝐴̅ . 𝐵̅
resistance, while the reverse biased CB-junction has a high resistance.  The corresponding truth table is,
 In a spoiled transistor, the resistance is low in both situations. Inputs Output
48. Which one of the transistors PNP and NPN is more useful and why? A B Y
 NPN transistor is more useful than PNP transistor. In NPN transistor, electrons
0 0 1
are the majority charge carriers while in PNP -transistor holes are the majority
0 1 0
charge carriers.
 Since electrons have higher mobility than holes, NPN -transistors are more 1 0 0
commonly used than PNP-transistor. 1 1 0
49. Why is logic gate so called?  So the inputs must be, A B = 0 0
 This is because, a logic gate follows a certain logical relationship between input 55. Give a single equation to express amplitude, frequency and phase modulation.
and output voltages.  The instantaneous voltage of carrier wave is ; 𝒆𝑪 = 𝑬𝑪 𝐬𝐢𝐧(𝝎𝑪 𝒕 + 𝝓)
50. How is a NOT gate different from AND or OR gate? Here, 𝐸𝐶 → amplitude of carrier wave, 𝝎𝑪 →angular frequency of carrier wave
 AND gate and OR gates can have two or more inputs while NOT gate has only one and 𝜙 → phase angle
input. (i) When amplitude 𝑬𝑪 of carrier wave changes inaccordance with modulating
 AND gate and OR gate can be made from junction diodes, but NOT gate cannot be signal (keeping 𝝎𝑪 and 𝝓 as constant), we get amplitude modulation (AM)
made from junction diodes and it is realized by using a transistor. (ii) When 𝝎𝑪 is varied with modulating signal (keeping 𝑬𝑪 and 𝝓 as constant),
51. Distinguish between the light emitting diode (LED) and photo diode. we get frequency modulation (FM)
Light emitting diode (LED) Photo diode (iii) When 𝝓 is varied , we get phase modulation (PM)
It is forward biased It is reverse biased 56. Why do we need carrier waves of very high frequency in the modulation of
Recombination of electrons and holes Energy (ℎ𝜈) is supplied by light to take signal?
takes place at the junction and emits an electron from valence band to  High frequency carrier waves are used to increase operation range, to reduce
electro magnetic radiation conduction band antenna length and convert the wide band signal into narrow band signal.
 Then the signal can be easily recovered and distinguished from other signals at
the receiving station.

victory R. SARAVANAN. M.Sc., M.Phil., B.Ed PG ASST [PHYSICS], GBHSS, PARANGIPETTAI - 608 502
12 PHYSICS UNIT – 10 ELECTRONICS AND COMMUNICATION COMPLETE GUIDE AND MODEL QUESTION
57. How many frequencies are in an amplitude modulated wave (AMW)? 66. What is the difference between telegraphy and teleprinting?
 There are three frequencies contained in AMW. They are  In telegraphy, the information is transmitted in the form of codes (dots and
(i) 𝑓𝐶 → original carrier wave frequency dashes)
(ii) 𝑓𝐶 − 𝑓𝑆 → Lower side band frequency (LSB)  Teleprinting is the advanced form of telegraphy in which text is transmitted to
(iii) 𝑓𝐶 + 𝑓𝑆 → Upper side band frequency (USB) distant places.
58. Which is better for high fidelity reception either FM or AM? 67. Why is an AM signal likely to be more noisy than FM signal upon transmission
 FM transmission gives high fidelity due to the presence of a large number of through channel?
sidebands.  In AM, the carrier wave’s instantaneous voltage is varied by modulating wave
59. Why do we need a higher band width for transmission of music compared to voltage. On transmission, noise signals can also be added and receiver assumes
that for commercial telephonic communication? noise as a part of the modulationg signal.
 Speech signals contains frequencies between 300 Hz to 3100 Hz. Such signals  However in FM, the carrier frequency is changed as per modulation wave
require a small band width of 2800 Hz for telephonic communication. voltage. This can be done at the mixing/modulation stage and not while signal is
 Audio signals have frequencies between 20 Hz to 20 kHz. So thetransmission of a transmitting in channel. Hence noise does not affect FM signal.
good music requires a higher band width of about 20 kHz 68. What is the range of frequencies used in satellite communication?
60. What should be the length of the dipole antenna for a carrier wave of frequency  The following two frequency bands are used for satellite communication,
𝟑 𝑿 𝟏𝟎𝟖 Hz (i) 5.9 GHz - 6.4 GHz ; for uplink
𝝀 𝒄 𝟑 𝑿 𝟏𝟎𝟖 𝟏 (ii) 3.7 GHz - 4.2 GHz ; for down link
 Length of a dipole antenna = = = = = 𝟎. 𝟓 𝒎
𝟐 𝟐𝝂 𝟐 𝑿 𝟑 𝑿 𝟏𝟎𝟖 𝟐 69. Give some advantages of digital communication.
61. Give one example each of a system that uses the (i) sky wave (ii) space wave  Digital signals can be easily received
mode of propagation.  Digital signals do not distorted by noise
(i) Short broadcast services use sky wave propagation  Digital signals can be coded
(ii) TV broadcast/microwave links/ satellite communication use space wave  Digital signals can be reproduced more accurately
propagation.
62. Why ground wave transmission of signals restricted to a frequency of 1500 kHz?
 As a ground wave passes over the surface of the earth, it is weekened due to the
absorption of energy by the earth.These energy losses increase with the increase
in frequency.
 Hence ground wave propagation can be sustained only up to a frequency of
1500 kHz. Higher frequencies are highly damped.
63. Why are short wave bands used for long distance transmission of signals?
 Radio waves of short waves bands can be easily reflected by the ionosphere. So
they are used in long distance transmission.
64. Why is the transmission of signals using sky waves restricted to frequencies up
to 30 MHz?
 This is because ionosphere cannot reflect electro magnetic waves having
frequency greater than 30MHz.
 For example television frequencies lie in the range 100 - 220 MHz which cannot
be reflected by the ionosphere. So sky wave propagation method is not used in
TV transmission
65. What is the essential requirement for transmitting a microwaves from one
point to another on the earth?
 For microwave transmission,the transmitting and receiving antenna must be in
the line of sight.

victory R. SARAVANAN. M.Sc., M.Phil., B.Ed PG ASST [PHYSICS], GBHSS, PARANGIPETTAI - 608 502
12 PHYSICS UNIT – 10 ELECTRONICS AND COMMUNICATION COMPLETE GUIDE AND MODEL QUESTION
EXAM NO 10. If the input to the NOT gate is A = 1011, its output is
NAME : (a) 0100 (b) 1000
UNIT -10 ELECTRONICS AND COMMUNICATION (c) 1100 (d) 0011
11. Which one of the following represents forward bias diode?
Time - 2 : 30 hours Total - 60 marks
PART - I 15 X 1 = 15 (a) (b)
Note : (i) Answer all the questions
(ii) Choose the best answer and write the option code and (c) (d)
corresponding answer 12. The given electrical network is equivalent to
1. The barrier potential of a silicon diode is approximately,
(a) 0.7 V (b) 0.3V
(c) 2.0 V (d) 2.2V
2. Doping a semiconductor results in
(a) The decrease in mobile charge carriers (b) The change in chemical properties (a) AND gate (b) OR gate
(c) The change in the crystal structure (d) The breaking of the covalent bond (c) NOR gate (d) NOT gate
3. In an unbiased p-n junction, the majority charge carriers (that is, holes) in the 13. The output of the following circuit is 1 when the input ABC is
p -region diffuse into n-region because of
(a) the potential difference across the p-n junction
(b) the higher hole concentration in p-region than that in n-region
(c) the attraction of free electrons of n-region
(d) the higher concentration of electrons in the n-region than that in the p-region
4. If a positive half –wave rectified voltage is fed to a load resistor, for which part
of a cycle there will be current flow through the load? (a) 101 (b) 100
(a) 00–900 (b) 900–1800 (c) 110 (d) 010
(c) 0 –180
0 0 (d) 00–3600 14. The variation of frequency of carrier wave with respect to the amplitude of
5. The zener diode is primarily used as the modulating signal is called
(a) Rectifier (b) Amplifier (a) Amplitude modulation (b) Frequency modulation
(c) Oscillator (d) Voltage regulator (c) Phase modulation (d) Pulse width modulation
6. The principle based on which a solar cell operates is 15. The frequency range of 3 MHz to 30 MHz is used for
(a) Diffusion (b) Recombination (a) Ground wave propagation (b) Space wave propagation
(c) Photovoltaic action (d) Carrier flow (c) Sky wave propagation (d) Satellite communication
7. The light emitted in an LED is due to
(a) Recombination of charge carriers PART - II 6 X 2 = 12
(b) Reflection of light due to lens action Note : (i) Answer any 6 of the following questions .
(c) Amplification of light falling at the junction
(ii) Question No. 23 is compulsory
(d) Large current capacity.
16. What is extrinsic semiconductor?
8. The barrier potential of a p-n junction depends on (i) type of semiconductor
17. Define junction potential (barrier potential)
material (ii) amount of doping (iii) temperature. Which one of the following is
correct? 18. What is called LED? Give its symbol
(a) (i) and (ii) only (b) (ii) only 19. Define input resistance of transistor.
(c) (ii) and (iii) only (d) (i) (ii) and (iii) 20. What is Barkhausen condition for sustained oscillations?
9. To obtain sustained oscillation in an oscillator, 21. Distinguish between analog and digital signals?
(a) Feedback should be positive (b) Feedback factor must be unity 22. Define amplitude modulation.
(c) Phase shift must be 0 or 2π (d) All the above 23. In a transistor connecten in common base configuration 𝛼 = 0.95, 𝐼𝐸 = 1 𝑚𝐴 .
Calculate the value of 𝐼𝐶 and 𝐼𝐵
victory R. SARAVANAN. M.Sc., M.Phil., B.Ed PG ASST [PHYSICS], GBHSS, PARANGIPETTAI - 608 502
12 PHYSICS UNIT – 10 ELECTRONICS AND COMMUNICATION COMPLETE GUIDE AND MODEL QUESTION
PART - III 6 X 3 = 18
Note : (i) Answer any 6 of the following questions .
(ii) Question No. 30 is compulsory
24. Write a note on Zener breakdown.
25. Draw the circuit diagram of common base configuration of NPN transistor.
26. Give the relation between 𝛼 and 𝛽
27. State and prove Demorgan’s theorems.
28. Give the advantages and limitations of frequency modulation (FM)
29. Give the logic symbol, Boolean expression and truth table of EX-OR gate
30. Prove the following Boolean expressions using the laws and theorems of Boolean
algebra. (i) (A+B) (A+𝐵̅ ) = A (ii) A(𝐴̅+B) = AB (iii) (A+B) (A+C) = A+BC
PART - IV 3 X 5 = 15
Note : (i) Answer all the questions
31. Elucidate the formation of a N –type and P –type semiconductors
(OR)
Explain the construction and working of a full wave rectifier
32. Explain the working of Zener diode as a voltage regulator
(OR)
Transistor functions as a switch. Explain
33. Fiber optic communication is gaining popularity among the various transmission
media -justify.
(OR)
Explain the action transistor as an oscillator.

அக்கினிக் குஞ்ச ொன்று கண்டேன் – அதத


அங்சகொரு கொட்டிட ொர் சபொந்திதே தைத்டதன்
சைந்து தனிந்தது கொடு – தழல்
வீரத்தில் குஞ்ச ொன்றும் மூப்சபன்றும் உண்டேொ?
தத்தரிகிே தத்தரிகிே தித்டதொம்
- மகொகவி சுப்பிரமண்ய பொரதியொர்

victory R. SARAVANAN. M.Sc., M.Phil., B.Ed PG ASST [PHYSICS], GBHSS, PARANGIPETTAI - 608 502
பசித்திரு (Be hungry) தனித்திரு (Be individual) விழித்திரு (Be conscious)

HIGHER SECONDARY SECOND YEAR-PHYSICS

NAME :
STANDARD : 12 SECTION :
SCHOOL :
EXAM NO :

victory R. SARAVANAN. M.Sc, M.Phil, B.Ed.,


PG ASST (PHYSICS)
GBHSS, PARANGIPETTAI - 608 502
12 PHYSICS UNIT – 11 RECENT DEVELOPMENT IN PHYSICS COMPLETE GUIDE AND MODEL QUESTION

PART – I 1 MARK MULTIPLE CHOICE QUESTIONS & ANSWERS 6. The materials used in Robotics are
1. The particle size of ZnO material is 30 nm. Based on the dimension it is (a) Aluminium and silver (b) Silver and gold
classified as (c) Copper and gold (d) Steel and aluminum
(a) Bulk material (b) Nano material Solution :-
(c) Soft material (d) Magnetic material  For robots aluminium and steel are the most common metals. Alluminium is a
Solution :- softer metal but steel is several times stronger.
 If the particle of a solid size less than 100 nm,it is said to be a ‘nano solid’ Answer (d) Steel and aluminum
 When the particle size exceed 100 nm, it is ‘bulk solid’ 7. The alloys used for muscle wires in Robots are
Answer (b) Nano material (a) Shape memory alloys (b) Gold copper alloys
2. Which one of the following is the natural nano material. (c) Gold silver alloys (d) Two dimensional alloys
(a) Peacock feather (b) Peacock beak Solution :-
(c) Grain of sand (d) Skin of the Whale  Muscle wires are thin strands of wire made of shape memory alloys. They can
Solution :- contracted by 5% when electric current is passed through them.
 Single strand DNA, double strand DNA, wings of a morpho butterfly, peacock Answer (a) Shape memory alloys
feathers, lotus leaf surface, parrot fist teeth are examples for nono in nature 8. The technology used for stopping the brain from processing pain is
Answer (a) Peacock feather (a) Precision medicine (b) Wireless brain sensor
3. The blue print for making ultra durable synthetic material is mimicked from (c) Virtual reality (d) Radiology
(a) Lotus leaf (b) Morpho butterfly Solution :-
(c) Parrot fish (d) Peacock feather  Medical virtual reality is effectively used to stop the brain from processing pain
Solution :- an cure soreness in the hospitalized patients. It helpsin the treatment of Autism.
 Parrot fish crunches up coral all day because of interwoven fibre nano structure Memory loss and Mental illness
of its teeth. Crystal of a mineral called fluorapatite are woven together in a Answer (c) Virtual reality
chain mail like arrangement which gives parrot fish teeth incredible durability. 9. The particle which gives mass to protons and neutrons are
Mimic of this nano structure provides the blue print for making ultra durable (a) Higgs particle (b) Einstein particle
synthetic materials (c) Nanoparticle (d) Bulk particle
Answer (c) Parrot fish Solution :-
4. Method of making nanomaterial by assembling the atoms is called  ‘Higgs particles’ also known as “God” particles were discovered by Peter Higgs
(a) Top down approach (b) Bottom up approach and Englert which gives mass to many particles like protons, neutrons etc
(c) Cross down approach (d) Diagonal approach Answer (a) Higgs particle
Solution :- 10. The gravitational waves were theoretically proposed by
 Top Down approach - breaking down bulk solids in to nano sizes (a) Conrad Rontgen (b) Marie Curie
 Bottom Up approach - assembling the atoms/molecules together (c) Albert Einstein (d) Edward Purcell
Answer (b) Bottom up approach Solution :-
5. “ Ski wax” is an application of nano product in the field of  Gravitational waves are the disturbances in the curvature of space-time and it
(a) Medicine (b) Textile travels with speed of light. Any accelerated charge emits electromagnetic wave.
(c) Sports (d) Automotive industry Similarly any accelerated mass emits gravitational waves.
Answer (c) Sports  The strongest source of gravitational waves are black holes.
 Albert Einstein theoretically proposed the existence of ‘gravitational waves’ in
the year 1915. After 100 years, it is experimentally proved that his predictions
are correct.
Answer (c) Albert Einstein

victory R. SARAVANAN. M.Sc., M.Phil., B.Ed PG ASST [PHYSICS], GBHSS, PARANGIPETTAI - 608 502
12 PHYSICS UNIT – 11 RECENT DEVELOPMENT IN PHYSICS COMPLETE GUIDE AND MODEL QUESTION
 Optical engineering and communication
PART – II & III 2 AND 3 MARK SHORT ANSWER QUESTIONS & ANSWERS  Biomedical and drug delivery
1. Distinguish between Nanoscience and Nanotechnology.  Agriculture and food
Nanoscience :  Cosmetics and paints
 It is the science of objects with typical size 1 - 100 nm  Bio-technology
 Nano means one - billionth of a metre. (i.e) 10−9 𝑚  Textiles
Nanotechnology : 6. What is robotics?
 It is a technology involving the design, production, characterization and  Robotics is an integrated study of mechanical engineering, electronic
application of nano structured materials engineeting, computer engineering and science.
2. What is the difference between Nanomaterials and Bulk materials?  Robot is a mechanical device designed with electronic circuitry and
 If the particle of a solid is of size less than 100 nm, it is said to be a ‘nano solid’. programmed to perform a specific task.
When the particle size exceeds 100 nm, it is a ‘bulk solid’ 7. What are the components of robotics?
 Though nano and bulk solids have same chemical composition, nano form of the  The robotic system mainly consists of
material shows strikingly different properties when compared to its bulk (1) Sensors
counterpart. (2) Power supplies
3. Give the interdisciplinary nature of nano technology. (3) Controm systems
 Nano science and technology is the interdisciplinary area covering its (4) Manipulators
applications in various fields. They are, (5) Necessary softwares
(1) Physics 8. Give the types of robots.
(2) Chemistry (1) Human robot :
(3) Electrical & Mechanical Engineering  Certain robots are made to resemble humans in appearance and replicate
(4) Material science the human activities like walking, lifting and sensing etc
(5) Molecular Biology (2) Industrial robots :
(6) Applied Mathematics & Computer science  Six main types of industrial robots are Cartesian, SCARA, Cylindrical, Delta,
4. Explain how nano structures are made in the laboratory? Polar and Vertically articulated.
Nano in laboratories :  They are ideal for Arc welding, Spot welding, Material handling, machne
 The nano structrures made in the laboratory mimic some of the nature’s tending and other applications.
amazing nano structrures. 9. What is artificial intelligence? What are its work?
 There are two ways of preparing the nanomaterials. They are.  The aim of artificial intelligence is to bring in human like behaviour in robots.
(1) Top - Down approach  It works on,
(2) Bottm - Up approach (1) Face recognition
(1) Top - Down approach : (2) Providing response to player’s actions is computer games
 Nano materials are synthesized by breaking down bulk solids in to nano (3) Taking decisions based on previous actions
sizes. (4) To regulate the traffic by analyzing the density of traffic on roads
(e.g) Ball milling, sol-gel, lithography (5) Translate words from one language to another
(2) Bottom - up approach : 10. Give the applications of robot in various fields.
 Nano materials are synthesized by assembling the atoms or molucles Applications of robot :
together. Selectively atoms are added to create structures.  Weaponry, packing, Lawn mowing, cutting, under water, agriculture, pool
(e.g.) plasma etching and chemical vapour cleaning
deposition  Welding, cutting, assembling, litter robot, transport.
5. List the applications of Nano technology.  Vacuum cleaners, hospitals, surgery, laboratory
 Energy storage  Exploring stars, planets etc., investigation of the mineralogy of the rocks and
 Defence and security soils on Mars, analysis of elements found in rocks and soils
 Metallurgy and materials
 Electronics

victory R. SARAVANAN. M.Sc., M.Phil., B.Ed PG ASST [PHYSICS], GBHSS, PARANGIPETTAI - 608 502
12 PHYSICS UNIT – 11 RECENT DEVELOPMENT IN PHYSICS COMPLETE GUIDE AND MODEL QUESTION
11. Write a note on nano robots.
Nano robots : PART – IV 5 MARK LONG ANSWER QUESTIONS & ANSWERS
 The size of the nano ronots is reduced to microscopic level to perform a task in 1. Explain Nano structure in nature with examples.
very small spaces. Nano in nature :
 In future nano robots are used in the medical fields. (1) Single strand DNA :
 Nano robots in blood stream to perform small surgical procedures, to fight  It is the basic building block of all living things.
against bacteria, repairing individual cell in the body.  It is about 3 nm wide
 It can travel into the body and once after the job is performed it can find its way out. (2) Morpho butterfly :
12. Why steels are preferred to make robots?  The scales of the wings of this butterfly contains nano structures.
 For robots, aluminum and steel are the most common metals.  When light wave interact with this giving the wings brilliant metallic blue
 Aluminum is a softer metal and is therefore easier to work with it. and green hues.
 But steel is several times stronger and because of the inherent strength of steel, (3) Peacock feathers :
robot bodies are made using sheet, bar, rod, channel and other shapes.  They get their iridescent coloration from light interacting with 2
13. What is particle physics? Write down its recent development. dimensional photonic crystal structres just tens of nanometers thick
Particle physics and its development :  Similar nano structrures are made in lab to glow in different colors
 The study of the theory of fundamental particles of nature is called particle physics. (4) Parrot fish :
 Initially it was thought that atom is the fundamental entity of matter. But in  It crunches up coral all day.
1930, it was established that atoms are made up of electrons, protons and  The source of powerful bite is the interwoven fibre nanostructure.
neutrons  Crystals of a mineral called fluorapatite are woven together in a chain
 In 1960, it was discovered that protons and neutrons were made up of quarks. mail-like arrangement. This structure gives parrot fish teeth incredible
 Later it was found that quarks interact through gluons. durability.
 Recently in 2013, famous Higgs particles also known as God particles were  It provides a blue print for creating ultr-durable synthetic materials that
discovered which gives mass to many particles like protons, neutrons etc ., could be useful for mechanical components in electronics and in other
14. Write a note on Cosmology. devices that undergo repetitive movement, abrasion and contact stress
 Cosmology is the branch that involves the origin and evolution of the universe. (5) Lotus Leaf surface :
 It deals with the formation of stars, galaxy etc.  Scaning electron micrograph (SEM) gives the nano structures on the
15. What are called gravitational waves? surface of a leaf from a lotus plant.
 The disturbances in the curvature of space-time is called gravitational waves.  This is the reason for self cleaning process on lotus leaf.
Its travels with speed of light. 2. Discuss the applications of Nanomaterials in various fields.
 Any accelerated charge emits electromagnetic waves. Similarly any accelerated Automotive industry :
mass emits gravitational waves.  Lightweight construction
 But these gravitational waves are very weak even for masses like earth.  Painting (fillers, base coat, clear coat)
 The strongest source of gravitational waves are black holes.  Catalysts
 The recent discoveries of gravitational waves are emitted by two black holes  Tires (fillers)
when they merge to a single black hole.  Sensors
 In 1915, Albert Einstein theoretically proposed the existence of gravitational  Coatings for windscreen and car bodies
waves. After 100 years, it is experimentally proved that his predictions are correct. Chemical industry :
16. Write a note on black holes.  Fillers for paint systems
Black holes :  Coating systems based on nanocomposites
 Black holes are end stage of stars which are highly dense massive object.  Impregnation of papers
 Its mass ranges 20 times mass of the sun to 1 million times mass of the sun.  Switchable adhesives
 It has very strong gravitational force such that no particle or even light can escape from it .  Magnetic fluids
 The existence of black hole is studied when the stars orbiting the black hole Engineering :
behave differently from the other stars.  Wear protection for tools and machines
 Every galaxy has black hole at its centre.  Lubricant-free bearings
 Sagittarius A* is the black hole at the centre of the Milky Way galaxy.
victory R. SARAVANAN. M.Sc., M.Phil., B.Ed PG ASST [PHYSICS], GBHSS, PARANGIPETTAI - 608 502
12 PHYSICS UNIT – 11 RECENT DEVELOPMENT IN PHYSICS COMPLETE GUIDE AND MODEL QUESTION
Electronic industry :  Skin creams
 Data memory  Tooth paste
 Displays Sports/ outdoor :
 Laser diodes  Ski wax
 Glass fibres  Antifogging of glasses/goggles
 Optical switches  Antifouling coatings for ships/boats
 Filters (IR-blocking)  Reinforced tennis rackets and balls
 Conductive, antistatic coatings 3. What are the possible harmful effects of usage of Nanoparticles? Why?
Construction : Possible harmful effects of nano particles :
 Construction materials  The major concern here is that the nanoparticles have the dimensions same as
 Thermal insulation that of the biological molecules such as proteins. They may easily get absorbed
 Flame retardants on to the surface of living organisms and they might enter the tissues and fluids
 Surface-functionalised building materials for wood, floors, stone, facades, tiles, of the body.
roof tiles, etc.  The adsorbing nature depends on the surface of the nanoparticle. Indeed, it is
 Facade coatings possible to deliver a drug directly to a specific cell in the body by designing the
 Groove mortar surface of a nanoparticle so that it adsorbs specifically on to the surface of the
Medicine : target cell.
 Drug delivery systems  The interaction with living systems is also affected by the dimensions of the
 Active agents nanoparticles.
 Contrast medium  Nanoparticles can also cross cell membranes. It is also possible for the inhaled
 Medical rapid tests nanoparticles to reach the blood, to reach other sites such as the liver, heart or
 Prostheses and implants blood cells.
 Antimicrobial agents and coatings  Researchers are trying to understand the response of living organisms to the
 Agents in cancer therapy presence of nanoparticles of varying size, shape, chemical composition and
Food and drinks : surface characteristics.
 Package materials 4. Mention the advantages and disadvantages of Robotics.
 Storage life sensors Advantages of robotics :
 Additives  The robots are much cheaper than humans.
 Clarification of fruit juices  Robots never get tired like humans. Hence absenteeism in work place can be
Energy : reduced.
 Fuel cells  Robots are more precise and error free in performing the task.
 Solar cells  Stronger and faster than humans.
 Batteries  Robots can work in extreme environmental conditions: extreme hot or cold,
 Capacitors space or underwater. In dangerous situations like bomb detection and bomb
Textile/fabrics/non-wovens : deactivation.
 Surface-processed textiles  In warfare, robots can save human lives.
 Smart clothes  Robots are significantly used in handling materials in chemical industries
Household : especially in nuclear plants which can lead to health hazards in humans.
 Ceramic coatings for irons Disadvantages of robotics :
 Odors catalyst  Robots have no sense of emotions or conscience.
 Cleaner for glass,  They lack empathy and hence create an emotionless workplace.
 ceramic, floor, windows  If ultimately robots would do all the work, and the humans will just sit and
Cosmetics : monitor them, health hazards will increase rapidly.
 Sun protection  Unemployment problem will increase.
 Lipsticks  Robots can perform defined tasks and cannot handle unexpected situations

victory R. SARAVANAN. M.Sc., M.Phil., B.Ed PG ASST [PHYSICS], GBHSS, PARANGIPETTAI - 608 502
12 PHYSICS UNIT – 11 RECENT DEVELOPMENT IN PHYSICS COMPLETE GUIDE AND MODEL QUESTION
 The robots are well programmed to do a job and if a small thing goes wrong it 6. Discuss the functions of key components in Robots?
ends up in a big loss to the company. Functions of key components of Robots :
 If a robot malfunctions, it takes time to identify the problem, rectify it, and even  Most robots are composed of 3 main parts:
reprogram if necessary. This process requires significant time. (1) Controller
 Humans cannot be replaced by robots in decision making. (2) Mechanical parts
 Till the robot reaches the level of human intelligence, the humans in work place will exit. (3) Sensors
5. Comment on the recent advancement in medical diagnosis and therapy. Controller :
(1) Virtual reality :  It is also known as the "brain" which is run by a computer program.
 Medical virtual reality is effectively used to stop the brain from processing  It gives commands for the moving parts to perform the job.
pain and cure soreness in the hospitalized patients. Mechanical parts :
 It helps in the treatment of Autism, Memory loss, and Mental illness.  It consists motors, pistons, grippers, wheels, and gears that make the robot
(2) Precision medicine : move, grab, turn, and lift.
 Precision medicine is an emerging approach for disease treatment and Sensors :
prevention that takes into account individual variability in genes,  It tells the robot about its surroundings. It helps to determine the sizes and
environment, and lifestyle for each person. shapes of the objects around, distance between the objects, and directions as well.
(3) Health wearables : 7. Explain the various components of robotics.
 A health wearable is a device used for tracking a wearer's vital signs or Power conversion unit:
health and fitness related data, location, etc.  Robots are powered by batteries, solar power, and hydraulics.
 Medical wearables with articial intelligence and big data provide an added Actuators:
value to healthcare with a focus on diagnosis, treatment, patient monitoring  Converts energy into movement. The majority of the actuators produce
and prevention. rotational or linear motion.
(4) Articial organs : Electric motors:
 An articial organ is an engineered device or tissue that is implanted or  They are used to actuate the parts of the robots like wheels, arms, fingers, legs,
integrated into a human. sensors, camera, weapon systems etc.
 It is possible to interface it with living tissue or to replace a natural organ. It  Different types of electric motors are used. The most often used ones are AC
duplicates or augments a specic function or functions of human organs so motor, Brushed DC motor, Brushless DC motor, Geared DC motor, etc.
that the patient may return to a normal life as soon as possible. Pneumatic Air Muscles:
(5) 3D printing :  They are devices that can contract and expand when air is pumped inside.
 Advanced 3D printer systems and materials assist physicians in a range of  It can replicate the function of a human muscle. ey contract almost 40% when
operations in the medical field from audiology, dentistry, orthopedics and the air is sucked inside them.
other applications. Muscle wires:
(6) Wireless brain sensors :  They are thin strands of wire made of shape memory alloys. ey can contract by
 Wireless brain sensors monitor intracranial pressure and temperature and 5% when electric current is passed through them.
then are absorbed by the body. Hence there is no need for surgery to Piezo Motors and Ultrasonic Motors:
remove these devices.  Basically, we use it for industrial robots.
(7) Robotic surgery : Sensors:
 Robotic surgery is a type of surgical procedure that is done using robotic systems.  Generally used in task environments as it provides information of real-time
 Robotically-assisted surgery helps to overcome the limitations of pre- knowledge.
existing minimally-invasive surgical procedures and to enhance the Robot locomotion:
capabilities of surgeons performing open surgery.  Provides the types of movements to a robot.
(8) Smart inhalers :  The different types are
 Inhalers are the main treatment option for asthma. Smart inhalers are (a) Legged
designed with health systems and patients in mind so that they can offer (b) Wheeled
maximum benefit. (c) Combination of Legged and Wheeled Locomotion
 Smart inhalers use Bluetooth technology to detect inhaler use, remind (d) Tracked slip/skid
patients when to take their medication and gather data to help guide care.
victory R. SARAVANAN. M.Sc., M.Phil., B.Ed PG ASST [PHYSICS], GBHSS, PARANGIPETTAI - 608 502
12 PHYSICS UNIT – 11 RECENT DEVELOPMENT IN PHYSICS COMPLETE GUIDE AND MODEL QUESTION

NAME : EXAM NO PART - II 2X 2 = 4


UNIT – 11 RECENT DEVELOPMENT IN PHYSICS Note : (i) Answer any 2 of the following questions .
11. Distinguish between Nano science and Natotechnology
Time - 1 : 15 hours Total - 25 marks 12. What is robotics?
PART - I 10 X 1 = 10 13. Why steels are preferred to make robots?
Note : (i) Answer all the questions PART - III 2X3=6
(ii) Choose the best answer and write the option code and
corresponding answer Note : (i) Answer any 2 of the following questions .
1. The particle size of ZnO material is 30 nm. Based on the dimension it is 14. What is the difference between Nano material and bulk material?
classified as 15. Write a note on nano robots.
(a) Bulk material (b) Nano material 16. What is artificial inteligence? What are its work?
(c) Soft material (d) Magnetic material PART - IV 1X5=5
2. Which one of the following is the natural nano material. Note : (i) Answer all the questions
(a) Peacock feather (b) Peacock beak 17. Explain Nano structure in nature with examples.
(c) Grain of sand (d) Skin of the Whale
(OR)
3. The blue print for making ultra durable synthetic material is mimicked from Discuss the functions of key components in Robots.
(a) Lotus leaf (b) Morpho butterfly
(c) Parrot fish (d) Peacock feather
4. Method of making nanomaterial by assembling the atoms is called
(a) Top down approach (b) Bottom up approach
(c) Cross down approach (d) Diagonal approach
5. “ Ski wax” is an application of nano product in the field of
(a) Medicine (b) Textile
(c) Sports (d) Automotive industry
6. The materials used in Robotics are
(a) Aluminium and silver (b) Silver and gold
(c) Copper and gold (d) Steel and aluminum
7. The alloys used for muscle wires in Robots are
(a) Shape memory alloys (b) Gold copper alloys
(c) Gold silver alloys (d) Two dimensional alloys
8. The technology used for stopping the brain from processing pain is
(a) Precision medicine (b) Wireless brain sensor
(c) Virtual reality (d) Radiology
9. The particle which gives mass to protons and neutrons are
(a) Higgs particle (b) Einstein particle
(c) Nanoparticle (d) Bulk particle யாதும் ஊரே யாவரும் ரேளிர்
10. The gravitational waves were theoretically proposed by - எல்லா ஊரும் நம் ஊர், எல்லா உயிர்ேளும் எம் உறவினர்ேள்
(a) Conrad Rontgen (b) Marie Curie
(c) Albert Einstein (d) Edward Purcell தீதும் நன்றும் பிறர் தே வாோ
- தீமையும் நம்மையும் அடுத்தவர்ேளால் நைக்கு வருவதல்ல

புறநானூறு - ேணியன் பூங்குன்றனார்

victory R. SARAVANAN. M.Sc., M.Phil., B.Ed PG ASST [PHYSICS], GBHSS, PARANGIPETTAI - 608 502

You might also like